You are on page 1of 479

Article III – Bill of Rights

Section 1. No person shall be deprived of life, liberty, or property without due process of law nor shall
any person be denied the equal protection of the law.

1. Purpose of the Bill of Rights


2. Three Great Powers of Government
3. Police Power

Lozano v. Martinez, GR No. L-63419, December 18, 1986 - Anton


FACTS:
The constitutionality of Batas Pambansa Bilang 22 (BP 22 for short), popularly known as the Bouncing
Check Law, which was approved on April 3, 1979, is the sole issue presented by these petitions for
decision. It is contended that the law is unconstitutional for violating the non-imprisonment for non-
payment of debt clause in the Constitution.

ISSUE: Whether or not B.P. 22 is unconstitutional for violating the non-imprisonment for non-payment
of debt clause.

RULING:

NO. It may be constitutionally impermissible for the legislature to penalize a person for non-payment of
a debt ex contractu But certainly it is within the prerogative of the lawmaking body to proscribe certain
acts deemed pernicious and inimical to public welfare. Acts mala in se are not the only acts which the
law can punish. An act may not be considered by society as inherently wrong, hence, not malum in se
but because of the harm that it inflicts on the community, it can be outlawed and criminally punished
as malum prohibitum. The state can do this in the exercise of its police power.

The police power of the state has been described as "the most essential, insistent and illimitable of
powers" which enables it to prohibit all things hurtful to the comfort, safety and welfare of society. It is
a power not emanating from or conferred by the constitution, but inherent in the state, plenary,
"suitably vague and far from precisely defined, rooted in the conception that man in organizing the state
and imposing upon the government limitations to safeguard constitutional rights did not intend thereby
to enable individual citizens or group of citizens to obstruct unreasonably the enactment of such
salutary measures to ensure communal peace, safety, good order and welfare."

The enactment of BP 22 is a declaration by the legislature that, as a matter of public policy, the making
and issuance of a worthless check is deemed public nuisance to be abated by the imposition of penal
sanctions.

4. The Seat of Police Power

MMDA v. Bel-Air Village Association, etc GR No. 135962, March 27, 2000- Anton
FACTS:
Petitioner, in the exercise of its authority as an agent of the state possessed with police power, required
the opening of Neptune St. of Bel-air subdivision, a private subdivision, in order to rationalize the use of
roads and/or thoroughfares for the safe and convenient movement of persons. It is further contended
that since petitioner possesses police power, it is no longer necessary for the City of Makati to enact an
ordinance to effectuate the opening of the said street.

ISSUE: HAS THE METROPOLITAN MANILA DEVELOPMENT AUTHORITY (MMDA) THE MANDATE TO OPEN
NEPTUNE STREET TO PUBLIC TRAFFIC PURSUANT TO ITS REGULATORY AND POLICE POWERS?

RULING:
NO. Police power is vested in the Legislative department. It cannot be exercised by petitioner by the
mere fact that it is an agent of the State. It bears stressing that police power is lodged primarily in the
National Legislature. It cannot be exercised by any group or body of individuals not possessing
legislative power. The National Legislature, however, may delegate this power to the President and
administrative boards as well as the lawmaking bodies of municipal corporations or local government
units. Once delegated, the agents can exercise only such legislative powers as are conferred on them by
the national lawmaking body. In sum, petitioner’s charter does not grant police power unto it. Neither
can it be inferred that there is a valid delegation of police power to the MMDA by the National
Legislature pursuant to the general welfare clause. Hence, contrary to its contention, it does not possess
police power in order to effectuate the opening of Neptune street to the public.

4. The Seat of Police Power

MMDA v. Bel-Air Village Association, etc GR No. 135962, March 27, 2000- Anton

5. Primacy of Human Rights

Republic v. Sandiganbayan GR 104768, July 21, 2003 - Anton


FACTS:
The PCGG was created immediately after the EDSA revolution. Its primary tasked is to recover the ill-
gotten wealth of the Marcoses. The properties of private respondent were seized through the mere
affidavits to the effect that the acquisition of the properties were unreasonable.

ISSUE: Whether the sequestration is valid.

RULING:
NO. While the Philippines is not bound by any Constitution at that the time, there can still be no
arbitrary deprivation of property as this is protected by the Universal Declaration of Human Rights. The
primacy of Human Rights must prevail at all times. Needless to emphasize, the right of the people to be
secure in their properties is one of the most basic human rights. An alleged illegal ownership over the
properties does not warrant the illegal seizure in the case at bar. The seizure of these items was
therefore void, and unless these items are contraband per se, and they are not, they must be returned
to the person from whom the raiding seized them.

Mijares v. Ranada, GR 139325, April 12, 2005 - Anton


FACTS:
Petitioners who each alleged having suffered human rights abuses such as arbitrary detention, torture
and rape in the hands of police or military forces during the Marcos regime, brought an action before
the U.S. District Court in Hawaii against the Marcos estate. They obtained a favorable decision therein
and they now seek to enforce the foreign judgment before Philippine Courts.

ISSUE: Whether or not the said foreign judgment may be recognized before our Philippine Courts.

RULING:
YES. Sound principles of comity and vested rights dictate that the supremacy of human rights must
prevail and it cannot be limited by the mere fact that a judgment was rendered by a foreign court. Strict
rules and procedures of the enforcement of judgment must yield to the preservation of human rights.
Besides, the judgment sought to be enforced in the case at bar is one in personam. Hence, it is merely
presumptive rather than conclusive. The other party may adduce evidence to rebut the said judgment.

6. Hierarchy of Rights: Life, Liberty, Property

Philippine Blooming Mills Employees Organization v. Philippine Blooming Mills Co. Inc., 51 SCRA 189-
Anton
FACTS:
Petitioner decided to stage a mass demonstration in front of Malacañang to express their grievances
against the alleged abuses of the Pasig Police. This was assailed by respondent as according to the, it
would disrupt the ordinary flow of their business, tantamount of depriving them of their peroperty.

ISSUE: Whether or not respondent’s property rights is supreme so that respondent’s contention may be
given due weight.

RULING:
No. While the Bill of Rights also protects property rights, the primacy of human rights over property
rights is recognized. Because these freedoms are "delicate and vulnerable, as well as supremely precious
in our society" and the "threat of sanctions may deter theirexercise almost as potently as the actual
application of sanctions," they "need breathing space to survive," permitting government regulation
only "with narrow specificity." Property and property rights can be lost thru prescription; but human
rights are imprescriptible. In the hierarchy of civil liberties, the rights of free expression and of assembly
occupy a preferred position as they are essential to the preservation and vitality of our civil and political
institutions; and such priority "gives these liberties the sanctity and the sanction not permitting dubious
intrusions."

Salonga v. Pano, GR No. L-59524, February 18, 1985- Anton


FACTS:
The petitioner invokes the constitutionally protected right to life and liberty guaranteed by the due
process clause, alleging that no prima facie case has been established to warrant the filing of an
information for subversion against him. Petitioner asks this Court to prohibit and prevent the
respondents from using the iron arm of the law to harass, oppress, and persecute him, a member of the
democratic opposition in the Philippines. as amended by PD 885, BP 31 and PD 1736. On 15 October
1981, the counsel for Salonga filed a motion to dismiss the charges against Salonga for failure of the
prosecution to establish a prima facie case against him but this was denied by respondent judge.

ISSUE: Whether or not the right to life and liberty is deemed violated upon filing an information without
prima facie case.
RULING:
YES. The right to a preliminary investigation is a statutory grant, and to withhold it would be to
transgress constitutional due process. A preliminary investigation serves not only the purposes of the
State. More important, it is a part of the guarantees of freedom and fair play which are birthrights of all
who live in our country. It is, therefore, imperative upon the fiscal or the judge as the case may be, to
relieve the accused from the pain of going through a trial once it is ascertained that the evidence is
insufficient to sustain a prima facie case or that no probable cause exists to form a sufficient belief as to
the guilt of the accused.

Victoriano v. Elizalde Rope Workers’ Union, GR No. L-25246, Sept. 12, 1974- Anton
FACTS:

Petitioner, a member of the religious sect known as the "Iglesia ni Cristo", had been in the employ of the
Elizalde Rope Factory, Inc. As such employee, he was a member of respondent which had with the
Company a collective bargaining agreement containing a closed shop provision which reads as follows:

Membership in the Union shall be required as a condition of employment for all


permanent employees workers covered by this Agreement.

Petitioner’s religious sect prohibits affiliation to any labor organization such as respondent. Respondent
on the other hand contends that R.A. 3350 in exempting the members of said religious sects from
coverage of union security agreements.

ISSUE: Whether or not petitioner may be exempt from affiliating with respondent.

RULING:
YES. The means adopted by the Act to achieve that purpose — exempting the members of said religious
sects from coverage of union security agreements — is reasonable.

It may not be amiss to point out here that the free exercise of religious profession or belief is superior to
contract rights. In case of conflict, the latter must, therefore, yield to the former. Religious freedom,
although not unlimited, is a fundamental personal right and liberty, and has a preferred position in the
hierarchy of values. Contractual rights, therefore, must yield to freedom of religion. It is only where
unavoidably necessary to prevent an immediate and grave danger to the security and welfare of the
community that infringement of religious freedom may be justified, and only to the smallest extent
necessary to avoid the danger.

Social Justice Society, et al v. Atienza, Jr., GR No. 156052, February 13, 2008- Anton
FACTS:
Ordinance No. 8027, approved by Manila City Council on November 28, 2001 and effective December
28, 2001, reclassifies portions of Pandacan and Sta. Ana from industrial to commercial and directs the
owners and operators of businesses to cease and desist from operating their businesses within 6 months
from the ordinance’s effectivity. Among the businesses in the area are the so-called Pandacan Terminals
of Chevron, Petron, and Shell.
Chevron, Petron and Shell, questioned the validity of the said ordinance. They argued that they are
fighting for their right to property alleging that they stand to lose billions of pesos if forced [to] relocate.

ISSUE: Are the contentions of the oil companies tenable?

RULING:
NO. The Court described Ordinance No. 8027 as a measure enacted pursuant to the delegated police
power of local government units “to promote the order, safety, and health, morals, and general welfare
of the society.” It explained that “based on the hierarchy of constitutionally protected rights, the right to
life enjoys precedence over the right to property. The reason is obvious: life is irreplaceable, property is
not.

When the state or [local government unit] LGU’s exercise of police power clashes with a few individuals’
right to property, the former should prevail.”

7. Due Process: In General

Tupas v. CA, 193 SCARA 597- Anton


Asilo v. People, 645 SCRA 41- Anton

8. Procedural Due Process

In General

Banco Espanol Filipino v. Palanca 37 P 921- Anton


FACTS:
Engracio Palanca was indebted to El Banco and he had his parcel of land as security to his debt. His debt
amounted to P218,294.10. His property is worth 75k more than what he owe. Due to the failure of
Engracio to make his payments, El Banco executed an instrument to mortgage Engracio’s property.
Engracio however left for China and he never returned til he died. Since Engracio is a non resident El
Banco has to notify Engracio about their intent to sue him by means of publication using a newspaper.
The lower court further ordered the clerk of court to furnish Engracio a copy and that it’d be sent to
Amoy, China. The court eventually granted El Banco petition to execute Engracio’s property. 7 years
thereafter, Vicente surfaced on behalf of Engracio as his administrator to petition for the annulment of
the ruling. Vicente averred that there had been no due process as Engracio never received the
summons.

ISSUE: What are the requisites of due process in general?

RULING:
The SC ruled against Palanca. The SC ruled that the requisites for judicial due process had been met. The
requisites are:

1. There must be an impartial court or tribunal clothed with judicial power to hear and decide the matter
before it.
2. Jurisdiction must be lawfully acquired over the person of the defendant or over the property subject of
the proceedings.
3. The defendant must be given the opportunity to be heard.
4. Judgment must be rendered only after lawful hearing.

Aspects of the Proceedings


Galvez v. CA 237 SCRA 685- Anton
FACTS:
An ex parte motion to withdraw the information was filed without notice and hearing. This was sought
for in order that the proper charges against petitioner may be determined. It is petitioners' submission
that the prosecution's failure to serve them a copy of the motion to withdraw the original informations
and to set said motion for hearing constitutes a violation of their right to be informed of the proceedings
against them. They now challenge that the trial court may not pass of the merits of the motion.

ISSUE: Whether or not the trial court can pass on the merits of the motion to withdraw the information.

RULING:
YES. The rule is now well settled that once a complaint or information is filed in court any disposition of
the case, whether as to its dismissal or the conviction or the acquittal of the accused, rests in the sound
discretion of the court. Although the prosecutor retains the direction and control of the prosecution of
criminal cases even when the case is already in court, he cannot impose
his opinion upon the tribunal. For while it is true that the prosecutor has the quasi-judicial discretion to
determine whether or not a criminal case should be filed in court, once the case had already been
brought therein any disposition the prosecutor may deem proper thereafter should be addressed to the
court for its consideration and approval. 23 The only qualification is that the action of the court must not
impair the substantial rights of the accused or the right of the People to due process of law. It must here
be emphasized that respondent prosecutor sought, and was subsequently granted, permission by the
court to dismiss the original informations. It cannot therefore be validly claimed that the prosecutor
exceeded his authority in withdrawing those informations because the same bore the imprimatur of the
court. As such, there is no violation of petitioner’s due process.

State Prosecutor v. Muros 236 SRCA 505- - Anton


FACTS:
The case at bar involves the prosecution of the 11 charges against Imelda Marcos in violation of the
Central Bank Foreign Exchange Restriction in the Central Bank Circular 960. The respondent judge
dismissed all 11 cases solely on the basis of the report published from the 2 newspapers, which the
judge believes to be reputable and of national circulation, that the Pres. of the Philippines lifted
all foreign exchange restrictions.

ISSUE: Whether or not the respondent judge committed grave abuse of discretion in taking judicial
notice on the statement of the president lifting the foreign exchange restriction published in the
newspaper as basis for dismissing the case?

RULING:
YES. The Supreme Court held the respondent judge guilty for gross ignorance of the law. It cannot
comprehend his assertion that there is no need to wait for the publication of the circular no. 1353 which
is the basis of the President’s announcement in the newspaper, believing that the public announcement
is absolute and without qualification and is immediately effective and such matter becomes a public
knowledge which he can take a judicial notice upon in his discretion. It is a mandatory requirement that
a new law should be published for 15 days in a newspaper of general circulation before its effectivity.
When the President’s statement was published in the newspaper, the respondent admitted of not
having seen the official text of CB circular 1353 thus it was premature for him to take judicial notice on
this matter which is merely based on his personal knowledge and is not based on the public knowledge
that the law requires for the court to take judicial notice of.

Martinez v. CA 237 SCRA 395 - Anton


FACTS:
Petitioner was charged with libel arising from the allegedly derogatory and scurrilous imputations and
insinuations against Laurel contained in Martinez' article entitled "The Sorrows of Laurel. After a motion
for reinvestigation for determination of probable cause, the trial court dismissed the case relying solely
upon the recommendation of the prosecutors that there is no probable court.

ISSUE: Is the dismissal by the judge correct?

RULING:
NO. The grant of the motion to dismiss was based upon considerations other than the judge's own
personal individual conviction that there was no case against the accused. Whether to approve or
disapprove the stand taken by the prosecution is not the exercise of discretion required in cases like
this. The trial judge must himself be convinced that there was indeed no sufficient evidence against the
accused, and this conclusion can be arrived at only after an assessment of the evidence in the
possession of the prosecution. What was imperatively required ws the trial judge's own assessment of
such evidence, it not being sufficient for the valid and proper exercise of judicial discretion merely to
accept the prosecution's word for its supposed insufficiency.

As aptly observed by the Office of the Solicitor General, in failing to make an independent finding of the
merits of the case and merely anchoring the dismissal on the revised position of the prosecution, the
trial judge relinquished the discretion he was duty bound to exercise. In effect, it was the prosecution,
through the Department of Justice which decided what to do and not the court which was reduced to a
mere rubber stamp

Espeleta v. Avelino 62 SCRA 395- Anton

FACTS:
Shell’s counsel Bellaflor forwarded an oral motion for the revocation of appearance of Espleta’s witness
Montano for cross-examination and the conlusion of her testimony. She was unable to appear in one of
the trials due to her an audit for a job in the Department of Local Government at the day she was
supposed to finish her testimony and cross-examination. Judge Avelino accepted this proposal and even
allowed Shell to present its rebuttal witness for Espeleta’s testimony. The judge granted this request to
the prejudice of Espeleta. The magistrate also did not consider Espeleta’s counsel’s letter for
postponement. The judge told parties to submit documentary evidence afterwards but rejected the
ones from Montano’s testimony due to her being stricken from the records.

ISSUE: Whether the concept of fairness that is basic to procedural due process would be satisfied if the
right to be heard of petitioner was revoked by the respondent Judge?
RULING:
NO. Espeleta presented Montano as an accountant to testify for the reduced balance to Shell in the
form of 14,000 from Shell’s proposed amount of 22,000. The deductions included payment for damage
due to gasoline leakages.
Under the circumstances, the stress on the absence of procedural due process is understandable for as a
result of the order of respondent Judge, there is more than just a probability that petitioner would be
condemned to pay before he had been fully heard. The trial didn’t satisfy the standard for a
judicious inquiry, because there was a mockery of the requirement that the litigants should be given full
opportunity to sustain their claims and have their evidence considered and weighted. The petitioner can
assert due process.

Rabino v. Cruz 222 SCRA 493 – Anton


FACTS:
A complaint was filed for the recovery of portions of land. Petitioners, although occupants of a portion
of the subject parcel of land in Civil Cases No. 630 and No. 631, were not impleaded as defendants in
said.

ISSUE: Is the judgment in the case enforceable against petitioners?

RULING:
NO. It is to be stressed that petitioners have not been impleaded as party defendants although they are
occupying portions of the parcel of land, subject-matter of said cases. The rule is that judgment can not
bind persons who are not parties to the action (Vda. de Sengbengco vs. Arellano, 1 SCRA 711 [1961];
Hollero vs. Court of Appeals, 11 SCRA 310 [1964]; Plata vs. Yatco, 12 SCRA 718 [1964]). This rule is
anchored on the constitutional right of a person to due process of law. No person shall be condemned
or judgment rendered against him without due process of law.

It is clear that petitioners were denied due process of law. They are possessors of portion of the parcel
of land in question yet they were not impleaded as defendants in Civil cases No. 630 and 631 for which
reason any judgment rendered in said cases and any order of writ issued therein cannot be enforced
against them.

Ysmael v. CA 273 SCRA 165 - Anton

FACTS:
This case involved a sale of land by petitioner. Due to petitioner’s failure to deliver the titles of the
subject lots, Jamlang sought relief from the Housing and Land Use Regulatory Board (HLURB) by filing a
complaint for specific performance with damages. The HLURB ruled against petitioner. Petitioner
complains that he was unjustly denied due process in the execution proceedings, adding that his
inaction and default status at the trial stage was unduly extended to the execution stage.

ISSUE: Whether or not petitioner was denied due process.

RULING:
NO. Petitioner’s attempt to separate and make a distinction between due process at the trial stage and
due process at the execution stage will not succeed. “A case in which execution has been issued is
regarded as still pending so that all proceedings in the execution are proceedings in the
suit. Unquestionably, the court which rendered the judgment has a general supervisory control over its
process of execution. This power carries with it the right to determine every question of fact and law
which may be involved in the execution.” Trial and execution proceedings constitute one whole action
or suit. Petitioner cannot unduly separate the two so that he could conveniently escape the effects of
being declared in default. The essence of due process is the opportunity to be heard. Petitioner was
given this opportunity, yet he chose to ignore it. Thus, he cannot now complain that he was denied due
process.

Carvajal v CA 280 SCRA 351 – Anton


FACTS:
Petitioner is the applicant in a land registration case. During the hearings conducted, petitioner
presented his evidence on the question as to whether or not he had a registrable right over the land in
question. The National Land Titles and Deeds Registration Administration (NLTDRA) ruled against
petitioner. Petitioner maintains that he was denied due process when he, as an applicant in a land
registration case, was not able to take the witness stand.

ISSUE: Is there denial of due process if an applicant for land registration is unable to testify?

RULING:
NO. Petitioner claims that he was denied due process because he was unable to take the witness
stand. We disagree. The essence of due process is the opportunity to be heard. It is the denial of this
opportunity that is repugnant to due process.[19] In this case, petitioner was afforded an opportunity to
present witnesses, and he did present three. However, petitioner did not invoke his right to take the
witness stand even when the trial court ordered the submission of the parties’ memoranda which
signified the termination of the proceedings. Because he acquiesced to the termination of the case, he
forfeited his right to take the witness stand.

People v. Castillo 289 SCRA 213 – Czar


F: A complaint was filed against Robert Castillo for stabbing a certain Tony Dometita, which brought
about the death of the latter. The court found Castillo guilty beyond reasonable doubt. Accused
appealed before this Court alleging that the court was partial in favor of the prosecution as shown by his
participation in the examination of witnesses by questioning numerous questions.

I: Whether or not the appellant was given an impartial trial by the court?

H: The allegation of bias and prejudice is not well-taken. It is a judge’s prerogative and duty to ask
clarificatory questions to ferret out the truth. On the whole, the Court find that the questions
propounded by the judge were merely clarificatory in nature. Questions which merely clear up dubious
points and bring out additional relevant evidence are within judicial prerogative. Moreover,
jurisprudence teaches that allegations of bias on the part of the trial court should be received with
caution, especially when the queries by the judge did not prejudice the accused. The propriety of judge’s
queries is determined not necessarily by their quantity but by their quality and, in any event, by the test
of whether the defendant was prejudiced by such questioning. In this case, appellant failed to
demonstrate that he was prejudiced by the questions propounded by the trial judge. In fact, even if all
such questions and the answers thereto were eliminated, appellant would still be convicted.
Cosep v. People 290 SCRA 378 – Czar
F: The Municipality of Olutanga secured the services of private complainant Alegre to undertake the
construction of an artesian well under a “pakyaw” arrangement for the contract price of P5000 payable
after completion of the project. Petitioner Cosep, the municipal planning and development coordination
officer, monitored the progress of the construction. After the project, petitioner secured the amount of
P5000 from the Municipal treasurer, however only P4500 was given to the private complainant, the
balance being allegedly withheld by petitioner as reimbursement for his expenses in processing the
papers in the Municipal Treasurers Office. A case was filed against the Petitioner for allegedly
withholding the P500 before the Sandiganbayan. The Sandiganbayan convicted Cosep guilty beyond
reasonable doubt of the crime. Hence this petition, contending that he was not accorded an impartial
trial by the Sandiganbayan and his guilt was not proven beyond reasonable doubt.
I: Whether or not Cosep was not accorded an impartial trial by the Sandiganbayan?
H: Admittedly, petitioner, like any other accused individual, is entitled to a fair trial before an “impartial
and neutral judge” as an indispensable imperative of due process.[6] Judges must not only be impartial,
but must also appear to be impartial as an added assurance to the parties that the decision will be
just.[7] However, this is not to say that judges must remain passive or silent during the
proceedings. Since they are in a better position to observe the demeanor of the witness as he testifies
on the witness stand, it is only natural for judges to ask questions to elicit facts with a view to attaining
justice for the parties. Questions designed to clarify points[8] and to elicit additional relevant evidence
are not improper.[9] Also, the judge, being the arbiter, may properly intervene in the presentation of
evidence to expedite and prevent unnecessary waste of time.[10]
With the above doctrines serving as guidelines, we have scrutinized carefully the questions
propounded by the Justices, and none was indicative of their partiality for the prosecution in proving its
case against the petitioner. More precisely, on pages 34 to 35 of the Transcript of Stenographic Notes,
the gist of the questions were on the monitoring procedure being undertaken by the petitioner in
supervising the project. While on pages 36 to 39, the questions dealt with the identities and
qualifications of the workers who participated in the construction of the project. Those on pages 41 to
42, referred to queries which sought to clarify the facts and circumstances of another case filed against
the petitioner by a certain Mr. Macapala. All told, these questions cannot be said to have crossed the
limits of propriety. In propounding these questions, the Justices merely attempted to ferret the the
truth as to the facts to which the witness was testifying.
In any case, if petitioner were under the impression that the Justices were unduly interfering in his
testimony, he was free to manifest his objection.[11] However, the records show that he answered the
questions freely and without any objection from his counsel on the alleged active participation of the
Justices when he gave his testimony.
While we do not see any merit in petitioner’s first assigned error, we, however, agree with him that
his guilt was not adequately proven beyond reasonable doubt by the prosecution. Petitioner Cosep is
ACQUITTED on ground of reasonable doubt.

Rodrigo v. Sandiganbayan GR 125498 Feb. 18, 1999- Czar


F: The provincial auditor filed a criminal complaint for estafa before the Ombudsman against petitioners.
Thereafter, the Ombudsman approved the filing of information for violation of Sec. 3 (e) of RA 3019
before the Sandiganbayan. Petitioners thereafter filed before the Sandiganbayan a motion to quash the
information, but was denied by the latter. Hence, this petition for Certiorari.

I: Whether the Ombudsman committed grave abuse of discretion in filing the information against the
petitioners

H: This Court, moreover, has maintained a consistent policy of non-interference in the determination of
the Ombudsman regarding the existence of probable cause, provided there is no grave abuse in the
exercise of such discretion. In a recent decision, this Court, quoting Young vs. Office of the Ombudsman,
stated the rationale for this rule… the rule is based not only upon respect for the investigatory and
prosecutor powers granted by the Constitution to the Office of the Ombudsman but only upon
practicality as well. Otherwise, the functions of the Court will be grievously hampered by innumerable
petitions assailing the dismissal of investigatory proceedings conducted by the Office of the Ombudsman
with regard to complaints filed before it, in much the same way that the courts would be extremely
swamped if they could be compelled to review the exercise of discretion on the part of the fiscals or
prosecuting attorneys each time they decide to file an information in court or dismiss a complaint by a
private complainant

People v. Hui 338 SCRA 2000- Czar


F: Zheng Bai Hui and Nelson Hong Ty were sentenced by the Court to suffer the death penalty for the
sale of methamphetamine hydrochloride, otherwise known as “shabu.” Hence this automatic review
contending that the trial court erred in depriving the accused of the prized ideal of the “cold neutrality
of an impartial judge as part of the procedural process.

I: Whether or not the accused was unduly and arbitrarily convicted by the court?

H: In any case, a severe examination by a trial judge of some of the witness for the defense in an effort
to develop the truth and to get at the real facts affords no justification for a charge that he has assisted
the prosecution with an evident desire to secure a conviction, or that he intimidated the witnesses for
the defense. The trial judge must be accorded a reasonable leeway in putting such questions to
witnesses as maybe essential to elicit relevant facts to make the record speak the truth. Trial judges in
this jurisdiction are judges of both the law and the facts, and they would be negligent in the
performance of their duties of they permitted a miscarriage of justice as a result of a failure to propound
a proper question to a witness which might develop some material hearing upon the outcome. In the
exercise of sound discretion, he may put such questions to test the credibility of the witness and to
extract the truth. He may seek to draw out relevant and material testimony though that testimony may
tend ti support or rebut the position taken by one or the other party. It cannot be taken against him is
the clarificatory questions he propounds happen to reveal certain truths which tend to destroy the
theory of one party.

People v. Cabiles 341 SCRA 2000 – Czar


F: In view of all foregoing, the Court finds:

In Criminal Case No. U -8390:

Accused , Cabiles GUILTY beyond reasonable doubt of the crime of Attempted Homicide.

In Criminal Case No. U-8389:


Accused, De Los Reyes GUILTY beyond reasonable doubt of the crime of Illegal Possession of Firearms
and Ammunitions.

In Criminal Case No. U-8391:


Accused, Cabiles GUILTY beyond reasonable doubt of the crime of Illegal Possession of Firearms and
Ammunitions.

In Criminal Case No. U-8392:


Accused, Cabiles GUILTY beyond reasonable doubt of the crime of Murder.

Hence, this automatic review.

I: Whether or not the Judge rendered a judgment convicting both the accused of the crimes charged?

H: Appellants complain that the trial judge actively participated in the prosecution of the case and thus
rendered a biased judgment. They particularly assail the questions asked by the judge, of witness Arman
Pamarang, following the redirect examination, as indicative of his desire to convict appellants. Our
scrutiny of the detailed questions asked by the trial judge, however, fails to disclose any bias on his part
which would prejudice appellants. The questions were clarificatory. It is a judge’s prerogative to ask
clarificatory queries out of the truth.

Gozum v. Liangco 339 SCRA 253 – Czar


F: This is an administrative complaint for the dismissal of Judge Liangco for serious misconduct, gross
inefficiency and incompetence. This is relation to his handling of a petition for declaratory relief by the
Sangunniang Bayan of San Luis, Pampanga which sought his legal opinion on the validity of Resolution
No. 34-96 which provided that Lot No. 11 belonging to the municipality of San Luis, but occupied by the
family of Gozun, be used for the construction of Rural Health Center of San Luis, Pampanga.

I: What constitutes an impartial judge?

H: It is the duty of the members of the bench to avoid any impression of impropriety to protect the
image and integrity of the judiciary. A judge who tarnishes the image of the judiciary or bring it to public
contempt, dishonor or disrespect must be administratively dealth with and punished accordingly. The
rule is applied more strictly to municipal, metropolitan and regional trial court judges. This is because
they are judicial front liners who have direct contact with the litigating parties.

Soriano v. Angeles 339 SCRA 253 – Czar


F: In acquitting private respondent, private Judge Adoracion Angeles found it incredible that petitioner
did not resist or even say anything when private respondent allegedly assaulted him and that none of
the four barangay tanods who were near him came to his aid. She thought that if petitioner had indeed
been attacked, he would have suffered more serious injuries than a contusion on the forehead,
erythema on the chest, and a lacerated wound on the lower lip. Respondent judge also excluded from
the evidence the testimonies of petitioner and barangay tanod Montoya on the ground that their
testimonies had not been formally offered in evidence. Hence this petition for certiorari. Petitioner
alleges that the decision is void because it was not rendered by an impartial tribunal.

I: Whether or not the decision was rendered by an impartial court?


H: It is pertinent at this point to cite certain principles laid down by the Court regarding the
disqualification of a judge for lack of the objectivity that due process requires. It is settled that mere
suspicion that a judge is partial to one of the parties is not enough; there should be evidence to prove
the charge. Bias and prejudice cannot be presumed, especially weighed against a judges sacred
allegation under oath of office to administer justice without respect to any person and do equal right to
the poor and the rich. There must be a showing of bias and prejudice stemming from an extrajudicial
source resulting in an opinion in the merits on some basis other than the judge learned from his
participation in the case.

Petitioner’s claim that respondent judge was biased is belied by his failure to move for respondent
judges inhibition. Petitioner’s claim that he did not so because of his “belief and desire for said
respondent judge to finally return to her normal sense of fairness is a feeble excuse. His failure to file
such motion stands as one mark stark difference between this case and Galman since the private
prosecutors in the latter case lost no time in seeking the disqualification of the members of the
Sandiganbayan on ground of manifest bias and partiality for the defense.

Villanueva v. Malaya 330 SCRA 278- Czar


F: Petitioner Erlinda Mariano Villanueva, legally adopted daughter of Irene Mariano, alleges that the
subject property is one of several properties that she inherited from Don Mariano. An Indenture of
Extrajudicial Settlement of Estate was executed by Irene and Jose, which designates a pro-indiviso
shares to Jose, Erlinda and Irene (administrator). By virtue of the indenture, TCT in the name of Don
Mariano was cancelled and issued in lieu thereof in the names of the heirs. Based on a Deed of Sale, it
appears that Irene conveyed the disputed property to a certain Raul Santos. Irene died, which prompted
Jose and Erlinda to file a complaint against Santos for annulment of the deed of sale on the ground of
forgery and simulated sale.

I: Whether or not the court rendered a just and impartial decision?

H: In holding Santos to be Irene’s successor-in-interest because of his being a subsequent vendee to the
property, respondent court in effect ruled upon the validity of the Deed of Sale of April 15, 197, a matter
over which it had no jurisdiction to decide and which is properly the subject of the pending Civil Case
No. 88-1506. Third, respondent court based the right to redeem upon the purported transfer of
ownership to Santos of the property. The right of redemption is explicitly conferred by Sec. 29 Rule 39 of
the Rules of Court on the judgment debtor and his sucessors-in-interest; it is not conditioned upon
ownership of the property but by virtue of a writ of execution directed against the judgment debtor.

Almendra v. Asis 330 SCRA 69 AM RTJ-00-1550– Czar


F: Three administrative complaints were filed by Antonio Almendra against Judge Asis for partiality,
gross ignorance of the law, knowingly rendering unjust judgment and violation of the Anti-Graft and
Corrupt Practices Act. Thus, Almendra prayed that respondent judge inhibit himself from all pending
cases involving complainant, having exhibited manifest partiality against him.

I:Whether or not Judge Asis was partial and unfair in the rendering of judgment?

H: In his three administrative complaints, Antonio Almendra repeatedly imputed charges of bias,
partiality and unfairness against respondent judge. However, the fact that respondent judge ruled
against complainant in the three cases filed before him did not amount to partiality against said
complainant or warrant the conclusion that respondent rendered an unjust judgment where he acts in
bad faith, malice, revenge or some other similar motive. Absent the element of bad faith, an error of
judgment not being a ground for disciplinary.

Consequently, respondent judge may be held liable for rendering the wrongful judgment in Civil Case
No. 214 despite the existence of a final and executory judgment of the Court of Appeals involving the
same parties, subject matter and cause of action. Aside from violating a fundamental rule that no
amendments may be made to a final and executory decision, respondent judge acted contrary to the
ruling of a higher court. Thus, respondent judge is liable for serious inefficiency.

Dayot v. Garcia 353 SCRA 280- Czar


F: Complainant Dayot was convicted by Respondent Judge Garcia of the MTC for Grave Salnder. He was
sentenced to suffer the penalty of imprisonment of 1 month and 1 day to 4 months of arresto mayor,
which was affirmed by the RTC but increased the penalty to 3 months of arresto mayor as minimum to 1
year and 1 day of prission correctional as maximum.. Likewise amount of moral damages are increased
to P10,000.00. He filed for review but was denied due course In the present case, complainant alleges
that respondent judge committed misconduct of office,abuse of authority and oppression when he
issued the warrant of arrest and ordered complainants detention despite the pendency of a motion for
reconsideration as this Court had yet to resolve the petition with finality; that he filed a motion to lift an
order discrediting his service of sentence after considering that his service was made outside the prison
cell.

I: Whether or not Judge acted with partiality and arbitrarily?

H: Complainant charges respondent Judge with misconduct of office, abuse of authority and oppression
when he issued the warrant of arrest and ordered complainant's detention despite the pendency of the
motion for reconsideration in before this Court. It should be noted that complainant filed a
Manifestation in reiterating the fact that a warrant of arrest was issued by respondent judge despite the
pendency of the motion for reconsideration but the said Manifestation was merely noted without action
in view of the fact that the petition for review on certiorari had already been denied for lack of merit
and the motion for reconsideration was likewise denied with finality. As to whether there was error on
the part of the respondent Judge in ordering the issuance of the warrant of arrest, complainant
addressed this matter in the Motion to Lift the said warrant of arrest which he filed with the respondent
Judge, wherein complainant argued that the petition before this Court is still pending. This motion was
however denied by respondent Judge in his Order. Whether the respondent Judge correctly denied the
motion is a judicial matter which is not a proper subject in an administrative proceeding.
Consequently, complainant's charge that respondent Judge failed to act on the Motion to Lift the arrest
warrant is untenable as he had issued an Order on June 25, 1998 denying the said motion.

While a judge may not always be subjected to disciplinary action for every erroneous order or decision
he renders, that relative immunity is not a license to be negligent or abusive and arbitrary in performing
his adjudicatory prerogatives (De Vera vs. Dames II, 310 SCRA 213). The issuance of the Order of
November 6, 1998 without the benefit of a hearing is a clear evidence of the judge's failure to
understand the limitations of his power and betrays his ignorance of the cardinal principles of due
process (Macasasa vs. Imbing, 312 SCRA 385). By unilaterally discrediting the period served outside the
jail without giving complainant a chance to be heard, respondent Judge failed to observe the
requirements of due process. HE WAS FINED.
People v. Hapa GR 125698 July 19, 2001 – Czar
F: The trial court rendered a decision finding accused Hapa, et.al GUILTY beyond reasonable doubt of
the crime of MURDER. Accused contend that the trial court erred in finding them guilty of murder,
instead of homicide only. They likewise questioned the authority of the judge who penned the decision
because he was not the one who conducted the trial and heard the proceedings of the case.

I: Whether or not the accused was not afforded an impartial judgment?

H: The fact that Judge Valencia who decided the case was not the one who heard the testimonies of the
witnesses would not automatically warrant a reversal of the decision.[33]Such fact constitutes no
compelling reason to jettison his findings and conclusions, and does not per se render his decision void.
It may be true that the trial judge who conducted the hearing would be in a better position to ascertain
the truth or falsity of the testimonies of the witnesses. However, it does not necessarily follow that a
judge who was not present during the trial can not render a valid and just decision. For the judge who
was not present during the trial can rely on the transcript of stenographic notes taken during the trial as
basis of his decision. Such reliance does not violate substantive and procedural due process of law.[34]

Aguirre v. people GR 144142 August 23, 2001 – Czar


F: Petitioner Yolanda Aguirre filed the instant petition for review on certiorari seeking to reverse and set
aside the Decision, dated November 25, 1999, of the Court of Appeals (CA) which affirmed her
conviction for violation of the Batasang Pambansa Blg. 22 (B.P. Blg. 22). She likewise assails that the
Court denied her due process when she was convicted for violation of BP 22, after the court declared
that she waived her right to present evidence when she tried to postponed the proceedings several
times.

I: Whether or not she was granted a fair trial?

H: The essential requirements of due process in this jurisdiction are well-established:


(1) There must be a court or tribunal clothed with judicial authority to hear and determine the
matter before it;
(2) Jurisdiction must be lawfully acquired over the person of the defendant or property which
is the subject of the proceeding;
(3) The defendant must be given an opportunity to be heard; and
(4) Judgment must be rendered upon lawful hearing.[9]
Applying the above test, the Court finds that petitioner in this case cannot feign denial of due process
because she had been given the opportunity to present her side.

In any case, the Court finds no cogent reason to deviate from the settled rule that factual findings of the
trial court are binding on the Supreme Court when supported by substantial evidence on record and
carry more weight when affirmed by the appellate court, as in this case. However, there is need to
modify the penalty imposed on petitioner

Puyat v. Zabarte 352 SCRA 738 – Czar


F: Zabarte commenced to enforced the money judgment rendered by the superior court of the state of
California. Petitioner challenged the decision of the CA which affirmed the RTC decision, who was
alleging that the answer filed by petitioner failed to tender any genuine issue as to the material facts.

I: Whether or not the CA acted in a manner contrary to law when it affirmed the order of the trial court
granting respondent’s motion for summary judgment rendering judgment against Puyat.

H: Summary judgment in litigation is resorted to if there is no genuine issue as to the material fact, other
than the amount of damages. If this verity is evident from the pleadings and the supporting affidavits,
depositions and admissions on file with the court, the moving party is entitled to such remedy as a
matter of course. The grounds relied upon by petitioner are contradictory, on the one hand, he insists
that the RTC take jurisdiction over the enforcement case in order to invalidate the foreign judgment,
yet, he avers that the trial court should not exercise jurisdiction over the same case on the basis of
forum non conveniens. Not only do these defenses weaken each other but they bolster the finding of
the lower courts that he was merely maneuvering to avoid or delay payment of his obligation. His
petition is hereby denied.

Baritua v. Mercader 350 SCRA 86- Czar


F: Dominador Mercader boarded with Petitioner JB Lines, Inc Bus No. 142. However, the bus met an
accident, which resulted to the death of the former. The RTC, after dure trial, rendered the aforesaid
assailed decision in favor of Mercader, which was affirmed by the Court of Appeals. Hence, this Petition
assailing that CA likewise arbitrarily disregard petitioners constitutional right to procedural due process
and fairness when it ignored and thrust aside their right to present evidence.

I: Whether or not the judges who heard the case were biased and impartial?

H: These arguments are not meritorious. First, judges cannot be expected to rely on the testimonies of
every witness. In ascertaining the facts, they determine who are credible and who are not. In doing so,
they consider all the evidence before them. In other words, the mere fact that Judge Noynay based his
decision on the testimonies of respondents’ witnesses does not necessarily mean that he did not
consider those of petitioners. Second, we find no sufficient showing that Judge Operario was
overzealous in questioning the witnesses. His questions merely sought to clarify their testimonies. In
all, we reject petitioners’ contention that their right to adduce evidence was violated.

We therefore believe that there is no reason to overturn the assailed CA Decision, which affirmed that
of the RTC. It is a well-settled rule that the trial court’s factual findings, when affirmed by the appellate
court, are conclusive and binding, if they are not tainted with arbitrariness or oversight of some fact or
circumstance of significance and influence.[24] As clearly discussed above, petitioners have not
presented sufficient ground to warrant a deviation from this rule.

Barbers v. Laguio 351 SCRA 606 – Czar


F: A complaint for alleged violation of Art. 20 (knowingly rendering unjust judgment) of RPC, Sec. 3 (e) of
RA 3019 and 4 (b) and (c) of RA 6713 were filed against respondent judge in connection with his
acquittal of the accused Lawrence Wang in 3 criminal cases. Petitioners now claim that the Judge acted
with grave partiality in acquitting the accused.

I: Whether or not respondent judge was partial in acquitting the accused.


H: Anent the charge that respondent judge displayed bias and partiality during the trial when he asked
numerous clarificatory questions, we note that the participation of respondent judge in the conduct of
the trial was not by itself condemnable. He could not be expected to remain an impassive and remote
presence during the proceedings, prohibited from asking questions when proper and necessary, yet all
these were done without necessarily transgressing the boundaries of impartiality decreed by judicial
ethics. The mere fact that the presiding judge asked clarificatory questions during the trial does not
make him a biased judge.[26] “He must be accorded a reasonable leeway in asking questions to
witnesses as may be essential to elicit relevant facts and to bring out the truth.”[27] Differently stated,
“questions to clarify points and to elicit additional relevant evidence are not improper. The judge being
the arbiter may properly intervene in the presentation of evidence to expedite and prevent unnecessary
waste of time.”[28] Complaint against Judge was dismissed.

People v. Herida 353 SCRA 650 – Czar


F: Accused Herida is found by the court guilty beyond reasonable doubt of the crime of Murder and was
sentenced to suffer the penalty of reclusion perpetua. Hence this instant petition alleging that the court
seriously erred when it denied the accused his constitutional right to due process of law by acting with
obvios bias and prejudice during the trial of this case.

Appellant avers that the trial court judge exhibited bias or prejudice against him. Appellant points out
that over seventy percent (70%) of the testimonies of the prosecution’s material witnesses were elicited
by the judge, while the cross-examination of the defense witnesses was to a large extent conducted by
the judge himself. He submits that under these circumstances, his right to a fair and impartial trial was
violated.

I: Whether or not accused was denied due process with obvious bias and prejudice?

H: The transcripts of the proceedings show that the trial court did intensively question the witnesses.
For instance, of the 182 questions asked of prosecution eyewitness Tomas Baniquid, 79 or roughly 43%
of the total came from the judge. However, we note that the judge also intensively questioned
witnesses of the defense. When appellant took the stand, 63 questions were added, with 27 or
approximately 43% asked by the judge. The intensive questioning of the witnesses, however, was
necessary. The sworn affidavits of the material witnesses were adopted as their direct testimonies,
subject to cross-examination. Since affidavits are generally takenex parte and are often incomplete or
even inaccurate for lack of searching inquiries by the investigating officer,[24] the trial court had to ask
many questions to clarify important matters. The judge’s behavior under this circumstance cannot be
considered biased or prejudiced. Judges are, after all, not mere referees in a boxing bout, whose only
task is to watch and decide the results.[25] Judges have as much interest as counsel in the orderly and
expeditious presentation of evidence and have the duty to ask questions that would elicit the facts on
the issues involved, clarify ambiguous remarks by witnesses, and address the points that are overlooked
by counsel. DECISION AFFIRMED.

People v. Medenilla GR 1311638 Mar. 26, 2001 – Czar


( I read the full case, its all about the contempt proceedings no related issue on aspects in
proceedings)
F: In a decision finding Medenilla guilty of the offense of violation of Dangerous Drugs Act of 1972, the
counsel for the convict was directed to comment on why he should not be cited for contempt for
anchoring the defense of his client on an alleged Supreme Court circular, which in reality was never
issued by this Court. He submitted his comment, in which he claimed that he immediately assumed that
the circular was issued by the Supreme Court and as such he used this alleged circular to seek the
acquittal or at least the reduction of the penalty imposed on his client. The Court found Atty. Arias guilty
of contempt.

I: Whether or not the Court correctly found Atty. Arias in contempt?

H: In the present case, Atty. Arias was evidently remiss in his duties towards his client and this Court.
We find it hard to believe that Atty. Arias was not aware that a circular regarding the requirement of
conducting qualitative and quantitative tests of seized illegal drugs does not exist. We cannot fathom
his excuse that he merely relied on the assertion given to him "off the record" by the forensic chemist
witness regarding the alleged circular and, from there, made a leap of faith and anchored the life and
case of his client on such an unfounded assertion. This kind of conduct is undeniably contradictory to
the training of a lawyer which is to always verify the validity of the legal provisions which he will use in
his case. Thus, we cannot accept the excuse offered by Atty. Arias that he was misled by the claim of
the forensic chemist witness that a circular requiring qualitative and quantitative tests of seized illegal
drugs was issued by this Court. It is our view that Atty. Arias deliberately tried to mislead the trial court
and this Court into believing the existence of such alleged circular. SUSPENDED.

People v. Rivera GR 139180 July. 31, 2001 – Mikee


Rivera was found guilty by the RTC of raping his 13 year old daughter. He invokes his right to due
process of law upon these claims: (a) the trial judge disallowed his lawyer from cross-examining his
daughter concerning the latter’s sworn statements on the ground of irrelevance and immateriality; (b)
the trial court denied the motion made by accused-appellant’s counsel de oficio to postpone the cross-
examination of Dr. Barin, the examining physician, because of which the said counsel consequently
waived the cross-examination of Dr. Barin; (c) the judge propounded numerous questions to accused-
appellant during his cross-examination by the prosecutor; and (d) the trial court’s decision was
promulgated just one day after accused-appellant submitted his memorandum.
I: was Rivera accorded due process?
R: Yes.
Procedural due process simply means that a person must be heard before he is condemned. This dure
process requirement is a part of a person’s basic rights, not a mere formality that may be dispensed with
or performed perfunctorily. Considering both the evidence and the law applicable to this case, we hold
that accused –appellant has been accorded his right to due process.
a- The issues of acts of lasciviousness in the Sworn Statements were not asked during the direct
examination, she may not be questioned on this matter.
b- The witness of Dr. Barin did not require considerable study and extraordinary preparation on
the part of defense counsel for the purpose of cross-examination. It seems Atty. Bansil, counsel
de oficio no longer found it necessary to cross-examine Dr. Barin.
c- Where the trial court is judge both of the law and of the facts, it is oftentimes necessary in the
due and faithful administration of justice for the presiding judge to re-examine a witness so that
his judgment, when rendered, may rest upon a full and clear understanding of the facts.[44] Our
reading of the transcript of stenographic notes in this case shows that the trial judge merely
wanted to clarify certain points relating to the defense of accused-appellant and not to establish
his guilt.
d- The decision rendered by the trial court gives a clear account of the facts and the law on which it
is based. It discusses in full the court’s findings on the credibility of both the prosecution and
defense witnesses and its evaluation of the evidence of both parties.

People v. Basques GR 144035 Sept. 27, 2001 – Mikee


Basquez appealed the RTC’s finding of him being guilty for rape on the fround that the trial judge
showed manifest bias and partiality against the accused by virtually acting as prosecutor.
R:The participation of judges in the conduct of trials cannot be condemned outrightly. They cannot be
expected to remain always passive and stoic during the proceedings. After all, they are not prohibited
from asking questions when proper and necessary. In fact, this Court has repeatedly ruled that judges
“must be accorded a reasonable leeway in asking questions to witnesses as may be essential to elicit
relevant facts and to bring out the truth.”[17]
In the present case, there was an effort by the trial judge to arrive at the truth and to do justice to both
parties. It would be a distorted concept of due process if, in pursuance of that valid objective, he is to be
accused of unfairness. His inquisitiveness did not unduly harm appellant’s substantial rights. Rather, the
questions he propounded to the witnesses showed his intention to elicit the truth. This conduct is
expected of judges who, conscious of their responsibilities as magistrates, propound questions to
witnesses who give incomplete and obscure answers.

Cooperative Development v. DOLEFIL GR 137489 May 29, 2002 – Mikee


The intervenors aver that the Resolution of the Court of Appeals which denied the motion for
reconsideration of herein petitioner Cooperative Development Authority and also invalidated the
election of officers and members of the board of directors of DARBCI held during the special general
assembly on July 12, 1998, thus adversely affecting their substantial rights including their right to due
process when the validity of the election was never an issue in the appeal and they were never asked to
comment on the “Twin Motions For Contempt of Court and to Nullify Proceedings”.
Ruling:
We find, however, that the action taken by the Court of Appeals, 13th Division, on the “Twin Motions for
Contempt of Court and to Nullify Proceedings” insofar as it nullified the election of the officers and
members of the Board of Directors of DARBCI, violated the constitutional right of the petitioners-in-
intervention to due process. The requirement of due process is satisfied if the following conditions are
present, namely: (1) there must be a court or tribunal clothed with judicial power to hear and determine
the matter before it; (2) jurisdiction must be lawfully acquired over the person of the defendant or over
the property which is the subject of the proceedings; (3) the defendant must be given an opportunity to
be heard; and (4) judgment must be rendered upon lawful hearing.[49] The appellate court should have
first required the petitioners-in-intervention to file their comment or opposition to the said “Twin
Motions For Contempt Of Court And to Nullify Proceedings” which also refers to the elections held
during the general assembly on July 12, 1998. It was precipitate for the appellate court to render
judgment against the petitioners-in-intervention in its Resolution dated February 9, 1999 without due
notice and opportunity to be heard. Besides, the validity of the general assembly held on July 12, 1998
was not raised as an issue in CA-G.R. SP No. 47933.
Garcia v. Pajaro GR 141149 July 5, 2002 – Mikee
Petitioner Garci, an employee under the City Treasurer, was formally charged and was preventively
suspended for Unsatisfactory ratings and when an investigation was scheduled, he failed to Answer and
refused to honor the subpoena to submit himself for investigation. ex-parte investigation proceeded
and the information gathered and submitted testimonies to support the allegations became the basis
for the decision by the Department of Finance.
I: Was there due process?
R: Yes.
The essence of due process in an administrative proceeding is the opportunity to explain one’s side,
whether written or verbal. The constitutional mandate is satisfied when a petitioner complaining about
an action or a ruling is granted an opportunity to seek reconsideration. So long as the parties are given
the opportunity to explain their side, the requirements of due process are satisfactorily complied
with.[44] Moreover, this constitutional mandate is deemed satisfied if a person is granted an
opportunity to seek reconsideration of an action or a ruling
Brioso v. Mariano, (not Briaso) GR 137265, Jan. 31, 2003 – Mikee
In a land dispute, one of the parties died during the proceedings. The trial court failed to order the
appearance of his legal representative or heirs and merely issued an Order merely admitting
respondents' motion for substitution.
I: Was the substituted heir accorded due process?
R: Yes. despite the trial court's failure to adhere to the rule on substitution of a deceased party, its
judgment remains valid and binding on the following heirs, namely, Salvador, Concepcion and Ernesto.
Formal substitution of heirs is not necessary when the heirs themselves voluntarily appeared, shared in
the case and presented evidence in defense of deceased defendant.23 This is precisely because, despite
the court's non-compliance with the rule on substitution, the heirs' right to due process was obviously
not impaired.24 In other words, the purpose of the rule on substitution of a deceased party was already
achieved.

Macias v. Macias GR 1461617, Sept. 3, 2003- Mikee


Macias, the husband, filed a petition for nullity of marriage against his wife. Instead of filing her answer,
respondent filed a motion to dismiss which was denied. An order was subsequently issued setting the
case for hearing, however,respondent and counsel did not appear because it was received by the
respondent five days after the scheduled trial dates. On those days, petitioner was allowed to present
evidence ex parte. And unaware that the case was already submitted for decision, respondent filed an
MR on the decision denying the Motion to dismiss. The trial court granted the petition and declared the
nullity of the marriage.
Issue: was due process accorded to respondent during the proceedings?
No.
Due process is the very essence of justice itself. Where the rule of law is the bedrock of our free society,
justice is its very lifeblood. Denial of due process is thus no less than a denial of justice.
In the case at bar, the trial court did not observe the rudimentary principle of due process enshrined in
our Constitution. Neither did it comply with pertinent procedural rules.
More to the point, the trial court, without even waiting for respondent’s motion for reconsideration of
the Order denying her motion to dismiss, hurriedly set the case for hearing. Also, without allowing the
respondent to file her answer to the petition and knowing there was no joinder of issues as yet, the trial
court hastily authorized petitioner to present his evidence ex-parte.
Under the Rules, where the defending party fails to file his or her answer to the petition, the trial court
should order the prosecutor to intervene for the State by conducting an investigation to determine
whether or not there was collusion between the parties. Here, the trial court disregarded such
procedure. Obviously, the summary proceeding is a patent nullity.
And assuming arguendo that there was an answer filed by the respondent, still, the hearing of the case
on May 2 and 3, 2001 is a procedural flaw. As stated at the outset, respondent received the notice of
hearing only on May 8, 2001. So how could she be present in court on May 2 and 3?
We are convinced that respondent’s fundamental right to due process was blatantly transgressed by the
trial court. And resultantly, the proceedings conducted, including the trial court’s Decision, are void for
lack of due process.
Albior v. Auguis, AM P-01-1472, June 6, 2003 – Mikee
Two complaints for rape against the son of complainant Albior were filed in the MCTC where
respondent worked as Clerk of Court. He issued a detention order for the accused even without a
preliminary investigation or a warrant of arrest, and failed to inform the Judge regarding the filing of the
complaints. When a motion for his release was filed, respondent only issued an order to file the counter-
affidavits for the preliminary investigation. Only upon the filing of a petition for habeas corpus with the
RTC of Bohol was the accused granted an order for release. A complaint for usurpation of judicial
functions and negligence was filed against Auguis.
Issue: Did the acts of the Clerk of Court constitute a violation of the right to due process?
R: yes.
Respondent might have been motivated by a sincere desire to help the accused and his relatives. But as
an officer of the court, he should be aware that by issuing such detention order, he trampled upon a
fundamental human right of the accused. Because of the unauthorized order issued by respondent, the
accused Edilberto Albior was deprived of liberty without due process of law for a total of 56 days,
counted from his unlawful detention on January 27, 1999 until the issuance of the appropriate order of
commitment by the municipal judge on March 25, 1999.
Thus, the Court cannot condone nor take lightly the serious violation committed by the respondent.
Article III, Section 1 of the Constitution mandates:
No person shall be deprived of life, liberty or property without due process of law, nor shall any person
be denied the equal protection of the laws. (Underscoring ours)
Republic v. Sandiganbayan, GR 152154, Nov. 18, 2003- Mikee
respondents Imelda R. Marcos, Irene Marcos-Araneta, Ma. Imelda Marcos and Ferdinand R. Marcos, Jr.,
, sought reconsideration of the SC decision dated July 15, 2003 which ordered the forfeiture in favor of
the Republic of the Philippines of the Swiss deposits in escrow at the Philippine National Bank (PNB) in
the estimated aggregate amount of US$658,175,373.60 as of January 31, 2002. They contend that
FORFEITURE PROCEEDINGS UNDER R.A. 1379, IN RELATION TO THE EXECUTIVE ORDERS ARE
CRIMINAL/PENAL IN NATURE, HENCE, RESPONDENT HAS ALL THE RIGHTS IN FAVOR OF THE ACCUSED
UNDER THE CONSTITUTION
Issue: what is the nature of forfeiture proceedings under RA 1379?
Civil. In Almeda Sr., et al. vs. Perez, et al.,[6] we suggested a test to determine whether the proceeding
for forfeiture is civil or criminal:
“ . . . Forfeiture proceedings may be either civil or criminal in nature, and may be in rem or in personam.
If they are under a statute such that if an indictment is presented the forfeiture can be included in the
criminal case they are criminal in nature, although they may be civil in form; and where it must be
gathered from the statute that the action is meant to be criminal in its nature it cannot be considered as
civil. If, however, the proceeding does not involve the conviction of the wrongdoer for the offense
charged the proceeding is of a civil nature; and under statutes which specifically so provide, where the
act or omission for which the forfeiture is imposed is not also a misdemeanor, such forfeiture may be
sued for and recovered in a civil action.” (37 CJS, Forfeiture, Sec. 5, pp. 15-16)
In the case of Republic vs. Sandiganbayan and Macario Asistio, Jr.,[7] this Court categorically declared
that: The rule is settled that forfeiture proceedings are actions in rem and therefore civil in nature.The
proceedings under RA 1379 do not terminate in the imposition of a penalty but merely in the forfeiture
of the properties illegally acquired in favor of the State.
The preliminary investigation required prior to the filing of the petition, in accordance with Section 2 of
the Act, is expressly provided to be similar to a preliminary investigation in a criminal case. The
similarity, however, ends there for, if the investigation were akin to that in a criminal case but all the
other succeeding steps were those for a civil proceeding, then the process as a whole is definitely not
criminal. Were it a criminal proceeding, there would be, after preliminary investigation, a reading of the
information, a plea of guilty or not guilty, a trial and a reading of judgment in the presence of
respondents. But these steps, as above set forth, are clearly not provided for in the law.
Respondents were repeatedly accorded full opportunity to present their case, their defenses and their
pleadings. Not only did they obstinately refuse to do so. Respondents time and again tried to confuse
the issues and the Court itself, and to delay the disposition of the case.
Ty v. Banco Filipino Savings and Mortgage Bank, 422 SCRA 649 – Mikee 149797-98 13Feb20014
The judge in this case showed obvious impartiality towards the respondent bank, such as assuming that
service was made upon the parties without proof, and ordering Tala et al( parties in the case) to
produce certain documents despite failure by the respondent to tender the costs for the production.
And when these were not produced, the court ordered other documents to be produced by Tala which
were not sought for by bank.
I: was there undue interference by the judge?
R: Yes.
There is undue interference where the judge’s participation in the conduct of the trial tends to build or
bolster a case for one of the parties. The role of the trial judge in the conduct of judicial proceedings
should only be confined to promote the expeditious resolution of controversies and prevent
unnecessary waste of time or to clear up some obscurity.
While the trial court may interfere in the manner of presenting evidence in order to promote the
orderly conduct of the trial, the final determination of what evidence to adduce is the sole prerogative
of the contending parties. Courts, while not unmindful of their primary duty to administer justice,
without fear or favor and to dispose of cases speedily and in as inexpensive a manner as us possible for
the court and the parties, should refrain from showing any false impression in the minds of the litigants.
For obvious reasons, it is the bounden duty of all to strive for the preservation of the people’s faith in
our courts.
People v. Larranaga, *421 SCRA 530 – Mikee
Respondents were found guilty by the RTC of kidnapping and serious illegal detention. They appealed,
and one of the ground was violation of due process.
Issue: was there a violation of appellant’s due process?
R:no, there was none.
In evaluating a due process claim, the court must determine whether life, liberty or property interest
exists, and if so, what procedures are constitutionally required to protect that right.
Due process of law is the primary and indispensable foundation of individual freedoms; it is the basic
and essential term in the social compact which defines the rights of the individual and delimits the
powers which the State may exercise. In evaluating a due process claim, the court must determine
whether life, liberty or property interest exists, and if so, what procedures are constitutionally required
to protect that right. Otherwise stated, the due process clause calls for two separate inquiries in
evaluating an alleged violation: did the plaintiff lose something that fits into one of the three protected
categories of life, liberty or property? And if so, did the plaintiff receive the minimum measure of
procedural protection warranted under the circumstances?
For our determination, therefore, is whether the minimum requirements of due process were accorded
to appellants during the trial of these cases.
R. Transport v. Philhino 494 SCRA 630 – Mikee
Due to a witness’ and the petitioner’s counsel non-appearance for her cross-examination, the trial court
denied the motion to reset the cross-examination and the testimony of that witness was striken out. The
case was already dragged out for four years by petitioner and it had endured more than the usually
acceptable number of continuances.
Issue: was the trial court’s order violative of due process?
Ruling: no. Petitioner’s contention that it was denied due process is not well-taken. The most basic
tenent of due process is the right to be heard. Where a party had been afforded an opportunity to
participate in the proceedings but failed to do so, he canot complain of deprivation of due process. Due
process is satisfied as long as the party is accorded an opportunity to be heard. If it is not availed of, it is
deemed forfeited without violating the Bill of Rights.
Trans Middle East v. Sandiganbayan 499 SCRA 308- Mikee
The PCGG sequestered shares held bu petitioner in PCIB and the shares of Atty. Narciso in the same
bank. It now raises the issue that due process was not afforded to it.
R:When petitioner’s contention that it was denied due process, let it be stressed that where a party was
given an opportunity to participate in the proceedings but failed to do so, he cannot complain of
deprivation of due process. Here, petitioner was afforded the opportunity to be heard. To be heard does
not only mean presentation of testimonial evidence in court-one may also be heard through pleadings,
and where opportunity to be heard through pleadings is accorded, as in this case, there is no denial of
due process.
Respondent court directed petitioner to file its comment on the manifestations and motions of Atty.
Narciso. However, despote notice petitioner. Records disclose that it filed the same only on May 29,
1997 after the incidents were deemed submitted for resolution. Therefore, petitioner cannot complain
it was deprived of due process.
Uy v. First Metro 503 SCRA 704 - Mikee
A collection suit was filed against petitioner by respondent. Hearings were scheduled and cancelled and
moved due to petitioner’s counsel’s request. Instead of presenting evidence, counsel requested for a
postponement and resetting of hearing. Thus, later the court ordered that petitioner’s right to present
evidence was deemed waived and the case was submitted for decision. Judgment was made in favor
ofrespondent.
Issue: can petitioner seek for new trial on the ground of gross negligence of petitioner’s previous
counsel in failing to attend the hearing for the reception of evidence thus impairing his rights to due
process?
Ruling: in this instance, no.
Petitioner’s counsel’s inexcusable neglect did not amount to petitioner’s deprivation of due process of
law. The right to due process safeguards the opportunity to be heard and to submit any evidence one
may have in support of his claim or defense. In the instant case, petitioner was given several
opportunities to be heard and to submit evidence but he squandered them. Indeed, from lethargy is
misfortune born.

Deutsche Bank v. Chua 481 SCRA 672 – Mikee


Courts are required to resolve the admissibility of the evidence offered immediately after the objection
is made or within a reasonable time. It must be made during the trial and at such time as will give the
other party an opportunity to meet the situation presented by the ruling. Courts are further mandated
to state the reason or reasons for their ruling if there are two or more objections by the other party. Due
process requires no less. In this case, we note that, in the Order of September 5, 2003, the trial court
failed to state its reasons for reconsidering its earlier order , and for admitting the documentary
evidence of the petitioners only as part of the testimonies of its witnesses.
People v. Santos 501 SCRA 325 - Mikee
Santos was found guilty of raping a five year old girl.he faults the trial court with acting as the
prosecutor and the judge at the same time for allegedly initiating and propounding the questions, short
of supplying the desired answer from the witness.
Judges are not mere referees like those of a boxing bout, only to watch and decide the results of a
game—they should hace as much interest as counsel in the orderly and expeditious presentation of
evidence, calling attention of counsel to points at issue that are overlooked, directing them to ask
question that would elicit the facts on the issues involved, and clarifying the ambiguous remarks by
witnesses.
Victoriano v. People 509 SCRA 483- Mikee
Right to cross-examinatin: where a party has had the opportunity to cross-examine a witness but failed
to avil himself of it, he necessarily forfeits the right to cross-examine and the testimony given on direct
examination of the witness will be received and or allowed to remain in the record.
Santos v. DOJ 543 SCRA 70- Mikee

DBP v. Teston 545 SCRA 422 – Val

FACTS: Respondent purchased from petitioner two (2) parcels of land which he later defaulted on, thus
causing the rescission of the contract. Respondent had voluntarily offered the two parcels of land for
inclusion in the CARP. Respondent filed a Petition against DBP, alleging that his obligation to DBP was
assumed by the government through the Land Bank after such became covered by the CARP, and that
the operation of said law extinguished DBP’s right to rescind the sale. On appeal the CA ordered DBP
to return the alleged down payment made by petitioner in the amount of p1,000,000.00 but this was
neither raised as an issue.

ISSUE: Whether the CA’s order was valid

RULING: No. x x x It is elementary that a judgment must conform to, and be supported by, both the
pleadings and the evidence, and must be in accordance with the theory of the action on which the
pleadings are framed and the case was tried. The judgment must be secudum allegata et probate.

Due process considerations justify this requirement. It is improper to enter an order which
exceeds the scope of relief sought by the pleadings, absent notice which affords the opposing party an
opportunity to be heard with respect to the proposed relief. The fundamental purpose of the
requirement that allegations of a complaint must provide the measure of recovery is to prevent surprise
to the defendant.

Ruivivar v. OMB 565 SCRA 324- Val

FACTS: The Ombudsman found the petitioner administratively liable and imposed the penalty of
reprimand for serious misconduct.
The records show that the petitioner duly filed a motion for reconsideration on due process
grounds (i.e., for the private respondent’s failure to furnish her copies of the affidavits of
witnesses). The Ombudsman issued and its Order furnishing her with copies of the private
respondent’s witnesses, together with the “directive to file, within ten (10) days from receipt, such
pleading which she may deem fit under the circumstances.” Petitioner received the private respondent’s
witnesses’ affidavits but she did not choose to controvert these affidavits or to file a supplement to her
motion for reconsideration.

ISSUE: WHETHER OR NOT PETITIONER WAS DENIED OF THE CONSTITUTIONAL GUARANTEE


TO DUE PROCESS
RULING: No. Under the concept of “due process,” on the other hand, a violation occurs when a
court or tribunal rules against a party without giving him or her the opportunity to be heard. At
the same time, the petitioner – who had the same opportunity to rebut the belatedly-furnished
affidavits of the private respondent’s witnesses – was not denied and cannot now claim denial
of due process because she did not take advantage of the opportunity opened to her at the
Ombudsman level.
On this score, the Supreme Court held that “a party cannot feign denial of due process where
he had the opportunity to present his side”. The respondent herein was given the opportunity
not normally accorded, to present her side, but she opted not to do so which is evidently fatal
to her cause.” [

Borromeo v. Garcia 546 SCRA 543 – Val


FACTS: This is a petition for review on certiorari which seeks the reversal of the Decision and
Resolution of the Court of Appeals which affirmed the Orders of the Regional Trial Court (RTC) to
annotate an easement of road right of way on the title of petitioner Borromeo Bros. Estate, Inc. in favor
of respondent Edgar John A. Garcia.
The records reveal that the cadastral court furnished petitioner its Order, which reiterated its
previous order. More importantly, the cadastral court heard petitioner’s motion for reconsideration in
open court and likewise allowed the parties to file their respective memoranda prior to ruling on the
motion for reconsideration.
Petitioner alleged these errors: (1) the appellate court erred in not reversing the Order of the
cadastral court despite (a) the nullity of the Order for the denial of petitioner’s substantive and
procedural right to due process
ISSUE: Whether there was denial of due process

RULING: No. The cadastral court did not deny petitioner of its right to due process of the law. The
essence of due process is found in the reasonable opportunity to be heard and submit any evidence in
support of one’s defense. What the law proscribes is the lack of opportunity to be heard.
Indeed, deprivation of the right to due process cannot be successfully invoked where a
party was given the chance to be heard on his Motion for Reconsideration as what happened in the
instant case.
In fine, the records of the instant case show that (1) there was substantial evidence to support
the annotation of the easement of right of way on the title of petitioner in favor of respondent and (2)
the requirements of due process were sufficiently met.

Cesa v. OMB 553 SCRA 357- Val


FACTS: The Office of the Ombudsman suspended Cebu City Treasurer Cesa for six months without pay
for tolerating illegal practices relative to the granting of cash advances to paymasters. During a
surprise audit it was discovered that the paymaster was granted more advances without liquidating
previous advances and she along with other city officials were charged before the OMB .the
Ombudsman found Cesa and the other city officials guilty of neglect of duty. Before the Court of
Appeals, Cesa argued that there was lack of due process because the complaint filed against him was
not verified.

ISSUE: Was Cesa’s right to due process violated when he was suspended for six months as city
treasurer?
RULING: The basic due process requirements in administrative cases: opportunity to explain one’s side or
seek a reconsideration of the ruling.
There is no denial of due process if records show that hearings were held with prior notice to
adverse parties. Even without notice, there is no denial of procedural due process if the parties were
given the opportunity to be heard.
The procedural lapses, if any, were cured when Cesa participated in the preliminary conference,
submitted his counter-affidavit and supplemental counter-affidavit, actively participated in the
proceedings by cross-examining witnesses, and filed a motion for reconsideration before the Office of
the Ombudsman. Cesa was given every opportunity to explain his side and to present evidence in his
defense during the administrative investigation.

DAR v. Samson 554 SCRA 500- Val


FACTS: Samson applied for exemption from the coverage of CARP over nine (9) parcels of land which
was granted. Petitioners-farmers filed an Opposition/Petition alleging that the lands were fully
developed and planted with variety of plants, and to which some of them have their farm houses built.
DAR ordered to segregate the areas with agricultural developments and cover the same under CARP and
exempting the balance.
Samson assailed the Order before the Office of the President arguing that he was not notified of the
appeal;. Respondents argue that they were denied due process because they were not able to
participate in the proceedings before the DAR and that their appeal with the Office of the President did
not cure the said procedural lapse.

ISSUE: WHETHER THE COURT OF APPEALS ERRED IN HOLDING THAT RESPONDENTS WERE DENIED DUE
PROCESS OF LAW.

RULING: No. It is important to reiterate that administrative agencies are not bound by the technical
niceties of law and procedure and the rules obtaining in the courts of law. In administrative
proceedings, a fair and reasonable opportunity to explain one’s side suffices to meet the requirements
of due process.
In administrative proceedings, procedural due process has been recognized to include the following: (1)
the right to actual or constructive notice of the institution of proceedings which may affect a
respondent’s legal rights; (2) a real opportunity to be heard personally or with the assistance of counsel,
to present witnesses and evidence in one’s favor, and to defend one’s rights; (3) a tribunal vested with
competent jurisdiction and so constituted as to afford a person charged administratively a reasonable
guarantee of honesty as well as impartiality; and (4) a finding by said tribunal which is supported by
substantial evidence submitted for consideration during the hearing or contained in the records or made
known to the parties affected.27
In any event, the Court agrees with petitioners that any procedural defect in the proceedings
before the DAR was cured when Samson appealed before the Office of the President. Although the
ruling of the DAR had attained finality, the Office of the President still entertained respondents’ appeal
thus giving them the opportunity to be heard.
Hilano v. People 551 SCRA 191 – Val

FACTS|: Petitioner was charged with two counts of Murder. During trial, Public Attorney's Office (PAO), took over
representing petitioner in view of the death of the latter's counsel. RTC found him guilty. Petitioner, filed with the RTC
a Petition for Relief and stated that he instructed his lawyer to file the necessary motion for reconsideration or
notice of appeal but his lawyer failed to file such
Petitioner contends that the negligence of his counsel de oficio cannot be binding on him for the latter's
defiance of his instruction to appeal automatically breaks the fiduciary relationship between counsel-client and
cannot be against the client who was prejudiced;

ISSUE: Whether there was a denial of due process

RULING: yes. In criminal cases, the right of an accused person to be assisted by a member of the bar is
immutable. Otherwise, there would be a grave denial of due process. Thus, even if the judgment had
become final and executory, it may still be recalled, and the accused afforded the opportunity to be heard
by himself and counsel.
While as a general rule, the failure of petitioner to file his motion for reconsideration within the 15-
day reglementary period fixed by law rendered the resolution final and executory, we have on some occasions
relaxed this rule. A strict and rigid application of rules that would result in technicalities that tend to frustrate rather
than promote substantial justice must be avoided.
In all criminal prosecutions, the accused shall have the right to appeal in the manner prescribed by law. While
this right is statutory, once it is granted by law, however, its suppression would be a violation of due process, a
right guaranteed by the Constitution.

Pasiona v. CA 559 SCRA 137- Val


FACTS: From an adverse judgment of the NLRC affirming the petitioner’s termination of employment for just
cause, petitioner elevated his case to the CA via a petition for certiorari which dismissed such. Almost five
months later, petitioner, filed the present petition for certiorari, alleging that despite his inquiries with his former
counsel, said counsel never informed him of the CA Decision and it was only upon coming to Manila to get his Bar
Examination Permit from the Supreme Court, when he discovered that a CA Decision had already been
promulgated. Petitioner asserts that he should be allowed to avail of the remedy of certiorari because he was
denied due process due to the recklessness and gross negligence of his former counsel.

ISSUE: was petitioner deprived of due process of law by reason of counsel's failure to file a motion for
reconsideration of the CA Decision

RULING: No. However, it is not only the gross negligence of counsel that would justify deviation from the principle
of finality of judgment. It should be coupled with the denial of due process to the client by reason of said negligence.
“The essence of due process is to be found in the reasonable opportunity to be heard and submit any
evidence one may have in support of one's defense. x x x Where opportunity to be heard, either through oral
arguments or pleadings, is accorded, there is no denial of due process.”
In this case, petitioner's situation is far better than those of the aggrieved parties in the above-
cited cases because he had been able to present all his evidence and fully ventilate his arguments before the LA,
then on appeal before the NLRC, and even in his petition for certiorari before the CA.

Bibas v. OMB 559 SCRA 591 – Val


FACTS: Subject of petitioner’s petition before the appellate court was the Order of the Office of the
Ombudsman finding her guilty of Dishonesty and dismissing her from government service due to a cash
shortage of P989,461.10.
Petitioner thereupon filed a petition for certiorari with the Court of Appeals which, dismissed it
outright on procedural grounds, namely, an original action for certiorari was the wrong remedy, the
proper remedy being appeal; petitioner failed to state the date she received the assailed orders; only
photocopies of the assailed orders were submitted; and there was no explanation why service was not
done personally . The CA dismissed such hence, the present petition.

ISSUE: whether the Court of Appeals erred in dismissing petitioner’s action for certiorari by a strict
application of procedural rules and of the rule that negligence of counsel is binding on the client.

RULING: No. petition denied.


If the incompetence, ignorance or inexperience of counsel is so great and the
error committed as a result thereof is so serious that the client, who otherwise has a
good cause, is prejudiced and denied his day in court, the litigation may be reopened to
give the client another chance to present his case. In a criminal proceeding, where
certain evidence was not presented because of counsel’s error or incompetence, the
defendant in order to secure a new trial must satisfy the court that he has a good
defense and that the acquittal would in all probability have followed the introduction of
the omitted evidence.

Interestingly, petitioner admits that “[t]he merits of [her] case have been ventilated well enough
both in the Petition itself and the Reply to the Comments of [the] COA” which she filed with this
Court. If she fails then to present a strong case through the pleadings, there would be no point
remanding her case to the appellate court. As will be shown below, petitioner has failed to do just
that.
A finding of guilt in an administrative case would have to be sustained for as long as it is supported
by substantial evidence .

Espina v. Cerujano 550 SCRA 107 – Val


FACTS: In a Criminal Case for Robbery in Band with Multiple Homicide, Regional Trial Court convicted as
charged all the accused therein and the decision became final and executory .Petitioner filed a Motion
to Dismiss Criminal case on the ground that the Anti-Subversion Law had been repealed; such was
granted.
Respondents, thus filed an administrative complaint for conduct prejudicial to the best interest
of the service against petitioner before the Department of Justice (DOJ), for The grounds relied upon by
Espina ha[ve] nothing to do with the case at all, as the case decided with finality by the Court is Robbery
in Band with Multiple Homicide and not a violation of the Anti-Subversion Law. After investigation, the
Secretary of Justice found petitioner liable for grave misconduct and recommended to the President his
dismissal which was later granted.

ISSUE: Can one charged with Conduct Grossly Prejudicial to the Best Interest of the Service be found
guilty of Grave Misconduct?

RULING: NO. (a) a basic requirement of due process is that a person must be duly informed of the
charges against him and that (b) a person can not be convicted of a crime which he was not
charged. Administrative proceedings are not exempt from basic and fundamental procedural principles,
such as the right to due process in investigations and hearings
Grave misconduct is penalized by dismissal from service. On the other hand, conduct grossly
prejudicial to the best interest of the service is penalized by dismissal from service only on the second
offense; on the first offense, the penalty is suspension for six months and one day, to one year.
Geronga v. Varela 546 SCRA 429 – Val
FACS: Petitioner was involved in two administrative cases. Respondent referred such to the City Legal
Officer for investigation which resulted in a Resolution/Recommendation for the dismissal of petitioner
for grave misconduct.
Nowhere in the records of Administrative Case No. 96-04 does it appear that petitioner was charged
with grave misconduct, or that he was held to answer for his alleged defamatory statements in his
letter.

ISSUE: Whether or not the dismissal of the constitutes a denial of his constitutional right to due process

RULING: YES. Two fundamental requirements of due process in administrative cases are that a person
must be duly informed of the charges against him; and that he cannot be convicted of an offense or
crime with which he was not charged.
In the present case, the records of Administrative Case No. 96-04 reveal that petitioner was dismissed
for an act which was not alleged in the administrative charge filed against him.
The basis of the dismissal of petitioner, has no bearing whatsoever on the offenses with which the latter
was charged under the Sworn Complaint nor to the incidents/acts described therein.

OMB v. Magno GR 178923, Nov. 27, 2008 – Val


FACTS: A complaint before the Ombudsman against Magno and other officials of Parañaque City, for
with violation of Republic Act No. 3019, otherwise known as the Anti-Graft and Corrupt Practices Act.
The administrative charges against Magno, et al. were particularly for Misconduct and Oppression.The
Office of the Ombudsman rendered its Decision holding only Magno and Jimenez guilty of Grave
Misconduct and dismissing them from service.
ISSUE: whether petitioner ombudsman violated private respondent magno’s right to due process when
it declared him administratively liable for grave misconduct.
RULING: Yes. (a) a basic requirement of due process is that a person must be duly informed of the
charges against him and that (b) a person can not be convicted of a crime with which he was not
charged.
Administrative proceedings are not exempt from basic and fundamental procedural principles, such as
the right to due process in investigations and hearings.
The essence of due process in administrative proceedings is the opportunity to explain one’s side or
seek a reconsideration of the action or ruling complained of. As found by the Court of Appeals, Magno
was clearly deprived of his right to due process when he was convicted of a much serious offense,
carrying a more severe penalty, without him being properly informed thereof or being provided with the
opportunity to be heard thereon.

Avenido v. CSC 553 SCRA 711- Val

FACTS: Petitioner claims that he was deprived of due process of law when the NTC, thru a Show Cause
Order, charged him with Dishonesty, Falsification of Public Documents and Usurpation of Authority, and
then found him guilty of Conduct Prejudicial to the Best Interest of the Service, an offense which he
avers is so different from the offenses with which he was earlier charged; the offense warranted his
dismissal.
ISSUE: whether petitioner was deprived of due process
RULING: No. This Court has already ruled that the designation of the offense or offenses with which a
person is charged in an administrative case is not controlling and one may be found guilty of another
offense, where the substance of the allegations and evidence presented sufficiently proves one’s guilt:
The charge against the respondent in an administrative case need not be drafted with the
precision of an information in a criminal prosecution. It is sufficient that he is apprised of the
substance of the charge against him; what is controlling is the allegation of the acts complained of,
not the designation of the offense.
In administrative proceedings such as the one at bench, due process simply means the
opportunity to explain one's side or the opportunity to seek a reconsideration of the action or ruling
complained of. In the instant case, petitioner was furnished a copy of the charges against him and he
was able to file an answer and present evidence in his defense. Clearly, therefore, due process was
observed in this case.

Romualdez v. COMELEC 553 SCRA 370 – Val


FACTS: Private respondent filed a Complaint-Affidavit with the COMELEC charging petitioners with
violation of of the Omnibus Election Code,. Petitioners filed a Joint Counter-Affidavit with Motion to
Dismiss. COMELEC filed with the RTC separate Informations against petitioners for violation of Repblic
Act No. 8189.
Petitioners contend that the election offenses for which they are charged by private respondent
are entirely different from those which they stand to be accused of before the RTC by the COMELEC.
Essentially, petitioners are of the view that they were not accorded due process of law. Specifically, their
right to refute or submit documentary evidence against the new charges which COMELEC ordered to be
filed against them.
ISSUE: Whether there was denial of due process
RULING: NO. Significantly, the allegations in the Complaint-Affidavit which was filed with the Law
Department of the COMELEC, support the charge directed by the COMELEC En Banc to be filed against
petitioners with the RTC. Even a mere perusal of the Complaint-Affidavit would readily show that
Section 10 of Republic Act No. 8189 was specifically mentioned therein.
In fact, the nature of the criminal charges in private respondent’s Complaint-Affidavit and that
of the charges contained in the Informations filed with the RTC, pursuant to the COMELEC Resolution En
Banc are the same, such that, petitioners cannot claim that they were not able to refute or submit
documentary evidence against the charges that the COMELEC filed with the RTC. Petitioners were
afforded due process because they were granted the opportunity to refute the allegations in private
respondent’s Complaint-Affidavit.

Multi-Trans Agency v. Oriental 590 SCRA 675- Val

FACTS: The instant case arose from a complaint for sum of money filed by respondent Oriental
Assurance Corporation (Oriental) against petitioner.
The records show that petitioner was declared in default for failure of its former counsel to file
an answer to the complaint after the motion to dismiss he filed was denied by the trial
court. Atty. Austria did not do anything to protect the interests of petitioner. Petitioner failed to
adduce any evidence to rebut the allegations contained in the complaint.
Respondent filed a motion to declare defendant Multi-Trans in default for failure to file its
answer to the complaint which was granted.

ISSUE: Whether there was a denial of due process


RULING: Yes. The mistake or negligence of counsel may result in the rendition of an unfavorable
judgment against the client. We have, however, carved out exceptions to this rule; as where the reckless
or gross negligence of counsel deprives the client of due process of law; or where the application of the
rule will result in outright deprivation of the client’s liberty or property; or where the interests of justice
so requires and relief ought to be accorded to the client who suffered by reason of the lawyer’s gross or
palpable mistake or negligence.
Gross negligence has been defined as the want or absence of or failure to exercise slight care or
diligence, or the entire absence of care. In the case before us, we find the negligence of petitioner’s
former counsel to be so gross that it was deprived of its day in court, thus denying it due process. It
was deprived of due process.

Siochi v. BPI 193872, October 18, 2011 – Val


FACTS: petitioners filed with the RTC a petition for corporate rehabilitation which was granted. But the
RTC confined the initial hearing to the issue of jurisdiction and failed to address other more important
matters relating to the petition and comment. The RTC also failed to refer for evaluation the
rehabilitation plan to the rehabilitation receiver. Thus, the rehabilitation receiver was unable to submit
his recommendations and make modifications or revisions to the rehabilitation plan as necessary.
Moreover, the RTC denied the rehabilitation receiver’s motion to issue an order directing petitioners
and their creditors to attend a meeting. The CA reversed the ruling for the failure of the RTC to abide by
the Rules of Procedure.

ISSUE: whether it is within [the RTC’s] discretion to disregard the procedural formalities,” and “the
lower court has factual basis in its finding that petitioners are capable of rehabilitated

RULING: No. indeed, the Rules are construed liberally. However, this does not mean that courts may
disregard the Rules. The Court held that, “These rules are to be construed liberally to obtain for the
parties a just, expeditious, and inexpensive disposition of the case. The parties may not, however,
invoke such liberality if it will result in the utter disregard of the rules.”

Catacutan v. People 656 SCRA 524- Val

FACTS: Petitioner was charged before the RTC with violation of Section 3(e) of RA 3019 . The RTC
rendered its Decision holding that the act of the petitioner in defying the orders of the CHED and the
CSC to implement the subject promotional appointments demonstrates his palpable and patent
fraudulent and dishonest purpose hence, it held petitioner guilty of the crime charged . The RTC denied
the introduction in evidence the Court of Appeals Decision in CA-G.R. SP No. 51795 entitled "Jose R.
Catacutan, petitioner, versus OMB”

ISSUE: Whether the [petitioner’s] constitutional rights to due process was violated when he was denied
the opportunity to present in evidence

RULING: no. It is well within the Court’s discretion to reject the presentation of evidence which it
judiciously believes irrelevant and impertinent to the proceeding on hand.

"Due process simply demands an opportunity to be heard." "Due process is satisfied when the parties
are afforded a fair and reasonable opportunity to explain their respective sides of the controversy."

Guided by these established jurisprudential pronouncements, petitioner can hardly claim denial of his
fundamental right to due process. Records show that petitioner was able to confront and cross-examine
the witnesses against him, argue his case vigorously, and explain the merits of his defense.
The RTC committed no error in judgment when it did not allow the Accused-appellant to present the
Decision of the Court of Appeals. The findings in administrative cases are not binding upon the court
trying a criminal case, even if the criminal proceedings are based on the same facts and incidents which
gave rise to the administrative matter. The dismissal of an administrative case does not operate to
terminate a criminal proceeding with the same subject matter.

Mortel v. Kerr 685 SCRA 1 (clear violation and errors of counsel) – Val

FACTS: Respondent instituted a complaint for foreclosure of mortgage, against Mortel. The pre-trial was
re-set four times on the fifth setting Mortel and Atty. Mas were not present thus the RTC declared
Mortel as in default.

Mortel’s counsel erred in failing to state the specific material dates to show that the petition for
review was filed within the reglementary period; and that Mortel resorted to the wrong remedy by filing
a petition for review instead of a petition for certiorari Mortel’s counsel committed another error when
he filed his urgent motion for extension of time to file an appeal in the CA, instead of in the SC, resulting
in not stopping the running of the period of appeal and in thereby rendering the Resolution of the CA
final.

The client seeks the reversal of the resolution whereby the Court of Appeals (CA) denied his
petition for review on certiorari from the order of the Regional Trial Court. He pleads that the rules of
procedure should be liberally construed in his case, and that he should not be bound by the negligence
and errors of his previous counsels that deprived him of his property without being afforded his day in
court.

ISSUE: Whether petitioner was deprived of due process

RULING: YES. When the incompetence, ignorance or inexperience of counsel is so great and the resulting
error is so serious that the client, who otherwise has a good cause, is prejudiced and denied his day in
court, the client deserves another chance to present his case. Hence, the litigation may be reopened for
that purpose.

The Court declared that an exception to the rule that a client is bound by the mistakes of his counsel is
when the negligence of the counsel is so gross that the client was deprived of his day in court, thereby
also depriving the client of his property without due process of law.

The negligence and mistakes committed by his several counsels were so gross and palpable that they
denied due process to Mortel and could have cost him his valuable asset. They thereby prevented him
from presenting his side, which was potentially highly unfair and unjust to him on account of his defense
being plausible and seemingly meritorious.

Gravides v. COMELEC 685 SCRA 382 (error of counsel) - Val


FACTS: Borjal and Gravides both ran for the position of Punong Barangay of Barangay U.P. Campus in
Diliman,. Gravides won. Borjal filed an Election Protest alleging irregularities and violation of election
laws. Gravides filed her Answer with Compulsory Counterclaim denying the allegations. She pointed out
that the protest failed to provide a detailed specification of the acts or omissions complained of, which
would show the alleged fraud or irregularities in the protested precincts. Such general and sweeping
allegations violate the provisions of the Rules of Procedure in Election Contests

ISSUE: Whether the rules of procedure in election contests are to applied strictly

RULING: No. Courts have the prerogative to relax rules of even the most mandatory character, mindful
of the duty to reconcile both the need to speedily end litigation and the parties’ right to due process."
While procedural rules are intended for the expeditious disposition of election cases, this should not
impede this Commission from compliance with the established principles of fairness and justice and
adjudication of cases not on technicality but on their substantive merits.

The paramount interest of determining the true will of the electorate thus justified a relaxation of
procedural rules.Indeed, an election protest is imbued with public interest so much so that the need to
dispel uncertainties which becloud the real choice of the people is imperative.

Publicity and T.V. Coverage

Webb v. de Leon 247 SCRA 652 - Prince


(G.R. No. 121234, August 23, 1995)
1. Mass Media exploited the Vizconde Massacre Case.
2. DOJ panel conducted the preliminary investigation in two separate hearings.
3. Webb's counsel filed a petition for injunction, certiorari, prohibition and mandamus compelling
the NBI agents to release the first and original sworn affidavit of Alfaro and the NBI Report on
Webb's location at the time of the commission of the crime.
4. NBI released only a photo-copy of Alfaro's affidavit since it was misplaced.
5. The first sworn affidavit of Alfaro and the NBI report were neither released nor filed before the
DOJ panel.
6. After the second and last hearing, DOJ allowed the parties to file more evidence.
7. DOJ panel did not file an information within 10 days, as provided by the ROC, from the date of
the last hearing for preliminary investigation since it waited for more evidence.
8. DOJ filed their information 27 days after the last hearing for preliminary investigation but the
RTC still appreciated the information.
ISSUE: Whether prejudicial publicity made the preliminary investigation partial.
HELD: NO PREJUDICIAL PUBLICITY and PARTIALITY; PRELIMINARY INVESTIGATION FOLLOWS DUE
PROCESS. Petition DISMISSED.
DOJ panel's Resolution determining probable cause carries no sign of bias since they did not
consider any extra-record evidence except evidence properly adduced by the parties, and extended the
time of investigation upon the motions of the petitioners despite its summary nature.
Publicity gives assurance that the proceedings were conducted fairly to all concerned and
discouraging perjury, the misconduct of participants, or decisions based on secret bias or partiality.
However, probable cause in preliminary investigations is not synonymous with guilt, hence too much or
pervasive publicity during preliminary investigation may deprived one's right to fair trial. Although not
judges, prosecutors conducting preliminary investigation must be fair and impartial because, under Rule
112 of the ROC, preliminary investigations are not inquisitorial but quasi-judicial. And since the right to
have a preliminary investigation is a substantive right, a preliminary investigation should be scrupulously
conducted so that the constitutional right to liberty can be protected from any material damage.
Pervasive and prejudicial publicity under certain circumstances can deprive an accused of his
due process right to fair trial. But to warrant a finding of prejudicial publicity there must be allegation
and proof that the judges have been unduly influenced, not simply that they might be, by the barrage of
publicity (Martelino, et al. vs. Alejandro et al.).

People v. Teechankee 249 SCRA 54- Prince


(G.R. No. Nos. 111206-08, October 6, 1995)
1. Teehankee JR shot John Chapman, Jussi Leino, and Maureen Hultman.
2. He is charged with murder, frustrated murder and homicide separately.
3. Media-men and photographers were present inside and outside of the courtroom.
4. The trial judge allowed taking of photographs but not during the trial proper.
5. When the media-men and the spectators became unruly, he excluded them from the
courtroom.
6. The trial judge voluntarily inhibited himself from further hearing the case.
7. But SC directed the trial judge to proceed with the trial since they found that there was no sign
of partiality.
8. RTC judge found accused guilty of the crimes charged beyond reasonable doubt.
ISSUE: Whether the media-men's detailed coverage of the trial impaired the trial judge's partiality.
HELD: TRIAL JUDGE REMAINED IMPARTIAL THROUGHOUT. JUDGES MAY EXCLUDE THE PUBLIC OR
ADMONISH MEDIAMEN. RTC AFFIRMED with MODIFICATION.
The trial judge remained impartial since he consistently excludes the media-men from the trial
or admonishes them upon the manifestation of the defense counsel who complained that the media-
men became unruly or improper.
The press guards against the miscarriage of justice by subjecting in the police, prosecutors, and
judicial processes to extensive public scrutiny and criticism. Thus, pervasive publicity is not per se
prejudicial to the right of an accused to fair trial. The mere fact that a criminal trial is given a detailed
coverage by the press does not by itself prove that the publicity influenced the trial judge and impaired
his impartiality since a judge's training and learning includes disregarding off-court evidence and on-
camera performances of parties to a litigation. There must be actual prejudice to warrant a finding of
prejudicial publication. There must be allegation and proof that the judges have been unduly influenced,
not simply that they might be, by the barrage of publicity. (Martelino, et al. vs. Alejandro, et al.)
NOTE: In Webb vs. de Leon, pervasive publicity may be sufficient in preliminary investigation to impair
right to an impartial trial while in People vs. Teehankee, pervasive publicity is NOT sufficient in TRIAL
PROPER to impair right to an impartial trial. There must be ACTUAL prejudice.

People v. Sanchez GR 121039-45 Jan. 25, 1999 - Prince


 Mayor Sanchez, and his policemen-bodyguards, are charged of the crime rape with homicide.
 Mass Media exploited the story of the rape-murder of a girl and murder of his boyfriend, all UP
Los Banos victims.
 Mayor Sanchez claimed a framed-up; Insisted that General Alqueza maneuvered the
investigation since his son was the primary suspect.
 UP student-activist believed that Sanchez is linked to PACC Chief Joseph Estrada and Lacson, and
General Alqueza's son is a mere fall-guy. The activist continued support for the Alqueza family.
 Two of the policemen-bodyguards turned state witnesses. They admitted to the abduction of
the victims but denied participation in the rape and homicide.
 RTC judge found the accused guilty of the crime charge guilty beyond reasonable doubt.
 Sanchez claimed that the RTC judge was influenced by publicity.
ISSUE: Whether there is prejudicial publicity influencing the judge.
HELD: NO PREJUDICIAL PUBLICITY. Compare with People vs. Teehankee.

People v. Sanchez GR 121039 Oct. 18, 2001 – Prince


1. Motion for Reconsideration of the Decision dated January 25, 2009 and penned by Justice Martinez.
2. Justice Melo penned this motion for reconsideration since Justice Martinez retired.
ISSUE: Whether there is prejudicial publicity influencing the judge,
HELD: NO PREJUDICIAL PUBLICITY. COMPARE WITH People v. Teehankee 249 SCRA 54.
AFFIRM THE CONVICTION with MODIFICATION of award of damages.
Sanchez failed to show proof of actual prejudice or bias. Compare with Martelino, et al. vs.
Alejandro.

Perez v. Estrada A.M. No. 01-4-03-SC June 29, 2001 – Prince


RE: REQUEST RADIO-TV COVERAGE OF THE TRIAL OF IN THE SANDIGANBAYAN OF THE PLUNDER
CASES AGAINST THE FORMER PRESIDENT JOSEPH E. ESTRADA

1. Several criminal cases against Former President Estrada were filed after the impeachment trial.
2. KBP requested for live media coverage of Estrada's trial before the Sandiganbayan.
ISSUE: Whether live television and radio coverage creates prejudicial publicity in the courtrooms.
HELD: LIVE TV AND RADIO COVERAGE MAY CREATE PREJUDICIAL PUBLICITY ON THE JUDGE, LAWYERS,
AND WITNESSES.
Video footages of court hearings for news purposes shall be restricted and limited to shots of
the courtroom, the judicial officers, the parties and their counsel taken prior to the commencement of
official proceedings. No video shots or photographs shall be permitted during the trial proper.
Live television and radio coverage of judicial proceedings inherently denies the accused of his
right to due process by intimidating the judge, witnesses, lawyers, and other court personnel. Witnesses
might be frightened, play to the camera, or become nervous. They are subject to extraordinary out-of
court influences which might affect their testimony. Also, telecasting not only increases the trial judge's
responsibility to avoid actual prejudice to the defendant, it may as well affect his own performance.
Judges are human beings also and are subject to the same psychological reactions as laymen. For the
defendant, telecasting is a form of mental harassment and subjects him to excessive public exposure
and distracts him from the effective presentation of his defense. (Estes vs. Texas).
Right of the accused to a fair and public trial is preferred over the right to public information and
freedom of the press since the superior right to life and liberty is at stake. Thus, judgment must be
rendered after presentation of credible evidence, witnesses unswayed by undue influence or pressure,
and given by the dispassionate and unprejudiced mind of the judge.
Right to a public trial is right solely belonging to the accused. Although publicity assures that the
proceedings conducted fairly by scrutinizing the judicial processes, its ultimate aim is to protect the
accused from unjust condemnation and secret deprivation of his life or liberty. Thus, the public has
corresponding duty to preserve the right of the accused to a fair and impartial trial.
A public trial is not synonymous with publicized trial. Public trial means that the public may
observe the proceedings with proper decorum and submission to the rules. The public should not
distract the trial participants from their proper functions but they shall be totally free to report what
they have observed during the proceedings.

Perez v. Estrada A.M. No. 01-4-03-SC Sept. 13, 2001- Prince


1. Motion for Reconsideration of the resolution dated June 29, 2001.

HELD: LIVE TV AND RADIO COVERAGE IS STILL PROHIBITED. AUDIO-VISUAL RECORDINGS ARE
ALLOWED. 9-6 DECISION.
The recordings will not be for live or real time broadcast but for documentary purposes:
1. The trial shall be recorded in its entirety, excepting such portions thereof as the Sandiganbayan
may determine should not be held public under Rule 119, §21 of the Rules of Criminal
Procedure;
2. Cameras shall be installed inconspicuously inside the courtroom and the movement of TV crews
shall be regulated consistent with the dignity and solemnity of the proceedings;
3. The audio-visual recordings shall be made for documentary purposes only and shall be made
without comment except such annotations of scenes depicted therein as may be necessary to
explain them;
4. The live broadcast of the recordings before the Sandiganbayan shall have rendered its decision
in all the cases against the former President shall be prohibited under pain of contempt of court
and other sanctions in case of violations of the prohibition;
5. To ensure that the conditions are observed, the audio-visual recording of the proceedings shall
be made under the supervision and control of the Sandiganbayan or its Division concerned and
shall be made pursuant to rules promulgated by it;
6. Simultaneously with the release of the audio-visual recordings for public broadcast, the original
thereof shall be deposited in the National Museum and the Records Management and Archives
Office for preservation and exhibition in accordance with law.

These are the reasons of the above-mentioned conditions:

 The hearings are of historic significance. They are an affirmation of our commitment to the rule
that "the King is under no man, but he is under God and the law."
 The Estrada cases involve matters of vital concern to our people who have a fundamental right
to know how their government is conducted. This right can be enhanced by audio visual
presentation.
 Audio-visual presentation is essential for the education and civic training of the people.
 The accuracy of the transcripts of stenographic notes taken during the trial can be checked by
reference to the tapes.
 By delaying the release of the tapes for broadcast, concerns that those taking part in the
proceedings will be playing to the cameras and will thus be distracted from the proper
performance of their roles -- whether as counsel, witnesses, court personnel, or judges -- will be
allayed. (NOTE: Compare with the Doctrine in Estes vs. Texas).
 Concerns about the regularity and fairness of the trial will be addressed since the tapes will not
be released for public showing until after the decision of the cases by the Sandiganbayan.
 If a documentary record is made of the proceedings, any movie that may later be produced can
be checked for its accuracy against such documentary and any attempt to distort the truth can
thus be averted.
 Prof. Paul Freund of the Harvard Law School also made the similar conclusions.

People v. Roxas- 628 SCRA 378- Prince


G.R. No. 172604, August 17, 2010
1. Roxas and his companions stole a car with the woman-owner still inside.
2. He shot the woman in the face and left. But the woman later survived and woke up with holes in
her face and neck. The bullet failed to hit vital points.
3. The woman testified in trial and consistently identified the accused.
4. During trial, Secretary of Justice and media-men were present.
5. Trial Judge found Roxas guilty beyond reasonable doubt of Kidnapping and Serious Illegal
Detention with Frustrated Murder, and separate crime of Carnapping under RA 6539.
ISSUE: Whether the presence of the Secretary of Justice and media-men affected the trial judge's cold
neutrality.
HELD: NO SUBSTANTIAL PROOF OF PREJUDICIAL PUBLICITY, pervasive publicity is not enough. CA and
RTC AFFIRMED with MODIFICATION. Compare with People vs. Teehankee.
The fact that the trial judge opted to believe the prosecution's evidence rather than that of the defense
is not a sign of bias. Before we could conclude that the accused was prejudiced by the presence of the
media and Secretary of Justice, he must first show substantial proof, not merely cast suspicions. There
must be a showing that adverse publicity indeed influenced the court's decision.

Administrative; Quasi-Judicial Proceedings; Arbitration

1. In General Administrative due process

Ang Tibay v. CIR 69 P 635 – Prince


G.R. No. L-46496, February 27, 1940.
 Massive lay-off of workers by Ang Tibay, violations of the CBA, and other unfair labor practices.
Ang Tibay did not lay-off NWB workers.
 Two Labor Unions, NLU and NWB; NLU sought for vacation of the judgment of the SC and the
remand of the case to the CIR to open a new trial.
 NLU alleged that Ang Tibay discriminate against NLU by laying-off NLU worker's for the purpose
of reducing loses caused by Ang Tibay's breach of contract with the Philippine Army.
ISSUE: Whether the NLU was given due process.
HELD: NO DUE PROCESS BECAUSE THERE WAS NO SUBSTANTIAL EVIDENCE TO SUPPORT CIR'S
CONCLUSION. MOTION FOR NEW TRIAL GRANTED. REMAND TO THE CIR.
Commonwealth Act No. 103 created the Court of Industrial Relations as a special court exercising not
only quasi-judicial judicial functions but executive functions as well. It has the mingling of executive and
judicial functions, which is a departure from the rigid doctrine of the separation of governmental
powers. It is more administrative in nature, thus CIR is not narrowly constrained by technical rules of
procedures (Goseco vs. CIR, G.R. No. 46673, September 13, 1939). Sec 20 of CA No 103 requires it to
"act according to justice and equity and substantial merits of the case, without regard to technicalities
or legal forms and shall not be bound by any technicalities or legal forms and shall not be bound by any
technical rules of legal evidence but may inform its mind in such manner as it may deem just and
equitable." However, It does not mean that it can, in justifiable cases before it, entirely ignore or
disregard the fundamental and essential requirements of due process in trials and investigations of an
administrative character. There are primary rights which must be respected even in proceedings of this
character:
(1) The right to a hearing which includes the right of the party interested or affected to present his
own case and submit evidence in support thereof.
(2) Not only must the party be given an opportunity to present his case and to adduce evidence
tending to establish the rights which he asserts but the tribunal must consider the evidence presented.
(3) While the duty to deliberate does not impose the obligation to decide right, it does imply a
necessity which cannot be disregarded, namely, that of having something to support its decision. A
decision with absolutely nothing to support it is a nullity, a place when directly attached.
(4) Not only must there be some evidence to support a finding or conclusion but the evidence must be
“substantial.” Substantial evidence is more than a mere scintilla It means such relevant evidence as a
reasonable mind might accept as adequate to support a conclusion.
(5) The decision must be rendered on the evidence presented at the hearing, or at least contained in
the record and disclosed to the parties affected.
(6) The Court of Industrial Relations or any of its judges, therefore, must act on its or his own
independent consideration of the law and facts of the controversy, and not simply accept the views of a
subordinate in arriving at a decision.
(7) The Court of Industrial Relations should, in all controversial questions, render its decision in such a
manner that the parties to the proceeding can know the vario issues involved, and the reasons for the
decisions rendered. The performance of this duty is inseparable from the authority conferred upon it.

Dazon v. Yap - 610 SCRA 19- Prince


G.R. No. 157095, January 15, 2010 - CORRECT CITATION
 Dazon made down-payment and several installments to Primetown Property President Yap for
the purchase of a condominium unit. But Primetown failed to complete the unit. Dazon
demanded refund but Primetown did not comply.
 Hence, Dazon filed a criminal complaint under PD No. 957 (1976), "The Subdivision and
Condominium Buyers' Protective Decree.”PD 1344 already expanded the jurisdiction of the
HLURB.
 Prosecutor found probable cause and filed the information before the RTC.
 Primetown appeal the findings of the prosecutor before the DOJ. And DOJ ordered the
prosecutor to withdraw information.
 RTC granted the withdrawal.
ISSUE: Whether the RTC has jurisdiction over the case and not the HLURB.
HELD: RTC'S JURISDICTION UPHELD. HLURB DOES NOT HAVE JURISDICTION. PETITION GRANTED. RTC'S
GRANT OF WITHDRAWAL REVERSED AND SET ASIDE.
NOTE: Due Process requires a Tribunal with the JURISDICTION to hear the case.
The primordial function of the Housing and Land Use Regulatory Board (HLURB) is the regulation
of the real estate trade and business. Section 1 of PD 1344 did not grant it the power to hear, decide,
and impose penalties in criminal cases. Since HLURB does not have such power, it has no jurisdiction
over criminal actions arising from violations of PD 957. Only section 38 of PD 957 grant HLURB the
limited power to impose administrative fines after conviction and/or punishment.
Conviction and punishment of criminals is excluded because of the rule of statutory construction
that the express mention of one thing in the law means the exclusion of others not expressly
mentioned. Indeed, administrative agencies being tribunals of limited jurisdiction can only wield such
powers as are specifically granted to them by their enabling statutes. The legislature must state its
intention in express terms that would leave no doubt, as even such quasi-judicial prerogatives must be
limited, if they are to be valid, only to those incidental to or in connection with the performance of
administrative duties, which do not amount to conferment of jurisdiction over a matter exclusively
vested in the courts.

2. Judges and Disciplinary Process

OCA v. Pascual 259 SCRA 125 – Prince


[A.M. No. MTJ-93-783, July 29, 1996] correct citation.
 Anonymous letter of allegations of extortion and bribery against MTC Judge Pascual. The
anonymous letter did not specify any illegal acts. Allegations were in general terms.
 NBI conducted an entrapment operation. NBI agents made manifest contradiction in their
testimonies. It appears that the marked money was placed inside an envelope which the judge
thought contained pleadings. Upon his knowledge that the envelope contained money, he
drove the NBI's asset, who had the envelope, out of his chamber. NBI agents came in declared
an entrapment. But the envelope was nowhere to be found. An NBI agent called the same
asset to search for the envelope and the asset found the envelope. The asset claimed that the
money came from the NBI while the agents claimed that it belong to the asset.
 The evidence for the Report and Recommendation of the Investigating Judge are: The
Complaint, the Answer, the Memorandum of the respondent, and the transcript of stenographic
notes of the hearing of the bribery case at the Sandiganbayan.
 No open trial.
ISSUE: Whether there was due process.
HELD: NO DUE PROCESS: NO SUBSTANTIAL OR COMPETENT EVIDENCE AND NO OPEN TRIAL.
ADMINISTRATIVE CASE DISMISSED.
The Report and Recommendation of the investigating judge lacks the requirements of due
process for being based on incompetent evidence and issued without giving the respondent judge the
opportunity to meet the witnesses against him face to face. Before any judge could be faulted, it should
be only after due investigation and after presentation of competent evidence, especially since the
charge is penal in character. Even in an administrative cases, it is demanded that, if the respondent
judge should be disciplined for grave misconduct of any graver offense, the evidence against him should
be competent and should be derived from direct knowledge.
NBI acted on the anonymous letter without verifying the source or identity of the sender. Thus,
the letter is incompetent evidence for being hearsay. NBI agents, having acted upon incompetent
evidence, in effect, instigated or induced the judge to commit a crime. And even planted evidence.
Since the evidence were gathered from illegal entrapment, the evidence themselves are inadmissible
and incompetent.

Valenzuela v. Bellosillo 322 SCRA 536 – Prince


Adm. Matter No. MTJ-00-1241, January 20, 2000
1. Ms. Colapo hired Atty. Valenzuela as counsel for a BP 22 case, bouncing checks. Judge Bellosillo
suggested to Colapo to replace Valenzuela.
2. Colapo complained that Valenzuela was charging excessive fees for merely drafting a
manifestation for and approval of bail bond. And Valenzuela did not even appear for his client in
the approval of the bail.
3. Colapo eventually replaced Valenzuela.
4. Valenzuela administratively charge the judge of gross misconduct.
5. The executive judge found the evidence to be insufficient, hence dismissed the charges.
ISSUE:Whether the respondent judge was afforded due process.
HELD: DUE PROCESS PRESENT: EXECUTIVE JUDGE CORRECTLY DISMISSED THE CASE BECAUSE OF
INSUFFICIENCY OF EVIDENCE. ADMINISTRATIVE CASE DISMISSED. Compare with OCA vs. Pascual.
Evidence was insufficient because Colapo, who made the affidavit, was not present to testify for or
against Judge Bellosillo. The affidavit of Colapo is inadmissible as evidence since she was not placed on
the witness stand to give the respondent Judge an opportunity to test the veracity of the allegations.

3. Aspects of the Proceedings

Lumiqued v. Exevea 282 SCRA 125 – Prince


G.R. No. 117565 November 18, 1997
1. A DAR Cashier and a private person filed an administrative complaint of malversation of public
funds against DAR Reg. Director Lumiqued.
2. DOJ formed an investigating committee. The committee sent subpoena to Lumiqued for
submission of counter-affidavit.
3. Hearings were conducted before the committee but Lumiqued was not assisted by counsel. The
committee repeatedly asked Lumiqued if he can confidently represent himself without
assistance of counsel and he answered in the affirmative. But Lumiqued move for resetting of
the subsequent hearings to give him time to look for a lawyer.
4. At last hearing date, Lumiqued nor his lawyer appeared before the committee since he suffered
a stroke. He failed to inform the committee. Hence, committee terminated the investigation
and resolved the case. It finally issued a report finding Lumiqued liable for all charges. It
recommended dismissal from office without prejudice to filing of a criminal case.
5. Eventually, his case reached the Office of the President despite many motions. Pres Ramos
dismissed Lumiqued through an administrative order.
ISSUE: Whether due process in administrative inquiry includes right to be represented by counsel.
HELD: ENGAGING THE SERVICES OF COUNSEL IN ADMINISTRATIVE PROCEEDINGS IS AN OPTION AND
NOT AN ABSOLUTE RIGHT; SUBSEQUENT FILING OF MOTIONS CURED DEFECT IN DUE PROCESS.
DISMISSAL AFFIRMED.
While investigations conducted by an administrative body may at times be akin to a criminal
proceeding, under existing laws, a party in an administrative inquiry may or may not be assisted by
counsel, irrespective of the nature of the charges and of the respondent's capacity to represent himself,
and no duty rests on such a body to furnish the person being investigated with counsel. In an
administrative proceeding, engaging the service of counsel is an OPTION (Section 32, Article VII of RA
No. 2260; and Section 39, par 2, Rule XIV of Book V of EO No. 292). A respondent, when confident, may
choose not to engage the services of counsel. Thus, the right to counsel is not imperative in
administrative investigations because such inquiries are conducted merely to determine whether there
are facts that merit disciplinary measures against erring public officers and employees, with the purpose
of maintaining the dignity of government service.
The right to counsel is not an absolute right and may, thus, be invoked or rejected in a criminal
proceeding and, with more reason, in an administrative inquiry. Thus, the right of an accused in criminal
proceedings to have competent and independent counsel of his own choice does not apply in a
investigation to determine administrative liability because he was not accused of any crime and the
investigation was not part of a criminal prosecution. Moreover, the right to counsel, which cannot be
waived unless the waiver is in writing and in the presence of counsel, is a right afforded a suspect or an
accused during custodial investigation.
Lumiqued's appeal and his subsequent filing of motions for reconsideration cured whatever irregularity
attended the proceedings conducted by the committee. In administrative proceedings, as long as a party
was given the opportunity to defend his interests in due course; he cannot be said to have been denied
due process of law. This constitutional mandate is deemed satisfied if a person is granted an opportunity
to seek reconsideration of the action or ruling complained of.

Fabella v. CA 282 SCRA 256- Prince


G.R. No. 110379, November 28, 1997
• Public School Teachers made a protest or mass actions on the streets demanding better
compensation.
• DECS Secretary formed investigating committees. Committees are made of members from
either the Quezon City Secondary Teachers Federation or the Quezon City Elementary Teachers
Federation designated by the DECS Secretary.
• Administrative hearings were held before the committees. Hence, the teachers petitioned for
certiorari and mandamus with the RTC for purpose of restraining the committees.
• Pending petition before the RTC, the committees finished their investigation and found the
teachers administratively liable and recommended dismissal.
• RTC dismissed the petition for certiorari and mandamus. Thus, the teachers petitioned for
certiorari with the SC. The SC reversed the RTC's dismissal and ordered the RTC proceed with
the case.
• RTC found that there was no due process with the committees' investigation. CA affirmed the
RTC.
ISSUE: Whether there is due process.
HELD: NO DUE PROCESS BECAUSE THE COMMITTEE LACKS COMPETENT JURISDICTION. CA AFFIRMED.
DECS PETITION DENIED.
There is no due process because the committees hearing the case lacks competent jurisdiction, thus all
proceedings made by them were void. Sec 9 of RA 4670 (Magna Carta for Public School Teachers)
expressly provides that the committee to hear public schoolteachers’ administrative cases should be
composed of the school superintendent of the division as chairman, a representative of the local or any
existing provincial or national teachers’ organization and a supervisor of the division. Under Sec 9, it is
an indispensable requirement that the representative must be designated or authorized by the teacher's
organization, not designated by the DECS, to ensure an impartial tribunal. The provision for the
representation of a teacher's organization is intended by law for the protection of the rights of teachers
facing administrative charges.

In administrative proceedings, due process has been recognized to include the following:
1. The right to actual or constructive notice of the institution of proceedings which may affect a
respondent’s legal rights;
2. A real opportunity to be heard personally or with the assistance of counsel, to present witnesses
and evidence in one’s favor, and to defend one’s rights;
3. A tribunal vested with competent jurisdiction and so constituted as to afford a person charged
administratively a reasonable guarantee of honesty as well as impartiality;
4. A finding by said tribunal which is supported by substantial evidence submitted for
consideration during the hearing or contained in the records or made known to the parties
affected.

Joson v.Exec. Sec. 290 SCRA 279- Prince


G.R. No. 131255, May 20, 1998
1. Provincial Council of Nueva Ecija disapproved Gov. Joson's application of PhP 150 M loan.
Hence, Gov Joson and armed men came to the Council meeting and threatened the Provincial
Council.
2. Provincial Council of Nueva Ecija filed an administrative complaint for grave misconduct and
abuse of authority against Gov. Joson.
3. President instructed the DILG Secretary to investigate. DILG Sec directed Joson to submit an
answer to the complaint but he did not comply despite numerous extensions of time. Hence,
DILG declared Joson in default.
4. DILG Sec submitted his report and recommendation to the Office of the President. Then, the
Exec Secretary ordered Joson to be under preventive suspension for 60 days pending
investigation.
5. Joson moved for a Conduct of Formal Investigation with the DILG of pursuant to the provisions
of the LGC and Rule 7 of Administrative Order No. 23. But it was denied. DILG terminated the
investigation and resolved the case. DILG found Joson administratively liable and suspend him
for 6 months without pay.
ISSUE: Whether the petitioner was afforded due process.
HELD: DUE PROCESS PRESENT DESPITE THE ABSENCE OF HEARING SINCE PETITIONER FAILED TO USE
THE OPPORTUNITY TO BE HEARD; DISCIPLINING AUTHORITY GAVE HIM SUCH OPPORTUNITY.
When petitioner failed to submit his position paper as directed and insisted for the conduct o formal
investigation, he was not denied of his right of procedural process because he was given the opportunity
to be heard.
 The essence of due process is to be found in the reasonable opportunity to be heard and to
submit evidence one may have in support of one’s defense. To be heard does not only mean
verbal arguments in court; one may be heard also through pleadings. Where opportunity to be
heard, either through oral arguments or pleadings, is accorded, there is no denial of due
process. Settled is the rule that in administrative proceedings, technical rules of procedure and
evidence are not strictly applied.
 Preventive suspension is authorized under Sec. 63 of the LGC. It may be imposed by the
Disciplining Authority at any time (a) after the Issue are joined; (b) when the evidence of guilt is
strong; and (c) given the gravity of the offense, there is great probability that the respondent,
who continues to hold office, could influence the witnesses or pose a threat to the safety and
integrity of the records and other evidence.
 The DILG has jurisdiction over the case. Jurisdiction over administrative disciplinary actions
against elective local officials is lodged in 2 authorities: the Disciplining Authority and the
Investigating Authority. The Disciplining Authority is the President, whether acting by himself or
through the Exec. Secretary. The Sec of the DILG is the Investigating Authority, who may act by
himself or constitute an Investigating Committee. He is not, however, the exclusive Investigating
Authority for the DILG Sec may designate a Special Investigating Committee.

Busuego v. CA GR 95325 Mar. 11, 1999- Prince


G.R. No. 95326, March 11, 1999(CORRECT CITATION)
1. Central Bank Examiners conducted their regular examination of the accounts and records of PAL
Employees Savings and Loan Association, Inc. ("PESALA").
2. CB Examiners found irregularities and anomalies.
3. CB Examination Section Director invited the petitioners and the PESALA's Board of Director's to
a conference to discussed the findings but the petitioners did not appear.
4. One of the petitioner sent an explanatory letter which was forwarded to the CB Monetary
Board.
5. The PESALA Board likewise submitted their explanatory letter and report.
6. The Monetary Board issued Resolution 805: it Requires the PESALA Board to file civil and/or
criminal complaints against the petitioners.
ISSUE: Whether there is due process.
HELD: NOTICE IS GIVEN; OPPORTUNITY TO BE HEARD WAS GIVEN BUT NOT USED; MONETARY BOARD
HAS COMPETENT JURISDICTION. HENCE, DUE PROCESS. Compare with Ang Tibay vs. CIR; Lumiqued vs.
Exevea; Joson vs. Exec. Sec.
Sec 28 of RA No. 3779 (Savings and Loan Association Act) authorizes the Monetary Board to conduct
regular yearly examinations of the books and records of savings and loan associations, to suspend, a
savings and loan association for violation of law, to decide any controversy over the obligations and
duties of directors and officers, and to take remedial measures, among others.

CSC v. Lucas GR 127838 Jan. 21, 1999- Prince


1. A Female Asst Information officer claimed that the Dept of Agriculture (DA) photographer
(LUCAS) maliciously touched her thigh. Hence, she filed a complaint for simple misconduct
before the Board of Personnel Inquiry (BOPI) of the DA.
2. After due investigation, the BOPI found LUCAS guilty of the offense charged and recommended
a 1 month suspension. The Sec of the DA approved the recommendation.
3. LUCAS appealed the BOPI decision to the CSC. But the CSC found LUCAS guilty of grave
misconduct and imposed the penalty of dismissal.
4. LUCAS was not informed that the charge against him was modified by the CSC.

ISSUE: Whether the formal charge against a respondent in an administrative case should be drafted
with the precision or detail of an information in a criminal prosecution.
HELD: FORMAL CHARGE FAILED TO INFORMED RESPONDENT ON THE NATURE OF THE OFFENSE;
CONVICTION AND PENALTY ARE GREATER THAN THE CHARGE. HENCE, NO DUE PROCESS.
CSC deprived respondent of due process when it failed to informed the respondent of the
nature of the offense and it found him liable for grave misconduct without proof of malice when he was
charged with simple misconduct. Under the CSC Guidelines, grave misconduct, as distinguished from
simple misconduct, have the elements of manifest corruption, clear intent to violate the law or flagrant
disregard of established rule (Landrito vs. CSC).
The right to substantive and procedural due process is applicable in administrative proceedings:
1. A person must be duly informed of the charges against him.
2. A person cannot be convicted of a crime with which he was not charged.

NPC v. Bernabe 332 SCRA 74- Prince


G.R. No. 129914, May 12, 2000.
1. Bernabe, a policeman, is accused of en-cashing treasury warrants of dead Constabulary officers
and appropriating the money himself.
2. PNP Director General ordered the Criminal Investigation Service Command (CICS) to investigate
Bernabe.
3. Bernabe was informed of the accusation and ordered to file an affidavit, and he submitted his
affidavit.
4. CICS submitted a memorandum to the Director General. The memorandum discussed
competent evidence supporting the accusation. Hence, Director General suspended Bernabe
and ordered him to file an answer after receiving the notice of complaint.
5. Bernabe receive the notice of complaint for grave misconduct and conduct unbecoming of a
police officer. After filing his answer, the PNP Summary Dismissal Board recommended dismissal
and PNP Inspector General concurred with the dismissal.
6. He appealed the Insp General's decision to the NAPOLCOM National Appellate Board which
affirmed the dismissal.

ISSUE: Whether there is due process.


HELD: ADMINISTRATIVE DUE PROCESS DOES NOT REQUIRE TRIAL PROPER AS LONG THERE IS NOTICE
AND OPPORTUNITY TO BE HEARD; SUBSEQUENT FILING OF MOTIONS CURES DEFECTS IN DUE
PROCESS. DISMISSAL of police officer AFFIRMED.

Due process as a constitutional precept does not always and in all situations require a trial-type
proceeding. Compare with Joson vs. Exec Sec.; Lumiqued vs. Exevea.

Summary Dismissal Board v. Torcita, GR No 130442, April 6, 2000, 330 SCRA 153-Prince
1. A administrative complaint was filed against Policeman Torcita for conduct unbecoming of an
officer. No complaint was filed for breach of internal discipline.
2. However, the Summary Dismissal Board's investigation found respondent to have committed a
breach of internal discipline by taking alcoholic drinks while on duty.
3. Other charges lacks sufficient evidence except the intoxication.
4. Hence, the Board suspended respondent from service for 20 days, for “simple irregularity in the
performance of service”.
ISSUE: Whether there was due process.
HELD: NO WRITTEN OR FORMAL CHARGE OF THE OFFENSE OF WHICH HE WAS FOUND GUILTY. THUS,
NO DUE PROCESS.
Respondent was entitled to know that he was being charged with being drunk while in the performance
of duty. Although he was given the opportunity to be heard on the multiple and broad charges filed
against him, the absence of specification of the offense for which he was eventually found guilty is not a
proper observance of due process.

Velayo v. Comelec 327 SCRA 713 – Rhino

FACTS: Private Respondent (PR) Natividad filed several petitions for the exclusion of some election
returns before the Mun. Board of Canvassers (MBC) and later the disqualification of the Chairman and
Vice-Chairman of the MBC before the Comelec Division. But none of these named his rival Petitioner
Velayo as respondent therein who was later proclaimed as the winning mayor of Gapan, Nueva Ecija.
The Comelec en banc issued a resolution annulling the proclamation of Velayo. Velayo claimed that he
was denied due process for lack of notice other than the questioned resolution.

ISSUE: Whether there is denial of due process.

RULING: YES! Petitioner is a real party in interest as the proclaimed Mayor. His non-inclusion as
respondent and his lack of notice of the proceedings in the COMELEC which resulted in the cancellation
of his proclamation constitute clear denial of due process.

Summary proceedings in pre-proclamation cases, although may not require a trial type hearing, does
not mean ex parte proceedings. Summary simply means with dispatch, with the least possible delay. The
adverse party must be notified so that he can be apprised of the nature and purpose of the proceeding.

The resolution was based on new and additional evidence presented by PR which were not presented
before the MBC. Petitioner was not furnished these evidence and given the chance to refute them.

Ramoran v. Jardine 326 SCRA 208 G.R. No. 131943. February 22, 2000– Rhino

FACTS: Petitioner was dismissed from Respondent Jardine CMG Life Insurance Company, Inc. for
falsification of her overtime authorization slips. Respondent’s labor union staged a strike with
petitioner’s dismissal as one of the causes. Then respondent filed a complaint in the Provincial
Prosecution Office of Rizal for falsification. Later, respondent and the union entered into a Compromise
Agreement, among others, that legality of the dismissal shall be decided by a panel of voluntary
arbitrators. Such panel sustained the dismissal. Meanwhile two informations for falsification were filed
in the MTC, one of which petitioner was convicted prompting her to appeal to the RTC which overruled
the conviction. Petitioner went to the CA praying the panel’s ruling to be overturned following her
acquittal in the two criminal cases. CA denied. Petitioner claims that she was deprived of her right to
due process of law because of the composition and actuations of the Panel of Voluntary Arbitrators.

ISSUE: Whether there is denial of due process.

RULING: NO! The essence of due process is to be found in the reasonable opportunity to be heard and
submit any evidence one may have in support of one’s defense. Due process does not necessarily
require conducting an actual hearing but simply giving the party concerned due notice and affording an
opportunity or right to be heard.
In the instant case, petitioner was apprised of the charges against her; she consented to resorting to
voluntary arbitration and participated in the selection of arbitrators. Petitioner submitted herself to the
jurisdiction of the Panel of Voluntary Arbitrators, by presenting her evidence and sought affirmative
relief therein.

It is an undesirable practice for a party, after encountering an adverse judgment, to complain and
question the proceedings where she had submitted her own evidence and claimed affirmative relief.

Immam v. Comelec 322 SCRA 866 G.R. No. 134167. January 20, 2000 – Rhino

FACTS: Private respondent (PR) filed with the COMELEC, a "Petition to Count the Ballots and for Holding
of Special Elections alleging that the election inspectors of fourteen (14) precincts left the polling places
due to terrorism and threats thereof. His rival, petitioner, was proclaimed mayor of Matanog,
Maguindanao prompting him to question it with the COMELEC which later suspended the proclamation.
Petitioner now alleges that the questioned order was issued without any motion for its issuance and
without notice and hearing. Thus, he claims, his right to due process was violated.

ISSUE: Whether there is denial of due process.

RULING: NO! The essence of due process is the opportunity to be heard. The right to be heard does not
only refer to the right to present verbal arguments in court. A party can be heard through the pleadings
he submits. In this case, petitioner was heard through the memorandum (in Jardiel v. Commission on
Elections: affidavits) he submitted.

Villarosa v. Comelec GR 133927 Nov. 29, 1999- Rhino

FACTS: Petitioner was a candidate for Representative of the lone district of Occidental Mindoro. Private
respondent (PR) Restor filed a letter-petition with the COMELEC questioning the nickname JVT declared
by petitioner in her COC as she is not so publicly known as such and that it pertains to her husband
(former congressman) and prayed for the nullification of all votes made to JVT. Commission en banc,
issued a Resolution granting private respondent, a copy of which petitioner received on the same date
of the resolution and on the day of the election at the time when voting has ceased and canvassing of
votes already started. Petitioner filed an "Urgent Manifestation and Motion" with the Commission on
the next day, which the Commission promptly denied the following day. Petitioner now alleges denial of
due process.

ISSUE: Whether there is denial of due process.

RULING: YES! Petitioner came to know about the letter-petition only upon receipt of the Comelec
resolution which makes it clear that the Commission passed upon the letter-petition without affording
petitioner the opportunity to explain her side and to counter the allegations therein. Due process
dictates that before any decisions can be validly rendered in a case, the twin requirements of notice and
hearing must be observed.

In Salonga vs. Court of Appeals, 269 SCRA 534: Deprivation of due process cannot be successfully
invoked where a party was given the chance to be heard on his motion for reconsideration. This is not
applicable here because the "Urgent Manifestation and Motion" does not purport to embody
petitioner's grounds and arguments for reconsideration as it states that “petitioner reserves all rights
and waives none, including filing a supplemental motion for reconsideration pending retaining
additional counsel.”

Go v. Comelec GR 147741 May 10, 2001- Rhino

FACTS: Petitioner is the incumbent representative of the Fifth District, province of Leyte. As her term is
about to expire, she filed her COC for Mayor of Baybay, Leyte to the Municipal Election Supervisor
(MES). The next day, she filed a COC for governor of the province of Leyte to the Provincial Election
Supervisor (PES) and withdrew her COC for mayor. The PES refused to accept and advised that the
withdrawal shall be filed with the MES. But since it was almost deadline to withdraw the same, she sent
her affidavit of withdrawal to her father who forwarded the same to Comelec. Private Respondents
Montejo and Antoni petitioned before the PES to disqualify the petitioner on the basis of filing two
COCs. The PES inhibited for having previously refused the withdrawal and referred the cases to the
Comelec, Manila, Law Department. Then Comelec First Division required the PES to forward the case to
the Regional Election Director where hearing ensued. On the other hand, the Law Department,
COMELEC made a study of the cases without affording petitioner an opportunity to be heard or to
submit responsive pleadings and they submitted a report and recommendation to the COMELEC en banc
which approved the same.

ISSUE: Whether there is denial of due process.

RULING: YES! The Law Department, COMELEC conducted an ex-parte study of the cases. Petitioner
was not required to submit a comment or opposition to the petitions for cancellation of her certificates
of candidacy and/or for disqualification. It did not set the cases for hearing. It was not even aware of
the proceedings before the Regional Election Director. The COMELEC Rules of Procedure requires that
notice be given to the respondent. The COMELEC en banc in approving the report and recommendation
of the Law Department, deprived the petitioner of procedural due process of law. The COMELEC, acting
as a quasi-judicial tribunal, cannot ignore the requirements of procedural due process in resolving cases
before it.

Mollaneda v. Umacob GR 140128 June 6, 2001 – Rhino

FACTS: Respondent Umacob, a public school, teacher went to the Admin Office to follow up her request
for transfer to a different district. But it was Petitioner Mollaneda, school Division Superintendent, who
was at the office at that time. After entertaining her request hugged her, embraced her, kissed her nose
and lip in a torrid manner, and mashed her breast. Petitioner did these acts for several times then
warned Respondent not to tell the incident to anybody. Respondent reported the incident to the police
station and filed a complaint for acts of lasciviousness before the Municipal Trial Court. She also filed an
administrative complaint as well with the CSC and furnished the DECS a copy of her affidavit-complaint.
A DECS investigating committee was formed, which later recommended to the DECS Regional Director
"the dropping of the case" for lack of merit. CSC designated Atty. Anacleto Buena to hear and receive
the evidence in the case; the hearing was attended by both parties and their counsel. CSC found
Petitioner guilty, which was affirmed by the Court of Appeals. Petitioner alleges that he was denied due
process on the basis that the Commissioners did not personally hear the case but only via hearing
officer.

ISSUE: Whether there is denial of due process.


RULING: NO! During the hearing, Petitioner actively participated in all proceedings and cross-examined
all witnesses. Neither does due process of law nor the requirements of fair hearing require that the
actual taking of testimony be before the same officer who will make the decision in the case. Due
process in administrative proceedings merely requires that the tribunal or body or any of its judges must
act on its or his own independent consideration of the law and facts of the controversy, and not simply
accept the views of a subordinate.

Cruz v. CSC GR 144469 Nov 27, 2001- Rhino

FACTS: CSC received a letter from a private individual claiming that, during the examinations for non-
professional in the career civil service Petitioner Paitim, the Municipal Treasurer of Norzagaray, Bulacan,
falsely pretending to be the examinee, Petitioner Cruz, a co-employee in the said office, took the
examinations for the latter. Investigation ensued and an administrative case was filed before the CSC.
They were ordered to file their Answer where they should state whether they elect to have a formal
investigation or waive their right to said investigations should their Answer be found not satisfactory.
Petitioners elected a formal investigation. The petitioners subsequently filed a Motion to Dismiss
averring that if the investigation will continue, they will be deprived of their right to due process
because the Civil Service Commission was the complainant, the Prosecutor and the Judge, all at the
same time.

ISSUE: Whether there is denial of due process.

RULING: NO! The CSC is mandated to hear and decide administrative case instituted by it or instituted
before it directly or on appeal including actions of its officers and the agencies attached to it pursuant
Administrative Code of 1987. The fact that the complaint was filed by the CSC itself does not mean that
it could not be an impartial judge. As an administrative body, its decision was based on substantial
findings. Factual findings of administrative bodies, being considered experts in their field, are binding on
the Supreme Court. The records clearly disclose that the petitioners were duly investigated by the CSC.

Condilla v. De Venecia GR 150605 Dec 10, 2002 – Rhino

FACTS: A petition for disqualification was filed against petitioner for indirectly soliciting votes (by using
the equipment and vehicles owned by the City Government of Ormoc to extract, haul and distribute
gravel and sand to the residents of Kananga and Matag-ob, Leyte, for the purpose of inducing,
influencing or corrupting them to vote for him) which was heard only after the election. Before the
counting could be finished, the sitting representative of 4th District of Leyte respondent Locsin filed a
Most Urgent Motion before the Commission to set the hearing of the disqualification case and prayed
for the suspension of the proclamation of the petitioner “so as not to render the present disqualification
case moot and academic.” Petitioner did not receive a copy of such motion. By virtue of the COMELEC
Second Division Ex-Parte Order, Petitioner garnered the highest votes as representative of 4th District of
Leyte but his proclamation was suspended. Petitioner filed a Motion to Lift the Order of Suspension of
his proclamation but was not acted upon. The COMELEC Second Division found the petitioner guilty of
indirect solicitation of votes and ordered his disqualification; hence respondent Locsin was proclaimed
the winner and took her oath. This proclamation was later annulled by Comelec en banc and
proclaimed petitioner. But respondent refuse to vacate her position.
ISSUE: Whether there is denial of due process that will invalidate the proclamation of respondent
Lacson.

RULING: YES!
(a) Petitioner was not notified of the petition for his disqualification through the service of
summons nor of the Motions to suspend his proclamation. Respondent’s Most Urgent Motion
does not fall under the exceptions to notice and service of motions.
(b) The COMELEC Second Division did not give ample opportunity to the petitioner to adduce
evidence in support of his defense in the petition for his disqualification. The hearing for
Motion to Lift the Order of Suspension cannot be substituted for the hearing in the
disqualification case. Although intrinsically linked, it is not to be supposed that the evidence of
the parties in the main disqualification case are the same as those in the Motion to Lift the
Order of Suspension.
(c) The Resolution of the COMELEC Second Division disqualifying the petitioner is not based on
substantial evidence. The Resolution of the COMELEC Second Division cannot be considered to
be based on substantial evidence. It relied merely on affidavits of witnesses attached to the
petition for disqualification.
(d) Exclusion of the votes in favor of the petitioner and the proclamation of respondent Locsin
was done with undue haste. The COMELEC Second Division ordered the exclusion of the votes
cast in favor of the petitioner, and the proclamation of the respondent Locsin, without affording
the petitioner the opportunity to challenge the same.
(e) Even assuming, however, that the petitioner was validly disqualified, it is still improper for the
COMELEC Second Division to order the immediate exclusion of votes cast for the petitioner as
stray, and on this basis, proclaim the respondent as having garnered the next highest number
of votes. Section 6 of R.A. No. 6646 and section 72 of the Omnibus Election Code require a final
judgment before the election for the votes of a disqualified candidate to be considered
“stray.” Petitioner timely filed a Motion for Reconsideration.
(f) In the recent case of Trinidad v. COMELEC: this Court ruled that the effect of a judgment
disqualifying a candidate, after winning the election, based on personal circumstances or section
68 of the Omnibus Election Code (election offenses) is the same: the second placer could not
take the place of the disqualified winner.

Associated Communication v. Dumlao GR 136762 Nov. 21, 2002 – Rhino

FACTS: Petitioner Associated Communications and Wireless Services, Ltd. (ACWS) operated several
radio and television stations nationwide by virtue of a legislative franchise. Then PD 576-A terminated
all franchises to operate radio or television broadcasting systems. ACWS continued to operate its radio
and television stations through permits issued by the Board of Communications and the Secretary of
Public Works and Communications pursuant to PD 576-A which became the NTC later on. Then the NTC
forwarded ACWS a LETTER informing it to submit, within thirty (30) days from expiration of its
Temporary Permit, a new congressional franchise pursuant to Memorandum of Understanding between
the House Committee on Legislative Franchises and the KBP of which ACWS is a member and failure to
do so may cause a denial of their application for renewal of their Temporary Permits. Despite this
directive, respondents failed to submit a new congressional franchise; hence, the NTC issued an ORDER
to cease and desist from operating Channel 25. Petitioner filed its Answer. While the case was still
pending before the NTC, ACWS went to the CA which dismissed the case and its motion to reconsider
the same. Hence, this petition where ACWS argues that NTC failed to observe due process in the
issuance of the Order because ACWS did not receive the letter mentioned in the Order.
ISSUE: Whether there is denial of due process.

RULING: NO! In order to fall within the protection of this provision (Section 1 of the Bill of Rights ), two
conditions must concur, namely, that there is a deprivation and that such deprivation is done without
proper observance of due process. When one speaks of due process of law, a distinction must be made
between matters of procedure and matters of substance. In essence, procedural due process “refers to
the method or manner by which the law is enforced,” while substantive due process “requires that the
law itself, not merely the procedures by which the law would be enforced, is fair, reasonable, and just.”
ACWS argues that NTC failed to observe due process in the issuance of the Order
because ACWS did not receive the letter mentioned in the Order. Obviously, ACWS is referring to the
procedural aspect of the due process clause.
Notice and hearing are fundamental requirements of procedural due process when an
administrative body exercises its quasi-judicial functions. Both were complied with in this case.
Moreover, this constitutional mandate is deemed satisfied if a person is granted an opportunity to
seek reconsideration of the action or ruling complained of.
The NTC afforded ACWS an opportunity to be heard by requiring it to submit an answer and by
conducting hearings on the matter. All told, ACWS had an opportunity to seek a reconsideration of the
Order before the NTC.

Velllarosa v. Pomperada, Admin Case (AC) No. 5310, Jan. 28, 2003 – Rhino
FACTS: Complainants Lina Villarosa and her son Jose Villarosa filed before this Court a complaint for
disbarment against respondent Atty. Osmondo Pomperada, charging him with deceit and gross
misconduct by falsifying a deed of sale to have been notarized by him when in fact it pertains to another
document based on the Certification from the Records Management and Archives Office. The Court,
required respondent lawyer to comment on the complaint which the latter complied. The Court
indorsed the matter to the IBP for investigation, report and recommendation. The resolution of the IBP
Board of adopting the recommendation of Investigating Commissioner San Juan was submitted to the
Court. Respondent filed a MR contending that he was utterly deprived of his fundamental right of due
process when San Juan submitted her report, recommending the penalty of suspension from the
practice of law and perpetual disqualification from being appointed Notary Public, without any formal
investigation having first been conducted by the IBP.

ISSUE: Whether there is denial of due process.

RULING: YES! If, at the outset, the Court finds a complaint to be clearly wanting in merit, it outrightly
dismisses the case. If, however, the Court deems it necessary that further inquiry should be made, such
as when the matter could not be resolved by merely evaluating the pleadings submitted, a referral is
made to the IBP for a formal investigation of the case during which the parties are accorded an
opportunity to be heard. An ex parte investigation may only be conducted when respondent fails to
appear despite reasonable notice. The rules are enunciated in Rule 139-B of the Rules of Court. (N.B.:
It’s Villarosa, not Vellarosa.)

Alauya v. Comelec, GR 152151-52, Jan. 22, 2003- Rhino


FACTS: Regular elections for regional governor, regional vice-governor, and members of the Regional
Legislative Assembly for the ARMM were held. There was a failure of election in the municipality of
Lumbatan necessitating the holding of special elections. Considering the number of registered voters in
Lumbatan, the special elections would not affect Menor’s election as regional assemblyman. Private
respondent Tamano filed two (2) petitions to declare a failure of election in the 5 municipalities.
Tamano also claimed that in almost all of the precincts in these 5 municipalities, either petitioner
Alauya, Jr. (“Alauya” for brevity) or private respondent Sarangani (“Sarangani” for brevity) obtained
100% of the votes such that their votes equalled the total number of registered voters. Tamano prayed
for the suspension of proclamation of the winning candidates. The COMELEC en banc issued the order
directing “the Provincial Board of Canvassers NOT to proclaim the alleged or supposed winners and to
continue and complete the canvass of election results in the Second District of Lanao del Sur.”
Petitioner went to the Comelec then to the SC questioning the Order arguing that COMELEC could only
suspend proclamation “after due notice and hearing.”

ISSUE: Whether there is denial of due process.

RULING: NO! A party cannot successfully invoke deprivation of due process if he was accorded the
opportunity of a hearing, through either oral arguments or pleadings. There is no denial of due process
when a party is given an opportunity through his pleadings. Contrary to Alauya’s claim, it appears that
notices dated December 21, 2001 were given to all concerned parties requiring them to file their answer
to the petition and setting the case for hearing on January 4, 2002. In an Order dated January 4, 2002,
the COMELEC noted that Alauya did not appear during the hearing. Subsequently, Alauya filed his
Answer with Motion to Dismiss to the petitions. Verily, Alauya was given an opportunity to be heard
during the hearing held on January 4, 2002 which he failed to attend and was in fact heard through the
pleadings he filed with the COMELEC.

Spouses Casimiro v. CA G.R. 135911, Feb. 11, 2003- Rhino

FACTS: During a relocation survey conducted by Geodetic Engineer Emilio Paz at the instance of
respondents, it was discovered that the Casimiro Village Subdivision, owned by petitioners, encroached
by 3,110 square meters into respondents’ land. Respondents notified petitioners and demanded that
they desist from making further development in the area. Unheeded, respondents sued in the CFA
which ruled against petitioners and was overturned by the RTC. In the CA, it ordered that a relocation
survey be conducted by a team of surveyors composed of a surveyor designated by the respondents,
petitioners, and a third member-surveyor chosen by the said two surveyors. The CA ruled in
respondents’ favor. Petitioners raise the irregularity of the surveys made as their representative was
not informed of the schedule; hence, was absent when the survey was conducted. (N.B.: Should be
G.R. No. 136911, not G.R. No. 135911.)

ISSUE: Whether there is denial of due process.

RULING: YES! It appears that the actual field work was performed by engineers from LRA, without the
representatives of petitioners and respondents being present. There was no clear showing that notices
of the field work were sent to petitioners and respondents. Worse, the actual field work was
undertaken by only four engineers, all of whom were designated from the LRA. This is in violation of the
agreement of the parties that the actual field work should be done by five technical personnel, three of
whom shall come from the Land Registration Authority and the remaining two shall be Engrs. Lopez and
Bernardo or their respective representatives.

Sy v. CA, GR 147572, Feb. 27, 2003- Rhino

FACTS: A complaint for illegal dismissal with money claims was filed by petitioner Rosario against
respondent Victory Ricemill. The regional labor arbiter found that there were valid causes, i.e., willful
disobedience to the lawful orders of the employer and commission of a crime or offense against the
employer’s duly authorized representative (attempting to kill a co-employee who replaced petitioner as
driver), for the termination of petitioner’s employment. The NLRC, initially found that petitioner was
denied due process during the proceedings with the regional labor arbiter as he (petitioner) was not
given the opportunity to present his additional rebuttal evidence. After the remand, the NLRC affirmed
the same outcome of the arbiter’s decision which was affirmed by the CA. Petitioner now claims that he
was not afforded due process of law because prior to the termination letter, he was not furnished a
written notice detailing the particular acts and/or omissions which he allegedly committed to warrant
his dismissal. Petitioner thus prays that respondent be directed to reinstate him and pay his money
claims.

ISSUE: Whether there is denial of due process.

RULING: NO! The SC is not a trier of facts. The unanimous finding of the regional labor arbiter, the
NLRC and the CA that petitioner is guilty of willful disobedience is based on substantial evidence on
record. However, the records reveal that indeed, respondent’s manner of dismissing petitioner fell short
of the two-notice requirement. While it furnished petitioner the written notice informing him of his
dismissal, respondent failed to furnish petitioner the written notice apprising him of the charge or
charges against him.

However, the prevailing rule is that when the dismissal is effected for a just and valid cause, as in this
case, the failure to observe procedural requirements does not invalidate nor nullify the dismissal of an
employee. Not all notice requirements are requirements of due process. Some are simply part of a
procedure to be followed before a right granted to a party can be exercised. Others are simply an
application of the Justinian precept, embodied in the Civil Code, to act with justice, give everyone his
due, and observe honesty and good faith toward one’s fellowmen, violation of which, gives rise to
damages.

Such is the notice requirement in Arts. 282-283. It is similar to the failure to observe the provisions of
Art. 1592, in relation to Art. 1191, of the Civil Code in rescinding a contract for the sale of immovable
property. Under these provisions, while the power of a party to rescind a contract is implied in
reciprocal obligations, nonetheless, in cases involving the sale of immovable property, the vendor
cannot exercise this power even though the vendee defaults in the payment of the price, except by
bringing an action in court or giving notice of rescission by means of a notarial demand.
The procedural infirmity is then remedied by ordering the payment to the employee his full backwages
from the time of his dismissal until the court finally rules that the dismissal has been for a valid cause.
(N.B.: The citation pertains to this case TEODORICO ROSARIO, petitioner, vs. VICTORY
RICEMILL, respondent to which the foregoing digest was culled.)

Namil v. Comelecc, GR 15040, Oct. 28, 2003- Rhino (should be G.R. No. 150540):

FACTS: The Municipal Board of Canvassers of Palimbang proclaimed the petitioners as the Sangguniang
Bayan winning candidates. They took their oath, and assumed their offices. Later the Municipal Board
of Canvassers of Palimbang proclaimed the respondents as winners. Given the double proclamation, the
concerned election officers submitted their memorandum which points to the validity of the latter
proclamation. Commissioner Sadain submitted his Recommendation to the COMELEC based on said
memoranda and the Comelec thus based its assailed order based on such recommendation. Petitioners
now contend that the order is null and void since it was issued without according them due notice and
hearing.

ISSUE: Whether there is denial of due process.

RULING: YES! Although public office is not property under Section 1 of the Bill of Rights of the
Constitution, and one cannot acquire a vested right to public office, it is, nevertheless, a protected
right. Due process in the proceedings before the public respondent exercising its quasi-judicial functions,
requires due notice and hearing, among others. Comelec is without power to partially or totally annul a
proclamation or suspend the effects of a proclamation without notice and hearing. The proclamation of
a winning candidate cannot be annulled if he has not been notified of any motion to set aside his
proclamation. Due process requires that a party be given an opportunity to adduce his evidence to
support his side of the case and that the evidence should be considered in the adjudication of the case.

The Court, in Utto vs. Commission on Elections, held that the twin-requirement of notice and hearing in
an annulment of proclamation is not applicable because of the illegality of petitioner’s proclamation. In
the Utto case, a notice of appeal was filed questioning the ruling of the board of canvassers but, the
latter proceeded in proclaiming Utto as the winning candidate. This made the proclamation illegal. In
the present case, nobody questioned the petitioners’ proclamation.

Bautista v. Comelec, GR 154796-97, Oct. 23, 2003 - Rhino

FACTS: Petitioner Bautista filed his certificate of candidacy for Punong Barangay in Lumbangan but it
was refused because he was not a registered voter in Lumbangan. The RTC, via mandamus, ordered the
acceptance of petitioner’s COC. The COMELEC Law Department recommended the cancellation of
Bautista’s COC but the COMELEC en banc failed to act on it before the barangay elections. Petitioner
garnered the highest votes while private respondent Alcoreza came in second and took oath. The
Comelec en banc issued two resolutions: (1) cancelling petitioner’s candidacy and (2) to cease and desist
from taking his oath of office or from assuming the position to which he was elected, unless a
temporary restraining order was issued by the Supreme Court. Bautista wrote a letter to COMELEC
requesting the latter for reconsideration of the COMELEC Resolutions and while pending, he went to the
SC raising denial of due process in the issuance of the orders.

ISSUE: Whether there is denial of due process.


RULING: YES! The essence of due process is simply the opportunity to be heard, or as applied to
administrative proceedings, an opportunity to explain one’s side or an opportunity to seek a
reconsideration of the action or ruling complained of. A formal or trial-type hearing is not at all times
and in all instances essential. What is frowned upon is absolute lack of notice and hearing, as in this
case. There is due process when a party is able to present evidence in the form of pleadings.

The fact that Bautista was able to file a letter with the COMELEC en banc requesting for reconsideration
of the Resolutions is beside the point. To reiterate, the 1993 COMELEC Rules of Procedure prohibit a
motion for reconsideration of a COMELEC en banc resolution except in cases involving election offenses.

Respondents likewise submit that there was no need for presentation and evaluation of evidence since
the issue of whether Bautista was a registered voter is easily resolved by looking at the COMELEC
registration records. This reasoning fails to consider the instances where a voter may be excluded
through inadvertence or registered with an erroneous or misspelled name.

Office of OMB v. Coronel 493 SCRA 392 G.R. No. 164460 June 27, 2006 – Rhino

FACTS: Respondent Coronel is a Senior Accounting Processor of the Linamon Water District. A graft
case for dishonesty was filed against her before the OMB for falsifying the cash invoice (from P213 as
reflected on the photocopy to P1,213) she reimbursed for the luncheon meeting of which she was found
guilty. Her MR was granted but Ombudsman Desierto DISAPPROVED the said order with a marginal
note, ‘the original decision stands.’ Respondent went to the SC for nullification of petitioner’s
Disapproval Order for having deprived her of due process.

ISSUE: Whether there is denial of due process.

RULING: NO! The ombudsman’s disapproval orders written as marginal notations (“The original
decision stands.”) were valid, even if they did not specifically spell out their factual and evidentiary basis.
Indeed, this doctrine was first established in Cruz v. People 233 SCRA 439, June 27, 1994 and has
consistently been followed in recent cases.

The notation means that since it is merely a review of the conclusions arrived at by the investigating
prosecutor, another or a new preliminary investigation is no longer necessary. It actually adopted the
factual and legal conclusions of the original Decision.

In administrative proceedings, the essence of due process lies simply in the opportunity to explain one’s
side or to seek reconsideration of the action or ruling complained of. What is proscribed is the absolute
lack of notice or hearing. Significantly, respondent’s intelligible pleadings before the CA and this Court
indicate that she knew the bases for the ombudsman’s Decision.

Erece v. Macalingay 552 SCRA 320 G.R. No. 166809 April 22, 2008- Rhino

FACTS: Petitioner Atty. Erece is the Regional Director of CHR. Respondent employees of the CHR filed
an Affidavit-Complaint before the CSC against petitioner alleging that he denied them the use of the
office vehicle assigned to petitioner, that petitioner still claimed transportation allowance even if he was
using the said vehicle. At his instance, in order to prevent delay in the disposition of the case, he was
allowed to present evidence first to support the allegations in his Counter-Affidavit. After he rested his
case, respondents did not present their evidence, but moved to submit their position paper and formal
offer of evidence, which motion was granted by the CSC over his (petitioner’s) objection. The CSC
charged petitioner with Dishonesty and Grave Misconduct. Petitioner filed a petition for review of the
CSC Resolution with the CA which upheld the CSC decision. Petitioner now alleges that he was denied
due process as respondents failed to identify and testify on their Affidavit-Complaint and that petitioner
was denied of his right to cross-examine respondents on their Affidavit-Complaint.

ISSUE: Whether there is denial of due process.

RULING: NO! Petitioner was given the opportunity to be heard and present his evidence. In
administrative proceedings, the essence of due process is simply the opportunity to explain one’s side.
Due process of law in administrative cases is not identical with “judicial process” for a trial in court is not
always essential to due process. While a day in court is a matter of right in judicial proceedings, it is
otherwise in administrative proceedings since they rest upon different principles. The due process
clause guarantees no particular form of procedure and its requirements are not technical. The right to
cross-examine is not an indispensable aspect of due process. Nor is an actual hearing always essential.

Marcelo v. Bungubung 552 SCRA 589 G.R. No. 175201 April 23, 2008
- Rhino

FACTS: Respondent Bungubung is the Manager of the Port District Office of Manila, Philippine Ports
Authority (PPA and the Chairman of the Ports District Security Bids and Awards Committee of the PPA.
Doromol, the President of a security agency that participated in the bidding, filed a Complaint-Affidavit
(requesting bribe for the award from money to pajero) against respondent before PPA Resident
Ombudsman. After investigation, complaints were filed against Bungubung before the Ombudsman.
After the parties submitted the required pleadings, respondent manifested therein that he was
submitting the case for resolution. The Ombudsman ordered the submission of the case for resolution.
The parties were then required to submit their respective Memoranda. Graft Investigation Officer’s
dismissed the case without prejudice which was disapproved by petitioner Ombudsman Marcelo and
issued another Order finding respondent liable prompting respondent to file MR which was denied, thus
he went to the CA. Meanwhile, Doromal retracted and confessed that the complaint was all fabricated.
The CA set aside the petitioner’s order and absolved respondent. The petitioner now claims that
reliance by him on the affidavits of Doromal and his witness in determining respondent’s administrative
liability did not deprive respondent due process.

ISSUE: Whether there is denial of due process.

RULING: NO! The standard of due process that must be met in administrative tribunals allows a certain
degree of latitude as long as fairness is not ignored. It is, therefore, not legally objectionable for being
violative of due process for an administrative agency to resolve a case based solely on position papers,
affidavits or documentary evidence submitted by the parties as affidavits of witnesses may take the
place of their direct testimonies.

After the filing of the Complaint, respondent was allowed by the Ombudsman to submit the following:
(a) a counter-affidavit refuting the charges against him; (b) a rejoinder-affidavit; and (c) a Motion for
Reconsideration of the Order of the Ombudsman. Moreover, respondent had the option to subject the
case to a formal investigation, but he opted to submit the case for resolution on the basis of the
affidavits on record.

SEC v. Interport 567 SCRA 354 - Bryan


G.R. No. 135808

Facts:

1) Board of Directors of IRC approved a Memorandum of Agreement (MoA) with Ganda Holdings
Berhad (GHB).
a. Under the MoA, IRC acquired 100% or the entire capital stock of Ganda Energy Holdings, Inc.
(GEHI), which would own and operate a 102 megawatt gas turbine power-generating barge.
b. Also stipulated is that GEHI would assume a five-year power purchase contract with National
Power Corp. At that time, GEHI’s power-generating barge was 97% complete and would go on-
line by mid-Sept 1994.

2) SEC averred that it received reports that IRC failed to make timely public disclosures of its
negotiations with GHB and that some of its directors heavily traded IRC shares utilizing this
material insider information.
3) IRC sent a letter to SEC, attaching copies of MoA and its directors appeared to explain IRC’s alleged
failure to immediately disclose material information as required under the Rules on Disclosure of
Material Facts.
4) 19 Sept 1994 – SEC Chairman issued an Order finding that IRC violated the Rules on Disclosure
when it failed to make timely disclosure, and that some of the officers and directors of IRC entered
into transactions involving IRC shares in violation of Sec 30, in relation to Sec 36 of the Revised
Securities Act.
5) 20 Aug 1998 – CA promulgated a Decision
a. Determined that there were no implementing rules and regulations regarding disclosure, insider
trading, or any of the provisions of the Revised Securities Acts which respondents allegedly
violated.
b. Further decided that the Rules of Practice and Procedure before the PED did not comply with the
statutory requirements contained in the Administrative Code of 1997. Section 9, Rule V of the
Rules of Practice and Procedure before the PED affords a party the right to be present but without
the right to cross-examine witnesses presented against him, in violation of Sec 12(3), Chap 3, Book
VII of the Administrative Code.

Issue: Whether or not PED Rules of Practice and Procedure was invalid since Section 8, Rule V thereof
failed to provide for the parties’ right to cross-examination.

Ruling:
As such, the PED Rules provided that the Hearing Officer may require the parties to submit their
respective verified position papers, together with all supporting documents and affidavits of witnesses.
A formal hearing was not mandatory; it was within the discretion of the Hearing Officer to determine
whether there was a need for a formal hearing. Since, according to the foregoing rules, the holding of a
hearing before the PED is discretionary, then the right to cross-examination could not have been
demanded by either party.

Calinisan v. Roaquin 630 456- Bryan


G.R. No. 159588

Facts:
SPO2 Roaquin of the PNP was charged with murder before the RTC of Olongapo City for killing
Alfredo Taluyo in a nightclub squabble.

While Roaquin was under detention, the PNP Headquarters of Regional Command 3 issued Special
Order 74, discharging him from the service. They discharged him notwithstanding that he had not been
administratively charged in connection with the offense of which he was charged in court.

Seven years later the RTC acquitted him of the crime of which he was charged upon a finding that
he acted in complete self-defense. Following this development, Roaquin asked the PNP to reinstate him
into the police service.

P/Chief Superintendent Roberto Calinisan, Director of the PNP Regional Office III, reinstated
Roaquin into service, citing Section 48 of R.A. 6975. However, P/Chief Superintendent Reynaldo Acop,
Head of the PNP Directorate for Personnel and Records Management, issued a memorandum, directing
Calinisan to nullify Roaquin’s reinstatement. Acop said that Roaquin could not be entitled to
reinstatement since he failed to file a motion for reconsideration within 10 days of being notified of his
discharge.

Acting on his superior’s order, Calinisan issued Special Orders 102, nullifying Roaquin’s
reinstatement. Roaquin sought reconsideration, but this was denied with an advice that he seek redress
in court.

Issue: Whether or not there is due process in the dismissal of respondent.

Ruling:
Section 45 that Roaquin’s superior officers invoked cannot apply to him since no one filed an
administrative action against him in connection with the crime of which he was charged in court. His
superiors did not adduce evidence during the trial before the RTC that such action had been filed. They
subsequently alleged in their pleadings the filing of some administrative case against him but they
provided neither the specifics of that case nor a document evidencing its existence.

At any rate, assuming that someone filed an administrative charge against Roaquin, still the law
required the PNP to give him notice of such charge and the right to answer the same. This does not
appear in the record. Additionally, Special Order 74 provided that Roaquin’s mode of discharge was
to be determined by higher headquarters. Again, nothing in the record of this case indicates that the
PNP investigated Roaquin or conducted a summary proceeding to determine his liability in connection
with the murder of which he was charged in court. The PNP gave him no chance to show why he
should not be discharged.
IBP v. Atienza 613 SCRA 518- Bryan
G.R. No. 175241

Facts:
The IBP, through its then National President Jose Anselmo Cadiz, filed with the Office of the
City Mayor of Manila a letter application for a permit to rally at the foot of Mendiola Bridge on June 22,
2006.

Respondent issued a permit dated June 16, 2006 allowing the IBP to stage a rally on given date but
indicated therein Plaza Miranda as the venue, instead of Mendiola Bridge, which permit the IBP received
on June 19, 2006.

Aggrieved, petitioners filed on June 21, 2006 before the Court of Appeals a petition for certiorari
docketed as CA-G.R. SP No. 94949.
The appellate court ruled, in CA-G.R. SP No. 94949, by the first assailed issuance, that the
petition became moot and lacked merit. The appellate court also denied petitioners’ motion for
reconsideration by the second assailed issuance.

Hence, the filing of the present petition for review on certiorari.

Issue: Whether or not there was grave abuse of discretion in modifying the venue of the rally.

Ruling:
In modifying the permit outright, respondent gravely abused his discretion when he did not
immediately inform the IBP who should have been heard first on the matter of his perceived imminent
and grave danger of a substantive evil that may warrant the changing of the venue. The opportunity to
be heard precedes the action on the permit, since the applicant may directly go to court after an
unfavorable action on the permit.

Domingo v. OMB 577 SCRA 476 - Bryan


G.R. No. 176127

Facts:
A complaint-affidavit was filed before the Office of the Ombudsman by Sangguniang Kabataan
(SK) officials against petitioner as Barangay Chairman and Barangay Treasurer Fe T. Lao, both of
Barangay 686, Zone 75, District V, Manila, for malversation, falsification of public document, dishonesty
and grave misconduct.

Respondents alleged that petitioner and Lao misappropriated the cash advance taken by
respondents from the SK funds amounting to P16,784.00 in the year 2002. They added that
petitioner gave a false statement in his Justification supporting the 2003 Barangay Budget and
Expenditures by declaring that his barangay had no incumbent SK officials at that time contrary to the
fact that respondents are duly elected and incumbent SK officials of the barangay.

The OMB rendered judgment finding petitioner guilty of violation of Section 4(b)
(professionalism) of R.A. No. 6713 (AN ACT ESTABLISHING A CODE OF CONDUCT AND ETHICAL
STANDARDS FOR PUBLIC OFFICIALS AND EMPLOYEES), and meted the penalty of suspension from office
for a period of 6 months.
After denial of his motion for reconsideration, petitioner filed a petition for review with the
Court of Appeals. The appellate court denied the petition and affirmed the OMB’s decision in toto.

Aggrieved, petitioner filed the instant petition seeking the reversal of the Court of Appeals’
decision on the ground that he cannot be held administratively liable for any act beyond his control and
knowledge under R.A. No. 6713.

Issue: Whether or not the petitioner was properly informed of the nature and cause of the charge
against him.

Ruling:
There is obviously a denial of due process in this case. The due process requirement mandates
that every accused or respondent be apprised of the nature and cause of the charge against him, and
the evidence in support thereof be shown or made available to him so that he can meet the charge with
traversing or exculpatory evidence. A cursory reading of the complaint-affidavit does not reveal that
petitioner was charged with violation of Section 4(b) of R.A. No. 6713. Likewise, in the OMB’s Evaluation
Report, the charges indicated were for malversation, falsification, dishonesty and grave misconduct.

Zambales v. CAstellejos 581 SCRA 320 - Bryan


G.R. Nos. 176935-36, March 13, 2009

Facts:
Jose S. Dominguez, Isaias Q. Vidua, Vicente M. Barretto, Jose M. Santiago, Jose
Naseriv C. Dolojan, Juan Fernandez and Honorio Dilag, Jr. are members of the Board of
Directors of the Zambales II Electric Cooperative, Inc. (hereafter ZAMECO II). ZAMECO II is an
electric cooperative organized and registered under Presidential Decree No. 269, as
amended.

National Electrification Administration (NEA) is a government owned and controlled


corporation organized under Presidential Decree (PD) No. 269.
Castillejos Consumers Associations, Inc. (hereafter CASCONA) is an organization of
electric consumers from the municipality of Castillejos, Zambales under the coverage area of
ZAMECO II.

CASCONA, through its Board of Trustees, filed a letter-complaint with NEA seeking
the removal of the petitioners for the following alleged offenses:

a. illegal payment of 13th Month Pay and Excessive Mid-Year and Christmas Bonus to
petitioners; and

b. overstaying as members of the Board of Directors of ZAMECO II.

The appellate court denied the consolidated petitions on the ground that NEA properly
exercised its supervisory power over ZAMECO II. Corollary to this ruling is the Court of Appeals’
declaration that petitioners have not been deprived of due process in the administrative proceedings.
Petitioners insist that they were denied due process as they were never notified of the charges
against them based on the July 24, 2003 Audit Report (2003 Audit Report). Allegedly, petitioners had
been asked to respond only to the charges under the June 25, 1998 Audit Report.

Issue: Whether or not there was a denial of due process.

Ruling:
It may be pointed out that in the Order dated November 6, 2003, the ADCOM mentioned an
agreement between the parties that the submission of their respective position papers shall be in lieu of
formal trial-type proceedings. This agreement, however, preceded CASCONA’s mention of the 2003
Audit Report on November 13, 2003. Therefore, it binds petitioners only insofar as they have effectively
waived a “hearing proceeding” on the 1998 Audit Report but not with respect to the 2003 Audit Report.

As such, NEA’s compliance with due process requirements should be evaluated based on the
standard set forth in Ang Tibay v. CIR, pertaining to the cardinal rights which must be observed in
proceedings before administrative tribunals, synthesized in a subsequent case as follows:

There are cardinal primary rights which must be respected even in proceedings of this character. The first
of these rights is the right to a hearing, which includes the right of the party interested or affected to
present his own case and submit evidence in support thereof. Not only must the party be given an
opportunity to present his case and to adduce evidence tending to establish the rights which he asserts
but the tribunal must consider the evidence presented. While the duty to deliberate does not impose the
obligation to decide right, it does imply a necessity which cannot be disregarded, namely, that of having
something to support its decision. Not only must there be some evidence to support a finding or
conclusion, but the evidence must be substantial. The decision must be rendered on the evidence
presented at the hearing, or at least contained in the record and disclosed to the parties affected.

OMB v. Evangelista 581 SCRA 350- Bryan


G.R. No. 177211; March 13, 2009

FACTS:
 Priscila Villanueva, co-chair of the Local School Board (LSB) of Aguirre, Pangasinan, filed an
administrative case with the Ombudsman against Mayor Evangelista, municipal treasurer
Melican, and municipal accountant Limos. She alleged that the three made use of the Special
Education Fund (SEF) to purchase speech kits and textbooks without the authorization of the
LSB and that the said speech kits and textbooks were not even received by the recipient schools.
 Villanueva also prayed for the preventive suspension of the three.
 Without furnishing the respondents with a copy of the complaint, the Ombudsman issued an
Order preventively suspending them for four (4) months, without pay.
 The respondents filed a petition for certiorari with the Court of Appeals.
 The Court of Appeals reversed the preventive suspension order by the Ombudsman on the
following grounds: (1) the Order was issued without compliance with Sec. 26 of the Ombudsman
Act which requires that the respondents be informed of the charges against them; (2) there was
haste in the issuance of the Order since the Ombudsman did not await the recommendation of
his deputy.
 Thus, the Ombudsman filed a Rule 65 certiorari with the Supreme Court.

Issue: Whether or not a preventive suspension order may issue even without notifying the respondent
in an administrative case of the charge/s against him.

Ruling:
YES. A preventive suspension order by the Ombudsman may issue even without notifying the
respondent of the charge as per the Ombudsman Act and the prevailing jurisprudence on the matter.

Under Sec. 24 of the Ombudsman Act, a preventive suspension order may issue when two
requisites concur: (1) evidence of guilt is strong; AND (2) the charge involves dishonesty, oppression,
grave misconduct, neglect in the performance of duty; OR the charge warrants the removal of the
officer from service; OR the respondent’s continued stay in office may prejudice the case against him.

Clearly, the plain language of the above-quoted provision debunks the appellate court’s position
that the order meting out preventive suspension may not be issued without prior notice and hearing
and before the issues are joined. Under Section 24, two requisites must concur to render the preventive
suspension order valid. The first requisite is unique and can be satisfied in only one way. It is that in the
judgment of the Ombudsman or the Deputy Ombudsman, the evidence of guilt is strong. The second
requisite, however, may be met in three (3) different ways, to wit: (1) that the offense charged involves
dishonesty, oppression or grave misconduct or neglect in the performance of duty; (2) the charge would
warrant removal from the service; or (3) the respondent’s continued stay in office may prejudice the
case filed against him.

Phil Export v. Pearl City 608 SCRA 280 - Bryan

Facts:
Petitioner Corporation [herein respondent Pearl City Manufacturing Corporation] is a PEZA-
registered Ecozone Export Enterprise located at the Mactan Economic Zone (MEZ) I in Lapu-Lapu City,
[province of Cebu] engaged in the business of recycling and processing, for export, of used clothing into
wool, fiber, cotton fiber, polyester fiber, useable clothing and industrial rags. Individual petitioners are
the employees of the petitioner Corporation.

Sometime in March 2004, petitioner Corporation, along with two (2) other PEZA-registered
companies importing used clothing, was informed of a physical inventory to be conducted by the PEZA
officers in their respective zones on their businesses.

After the completion of the physical inventory on the petitioner Corporation, PEZA officers
discovered that it had an unaccounted importation of 8,259,645 kilograms of used clothing for the
period of fifteen (15) months covering January 2003 up to March 2004.

Petitioner Corporation was then instructed to submit its explanation regarding the said
unaccounted shortage in its import-export liquidation. After submitting the required explanation,
petitioner Corporation was subjected to a special audit conducted by PEZA to determine the amount of
wastage generated by the company.

On the basis of the results of the physical inventory and the special audit conducted on the
petitioner Corporation, respondent [herein co-petitioner] PEZA Board passed a resolution [Resolution
No. 04-236] canceling the PEZA Registration of petitioner Corporation as an Ecozone Export Enterprise
at MEZ I.

An administrative appeal was filed by the petitioners to the Office of the President from the
resolution canceling its registration.

Issue: Whether or not in order to afford due process, interrogations or inquiries need to be conducted
by the PEZA Board to give PCMC the opportunity to defend itself from any charge directed against it.

Ruling:
It is settled that in administrative proceedings, a fair and reasonable opportunity to explain
one’s side suffices to meet the requirements of due process. The essence of procedural due process is
embodied in the basic requirement of notice and a real opportunity to be heard.

In the present case, since PCMC was properly informed of the supposed discrepancy in its import
and export liquidations, that it was given ample opportunity by the PEZA management to be heard or to
explain its side in relation to its unaccounted imported materials and that it was subsequently informed
of the decision of the PEZA Board to cancel its registration on the basis of its assessment of the evidence
presented or lack thereof, petitioners cannot claim that they were denied their right to due process of
law.

Pichay v. Office of the Deputy Executive Secretary 677 SCRA 408 – Bryan

Facts:
President Aquino issued EO 13 abolishing the Presidential Anti-Graft Commission (PAGC) and
transferring its functions to the Office of the Deputy Executive Secretary for Legal Affairs (ODESLA),
more particularly to its newly-established Investigative and Adjudicatory Division (IAD). Respondent
Purisima filed before the IAD-ODESLA a complaint against Pichay, chairman of the Board of Trustees of
the LWUA as well as the incumbent members of the LWUA Board of Trustees which arose from the
purchase by the LWUA shares of stock of Express Savings Bank Inc.

Issue: Whether or not there was due process.

Ruling:
Contrary to petitioner's assertions, his right to due process was not violated when the IAD-
ODESLA took cognizance of the administrative complaint against him since he was given sufficient
opportunity to oppose the formal complaint filed by Secretary Purisima. In administrative proceedings,
the filing of charges and giving reasonable opportunity for the person so charged to answer the
accusations against him constitute the minimum requirements of due process, which simply means
having the opportunity to explain one’s side. Hence, as long as petitioner was given the opportunity to
explain his side and present evidence, the requirements of due process are satisfactorily complied with
because what the law abhors is an absolute lack of opportunity to be heard. The records show that
petitioner was issued an Order requiring him to submit his written explanation under oath with respect
to the charge of grave misconduct filed against him. His own failure to submit his explanation despite
notice defeats his subsequent claim of denial of due process.

Arroyo v. DOJ 681 SCRA 181 - Bryan

Facts:
 This case stemmed from the allegations of massive electoral fraud and manipulation of election
results in the 2004 and 2007 National Elections.
 On August 15, 2011, the Comelec and the DOJ issued Joint Order No. 001-2011 creating and
constituting a Joint Committee and Fact-Finding Team on the 2004 and 2007 National Elections
electoral fraud and manipulation cases with the mandate to conduct the necessary preliminary
investigation on the basis of the evidence gathered and the charges recommended by the Fact-Finding
Team created and referred.
 The Fact-Finding Team recommended that petitioner Abalos and ten (10) others be subjected to
preliminary investigation for electoral sabotage for conspiring to manipulate the election results in
North and South Cotabato and Mauindanao.
 The Comelec’s Law Department filed with the Regional Trial Court (RTC), Pasay City, an Information
against petitioner GMA, Governor Andal Ampatuan, Sr., and Atty. Lintang H. Bedol. The case was
raffled to Branch 112 and the corresponding Warrant of Arrest was issued which was served on GMA
on the same day.
 On November 18, 2011, petitioner GMA filed with the RTC an Urgent Omnibus Motion Ad Cautelam
with leave to allow the Joint Committee to resolve the motion for reconsideration filed by GMA, to
defer issuance of a warrant of arrest and a Hold Departure Order, and to proceed to judicial
determination of probable cause. She, likewise, filed with the Comelec a Motion to Vacate Ad
Cautelam praying that its Resolution be vacated for being null and void. The RTC nonetheless issued a
warrant for her arrest which was duly served. GMA thereafter filed a Motion for Bail which was
granted.

Issues:
1. Whether or not Joint Order No. 001-2011 "Creating and Constituting a Joint DOJ-COMELEC
Preliminary Investigation Committee and Fact-Finding Team on the 2004 and 2007 National Elections
Electoral Fraud and Manipulation Cases" is constitutional in light of the following:
 due process clause –YES, it is constitutional

Ruling:
It is settled that the conduct of preliminary investigation is, like court proceedings, subject to the
requirements of both substantive and procedural due process. Preliminary investigation is considered as
a judicial proceeding wherein the prosecutor or investigating officer, by the nature of his functions, acts
as a quasi-judicial officer. The authority of a prosecutor or investigating officer duly empowered to
preside over or to conduct a preliminary investigation is no less than that of a municipal judge or even
an RTC Judge.

It must also be emphasized that Joint Order No. 001-2011 created two bodies, namely: (1) the
Fact-Finding Team tasked to gather real, documentary and testimonial evidence which can be utilized in
the preliminary investigation to be conducted by the Joint Committee; and (2) the Joint Committee
mandated to conduct preliminary investigation. It is, therefore, inaccurate to say that there is only
onebody which acted as evidence-gatherer, prosecutor and judge.

4. Extradition Proceedings

Sec, of Justice v. Lantion 343 SCRA 377 - Bryan

Facts:
Pursuant to the extradition treaty with the United States, a request was made by the U.S. for the
extradition of Mark Jimenez. While the petition for extradition was being evaluated by the Department
of Justice and before the filing of an extradition case in the proper court, a request was made by Jimenez
that documents coming from the US related to the extradition related to the extradition request be
made available to him. The request was granted by Judge Lantion. Whereupon the Secretary of Justice
asked the Court to reverse the lower courts oreder.

Issue: Whether or not the extraditee has the right to notice and hearing.

Ruling:
During the evaluation stage in the office of the Department of Justice the subject of the
extradition request does not have the right to notice and hearing. “An extradition is not a criminal
prosecution, and the constitutional safeguards that accompany a criminal trial in this country do not
shield an accused from extradition pursuant to a valid treaty.” As an extradition proceeding is not a
proceeding akin to a preliminary investigation, the due process safeguards in the latter do not
necessarily apply to the former.

Cuevas v. Munoz GR 140520 Dec. 18, 2000- Bryan

Facts:
The Hong Kong Magistrate’s Court at Eastern Magistracy issued a warrant for the arrest of
respondent Juan Antonio Muñoz for seven (7) counts of accepting an advantage as an agent and
seven(7) counts of conspiracy to defraud, contrary to the common law of Hong Kong. The Department
of Justice received a request for the provisional arrest of the respondent from the Mutual Legal
Assistance Unit, International Law Division of the Hong Kong Department of Justice pursuant to Article
11(1) of the RP-Hong Kong Extradition Agreement. Upon application of the NBI, RTC of Manila issued an
Order granting the application for provisional arrest and issuing the corresponding Order of Arrest.
Consequently, respondent was arrested pursuant to the said order, and is currently detained at the NBI
detention cell. Respondent filed with the Court of Appeals, a petition for certiorari, assailing the validity
of the Order of Arrest. The Court of Appeals rendered a decision declaring the Order of Arrest null and
void on the grounds, among others that the request for provisional arrest and the accompanying
warrant of arrest and summary of Facts were unauthenticated and mere facsimile copies which are
insufficient to form a basis for the issuance of the Order of Arrest. Thus, petitioner Justice Serafin R.
Cuevas, in his capacity as the Secretary of the Department of Justice, lost no time in filing the instant
petition.

Respondent posits that the admission by the RTC of the request for provisional arrest and its
supporting documents despite lack of authentication is a violation of the respondent’s right to due
process.

Issue: Whether or not the request for provisional arrest of respondent and its accompanying documents
must be authenticated.

Ruling:
We have explained why an extraditee has no right to notice and hearing during the evaluation
stage of the extradition process. As aforesaid, P.D. 1069 xxx affords an extraditee sufficient opportunity
to meet the evidence against him once the petition is filed in court. The time for the extraditee to know
the basis of the request for his extradition is merely moved to the filing in court of the formal petition for
extradition. The extraditee’s right to know is momentarily withheld during the evaluation stage of the
extradition process to accommodate the more compelling interest of the State to prevent escape of
potential extraditees which can be precipitated by premature information of the basis of the request for
his extradition.

Gov’t. of U.S.A v. Purganan GR 148571 Sept. 24, 2002- Bryan

Facts:
This Petition is really a sequel to GR No. 139465 entitled Secretary of Justice v. Ralph C. Lantion.
Pursuant to the existing RP-US Extradition Treaty, 6 the United States Government, through diplomatic
channels, sent to the Philippine Government Note Verbale No. 0522 dated June 16, 1999, supplemented
by Note Nos. 0597, 0720 and 0809 and accompanied by duly authenticated documents requesting the
extradition of Mark B. Jimenez, also known as Mario Batacan Crespo. Upon receipt of the Notes and
documents, the secretary of foreign affairs (SFA) transmitted them to the secretary of justice (SOJ) for
appropriate action, pursuant to Section 5 of Presidential Decree (PD) No. 1069, also known as the
Extradition Law. Upon learning of the request for his extradition, Jimenez sought and was granted a
Temporary Restraining Order (TRO) by the RTC of Manila, Branch 25. 7 The TRO prohibited the
Department of Justice (DOJ) from filing with the RTC a petition for his extradition. The validity of the TRO
was, however, assailed by the SOJ in a Petition before this Court in the said GR No. 139465. Initially, the
Court — by a vote of 9-6 — dismissed the Petition. The SOJ was ordered to furnish private respondent
copies of the extradition request and its supporting papers and to grant the latter a reasonable period
within which to file a comment and supporting evidence.
Issue: Whether Jimenez is entitled to notice and hearing before a warrant for his arrest can be issued.

Ruling:
It is significant to note that Section 6 of PD 1069, our Extradition Law, uses the word
“immediate” to qualify the arrest of the accused. This qualification would be rendered nugatory by
setting for hearing the issuance of the arrest warrant. Hearing entails sending notices to the opposing
parties, receiving Facts and arguments from them, and giving them time to prepare and present such
Facts and arguments. Arrest subsequent to a hearing can no longer be considered “immediate.” The law
could not have intended the word as a mere superfluity but, on the whole, as a means of imparting a
sense of urgency and swiftness in the determination of whether a warrant of arrest should be issued.

By using the phrase “if it appears,” the law further conveys that accuracy is not as important as speed at
such early stage. The trial court is not expected to make an exhaustive determination to ferret out the
true and actual situation, immediately upon the filing of the petition. From the knowledge and the
material then available to it, the court is expected merely to get a good first impression — a prima facie
finding — sufficient to make a speedy initial determination as regards the arrest and detention of the
accused.

WHEREFORE, the Petition is GRANTED.

Rodriguez v. Presiding Judge, 483 SCRA 290 - Bryan

Facts:
The case stemmed from the petition for extradition filed on March 12, 2001 by the
Government of the United States of America (US government) through the Department of Justice (DOJ)
against the petitioners.

After their arrest, petitioners applied for bail which the trial court granted on September 25,
2001. The bail was set for one million pesos for each. Petitioners then posted cash bonds. The US
government moved for reconsideration of the grant of bail, but the motion was denied by the trial
court. Unsatisfied, the US government filed a petition for certiorari with this Court.
Thereafter, we directed the trial court to resolve the matter of bail which, according to its
November 28, 2001 Order, shall be subject to whatever ruling that this Court may have in the similar
case of Mark Jimenez entitled Government of the United States of America v. Purganan. In compliance
with our directive, the trial court, without prior notice and hearing, cancelled the cash bond of the
petitioners and ordered the issuance of a warrant of arrest.

Issue: Whether or not prior notice and hearing is necessary for cancellation of bail.

Ruling:
Under these premises, and with the trial court’s knowledge that in this case, co-petitioner has
offered to go on voluntary extradition; that she and her husband had posted a cash bond of P1 million
each; that her husband had already gone on voluntary extradition and is presently in the USA
undergoing trial; that the passport of co-petitioner is already in the possession of the authorities; that
she never attempted to flee; that there is an existing hold-departure order against her; and that she is
now in her sixties, sickly and under medical treatment, we believe that the benefits of continued
temporary liberty on bail should not be revoked and their grant of bail should not be cancelled, without
the co-petitioner being given notice and without her being heard why her temporary liberty should
not be discontinued.

We emphasize that bail may be granted to a possible extraditee only upon a clear and
convincing showing (1) that he will not be a flight risk or a danger to the community, and (2) that
there exist special, humanitarian and compelling circumstances.

Gov’t. of Hong Kong v. Olalia, GR 153675 April 19, 2007- Bryan

Facts:
The Philippines and Hong Kong signed an “Agreement for the Surrender of Accused and
Convicted Persons.”
Private respondent Muñoz was charged before the Hong Kong Court. Department of
Justice(DOJ) received from the Hong Kong Department of Justice a request for the provisional arrest of
private respondent Muñoz. The DOJ then forwarded the request to the National Bureau of Investigation
(NBI) which, in turn, filed with the RTC of Manila, Branch 19 an application for the provisional arrest of
private respondent. The NBI agents arrested and detained him.
Muñoz filed a petition for bail which was denied by Judge Bernardo, Jr. holding that there is no
Philippine law granting bail in extradition cases and that private respondent is a high “flight risk.” After
Judge Bernardo, Jr. inhibited himself from further hearing the case, it was then raffled off to Branch 8
presided by respondent judge. Private respondent filed a motion for reconsideration of the Order
denying his application for bail and this was granted by respondent judge.

ISSUE
Whether or not the trial court committed grave abuse of discretion amounting to lack or excess of
jurisdiction in allowing private respondent to bail?

Ruling:
No, the trial court did not commit grave abuse of discretion amounting to lack or excess of
jurisdiction in allowing private respondent to bail. Accordingly, although the time-honored principle of
pacta sunt servanda demands that the Philippines honor its obligations under the Extradition Treaty it
entered into with the Hong Kong Special Administrative Region it does not necessarily mean that in
keeping with its treaty obligations, the Philippines should diminish a potential extraditee’s rights to life,
liberty, and due process guaranteed by the Constitution.
We should not, therefore, deprive an extraditee of his right to apply for bail, provided that a
certain standard for the grant is satisfactorily met. In his Separate Opinion in Purganan, then Associate
Justice, now Chief Justice Reynato S. Puno, proposed that a new standard which he termed “clear and
convincing evidence” should be used in granting bail in extradition cases. According to him, this standard
should be lower than proof beyond reasonable doubt but higher than preponderance of evidence. The
potential extraditee must prove by “clear and convincing evidence” that he is not a flight risk and will
abide with all the orders and processes of the extradition court. In this case, there is no showing that
private respondent presented evidence to show that he is nota flight risk. Consequently, this case should
be remanded to the trial court to determine whether private respondent may be granted bail on the
basis of “clear and convincing evidence.”

5. Arbitration

RCBC v. Banco de Oro 687 SCRA 583 – Bryan


G.R. No. 196171 December 10, 2012

Facts:
On May 24, 2000, RCBC entered into a Share Purchase Agreement (SPA) with Equitable-PCI
Bank, Inc. (EPCIB), George L. Go and the individual shareholders of Bankard, Inc. (Bankard) for the sale to
RCBC of 226,460,000 shares (Subject Shares) of Bankard, constituting 67% of the latter’s capital stock.
After completing payment of the contract price (P1,786,769,400), the corresponding deeds of sale over
the subject shares were executed in January 2001.

The dispute between the parties arose sometime in May 2003 when RCBC informed EPCIB and the other
selling shareholdersof an overpayment of the subject shares, claiming there was an overstatement of
valuation of accounts amounting to P478 million and that the sellers violated their warrantyunder
Section 5(g)of the SPA.

Meanwhile, EPCIB’s corporate name was officially changed to Banco De Oro (BDO)-EPCIB after
its merger with BDO was duly approved by the Securities and Exchange Commission. As such, BDO
assumed all the obligations and liabilities of EPCIB under the SPA.

As no settlement was reached, RCBC commenced arbitration proceedings with the International
Chamber of Commerce-International Court of Arbitration (ICC-ICA)

RCBC refuted Respondents’ allegation of partiality on the part of Chairman Barker and reiterated the
prayer in its application for reimbursement of advance on costs paid to the ICC-ICA. RCBC contended
that based on Mr. Secomb’s article, whether the "contractual" or "provisional measures" approach is
applied, the Arbitration Tribunal is vested with jurisdiction and authority to render an award with
respect to said reimbursement of advance cost paid by the non-defaulting party.

Respondents, on the other hand, maintained that RCBC’s application for reimbursement of advance cost
has no basis under the ICC Rules. They contended that no manifest injustice can be inferred from an act
of a party paying for the share of the defaulting party as this scenario is allowed by the ICC Rules.
Neither can a partial award for advance cost be justified under the "contractual approach" since the
matter of costs for arbitration is between the ICC and the parties, not the Arbitration Tribunal and the
parties. An arbitration tribunal can issue decisions on costs only for those costs not fixed by the ICC.

The ICC ruled in favor of respondent. This was however reversed by the CA. Hence this petition.

Issue: Whether or not there was evident partiality.


Ruling:
That there was an action to be taken beforehand is confirmed by Chairman Barker’s furnishing
the parties with a copy of the Secomb article. This article ultimately favored RCBC by advancing its
cause. Chairman Barker makes it appear that he intended good to be done in doing so but due process
dictates the cold neutrality of impartiality. This means that "it is not enough…[that] cases [be decided]
without bias and favoritism. Nor is it sufficient that…prepossessions [be rid of]. [A]ctuations should
moreover inspire that belief." These put into the equation, the furnishing of the Secomb article further
marred the trust reposed in Chairman Barker. The suspicion of his partiality on the subject matter
deepened. Specifically, his act established that he had pre-formed opinions.

We affirm the foregoing findings and conclusion of the appellate court save for its reference to the
obiter in Commonwealth Coatings that arbitrators are held to the same standard of conduct imposed on
judges. Instead, the Court adopts the reasonable impression of partiality standard, which requires a
showing that a reasonable person would have to conclude that an arbitrator was partial to the other
party to the arbitration. Such interest or bias, moreover, "must be direct, definite and capable of
demonstration rather than remote, uncertain, or speculative." When a claim of arbitrator’s evident
partiality is made, "the court must ascertain from such record as is available whether the arbitrators’
conduct was so biased and prejudiced as to destroy fundamental fairness."

Applying the foregoing standard, we agree with the CA in finding that Chairman Barker’s act of
furnishing the parties with copies of Matthew Secomb’s article, considering the attendant
circumstances,is indicative of partiality such that a reasonable man would have to conclude that he was
favoring the Claimant, RCBC. Even before the issuance of the Second Partial Award for the
reimbursement of advance costs paid by RCBC, Chairman Barker exhibited strong inclination to grant
such relief to RCBC, notwithstanding his categorical ruling that the Arbitration Tribunal "has no power
under the ICC Rules to order the Respondents to pay the advance on costs sought by the ICC or to give
the Claimantany relief against the Respondents’ refusal to pay." That Chairman Barker was predisposed
to grant relief to RCBC was shown by his act of interpreting RCBC’s letter, which merely reiterated its
plea to declare the Respondents in default and consider all counterclaims withdrawn – as what the ICC
Rules provide – as an application to the Arbitration Tribunal to issue a partial award in respect of BDO’s
failure to share in the advance costs. It must be noted that RCBC in said letter did not contemplate the
issuance of a partial order, despite Chairman Barker’s previous letter which mentioned the possibility of
granting relief upon the parties making submissions to the Arbitration Tribunal. Expectedly, in
compliance with Chairman Barker’s December 18, 2007 letter, RCBC formally applied for the issuance of
a partial award ordering BDO to pay its share in the advance costs.

Academic Discipline

1. In General

Angeles v. Sison 112 SCRA 26- Bryan

FACTS:
The petitioner was mauled by two of his students in FEU outside of the school campus,
specifically in Oak Barrel Restaurant, prompting his filing of criminal complaint for assault which was
dismissed on the basis of an affidavit of desistance submitted by the petitioner.
The administrative complaint filed before the Dean, however, was acted on by the creation of a
committee to conduct administrative investigation, headed by the Dean himself.
Over the opposition of the offending parties, the respondent Judge issued an order denying their motion
against the conduct of administrative investigation, hence this appeal before the Court.

ISSUE: Does the school, through its authorized representative, have jurisdiction to investigate over an
alleged misconduct committed outside the school premises and beyond school hours?

Ruling:
YES. A college or any school for that matter has a dual responsibility to its students. One is to
provide opportunities for learning and the other is to help them grow and develop into mature,
responsible, effective and worthy citizens of the community. Discipline is one of the means to carry out
the second responsibility.
Thus, there can be no doubt that the establishment of an educational institution requires rules and
regulations necessary for the maintenance of an orderly educational program and the creation of an
educational environment conducive to learning. Such rules and regulations are equally necessary for the
protection of the students, faculty, and property. The power of school officials to investigate, an adjunct
of its power to suspend or expel, is a necessary corollary to the enforcement of such rules and
regulations and the maintenance of a safe and orderly educational environment conducive to learning.
Common sense dictates that the school retains its power to compel its students in or off-campus to a
norm of conduct compatible with their standing as members of the academic community. Hence, when
as in the case at bar, the conduct complained of directly affects the suitability of the alleged violators as
students, there is no reason why the school cannot impose the same disciplinary action as when the act
took place inside the campus.

Malabanan v. Ramento 129 SCRA 359 – Bryan


G.R. NO.62270; 21 MAY 1984

Facts:
Petitioners were officers of the Supreme Student Council of respondent University. They sought
and were granted by the school authorities a permit to hold a meeting from 8:00 A.M. to 12:00 P.M, on
August 27, 1982. Pursuant to such permit, along with other students, they held a general assembly at
the Veterinary Medicine and Animal Science basketball court (VMAS), the place indicated in such permit,
not in the basketball court as therein stated but at the second floor lobby. At such gathering they
manifested in vehement and vigorous language their opposition to the proposed merger of
the Institute of Animal Science with the Institute of Agriculture. The same day, they marched toward the
Life Science Building and continued their rally. It was outside the area covered by their permit. Even they
rallied beyond the period allowed. They were asked to explain on the same day why they should not be
held liable for holding an illegal assembly. Then on September 9, 1982, they were informed that they
were under preventive suspension for their failure to explain the holding of an illegal assembly. The
validity thereof was challenged by petitioners both before the Court of First Instance of Rizal against
private respondents and before the Ministry of Education, Culture, and Sports. Respondent Ramento
found petitioners guilty of the charge of illegal assembly which was characterized by the violation of the
permit granted resulting in the disturbance of classes and oral defamation. The penalty was suspension
for one academic year. Hence this petition.

Issue: Whether or not the penalty of one year suspension, for illegal assembly is proper.

Ruling:
The rights to peaceable assembly and free speech are guaranteed students of educational
institutions. Necessarily, their exercise to discuss matters affecting their welfare or involving public
interest is not to be subjected to previous restraint or subsequent punishment unless there be a
showing of a clear and present danger to a substantive evil that the state, has a right to present. As a
corollary, the utmost leeway and scope is accorded the content of the placards displayed or utterances
made. The peaceable character of an assembly could be lost, however, by an advocacy of disorder under
the name of dissent, whatever grievances that may be aired being susceptible to correction through the
ways of the law. If the assembly is to be held in school premises, permit must be sought from its school
authorities, who are devoid of the power to deny such request arbitrarily or unreasonably. In granting
such permit, there may be conditions as to the time and place of the assembly to avoid disruption of
classes or stoppage of work of the non-academic personnel. Even if, however, there be violations of its
terms, the penalty incurred should not be disproportionate to the offense.

Guzman v. NU 142 SCRA 699 - Bryan


G.R. No. L-68288 July 11, 1986 (142 SCRA 699)

Facts:
The petitioners sought re-enrolment in the National University (NU). NU refused admission of
petitioners because the latter actively participated in mass actions, some have no permit, within the
premises of the University. In effect, according to NU, they subject themselves to expulsion without
cause, notice, or hearing. One of petitioners do not have any disciplinary charges but was denied re-
enrollment. Some of petitioners have “poor” grades and NU started a pending criminal action for
malicious mischief against Guzman as a consequence of destruction to NU's property. In connection to
this criminal action, the Supreme Court in past ruled that Guzman should be re-enrolled with
continuance of NU's disciplinary proceeding. This is a new action against Guzman and others.

Issue: Must the imposition of a school's disciplinary penalty observe due process similar to the due
process in the courts of justice?

Ruling:
NO. The due process in disciplinary cases involving students does not entail proceedings and
hearings similar to those prescribed for actions and proceedings in courts of justice. The proceedings in
student discipline cases may be summary; and cross-examination is not, 'contrary to petitioners' view,
an essential part thereof.

There are withal minimum standards which must be met to satisfy the demands of procedural
due process; and these are, that (1) the students must be informed in writing of the nature and cause of
any accusation against them; (2) they shall have the right to answer the charges against them, with the
assistance of counsel, if desired; (3) they shall be informed of the evidence against them; (4) they shall
have the right to adduce evidence in their own behalf; and (5) the evidence must be duly considered by
the investigating committee or official designated by the school authorities to hear and decide the case.

From the Facts presented by petitioners and respondent, there was no disciplinary proceeding instituted
and it leaved doubt as to the presence and extent of the petitioners' participation in the mass actions
against the school. Respondent also failed to present a duly published rules and regulations as basis for
expulsion and non-admission. Moreover, the respondent violated a number the provisions of Under the
Education Act of 1982 Manual of Regulations for Private Schools such as the student's right to continue
education until graduation, and the imposition of disciplinary penalty after due investigation.

Alcuaz v. PSBA 161 SCRA 7 – Kaiser


FACTS:In 1986, some PSBA students, herein petitioners Alcuaz et. al. staged demonstrations in the
premises of the school. In order for the demonstration to be settled, an agreement was entered into
among others the regulations for the conduct of protest action. In spite of the agreement, it was alleged
that the petitioners, committed tumultuous and anarchic acts within the premises of the school, fanned
by the cooperation of the intervening professors, causing disruption of classes to the prejudice of the
majority students. The school took administrative sanctions upon them in view of their participation in
the demonstration. The students and the intervening professors were sanctioned. They were dismissed
and terminated.
ISSUE: Whether or not there has been a deprivation of constitutional rights of expression and assembly
and of due process of law of the students who have been barred from re-enrollment.
HELD: The Supreme Court held that due process in disciplinary cases such as the case at bar does not
entail proceedings and hearings similar to those prescribed for actions and proceedings in the courts of
justice. The Court has already recognized the right of the school to refuse re-enrollment of students for
academic delinquency and violation of disciplinary regulations. In the school’s administrative process,
both students and professors were given three (3) days from receipt of letter to explain in writing why
the school should not take administrative sanction against them. With respect to the academic activities
of the students and the teaching loads of the teachers, the respondent school has created new class for
the petitioners and the intervening professors during and when the investigation was going on.
The Court then upheld that there is no denial of due process where all requirements of administrative
due process were met by the school and the students were given the opportunity to be heard and that
the right of expression and assembly are not absolute especially when parties are bound to certain rules
under a contract.
Non v. Judge Dames 185 SCRA 523
FACTS: Petitioners, students in private respondent Mabini Colleges, Inc. were not allowed to re-enroll by
the school for the academic year 1988-1989 for leading or participating in student mass actions against
the school in the preceding semester. The subject of the protests is not, however, made clear in the
pleadings.
The trial court dismissed the petition referring to the ruling in Alcuaz vs. PSBA stating, that being a mere
privilege and not a legal right for a student to be enrolled or re-enrolled, respondent Mabini College is
free to admit or not admit the petitioners for re-enrollment in view of the academic freedom enjoyed by
the school.
The respondents, in justifying their action, stated that 8 of the petitioners have incurred failing grades.
In response, the petitioners stated that: (a) three of them were graduating. (b) Their academic
deficiencies do not warrant non-readmission. (c) The improper conduct attributed to them was during
the exercise of the cognate rights of free speech and peaceable assembly. (d) There was no due
investigation that could serve as basis for disciplinary action. (e) Respondent school is their choice
institution near their places of residence, which they can afford to pay for tertiary education.
ISSUE: Whether or not the school has the right not to re-admit the petitioners.
HELD: The Supreme Court ruled that the trial court cannot anchor the “Termination of Contract” theory
the contract between the school and the student is not an ordinary contract. It is imbued with public
interest, considering the high priority given by the Constitution to education and the grant to the State
of supervisory and regulatory powers over all educational institutions. It is intended merely to protect
schools wherein tuition fees are collected and paid on installment basis. It cannot be construed to mean
that a student shall be enrolled for only one semester.
The right of an institution of higher learning to set academic standards cannot be utilized to discriminate
against students who exercise their constitutional rights to speech and assembly, for otherwise there
will be a violation of their right to equal protection. It provides that every student has the right to enroll
in any school college or university upon meeting its specific requirements and reasonable regulations; . .
. and that “the student is presumed to be qualified for enrollment for the entire period he is expected to
complete the course, without prejudice to his right to transfer.”
ADMU v. Capulong 222 SCRA 644
FACTS:On February 8, 9, and 10 of 1991, a fraternity in Ateneo Law School named Aquila Legis
conducted its initiation rites upon neophytes. Unfortunately, one neophyte died as a result thereof and
one was hospitalized due to serious physical injuries. In a resolution dated March 9, 1991, the
Disciplinary Board formed by Ateneo found seven students guilty of violating Rule 3 of the Rules on
Discipline. Fr. Joaquin Bernas, then president of Ateneo, on the basis of the findings, ordered the
expulsion of the seven students. However, on May 17, 1991, Judge Ignacio Capulong of the Makati RTC,
upon the students’ petition for certiorari, prohibition, and mandamus, ordered Ateneo to reverse its
decision and reinstate the said students.
ISSUE: Whether or not the Ateneo Law School may order for dismissing such students pursuant to its
rules.
HELD: Yes, Ateneo has the competence and the power to dismiss its erring students and therefore it had
validly exercised such power. The students do not deserve to claim such a venerable institution such as
Ateneo as their own a minute longer for they may forseeably cast a malevolent influence on students
currently enrolled as well as those who come after them. This is academic freedom on the part of the
school which includes:

a. freedom to determine who may teach;

b. freedom to determine what may be taught;

c. freedom to determine how it shall be taught;

d. freedom to determine who may be admitted to study.

U.P. v. Ligot-Telan 227 SCRA 342


Facts: Petitioner was a student of U.P Dilliman, on her dissertation certain corrections were requested of
her to be made for her to complete her requirements to get her Doctor’s degree in anthropology,
petitioner was still able to graduate, however after an investigation was conducted it was found out that
she plagiarized certain part of her dissertation hence the school revoked her doctors diploma.
Issue W/N there was a violation of her rights when her diploma was revoked.
Held: No. In the case at bar, the Board of Regents determined, after due investigation conducted by a
committee composed of faculty members from different U.P. units, that private respondent committed
no less than ninety (90) instances of intellectual dishonesty in her dissertation. The Board of Regents'
decision to withdraw private respondent's doctorate was based on documents on record including her
admission that she committed the offense.
private respondent was afforded the opportunity to be heard and explain her side but failed to refute
the charges of plagiarism against her. Her only claim is that her responses to the charges against her
were not considered by the Board of Regents before it rendered its decision. However, this claim was
not proven. Accordingly, we must presume regularity in the performance of official duties in the
absence of proof to the contrary.

Go v. Colegio De San Juan de Letran 683 SCRA 358


Facts: after an investigation conducted by the respondent, petitioner was suspended from school for
being a senior member of a fraternity which was not allowed in school. Petitioners now claim that they
were denied of due process as their side was never heard.
Issue: W/N there was a denial of due process.
Held: No. disciplinary proceedings may be summary, the insistence that a "formal inquiry" on the
accusation against Kim should have been conducted lacks legal basis. It has no factual basis as well.
While the petitioners state that Mr. and Mrs. Go were "never given an opportunity to assist Kim,"56 the
records show that the respondents gave them two (2) notices, dated December 19, 2001 and January 8,
2002, for conferences on January 8, 2002 and January 15, 2002. The notices clearly state: "Dear
Mr./Mrs. Go, We would like to seek your help in correcting Kim’s problem on: Discipline & Conduct
Offense: Membership in Fraternity." Thus, the respondents had given them ample opportunity to assist
their son in his disciplinary case.
The records also show that, without any explanation, both parents failed to attend the January 8, 2002
conference while Mr. Go did not bother to go to the January 15, 2002 conference. "Where a party was
afforded an opportunity to participate in the proceedings but failed to do so, he cannot [thereafter]
complain of deprivation of due process."
Through the notices, the respondents duly informed the petitioners in writing that Kim had a disciplinary
charge for fraternity membership. At the earlier November 23, 2001 Parents-Teachers Conference, Mr.
Rosarda also informed Mrs. Go that the charge stemmed from the fraternity neophytes’ positive
identification of Kim as a member; thus the petitioners fully knew of the nature of the evidence that
stood against Kim.

Deportation Proceeding

1. In General

Lao Gi v. CA 180 SCRA 756- Kaiser


Facts: Filomeno Chia Jr. was made a Filipino citizen by virtue of Opinion 191 by the Secretary of justice.
However, this was revoked when his father’s citizenship was cast aside due to fraud and
misrepresentation.Charges of deportation were filed against the Chias. Charges also alleged that they
refused to register as aliens and that they committed acts of undesirability. The Chias said that the CID
has no authority to deport them which was denied by the CID. They filed a petition with the Supreme
Court for a writ of preliminary injunction which was dismissed for lack of merit. Their MFR was also
denied. Earlier, Manuel Chia’s case of falsification of public documents in alleging he was a Filipino
citizen. He was alleged to have done this for the sale of real property. The trial court acquitted him by
saying that Opinion 191 was res judicata and cant be contravened by Opinion 147. The CID set the
hearing for the deportation case against the Chias and told them to register as aliens.
In their SC petition, they seek to set aside the CA decision. They argued that they weren’t subject to
immediate deportation, the presence of fraud in the citizenship, the CA’s overstepping of appellate
jurisdiction, and the resolution of the SC didn’t make a ruling that the petitioner entered the Philippines
by false pretenses.
Issue: Does the CID have the jurisdiction to determine the deportation?
Held: Yes. Petition granted Hearing must be continued to determine if they are really aliens
Ratio:
Section 37 of the Immigration act states:
SEC. 37. (a) The following aliens shall be arrested upon the warrant of the Commissioner of
Immigration or of any other officer designated by him for the purpose and deported upon the warrant
of the Commissioner of Immigration after a determination by the Board of Commissioners of the
existence of the ground for deportation as charged against the alien:
(1) Any alien who enters the Philippines after the effective date of this Act by means of false and
misleading statements or without inspection and admission by the immigration authorities at a
designated port of entry or at any place other than at a designated port of entry. (As amended by Sec.
13, Rep. Act No. 503.) ...
There must be a determination of the existence of the ground charged, particularly illegal entry into the
country. Only after the hearing can the alien be deported. Also, there must be appositive finding from
the CID that they are aliens before compelling them to register as such. This power is the police power
to protect the state from undesirable aliens injurious to the public good. Since the deportation is a harsh
process, due process must be observed. In the same law, it is provided that:
No alien shall be deported without being informed of the specific grounds for deportation nor without
being given a hearing under rules of procedure to be prescribed by the Commissioner of Immigration.
The acts or omissions that they are charged of must be in ordinary language for the person to be
informed and for the CID to make a proper judgment. Also, the warrants of arrest must be in accordance
with the rules on criminal procedure.
On the information of a private prosecutor in the case: Deportation is the sole concern of the state.
There is no justification for a private party to intervene.
Domingo v Scheer, 421 SCRA 468
Facts:in 1995 respondent a German citizen was being deported by the BID by merely relying on the
letter of the vice-consul of Germany in the Philippines, no hearing was conducted to hear petitioners
side. On 2002 he was arrested and subjected to summary deportation proceedings, respondent allege
that he now has regained his passport and is was a law-abiding citizen during his stay in the Philippines.
The CA granted petitioners motion not to be deported.
Issue: W/N CA has jurisdiction over the BID.
Held: We do not agree with the petitioner’s contention that the issue before the CA, as to the power of
the President to determine whether an alien may remain or be deported from the Philippines, is beyond
the appellate court’s competence to delve into and resolve. The contention of the petitioner is based
on a wrong premise.
The settled rule is that the authority to exclude or expel aliens by a power affecting international
relation is vested in the political department of the government, and is to be regulated by treaty or by
an act of Congress, and to be executed by the executive authority according to the regulations so
established, except in so far as the judicial department has been authorized by treaty or by statute, or is
required by the Constitution to intervene. The judicial department cannot properly express an opinion
upon the wisdom or the justice of the measures executed by Congress in the exercise of the power
conferred on it, by statute or as required by the Constitution. Congress may, by statute, allow the
decision or order of the Immigration Commissioner or the BOC to be reviewed by the President of the
Philippines or by the courts, on the grounds and in the manner prescribed by law.
Regulations: Fixing of Rates and Regulation of Profession

1. Rates

Philcomsat v. Alcuaz 180 SCRA 218


FACTS: By virtue of RA 5514, Philippine Communications Satellite Corporation was granted “a franchise
to establish, construct, maintain and operate in the Philippines, at such places as the grantee may select,
station or stations and associated equipment and facilities for international satellite communications.”
Under this franchise, it was likewise granted the authority to “construct and operate such ground
facilities as needed to deliver telecommunications services from the communications satellite system
and ground terminal or terminals.” Under Sec 5 of the same law, PhilComSat was exempt from the
jurisdiction, control and regulation of the Public Service Commission later known as the National
Telecommunications Commission. However, EO 196 was later proclaimed and the same has placed
PhilComSat under the jurisdiction of NTC. Consequently, PhilComSat has to acquire permit to operate
from NTC in order to continue operating its existing satellites. NTC gave the necessary permit but it
however directed PhilComSat to reduce its current rates by 15%. NTC based its power to fix the rates on
EO 546. PhilComSat assailed the said directive and holds that the enabling act (EO 546) of respondent
NTC empowering it to fix rates for public service communications does not provide the necessary
standards constitutionally required hence there is an undue delegation of legislative power, particularly
the adjudicatory powers of NTC. PhilComSat asserts that nowhere in the provisions of EO 546, providing
for the creation of respondent NTC and granting its rate-fixing powers, nor of EO 196, placing petitioner
under the jurisdiction of respondent NTC, can it be inferred that respondent NTC is guided by any
standard in the exercise of its rate-fixing and adjudicatory powers. PhilComSat subsequently clarified its
said submission to mean that the order mandating a reduction of certain rates is undue delegation not
of legislative but of quasi-judicial power to respondent NTC, the exercise of which allegedly requires an
express conferment by the legislative body.
ISSUE: Whether or not there is an undue delegation of power.
HELD: Fundamental is the rule that delegation of legislative power may be sustained only upon the
ground that some standard for its exercise is provided and that the legislature in making the delegation
has prescribed the manner of the exercise of the delegated power. Therefore, when the administrative
agency concerned, NTC in this case, establishes a rate, its act must both be non-confiscatory and must
have been established in the manner prescribed by the legislature; otherwise, in the absence of a fixed
standard, the delegation of power becomes unconstitutional. In case of a delegation of rate-fixing
power, the only standard which the legislature is required to prescribe for the guidance of the
administrative authority is that the rate be reasonable and just. However, it has been held that even in
the absence of an express requirement as to reasonableness, this standard may be implied. In the case
at bar, the fixed rate is found to be of merit and reasonable.

Radiocom v. NTC 184 SCRA 517


Facts: RCPI operated a radio communications system since 1957 under legislative franchise granted by
Republic Act No. 2036 (1957). The petitioner established a radio telegraph service in Sorsogon, Sorsogon
(1968). in San Jose, Mindoro (1971), and Catarman, Samar (1983).
Kayumanggi Radio, on the other hand, was given the rights by the NTC to operate radio networks in the
same areas.
RCPI filed a complaint in the NTC and sought to prohibit Kayumanggi Radio to operate in the same areas.
The NTC ruled against the RTC’s favor and commanded RCPI to desist in the operation of radio
telegraphs in the three areas.
Issue: Whether or not petitioner RCPI, a grantee of a legislative franchise to operate a radio company, is
required to secure a certificate of public convenience and necessity before it can validly operate its radio
stations including radio telephone services in the aforementioned areas
Held: Yes. The position of the petitioner that by the mere grant of its franchise under RA No. 2036 it can
operate a radio communications system anywhere within the Philippines is erroneous.
Sec. 4(a). This franchise shall not take effect nor shall any powers thereunder be exercised by the
grantee until the Secretary of Public works and Communications shall have allotted to the grantee the
frequencies and wave lengths to be used, and issued to the grantee a license for such case.
Thus, in the words of R.A. No. 2036 itself, approval of the then Secretary of Public Works and
Communications was a precondition before the petitioner could put up radio stations in areas where it
desires to operate.
The records of the case do not show any grant of authority from the then Secretary of Public Works and
Communications before the petitioner installed the questioned radio telephone services in San Jose,
Mindoro in 1971. The same is true as regards the radio telephone services opened in Sorsogon,
Sorsogon and Catarman, Samar in 1983. No certificate of public convenience and necessity appears to
have been secured by the petitioner from the public respondent when such certificate,was required by
the applicable public utility regulations.
The Constitution mandates that a franchise cannot be exclusive in nature nor can a franchise be granted
except that it must be subject to amendment, alteration, or even repeal by the legislature when the
common good so requires.

Maceda v. ERB 199 SCRA 454


Facts: petitioner Maceda seeks nullification of the Energy Regulatory Board (ERB) Orders dated
December 5 and 6, 1990 on the ground that the hearings conducted on the second provisional increase
in oil prices did not allow him substantial cross-examination, in effect, allegedly, a denial of due process.
Issue: W/N not allowing petitioner to Cross Examine is denial of due process
Held: No. What must be stressed is that while under Executive Order No. 172, a hearing is indispensable,
it does not preclude the Board from ordering, ex-parte, a provisional increase, as it did here, subject to
its final disposition of whether or not: (1) to make it permanent; (2) to reduce or increase it further; or
(3) to deny the application. Section 3, paragraph (e) is akin to a temporary restraining order or a writ of
preliminary attachment issued by the courts, which are given ex-parte and which are subject to the
resolution of the main case.
The order of testimony both with respect to the examination of the particular witness and to the general
course of the trial is within the discretion of the court and the exercise of this discretion in permitting to
be introduced out of the order prescribed by the rules is not improper (88 C.J.S. 206-207).Such a relaxed
procedure is especially true in administrative bodies, such as the ERB which in matters of rate or price
fixing is considered as exercising a quasi-legislative, not quasi-judicial, function As such administrative
agency, it is not bound by the strict or technical rules of evidence governing court proceedings (Sec. 29,
Public Service Act; Dickenson v. United States, 346, U.S. 389, 98 L. ed. 132, 74 S. St. 152). (Emphasis
supplied)

Globe Telecom c. NTC, 435 SCRA 110


Facts: On 4 June 1999, Smart filed a Complaint with NTC to interconnect Smart's and Globe's GSM
networks, particularly their respective SMS or texting services. The Complaint arose from the inability of
the two leading CMTS providers to effect interconnection. NTC also declared that both Smart and Globe
have been providing SMS without authority from it, in violation of Section 420 (f) of MC No. 8-9-95
which requires PTEs intending to provide value-added services (VAS) to secure prior approval from NTC
through an administrative process. directed the parties to secure the requisite authority to provide SMS
within thirty (30) days, subject to the payment of fine in the amount of two hundred pesos (P200.00)
"from the date of violation and for every day during which such violation continues."
Issue: W/N NTC may fine the petitioners for not securing the requisite authority.
Held: Yes. But in light that NTC did not provide Globe with due process it may not compel the said
company to pay the said fines. Globe and Smart were denied opportunity to present evidence on the
issues relating to the nature of VAS and the prior approval.
Another disturbing circumstance attending this petition is that until the promulgation of the assailed
Order Globe and Smart were never informed of the fact that their operation of SMS without prior
authority was at all an issue for consideration. As a result, neither Globe or Smart was afforded an
opportunity to present evidence in their behalf on that point.
the Court is convinced that prior to the promulgation of the assailed Order Globe was never notified that
its authority to operate SMS was put in issue. There is an established procedure within NTC that
provides for the steps that should be undertaken before an entity such as Globe could be subjected to a
disciplinary measure

2. Profession

Corona v. UHPAP 283 SCRA 31


FACTS: IN ISSUING ADMINISTRATIVE ORDER NO. 04-92 (PPA-AO NO. 04-92), LIMITING THE TERM OF
APPOINTMENT OF HARBOR PILOTS TO ONE YEAR SUBJECT TO YEARLY RENEWAL OR CANCELLATION.ON
AUGUST 12, 1992, RESPONDENTS UNITED HARBOUR PILOTS ASSOCIATION AND THE MANILA PILOTS
ASSOCIATION, THROUGH CAPT. ALBERTO C. COMPAS, QUESTIONED PPA-AO NO. 04-92. ON DECEMBER
23, 1992, THE OP ISSUED AN ORDER DIRECTING THE PPA TO HOLD IN ABEYANCE THE IMPLEMENTATION
OF PPA-AO NO. 04-92 ON MARCH 17, 1993, THE OP, THROUGH THEN ASSISTANT EXECUTIVE SECRETARY
FOR LEGAL AFFAIRS RENATO C. CORONA, DISMISSED THE APPEAL/PETITION AND LIFTED THE
RESTRAINING ORDER ISSUED EARLIER RESPONDENTS FILED A PETITION FOR CERTIORARI, PROHIBITION
AND INJUNCTION WITH PRAYER FOR THE ISSUANCE OF A TEMPORARY RESTRAINING ORDER AND
DAMAGES, BEFORE BRANCH 6 OF THE REGIONAL TRIAL COURT
ISSUE: WON PPA-AO-04-92 IS CONSTITUTIONAL
HELD: THE COURT IS CONVINCED THAT PPA-AO NO. 04-92 WAS ISSUED IN STARK DISREGARD OF
RESPONDENTS' RIGHT AGAINST DEPRIVATION OF PROPERTY WITHOUT DUE PROCESS OF LAW. THE
SUPREME COURT SAID THAT IN ORDER TO FALL WITHIN THE AEGIS OF THIS PROVISION, TWO
CONDITIONS MUST CONCUR, NAMELY, THAT THERE IS A DEPRIVATION AND THAT SUCH DEPRIVATION IS
DONE WITHOUT PROPER OBSERVANCE OF DUE PROCESS. AS A GENERAL RULE, NOTICE AND HEARING,
AS THE FUNDAMENTAL REQUIREMENTS OF PROCEDURAL DUE PROCESS, ARE ESSENTIAL ONLY WHEN
AN ADMINISTRATIVE BODY EXERCISES ITS QUASI-JUDICIAL FUNCTION. IN THE PERFORMANCE OF ITS
EXECUTIVE OR LEGISLATIVE FUNCTIONS, SUCH AS ISSUING RULES AND REGULATIONS, AN
ADMINISTRATIVE BODY NEED NOT COMPLY WITH THE REQUIREMENTS OF NOTICE AND HEARING
THERE IS NO DISPUTE THAT PILOTAGE AS A APROFESSION HAS TAKEN ON THE NATURE OF A PROPERTY
RIGHT. IT IS READILY APPARENT THAT PPA-AO NO. 04-92 UNDULY RESTRICTS THE RIGHT OF HARBOR
PILOTS TO ENJOY THEIR PROFESSION BEFORE THEIR COMPULSORY RETIREMENT

Dismissals, Suspension, Reinstatement etc….

1. Dismissals in Government Boards and Commissions

Abalos c. CSC 196 SCRA 81


Facts: Shortly after assuming office as Provincial Governor of Lanao del Norte, the petitioner, by
resolution of the provincial board, filed with the Tanodbayan a complaint for malversation against his
predecessor, Arsenio A. Quibranza, for having used for personal purposes several bulldozers belonging
to the province. Cited as witnesses were the operators of the bulldozers, private respondents Villabona
and Yap, who allegedly executed on September 6, 1986, affidavits in support of the complaint. These
sworn statements were disowned in a joint affidavit executed on September 25, 1986, by the private
respondents, who claimed that the earlier statements were spurious. The petitioner also invokes Section
40 of P.D. No. 807 and argues that in view of the private respondents' admission of the charge against
them, they could be summarily dismissed under this section.

Sec. 40. Summary Proceedings. — No formal investigation is necessary and the respondent may be
immediately removed or dismissed if any of the following circumstances is present:

(a) When the charge is serious and the evidence of guilt is strong.

Issue: W/N the said PD 807 may be used against the respondents.
Held: No. The Court had earlier entertained serious misgivings about the constitutionality of Section 40
as against strong protests that it was violative of due process in so far as it deprived the civil servant of
the right to defend himself against the ex parte decision to dismiss him. While it is true that this section
had been upheld in earlier decisions (albeit not very categorically), there was a growing sentiment that
the law should be re-examined more closely in deference to the right to a hearing that it was
foreclosing.
Fortunately, the question has been rendered moot and academic by the Congress of the Philippines,
which has itself seen fit to remove it from our statute books.

GSIS v. CSC 201 SCRA 661


Facts: This is a petition for certiorari to review the order of the Civil Service Commission(CSC) dated June
20, 1990 which directed the Government Service Insurance System(GSIS) to pay the compulsory heirs of
deceased Elizar Namuco and Eusebio Manuel for the period from the date of their illegal separation up
to the date of their demise. The Order dated November 22, 1990, however, denied herein petitioner’s
motion for reconsideration of CSC’s Order dated June 20, 1990. Deceased Elizar Namuco andEusebio
Manuel were illegally dismissed by the GSIS for allegedly being involved in irregularities in the canvass of
supplies and materials.
Issue: Whether or not the Civil Service Commission has the power to execute its judgments, final orders
or resolutions?
Whether or not the writ of execution issued on June 20, 1990 is void because it varies with the Court’s
Resolution of July 4, 1988?
Held:: The Civil Service Commission has the power to execute its judgment, final orders or resolutions.
The CSC is a constitutional commission invested by the Constitution and relevant laws not only with the
authority to administer the civil service but is also vested with quasi-judicial powers. It has the authority
to hear and decide administrative disciplinary cases instituted directly with it or brought to it on appeal.
The grant to a tribunal or agency of adjudicatory power or the authority to hear and adjudge cases,
normally and logically is deemed to include the grant of authority to enforce or execute the judgments it
thus renders unless the law otherwise provides. It is quite obvious that the authority to decide cases
would be inutile unless accompanied by the authority to see that what has been decided is carried out.
The writ of execution issued on June 20, 1990 is valid. The Court upholds the same, simply because
there is no fair and feasible alternative in the circumstances. The binding force of Resolution of July 4,
1988, for all intents and purposes, is that it makes exoneration in the administrative proceedings a
condition precedent to payment of said back salaries, it cannot however exact an impossible
performance or decree a useless exercise such as that the subsequent disciplinary proceedings is an
empty, and inutile procedure as to the deceased employees, they cannot possibly be bound by any
substantiation in the said proceedings of the abovementioned charges.

Macayayong v. Ople 204 SCRA 372


Facts: Petitioner Macayayong had been detailed with the various offices in the Office of the President
(the last one being the Board of Liquidators) from 1968 up to the time he was dropped from the roster
of the Department of Labor effective January 31, 1976. The Undersecretary of Labor terminated the
detail of the petitioner with the Malacañang Complaint and Action Unit effective September 26, 1974.
The Chairman, Board of Liquidators, Office of the President, requested extension of petitioner's detail
with said office until after the cases he was handling are terminated. Notwithstanding the request, the
Secretary of Labor in a telegram manifested that the detail of Atty. Macayayong was not sanctioned by
his Office, therefore, the recall order still stood with a warning that unless he (Macayayong) reported to
his home office within 72 hours from notice, he would be dropped from the rolls of personnel. In
obedience thereto, Macayayong reported to the Bureau of Labor Standards on January 6, 1975 and
rendered service therein up to March 11, 1975, when he applied for a leave of absence.
Without the knowledge of the Secretary of labor he gain reported with the Office of the President,
hence he was told to report to the Secretary of labor within 5 days else he will be dismissed which he
did not comply with.
Issue: W/N Petitioner was denied due process
Held: No. The records show that before petitioner was dropped from the roster of the Department of
Labor, he was notified twice by his home office to report back to work, on January 5 and on January 19,
1976. In the aforesaid notices, the petitioner was urged to come back to his home office because his
services were badly needed, with a warning that in case he (petitioner) fails to report, he would be
dropped from the roster of the Department of Labor.
In the case at bar, petitioner was given ample opportunity to be heard and to present his case. Thus, as
previously stated, he appealed his case to the Civil Service Commission and then to the Office of the
President. In both appeals, he lost.
This Court had ruled that a motion for reconsideration or appeal is curative in character on the issue of
alleged denial of due process by way of lack of hearing of the case

Gonzales v. CSC 226 SCRA 66


Facts: Petitioner Jesus R. Gonzales was one of the two Utility Workers II assigned at the Pharmacy
Section of respondent Philippine Children’s Medical Center (PCMC). On March 2, 1998, petitioner
started absenting himself without an approved leave (AWOL) and without explaining the reason for his
absence to his superiors. In view of the exigency of petitioner’s functions, Ms. Jara Corazon O. Ehera,
Human Resources Management Officer III, wrote a letter-notice dated March 5, 1998 to petitioner
directing him to report for work within three (3) days from receipt of said notice, otherwise, he would
be dropped from the rolls. Petitioner now avers that he received PCMC’s letter dated March 5, 1998
only on March 20, 1998 and thus, had until March 23, 1998 within which to comply with the directive;
that in fact, he did report for work on March 21, 1998, but was barred by security personnel from
entering the company because his name had already been dropped from the rolls effective March 20,
1998. He contends that his non-compliance with the return to work directive does not constitute
abandonment of work as no person in his right mind would abandon his job to his own detriment.
Issue: W/N petitioner was denied due process.
Held: No. Petitioner’s assertion that he was denied due process is untenable. The essence of due
process is simply an opportunity to be heard or as applied to administrative proceedings, an opportunity
to explain one’s side or opportunity to seek a reconsideration of the action or ruling complained of.

Records would show that respondent PCMC had written a letter to petitioner to report for work and
another letter informing him that he was being dropped from the rolls, due to his Absence Without
Official Leave, with the enumeration of the reasons and basis thereof. That petitioner only received the
notice on March 20, 1998 is of no moment. In fact, two notices were given him: (a) the notice requiring
him to report for work; and (b) the notice that he would be dropped because of his absences without
official leave. Further, petitioner was given sufficient opportunity to report for duty after he received
the return-to-work order, but he did not report. Thus, the CSC found no error when respondent PCMC
dropped petitioner from the rolls for his refusal to comply with the return-to-work order within a
prescribed period.

Go. V. NPC 271 SCRA 447


Facts: Petitioner Edgar M. Go had been a member of the Olongapo City Police Department since April
18, 1974. On December 16, 1983, he was dismissed for alleged involvement in illegal gambling, more
particularly the operation of jai-alai bookies. Petitioner claims that he was not served written charges
and informed of the nature of such charges; that no hearing had actually been held by the summary
dismissal board; and that at any rate he was not heard.
Issue: W/N petitioner was denied due process.
Held: No. Petitioner might have been told what the charge or charges against him were, but not the
details thereof, and, certainly, not what the alleged witnesses against him might have said because, as
already stated, the record of the INP simply did not contain their alleged testimonies.
Nor does it appear that petitioner was heard in his defense. His claim, that thrice he appeared before
the summary dismissal board but no hearing was ever held either because complainant and his
witnesses did not appear or the members of the board were absent or both complainant and witnesses
and members of the board were absent, was never specifically denied in any of the decisions of the
administrative authorities, beyond saying that the claim was belied by the record. There is simply
nothing in the INP record of the case to show this. It may be argued that the requirements of due
process are satisfied if a party initially denied a hearing is subsequently granted one by means of motion
for reconsideration. That is true indeed if the charges and the evidence against him are set forth in the
record of the case, but not where, as here, they are not. If in his appeal to the PC/INP Director General,
petitioner presented the affidavits of retraction of two of the witnesses against him, it was only because
the decision of the board mentioned that these witnesses allegedly said they had acted as petitioner's
collectors and not because their prior statements were in the record.

CHR v. CSC 227 SCRA 42-


Facts: Atty. Elias Pacete, a permanent appointee since February 1, 1988 to the position of Division Chief
of Region IX of the Commission of Human Rights (CHR), filed an application for optional retirement
pursuant to Rep. Act No. 1616, 1 dated June 17, 1989 effective July 31, 1989 because of failing eyesight.
On July 7, 1989, however, Pacete sent a notice of withdrawal of his application for retirement. On July
17, 1989, Pacete was informed by the Chairman of the CHR through a telegram of the acceptance and
approval of his application for optional retirement effective July 31, 1989 and the appointment of Atty.
Rodrigo Roy as his successor effective August 1, 1989. Pacete requested the CHR that he be reinstated
to his former position with back wages and allowances and the recall of the appointment of his
successor, Atty. Roy. On October 18, 1989, the CHR through a resolution, denied his request and instead
formally charged him with incompetence, gross inefficiency in the performance of official duty and
failure to account for public funds.
Issue: W/N the respondent Pacete was accorded due process.
Held: No. Consequently, since the resolution denying reinstatement was issued without conforming to
the requirements of due notice and hearing, private respondent's dismissal from service was illegal. It
constituted a blatant violation of Section 46 of the Administrative Code of 1987 and Section 36 of Pres.
Decree No. 807 which provides that "No officer or employee in the Civil Service shall be suspended or
dismissed except for cause as provided by law and after due process," not to speak of a similar provision
under Article IX B, Sec. 2 (3) of the 1987 Constitution on the Civil Service Commission. If petitioner
wishes to dismiss private respondent for cause, the latter must be allowed to return to his previous
position so that he may avail himself of the opportunity to refute the charges imputed to him. It should
be pointed out, however, that he may not be able to fulfill the three-year requirement if the CHR
succeeds in proving the charges of incompetence and inefficiency through a hearing as laid down in
paragraph (d) of Section 12 of CA 186.

Uy v. COA 328 SCRA 607 - JM

Facts: Petitioners were among the more than sixty permanent employees of the Provincial Engineering
Office, who were dismissed from the service by then Governor Ceferino S. Paredes, Jr. when the latter
assumed office, allegedly to scale down the operations of the said office. The MSPB issued an Order
which directed the Provincial Government of Agusan del Sur pay petitioners their back salaries and
other money benefits for the period that they had been out of the service until their reinstatement. COA
disallowed.

Issue: Due process.

Ruling: The fundamental requirements of procedural due process cannot be violated in proceedings
before the COA. In the case at bar, former Governor Paredes was never made a party to nor served a
notice of the proceedings before the COA. While administrative agencies exercising quasi-judicial
powers are not hide bound by technical procedures, nonetheless, they are not free to disregard the
basic demands of due process. Notice to enable the other party to be heard and to present evidence is
not a mere technicality or a trivial matter in any administrative proceedings but an indispensable
ingredient of due process. It would be unfair for COA to hold former Governor Paredes personally liable
for the claims of petitioners amounting to millions of pesos without giving him an opportunity to be
heard and present evidence in his defense.

Lameyra v. Pangilinan 322 SCRA 117- JM

Facts: Petitioner received a letter from the mayor informing him that he is dropped from the roll of
employees of the local government unit. Petitioner claims that he was terminated without prior written
notice of the charges and without investigation and hearing.
Issue: Due process.

Held: Although it is clear from the Civil Service Memorandum Circular that no prior notice is required to
drop from the rolls an employee who has been continuously absent without leave for at least thirty
days, petitioner contests the finding that he was absent at all. He claims that he reported for work but
was prevented from signing the log book. In view of the circumstances prevailing in this case, the Civil
Service Commission should have considered the new evidence annexed by petitioner to his motion for
reconsideration.

NPC v. Zozobrado, 487 SCRA 16- JM

Facts: Respondent Agustin A. Zozobrado, a permanent employee of petitioner National Power


Corporation (NPC) assigned as Pilot in the aviation group, received a letter from NPC President
informing him that that he was being dropped from the rolls. Respondent Zozobrado filed an appeal
before the CSC questioning NPC’s implementation of dropping him from the rolls. CSC issued a
Resolution dismissing petitioner’s appeal. Respondent Zozobrado filed a Motion for Reconsideration of
the said Resolution, which the CSC denied in another Resolution.
Issue: Due process.

Held: In the instant case, the notice required by law was not given to the petitioner. He was not given
notice after the rating of unsatisfactory during the first semester of 1997 within thirty (30) days
therefrom. The alleged verbal notice, to our mind, is not sufficient for the reason that it is easily
concocted. And when there are conflicting allegations as to the alleged verbal notice, such that there is
a clash between the word of a superior officer and that of a subordinate, the latter is usually at a
disadvantage. Hence, a verbal notice cannot be considered as substantial compliance with the Civil
Service Rules.

PAGCOR v. CA, GR 185668, December 13, 2011- JM

Facts: Private respondent Manahan was a Treasury Officer of petitioner Philippine Amusement and
Gaming Corporation (PAGCOR) assigned in Casino Filipino-Manila Pavilion (CF-Pavilion). Among her
functions as Treasury Officer was the handling of fund transfer requests received by CF-Pavilion and the
supervision of the office's Vault-in-Charge and Senior Cashier. A notice of preventive suspension dated
April 15, 2004 and signed by Dan N. Dia, Senior Branch Manager of CF-Pavilion, was received by
.Manahan on April 16, 2004. The pertinent portions of the notice read:

You are hereby informed of the charge against you of SERIOUS PROCEDURAL
DEVIATION/GROSS NEGLIGENCE, arising from the anomalous fund transfer transaction
in the amount of [P]4.2 million, consummated at the VIP Booth last April 14, 2004
wherein you were on the 6-2PM duty.

Issue: Due process.

Ruling: Evidently, the petitioner failed to substantially comply with the requisite formal charge, as well
as with the other requirements under CSC Resolution No. 99-1936 concerning the procedure for the
conduct of an administrative investigation. What PAGCOR claims to be the formal charge it issued in
compliance with the CSC rules was the memorandum addressed to Manahan under the subject
“Preventive Suspension,” which was issued by CF-Pavilion's Senior Branch.
2. Dismissals in Private Sector

Hellinic v. Siete 195 SCRA 179- JM

Facts: Siete was employed as Master of M/V Houda G by Sultan Shipping Co., Ltd., through its crewing
agent, herein petitioner. After considering the position papers and documentary evidence of the parties,
Administrator Tomas D. Achacoso of the Philippine Overseas Employment Administration (POEA)
dismissed the complaint, holding that there was valid cause for Siete's removal. The decision placed
much value on the various communications presented by the petitioner to show that Siete was indeed
guilty of the charges that justified his separation.

Issue: Due process.

Ruling: It is not correct to say that managerial employees may be arbitrarily dismissed, at any time and
without cause as established in an appropriate investigation. Managerial employees, no less than rank-
and-file laborers, are entitled to due process. Loss of confidence, which is the usual ground for the
removal of the managerial employee, must be established like any other lawful cause. Even if it be
assumed that Siete was a managerial employee — an issue which, incidentally, was not earlier raised or
resolved — the petitioner has not satisfactorily proved the reason for its supposed loss of confidence in
him.

Salaw v. NLRC 202 SCRA 7 – JM

Facts: Herein petitioner was employed by the private respondents and was terminated. That
prior to petitioner’s termination during the extraction of sworn statement from the latter he wasn’t
assisted by his counsel and thus during the meeting with the PDIC private
respondent told petitioner to appear before the said committee without his counsel.

Issue: W/N dismissal of the petitioner by the private respondents was in accordance with due process.

Held: No. Investigating committee violated petitioners constitutional right to due process as much as he
was not given a chance to defend himself by a competent and independent counsel which is a very basic
requirement of substantive due process and this right can’t be waived except in writing however the
waiver must be in the presence of a counsel.

Conti v. NLRC, GR 119253 April 10, 1997- JM

Facts: Petitioner Amor Conti was employed by respondent Corfarm as cashier on. Petitioner Leopoldo
Cruz was employed by the same respondent corporation as a warehouseman. Petitioners received a
memorandum from private respondents terminating their services effective on said date, allegedly for
two reasons: 1) the expiration of their employment contracts, these being coterminous with the
management contract between Corfarm and MERALCO, and; 2) the on-going evaluation of their past
performances, and investigation of the internal auditor of Corfarm of certain anomalous transactions
involving them (petitioners).
Issue: Due process.
Held: This Court has consistently held that the twin requirements of notice and hearing constitute
essential elements of due process in the dismissal of employees. As to the requirement of notice, it has
been held that the employer must furnish the worker with two written notices before termination of
employment can be legally effected: (a) notice which apprises the employee of the particular acts or
omissions for which his dismissal is sought, and; (b) subsequent notice which informs the employee of
the employer's decision to dismiss him.
With regard to the requirement of a hearing, this Court has held that the essence of due process is
simply an opportunity to be heard and not that an actual hearing should always and indispensably be
held. In the case at bar, neither notice nor hearing was afforded the petitioners.

Aparente v. NLRC, GR 117652 - JM

Facts: Aparente was employed by Coca Cola as assistant mechanic. He drove the company’s truck to
install a panel sign and accidentally sideswiped a ten year old girl whose injuries incurred hospitalization
expenses of up to P19,534.45. Such amount was not reimbursed by insurance as Aparente had no
driver’s license at the time of the accident; therefore Coca Cola shouldered the expenses. Coca Cola
conducted an investigation where Aparente was given the opportunity to defend himself. He was then
dismissed for violating the company rules and regulation for blatant disregard of established control
procedures resulting in company damages.

Issue: Due process.

Ruling: Although Aparente contends that he was investigated simply for the offense of driving without a
valid driver’s license, it was clear that he was fully aware that he was being investigated for his
involvement in the vehicular accident. It was also known to him that the accident caused the victim to
suffer serious injuries leading to expenses which the insurance refused to cover. Due process does not
necessarily require a hearing, as long as one is given reasonable opportunity to be heard

Lopez v. Alturas 647 SCRA 566- JM

Facts: Quirico Lopez (petitioner) was hired by Alturas Group of Companies (respondent) as truck driver.
Ten years later, he was dismissed after he was allegedly caught by respondent’s security guard in the act
of attempting to smuggle out of the company premises 60 kilos of scrap iron aboard respondent’s cargo
van that was assigned to him.

Issue: Due process.

Ruling: This Court has held that there is no violation of due process even if no hearing was
conducted, where the party was given a chance to explain his side of the controversy. What is
frowned upon is the denial of the opportunity to be heard.

3. Preventive Suspension
Alonzo v. Capulong 244 SCRA 80 - JM

Facts: Fajardo was preventively suspended from her post at the Pag-ibig Fund Foundation. The decision
for her preventive suspension was based on a recommendation by Pag-ibig’s legal department, which
found a prima facie case after investigating the circumstances surrounding a letter sent to the CEO of
Pag-ibig by a contractor complaining of improper conduct on Fajardo’s part. Fajardo claims she was
deprived of due process for being suspended on the basis of an unverified letter and not being allowed
to give her side.
Issue: Due process.
Held: It is now settled that the preventive suspension of a civil service employee or officer can be
ordered even without a hearing because such suspension is not a penalty but only a preliminary step in
an administrative investigation. The purpose is to prevent the accused from using his position or office
to influence prospective witnesses or tamper with the records which may be vital in the prosecution of
the case against him.

Castillio – Co v. Barbers 290 SCRA 717- JM


Facts: Governor Castillo-Co and Provincial Engineer Ringor were placed under preventive suspension for
a period of six (6) months. Petitioner questions the manner by which the suspension order was
issued. She claims that she was denied due process because she was not afforded the opportunity to
controvert the evidence against her before the order of preventive suspension was issued
Ruling: A preventive suspension can be decreed on an official under investigation after charges are
brought and even before the charges are heard since the same is not in the nature of a penalty but
merely a preliminary step in an administrative investigation. In connection with the suspension of
petitioner before he could file his answer to the administrative complaint, suffice it to say that the
suspension was not a punishment or penalty for the acts of dishonesty and misconduct in office, but
only as a preventive measure. Suspension is a preliminary step in an administrative investigation. If
after such investigation, the charges are established and the person investigated is found guilty of acts
warranting his removal, then he is removed or dismissed. This is the penalty

Bacsasar v. CSC 576 SCRa 787 – JM

Facts: Petitioner was charged with dishonesty by the Civil Service Commission-Autonomous Region in
Muslim Mindanao (CSC-ARMM). Petitioner asserts denial of due process because her case was decided
without a formal investigation. She claims that she was denied opportunity to present evidence, to
confront the witnesses against her, and to object to the evidence adduced against her.

Ruling: Due process in the administrative context does not require trial-type proceedings similar to
those in the courts of justice. A formal trial-type hearing is not at all times and in all instances essential
to due process. What is simply required is that the party concerned is given due notice and is afforded
an opportunity or right to be heard. It is enough that the parties are given a fair and reasonable
opportunity to explain their respective sides of the controversy and to present evidence on which a fair
decision can be made. Records show that petitioner answered the charges against her. She even
interposed an appeal from the decision of the CSC-ARMM to the CSC, and then to the CA. Clearly, she
was afforded an opportunity to be heard through her pleadings; hence, her right to due process was not
impaired.

Carabeo v. CA 607 SCRA 390- JM

Facts: The Department of Finance-Revenue Integrity Protection Service (DOF-RIPS), composed of private
respondents Troy Francis Pizarro, Joel Apolonio, Reynalito L. Lazaro, Ismael Leonor, and Melchor Piol,
filed a complaint with the Office of the Ombudsman against Carabeo, Officer-in-Charge (OIC) of the
Office of the Treasurer of Parañaque City. Carabeo contends that there must be prior notice and hearing
before the Ombudsman may issue a preventive suspension order.

Ruling: In connection with the suspension of petitioner before he could file his answer to the
administrative complaint, suffice it to say that the suspension was not a punishment or penalty
for the acts of dishonesty and misconduct in office, but only as a preventive measure.
Suspension is a preliminary step in an administrative investigation. If after such investigation,
the charges are established and the person investigated is found guilty of acts warranting his
removal, then he is removed or dismissed. This is the penalty. There is, therefore, nothing
improper in suspending an officer pending his investigation and before the charges against him
are heard and be given an opportunity to prove his innocence.

Ordinance/Status/Memo Cir/Rules

People v. Nazario 165 SCRA 136- JM

Facts: Eusebio Nazario was charged in violation of refusal and failure to pay his municipal taxes
amounting to Php 362.62 because of his fishpond operation provided under Ordinance 4, Series of 1955,
as amended. He is a resident of Sta. Mesa Manila and just leases a fishpond located at Pagbilao, Quezon
with the Philippine Fisheries Commission. The years in question of failure to pay was for 1964, 1965, and
1966. Nazario did not pay because he was not sure if he was covered under the ordinance. He was found
guilty thus this petition.

Ruling: As a rule, a statute or act may be said to be vague when it lacks comprehensible standards that
men "of common intelligence must necessarily guess at its meaning and differ as to its application." 11 It
is repugnant to the Constitution in two respects: (1) it violates due process for failure to accord persons,
especially the parties targetted by it, fair notice of the conduct to avoid; and (2) it leaves law enforcers
unbridled discretion in carrying out its provisions and becomes an arbitrary flexing of the Government
muscle.

Franscisco v. CA 199 SCRA 595- JM

Facts: Basilan Municipal Mayor Benjamin Valencia summarily ordered the demolition of an antiquated
and dilapidated Quonset warehouse situated in Port Area, Strong Boulevard, Isabela, Basilan, outside
the zone for warehouses. The legal possessor of the Quonset sought the prohibition of the Order but
was denied by the RTC.

Issue: Whether or not Respondent Mayor could summarily and extra-judicially order the demolition
of petitioner's Quonset building.
Ruling: NO. Ordinance No. 147 relied upon by Respondents should not be interpreted as authorizing the
summary removal of a non-conforming building by the municipal government. For if it does, it must be
struck down for being in contravention of the requirements of due process, as originally held by the
Court of Appeals. Moreover, the enforcement and administration of the provisions of the Ordinance
resides with the Zoning Administrator. It is said official who may call upon the City Fiscal to institute the
necessary legal proceedings to enforce the provisions of the Ordinance.

Misamis Or. V. DOF 238 SCRA 63- JM


Facts: Petitioner Misamis Oriental Association of Coco Traders, Inc. is a domestic corporation whose
members, individually or collectively, are engaged in the buying and selling of copra in Misamis Oriental.
The petitioner alleges that prior to the issuance Revenue Memorandum Circular 47-91 on June 11, 1991,
which implemented VAT Ruling 190-90, copra was classified as agricultural food product under $ 103(b)
of the National Internal Revenue Code and, therefore, exempt from VAT at all stages of production or
distribution. The reclassification had the effect of denying to the petitioner the exemption it previously
enjoyed when copra was classified as an agricultural food product under §103(b) of the NIRC. Petitioner
challenges RMC No. 47-91 on various grounds.

Issue: Whether RMC No. 47-91 is discriminatory and violative of theequal protection clause of the
Constitution.

Ruling: The court ruled in the negative. Petitioner likewise claims that RMC No. 47-91 is violative of
the equal protection clause because while coconut farmers and copra producers are
exempt, traders and dealers are not, although both sell copra in its original state. Petitioners add that oil
millers do not enjoy tax credit out of the VAT payment of tradersand dealers. The argument has
no merit. There is a material or substantial difference between coconut farmers and copra producers, on
the one hand, and copra traders and dealers, on the other. The former produce and sell copra, the latter
merely sell copra. The Constitution does not forbid the differential treatment of persons so long as there
is a reasonable basis for classifying them differently. It is not true that oil millers are exempt from VAT.
Pursuant to § 102 of the NIRC, they are subject to 10% VAT on the sale of services.

Estrada v. Sandiganbayan GR 148560 Nov. 19, 2001- JM

Facts: Petitioner calls for the Court to subject RA 7080(An Act defining and Penalizing the Crime
of Plunder) to the crucible of constitutionality for reasons that the act is vague. dispenses with the
"reasonable doubt" standard in criminal prosecutions abolishes the element of mens rea in crimes
punishable under the Revised Penal Code

Issue: Is the Plunder Law unconstitutional for being vague?


Does the Plunder law require less evidence proving the predicate crimes of plunder thus violates the
rights of the accused to due process.

Held: Petitioner has miserably failed to show that the Plunder Law is unconstitutional due to its
vagueness.-"...the Plunder Law contains ascertainable standards and well-defined
parameterswhich would enable the accused to determine the nature of his violation. Section 2 is
sufficiently explicit in its description of the acts, conduct and conditions required or forbidden, and
prescribes the elements of the crime with reasonable certainty and particularity."
Motion for Reconsideration

Mendenilla v. CSC 194 SCRA 278- JM

Facts: Petitioner Ardeliza Medenilla was a contractual employee of the Department of Public Works and
Highways (DPWH) occupying the position of Public Relations Officer II. Respondents, all of whom are
employees in the Human Resource Training and Material Development Division, Administrative and
Manpower Management Service of the DPWH, jointly lodged a protest before the DPWH task force on
reorganization contesting the appointment of the petitioner to the position. The task force on
reorganization dismissed the protest. Commission promulgated the assailed resolution disapproving the
promotional appointment. Petitioner filed a motion for reconsideration of the resolution.

Ruling: The essence of due process is the opportunity to be heard. The presence of a party is not always
the cornerstone of due process. What the law prohibits is not the absence of previous notice but the
absolute absence thereof and lack of opportunity to be heard. In the case at bar, any defect was cured
by the filing of a motion for reconsideration.

B. Hot Pursuit

Two Requisites:
1. An offense had just been committed.
2. The person making the arrest has probable cause to believe, based on his personal knowledge of facts
and circumstances, that the person to be arrested committed it.
*There must be immediacy between the time the offense is committed and the time of the arrest.

Mendiola vs CSC 221 SCRA 295 (Note: It’s not Mendenilla vs CSC its Mendiola)

Facts: Petitioner was an employee of the Bureau (this is weird because the full case never mentioned
what bureau hehehe meron lang mga terms like by the bureau or by a bureau agent hehehehe although
the Dept of Finance was mentioned). Due to the order Pres. Corazon Aquino involving reorganization,
petitioner was terminated. He appealed to the CSC and averred that he was denied due process when
he was dismissed from the service. Petitioner contested his being part of those personnel who should be
terminated because he had been commended and satisfactorily rated for his performance. On
September 21, 1988, the Commission resolved the case in petitioner's favor; however, there was a
motion for reconsideration filed by the Bureau which was given due course by the CSC.

Issue: WON there was denial of due process against Mendiola

Ruling: With respect to petitioner's contention that he was denied due process when the Commission
heard the Bureau's motion for reconsideration without notice to him, We agree with respondent
Bureau's argument that the defect was cured by the filing by petitioner of his Omnibus Motion on July
30, 1990. Thus, in Medenilla v. Civil Service Commission, We said that the lack of notice to petitioner
regarding the pending appeal and the hearing of said appeal was cured by the filing of a motion for
reconsideration. Denial of due process cannot be successfully invoked where a party was given the
chance to be heard on his motion for reconsideration.
Rodreguez v. Proj. 6 247 SCRA 528- Mac

G.R. No. 79968 August 23, 1995

PETER RODRIGUEZ, petitioner,


vs.
PROJECT 6 MARKET SERVICE COOPERATIVE, INC. and HON. MAXIMIANO C. ASUNCION, JUDGE,
REGIONAL TRIAL COURT, BRANCH 104, QUEZON CITY, respondents.

Facts: Private respondent Cooperative sought to eject petitioner from the stall which the petitioner is
renting. TRC rendered judgment on Aug 31, 1987 denying the motion to quash of petitioner. Petitioner
alleged denial of due process since the decision was done without any hearing and without him having
been furnished a copy of the Cooperative's opposition to his motion to quash
Issue: WON there was due process
Ruling: Yes, because after the denial of the motion to quash, petitioner raised a motion for
reconsideration of the said order. He received his copy of the opposition and respondent judge
conducted a hearing on his motion for reconsideration. Indeed, deprivation of due process cannot be
successfully invoked where a party was given the chance to be heard on his motion for reconsideration.

Lazo v. CSC 236 SCRA 469 – Mac


Lazo boasted that he bought his civil service eligibility; hence, an investigation was made wherein his
exam papers were rechecked. It was revealed that his real score is 34% and not 76%. His civil service
elibility was revoked. He alleged that he was denied due process since he not given opportunity to look
at his own test papers and examine the findings that he really did get only 34%. There was also no due
notice and hearing. This was dismissed; hence, he filed a motion for reconsideration
ISSUE: WON the motion for reconsideration cured any defect in the process of rechecking
Ruling: Yes. the motion for reconsideration remedied whatever defect there might have been in
rechecking the examination papers of petitioner without his presence. 3 Petitioner was given the right to
be heard, but, as already said, he did not make good use of it by showing that his actual score was
76.46%, and not 34.48%

Salonga v. CA 269 SCRA 534- Mac


Private respondent sought to enjoin petitioner from continuing the operation of Club Ibiza. Petitioner
never attended trial so he the judge found him guilty of indirect contempt of court.
Issue: WON petitioner was denied due process
Ruling: No. The fact that petitioners and their counsel Garlitos failed to attend said hearing and adduce
evidence on their behalf is of no moment. What is important is that they were given the chance to do so
(when they filed a motion for reconsideration which was granted but they simply did not attend the
hearings). "Indeed, deprivation of due process cannot be successfully invoked where a party was given
the chance to be heard in his motion for reconsideration."

Bernardo v. CA 275 SCRA 413 – Mac


Facts: Private respondent Jimmy Tomas filed a complaint for recovery of possess and and quieting of
title against petitioner. Petitioner lost because his lawyer died and so he was not able to present his
case. CA agreed with trial court and did not allow for a new trial; hence, this case.
Issue: Did the trial court deny due process to the petitioner by its refusal to grant new trial and/or to
reopen the case in spite of the fact that the defendant was unable to participate and to present his
evidence due to the death of the handling lawyer of the law firm representing him and the failure of the
new attorney to follow the rules on substitution of counsel?

Ruling: A thorough review of the instant case reveals that the negligence of the law firm engaged by the
petitioner to defend his cause, and the error of his new counsel in giving a defective substitution and
notice of the death of his former counsel, did not result in deprivation of due process to said party.
Hence, a nullification of the Respondent Court’s Amended Decision grounded on grave abuse of
discretion is not warranted. Under Section 26, Rule 138 of the Rules of Court and established
jurisprudence, a valid substitution of counsel has the following requirements: (1) the filing of a written
application for substitution; (2) the client’s written consent; (3) the consent of the substituted lawyer if
such consent can be obtained; and, in case such written consent cannot be procured, (4) a proof of
service of notice of such motion on the attorney to be substituted in the manner required by the Rules.
Where death of the previous attorney is the cause of substitution of the counsel, a verified proof of the
death of such attorney (usually a death certificate) must accompany the notice of appearance of the
new counsel.

Clearly, petitioner failed to comply with the above requirements. His new counsel’s notice of
appearance merely mentioned that Atty. Jose B. Puerto “recently died.” A verified certificate of death
was not attached thereto. It has been held that courts may not presume that the counsel of record has
been substituted by a second counsel merely from the filing of a formal appearance by the latter.

The party-litigant should not also rely totally on his counsel to litigate his case even if the latter expressly
assures that the former’s presence in court will no longer be needed. No prudent party will leave the
fate of his case entirely to his lawyer. Absence in one or two hearings may be negligible but want of
inquiry or update on the status of his case for several months (four, in this case) is inexcusable.

Casuela v. Ombudsman 276 SCRA 635 – Mac


Petitioner (POEA official) was suspended by Ombudsman for 3 months because he received money in
processing certain applications. He filed a motion for recon on the ground of denial of due process but
this was denied
Issue: WON there was denial of due process
Ruling: No. Petitioner’s claim that ‘the Ombudsman virtually deprived [him] of his right to due process
by failing to inform [him] of the charges against him’ x x x is bereft of merit. In the first place, as
petitioner admits he had raised the alleged '‘lack of sufficient opportunity to be informed of the charges
against him in a motion for reconsideration.’ This Honorable Court has repeatedly stressed that the
requirements of due process are complied [with] when a party is heard on a motion for reconsideration.

Cordenillio v. Executive Secretary 276 SCRA 652 – Mac

Facts: Private respondent Bolivar was given a Nipa Bacauan permit by Bureau of Forest Development
and later on petitioner Cordenillo applied for a Miscellaneous Sales Application basically for the purpose
of constructing a fishpond. The area also included those granted to Bolivar and to another person.
Cordenillo already finished his fishpond construction but he was made to vacate because that area
belong to private respondent Bolivar. Motion for reconsideration denied
WON Due process denied
Ruling: No. petitioner pretends to be a victim of due process violation because he was not afforded the
opportunity to be heard vis-à-vis private respondent Bolivar’s “EX-PARTE MOTION (For Issuance of the
Writ of Implementation for the forced evacuation of cordenillo I think).” Suffice it to say that the mere
fact that petitioner assails two Resolution of the Office of the President, the one with the later date of
which was issued precisely upon petitioner’s filing of a Motion for Reconsideration of the Resolution
first issued on dated May 7, 1993, shows that petitioner was in fact heard, for purposes of
Administrative due process, when he filed said Motion for Reconsideration. As such, any contention of
denial of due process must fail as the same was cured by the filing of the Motion for Reconsideration
As far back as 1935, it has already been settled doctrine that a plea of denial of procedural due process
does not lie where a defect consisting of an absence of notice of hearing was thereafter cured by the
alleged aggrieved party having had the opportunity to be heard on a motion for reconsideration. What
the law prohibits is not the absence of previous notice, but the absolute absence thereof and lack of
opportunity to be heard. There is then no occasion to impute deprivation of property without due
process where the adverse party was heard on a motion for reconsideration constituting as it does
sufficient opportunity for him to inform the Tribunal concerned of his side of the controversy. x x x
[W]hat due process contemplates is freedom from arbitrariness and what it requires is fairness or
justice, the substance rather than the form being paramount, the conclusion being that the hearing on a
motion for reconsideration meets the strict requirement of the process.’

Chua v. CA 287 SCRA 33 (this is actually Vda. de chua vs CA)

Facts: Petitioner asserts that “the appointed commissioners failed to conduct a hearing to give
the parties the opportunity to present their respective evidence. According to [petitioner], the
Commissioners Valuation Report was submitted on October 8, 1999 in violation of the
appellant’s right to due process as it was deprived of the opportunity to present evidence on the
determination of the just compensation.”

Issue: WON due process was denied

Ruling: The filing by [petitioner] of a motion for reconsideration accorded it ample opportunity
to dispute the findings of the commissioners, so that [petitioner] was as fully heard as there
might have been hearing actually taken place. “Denial of due process cannot be successfully
invoked by a party who has had the opportunity to be heard on his motion for reconsideration.”
De la Cruz v. Abelle 352 SCRA 691- Mac
De la Cruz v. Abelle 352 SCRA 691 (Dapat Vda. De Dela Cruz vs Abille)

Facts: Petitioners filed a motion for reconsideration praying that another Order be issued declaring as
null and void the Order dated April 19, 1989 (which is about the transfer of certain properties to some of
the tenants under DAR), which was issued allegedly without giving them a day in court, hence, there was
absence of due process of law. The said motion for reconsideration was treated as an appeal and
elevated to the Secretary of the Department of Agrarian Reform who rendered a Decision on June 20,
1994, the dispositive portion of which reads:

WHEREFORE, premises considered, Order is hereby issued dismissing the instant motion for lack of merit
and the Order dated October 21, 1992 is hereby affirmed.

Petitioners’ motion for reconsideration of the said Decision of the Secretary of DAR having been denied,
they filed a petition for review with the Court of Appeals. However, the Court of Appeals also dismissed
the said petition for review. Their motion for reconsideration was also denied.
Issue: WON due process was denied
Ruling: No. We agree with the Court of Appeals that although the petitioners were not given the
opportunity to be heard when Regional Director Antonio Nuesa in his Order dated April 19, 1989
ordered the cancellation of Certificate of Land Transfer No. 0-064711 on the retained area, nevertheless,
in their petition for issuance of an emancipation patent, petitioners were given the opportunity to be
heard as they raised in issue the validity of the cancellation of the said CLT, which was resolved by DAR
Regional Director Eligio P. Pacis in his Order dated October 21, 1992, and also in their (petitioners’)
motion for reconsideration, which was treated as an appeal by the Secretary of Agrarian Reform and
resolved in his Order dated June 20, 1994. The essence of due process is simply an opportunity to be
heard or, as applied to administrative proceedings, an opportunity to seek a reconsideration of the
action or ruling complained of.

Rodreguez v. CA GR 134275 August 7, 2002 – Mac


Rodriguez vs CA GR 134278 Aug 7, 2002 (NOT GR 134275)
Facts: Petitioner was one of those kotong cops arrested. He was summarily dismissed by PNP Chief
despite the existence of an admin case pending involving the same incident; hence, there was denial of
due process. He raised the issue before CA. CA denied motion for recon denied.
Issue: WON there was denial of due process

Ruling: No. Petitioner cannot now claim that he was not afforded due process by the NAPOLCOM. In
administrative proceedings, the filing of charges and giving reasonable opportunity for the person so
charged to answer the accusations against him constitute the minimum requirements of due process.
The essence of administrative due process is the opportunity to be heard. As long as a party was given
the opportunity to defend his interests in due course, he was not denied due process.
In the instant case, a scrutiny of the NAPOLCOM decision denying petitioner’s appeal from the PNP
Chief’s order of dismissal clearly shows that petitioner was afforded an opportunity to present his side
and defend his interests.

Gonzales v. CSC 490 SCRA 741 – Mac

Berboso v. CA 494 SCRA 583 – Mac

Petitioners Berbosos further claimed they were denied due process in the application, issuance and
confirmation of the Conversion Order issued by DAR.

WON petitioner was denied due process

Ruling: When private respondents Carloses applied for the issuance of the Conversion Order with the
DAR in 1973, one of the original tenants in the subject land was petitioners Berbosos' father, Macario
Berboso. While the said application is still pending with the DAR, all tenants of private respondents
Carloses, including Macario Berboso, were notified and interviewed by DAR Officer Guillermo V. Sta. Ana
as regards the said application. All of the said tenants, including Macario Berboso, made written
declarations and manifestations with regard to the said application.Despite said notices, Macario
Berboso did not initiate any proceedings to contest the processing of the application and the
subsequent issuance of the Conversion Order.

Even assuming for the sake of argument that petitioners Berbosos were not notified of the application
for Conversion Order filed by private respondents Carloses, this lack of notice had been cured when they
actively intervened and participated in the proceedings before the DARAB, the PARAB, the Office of the
President, and the Court of Appeals. The petitioners Berbosos made appeals and had also repeatedly
moved for the reconsideration of each decision that was adverse to them. Time and again, we ruled that
what is repugnant to due process is the absolute lack of opportunity to be heard. The essence of due
process is simply an opportunity to be heard or, as applied to administrative proceedings, an
opportunity to seek a reconsideration of the action or ruling complained of. Due process is satisfied
when the parties are afforded fair and reasonable opportunity to explain their side of the controversy or
an opportunity to move for a reconsideration of the action or ruling complained of.

Pontejos v. Desierto 592 - Mac

Facts: Petitioner has charged with extortion, bribery and graft and corruption. He was suspended by
Ombudsman Desierto. Petitioner moved to reconsider the above decision but this was denied by the
Ombudsman in an Order dated July 21, 1999. Thereafter, he filed a petition for review under Rule 43 of
the Rules of Court in the CA. On August 21, 2000, the CA dismissed the petition and upheld the
Ombudsman’s decision finding petitioner guilty of grave misconduct. Petitioner moved for
reconsideration but the CA denied his motion

Issue:WON he was denied due process

Ruling: No. We have consistently held that the essence of due process is simply the opportunity to be
heard or, as applied to administrative proceedings, the opportunity to explain one’s side or the
opportunity to seek a reconsideration of the action or ruling complained of. Any seeming defect in its
observance is cured by the filing of a motion for reconsideration. Denial of due process cannot be
successfully invoked by a party who has had the opportunity to be heard on his motion for
reconsideration.7 As the records would show, petitioner had filed a motion for reconsideration of the
decision of the Ombudsman. Hence, petitioner’s protestations that he had been deprived of due process
must necessarily fail.

I. Suretyship

Stronghold Insurance v. CA 205 SCRA 605 – Mac

Facts: Leisure Club Inc filed a replevin against Northern Motors. This was approved provided that Lesire
Club file a bond which it did through petitioner Stronghold. Later on it was found out that Northern
Motors has a better right to those property obtained by Leisure Club Inc but then Leisure Club Inc never
appeared again in court after it got hold of those personal property. Eventually Northern Motors sought
for the issuance of the writ of execution of bond in its favor. At the hearing of the said motion as well as
the opposition thereto filed by Stronghold Insurance Co., Inc., Northern Motors Inc. presented one
witness in the person of its former manager Clarissa G. Ocampo Stronghold Insurance Co., Inc. did not
cross-examine the said witness. Instead it asked for continuance in order to present its own witness.
Stronghold, however, never presented any witness.
Issue: WON Stronghold should really be liable for the bond (due process was never mentioned in the
entire case)

Ruling: Yes. Under the terms of the said bond, Stronghold Insurance together with Leisure Club Inc.
solidarily bound themselves in the sum of P42,000 —

In the case at bar, all the necessary conditions for proceeding against the bond are present, to wit:(i) the
plaintiff a quo, in bad faith, failed to prosecute the action, and after relieving the property, it promptly
disappeared; (ii) the subject property disappeared with the plaintiff, despite a court order for their
return; and (iii) a reasonable sum was adjudged to be due to respondent, by way of actual and
exemplary damages, attorney's fees and costs of suit.

J. Tariff and Customs Code

Feeder v. CA 197 SCRA 842 – Mac


The M/T "ULU WAI" foreign vessel of Honduran registry, owned and operated by Feeder International
Shipping Lines of Singapore, left Singapore on May 6, 1986 carrying 1,100 metric tons of gas oil and
1,000 metric tons of fuel oil consigned to Far East Synergy Corporation of Zamboanga, Philippines.

The vessel anchored at the vicinity of Guiuanon Island in Iloilo without notifying the Iloilo customs
authorities. The presence of the vessel only came to the knowledge of the Iloilo authorities by
information of the civilian informer in the area. Acting on said information, the Acting District Collector
of Iloilo dispatched a Customs team to verify the report.

The Customs team found out that the vessel did not have on board the required ship and shipping
documents, except for a clearance from the port authorities of Singapore clearing the vessel for
"Zamboanga."In view thereof, the vessel and its cargo were held and a Warrant of Seizure and
Detention over the same was issued after due investigation. It was then found guilty of violating the
Tariff and Customs Code of the Phil. The decision was affirmed both by the Court of Tax Appeals and CA.

Issue: WON the petitioner was deprived of property without due process of law because its right to be
presumed innocent was not recognized.

Ruling: Proceedings for the forfeiture of goods illegally imported are not criminal in nature. Proof
beyond reasonable doubt is not required to justify forfeiture of goods. What is required is substantial
evidence which means relevant evidence which a reasonable mind might accept as adequate to support
a conclusion. In this case, the govt has sufficiently established that there was an illegal importation or an
attempt thereof which warrants the forfeiture of the vessel and its cargo. Aside from this the right to be
presumed innocent can be invoke only by natural personals and not by a corporate entity like petitioner.

K. Appeal
Alba v. Deputy Ombudsman 254 SCRA 753
Facts: Alba is the DECS Director who was suppose to settle a conflict between a school (Arriesgado
Institute of Medical Sciences Foundation, Inc) and its students but Alba showed bias in favor of the
school to the detriment of the students; hence, the Ombudsman suspended him. Hahaha

Issue: WON the power of Ombudsman to impose penalty involving public censure, reprimand,
suspension of not more than one month, or a fine equivalent to one month salary, the decision shall be
final and unappealable which is final and unappealable violate the right to due process of petitioner
Whether or not the thirty (30)-day suspension of Petitioner, without pay and ‘unappealable’, imposed
by herein respondent DEPUTY OMBUDSMAN for MINDANAO, Cesar E. Nitorreda, was in accordance
with a valid or constitutional law/legislation and/or in accordance with due process, supported by
substantial evidence and is not arbitrary, whimsical and a grave abuse of discretion or authority on the
part of said Nitorreda

Ruling: No. Petitioner assails the constitutionality of Section 27 of R.A. 6770 (Ombudsman Law) and
Section 7, Rule III of Administrative Order No.7 for their failure to provide for the right of appeal in
certain cases from the decision of the Ombudsman, maintaining that the same is tantamount to a
deprivation of property without due process of law. As regards this threshold matter, suffice it to say
that this Court has consistently held that:
“The right to appeal is not a natural right nor a part of due process; it is merely a statutory privilege,
and may be exercised only in the manner and in accordance with” the provisions of the law.”

Apparently, therefore, the constitutional requirement of due process may be satisfied notwithstanding
the denial of the right to appeal for the essence of due process is simply the opportunity to be heard
and to present evidence in support of one’s case.

The Office of the Ombudsman is vested by law with the power to promulgate its own rules of procedure,
and a perusal of the said rules of procedure in administrative cases manifest sufficient compliance with
the requirements of due process.

Telan v. CA 202 SCRA 246


The petitioners in this case was the defendant in an accion publiciana wherein they lost. They wanted
to appeal but their original lawyer was not available anymore so they hired Atty. Palma to raise their
appeal. Eventually, the court ruled that their appeal was abandoned and dismissed due to failure to
file an appeal brief on time. It was during this time that petitioners realize that Atty. Palma was a fake
lawyer.
Issue: Whether or not the representation of the petitioner by a fake lawyer amounts to a deprivation of
his right to counsel and hence a lack of due process.
Ruling: We rule for the petitioners. We hold that they had not been accorded due process of law
because they lost their to appeal when deprived of the right to counsel. The right to counsel is absolute
and may be invoked at all times. More so, in the case of an on-going litigation, it is a right that must be
exercised at every step of the way, with the lawyer faithfully keeping his client company.

Rivera v. CSC 240 SCRA 43


Rivera, a manager of Landbank was charged with dishonesty etc. Landbank ordered his forced
resignation but on appeal the Merit Systems Protection Board reduced the penalty to only suspension
for 1 year. Rivera and the LBP both appealed to the CSC. The CSC affirmed the decision of landbank
which was the forced resignation; hence, this case. Note that a member of the Merit Systems
Protection Board is also a member of CSC who participated in both decisions of the Merit Systems and
the CSC regarding this case.
Issue:WON this is allowed
Ruling: No. In order that the review of the decision of a subordinate officer might not turn out to be a
farce, then reviewing officer must perforce be other than the officer whose decision is under review;
otherwise, there could be no different view or there would be no real review of the case. The decision of
the reviewing officer would be a biased view; inevitably, it would be the same view since being human,
he would not admit that he was mistaken in his first view of the case. The resolution of CSC reviving the
forced resignation is therefore set aside.

Singson v. NLRC 274 SCRA 358 – Kaye

FACTS:
Singson was an employee of PAL. On 7 Jun 1991, a Japanese national alleged that Singson extorted
money from her ($200.00) by accusing her of having excess baggage; and that to settle the issue she
needs to pay said amount to him. Singson was later investigated and the investigating committee found
him guilty. PAL then dismissed Singson from employment. Singson then filed a case before NLRC against
PAL for illegal dismissal, atty’s fees and damages. Labor Arbiter Raul Aquino ruled in favor of Singson as
he found PAL’s side insufficient to dismiss Singson. PAL appealed to the NLRC. The 2nd Division,
composed of Calaycay, Rayala former Arbiter Raul Aquino, of the NLRC took cognizance of the case.
NLRC reversed the decision of Aquino. Singson moved for reconsideration which was denied by NLRC,
this time only Calaycay & Rayala voted.

ISSUE: Whether or not Singson was denied of due process.

HELD: The SC ruled that Singson was denied due process. The SC held that Singson was denied due
process when Aquino participated, as presiding commissioner of the 2nd Division of the NLRC, in
reviewing PAL’s appeal. He was reviewing his own decision as a former labor arbiter. Under Rule VII,
Section 2 (b) of the New Rules of Procedure of the NLRC, each Division shall consist of one member from
the public sector who shall act as the Presiding Commissioner and one member each from the workers
and employers sectors, respectively. The composition of the Division guarantees equal representation
and impartiality among its members. Thus, litigants are entitled to a review of three (3) commissioners
who are impartial right from the start of the process of review. Commissioner Aquino can hardly be
considered impartial since he was the arbiter who decided the case under review. He should have
inhibited himself from any participation in this case. The infirmity of the resolution was not cured by the
fact that the motion for reconsideration of Singson was denied by two commissioners and without the
participation of Aquino. The right of petitioner to an impartial review of his appeal starts from the time
he filed his appeal. He is not only entitled to an impartial tribunal in the resolution of his motion for
reconsideration. Moreover, his right is to an impartial review of three commissioners. The denial of
Singson’s right to an impartial review of his appeal is not an innocuous error. It negated his right to due
process.

Building Care v. Macaraeg 687 SCRA 643- Kaye

BUILDING CARE V MACARAEG (687 SCRA 643)


FACTS:
Petitioners are in the business of providing security services to their clients. They hired respondent as a
security guard beginning August 25, 1996, assigning her at Genato Building in Caloocan City. However,
on March 9, 2008, respondent was relieved of her post. She was re-assigned to Bayview Park Hotel from
March 9-13, 2008, but after said period, she was allegedly no longer given any assignment. Thus, on
September 9, 2008, respondent filed a complaint against petitioners for illegal dismissal, underpayment
of salaries, non-Payment of separation pay and refund of cash bond. Petitioners allegedly directed
respondent to explain why she committed such infractions, but respondent failed to heed such order.
Respondent was nevertheless temporarily assigned to Bayview Park Hotel from March 9-13, 2008, but
she also failed to meet said client's standards and her posting thereat was not extended.5 Respondent
then filed an administrative complaint for illegal dismissal with the PNP-Security Agencies and Guard
Supervision Division on June 18, 2008, but she did not attend the conference hearings for said case.
Petitioners brought to the conference hearings a new assignment order detailing respondent at the
Ateneo de Manila University but, due to her absence, petitioners failed to personally serve respondent
said assignment order. Petitioners then sent respondent a letter Ordering her to report to headquarters
for work assignment, but respondent did not comply with said order. Instead, respondent filed a
complaint for illegal dismissal with the Labor Arbiter. Labor Arbiter rendered a Decision, the dispositive
portion of which reads as follows:

WHEREFORE, judgment is hereby made dismissing the charge of illegal dismissal as wanting in merit but,
as explained above, ordering the Respondents Leopard Security and Investigation Agency and Rupert
Protacio to pay complainant a financial assistance in the amount of P5,000.00.

Respondent then filed a Notice of Appeal with the National Labor Relations Commission (NLRC), but in a
Decision dated October 23, 2009, the NLRC dismissed the appeal for having been filed out of time,
thereby declaring that the Labor Arbiter's Decision had become final and executory on June 16, 2009.

Respondent elevated the case to the CA via a petition for certiorari, and on March 24, 2011, the CA
promulgated its Decision, the dispositive portion of which reads as follows:

WHEREFORE, the petition for certiorari is GRANTED. The


Decision dated October 23, 2009 and Resolution dated March 2, 2010 rendered by public respondent in
NLRC LAC No. 07-001892-09 (NLRC Case No. NCR-09-12628-08) are REVERSED and SET ASIDE, and in lieu
thereof, a new judgment is ENTERED declaring petitioner to have been illegally dismissed and
DIRECTING private respondents to reinstate petitioner without loss of seniority rights, benefits and
privileges; and to pay her backwages and other monetary benefits during the period of her illegal
dismissal up to actual reinstatement. Public respondent NLRC is DIRECTED to conduct further
proceedings, for the sole purpose of determining the amount of private respondent's monetary
liabilities in accordance with this decision.

ISSUE: whether the CA erred in liberally applying the rules of procedure and ruling that respondent's
appeal should be allowed and resolved on the merits despite having been filed out of time.>>YES.

RULING: The Court cannot sustain the CA's Decision. It should be emphasized that the resort to a liberal
application, or suspension of the application of procedural rules, must remain as the exception to the
well-settled principle that rules must be complied with for the orderly administration of justice. In this
case, the justifications given by the CA for its liberality by choosing to overlook the belated filing of the
appeal are, the importance of the issue raised, i.e., whether respondent was illegally dismissed; and the
belief that respondent should be “afforded the amplest opportunity for the proper and just
determination of his cause, free from the constraints of technicalities,”14 considering that the belated
filing of respondent's appeal before the NLRC was the fault of respondent's former counsel. Note,
however, that neither respondent nor her former counsel gave any explanation or reason citing
extraordinary circumstances for her lawyer's failure to abide by the rules for filing an appeal.
Respondent merely insisted that she had not been remiss in following up her case with said lawyer.

It is, however, an oft-repeated ruling that the negligence and mistakes of counsel bind the client. A
departure from this rule would bring about never-ending suits, so long as lawyers could allege their own
fault or negligence to support the client’s case and obtain remedies and reliefs already lost by the
operation of law.15 The only exception would be, where the lawyer's gross negligence would result in
the grave injustice of depriving his client of the due process of law.16 In this case, there was no such
deprivation of due process. Respondent was able to fully present and argue her case before the Labor
Arbiter. She was accorded the opportunity to be heard. Her failure to appeal the Labor Arbiter's Decision
cannot, therefore, be deemed as a deprivation of her right to due process.

L. Closure Proceeding

CB v. CA 220 SCRA 536 – Kaye


Central Bank of the Philippines v. CA (1985)
Ponente: Makasiar, C.J.
Topic: Delay (Art. 1169)

Facts:
April 28, 1965 - Island Savings Bank (ISB) approved the loan application for P80,000 of Sulpicio
Tolentino, who, as a security for the loan, also executed a real estate mortgage over his 100-ha land. The
approved loan application called for P80,000 loan, repayable in semi-annual installments for a period of
3 years, with 12% interest.
May 22, 1965 – a mere P17,000 partial release of the loan was made by ISB, and Tolentino and
his wife Edita signed a promissory note for P17,000 at 12% annual interest, payable within 3 years from
the date of execution of the contract at semi-annual installments of P3,459.
An advance interest for the P80,000 loan covering a 6-mo period amounting to P4,800was
deducted from the partial release of P17,000, but this was refunded to Tolentino on July 23, 1965, after
being informed by ISB that there was no fund yet available for the release of the P63,000 balance.
Aug. 13, 1965 – the Monetary Board of the Central Bank issued Resolution No. 1049, which
prohibited ISB from making new loans and investments, after finding that it was suffering liquidity
problems.
June 14, 1968 – the Monetary Board issued Resolution No. 967, which prohibited ISB from doing
business in the Philippines, after finding that it failed to put up the required capital to restore its
solvency.
Aug. 1, 1968 – ISB, in view of non-payment of the P17,000 covered by the promissory note, filed
an application for the extra-judicial foreclosure of the real estate mortgage covering the 100-ha land;
and the sheriff scheduled auction.
Tolentino filed a petition with the CFI for injunction, specific performance or rescission and
damages with preliminary injunction, alleging that since ISB failed to deliver the P63,000 remaining
balance of the loan, he is entitled to specific performance by ordering ISB to deliver it with interest of
12% per annum from April 28, 1965, and if said balance cannot be delivered, to rescind the real estate
mortgage.
CFI issued a TRO enjoining ISB from continuing with the foreclosure of the mortgage, however,
after finding Tolentino’s petition unmeritorious, ordered the latter to pay ISB P17,000 plus legal interest
and legal charges and lifting the TRO so the sheriff may proceed with the foreclosure.

CA, on appeal by Tolentino, modified CFI’s decision by affirming dismissal of Tolentino’s petition for
specific performance, but ruled that ISB can neither foreclose the mortgage nor collect the P17,000 loan.

SC: The parties, in the P80,000 loan agreement, undertook reciprocal obligations, wherein the
obligation/promise of each party is the consideration for that of the other; and when one party has
performed or is ready and willing to perform his part of the contract, the other party who has not
performed or is not ready and willing to perform incurs in delay (Art. 1169, CC).
When Tolentino executed a real estate mortgage, he signified his willingness to pay the P80,000
loan, and from such date, the obligation of ISB to furnish the loan accrued. Thus, ISB’s delay started on
April 28, 1965 and lasted 3 years or when Resolution No. 967 was issued prohibiting ISB from doing
further business, which made it legally impossible from ISB to furnish the P63,000 of the loan.
Resolution No. 1049 cannot interrupt the default of ISB in complying with its obligation to release
the P63,000 balance because it merely prohibited ISB from making new loans and investments, not from
releasing the balance of loan agreements previously contracted.
The mere pecuniary inability to fulfill an engagement does not discharge the obligation of the
contract, nor does it constitute any defense to a decree of specific performance; and the mere fact of
insolvency of a debtor is never an excuse for the nonfulfillment of an obligation, but instead, is taken as
a breach of contract.
The fact that Tolentino demanded and accepted the refund of the pre-deducted interest cannot be
taken as a waiver of his right to collect the P63,000 balance. The act of ISB in asking for the advance
interest was improper considering that only P17,000 out of the P80,000 loan was released.
The alleged discovery by ISB of the overvaluation of the loan collateral cannot exempt it from
complying with its obligation to furnish the entire P80,000 loan because bank officials/employees have
the obligation to investigate the existence and valuation of the properties being offered as a loan
security before approving the loan application.

Issues/Held/Ratio
1) WON the action of Tolenitno for specific performance can prosper.  NO.
Since ISB was in default under the agreement, Tolentino may choose between specific performance
or rescission, but since ISB is now prohibited from doing further business, the only remedy left is
Rescission only for the P63,000 balance of the loan.

2) WON Tolentino is liable to pay the P17,000 debt covered by the promissory note.  YES.
The bank was deemed to have complied with its reciprocal obligation to furnish a P17,000 loan. The
promissory note gave rise to Tolentino’s reciprocal obligation to pay such loan when it falls due and his
failure to pay the overdue amortizations under the promissory note made him a party in default, hence
not entitled to rescission (Art. 1191, CC). ISB has the right to rescind the promissory note, being the
aggrieved party.

Since both parties were in default in the performance of their reciprocal obligations, both are liable
for damages. In case both parties have committed a breach of their reciprocal obligations, the liability of
the first infractor shall be equirably tempered by the courts (Art. 1192, CC). The liability of ISB for
damages in not furnishing the entire loan is offset by the liability of Tolentino for damages (penalties
and surcharges) for not paying his overdue P17,000 debt. Since Tolentino derived some benefit for his
use of the P17,000, he should account for the interest thereon (interest was not included in the
offsetting).

3) WON Tolentino’s real estate mortgage can be foreclosed to satisfy the P17,000 if his liability to
pay therefor subsists.  NO.
The fact that when Tolentino executed his real estate mortgage, no consideration was then in
existence, as there was no debt yet because ISB had not made any release on the loan, does not make
the real estate mortgage void for lack of consideration.
It is not necessary that any consideration should pass at the time of the execution of the contract of
real mortgage. When the consideration is subsequent to the mortgage, the latter can take effect only
when the debt secured by it is created as a binding contract to pay. And when there is partial failure of
consideration, the mortgage becomes unenforceable to the extent of such failure. Where the
indebtedness actually owing to the holder of the mortgage is less than the sum named in the mortgage,
the mortgage cannot be enforced for more than the actual sum due.
Since ISB failed to furnish the P63,000 balance, the real estate mortgage of Tolentino became
unenforceable to such extent. P63,000 is 78.75% of P80,000, hence the mortgage covering 100 ha is
unenforceable to the extent of 78.75 ha. The mortgage covering the remainder of 21.25 ha subsists as a
security for the P17,000 debt.

Judgment:
1) Tolentino is ordered to pay ISB P17,000 plus P41, 210 (12% interest per annum)
2) In case Tolentino fails to pay, his real estate mortgage covering 21.25 ha shall be foreclosed to
satisfy his total indebtedness
3) The real estate mortgage covering 78.75 ha is unenforceable and ordered released in favor of
Tolentino

Rural Bank v. CA 162 SCRA 288- Kaye


RURAL BANK OF BUHI VS. CA
procedural due process

 Buhi Bank was a rural bank. Its books were examined by the Rural Banks division of the Central Bank
 However, it refused to be examined. As a consequence, its financial assistance was suspended
 Later, a general examination of the bank’s affairs and operations were again conducted.
 The rural bank’s division found out massive irregularities in the operations, giving out loans to
unknown and fictitious borrowers, and sums amounting to millions past due to the Central Bank.
There were also promissory notes rediscounted with the Central Bank for cash.
 As a result, the Buhi Bank became insolvent.
 The division chief, Odra, recommended that Buhi be placed under receivership.
 Thus, the Monetary Board adopted a Resolution # 583, placing the bank under receivership. Odra,
the division chief, was made the receiver.
 Odra thus implemented the resolution, authorizing deputies to take control and possession of Buhi’s
assets and liabilities.
 Del Rosario, the Buhi Bank Manager, filed an injunction against the receiver, arguing that the
resolution violated the Rural Banks Act and constitutes gadalej. The bank claims that there was a
violation of due process. They claim that the bank was not given the chance to deny and disprove
the claim of insolvency or the other grounds and that it was hastily put under receivership.
 Later on, the Central Bank Monetary Board ordered the liquidation of the Bank.
 The judge ruled in favor of the Bank and issued a writ of execution.
 The CA however restrained the enforcement of execution, citing that the Judge did not follow the
orders, and thus required the Bank to yield to the CB.

ISSUE: Was due process observed?


SC: YES. CLOSURE VALID.

Under Sec 29 of the RA 265, on proceedings regarding insolvency, there is NO REQUIREMENT that a
hearing be first conducted before a bank may be placed under receivership. The law explicitly provides
that the Monetary Board can IMMEDIATELY forbid a banking institution from doing business and
IMMEDIATELY appoint a receiver when: 1) there has been an examination by CB, b) a report to the CB,
and c) prima facie showing that the bank is insolvent.
As to the claim that the RA 265 violates due process, the claim is untenable. The law could not have
intended to disregard the constitutional requirement of due process when it conferred power to place
rural banks under receivership.

The closure and liquidation of the bank is considered an exercise of POLICE POWER. It maybe subject to
judicial inquiry and could be set aside if found to be capricious, discriminatory, whimsical, arbitrary, etc.
The appointment of a receiver may be made by the Monetary Board, WITHOUT NOTICE AND HEARING,
but subject to the JUDICIAL INQUIRY, to insure protection of the banking institution.

Due process does NOT necessarily require a PRIOR HEARING. A hearing or an OPPORTUNITY TO BE
HEARD may be made SUBSEQUENT to the closure. One could just imagine the dire consequences of a
prior hearing: bank runs would happen, resulting in panic and hysteria. In that way, fortunes will be
wiped out, and disillusionment will run the gamut of the entire banking industry.

There is no question that the action of the MB may be subject to judicial review. Courts may interfere
with the MB’s exercise of discretion. Here, the RTC has jurisdiction to adjudicate the question of
whether the MB acted in bad faith when it directed the dissolution of Buhi Bank.

Phil. Merchants v. CA GR 112844 June 2, 1995- Kaye

G.R. No. 112844 June 2, 1995

PHILIPPINE MERCHANT MARINE SCHOOL, INC., represented by JUAN O. NOLASCO III, petitioner,
vs.
COURT OF APPEALS, THE OFFICE OF THE EXECUTIVE SECRETARY, EDELMIRO AMANTE, RENATO
CORONA, and the DEPARTMENT OF EDUCATION, CULTURE AND SPORTS, respondents.

FACTS:
Sometime in 1986 the DECS received a complaint from Felixberto B. Galvez, president of petitioner's
Faculty Association, NAFLU-KMU, concerning the issuance of summer permit to petitioner and of its
holding of classes for courses not recognized by the Government. Galvez requested that the matter be
looked into as well as the possible revocation of petitioner's authority due to persistent violation of the
orders of the DECS.
In response, the DECS through Director Boquiren recommended that petitioner's summer permit be
revoked and that the school be closed effective SY 1986-1987 on the ground that: (a) petitioner did not
have a renewal permit/recognition for SY 1986-1987; (b) several communications were sent to
petitioner's head telling him not to operate without permit and to explain within seventy-two (72) hours
from receipt of Director Boquiren's letter dated 9 July 1986 why no drastic action should be taken
against it but said communication was never answered; and, (c) petitioner did not correct the
deficiencies indicated in the renewal permit for 1985-1986.
Petitioner assailed both resolutions of the Office of the President before respondent Court of Appeals by
way ofcertiorari. It alleged that the resolutions failed to meet the constitutional requirement of due
process because the basis for affirming the DECS phase-out and closure orders was not sufficiently
disclosed. Furthermore, its letters dated 2 and 3 October 1992 which presented incontrovertible proof
that it had introduced substantial improvements on its facilities for the past two and a half years while
its appeal was pending were not taken into account, thereby gravely abusing its discretion.
ISSUE: WON due process was denied herein. >>NO.

RULING: Set against the records of the case, the assertion of petitioner that it was deprived of its right
to a hearing and any opportunity whatsoever to correct the alleged deficiencies readily collapses. The
earlier narration of facts clearly demonstrates that before the DECS issued the phase-out and closure
orders, petitioner was duly notified, warned and given several opportunities to correct its deficiencies
and to comply with pertinent orders and regulations.

Petitioner has gone all the way up to the Office of the President to seek a reversal of the phase-out and
closure orders. There is thus no reason to complain of lack of opportunity to explain its side as well as to
comply with the alleged deficiencies.

M. Biddings

G.R. No. 109113 January 25, 1995

CONCERNED OFFICIALS OF THE METROPOLITAN WATERWORKS AND SEWERAGE SYSTEM


(MWSS),petitioners,
vs.
HON. OMBUDSMAN CONRADO M. VASQUEZ AND MEMBERS OF THE PHILIPPINE LARGE DIAMETER
PRESSURE PIPE MANUFACTURERS ASSOCIATION (PLDPPMA), respondents.

FACTS:

The Ombudsman, in its 19th October 1992 Order, 1 directed the Board of Trustees of Metropolitan
Waterworks and Sewerage System ("MWSS") (a) to set aside the recommendation of its Pre-
qualification, Bids and Awards Committee for Construction Services and Technical Equipment ("PBAC-
CSTE") that Contract No. APM-01 be given to a contractor offering fiberglass pipes and (b) to instead
award the contract to a complying and responsive bidder pursuant to the provisions of Presidential
Decree No. 1594. 2 The subsequent motion for reconsideration was denied by the Ombudsman in its
Order 01 March 1993.

Petitioners were asked to comment on the letter-complaint of PLDPPMA. On 25 June 1992, petitioners
moved for an extension of time within which to comment. On July 16, 1992, petitioners filed their letter-
comment. Responding to the reply of PLDPPMA, petitioners later filed a rejoinder. When an adverse
order was rendered against them, petitioners moved for its reconsideration, albeit to no avail.to Joint
Venture.

ISSUES: whether or not the rudiments of due process have been properly observed.>>>YES.

RULING:

Relative to the first issue, we are more than convinced, after a scrutiny of the records of this case, that
petitioners have been amply accorded the opportunity to be heard.
The absence of due process is an opportunity to be heard. 32 One may be heard, not solely by verbal
presentation but also, and perhaps even many times more creditably and practicable than oral
argument, through pleadings. 33 In administrative proceedings, moreover, technical rules of procedure
and evidence are not strictly applied; administrative due process cannot be fully equated to due process
in its strict judicial sense.

N. UDHA – RA 7279

G.R. No. 184478 March 21, 2012

PEREZ V. MADRONA

FACTS:

Respondent-spouses Fortunito Madrona and Yolanda B. Pante are registered owners of a residential
property located in Greenheights Subdivision, Phase II, Marikina City and covered by TCT No. 169365 of
the Registry of Deeds of Marikina. In 1989, respondents built their house thereon and enclosed it with a
concrete fence and steel gate. In 1999, respondents received the following letter dated May 25,
1999 from petitioner Jaime S. Perez, Chief of the Marikina Demolition Office stating that the structure
that they built encroached on the sidewalk and that is in violation of PD 1096 of the National Building
Code and RA 917 on Illegally occupied/constructed improvements within the road right-of-way. The
respondent-spouses are given 7 days to remove the said structure. As response, respondent Madrona
sent petitioner a letter stating that the May 25, 1999 letter (1) contained an accusation libelous in
nature as it is condemning him and his property without due process; (2) has no basis and authority
since there is no court order authorizing him to demolish their structure; (3) cited legal bases which do
not expressly give petitioner authority to demolish; and (4) contained a false accusation since their fence
did not in fact extend to the sidewalk. More than a year later or on February 28, 2001, petitioner sent
another letter with the same contents as the May 25, 1999 letter but this time giving respondents ten
days from receipt thereof to remove the structure allegedly protruding to the sidewalk. This prompted
respondents to file a complaint for injunction before the Marikina City RTC on March 12,
2001. Respondents likewise sought the issuance of a temporary restraining order (TRO) and a writ of
preliminary injunction to enjoin petitioner and all persons acting under him from doing any act of
demolition on their property and that after trial, the injunction be made permanent. On March 16,
2001, the RTC issued a TRO against petitioner. On July 27, 2004, the RTC rendered a Decision in favor of
respondents. The RTC decision permanently enjoined defendant Perez from performing any act which
would tend to destroy or demolish the perimeter fence and steel gate of the respondents’ property.
The RTC held that respondents, being lawful owners of the subject property, are entitled to the peaceful
and open possession of every inch of their property and petitioner’s threat to demolish the concrete
fence around their property is tantamount to a violation of their rights as property owners who are
entitled to protection under the Constitution and laws. The RTC also ruled that there is no showing
that respondents’ fence is a nuisance per se and presents an immediate danger to the community’s
welfare, nor is there basis for petitioner’s claim that the fence has encroached on the sidewalk as to
justify its summary demolition. CA affirmed,
ISSUE: W/N respondents’ structure is a nuisance per se that presents immediate danger to the
community’s welfare and can be removed without need of judicial intervention since the clearing of the
sidewalks is an infrastructure project of the Marikina City Government and cannot be restrained by the
courts as provided in Presidential Decree No. 1818

HELD:

No. If petitioner indeed found respondents’ fence to have encroached on the sidewalk, his remedy is
not to demolish the same summarily after respondents failed to heed his request to remove it. Instead,
he should go to court and prove respondents’ supposed violations in the construction of the concrete
fence. Indeed, unless a thing is a nuisance per se, it may not be abated summarily without judicial
intervention. Our ruling in Lucena Grand Central Terminal, Inc. v. JAC Liner, Inc., on the need for judicial
intervention when the nuisance is not a nuisance per se, is well worth mentioning. In said case, we
ruled:

Respondents can not seek cover under the general welfare clause authorizing
the abatement of nuisances without judicial proceedings. That tenet applies to a
nuisance per se, or one which affects the immediate safety of persons and property and
may be summarily abated under the undefined law of necessity (Monteverde v.
Generoso, 52 Phil. 123 [1982]). The storage of copra in the quonset building is a
legitimate business. By its nature, it can not be said to be injurious to rights of property,
of health or of comfort of the community. If it be a nuisance per accidens it may be so
proven in a hearing conducted for that purpose. It is not per se a nuisance warranting
its summary abatement without judicial intervention. [Underscoring supplied.]

In Pampanga Bus Co., Inc. v. Municipality of Tarlac where the appellant-


municipality similarly argued that the terminal involved therein is a nuisance that may
be abated by the Municipal Council via an ordinance, this Court held: “Suffice it to say
that in the abatement of nuisances the provisions of the Civil Code (Articles 694-707)
must be observed and followed. This appellant failed to do.

Respondents’ fence is not a nuisance per se. By its nature, it is not injurious to the health or
comfort of the community. It was built primarily to secure the property of respondents and
prevent intruders from entering it. And as correctly pointed out by respondents, the sidewalk
still exists. If petitioner believes that respondents’ fence indeed encroaches on the sidewalk, it
may be so proven in a hearing conducted for that purpose. Not being a nuisance per se, but at
most a nuisance per accidens, its summary abatement without judicial intervention is
unwarranted.

O. Cancellation of Property Rights/Privileges

American Inter-Fashion v. OP, 197 SCRA 409- Kaye

G.R. No. 92422 May 23, 1991


AMERICAN INTER-FASHION CORPORATION, petitioner,
vs.
OFFICE OF THE PRESIDENT, GARMENTS & TEXTILE EXPORT BOARD & GLORIOUS SUN FASHION
GARMENTS MANUFACTURING CO. (PHILS.), INC., respondents.

FACTS: On April 27, 1984, respondent GLORIOUS was found guilty of dollar-salting and misdeclaration of
importations by the GTEB in OSC Case No. 84-B-1 and, as a result of which, the export quotas allocated
to it were cancelled. Soon after the rendition of the GTEB decision, respondent GLORIOUS filed a
petition for certiorari and prohibition with the Court, docketed as G.R. No. 67180, contending that its
right to due process of law was violated, and that the GTEB decision was not supported by substantial
evidence. Giving credence to the allegations of respondent GLORIOUS, the Court issued a resolution on
June 4, 1984, ordering GTEB to conduct further proceedings in the administrative case against
respondent GLORIOUS. However, on July 25, 1984, respondent GLORIOUS filed a manifestation of its
intention to withdraw the petition. On August 20, 1984, the Court granted respondent GLORIOUS'
motion for withdrawal. Respondent GLORIOUS filed another motion to dismiss with prejudice, which
was duly noted by the Court in a resolution dated September 10, 1984.

More than two years later, on October 15, 1986, respondent GLORIOUS filed with the GTEB a petition
for the restitution of its export quota allocation and requested for a reconsideration of the GTEB
decision dated April 27, 1984. Once again, respondent GLORIOUS alleged that the charges against it in
OSC Case No. 84-B-1 were not supported by evidence. Moreover, it alleged that the GTEB decision
cancelling its export quotas was rendered as a result of duress, threats, intimidation and undue
influence exercised by former Minister Roberto V. Ongpin in order to transfer GLORIOUS' export quotas
to "Marcos crony-owned" corporations De Soleil Apparel Manufacturing Corporation [DSA] and
petitioner AIFC Respondent GLORIOUS further alleged that it was coerced by Mr. Roberto Ongpin to
withdraw its petition in G.R. No. 67180 and to enter into joint venture agreements paving the way for
the creation of DSA and petitioner AIFC which were allowed to service respondent GLORIOUS' export
quotas and to use its plant facilities, machineries and equipment.

ISSUE: Whether or not due process was observed.>>NO.


RULING:
Contrary to the petitioner's posture, the record clearly manifests that in cancelling the export quotas of
the private respondent, GTEB violated the private respondent's constitutional right to due process.
Before the cancellation in 1984, the private respondent had been enjoying export quotas granted to it
since 1977. In effect the private respondent's export quota allocation which initially was a privilege
evolved into some form of property right which should not be removed from it arbitrarily and without
due process only to hurriedly confer it on another

Alliance of DFLO v. Laguesma, 254 SCRA 565- Kaye


ALLIANCE OF DEMOCRATIC FREE LABOR ORGANIZATION (ADFLO), petitioner, vs. UNDERSECRETARY OF
LABOR BIENVENIDO LAGUESMA and CONFEDERATION OF LABOR AND ALLIED SOCIAL SERVICES
(CLASS), respondents.
FACTS: On 07 August 1989, ADFLO was summoned to a conference by the Bureau. In said
conference, the Bureau disclosed the seriousness of the charges against ADFLO that may warrant the
cancellation of its certificate of registration.
On 15 August 1989, a hearing was conducted and both parties were duly represented. ADFLO
manifested that it would move to inhibit the Director of Labor Relations from taking further action over
the present petition. It further manifested that it would file its comment to the earlier memorandum
filed by CLASS. CLASS, for its part, requested that it be given five (5) days within which to file its
objection against the motion to inhibit the Bureau Director.
On 25 August 1989, ADFLO filed its answer, averring that it had complied with all the legal
requirements for registration including the affiliation of more than 10 local unions; that it did not
commit any fraud or misrepresentation in its application for registration; that it conducted itself as a
legitimate labor organization and that the cancellation of its registration certificate which was secured in
good faith will violate the Constitutional right of the workers to organize and will deprive the
membership of their rights granted by law.
On November 16, 1990, the Bureau of Labor Relations (BLR), through Director Pura Ferrer-Calleja,
rendered a Decision cancelling the registration of ADFLO.
On February 12, 1992, BLR Director Pura Ferrer-Calleja. without first ruling on the admissibility of the
exhibits of CLASS and without any further hearing, rendered an order affirming the cancellation of
registration of ADFLO.

ISSUE: Was the decision cancelling the registration of petitioner rendered in violation of the due
process clause?>>YES.
RULING:
The most basic tenet of due process is the right to be heard, and as applied in administrative
proceedings, an opportunity to explain one’s side.[8] Such opportunity was denied petitioner in this case.
The public respondent and his subaltern, the Director of the Bureau of Labor Relations, should have
learned their lessons when the latter’s resolution dated November 16, 1989 cancelling petitioner’s
registration due precisely to absence of due process was reversed by the then Secretary of Labor whose
decision was, in effect, affirmed by this Court. However, instead of taking a lesson in due process, said
director - this time abetted by public respondent - violated again the same fundamental principle.
After petitioner submitted its objections to the admission of the documentary evidence of CLASS,
the BLR director should have first ruled on their admissibility. However, without ruling on said offer and
without setting the case for reception of petitioner’s evidence, the said official proceeded to render
judgment affirming its earlier (but already ruled as improper) decision to cancel the registration of
ADFLO. This is a gross violation of petitioner’s right to due process.
The cancellation of a certificate of registration is the equivalent of snuffing out the life of a labor
organization. For without such registration, it loses - as a rule - its rights under the Labor Code. Under
the circumstances, petitioner was indisputably entitled to be heard before a judgment could be
rendered cancelling its certificate of registration.

ABAKADA v. Ermita, 469 SCRA 1- Kaye


Abakada Guro v. Ermita
G.R. No. 168056, July 5, 2005

J. Puno En Banc

Facts:

Motions for Reconsideration filed by petitioners, ABAKADA Guro party List Officer and et al., insist that
the bicameral conference committee should not even have acted on the no pass-on provisions since
there is no disagreement between House Bill Nos. 3705 and 3555 on the one hand, and Senate Bill No.
1950 on the other, with regard to the no pass-on provision for the sale of service for power generation
because both the Senate and the House were in agreement that the VAT burden for the sale of such
service shall not be passed on to the end-consumer. As to the no pass-on provision for sale of petroleum
products, petitioners argue that the fact that the presence of such a no pass-on provision in the House
version and the absence thereof in the Senate Bill means there is no conflict because “a House provision
cannot be in conflict with something that does not exist.”

Escudero, et. al., also contend that Republic Act No. 9337 grossly violates the constitutional imperative
on exclusive origination of revenue bills under Section 24 of Article VI of the Constitution when the
Senate introduced amendments not connected with VAT.

Petitioners Escudero, et al., also reiterate that R.A. No. 9337’s stand- by authority to the Executive to
increase the VAT rate, especially on account of the recommendatory power granted to the Secretary of
Finance, constitutes undue delegation of legislative power. They submit that the recommendatory
power given to the Secretary of Finance in regard to the occurrence of either of two events using the
Gross Domestic Product (GDP) as a benchmark necessarily and inherently required extended analysis
and evaluation, as well as policy making.

Petitioners also reiterate their argument that the input tax is a property or a property right. Petitioners
also contend that even if the right to credit the input VAT is merely a statutory privilege, it has already
evolved into a vested right that the State cannot remove.

Issue:

Whether or not the R.A. No. 9337 or the Vat Reform Act is constitutional?

Held:

The Court is not persuaded. Article VI, Section 24 of the Constitution provides that All appropriation,
revenue or tariff bills, bills authorizing increase of the public debt, bills of local application, and private
bills shall originate exclusively in the House of Representatives, but the Senate may propose or concur
with amendments.

The Court reiterates that in making his recommendation to the President on the existence of either of
the two conditions, the Secretary of Finance is not acting as the alter ego of the President or even her
subordinate. He is acting as the agent of the legislative department, to determine and declare the event
upon which its expressed will is to take effect. The Secretary of Finance becomes the means or tool by
which legislative policy is determined and implemented, considering that he possesses all the facilities
to gather data and information and has a much broader perspective to properly evaluate them. His
function is to gather and collate statistical data and other pertinent information and verify if any of the
two conditions laid out by Congress is present.

In the same breath, the Court reiterates its finding that it is not a property or a property right, and a
VAT-registered person’s entitlement to the creditable input tax is a mere statutory privilege. As the
Court stated in its Decision, the right to credit the input tax is a mere creation of law. More importantly,
the assailed provisions of R.A. No. 9337 already involve legislative policy and wisdom. So long as there is
a public end for which R.A. No. 9337 was passed, the means through which such end shall be
accomplished is for the legislature to choose so long as it is within constitutional bounds.

The Motions for Reconsideration are hereby DENIED WITH FINALITY. The temporary restraining order
issued by the Court is LIFTED.

British American Tobacco v. Camacho 562 SCRA 511, 585 SCRA 36- Kaye
BRITISH AMERICAN TOBACCO V CAMACHO

FACTS:
Petitioner alleges that assuming the assailed law is constitutional, its Lucky Strike brand should be
reclassified from the premium-priced to the high-priced tax bracket. Relying on BIR Ruling No. 018-2001
dated May 10, 2001, it claims that it timely sought redress from the BIR to have the market survey
conducted within three months from product launch, as provided for under Section 4(B)21 of Revenue
Regulations No. 1-97, in order to determine the actual current net retail price of Lucky Strike, and thus,
fix its tax classification. Further, the upward reclassification of Lucky Strike amounts to deprivation of
property right without due process of law. The conduct of the market survey after two years from
product launch constitutes gross neglect on the part of the BIR. Consequently, for failure of the BIR to
conduct a timely market survey, Lucky Strike's classification based on its suggested gross retail price
should be deemed its official tax classification. Finally, petitioner asserts that had the market survey
been timely conducted sometime in 2001, the current net retail price of Lucky Strike would have been
found to be under the high-priced tax bracket.
ISSUE: whether or not the petitioner’s contention is tenable
RULING:

These contentions are untenable and misleading.

First, BIR Ruling No. 018-2001 was requested by petitioner for the purpose of fixing Lucky Strike's initial
tax classification based on its suggested gross retail price relative to its planned introduction of Lucky
Strike in the market sometime in 2001 and not for the conduct of the market survey within three
months from product launch. In fact, the said Ruling contained an express reservation that the tax
classification of Lucky Strike set therein "is without prejudice, however, to the subsequent conduct of a
survey x x x in order to determine if the actual gross retail price thereof is consistent with [petitioner's]
suggested gross retail price."22 In short, petitioner acknowledged that the initial tax classification of
Lucky Strike may be modified depending on the outcome of the survey which will determine the actual
current net retail price of Lucky Strike in the market.

Second, there was no upward reclassification of Lucky Strike because it was taxed based on its
suggested gross retail price from the time of its introduction in the market in 2001 until the BIR market
survey in 2003. We reiterate that Lucky Strikes' actual current net retail price was surveyed for the first
time in 2003 and was found to be from P10.34 to P11.53 per pack, which is within the premium-priced
tax bracket. There was, thus, no prohibited upward reclassification of Lucky Strike by the BIR based on
its current net retail price.

Third, the failure of the BIR to conduct the market survey within the three-month period under the
revenue regulations then in force can in no way make the initial tax classification of Lucky Strike based
on its suggested gross retail price permanent. Otherwise, this would contravene the clear mandate of
the law which provides that the basis for the tax classification of a new brand shall be the current net
retail price and not the suggested gross retail price. It is a basic principle of law that the State cannot be
estopped by the mistakes of its agents.

Last, the issue of timeliness of the market survey was never raised before the trial court because
petitioner's theory of the case was wholly anchored on the alleged unconstitutionality of the
classification freeze provision. As a consequence, no documentary evidence as to the actual net retail
price of Lucky Strike in 2001, based on a market survey at least comparable to the one mandated by law,
was presented before the trial court. Evidently, it cannot be assumed that had the BIR conducted the
market survey within three months from its product launch sometime in 2001, Lucky Strike would have
been found to fall under the high-priced tax bracket and not the premium-priced tax bracket. To so hold
would run roughshod over the State's right to due process. Verily, petitioner prosecuted its case before
the trial court solely on the theory that the assailed law is unconstitutional instead of merely challenging
the timeliness of the market survey. The rule is that a party is bound by the theory he adopts and by the
cause of action he stands on. He cannot be permitted after having lost thereon to repudiate his theory
and cause of action, and thereafter, adopt another and seek to re-litigate the matter anew either in the
same forum or on appeal.23 Having pursued one theory and lost thereon, petitioner may no longer
pursue another inconsistent theory without thereby trifling with court processes and burdening the
courts with endless litigation.

WHEREFORE, the motion for reconsideration is DENIED.

P. Administrative and Preliminary Investigation-Ombudsman

Roxas v. Vasquez GR 114944 June 19, 2001- Kaye

MANUEL C. ROXAS, et al. vs. CONRADO M. VASQUEZ, et al. [G.R. No. 114944, May 29, 2002]

FACTS:

Petitioner Roxas was the Chairman, while Nacpil was a Member, of the Bids
and Awards Committee of the Philippine Constabulary-Integrated National Police (PC-INP). The PC-INP
invited bids for the supply of sixty-five units of fire trucks. The Bids and Awards Committee voted to
award the contract to the Tahei Co., Ltd., manufacturer of Nikko-Hino. Accordingly, the contract was
executed between PC-INP and Tahei Co.

The COA subsequently discovered that there was a discrepancy in the amounts indicatedon the
disbursement voucher and the purchase order. Consequently, the DILG Secretary filed a
complaint with the Ombudsman against the respondents.

After preliminary investigation, the Deputy Ombudsman for the Military recommended the
indictment of all respondents, except Ramirez. On review, the Office of the Special
Prosecutor r e c o m m e n d e d t h e d i s m i s s a l o f t h e c o m p l a i n t s a g a i n s t R o x a s , N a c p i l , C
odoy, Kairan andRamirez. Formal charges were filed with the
S a n d i g a n b a y a n a g a i n s t N a z a r e n o , F l o r e s , Tanchanco, Custodio, Osia, Espeña and
Santos. Petitioners were not included in the criminalinformation. Flores and

Tanchanco moved for a reinvestigation, which was granted. Thereafter, the Office of the
Special Prosecutor recommended the dismissal of the charges against Flores and Tanchanco. In the
same resolution, however, the Special Prosecutor made a sudden turn about as regards Roxas, Nacpil
and Kairan, and ordered their inclusion as accused.

ISSUE:

Whether or not the inclusion of the petitioners as accused violated their right to due process.

HELD:

YES. It appears that the charge against respondents was previously dismissed. For this reason,
there being no motion or reconsideration filed by the complainant, said respondents ceased to be
parties. Consequently, the mere filing of motions for reconsideration by those previously indicted,
without questioning the dismissal of the charge against the said respondents, could not and should not
be made the basis for impleading them as accused in this case without violating their right to due
process. Furthermore, it appears that petitioners were deprived of due process when the Special
Prosecutor reinstated the complaint against them without their knowledge. Due process of law requires
that every litigant must be given an opportunity to be heard. He has the right to be present and defend
himself in person at every stage of the proceedings.

Ocampo v. Ombudsman 322 SCRA 17- Kaye


OCAMPO vs OFFICE OF THE OMBUDSMAN
FACTS:
Jesus Ocampo is the Training Coordinator of NIACONSULT, a subsidiary of the National Irrigation
Administration.
The Agricultural Development Bank of Nepal (ADBN) requested NIACONSULT for training of
small-scale community irrigation development, and Ocampo attended to the request.
o Ocampo requested an advance of 30% of training fees to the amount of US $9600 = PhP
204,960.
o He also accepted two instalments of the training fee for P61,488 and P143,472.
NIACONSULT demanded Ocampo to turn-over the total training fee paid by ADBN which
Ocampo personally received, but failed to remit the amount.
This prompted NIACONSULT to file an administrative case before the OMBUDSMAN.
o Ocampo failed to present a counter-affidavit amid being ordered twice by the OMBUDSMAN to
do so.
o The decretal portion of the Resolution stated that Ocampo was to be discharged from the
service, with forfeiture of special benefits and special perpetual disqualification to hold office in
government; without prejudice to any civil action NIACONSULT may institute to recover the amount
retained by Ocampo.
o Ocampo now assails lack of due process for not having been given the opportunity to file a
counter-affidavit and to present his evidence. [Bulk issue of the case, but not our concern in this topic.
The decision just says that he waived his right to due process when he failed to produce a counter-
affidavit amid being ordered twice to present his side]
While the case is pending, a criminal complaint for estafa and falsification was filed against
Ocampo based on the same facts or incidents. The Regional Trial Court DISMISSED the case.
OCAMPO’S CONTENTION: RTC’s dismissal of the criminal case serves as a bar to the
administrative case that can no longer stand on its own and therefore should be dismissed.
ISSUE/HELD:
Whether or not the dismissal of the criminal case affect the validity of the administrative case’s
resolution? NO.

RATIONALE:
The dismissal of the criminal case will not foreclose administrative action filed against petitioner
or give him a clean bill of health in all aspects.
On quantum of evidence: The RTC’s dismissal simply means that the prosecution was unable to
prove the guilt of petitioner beyond reasonable doubt. The lack or absence of proof beyond reasonable
doubt does not mean absence of any evidence whatsoever. The rule in administrative proceedings is
substantial evidence which merely requires such relevant evidence as a reasonable mind might accept
as adequate to support a conclusion. Considering this difference, one decision cannot be binding on the
other.

Serapio v. Sandiganbayan GR 148468 Jan. 28, 2003- Kaye


SERAPIO VS. SANDIGANBAYAN
396 SCRA 443

Facts: Petitioner Edward Serapio was a member of the Board of Trustees an the legal counsel of the Erap
Muslim Youth Foundation. Sometime 2000, petitioner received on its behalf a donation in the amount
of Php 200M through Chavit Singson. Petitioner received he donation worth the Foundation’s account.
In 2000, Chavit Singson publicly accused President Estrada and his family members and friends of
engaging in several illegal activities which triggered the filing with the Office of the Ombudsman several
criminal complaints against the petitioner, Joseph Estrada and his son.

On April 4, 2001, Ombudsman filed with the Sandiganbayan Informations against the former president,
one of which, for plunder. No bail was recommended for the provisional release of all the accused
including the petitioner. The case was raffled to a special division which was subsequently created by
the Supreme Court. On 25 April 2001, Sandiganbayan issued a resolution finding probable cause to
justify the issuance of warrants of arrest for the accused. Arraignment was set on 27 January 2001. In
the meantime, petitioner filed with Sandiganbayan an Urgent Petition for bail, which was set for hearing
on May 4, 2001. Petitioner’s co-accused Jinggoy Estrada filed a motion alleging that he was entitle to
bail as a matter of right.

During the hearing on May 4, 2001 on petitioner’s Urgent Petition for Bail, the prosecution moved for
the resetting of the arraignment of the accused earlier than the June 27 schedule. However,
Sandiganbayan denied the motion of the prosecution and issued an order declaring that the petition for
bail can and should be heard BEFORE petitioner’s arraignment on 27 June. On June 1, Sandiganbayan
issued a resolution requiring the attendance of petitioner as well as all the other accused during the
hearing on the petitioner for bail considering that under Section 8, Rule 115 of the Revised Rules of
Court, whatever evidence adduced during the hearing shall be considered automatically reproduced at
the trial.

The people insist that arraignment is necessary before bail hearings may be commenced because it is
only upon arraignment that the issues are joined. The people further stress the it is only when an
accused pleads not guilty may he filed a petition for bail and if he pleads guilty, then there would be no
need for him to file said petition. It is also the contention of the people that it is only during
arraignment that the accused is informed of the precise charge against him. He must then be arraign
first prior to bail hearings to prevent him from late on assailing the validity of the bail hearings on the
ground that he was not properly informed of the charge considering that under section 8 of Rule 114,
evidence presented during bail hearings are reproduce in the trial. Arraignment before bail hearings also
diminished the possibility of accused’s flight since trial in absentia may be had only if an accused escapes
after he has been arraigned.

However, the bail hearing again did not proceed because the petitioner filed with the information a
motion to quash the amended information on the grounds that as against him, the amended
information does not allege a combination of series of over or criminal acts constitutive of plunder.
According to the prosecution, the motion to quash the amended information was antithetical to his
petition for bail.

Petitioner also prays for the issuance of habeas corpus.

Issues:
(a) W/N petitioner should first be arraigned before hearings of his petition for bail may be
conducted.
(b) W/N petitioner may file a motion to quash the amended Information during the
pendency of his petition for bail.
(c) W/N a joint hearing of petition for bail for all the accused is mandatory
(d) W/N petitioner should instead be released through a writ of habeas corpus.
Decision:

(a) Although the petitioner was already arraigned, no plea has yet been entered thereby rendering the
issue of whether an arraignment is necessary before the conduct of bail hearings in the petitioner’s case
moot. Nonetheless, the court held that arraignment of an accused is not a pre-requisite to the conduct
of hearings on his petition for bail. A person is allowed to petition for bail as soon as he is deprived of
his of his liberty by virtue of his arrest or voluntary surrender.

In Lavides vs. CA, the court ruled that in cases where it is authorized, bail should be granted before
arraignment otherwise the accused may be precluded from filing a motion to quash. However, this
pronouncement should not be taken to mean that the hearing on a petition for bail should at all times
precede arraignment, because the rule is that a person deprived of his liberty by virtue of his arrest or
voluntary surrender may apply for bail as soon as he is deprived of liberty even before a complaint or
information is filed against him. The case of Lavides must be understood in light of the fact that the
accused in said case filed a petition for bail as well as a motion to quash. Hence, in that case, the court
held that to condition the grant of bail to an accused on his arraignment would be to place him in a
position where he had to choose between filing a motion to quash and thus delay his petition for bail
and forgoing the filing of the motion to quash so that he can be arraign at once ad therefore be released
on bail. Such would undermine the constitutional right of the accused.

When a bail is matter of right, an accused may apply for and be granted bail even prior to arraignment.
The Lavides case also implies that an application for bail in a case involving an offense punishable by
reclusion perpetua to death may also be heard even before an accused is arraigned. Sandiganbayan
therefore committed grave abuse of discretion amounting to excess of jurisdiction in ordering the
arraignment of petitioner before proceeding with the hearing of his petition for bail.

(b) Court dins no inconsistency exists between an application of an accused for bail and his filing of a
motion to quash. Bail, is the security given for the release of the person in custody of the law. A motion
to quash on the other hand is a mode by which an accused assails the validity of a criminal complain
filed against him for insufficiency on its fact in posit of law. These tow relied have objectives which are
not necessarily antithetical to each other. However, it is true that if a motion to quash a criminal
complaint or information on the ground that the same does not charge any offense is granted and the
case is dismissed and the accused is ordered released, the petition for bail of an accused may become
moot and academic.

(c) Petitioner argues that a joint bail hearing would negate his right to have his petition for bail resolved
in a summary proceeding since said hearing might be converted into a full blown trial. Prosecution on
the other hand claims that joint hearings will save the court form having to hear the same witnesses and
the parties from presenting the same evidences. There is no provision in the Rules of Court governing
the hearings of two or more petitioner for bail filed by different accused or that a petition for bail of an
accused be heard simultaneously with the trial of the case against the other accused. The matter should
be addressed to the sound discretion of the trial court. In the exercise of its discretion, the
Sandiganbayan must take into account not only the convenience of the sate, including the prosecution
but also that of the petitioner and the witnesses.

In the case of Ocampo vs. Bernabe, the court ruled that in a petition or bail hearing, the court is to
conduct only a summary hearing, meaning such brief and speedy method of receiving and considering
the evidence of guilt as is practicable and consistent with the purpose of the hearing which is early to
determine the weight of evidence for purposes of bail. The court does not try the merits or enter into
the inquiry as to the weight that ought to be given to the evidence against the accused, nor will it
speculate on the outcome of the trial or on what further such evidence as has reference to substantial
matters. In the case at bar, the case against former President Estrada is an entirely different matter.
For, with the participation of the former president in the hearing of petitioner’s petition for bail, the
proceeding assumes completely different dimension. The proceeding will no longer be summary since
the proceedings will be full blown which is antithetical to the nature of a bail hearing. The joinder of the
petitioner’s bail will be prejudicial to the petitioner as it will unduly delay the determination of the issue
of the right of petitioner to obtain provisional liberty and seek relief from his court. The Sandiganbayn
again committed a grave abuse of discretion in ordering a simultaneous hearing of petitioner’s petition
for bail with the trial of the case against former president.

(d) In the case at bar, bail is not matter of rights since the accused is charged with a capital offense, but
discretionary upon the court. Under Section 8 of rule 114, there must be a showing that the evidence of
guilt against a person charged with a capital offense is not strong for the court to grant him bail., thus,
upon an application for bail, by the person charged with a capital offense, a hearing must be conducted
where the prosecution has the burden of showing that the evidence of guilt against an accused is strong.
When the evidence of guilt is strong, bail becomes a matter of right, which is not so in the case at bar.

In exceptional cases, habeas corpus may be granted ny the courts even when the person concerned is
detained pursuant to a valid arrest or his voluntary surrender. The writ may be issued where the
deprivation of liberty while initially valid under the lad had not later become invalid. However, there is
no basis fir the issuance of the writ in the case at bar. The general rule is that the writ does not lie
where the person alleged to be restrained of his liberty is in the custody of an officer under process
issued by a court which had jurisdiction to issued the same applied, because petitioner is under
detention pursuant to the order of arrest. Petitioner in fact voluntarily surrendered himself to the
authorities.

9. Substantive Due Process

US v. Toribio – 15 Phil. 85- Kaye


US V TORIBIO

FACTS:
Sometime in the 1900s, Toribio applied for a license to have his carabao be slaughtered. His request was
denied because his carabao is found not to be unfit for work. He nevertheless slaughtered his carabao
without the necessary license. He was eventually sued and was sentenced by the trial court. His counsel
in one way or the other argued that the law mandating that one should acquire a permit to slaughter his
carabao is not a valid exercise of police power.

ISSUE: Whether or not the said law is valid.

HELD: The SC ruled against Toribio. The SC explained that it “is not a taking of the property for public
use, within the meaning of the constitution, but is a just and legitimate exercise of the power of the
legislature to regulate and restrain such particular use of the property as would be inconsistent with or
injurious to the rights of the publics. All property is acquired and held under the tacit condition that it
shall not be so used as to injure the equal rights of others or greatly impair the public rights and
interests of the community.”

DOCTRINE: To justify the public in interposing its authority to the public,


1. Interest of general public must be catered, and;

2. Means are reasonably necessary.

Churchill v. Rafferty – 32 Phil. 580- Kaye


CHURCHILL vs. RAFFERTY, G.R. NO. L-10572, December 21, 1915 ( 32 Phil 580)
Facts:
The case arises from the fact that defendant, Collector of Internal Revenue, would like to destroy or
remove any sign, signboard, or billboard, the property of the plaintiffs, for the sole reason that such
sign, signboard, or billboard is, or may be offensive to the sight. The plaintiffs allege otherwise.
Issue: Was there valid exercise of police power in this case?
Held:
Yes. There can be no doubt that the exercise of the police power of the Philippine Government belongs
to the Legislature and that this power is limited only by the Acts of Congress and those fundamentals
principles which lie at the foundation of all republican forms of government. An Act of the Legislature
which is obviously and undoubtedly foreign to any of the purposes of the police power and interferes
with the ordinary enjoyment of property would, without doubt, be held to be invalid. But where the Act
is reasonably within a proper consideration of and care for the public health, safety, or comfort, it
should not be disturbed by the courts.
"The power vested in the legislature by the constitution to make, ordain, and establish all manner of
wholesome and reasonable laws, statutes, and ordinances, either with penalties or without, not
repugnant to the constitution, as they shall judge to be for the good and welfare of the commonwealth,
and of the subjects of the same."
"The police power of the State, so far, has not received a full and complete definition. It may be said,
however, to be the right of the State, or state functionary, to prescribe regulations for the good order,
peace, health, protection, comfort, convenience and morals of the community, which do not ... violate
any of the provisions of the organic law."
"It [the police power] has for its object the improvement of social and economic conditioned affecting
the community at large and collectively with a view to bring about "he greatest good of the greatest
number."Courts have consistently and wisely declined to set any fixed limitations upon subjects calling
for the exercise of this power. It is elastic and is exercised from time to time as varying social conditions
demand correction."
"It may be said in a general way that the police power extends to all the great public needs. It may be
put forth in aid of what is sanctioned by usage, or held by the prevailing morality or strong and
preponderant opinion to be greatly and immediately necessary to the public welfare."
"It is much easier to perceive and realize the existence and sources of this police power than to mark its
boundaries, or to prescribe limits to its exercise."

DOCTRINE: Even the removal of billboards is valid exercise of police power if welfare of the general
public is the reason.

People v. Fajardo – 104 Phil. 443- Kaye


People v Fajardo G.R. No. L-12172 August 29, 1958

J. B. L . Reyes

Facts:
Fajardo was mayor in Baao, Camrines Sur when the municipal council passed the ordinance that
prohibits the construction of a building that blocks the view of the town plaza. Moreover, it redirects
the grant of permission to the mayor.
After his incumbency, Fajardo applied for a permit to build a building beside the gasoline stationnear the
town plaza. His request was repeatedly denied. He continued with the constructionunder the rationale
that he needed a house to stay in because the old one was destroyed by a typhoon.
He was convicted and ordered to pay a fine and demolish the building due to its obstructing view.
He appealed to the CA, which in turn forwarded the petition due to the question of the ordinance’s
constitutionality.

Issue: Is the ordinance constitutional?


Held: No, petition granted.

Ratio:
The ordinance doesn’t state any standard that limits the grant of power to the mayor. It is an arbitrary
and unlimited conferment.

Ordinances which thus invest a city council with a discretion which is purely arbitrary, and which may be
exercised in the interest of a favored few, are unreasonable and invalid. The ordinance should have
established a rule by which its impartial enforcement could be secured. All of the authorities cited above
sustain this conclusion.

The ordinance is unreasonable and oppressive, in that it operates to permanently deprive appellants of
the right to use their own property; hence, it oversteps the bounds of police power, and amounts to a
taking of appellants property without just compensation.

While property may be regulated to the interest of the general welfare, and the state may eliminate
structures offensive to the sight, the state may not permanently divest owners of the beneficial use of
their property and practically confiscate them solely to preserve or assure the aesthetic appearance of
the community.

Fajardo would be constrained to let the land be fallow and not be used for urban purposes. To do this
legally, there must be just compensation and they must be given an opportunity to be heard.

An ordinance which permanently so restricts the use of property that it can not be used for any
reasonable purpose goes, it is plain, beyond regulation and must be recognized as a taking of the
property.

The validity was also refuted by the Admin Code which states:
SEC. 2243. Certain legislative powers of discretionary character. — The municipal council shall have
authority to exercise the following discretionary powers:
xxx xxx xxx
(c) To establish fire limits in populous centers, prescribe the kinds of buildings that may be constructed
or repaired within them, and issue permits for the creation or repair thereof, charging a fee which shall
be determined by the municipal council and which shall not be less than two pesos for each building
permit and one peso for each repair permit issued. The fees collected under the provisions of this
subsection shall accrue to the municipal school fund.
Since, there was absolutely no showing in this case that the municipal council had either established fire
limits within the municipality or set standards for the kind or kinds of buildings to be constructed or
repaired within them before it passed the ordinance in question, it is clear that said ordinance was not
conceived and promulgated under the express authority of sec. 2243 (c)

DOCTRINE:
Where an ordinance deprives a private person of his right to use his own land, it amounts to taking of
property without just compensation.

Ermita-Malate Hotel & Operator v. City of Manila – 20 SCRA 849- Kaye


ERMITA MALATE HOTEL V CITY OF MANILA

Facts: On June 13, 1963, the Municipal Board of Manila passed Ordinance No. 4760 with the following
provisions questioned for its violation of due process:
refraining from entertaining or accepting any guest or customer unless it fills out a prescribed form in
the lobby in open view;
prohibiting admission o less than 18 years old;
usurious increase of license fee to P4,500 and 6,000 o 150% and 200% respectively (tax issue also);
making unlawful lease or rent more than twice every 24 hours; and
cancellation of license for subsequent violation.
The lower court issued preliminary injunction and petitioners raised the case to SC on certiorari.

Issue: Is the ordinance compliant with the due process requirement of the constitution?

Held: Ordinance is a valid exercise of police power to minimize certain practices hurtful to public morals.
There is no violation o constitutional due process for being reasonable and the ordinance is enjoys the
presumption of constitutionality absent any irregularity on its face. Taxation may be made to implement
a police power and the amount, object, and instance of taxation is dependent upon the local legislative
body. Judgment of lower court reversed and injunction lifted.
DOCTRINE: There is no precise definition for due process. Reason and justice must be taken thoroughly.

Ynot v. Intermediate Court of Appeals – 148 SCRA 659 - Xavier


FACTS: Executive Order 626-A was effected by President Marcos which prohibited interprovincial
movement of carabaos and also prohibiting the transportation of carabeef which shall be confiscated
and forfeited by the government to be distributed to charitable institutions.

The carabaos of petitioner were confiscated for violation of Executive Order No 626-A while he was
transporting them from Masbate to Iloilo. Petitioner challenged the constitutionality of Executive Order
No. 626-A. The government argued that Executive Order No. 626-A was issued in the exercise of police
power to conserve the carabaos that were still fit for farm work or breeding.
ISSUE: Whether EO No. 626-A is a violation of Substantive Due Process.
HELD: YES. Due process is violated because the owner of the property confiscated is denied the right
to be heard in his defense and is immediately condemned and punished. The executive order defined
the prohibition, convicted the petitioner and immediately imposed punishment, which was carried out
forthright. The measure struck at once and pounced upon the petitioner without giving him a chance to
be heard, thus denying him the centuries-old guaranty of elementary fair play.
Agustin v. Edu, 88 SCRA 195- Xavier
FACTS: Agustin is the owner of a Volkswagen Beetle Car. He is assailing the validity of Letter of
Instruction No. 229 which requires all motor vehicles to have early warning devices particularly to
equip them with a pair of “reflectorized triangular early warning devices”. Agustin says that cars are
already equipped with blinking lights which is already enough to provide warning to other motorists and
that the mandate to compel motorists to buy a set of reflectorized early warning devices is redundant
and would only make manufacturers and dealers instant millionaires.
ISSUE: Whether LOI No 229 violates the due process clause and is thus unconstitutional.
HELD: NO. The Letter of Instruction in question was issued in the exercise of police power. Justice Laurel
in Calalang v. Williams, identified police power with state authority to enact legislation that may
interfere with personal liberty or property in order to promote the general welfare. Persons and
property could thus "be subjected to all kinds of restraints and burdens in order to secure the general
comfort, health and prosperity of the state." The statute here questioned deals with a subject clearly
within the scope of the police power. As underlying questions of fact may condition the
constitutionality of legislation of this character, the presumption of constitutionality must prevail in the
absence of some factual foundation of record in overthrowing the statute.
Balacuit v. CFI – 163 SCRA 182- Xavier
FACTS: The Municipal Board of the City of Butuan enacted Ordinance No. 640 in 1961 to address the
complaint of parents that it is too financially burdensome for them to pay the full admission price at
any movie or other public exhibitions for their children. The Ordinance provided that admission should
be sold at half the price for children between 7 and 12 years of age. Violators would be penalized with
imprisonment and/or a fine. Petitioners Balacuit, Tan and Carcel are managers of theaters which were
affected by Ordinance 640 and they attack the validity and constitutionality of Ordinance No. 640 on the
grounds that it is ultra vires and an invalid exercise of police power, and violation of due process.
ISSUE: Whether the Ordinance violates the Constitutional safeguard of due process.
HELD: YES. A valid exercise of police power requires that it should be for (a) the public’s interest, (b) the
means employed should be reasonable and it should not be oppressive. As for the case at bar, the
court did not find a tangible link between the ordinance and the promotion of public health, security,
morals, or welfare. Furthermore, the means employed were judged to be unfair since they unjustly
prejudice the affected businesses by restraining their right to trade and even violating their right to
enter into contracts.
National Development Co. and New Agrix v. Phil. Vet. Bank – 192 SCRA 257- Xavier
FACTS: Agrix Marketing executed in favor of Respondent bank a real estate mortgage over three parcels
of land and later on went bankrupt. In order to rehabilitate the company, President Marcos issued PD
1717 which mandated, the extinguishing all of the mortgages and liens attaching to the property of
Agrix, and creating a Claims Committee to process claims against the company.
Respondent Bank filed a claim against the company before the Committee but, petitioners
invoked the provision of the law which cancels all mortgage liens against it. Thus, respondent claims PD
1717 as unconstitutional due to the impairment of obligation of contracts, and violation of the equal
protection clause.
ISSUE: Whether the PD 1717 is unconstitutional in as much as it proclaimed the extinguishment of all
mortgage and liens attached to any assets of the dissolved corporation which impairs the due process
clause.
HELD: YES. PD 1717 was held as unconstitutional on the ground that it was an invalid exercise of
police power. It had no lawful subject and no lawful method. It violated due process by extinguishing
all mortgages and liens and interests which are property rights unjustly taken. It also violated the
equal protection clause by lumping together all secured and unsecured creditors. It also impaired the
obligation of contracts, even though it only involved purely private interests.
Maranaw Hotel v. NLRC and Gina G. Castro – 238 SCRA 190- Xavier
FACTS: Private respondent Gina G. Castro was hired on a probationary basis for six months as a guest
relations officer of the Century Park Sheraton Hotel, a five-star hotel located at Malate, Manila, owned
by the petitioner. She was later dismissed on the ground of failure to meet the standards set forth in
her probationary employment contract. She then filed with the Arbitration Branch of the NCR - NLRC a
complaint for illegal dismissal with reinstatement, back wages, and damages against the hotel and its
former general manager, Peter Grieder. The Labor Arbiter (LA) rendered a Decision in favor of the
private respondent. NLRC reversed the decision of the LA and declared that there was no illegal
dismissal however, despite the tenor of its resolution, ordered the payroll reinstatement of the
employee pursuant to Article 223 of the Labor Code.
ISSUE: Whether the NLRC acted with grave abuse of discretion in ordering the payroll reinstatement of
an employee despite its resolution reversing the decision of the LA and declaring that there was no
illegal dismissal thereby violating due process.
HELD: YES. In the absence then of an order for the issuance of a writ of execution on the
reinstatement aspect of the decision of the Labor Arbiter, the petitioner was under no legal obligation
to admit back to work the private respondent under the terms and conditions prevailing prior to her
dismissal or, at the petitioner's option, to merely reinstate her in the payroll. An option is a right of
election to exercise a privilege, and the option in Article 223 of the Labor Code is exclusively granted to
the employer.
The event that gives rise for its exercise is not the reinstatement decree of a Labor Arbiter, but the
writ for its execution commanding the employer to reinstate the employee, while the final act which
compels the employer to exercise the option is the service upon it of the writ of execution when, instead
of admitting the employee back to his work, the employer chooses to reinstate the employee in the
payroll only. If the employer does not exercise this option, it must forthwith admit the employee back to
work, otherwise it may be punished for contempt.
Magtajas v. Pryce Properties – 234 SCRA 255- Xavier
FACTS: PAGCOR expanded to CDO City by leasing a portion of a building belonging to Pryce Properties
Corporation Inc. for its casino. CDO city enacted Ordinance 3375-93, prohibiting the operation of casino
and providing penalty for violation therefore. In the CA, Pryce assailed the ordinances which in effect
annul P.D. 1869 and are therefore invalid on that point, meanwhile, the Philippine Amusement and
Gaming Corporation (PAGCOR) invoke P.D. 1869 which created PAGCOR to help centralize and regulate
all games of chance, including casinos on land and sea within the territorial jurisdiction of the
Philippines.
The CA ruled in favor of the respondents and found the ordinances invalid. CDO City and its mayor
filed a petition for review under Rules of Court with the Supreme Court. The Petitioners argue that the
Sangguniang Panlungsod may prohibit the operation of casinos because they involve games of chance,
which are detrimental to the people.
ISSUE: Whether the Sangguniang Panlungsod exercising police power under what is known as the
General Welfare Clause may prohibit the establishment of casino operated by PAGCOR through an
ordinance or resolution
HELD: NO. Firstly, the morality of gambling is not a justiciable issue. Gambling is not illegal per se. While
it is generally considered inimical to the interests of the people, there is nothing in the Constitution
categorically proscribing or penalizing gambling or, for that matter, even mentioning it at all.

The tests of a valid ordinance are well established. A long line of decisions has held that to be valid, an
ordinance must conform to the following substantive requirements:
1) It must not contravene the constitution or any statute.
2) It must not be unfair or oppressive.
3) It must not be partial or discriminatory.
4) It must not prohibit but may regulate trade.
5) It must be general and consistent with public policy.
6) It must not be unreasonable.

The rationale of the requirement that the ordinances should not contravene a statute is
obvious. Municipal governments are only agents of the national government. Local councils exercise
only delegated legislative powers conferred on them by Congress as the national lawmaking body. The
delegate cannot be superior to the principal or exercise powers higher than those of the latter. It is a
heresy to suggest that the local government units can undo the acts of Congress, from which they have
derived their power in the first place, and negate by mere ordinance the mandate of the statute.
Bennis v. Michigan – No. 94-8729 March 4, 1996- Xavier
FACTS: Tina B. Bennis was a joint owner, with her husband, of an automobile in which her husband
engaged in sexual activity with a prostitute. In declaring the automobile forfeit as a public nuisance
under Michigan's statutory abatement scheme, the trial court permitted no offset for petitioner's
interest, notwithstanding her lack of knowledge of her husband's activity.
The Michigan Court of Appeals reversed, but was in turn reversed by the State Supreme Court,
which concluded, among other things, that Michigan's failure to provide an innocent-owner defense
was without federal constitutional consequence under this Court's decisions.
ISSUE: Whether the innocent owner defense is applicable to herein petitioner.
HELD: The United States Supreme Court held that innocent owner defense is not constitutionally
mandated by Fourteenth Amendment Due Process in cases of civil forfeiture.
Cruzan v. Dir. Missouri – No. 88-1503 June 25 1990- Xavier
FACTS: Petitioner Nancy Cruzan is in coma and vegetative state due to an accident. Her parents
requested to terminate her artificial nutrition and hydration, however the Hospital (Missouri)
employees refused, without court approval, to honor the request, since that would result in death.
A state trial court authorized the termination, based on Cruzan's expression to a former
housemate that she would not wish to continue her life if sick or injured unless she could live at least
halfway normally suggested that she would not wish to continue on with her nutrition and hydration.
The State Supreme Court reversed the trial court and decided that the State Living Will statute
embodied a state policy strongly favoring the preservation of life, and that Cruzan's statements to her
housemate were unreliable for the purpose of determining her intent. It rejected the argument that her
parents were entitled to order the termination of her medical treatment, concluding that no person can
assume that choice for an incompetent in the absence of the formalities required by the Living Will
statute or clear and convincing evidence of the patient's wishes.
ISSUE: Whether Missouri’s procedural requirement for clear and convincing evidence of an incompetent
person’s desire to terminate life support before it is terminated violate the constitutionally protected
liberty interest in refusing medical treatment under the Due Process Clause.
HELD: NO. The right to terminate life-sustaining treatment of an incompetent, if it is to be exercised,
must be done for such incompetent by a surrogate. Missouri’s interest in the preservation of life is
unquestionably a valid State interest.
The Due Process Clause protects an interest in life as well as a right to refuse life-saving
treatment. Missouri may legitimately safeguard these personal decisions by imposing heightened
evidentiary requirements. Moreover, even when available, family members will not always act in the
best interests of a patient. The State is entitled to safeguard against such abuses.
JMM Promotion and Management Inc. v. CA – 260 SCRA 319- Xavier
FACTS: The deployment of female entertainers to Japan was controlled by the government through
Department Order No. 3, where said entertainers were required an Artist Record Book as a
precondition to the processing by the POEA of any contract for overseas employment. Petitioners
contends that overseas employment is a property right within the meaning of the Constitution and avers
that the alleged deprivation thereof through the onerous requirement of an ARB violates due process
and constitutes an invalid exercise of police power.
ISSUE: Whether an Artist Record Book (ARB) violates due process and constitutes an invalid exercise of
police power.
HELD: NO. The ARB requirement and the questioned Department order related to its issuance were
issued pursuant to a valid exercise of police power which considers the welfare of Filipino performing
artists, particularly the women. It cannot be gainsaid that this scheme at least lessens the room for
exploitation by unscrupulous individuals and agencies.
Corona v. United Harbor – 283 SCRA 31- Xavier
FACTS: The United Harbor Philippine Pilots, Inc. was created through PD 505 on 1974 with the power to
control, regulate and supervise pilots and the pilotage profession. United Harbor, respondent, passed
its Rules and Regulations regarding Pilotage Services and Fees on 1985, which states among others that
aspiring pilots must achieve proper pilot license and undergo probationary pilot training for three
months in outports and four months in the port of Manila. After such time shall PPA issue permanent
appointments to such pilots. Petitioner questions the constitutionality of this promulgated Rules and
Regulations on the ground that it deprives one of pilotage which is an alleged property right.
ISSUE: Whether AO 04-92 (Rules and Regulations) issued by the PPA unduly violates property rights of
pilotage in violation of Section I of the Bill of Rights.
HELD: NO. The questioned AO does not wrongfully deprived one of the right to pilotage but it merely
regulates the profession of pilotage by providing certain limits to issuance of permanent
appointments to pilots. This limitations are for the benefit of the public who relies on the competence
of such pilots.
Kelly v. Johnson – 425 US 238- Xavier
FACTS: Kelly is a police officer and member of the United States Police Force. He questions a local
regulation requiring police officers to follow a uniform style and length of hair cut for being an
infringement of his liberty in accordance with the Fourteenth Amendment. The lower court sustained
his contention stating that the police department failed to identify a public need that calls for such
uniformity of hair length and styles to members of the police force.
ISSUE: Whether a regulation on the hair styles and cuts of police officers is unconstitutional.
HELD: NO. Members of the police force are not simply ordinary citizens. They are agents of justice and
bringers of immediate help to the members of the citizenry. The need for uniform hairstyle and haircut
as iterated in the assailed local regulation is rooted on this need for easy recognition for the benefit of
the public at large. The need of the public is of greater weight compared to the liberty of police
officers, the servants of the people, to choose their own hairstyles or haircuts.
Chavez v. Romulo – 431 SCRA 534 (2004)- Xavier
FACTS: Petioner Chavez as a licensed gun-owner and holder of a Permit to Carry Firearms Outside of
Residence (PTCFOR) assails the constitutionality of guidelines issued by respondent PNP Chief Ebdane
that banned the carrying of firearms outside ones residence on the ground that the owning and
carrying a gun is inherent to ones right to life and protection and the questioned guidelines takes away
such rights without due process and without just cause. Chief Ebdane issued the said guidelines in line
with President Gloria Arroyo’s speech before members of the PNP wherein she reiterated the need to
suspend issuances of PTCFOR to meet the necessary gun ban and avert the rising crime incidents in the
country.
ISSUE: Whether Permit to Carry Firearms Outside of Residence (PTCFOR) represent an absolute right
whose deprivation constitutes a violation of due process.
HELD: NO. It is not a contract nor is it right and a revocation of which does not unduly deprive one of
his rights in the purview of the Bill of Rights. A license such as the PTCFOR is a mere privilege granted to
the license holders subject to conditions and limitations that may be reasonably imposed, and which
may be revoked any time at the pleasure of the one issuing the same.
Cruz v. Flavier, GR 135385, December 6, 2000- Xavier
FACTS: Petitioners question the provisions of the IPRA defining the powers and jurisdiction of the NCIP
and making customary law applicable to the settlement of disputes involving ancestral domains and
ancestral lands on the ground that these provisions violate the due process clause of the Constitution.
ISSUE: Whether the provisions of the IPRA defining the powers and jurisdiction of the NCIP and making
customary law applicable to the settlement of disputes involving ancestral domains and ancestral lands
on the ground that these provisions violate the due process clause of the Constitution.
HELD: Petition Dismissed IPRA constitutional. 7-7 decision. Justice Panganiban reserves judgment on the
constitutionality of Sections 58, 59, 65, and 66 of the law, which he believes must await the filing of
specific cases by those whose rights may have been violated by the IPRA.

POSITIVE OPINION. Ancestral land within the ancestral domain does not belong to the State. However,
the natural resources belong to the State. Indigenous people only have priority right for exploration and
conservation.
OPPOSITE OPINION. The State must have full control in the development of the natural resources. The
IPRA provided that all ancestral domain includes natural resources. This is unconstitutional.
Smith Kline v. CA, GR 121267, October 23, 2001- Xavier
FACTS: Petitioner Smith Kline Beckman Corporation sued private respondent Tryco Pharma Corporation
for infringement of patent and unfair competition. Petitioner argues that under the doctrine of
equivalents for determining patent infringement, Albendazole, the active ingredient it alleges was
appropriated by private respondent for its drug Impregon, is substantially the same as methyl 5
propylthio-2-benzimidazole carbamate covered by its patent since both of them are meant to combat
worm or parasite infestation in animals. It cites the “unrebutted” testimony of its witness Dr.
Godofredo C. Orinion (Dr. Orinion) that the two substances substantially do the same function in
substantially the same way to achieve the same results, thereby making them truly identical.
Petitioner thus submits that the appellate court should have gone beyond the literal wordings
used in Letters Patent No. 14561, beyond merely applying the literal infringement test, for in spite of the
fact that the word Albendazole does not appear in petitioner’s letters patent, it has ably shown by
evidence its sameness with methyl 5 propylthio-2-benzimidazole carbamate.
ISSUE: Whether private respondent committed patent infringement to the prejudice of petitioner.
HELD: NO. The doctrine of equivalents thus requires satisfaction of the function-means-and-result test,
the patentee having the burden to show that all three components of such equivalency test are met.
Yet again, a scrutiny of petitioner’s evidence fails to convince this Court of the substantial
sameness of petitioner’s patented compound and Albendazole. While both compounds have the effect
of neutralizing parasites in animals, identity of result does not amount to infringement of patent unless
Albendazole operates in substantially the same way or by substantially the same means as the patented
compound, even though it performs the same function and achieves the same result. In other
words, the principle or mode of operation must be the same or substantially the same.
Parreño v. COA 523 SCRA 390- Xavier
FACTS: Petitioner Parreño filed his money claim before the COA contesting the constitutionality of
Section 27 of PD 1638 or the AFP Military Personnel Retirement and Separation Decree as amended by
PD 1650, which provides that a retiree who loses his Filipino citizenship shall be removed from the
retired list and consequently have his retirement benefits terminated.
ISSUE: Whether the obligation imposed on petitioner to retain his Filipino citizenship as a condition for
him to remain in the AFP retired list and receive his retirement benefit is contrary to public policy and
welfare, oppressive, discriminatory, and violative of the due process clause of the Constitution.
HELD: The constitutional right to equal protection of the laws is not absolute but is subject to
reasonable classification. To be reasonable, the classification
(a) must be based on substantial distinctions which make real differences;
(b) must be germane to the purpose of the law;
(c) must not be limited to existing conditions only; and
(d) must apply equally to each member of the class.
There is compliance with all these conditions. There is a substantial difference between
retirees who are citizens of the Philippines and retirees who lost their Filipino citizenship by
naturalization in another country, such as petitioner in the case before us. Petitioner’s loss of Filipino
citizenship constitutes a substantial distinction that distinguishes him from other retirees who retain
their Filipino citizenship. If the groupings are characterized by substantial distinctions that make real
differences, one class may be treated and regulated differently from another.
Esponcilla v. Bagong Tanyag G.R. No. 151019 529 SCRA 654- Xavier
FACTS: Under the Community Mortage Program (CMP), BATAHAI obtained a loan from National Home
Mortgage Finance Corporation (NHMFC) to purchase the land from its owners and subdivide it among its
member-beneficiaries, subject to the supervision and guidance of the National Housing Administration
(NHA). Petitioners refused to submit the documents-requirements of the NHMFC thus they were
delisted as beneficiaries of BATAHAI.
Petitioners, former members of BATAHAI, alleged that respondents Bagong Tanyag
Homeowners’ Association, Inc.(BATAHAI) subdivided the lots which they have been occupying since
1978 without their knowledge and consent, reassigned the lots without observing due process of law,
omitted or deleted their names from the certified list of prospective beneficiaries, and unlawfully
replaced three BATAHAI directors.
ISSUE: Whether petitioners were deprived of property without due process of law.
HELD: NO. The essence of due process is the opportunity to be heard. What the law prohibits is not the
absence of previous notice but the absolute absence thereof and the lack of opportunity to be
heard. The records of the case show that petitioners had had more than sufficient notice and
opportunity to be heard before they were delisted as prospective beneficiaries.
Before respondents issued the questioned list of prospective beneficiaries, sufficient notices
were posted informing petitioners of the need to submit the documents required by the NHMFC. Even
after petitioners were delisted as beneficiaries, respondents set new deadlines for petitioners to submit
the requirements, sending each of them letters reminding them of the consequences of non-compliance
therewith.
BF v. City Mayor Of Parañaque City et. al 515 SCRA 1- Xavier
FACTS: Petitioners BF Homes Parañaque, several homeowners’ associations, and residents of BF Homes
Parañaque, the largest subdivision in the country, alleged that the reclassification of certain portions of
BF Homes Parañaque from residential to commercial zone is unconstitutional because it amounts to
impairment of the contracts between the developer of BF Homes Parañaque and the lot buyers.
Petitioners cited the annotation on the lot buyers’ titles which provides that "the property shall be used
for residential purposes only and for no other purpose."
On the other hand, public respondents alleged that the passage of Municipal Ordinance No. 97-
08 is a valid exercise of police power by the Municipal Council of Parañaque and that such ordinance can
nullify or supersede the contractual obligations entered into by the petitioners and the developer.
ISSUE: Whether the reclassification of certain portions of BF Homes Parañaque from residential to
commercial zone is unconstitutional because it amounts to impairment of the contracts.
HELD: NO. While non-impairment of contracts is constitutionally guaranteed, the rule is not absolute,
since it has to be reconciled with the legitimate exercise of police power, i.e., "the power to prescribe
regulations to promote the health, morals, peace, education, good order or safety and general welfare
of the people." Invariably described as "the most essential, insistent, and illimitable of powers" and "in a
sense, the greatest and most powerful attribute of government," the exercise of the power may be
judicially inquired into and corrected only if it is capricious, whimsical, unjust or unreasonable, there
having been a denial of due process or a violation of any other applicable constitutional guarantee.
Similarly, in this case, Municipal Ordinance No. 97-08 is a legitimate exercise of police power and the
reclassification of El Grande and Aguirre Avenues in BF Homes Parañaque is not arbitrary or
unreasonable.
St. Luke’s v. NLRC 517 SCRA 677- Xavier
FACTS: The private respondent Maribel Santos worked as an X-Ray technician at the petitioner hospital
(SLMC) but she does not possess a certificate of registration as required under the newly passed
Radiologic Act or RA 7431. Due to her non-compliance and her failure to pass the exams, she was
separated.
The private respondent filed a complaint for illegal dismissal and non-payment of salaries and
other monetary benefits. The Labor Arbiter ordered the petitioner to pay respondent separation pay
and this was affirmed by both NLRC and the Court of Appeals, hence this petition. The petitioner
contended that respondent dismissal was valid.
ISSUE: Whether the separation from work for failure to comply with the certification requirement
violated her constitutional right to security of tenure.
HELD: NO. No malice or ill-will can be imputed upon private respondent as the separation of petitioner
Santos was undertaken by it conformably to an existing statute. This notwithstanding, The records bear
out the fact that petitioner Santos was given ample opportunity to qualify for the position and was
sufficiently warned that her failure to do so would result in her separation from work in the event there
were no other vacant positions to which she could be transferred. Despite these warnings, petitioner
Santos was still unable to comply and pass the required exam. To reiterate, the requirement for Board
certification was set by statute. Justice, fairness and due process demand that an employer should not
be penalized for situations where it had no participation or control.

Carlos v. DSWD 526 SCRA 130 G.R. No. 166494 - JC

Facts: Petitioners are domestic corporations and proprietors operating drugstores in the Philippines.
They question Sec 4 of RA 9257(Expanded Senior Citizen’s Act) as it grants a 20% discount for senior
citizens and insofar as it grans them a tax deduction on their gross sales receipts per taxable year.

Issue: Whether the law constitutes deprivation of property

Held: No. The permanent reduction in their total revenues is a forced subsidy corresponding to the
taking of private property for public use or benefit. This constitutes compensable taking for which
petitioners would ordinarily become entitled to a just compensation.

The law is a legitimate exercise of police power which, similar to the power of eminent domain, has
general welfare for its object. Police power is not capable of an exact definition, but has been purposely
veiled in general terms to underscore its comprehensiveness to meet all exigencies and provide enough
room for an efficient and flexible response to conditions and circumstances, thus assuring the greatest
benefits. Accordingly, it has been described as “the most essential, insistent and the least limitable of
powers, extending as it does to all the great public needs.” It is “[t]he power vested in the legislature by
the constitution to make, ordain, and establish all manner of wholesome and reasonable laws, statutes,
and ordinances, either with penalties or without, not repugnant to the constitution, as they shall judge
to be for the good and welfare of the commonwealth, and of the subjects of the same.”
For this reason, when the conditions so demand as determined by the legislature, property rights must
bow to the primacy of police power because property rights, though sheltered by due process, must
yield to general welfare.

Perez v. LPG 531 SCRA 431 G.R. No. 159149 - JC

Facts: BP Blg. 33, as amended, penalizes illegal trading, hoarding, overpricing, adulteration,
underdelivery, and underfilling of petroleum products, as well as possession for trade of adulterated
petroleum products and of underfilled liquefied petroleum gas (LPG) cylinders. The said law sets the
monetary penalty for violators to a minimum of P20,000 and a maximum of P50,000. DOE issued a
circular which included penalties not included in the law.
The trial court nullified the Circular on the ground that it introduced new offenses not included in the
law. The court intimated that the Circular, in providing penalties on a per cylinder basis for each
violation, might exceed the maximum penalty under the law. Respondent maintains that the Circular is
not in conformity with the law it seeks to implement. Petitioner argues that the penalties for the acts
and omissions enumerated in the Circular are B.P. Blg. 33 and Republic Act No. 8479. Petitioner adds
that Republic Act No. 7638 also authorize the DOE to impose the penalties provided in the Circular.

Issue: Whether the circular is void as it introduced new offenses not contemplated in the law

Held: No. For an administrative regulation, such as the Circular in this case, to have the force of penal
law, (1) the violation of the administrative regulation must be made a crime by the delegating statute
itself; and (2) the penalty for such violation must be provided by the statute itself.

These specific acts and omissions are obviously within the contemplation of the law, which seeks to curb
the pernicious practices of some petroleum merchants. However, for the refillers, marketers,
and dealers, the Circular is silent as to any maximum monetary penalty. This mere silence, nonetheless,
does not amount to violation of the aforesaid statutory maximum limit. Further, the mere fact that the
Circular provides penalties on a per cylinder basis does not in itself run counter to the law since all that
B.P. Blg. 33 prescribes are the minimum and the maximum limits of penalties.

MMDA v. Viron 530 SCRA 341 GR No. 170656 - JC

Facts: Due to traffic congestion, the MMDA, through EO 179, was tasked to undertake measures to ease
traffic congestion. MMDA recommended a plan which involved the closure of bus terminals located
along major thoroughfares. Respondents maintain that it is tantamount to deprivation of their
constitutional right to property without due process of law.

Issue: Whether the EO violated their right to due process

Held: Yes. E.O. No. 179 is declared NULL and VOID for being ultra vires. MMDA’s exercise of such power
does not satisfy the two tests of a valid police power measure, viz: (1) the interest of the public
generally, as distinguished from that of a particular class, requires its exercise; and (2) the means
employed are reasonably necessary for the accomplishment of the purpose and not unduly oppressive
upon individuals.44 Stated differently, the police power legislation must be firmly grounded on public
interest and welfare and a reasonable relation must exist between the purposes and the means.

MMDA cannot order the closure of respondents’ terminals not only because no authority to implement
the Project has been granted nor legislative or police power been delegated to it, but also because the
elimination of the terminals does not satisfy the standards of a valid police power measure.

Sec. of DND v. Manalo 568 SCRA 42 (Amparo) October 7, 2008 - JC

Facts: Suspected of being NPA members, the Manalo brothers were abducted and tortured. This case
was originally a Petition for Prohibition, Injunction, and Temporary Restraining Order (TRO) filed before
this Court by herein respondents on August 23, 2007 to stop herein petitioners and/or their officers and
agents from depriving them of their right to liberty and other basic rights. Respondents also sought
ancillary remedies, Protective Custody Orders, Appointment of Commissioner, Inspection and Access
Orders, and all other legal and equitable reliefs under Article VIII, Section 5(5) of the 1987 Constitution
and Rule 135, Section 6 of the Rules of Court.

While the August 23, 2007 Petition was pending, the Rule on the Writ of Amparo took effect on October
24, 2007. Forthwith, therein petitioners filed a Manifestation and Omnibus Motion to Treat Existing
Petition as Amparo Petition, to Admit Supporting Affidavits, and to Grant Interim and
Final Amparo Reliefs. They prayed that: (1) the petition be considered a Petition for the Writ
of Amparo under Sec. 26 of the Amparo Rule; (2) the Court issue the writ commanding therein
respondents to make a verified return within the period provided by law and containing the specific
matter required by law; (3) they be granted the interim reliefs allowed by the Amparo Rule and all other
reliefs prayed for in the petition but not covered by the Amparo Rule; (4) the Court, after hearing,
render judgment as required in Sec. 18 of the Amparo Rule; and (5) all other just and equitable reliefs.

Issue: Whether actual deprivation of liberty is necessary for the right to security of a person may be
invoked.

Held: Deprivation of liberty is not necessary before the right to security may be invoked –While the
right to security of person appears in conjunction with the right to liberty under Article 9, the
Committee has ruled that the right to security of person can exist independently of the right to liberty.
In other words, there need not necessarily be a deprivation of liberty for the right to security of person
to be invoked.

First, the right to security of person is “freedom from fear.” In its “whereas” clauses, the Universal
Declaration of Human Rights (UDHR) enunciates that “a world in which human beings shall enjoy
freedom of speech and belief and freedom from fear and want has been proclaimed as the highest
aspiration of the common people.” (emphasis supplied) Some scholars postulate that “freedom from
fear” is not only an aspirational principle, but essentially an individual international human right. It is the
“right to security of person” as the word “security” itself means “freedom from fear.”

Second, the right to security of person is a guarantee of bodily and psychological integrity or
security. Article III, Section II of the 1987 Constitution guarantees that, as a general rule, one’s body
cannot be searched or invaded without a search warrant. Physical injuries inflicted in the context of
extralegal killings and enforced disappearances constitute more than a search or invasion of the body. It
may constitute dismemberment, physical disabilities, and painful physical intrusion. As the degree of
physical injury increases, the danger to life itself escalates. Notably, in criminal law, physical injuries
constitute a crime against persons because they are an affront to the bodily integrity or security of a
person.

Third, the right to security of person is a guarantee of protection of one’s rights by the government. In
the context of the writ of amparo, this right is built into the guarantees of the right to life and
liberty under Article III, Section 1 of the 1987 Constitution and the right to security of person (as
freedom from threat and guarantee of bodily and psychological integrity) under Article III, Section
2. The right to security of person in this third sense is a corollary of the policy that the State “guarantees
full respect for human rights” under Article II, Section 11 of the 1987 Constitution. As the government is
the chief guarantor of order and security, the Constitutional guarantee of the rights to life, liberty and
security of person is rendered ineffective if government does not afford protection to these rights
especially when they are under threat. Protection includes conducting effective investigations,
organization of the government apparatus to extend protection to victims of extralegal killings or
enforced disappearances (or threats thereof) and/or their families, and bringing offenders to the bar of
justice.
SJS v. DDB 570 SCRA 410 - JC

Facts: RA 9165 or the Dangerous Drugs Act of 2002 was passed; petitioner question Sec 36 insofar as it
requires mandatory drug testing of candidates for public office, students of secondary and tertiary
schools, officers and employees of public and private offices, and persons charged before the
prosecutor's office with certain offenses, among other personalities.

Issue: Whether petitioner’s constitutional rights are violated

Held: As to the candidates: legislative power remains limited in the sense that it is subject to substantive
and constitutional limitations which circumscribe both the exercise of the power itself and the allowable
subjects of legislation. The substantive constitutional limitations are chiefly found in the Bill of
Rights and other provisions, such as Sec. 3, Art. VI of the Constitution prescribing the qualifications of
candidates for senators.

As to students and officers/employees of public/private offices: the provisions of RA 9165 requiring


mandatory, random, and suspicionless drug testing of students are constitutional. Indeed, it is within the
prerogative of educational institutions to require, as a condition for admission, compliance with
reasonable school rules and regulations and policies. To be sure, the right to enroll is not absolute; it is
subject to fair, reasonable, and equitable requirements.
SJS v. Atienza 545 SCRA 92 G.R. No. 156052 - JC

Facts: Ordinance No. 8027 was enacted pursuant to the police power delegated to local government
units; it reclassified certain areas from industrial to commercial and directed the owners and operators
of business to cease and desist from operating their business within 6 months. City of Manila and the
DOE entered into a MOU with the oil companies in which they agreed that "the scaling down of the
Pandacan Terminals was the most viable and practicable option. SJS contends that the enforcement of
the ordinance is a legal mandatory duty to remove the oil terminals instead of allowing them to stay.

Issue: Whether the ordinance violated their right to property

Held: No. Essentially, the oil companies are fighting for their right to property. They allege that they
stand to lose billions of pesos if forced to relocate. However, based on the hierarchy of constitutionally
protected rights, the right to life enjoys precedence over the right to property. The reason is obvious:
life is irreplaceable, property is not. When the state or LGU’s exercise of police power clashes with a few
individuals’ right to property, the former should prevail.

Both law and jurisprudence support the constitutionality and validity of Ordinance No. 8027. Without a
doubt, there are no impediments to its enforcement and implementation. Any delay is unfair to the
inhabitants of the City of Manila and its leaders who have categorically expressed their desire for the
relocation of the terminals. Their power to chart and control their own destiny and preserve their lives
and safety should not be curtailed by the intervenors warnings of doomsday scenarios and threats of
economic disorder if the ordinance is enforced.
SEC v. Interport 567 SCRA 354 - JC

Facts: The Board of Directors of IRC approved a MOA with Ganda Holdings Berhad (GHB). This MOA was
transmitted through facsimile to the Philippine Stock Exchange and SEC announcing the approval of the
agreement. The SEC averred that it had received reports that IRC failed to disclose its negotiations and
that its directors heavily traded IRC shares utilizing material insider information. It also claimed that SEC
had violated their right to due process when it ordered respondent to show cause why no civil, criminal
or administrative sanction should be imposed on them and thus shifted the burden of proof to IRC when
it ordered that the matter be referred to a special investigating panel.

Issue: Whether the creation the of panel violated their right to due process specifically the right to cross
- examine

Held: No. PED Rules provided that the Hearing Officer may require the parties to submit their respective
verified position papers, together with all supporting documents and affidavits of witnesses. A formal
hearing was not mandatory; it was within the discretion of the Hearing Officer to determine whether
there was a need for a formal hearing. Since, according to the foregoing rules, the holding of a hearing
before the PED is discretionary, then the right to cross-examination could not have been demanded by
either party.
People v. Siton 600 SCRA 476 - JC

Facts: Respondents were arrested via Art 202 of the RPC for vagrancy. Instead of filing an answer, they
challenged its constitutionality on the ground that it discriminates against the poor.

Issue: Whether Art 202 violates the right to equal protection

Held: No. Article 202 (2) does not violate the equal protection clause; neither does it discriminate
against the poor and the unemployed. Offenders of public order laws are punished not for their status,
as for being poor or unemployed, but for conducting themselves under such circumstances as to
endanger the public peace or cause alarm and apprehension in the community. Being poor or
unemployed is not a license or a justification to act indecently or to engage in immoral conduct.

It must not be forgotten that police power is an inherent attribute of sovereignty. It has been defined
as the power vested by the Constitution in the legislature to make, ordain, and establish all manner of
wholesome and reasonable laws, statutes and ordinances, either with penalties or without, not
repugnant to the Constitution, as they shall judge to be for the good and welfare of the commonwealth,
and for the subjects of the same. The power is plenary and its scope is vast and pervasive, reaching and
justifying measures for public health, public safety, public morals, and the general welfare. As an
obvious police power measure, Article 202 (2) must therefore be viewed in a constitutional light.
White Light v. City of Manila 576 SCRA 416 - JC

Facts: On 3 Dec 1992, then Mayor Lim signed into law Ord 7774 entitled “An Ordinance” prohibiting
short time admission in hotels, motels, lodging houses, pension houses and similar establishments in the
City of Manila. White Light Corp is an operator of mini hotels and motels who sought to have the
Ordinance be nullified as the said Ordinance infringes on the private rights of their patrons. The RTC
ruled in favor of WLC. It ruled that the Ordinance strikes at the personal liberty of the individual
guaranteed by the Constitution. The City maintains that the ordinance is valid as it is a valid exercise of
police power. Under the LGC, the City is empowered to regulate the establishment, operation and
maintenance of cafes, restaurants, beerhouses, hotels, motels, inns, pension houses, lodging houses and
other similar establishments, including tourist guides and transports. The CA ruled in favor of the City.

Issue: Whether the ordinance is violative of due process

Held: Yes. The SC ruled that the said ordinance is null and void as it indeed infringes upon individual
liberty. It also violates the due process clause which serves as a guaranty for protection against arbitrary
regulation or seizure. The said ordinance invades private rights. Note that not all who goes into motels
and hotels for wash up rate are really there for obscene purposes only. Some are tourists who needed
rest or to “wash up” or to freshen up. Hence, the infidelity sought to be avoided by the said ordinance is
more or less subjected only to a limited group of people. The SC reiterates that individual rights may be
adversely affected only to the extent that may fairly be required by the legitimate demands of public
interest or public welfare.

We recognize the capacity of the petitioners to invoke as well the constitutional rights of their patrons –
those persons who would be deprived of availing short time access or wash-up rates to the lodging
establishments in question.
CREBA v. Romula 614 SCRA 605 - JC

Facts: Petitioner is an association of real estate developers and builders in the Philippines. It impleaded
former Executive Secretary Alberto Romulo, then acting Secretary of Finance Juanita D. Amatong and
then Commissioner of Internal Revenue Guillermo Parayno, Jr. as respondents.

Petitioner assails the validity of the imposition of minimum corporate income tax (MCIT) on
corporations and creditable withholding tax (CWT) on sales of real properties classified as ordinary
assets.

Section 27(E) of RA 8424 provides for MCIT on domestic corporations and is implemented by RR 9-98.
Petitioner argues that the MCIT violates the due process clause because it levies income tax even if
there is no realized gain.

Issue: Whether the CWT is violative of due process and equal protection of the laws

Held: No. CWT is creditable against the tax due from the seller of the property at the end of the taxable
year. The seller will be able to claim a tax refund if its net income is less than the taxes withheld.
Nothing is taken that is not due so there is no confiscation of property repugnant to the constitutional
guarantee of due process. More importantly, the due process requirement applies to the power to
tax. The CWT does not impose new taxes nor does it increase taxes. It relates entirely to the method
and time of payment.

All persons belonging to the same class shall be taxed alike. It follows that the guaranty of the equal
protection of the laws is not violated by legislation based on a reasonable classification. Classification, to
be valid, must (1) rest on substantial distinctions; (2) be germane to the purpose of the law; (3) not be
limited to existing conditions only and (4) apply equally to all members of the same class.

The taxing power has the authority to make reasonable classifications for purposes of taxation.
Inequalities which result from a singling out of one particular class for taxation, or exemption, infringe
no constitutional limitation.8= The real estate industry is, by itself, a class and can be validly treated
differently from other business enterprises.
Southern Hemisphere v. ATC 632 SCRA 146 - JC

Facts: Petitioners assail the constitutionality of RA 9372 or the Human Security Act of 2007

Issue: Whether RA 9372 is violative of due process as it impairs free speech

Held: No. RA 9372 seeks to regulate conduct, not speech. Facial challenge involving the void for
vagueness and overbreadth doctrines can only be applied to free speech cases. Facial challenge cannot
be likened to as applied challenge because the latter considers the extant facts affecting real litigants. In
this case, petitioner fail to establish locus standi and no case is filed against them; hence, the petition is
dismissed.

A statute or act suffers from the defect of vagueness when it lacks comprehensible standards that men
of common intelligence must necessarily guess at its meaning and differ as to its application. It is
repugnant to the Constitution in two respects: (1) it violates due process for failure to accord persons,
especially the parties targeted by it, fair notice of the conduct to avoid; and (2) it leaves law enforcers
unbridled discretion in carrying out its provisions and becomes an arbitrary flexing of the Government
muscle. The overbreadth doctrine, meanwhile, decrees that a governmental purpose to control or
prevent activities constitutionally subject to state regulations may not be achieved by means which
sweep unnecessarily broadly and thereby invade the area of protected freedoms.

As distinguished from the vagueness doctrine, the overbreadth doctrine assumes that individuals will
understand what a statute prohibits and will accordingly refrain from that behavior, even though some
of it is protected.

A “facial” challenge is likewise different from an “as-applied” challenge.

Distinguished from an as-applied challenge which considers only extant facts affecting real litigants,
a facial invalidation is an examination of the entire law, pinpointing its flaws and defects, not only on the
basis of its actual operation to the parties, but also on the assumption or prediction that its very
existence may cause others not before the court to refrain from constitutionally protected speech or
activities.

Justice Mendoza accurately phrased the subtitle in his concurring opinion that the vagueness and
overbreadth doctrines, as grounds for a facial challenge, are not applicable to penal laws. A litigant
cannot thus successfully mount a facial challenge against a criminal statute on either vagueness or
overbreadth grounds.

The application of the overbreadth doctrine is limited to a facial kind of challenge and, owing to the
given rationale of a facial challenge, applicable only to free speech cases.
Since a penal statute may only be assailed for being vague as applied to petitioners, a limited vagueness
analysis of the definition of “terrorism” in RA 9372 is legally impermissible absent an actual or imminent
charge against them.

There is no merit in the claim that RA 9372 regulates speech so as to permit a facial analysis of its
validity

From the definition of the crime of terrorism in the earlier cited Section 3 of RA 9372, the following
elements may be culled: (1) the offender commits an act punishable under any of the cited provisions of
the Revised Penal Code, or under any of the enumerated special penal laws; (2) the commission of the
predicate crime sows and creates a condition of widespread and extraordinary fear and panic among the
populace; and (3) the offender is actuated by the desire to coerce the government to give in to
an unlawful demand.

In insisting on a facial challenge on the invocation that the law penalizes speech, petitioners contend
that the element of “unlawful demand” in the definition of terrorism must necessarily be transmitted
through some form of expression protected by the free speech clause.

In sum: RA 9372 seeks to regulate conduct not speech. Facial challenge involving the void for vagueness
and overbreadth doctrines can only be applied to free speech cases.
Roxas v. Macapagal-Arroyo 630 SCRA 211 G.R. No. 189155 - JC

Facts: Roxas was abducted and tortured. When she was freed, she was told not to report the incident.
She applied for the privilege of the writs of amparo and habeas data. The Court of Appeals granted the
same.

Issue: Whether circumstantial evidence with regard to the identity and affiliation of the perpetrators is
enough ground for the issuance of the privilege of the writ of amparo.

Whether substantial evidence to prove actual or threatened violation of the right to privacy in life,
liberty or security of the victim is necessary before the privilege of the writ may be extended.

Held: EVIDENCE REQUIRED IN AMPARO PROCEEDINGS


In amparo proceedings, direct evidence of identity must be preferred over mere circumstantial
evidence – In amparo proceedings, the weight that may be accorded to parallel circumstances as
evidence of military involvement depends largely on the availability or non-availability of other pieces of
evidence that has the potential of directly proving the identity and affiliation of the perpetrators. Direct
evidence of identity, when obtainable, must be preferred over mere circumstantial evidence based on
patterns and similarity, because the former indubitably offers greater certainty as to the true identity
and affiliation of the perpetrators.

EVIDENCE REQURED IN HABEAS DATA PROCEEDINGS

Substantial evidence of an actual or threatened violation of the right to privacy in life, liberty or
security of the victim is an indispensable requirement before the privilege of the writ may be
extended – An indispensable requirement before the privilege of the writ may be extended is the
showing, at least by substantial evidence, of an actual or threatened violation of the right to privacy in
life, liberty or security of the victim. In the case at bar, Roxas failed to show that there is an actual or
threatened violation of such right. Hence, until such time that any of the respondents were found to be
actually responsible for the abduction and torture of Roxas, any inference regarding the existence of
reports being kept in violation of the petitioner’s right to privacy becomes farfetched, and premature.
The Court must, at least in the meantime, strike down the grant of the privilege of the writ of habeas
data.

Meralco v. Lim 632 SCRA 195 G.R. No. 184769 - JC


Facts: Respondent, due to an anonymous letter, was transferred by Meralco to another office. Failing to
disclose the matter of the investigation and details of the transfer, she applied for writ of habeas data
with the RTC which granted it ratiocinating that the writ is not limited to extralegal killings but also to
ordinary citizens.

Issue: Whether an ordinary citizen may petition for issuance of writ of habeas data against a purely
employment issue

Held: No. Section 1. Habeas Data. – The writ of habeas data is a remedy available to any person
whose right to privacy in life, liberty or security is violated or threatened by an unlawful act or
omission of a public official or employee or of a private individual or entity engaged in the gathering,
collecting or storing of data or information regarding the person, family, home and correspondence of
the aggrieved party.

In Tapuz v. del Rosario[15] that the writs of amparo and habeas data will NOT issue to protect
purely property or commercial concerns nor when the grounds invoked in support of the petitions
therefor are vague or doubtful.[16] Employment constitutes a property right under the context of the
due process clause of the Constitution.[17] It is evident that respondent’s reservations on the real reasons
for her transfer - a legitimate concern respecting the terms and conditions of one’s employment - are
what prompted her to adopt the extraordinary remedy of habeas data. Jurisdiction over such concerns
is inarguably lodged by law with the NLRC and the Labor Arbiters.

Briccio “Ricky” A. Pollo v. Chairperson Karina Constantino-David, et al., GR 181881 April 8, 2010 - JC

Facts: Through an unsigned-letter complaint alleging that an employee of the CSC was lawyering for an
accused government employee, an investigation was conducted. Chairperson Karina Constantino-David
ordered the backing-up of all files in the hard disk of computers. It turned out that the files copied from
the computer assigned to and being used by the petitioner obtained 40 to 42 documents which were
draft pleadings or letters in connection with administrative cases in the CSC and other tribunals. Based
on this petitioner was dismissed from the service. Petitioner now questions the legality of the search
conducted on his office computer and the copying of his personal files without his knowledge and
consent, alleged as a transgression on his constitutional right to privacy.

Issue: Whether petitioner’s right to privacy was violated

Ruling: NO. The first criteria to determine such violation is whether petitioner had a reasonable
expectation of privacy. The relevant surrounding circumstances to consider are: (1) the employee’s
relationship to the item seized; (2) whether the item was in the immediate control of the employee
when it was seized; and (3) whether the employee took actions to maintain his privacy in the item.”
Petitioner did not allege that he had a separate enclosed office which he did not share with anyone, or
that his office was always locked and not open to other employees or visitors. Neither did he allege that
he used passwords or adopted any means to prevent other employees from accessing his computer
files. Even if these circumstances exist, they will be negated by the fact that CSC in this case had
implemented a policy that put its employees on notice that they have no expectation of privacy in
anything they create, store, send or receive on the office computers, and that the CSC may monitor the
use of the computer resources using both automated or human means.

The second criteria is whether the search conducted on petitioner’s computer was reasonable. A search
by a government employer of an employee’s office is justified at inception when there are reasonable
grounds for suspecting that it will turn up evidence that the employee is guilty of work-related
misconduct. The search was prompted by a letter-complaint.

Sto. Tomas v. Salac 685 SCRA 245 [2012] - JC

Facts: In G.R. 152642 and G.R. 152710. RA 8042 or the Migrant Workers and Overseas Filipino Act of
1995 was enacted to establish a higher standard of protection for OFWs and their families. Section 29
and 30 of the Act commanded the DOLE to begin deregulating within one year of its passage the
business of handling the recruitment and migration of overseas Filipino workers and phase out within
five years the regulatory functions of the POEA. Petitioners sought to prohibit the DOLE, TESDA and
POEA from issuing further orders and comply with the deregulation per RA 8042 with the Quezon RTC.
Respondent officials filed a case with this Court seeking to nullify the RTC order; the SC issued a TRO.
RA 9422 was enacted which repealed Section 29 and 30 of RA 8042 and adopted the policy of close
government regulation of recruitment and deployment of OFWs. Hence the cases involving Sec 29 and
30 were deemed moot and academic.

In Gr No. 167590, respondent Philippine Association of Service Exporters, Inc. (PASEI) filed a petition to
nullify Sec 6, 7 and 9 of RA 8042. Sec 9 of which provides allowed the filing of criminal actions arising
from "illegal recruitment" before the RTC of the province or city where the offense was committed or
where the offended party actually resides at the time of the commission of the offense. The Manila RTC
declared the same as unconstitutional because venue in criminal cases is jurisdictional and the provision
which allows the filing of the criminal case where the offended party resides would negate the general
rule on venue in criminal cases which is that place where the crime or its essential elements were
committed.

Issue: Whether the filing of the criminal case in the place of the commission of the crime or in the place
where the offended party resides is violative of due process

Held: No. Sec 9 on venue is constitutional. But there is nothing arbitrary or unconstitutional in Congress
fixing an alternative venue for violations of Section 6 of R.A. 8042 that differs from the venue
established by the Rules on Criminal Procedure. Indeed, Section 15(a), Rule 110 of the latter Rules
allows exceptions provided by laws. Thus:

SEC. 15. Place where action is to be instituted.— (a) Subject to existing laws, the criminal action shall be
instituted and tried in the court of the municipality or territory where the offense was committed or
where any of its essential ingredients occurred. (Emphasis supplied)
xxxx
Section 9 of R.A. 8042, as an exception to the rule on venue of criminal actions is, consistent with that
law’s declared policy of providing a criminal justice system that protects and serves the best interests of
the victims of illegal recruitment.

10. Equal Protection of the Law

REQUISITES of VALID CLASSIFICATION:


It must rest on Substantial distinctions
It must be germane to the purpose of the law.
It must not be limited to existing conditions only.
It must apply equally to all members of the same class.

Standards of Judicial Review


a) Rational Basis Test: described as adopting a ‘deferential’ attitude towards legislative classifications.
It applies to legislative classifications in general, such as those pertaining to economic or social
legislation.

b) Strict Scrutiny Test: A legislative classification which impermissibly interferes with the exercise of a
fundamental right or operates to the peculiar disadvantage of a suspect class is presumed
unconstitutional, and the burden is upon government to prove that the classification is necessary to
achieve a compelling state interest and that it is the least restrictive means to protect such interest.
This is used on issues of speech, gender, and race.

c) Intermediate Scrutiny Test: government must show that the challenged classification serves an
important state interest and that the classification is at least substantially related to serving that
interest.

People v. Cayat – 68 PHIL. 12, 18 - JC

Facts: Cayat was charged for violation of Act No. 1639 because he belonged to a non – Christian tribe
and he possessed a bottle of non – native wine. RTC convicted him.

Issue: Whether the law is violative of equal protection of the laws

Held No. It is an established principle of constitutional law that the guaranty of the equal protection of
the laws is not equal protection of the laws is not violated by a legislation based on reasonable
classification. And the classification, to be reasonable, (1) must rest on substantial distinctions; (2) must
be germane to the purposes of the law; (3) must not be limited to existing conditions only; and (4) must
apply equally to all members of the same class.

Act No. 1639 satisfies these requirements. The classification rests on real and substantial, not merely
imaginary or whimsical, distinctions. It is not based upon "accident of birth or parentage," as counsel to
the appellant asserts, but upon the degree of civilization and culture. "That it is germane to the
purposes of law cannot be doubted.

The prohibition "to buy, receive, have in his possession, or drink any ardent spirits, ale, beer, wine, or
intoxicating liquors of any kind, other than the so-called native wines and liquors which the members of
such tribes have been accustomed themselves to make prior to the passage of this Act.," is
unquestionably designed to insure peace and order in and among the non-Christian tribes.

The law is not limited in its application to conditions existing at the time of its enactment. It is intended
to apply for all times as long as those conditions exist. The Act was not predicated, as counsel for
appellant asserts, upon the assumption that the non-Christians are "impermeable to any civilizing
influence." On the contrary, the Legislature understood that the civilization of a people is a slow process
and that hand in hand with it must go measures of protection and security.

Finally, that the Act applies equally to all members of the class is evident from a perusal thereof. That it
may be unfair in its operation against a certain number non-Christians by reason of their degree of
culture, is not an argument against the equality of its application.

Ichong v. Hernandez – 101 PHIL. 1155 - JC

Facts: Republic Act 1180 or commonly known as “An Act to Regulate the Retail Business” was passed.
The said law provides for a prohibition against foreigners as well as corporations owned by foreigners
from engaging from retail trade in our country. This was protested by the petitioner in this case.
According to him, the said law violates the international and treaty of the Philippines therefore it is
unconstitutional. Specifically, the Treaty of Amity between the Philippines and China was violated
according to him.
Issue: Whether RA 1180 is violative of due process of equal protection of the laws

Held: The equal protection of the law clause is against undue favor and individual or class privilege, as
well as hostile discrimination or the oppression of inequality. It is not intended to prohibit legislation,
which is limited either in the object to which it is directed or by territory within which is to operate. It
does not demand absolute equality among residents; it merely requires that all persons shall be treated
alike, under like circumstances and conditions both as to privileges conferred and liabilities enforced.
The equal protection clause is not infringed by legislation which applies only to those persons falling
within a specified class, if it applies alike to all persons within such class, and reasonable grounds
exists for making a distinction between those who fall within such class and those who do not.

The balancing is the essence or, shall it be said, the indispensable means for the attainment of legitimate
aspirations of any democratic society. There can be no absolute power, whoever exercise it, for that
would be tyranny. Yet there can neither be absolute liberty, for that would mean license and anarchy. So
the State can deprive persons of life, liberty and property, provided there is due process of law; and
persons may be classified into classes and groups, provided everyone is given the equal protection of
the law. The test or standard, as always, is reason. The police power legislation must be firmly grounded
on public interest and welfare, and a reasonable relation must exist between purposes and means. And
if distinction and classification has been made, there must be a reasonable basis for said distinction.

MAYOR ANTONIO J. VILLEGAS vs. HIU CHIONG TSAI PAO HO and JUDGE FRANCISCO ARCA [G.R. No. L-
29646 November 10, 1978]
Facts: Ordinance No. 6537 prohibits aliens from being employed or to engage or participate in any
position or occupation or business enumerated therein, whether permanent, temporary or casual,
without first securing an employment permit from the Mayor of Manila and paying the permit fee of
P50.00.
Private respondent HiuChiong Tsai PaoHo who was employed in Manila, filed a petition with the Court
praying for the issuance of the writ of preliminary injunction and restraining order to stop the
enforcement of Ordinance No. 6537 as well as for a judgment declaring said Ordinance No. 6537 null
and void.
Petitioner Mayor Villegas argues that Ordinance No. 6537 cannot be declared null and void on the
ground that it violated the rule on uniformity of taxation because the rule on uniformity of taxation
applies only to purely tax or revenue measures and that Ordinance No. 6537 is not a tax or revenue
measure but is an exercise of the police power of the state, it being principally a regulatory measure in
nature.

Issue: Whether or not ordinance no. 6537 is violative of the equal protection clause.

Ruling: Yes. Ordinance No. 6537 is void because it does not contain or suggest any standard or criterion
to guide the mayor in the exercise of the power which has been granted to him by the ordinance.
The ordinance in question violates the due process of law and equal protection rule of the
Constitution.Requiring a person before he can be employed to get a permit from the City Mayor of
Manila who may withhold or refuse it at will is tantamount to denying him the basic right of the people
in the Philippines to engage in a means of livelihood. While it is true that the Philippines as a State is not
obliged to admit aliens within its territory, once an alien is admitted, he cannot be deprived of life
without due process of law. This guarantee includes the means of livelihood. The shelter of protection
under the due process and equal protection clause is given to all persons, both aliens and citizens.
PATRICIO DUMLAO vs COMMISSION ON ELECTIONS[G.R. No. L-52245 January 22, 1980]
Facts: Petitioner Dumlao is a former retiredGovernor of Nueva Vizcaya, who has filed his certificate of
candidacy for said position of Governor in the forthcoming elections of January 30, 1980.
He specifically questions the constitutionality of section 4 of Batas PambansaBlg. 52 as discriminatory
and contrary to the equal protection and due process guarantees of the Constitution. Claiming that said
provision being purely arbitrary grounds and, therefore, class legislation.

Sec 4 -Any retired elective provincial, city of municipal official who has received payment of the
retirement benefits to which he is entitled under the law and who shall have been 65 years of age at
the commencement of the term of office to which he seeks to be elected, shall not be qualified to run
for the same elective local office from which he has retired.

Issue: Whether said sec 4 of BP 52 is violative of the equal protection clause.

Ruling:NO. The constitutional guarantee of equal protection of the laws is subject to rational
classification. If the groupings are based on reasonable and real differentiations, one class can be
treated and regulated differently from another class. For purposes of public service, employees 65
years of age, have been validly classified differently from younger employees. Employees attaining
that age are subject to compulsory retirement, while those of younger ages are not so compulsorily
retirable.

In the case of a 65-year old elective local official, who has retired from a provincial, city or municipal
office, there is reason to disqualify him from running for the same office from which he had retired, as
provided for in the challenged provision. The need for new blood assumes relevance. The tiredness of
the retiree for government work is present, and what is emphatically significant is that the retired
employee has already declared himself tired and unavailable for the same government work, but, which,
by virtue of a change of mind, he would like to assume again. It is for this very reason that inequality will
neither result from the application of the challenged provision. Just as that provision does not deny
equal protection neither does it permit of such denial. Persons similarly situated are similarly treated.
It is within the compentence of the legislature to prescribe qualifications for one who desires to become
a candidate for office provided they are reasonable, as in this case.
GOESAERT V. CLEARY , 335 U.S. 464 (1948)
Facts: As part of the Michigan system for controlling the sale of liquor, bartenders are required to be
licensed in all cities having a population of 50,000, or more, but no female may be so licensed unless she
be 'the wife or daughter of the male owner' of a licensed liquor establishment.

Issue: Whether or not said statute is violative of the equal protection clause.

Ruling: the statute which in effect forbids any female to act as a bartender unless she be "the wife or
daughter of the male owner" of a licensed liquor establishment, does not violate the Equal Protection
Clause of the Fourteenth Amendment.

(a) The classification which Michigan has made as between wives and daughters of owners of liquor
establishments and wives and daughters of nonowners is not without a reasonable basis.

Since bartending by women may, in the allowable legislative judgment, give rise to moral and social
problems against which it may devise preventive measures, the legislature need not go to the full
length of prohibition if it believes that as to a defined group of females other factors are operating
which either eliminate or reduce the moral and social problems otherwise calling for prohibition.
Michigan evidently believes that the oversight assured through ownership of a bar by a barmaid's
husband or father minimizes hazards that may confront a barmaid without such protecting
oversight. This Court is certainly not in a position to gainsay such belief by the Michigan legislature.
If it is entertainable, as we think it is, Michigan has not violated its duty to afford equal protection of
its laws. We cannot cross-examine either actually or argumentatively the mind of Michigan
legislators nor question their motives. Since the line they have drawn is not without a basis in
reason, we cannot give ear to the suggestion that the real impulse behind this legislation was an
unchivalrous desire of male bartenders to try to monopolize the calling.

(b) Nor is the statute rendered unconstitutional because Michigan allows women to serve as waitresses
where liquor is dispensed.
ORMOC SUGAR COMPANY, INCvs.THE TREASURER OF ORMOC CITY, THE MUNICIPAL BOARD OF
ORMOC CITY, HON. ESTEBAN C. CONEJOS as Mayor of Ormoc City and ORMOC CITY [G.R. No. L-23794,
February 17, 1968]

Facts: In 1964, the Municipal Board of Ormoc City passed Ordinance 4, imposing on any and all
productions of centrifuga sugar milled at the Ormoc Sugar Co. Inc. in Ormoc City a municpal tax
equivalent to 1% per export sale to the United States and other foreign countries. The company paid the
said tax under protest. It subsequently filed a case seeking to invalidate the ordinance for being
unconstitutional.

Issue: Whether the ordinance violates the equal protection clause.

Ruling: The Ordinance taxes only centrifugal sugar produced and exported by the Ormoc Sugar Co. Inc.
and none other. At the time of the taxing ordinance’s enacted, the company was the only sugar central
in Ormoc City. The classification, to be reasonable, should be in terms applicable to future conditions as
well. The taxing ordinance should not be singular and exclusive as to exclude any subsequently
established sugar central, of the same class as the present company, from the coverage of the tax. As
it is now, even if later a similar company is set up, it cannot be subject to the tax because the
ordinance expressly points only to the company as the entity to be levied upon.
ATTORNEYS HUMBERTO BASCO, EDILBERTO BALCE, SOCRATES MARANAN AND LORENZO SANCHEZ vs.
PHILIPPINE AMUSEMENTS AND GAMING CORPORATION (PAGCOR) [G.R. No. 91649 May 14, 1991]

Facts: PAGCOR was created under PD 1869 to enable the Government to regulate and centralize all
games of chance authorized by existing franchise or permitted by law, under the following declared
policy: Section 1. Declaration of Policy. It is hereby declared to be the policy of the State to centralize
and integrate all games of chance not heretofore authorized by existing franchises or permitted by law.

Basco and other lawyers assailed the validity of PAGCOR averring among others that it violates the equal
protection clause of the constitution in that it legalizes PAGCOR conducted gambling, while most other
forms of gambling are outlawed, together with prostitution, drug trafficking and other vices.

Issue: Whether or not the creation of PAGCOR violates the equal protection clause.

Ruling: No. Just how PD 1869 is violative of the equal protection is not clearly explained in their petition.
The mere fact that some gambling activities like cockfighting (PD 449) horse racing (RA 306),
sweepstakes, lotteries and races (RA 1169) are legalized under certain conditions, while others are
prohibited, does not render the applicable laws, PD. 1869 for one, unconstitutional.

“equal protection of the laws” The clause does not preclude classification of individuals who may be
accorded different treatment under the law as long as the classification is not unreasonable or arbitrary.
A law does not have to operate in equal force on all persons or things to be conformable to Article III,
Sec 1 of the Constitution. The “equal protection clause” does not prohibit the Legislature from
establishing classes of individuals or objects upon which different rules shall operate. The Constitution
does not require situations which are different in fact or opinion to be treated in law as though they
were the same.
ANTERO M. SISON, JR vs.RUBEN B. ANCHETA [G.R. No. L-59431 July 25, 1984]

Facts: Batas Pambansa 135 was enacted. Sison, as taxpayer, alleged that its provision (Section 1) unduly
discriminated against him by the imposition of higher rates upon his income as a professional, that it
amounts to class legislation, and that it transgresses against the equal protection and due process
clauses of the Constitution as well as the rule requiring uniformity in taxation.

Issue: Whether BP 135 violates the due process and equal protection clauses, and the rule on uniformity
in taxation.

Ruling: There is a need for proof of such persuasive character as would lead to a conclusion that there
was a violation of the due process and equal protection clauses. Absent such showing, the presumption
of validity must prevail. Equality and uniformity in taxation means that all taxable articles or kinds of
property of the same class shall be taxed at the same rate. The taxing power has the authority to make
reasonable and natural classifications for purposes of taxation. Where the differentiation conforms to
the practical dictates of justice and equity, similar to the standards of equal protection, it is not
discriminatory within the meaning of the clause and is therefore uniform.

Taxpayers may be classified into different categories, such as recipients of compensation income as
against professionals. Recipients of compensation income are not entitled to make deductions for
income tax purposes as there is no practically no overhead expense, while professionals and
businessmen have no uniform costs or expenses necessary to produce their income. There is ample
justification to adopt the gross system of income taxation to compensation income, while continuing the
system of net income taxation as regards professional and business income.
Republic v. Sandiganbayan, Ferdinand Marcos – 230 SCRA 710 [Gr no. 92594 March 4, 1994]

Facts: Before 1986, the Landoil Group of Companies spearheaded by then Congressman Jose de
Venecia, Jr., was able to obtain foreign loans syndicated by various banks aggregating approximately one
hundred twenty million US dollars ($120 M). These foreign loans were guaranteed by PHILGUARANTEE,
whose Board of Directors was then composed of private respondents, Rosendo D. Bondoc, Cesar E. A.
Virata, Ruben Ancheta, Jaime C. Laya, PlacidoMapa, Jr., Roberto Ongpin, and CezarZalamea.
Congressman de Venecia's group of companies was unable to seasonably service these foreign loans and
this compelled PHILGUARANTEE to assume its obligation as guarantor.

The EDSA revolution swept the Marcoses out of power. One of the first official acts of then President
Corazon C. Aquino was the creation of the Presidential Commission on Good Government (PCGG) under
E.O No. 1. It was given the difficult task of recovering the illegal wealth of the Marcoses, their family,
subordinates and close associates.

the de Venecia group of companies and PHILGUARANTEE were sequestered by the petitioner, through
the PCGG.Certain obligations were then assumed by de Venecia, Jr., and his group namely : transfer all
the shares of the capital stock identified as belonging to Marcos , to fully cooperate with the Philippine
Government, in the prosecution of any case against former President Marcos and his cronies.

In reciprocity, petitioner agreed to cause the dismissal against de Venecia and his group of co-signors.

Issue: Whether or not respondent Bondoc and company were denied equal protection by dismissing the
cases against de Venecia who are persons similarly situated in the case.

Ruling: Yes. The dismissal of the Complainant against Bondoc and company is compelled by the equal
protection clause of the constitution. De Venecia, Jr. and the respondents Bondoc and company are
similarly situated. Respondent Bondoc, et al. were included in the complaint only because they allegedly
gave unwarranted favors to de VeneciaJr, in guaranteeing the latter’s foreign loans. When Petitioner
admitted that no undue favor was granted to De Venecia Jr. in the grant of such guaranty facilities and
dismissed its complaint against respondents Bondoc and company. To give a more favored treatment to
de Venecia Jr., when the parties are equally situated is to indulge in invidious discrimination.
ISHMAEL HIMAGAN vs. PEOPLE OF THE PHILIPPINES [G.R. No. 113811 October 7, 1994]

Facts: Petitioner, a policeman assigned with the medical company of the Philippine National Police
Regional Headquarters was implicated for murder and attempted murder. the trial court issued an
Order suspending petitioner until the termination of the case on the basis of Section 47, R.A. 6975,
otherwise known as Department of Interior and Local Government Act of 1990.

petitioner filed a motion to lift the order for his suspension, relying on Section 42 of P.D. 807 of the
Civil Service Decree, that his suspension should be limited to ninety (90) days. respondent judge
denied the motion pointing out that under Section 47 of R.A. 6975, the accused shall be suspended from
office until his case is terminated.

Petitioner posits that as a member of the Philippine National Police, under Sec. 91 of RA 6975 -The Civil
Service Law and its implementing rules and regulations shall apply to all personnel of the Department.
he is covered by the Civil Service Law, particularly Sec. 42 of PD 807 of the Civil Service Decree, which
limits the maximum period of suspension to ninety (90) days, thus:

He claims that an imposition of preventive suspension of over 90 days is contrary to the Civil Service
Law and would be a violation of his constitutional right to equal protection of laws.

Issue: Whether petitioner’s right to equal protection was violated.

Ruling:NO. Petitioner misapplies Sec. 42 of PD 807, the section clearly shows that it refers to the lifting
of preventive suspension in pending administrative investigation, not in criminal cases, as here. the
Civil Service Decree merely supplements, therefore cannot apply to members of the PNP because Sec.
47 of R.A. 6995 provides differently, that is, the suspension where the penalty imposed by law exceeds
six (6) years shall continue until the case is terminated.

The reason why members of the PNP are treated differently from the other classes of persons charged
criminally or administratively insofar as the application of the rule on preventive suspension is
concerned is that policemen carry weapons and the badge of the law which can be used to harass or
intimidate witnesses against them, as succinctly brought out in the legislative discussions.

R.A. 6975 does not violate the suspended policeman's constitutional right to equal protection of the
laws.The equal protection clause exists to prevent undue favor or privilege. It is intended to eliminate
discrimination and oppression based on inequality. Recognizing the existence of real differences among
men, the equal protection clause does not demand absolute equality. It merely requires that all persons
shall be treated alike, under like circumstances and conditions both as to the privileges conferred and
liabilities enforced. Thus, the equal protection clause does not absolutely forbid classifications, such as
the one which exists in the instant case.
COMMISSIONER JOSE T. ALMONTE, VILLAMOR C. PEREZ, NERIO ROGADO, and ELISA RIVERA vs.
HONORABLE CONRADO M. VASQUEZ and CONCERNED CITIZEN. [G.R. No. 95367 May 23, 1995]

Facts: Petitioner Jose T. Almonte was formerly Commissioner of the Economic Intelligence and
Investigation Bureau (EIIB), while Villamor C. Perez is Chief of the EIIB's Budget and Fiscal Management
Division. Subpoena ducestecum was issued by the Ombudsman in connection with his investigation of
an anonymous letter alleging that funds representing savings from unfilled positions in the EIIB had
been illegally disbursed. The letter, purporting to have been written by an employee of the EIIB and a
concerned citizen, was addressed to the Secretary of Finance, with copies furnished several government
offices, including the Office of the Ombudsman.

respondent Ombudsman granted the motion to quash the subpoena in view of the fact that there
were no affidavits filed against petitioners. But he denied their motion to quash the subpoena
ducestecum. He ruled that petitioners were not being forced to produce evidence against themselves,
since the subpoena ducestecum was directed to the Chief Accountant, petitioner NerioRogado. In
addition the Ombudsman ordered the Chief of the Records a Section of the EIIB, petitioner Elisa Rivera,
to produce before the investigator "all documents relating to Personnel Service Funds, for the year
1988, and all documents, salary vouchers for the whole plantilla of the EIIB for 1988, within ten (10)
days from receipt hereof."

Petitioners contend that under Art. XI, § 13(4) the Ombudsman can act only "in any appropriate case,
and subject to such limitations as may be provided by law" and that because the complaint in this case is
unsigned and unverified, the case is not an appropriate one.

Issue: Whether to receive complaints in any form or manner is violative of the equal protection clause.

Ruling: This contention lacks merit. As already stated, the Constitution expressly enjoins the
Ombudsman to act on any complaint filed "in any form or manner" concerning official acts or omissions.

There is violation of petitioner's right to the equal protection of the laws. Petitioners complain that "in
all forum and tribunals . . . the aggrieved parties . . . can only hale respondents via their verified
complaints or sworn statements with their identities fully disclosed," while in proceedings before the
Office of the Ombudsman anonymous letters suffice to start an investigation. In the first place, there can
be no objection to this procedure because it is provided in the Constitution itself. In the second place, it
is apparent that in permitting the filing of complaints "in any form and in a manner," the framers of the
Constitution took into account the well-known reticence of the people which keep them from
complaining against official wrongdoings. As this Court had occasion to point out, the Office of the
Ombudsman is different from the other investigatory and prosecutory agencies of the government
because those subject to its jurisdiction are public officials who, through official pressure and influence,
can quash, delay or dismiss investigations held against them. On the other hand complainants are more
often than not poor and simple folk who cannot afford to hire lawyers.

No violation of right against self-incrimination. It is enough to state that the documents required to be
produced in this case are public records and those to whom the subpoena ducestecum is directed are
government official in whose possession or custody the documents are.
TELECOMMUNICATIONS AND BROADCAST ATTORNEYS OF THE PHILIPPINES, INC. and GMA NETWORK,
INC., vs. THE COMMISSION ON ELECTION. [G.R. No. 132922. April 21, 1998]

Facts: Petitioners challenge the validity of §92of B.P. Blg. No. 881, the Omnibus Election Code, against
claims that the requirement that radio and television time be given free air time on the ground it
singles out radio and television stations to provide free air time. They contend that newspapers and
magazines are not similarly required, that such law: (1) takes property without due process of law and
without just compensation; (2) denies radio and television broadcast companies the equal protection
of the laws; and (3) is in excess of the power given to the COMELEC to supervise or regulate the
operation of media of communication or information during the period of election.

Issue: Whether or not sec 92 of BP 881 violated the equal protection clause.

Ruling: No. It rests on the fallacy that broadcast media are entitled to the same treatment under the
free speech guarantee of the Constitution as the print media. There are important differences in the
characteristics of the two media, however, which justify their differential treatment for free speech
purposes. Because of the physical limitations of the broadcast spectrum, the government must, of
necessity, allocate broadcast frequencies to those wishing to use them. There is no similar justification
for government allocation and regulation of the print media.

In the allocation of limited resources, relevant conditions may validly be imposed on the grantees or
licensees. The reason for this is that, as already noted, the government spends public funds for the
allocation and regulation of the broadcast industry, which it does not do in the case of the print media.
To require the radio and television broadcast industry to provide free air time for the COMELEC Time is a
fair exchange for what the industry gets.

Petitioners’ assertion therefore that §92 of B.P. Blg. 881 denies them the equal protection of the law has
no basis. In addition, their plea that §92 (free air time) and §11(b) of R.A. No. 6646 (ban on paid political
ads) should be invalidated would pave the way for a return to the old regime where moneyed
candidates could monopolize media advertising to the disadvantage of candidates with less resources.
That is what Congress tried to reform in 1987 with the enactment of R.A. No. 6646. We are not free to
set aside the judgment of Congress, especially in light of the recent failure of interested parties to have
the law repealed or at least modified.

All broadcasting, whether by radio or by television stations, is licensed by the government. Airwave
frequencies have to be allocated as there are more individuals who want to broadcast than there are
frequencies to assign. A franchise is thus a privilege subject, among other things, to amendment by
Congress in accordance with the constitutional provision that “any such franchise or right granted . . .
shall be subject to amendment, alteration or repeal by the Congress when the common good so
requires.”
CONRADO L. TIU vs. COURT OF APPEALS [G.R. No. 127410. January 20, 1999]

Facts: RA 7227 entitled "An Act Accelerating the Conversion of Military Reservations Into Other
Productive Uses, Creating the Bases Conversion and Development Authority for this Purpose, Providing
Funds Therefor and for Other Purposes." Section 12 thereof created the Subic Special Economic Zone
and granted there to special privileges.

President Ramos issued EO 97, clarifying the application of the tax and duty incentives. The President
issued EO No. 97-A, specifying the area within which the tax-and-duty-free privilege was operative. The
petitioners challenged before this Court the constitutionality of EO 97-A for allegedly being violative of
their right to equal protection of the laws granting tax and duty incentives only to businesses and
residents within the “secured area” of the Subic Special Economic Zone and denying them to those who
live within the Zone but outside such “fenced-in” territory.

Issue: Whether or not Executive Order No. 97-A violates the equal protection clause of the Constitution

Ruling: No. there are substantial distinctions. It is this specific area which the government intends to
transform and develop from its status quo ante as an abandoned naval facility into a self-sustaining
industrial and commercial zone, particularly for big foreign and local investors to use as operational
bases for their businesses and industries. Undeniably, they are the ones who can pour huge
investments to spur economic growth in the country and to generate employment opportunities for the
Filipinos, the ultimate goals of the government for such conversion. The classification is, therefore,
germane to the purposes of the law.

Certainly, there are substantial differences between the big investors who are being lured to establish
and operate their industries in the so-called “secured area” and the present business operators
outside the area.

It is well-settled that the equal-protection guarantee does not require territorial uniformity of laws.
As long as there are actual and material differences between territories, there is no violation of the
constitutional clause. And of course, anyone, including the petitioners, possessing the requisite
investment capital can always avail of the same benefits by channeling his or her resources or business
operations into the fenced-off free port zone.

The classification set forth by the executive issuance does not apply merely to existing conditions.the
objective is to establish a “self-sustaining, industrial, commercial, financial and investment center” in the
area. There will, therefore, be a long-term difference between such investment center and the areas
outside it.Lastly, the classification applies equally to all the resident individuals and businesses within
the “secured area.” The residents, being in like circumstances or contributing directly to the
achievement of the end purpose of the law, are not categorized further. Instead, they are all similarly
treated, both in privileges granted and in obligations required.
RODOLFO E. AGUINALDO, FLORENCIO L. VARGAS, ROMEO I. CALUBAQUIB, AMADO T. GONZALES,
SILVERIO C. SALVANERA, ALBERTA O. QUINTO, and AURORA V. ESTABILLO, vs. COMMISSION ON
ELECTIONS [G.R. No. 132774. June 21, 1999]

Facts: Petitioners seek to prevent the COMELEC from enforcing during the 1998 elections Section 67 of
the Omnibus Election Code (B.P. Blg. 881) “in accordance with its own tenor or as modified by paragraph
3 of Section 11 of Republic Act No. 8436.”Petitioners contend that the classification in Section 67 is not
based on substantial distinctions and, thus, violative of the equal protection clause of the Constitution.

“SEC. 11.Official Ballot. -…Provided, That any elective official, whether national or local, running for any
office other than the one he/she is holding in a permanent capacity, except for president and vice-
president, shall be deemed resigned only upon the start of the campaign period corresponding to the
position for which he/she is running; …”

According to petitioners, candidates for elective office are classified into the following groups under
Section 67: “(a) First classification: an incumbent elective official who runs for the same position as his
present incumbency …(and) another incumbent elective official running for another position; and(b)
Second Classification: an incumbent elective official who runs for president or vice-president…(and)
another incumbent elective [official] running for any other position (i.e., not his incumbency nor for
president or vice president)…”

Petitioners argue that, in the first classification, the reelectionistis given an undue advantage since he is
able to use the resources, prestige, and influence of his position. The same is not available to one
seeking an office different from the one he is presently holding. This, according to petitioners, does not
equalize the playing field for all candidates.

The COMELEC asserts that the classification embodied in Section 67 is reasonable and based on
substantial distinction. It points out that incumbents running for the same position are not considered
resigned because the intention of the law is to allow them to continue serving their constituents and
avoid a disruption in the delivery of essential services. Those running for different positions are
considered resigned because they are considered to have abandoned their present position by their act
of running for other posts.

Issue: Whether Sec 67 as enunciated in the tenor of sec 11 of the omnibus election code is violative of
the equal protection clause.

Ruling: “... rather than cut short the term of office of elective public officials, this statutory provision
seeks to ensure that such officials serve out their entire term of office by discouraging them from
running for another public office and thereby cutting short their tenure by making it clear that should
they fail in their candidacy, they cannot go back to their former position. This is consonant with the
constitutional edict that all public officials must serve the people with utmost loyalty and not trifle with
the mandate which they have received from their constituents.”Indeed, we have dealt squarely with the
issue of the validity of Section 67 of the Omnibus Election Code in Dimaporo v. Mitra, Jr.Section 67 was
crafted with the intention of giving flesh to the constitutional pronouncement that public service is a
public trust.
AGRIPINO A. DE GUZMAN, JR. et al. v. COMMISSION ON ELECTIONS[G.R. No. 129118. July 19, 2000]
Facts: RA 8189 "The Voter’s Registration Act of 1996". Section 44 thereof provides: "SEC. 44.
Reassignment of Election Officers. - No Election Officer shall hold office in a particular city or
municipality for more than four (4) years. Any election officer who, either at the time of the approval of
this Act or subsequent thereto, has served for at least four (4) years in a particular city or municipality
shall automatically be reassigned by the Commission to a new station outside the original congressional
district."

the COMELEC issued several directives reassigning the petitioners, who are either City or Municipal
Election Officers, to different stations. Petitioners theorize that Section 44 of RA 8189 is violative of the
"equal protection clause" of the 1987 Constitution because it singles out the City and Municipal Election
Officers of the COMELEC as prohibited from holding office in the same city or municipality for more than
four (4) years. They maintain that there is no substantial distinction between them and other COMELEC
officials, and therefore, there is no valid classification to justify the objective of the provision of law
under attack.

Issue: Whether or not RA 8189 sec 44 is violative of the equal protection clause.

Ruling: The petition is barren of merit. Section 44 of RA 8189 enjoys the presumption of validity, and the
Court discerns no ground to invalidate it.

The Court is not persuaded by petitioners’ arguments. The "equal protection clause" of the 1987
Constitution permits a valid classification under the following conditions:1. The classification must rest
on substantial distinctions;2. The classification must be germane to the purpose of the law;3. The
classification must not be limited to existing conditions only; and4. The classification must apply equally
to all members of the same class.
After a careful study, the ineluctable conclusion is that the classification under Section 44 of RA 8189
satisfies the aforestated requirements.The singling out of election officers in order to "ensure the
impartiality of election officials by preventing them from developing familiarity with the people of
their place of assignment" does not violate the equal protection clause of the Constitution.
THE PEOPLE OF THE PHILIPPINES vs. ELPIDIO MERCADO y HERNANDO and AURELIO ACEBRON y
ADORA [G.R. No. 116239. November 29, 2000]

Facts: the Regional Trial Court, Branch 156, Pasig, convicting accused-appellants SPO2 Elpidio Mercado y
Hernando and SPO1 Aurelio Acebron y Adora, of the Philippine National Police of Tanay, Rizal, of
kidnapping with murder and sentencing them to death.

Accused-appellants argue that Republic Act 7659 violates the 1987 Constitution. accused-appellant in
his reply brief contends that the death penalty law is violative of the equal protection clause of the 1987
Constitution because it punishes only people like him, the poor, the uneducated, and the jobless.

Issue: Whether or not RA 7659 providing for the death penalty for 13 heinous crimes violates the equal
protection clause.

Ruling: No. The constitutionality of Republic Act No. 7659 has already been settled in the Court's 12-3
per curiam Resolution in People vs. Echegaray, wherein the following rulings were made:

1. The death penalty is not a "cruel, unjust, excessive or unusual punishment." It is an exercise of the
state's power to "secure society against the threatened and actual evil."
2. The offenses for which Republic Act No. 7659 provides the death penalty satisfy "the element of
heinousness" by specifying the circumstances which generally qualify a crime to be punishable by
death;
3. Republic Act No. 7659 provides both procedural and substantial safeguards to insure its correct
application.
4. The Constitution does not require that "a positive manifestation in the form of a higher incidence of
crime should first be perceived and statistically proven" before the death penalty may be prescribed.
Congress is authorized under the Constitution to determine when the elements of heinousness and
compelling reasons are present, and the Court would exceed its own authority if it questioned the
exercise of such discretion.

"The equality the Constitution guarantees is legal equality or, as it is usually put, the equality of all
persons before the law. Under this guarantee, each individual is dealt with as an equal person in the law,
which does not treat the person differently because of who he is or what he is or what he possesses
(Bernas)”

Apparently, as it should be, the death penalty law makes no distinction. It applies to all persons and to
all classes of persons - rich or poor, educated, or uneducated, religious or non-religious. No particular
person or classes of persons are identified by the law against whom the death penalty shall be
exclusively imposed.
PEOPLE OF THE PHILIPPINES vs. ROMEO G. JALOSJOS [G.R. Nos. 132875-76. February 3, 2000]

Facts: The accused-appellant, Romeo G. Jalosjos is a full-fledged member of Congress who is now
confined at the national penitentiary while his conviction for statutory rape on two counts and acts of
lasciviousness on six counts is pending appeal. The accused-appellant filed this motion asking that he be
allowed to fully discharge the duties of a Congressman, including attendance at legislative sessions and
committee meetings despite his having been convicted in the first instance of a non-bailable offense.

Issue: Does being an elective official result in a substantial distinction that allows different treatment? Is
being a Congressman a substantial differentiation which removes the accused-appellant as a prisoner
from the same class as all persons validly confined under law?

Ruling: No. The performance of legitimate and even essential duties by public officers has never been an
excuse to free a person validly in prison. A strict scrutiny of classifications is essential lest wittingly or
otherwise, insidious discriminations are made in favor of or against groups or types of individuals.

The Court cannot validate badges of inequality. The necessities imposed by public welfare may justify
exercise of government authority to regulate even if thereby certain groups may plausibly assert that
their interests are disregarded. We, therefore, find that election to the position of Congressman is not a
reasonable classification in criminal law enforcement. The functions and duties of the office are not
substantial distinctions which lift him from the class of prisoners interrupted in their freedom and
restricted in liberty of movement. Lawful arrest and confinement are germane to the purposes of the
law and apply to all those belonging to the same class.

It would amount to the creation of a privileged class, without justification in reason, if notwithstanding
their liability for a criminal offense, they would be considered immune during their attendance in
Congress and in going to and returning from the same. There is likely to be no dissent from the
proposition that a legislator or a delegate can perform his functions efficiently and well, without the
need for any transgression of the criminal law. Should such an unfortunate event come to pass, he is to
be treated like any other citizen considering that there is a strong public interest in seeing to it that
crime should not go unpunished.
THE PEOPLE OF THE PHILIPPINES, vs. CAROL M. DELA PIEDRA. [G.R. No. 121777. January 24, 2001]
Cebuana vs Zamboanguena

Facts: Carol dela Piedra was charged with illegal recruitment. Petitioner points out that although the
evidence purportedly shows that her accomplice Jasmine Alejandro handed out application forms and
even received payment, appellant was the only one criminally charged. Alejandro, on the other hand,
remained scot-free. From this, she concludes that the prosecution discriminated against her on grounds
of regional origins. Appellant is a Cebuana while Alejandro is a Zamboangueña, and the alleged crime
took place in Zamboanga City.

Issue: Whether or not accused was denied equal protection and therefore should be exculpated for the
non- inclusion of her accomplice in the prosecution of the crime where they are similarly situated.

Ruling: No. The prosecution of one guilty person while others equally guilty are not prosecuted, is not,
by itself, a denial of the equal protection of the laws. The unlawful administration by officers of a statute
fair on its face, resulting in its unequal application to those who are entitled to be treated alike, is not a
denial of equal protection unless there is shown to be present in it an element of intentional or
purposeful discrimination. But a discriminatory purpose is not presumed, there must be a showing of
“clear and intentional discrimination.” In the case at bar, Dela Piedra has failed to show that, in charging
her, there was a “clear and intentional discrimination” on the part of the prosecuting officials.

Furthermore, the presumption is that the prosecuting officers regularly performed their duties, and this
presumption can be overcome only by proof to the contrary, not by mere speculation. As said earlier,
accused has not presented any evidence to overcome this presumption. The mere allegation that dela
Piedra, a Cebuana, was charged with the commission of a crime, while a Zamboangueña, the guilty party
inappellant’s eyes, was not, is insufficient to support a conclusion that the prosecution officers denied
appellant equal protection of the laws.
International School Alliance of Educators (ISAE) vs. Quisumbing [GR 128845, 1 June 2000]

Facts: The International School, Inc., created pursuant to Presidential Decree 732, Section 2(c) of the
same decree authorizes the School to employ its own teaching and management personnel selected by
it either locally or abroad, from Philippine or other nationalities, such personnel being exempt from
otherwise applicable laws and regulations attending their employment, except laws that have been or
will be enacted for the protection of employees.

The School hires both foreign and local teachers as members of its faculty, classifying the same into two:
(1) foreign-hires and (2) local-hires.

The School grants foreign-hires certain benefits not accorded local-hires. These include housing,
transportation, shipping costs, taxes, and home leave travel allowance. Foreign-hires are also paid a
salary rate 25% more than local-hires. The School justifies the difference on two "significant economic
disadvantages" foreign-hires have to endure, namely: (a) the "dislocation factor" and (b) limited tenure.

Petitioner "a legitimate labor union and the collective bargaining representative of all faculty members"
of the School, contested the difference in salary rates between foreign and local-hires. ISAE filed a
notice of strike.

Issue: Whether the School unduly discriminated against the local-hires.

Ruling: Yes. That public policy abhors inequality and discrimination is beyond contention. Our
Constitution and laws reflect the policy against these evils.

If the employer pays one employee less than the rest, it is not for that employee to explain why he
receives less or why the others receive more. That would be adding insult to injury. The employer has
discriminated against that employee; it is for the employer to explain why the employee is treated
unfairly. Herein, the International School has failed to discharge this burden.

There is no evidence here that foreign-hires perform 25% more efficiently or effectively than the local-
hires. Both groups have similar functions and responsibilities, which they perform under similar working
conditions. The School cannot invoke the need to entice foreign-hires to leave their domicile to
rationalize the distinction in salary rates without violating the principle of equal work for equal pay. The
point-of-hire classification employed by respondent School to justify the distinction in the salary rates of
foreign-hires and local hires to be an invalid classification. There is no reasonable distinction between
the services rendered by foreign-hires and local-hires. The practice of the School of according higher
salaries to foreign-hires contravenes public policy.
CENTRAL BANK (now Bangko Sentral ng Pilipinas) EMPLOYEES ASSOCIATION, INC, vs.
BANGKO SENTRAL NG PILIPINAS and the EXECUTIVE SECRETARY, [G.R. No. 148208 December
15, 2004]

Facts: RA 7653 – otherwise known as the New Central Bank Act took effect. petitioner Central Bank
(now BSP) Employees Association Inc. filed a petition against the Executive Secretary of the Office of the
President to restrain BSP from implementing the last proviso in Section 15 (i), Article II of RA 7653 which
pertains to establishment of a Human resource management system and a compensation structure as
part of the authority of the Monetary Board. Employees whose positions fall under SG 19 and below
shall be in accordance with the rates in the salary standardization act. Petitioner contends that the
classifications is not reasonable, arbitrary and violates the equal protection clause. The said proviso has
been prejudicial to some 2994 rank- and –file BSP employees. Respondent on the other hand contends
that the provision does not violate the equal protection clause, provided that it is construed together
with other provisions of the same law such as the “fiscal and administrative autonomy” of the Bangko
Sentral and the mandate of its monetary board. The Solicitor General, as counsel of the Executive
Secretary defends the provision, that the classification of employees is based on real and actual
differentiation and it adheres to the policy of RA 7653 to “establish professionalism and excellence
within the BSP subject to prevailing laws and policies of the government.”

Issue: Whether or not the contended proviso if RA 7653 violates the equal protection of laws, hence
unconstitutional.

Held: Yes. unconstitutional as it operates on the salary grade or the officer employee status, it
distinguishes between economic class and status with the higher salary grade recipients are of greater
benefit above the law than those of mandated by the Salary Standardization Act. Officers of the BSP
receive higher wages that those of rank-and-file employees because the former are not covered by the
salary standardization act as provided by the proviso.

Considering that majority, if not all, the rank-and-file employees consist of people whose status and rank
in life are less and limited, especially in terms of job marketability, it is they - and not the officers - who
have the real economic and financial need for the adjustment This is in accord with the policy of the
Constitution "to free the people from poverty, provide adequate social services, extend to them a
decent standard of living, and improve the quality of life for all.” Any act of Congress that runs counter
to this constitutional desideratum deserves strict scrutiny by this Court before it can pass muster.

These BSP rank-and-file employees represent the politically powerless and they should not be
compelled to seek a political solution to their unequal and iniquitous treatment. Indeed, they have
waited for many years for the legislature to act. They cannot be asked to wait some more for
discrimination cannot be given any waiting time. Unless the equal protection clause of the Constitution
is a mere platitude, it is the Court's duty to save them from reasonless discrimination.
ARMANDO G. YRASUEGUI v. PHILIPPINE AIRLINES, INC. [G.R. No. 168081, October 17, 2008]

Facts: Petitioner was a former international flight steward of PAL, herein respondent. Petitioner was
dismissed because of his failure to adhere to the weight standards of the airline company
(overweight). Petitioner claims that he was illegally dismissed.

Issue: Whether petitioner was discriminated against when he was dismissed thus a violation of the equal
protection clause.

Ruling: Petition denied. To make his claim more believable, petitioner invokes the equal protection
clause guaranty of the Constitution. However, in the absence of governmental interference, the
liberties guaranteed by the Constitution cannot be invoked. Put differently, the Bill of Rights is not
meant to be invoked against acts of private individuals. Indeed, the US Supreme Court, in interpreting
the 14th Amendment, which is the source of our equal protection guarantee, is consistent in saying that
the equal protection erects no shield against private conduct, however discriminatory or wrongful.
Private actions, no matter how egregious, cannot violate the equal protection guarantee.

The burden of evidence lies with the party who asserts an affirmative allegation, petitioner has to prove
his allegation with particularity. There is nothing on the records which could support the finding of
discriminatory treatment.

SJS v. Atienza 545 SCRA 92 - Kenny

G.R. No. 156052 March 7, 2007

SOCIAL JUSTICE SOCIETY (SJS), VLADIMIR ALARIQUE T. CABIGAO, and BONIFACIO S. TUMBOKON,
Petitioners,
vs.
HON. JOSE L. ATIENZA, JR., in his capacity as Mayor of the City of Manila, Respondent.

Facts: Ordinance No. 8027 reclassified the area described therein from industrial to commercial and
directed the owners and operators of businesses disallowed under Section 1 to cease and desist from
operating their businesses within six months from the date of effectivity of the ordinance. Among the
businesses situated in the area are the so-called "Pandacan Terminals" of the oil companies Caltex
(Philippines), Inc., Petron Corporation and Pilipinas Shell Petroleum Corporation. However, the City of
Manila and the Department of Energy (DOE) entered into a memorandum of understanding
(MOU)6 with the oil companies in which they agreed that "the scaling down of the Pandacan Terminals
[was] the most viable and practicable option." Meanwhile, petitioners filed this original action
for mandamus on December 4, 2002 praying that Mayor Atienza be compelled to enforce Ordinance No.
8027 and order the immediate removal of the terminals of the oil companies.

Issue: Whether respondent has the mandatory legal duty to enforce Ordinance No. 8027 and order the
removal of the Pandacan Terminals
Ruling: YES. The Local Government Code imposes upon respondent the duty, as City Mayor, to "enforce
all laws and ordinances relative to the governance of the city.” One of these is Ordinance No. 8027. As
the chief executive of the city, he has the duty to enforce Ordinance No. 8027 as long as it has not been
repealed by the Sanggunian or annulled by the courts. He has no other choice. It is his ministerial duty to
do so. The objective of the ordinance is to protect the residents of manila from the catastrophic
devastation that will surely occur in case of a terrorist attack on the Pandacan terminals. No reason
exists why such a protective measure should be delayed.

Gobenciong v. CA 550 SCRA 302- Kenny

MIAA v. Olongapo 543 SCRA 269- Kenny

Facts: OMSI and TCSI were among the five contractors of MIAA which had janitorial and maintenance
service contracts covering various areas in the Ninoy Aquino International Airport. Before their service
contracts expired, the MIAA Board of Directors informed OMSI and TCSI that their contracts would no
longer be renewed. TCSI expressed its concern over the award of its concession area to a new service
contractor through a negotiated contract. It said that to award TCSI’s contract by mere negotiation
would violate its right to equal protection of the law. TCSI thus suggested that a public bidding be
conducted and that the effectivity of its service contract be meanwhile extended until a winning bid is
declared.

Issue: Whether in awarding the service contracts through negotiation, there was a violation of
respondent’s right to equal protection.

Ruling: YES. Given the antecedent facts of these consolidated cases, we agree with the courts a
quo that the constitutional right of OMSI and TCSI to equal protection is violated by MIAA and
Gana when no public bidding was called precisely because the latter were going to award the
subject service contracts through negotiation. Worse, the acts of MIAA and Gana smack of
arbitrariness and discrimination as they not only did not call for the required public bidding but
also did not even accord OMSI and TCSI the opportunity to submit their proposals in a public
bidding. What OMSI and TCSI got was a terse reply that their contracts will not be renewed and
that MIAA would negotiate contracts lower than those of OMSI and TCSI without granting them
the opportunity to submit their own bids or proposals. On the ground of uneven protection of
law, we could grant the prayer for an order directing a public bidding. Unfortunately, such
action is already foreclosed by the decision of MIAA not to hire any service contractor.

Nicolas v. Romulo 578 SCRA 438- Kenny

Facts: Respondent Lance Corporal (L/CPL) Daniel Smith is a member of the United States Armed
Forces. He was charged with the crime of rape committed against a Filipina, petitioner herein. Pursuant
to the Visiting Forces Agreement (VFA) between the Republic of the Philippines and the United States,
the latter was granted custody of defendant Smith pending the proceedings. The RTC of Makati found
the defendant Smith guilty. As a result, the Makati court ordered Smith detained at the Makati jail until
further orders. However, defendant Smith was taken out of the Makati jail by a contingent of Philippine
law enforcement agents, purportedly acting under orders of the Department of the Interior and Local
Government, and brought to a facility for detention under the control of the United States government.
Petitioners argued that to allow the transfer of custody of an accused to a foreign power is to provide
for a different rule of procedure for that accused, which also violates the equal protection clause of the
Constitution.

Issue: Whether in ordering the transfer of the accused violates the equal protection clause of the
Constitution.

Ruling: NO. The equal protection clause is not violated, because there is a substantial basis for a
different treatment of a member of a foreign military armed forces allowed to enter our territory and
all other accused. The rule in international law is that a foreign armed forces allowed to enter one’s
territory is immune from local jurisdiction, except to the extent agreed upon. The Status of Forces
Agreements involving foreign military units around the world vary in terms and conditions, according to
the situation of the parties involved, and reflect their bargaining power. But the principle remains, i.e.,
the receiving State can exercise jurisdiction over the forces of the sending State only to the extent
agreed upon by the parties.[12]

League of Cities v. COMELEC 608 SCRA 636- Kenny

Facts: During the 11th Congress, Congress enacted into law 33 bills converting 33 municipalities into
cities. However, Congress did not act on bills converting 24 other municipalities into cities. During the
12th Congress, Congress enacted into law Republic Act No. 9009 which amended Section 450 of the Local
Government Code by increasing the annual income requirement for conversion of a municipality into a
city from P20 million to P100 million. The rationale for the amendment was to restrain, in the words of
Senator Aquilino Pimentel, "the mad rush" of municipalities to convert into cities solely to secure a
larger share in the Internal Revenue Allotment despite the fact that they are incapable of fiscal
independence. After the effectivity of RA 9009, the House of Representatives of the
12th Congress7 adopted Joint Resolution No. 29,8 which sought to exempt from the P100 million income
requirement in RA 9009 the 24 municipalities whose cityhood bills were not approved in the
11th Congress.

Issue: Whether the Cityhood Laws violate the equal protection clause.

Ruling: YES. Even if the exemption provision in the Cityhood Laws were written in Section 450 of the
Local Government Code, as amended by RA 9009, such exemption would still be unconstitutional for
violation of the equal protection clause. The exemption provision merely states, "Exemption from
Republic Act No. 9009 ─ The City of x x x shall be exempted from the income requirement prescribed
under Republic Act No. 9009." This one sentence exemption provision contains no classification
standards or guidelines differentiating the exempted municipalities from those that are not exempted.

There is no substantial distinction between municipalities with pending cityhood bills in the
11thCongress and municipalities that did not have pending bills. The mere pendency of a cityhood bill in
the 11th Congress is not a material difference to distinguish one municipality from another for the
purpose of the income requirement. The pendency of a cityhood bill in the 11th Congress does not affect
or determine the level of income of a municipality. Municipalities with pending cityhood bills in the
11thCongress might even have lower annual income than municipalities that did not have pending
cityhood bills. In short, the classification criterion − mere pendency of a cityhood bill in the 11th Congress
− is not rationally related to the purpose of the law which is to prevent fiscally non-viable municipalities
from converting into cities.

Quinto v. COMELEC 613 SCRA 385- Kenny

G.R. No. 189698 February 22, 2010

ELEAZAR P. QUINTO and GERINO A. TOLENTINO, JR., Petitioners,


vs.
COMMISSION ON ELECTIONS, Respondent.

Facts: The assailed Decision granted the Petition for Certiorari and Prohibition filed by Eleazar P. Quinto
and Gerino A. Tolentino, Jr. and declared as unconstitutional the second proviso in the third paragraph
of Section 13 of Republic Act No. 9369,2 Section 66 of the Omnibus Election Code3 and Section 4(a) of
COMELEC Resolution No. 8678,4mainly on the ground that they violate the equal protection clause of
the Constitution and suffer from overbreadth. The assailed Decision thus paved the way for public
appointive officials to continue discharging the powers, prerogatives and functions of their office
notwithstanding their entry into the political arena.

Issue: Whether the equal protection clause is violated when they accord differential treatment to
elective and appointive officials, because such differential treatment rests on material and substantial
distinctions and is germane to the purposes of the law.

Ruling: Substantial distinctions clearly exist between elective officials and appointive officials. The
former occupy their office by virtue of the mandate of the electorate. They are elected to an office for
a definite term and may be removed therefrom only upon stringent conditions. On the other hand,
appointive officials hold their office by virtue of their designation thereto by an appointing authority.
Some appointive officials hold their office in a permanent capacity and are entitled to security of
tenure while others serve at the pleasure of the appointing authority.

Another substantial distinction between the two sets of officials is that under Section 55, Chapter 8, Title
I, Subsection A. Civil Service Commission, Book V of the Administrative Code of 1987 (Executive Order
No. 292), appointive officials, as officers and employees in the civil service, are strictly prohibited from
engaging in any partisan political activity or take (sic) part in any election except to vote. Under the
same provision, elective officials, or officers or employees holding political offices, are obviously
expressly allowed to take part in political and electoral activities. By repealing Section 67 but retaining
Section 66 of the Omnibus Election Code, the legislators deemed it proper to treat these two classes of
officials differently with respect to the effect on their tenure in the office of the filing of the certificates
of candidacy for any position other than those occupied by them. Again, it is not within the power of the
Court to pass upon or look into the wisdom of this classification.

CREBA v. Romulo 614 SCRA 605 (supra) – Kenny

G.R. No. 160756 March 9, 2010


CHAMBER OF REAL ESTATE AND BUILDERS' ASSOCIATIONS, INC., Petitioner,
vs.
THE HON. EXECUTIVE SECRETARY ALBERTO ROMULO, THE HON. ACTING SECRETARY OF FINANCE
JUANITA D. AMATONG, and THE HON. COMMISSIONER OF INTERNAL REVENUE GUILLERMO
PARAYNO, JR., Respondents.

FACTS: Petitioner Chamber of Real Estate and Builders’ Associations, Inc. is questioning the
constitutionality of Section 27 (E) of Republic Act (RA) 84242 and the revenue regulations (RRs) issued by
the Bureau of Internal Revenue (BIR) to implement said provision and those involving creditable
withholding taxes. Petitioner assails the validity of the imposition of minimum corporate income tax
(MCIT) on corporations and creditable withholding tax (CWT) on sales of real properties classified as
ordinary assets.

ISSUE: Whether the revenue regulations are violative of the equal protection clause because the CWT is
being levied only on real estate enterprises.

RULING: The taxing power has the authority to make reasonable classifications for purposes of
taxation.87 Inequalities which result from a singling out of one particular class for taxation, or exemption,
infringe no constitutional limitation.88 The real estate industry is, by itself, a class and can be validly
treated differently from other business enterprises. Petitioner, in insisting that its industry should be
treated similarly as manufacturing enterprises, fails to realize that what distinguishes the real estate
business from other manufacturing enterprises, for purposes of the imposition of the CWT, is not their
production processes but the prices of their goods sold and the number of transactions involved. The
income from the sale of a real property is bigger and its frequency of transaction limited, making it less
cumbersome for the parties to comply with the withholding tax scheme. On the other hand, each
manufacturing enterprise may have tens of thousands of transactions with several thousand customers
every month involving both minimal and substantial amounts. To require the customers of
manufacturing enterprises, at present, to withhold the taxes on each of their transactions with their
tens or hundreds of suppliers may result in an inefficient and unmanageable system of taxation and may
well defeat the purpose of the withholding tax system.

NPC v. Pinatubo 616 SCRA 611- Kenny


FACTS: The National Power Corporation (NPC)[1] questions the decision rendered by the Regional Trial
Court (RTC) of Mandaluyong City, Branch 213 declaring item[s] 3 and 3.1 of NAPOCOR Circular No. 99-
75, which [allow] only partnerships or corporations that directly use aluminum as the raw
material in producing finished products either purely or partly out of aluminum, to participate in the
bidding for the disposal of ACSR wires as unconstitutional for being violative of substantial due process
and the equal protection clause of the Constitution as well as for restraining competitive free trade and
commerce.

ISSUE: Whether items 3 and 3.1 of NPC Circular No. 99-75 violated the equal protection clause
of the Constitution.
RULING: Items 3 and 3.1 met the standards of a valid classification. Indeed, as juxtaposed by the RTC,
the purpose of NPC Circular No. 99-75 was to dispose of the ACSR wires. As stated by Pinatubo, it was
also meant to earn income for the government. Nevertheless, the disposal and revenue-generating
objective of the circular was not an end in itself and could not bar NPC from imposing conditions for the
proper disposition and ultimately, the legitimate use of the scrap ACSR wires. In giving preference to
direct manufacturers and producers, it was the intent of NPC to support RA 7832, which penalizes the
theft of ACSR in excess of 100 MCM. Items 3 and 3.1 clearly did not infringe on the equal protection
clause as these were based on a reasonable classification intended to protect, not the right of any
business or trade but the integrity of government property, as well as promote the objectives of RA
7832. Traders like Pinatubo could not claim similar treatment as direct manufacturers/processors
especially in the light of their failure to negate the rationale behind the distinction.

Biraogo v. PTC 637 SCRA 78- Kenny


FACTS: Barely a month after the issuance of Executive Order No. 1“Creating the Philippine Truth
Commission of 2010.”, the petitioners asked the Court to declare it unconstitutional citing among others
that E.O. No. 1 violates the equal protection clause as it selectively targets for investigation and
prosecution officials and personnel of the previous administration as if corruption is their peculiar
species even as it excludes those of the other administrations, past and present, who may be indictable.

Issue: Whether the creation of the Philippine Truth Commission of 2010 violates the equal protection
clause.

Ruling: Although the purpose of the Truth Commission falls within the investigative power of the
President, the Court finds difficulty in upholding the constitutionality of Executive Order No. 1 in view of
its apparent transgression of the equal protection clause. For a classification to meet the requirements
of constitutionality, it must include or embrace all persons who naturally belong to the classes. “The
classification will be regarded as invalid if all the members of the class are not similarly treated, both as
to rights conferred and obligations imposed. Applying these precepts to this case, Executive Order No. 1
should be struck down as violative of the equal protection clause. The clear mandate of the envisioned
truth commission is to investigate and find out the truth “concerning the reported cases of graft and
corruption during the “previous administration” only. The intent to single out the previous
administration is plain, patent and manifest.
League v. COMELEC 643 SCRA 149- Kenny

G.R. No. 176951 December 21, 2009

LEAGUE OF CITIES OF THE PHILIPPINES (LCP), et.al., Petitioners,


vs.
COMMISSION ON ELECTIONS, et.al., Respondent

FACTS: Petitioner LCP and the intervenors invokes the equal protection clause in assailing the
constitutionality of RA 9009.

ISSUE: Whether the favourable treatment accorded the sixteen municipalities by the cityhood laws
violate the equal protection clause.

RULING: NO. As a matter of settled legal principle, the fundamental right of equal protection does not
require absolute equality. It is enough that all persons or things similarly situated should be treated
alike, both as to rights or privileges conferred and responsibilities or obligations imposed. The equal
protection clause does not preclude the state from recognizing and acting upon factual differences
between individuals and classes.

As things stand, the favorable treatment accorded the sixteen (16) municipalities by the cityhood laws
rests on substantial distinction. Indeed, respondent LGUs, which are subjected only to the erstwhile PhP
20 million income criterion instead of the stringent income requirement prescribed in RA 9009, are
substantially different from other municipalities desirous to be cities. Looking back, we note that
respondent LGUs had pending cityhood bills before the passage of RA 9009. There lies part of the
tipping difference. And years before the enactment of the amendatory RA 9009, respondents LGUs had
already met the income criterion exacted for cityhood under the LGC of 1991. Due to extraneous
circumstances, however, the bills for their conversion remained unacted upon by Congress. The
classification is also germane to the purpose of the law. The exemption of respondent
LGUs/municipalities from the PhP 100 million income requirement was meant to reduce the inequality
occasioned by the passage of the amendatory RA 9009. Lastly and in connection with the third requisite,
the uniform exemption clause would apply to municipalities that had pending cityhood bills before the
passage of RA 9009 and were compliant with then Sec. 450 of the LGC of 1991, which prescribed an
income requirement of PhP 20 million.

PAGCOR v. BIR 645 SCRA 338- Kenny


G.R. No. 172087 March 15, 2011

PHILIPPINE AMUSEMENT AND GAMING CORPORATION (PAGCOR), Petitioner,


vs.
THE BUREAU OF INTERNAL REVENUE (BIR), represented herein by HON. JOSE MARIO BUÑAG, in his
official capacity as COMMISSIONER OF INTERNAL REVENUE, Public Respondent.

FACTS: Petitioner Philippine Amusement and Gaming Corporation (PAGCOR), sought the declaration of
nullity of Section 1 of Republic Act (R.A.) No. 9337 insofar as it amends Section 27 (c) of the National
Internal Revenue Code of 1997, by excluding petitioner from exemption from corporate income tax for
being repugnant to Sections 1 and 10 of Article III of the Constitution. Section 27 (c) of R.A. No. 8424
(NIRC) provides that government-owned and controlled corporations (GOCCs) shall pay corporate
income tax, except petitioner PAGCOR, the Government Service and Insurance Corporation, the Social
Security System, the Philippine Health Insurance Corporation, and the Philippine Charity Sweepstakes
Office. With the enactment of R.A. No. 9337, certain sections of the National Internal Revenue Code of
1997 were amended. The particular amendment that is at issue in this case is Section 1 of R.A. No. 9337,
which amended Section 27 (c) of the National Internal Revenue Code of 1997 by excluding PAGCOR from
the enumeration of GOCCs that are exempt from payment of corporate income tax.

ISSUE: WHETHER RA 9337, SECTION 1 (C) IS NULL AND VOID AB INITIO FOR BEING REPUGNANT TO THE
EQUAL PROTECTION [CLAUSE] EMBODIED IN SECTION 1, ARTICLE III OF THE 1987 CONSTITUTION.

RULING: NO. PAGCOR cannot find support in the equal protection clause of the Constitution, as the
legislative records of the Bicameral Conference Meeting dated October 27, 1997, of the Committee on
Ways and Means, show that PAGCOR’s exemption from payment of corporate income tax, as provided
in Section 27 (c) of R.A. No. 8424, or the National Internal Revenue Code of 1997, was not made
pursuant to a valid classification based on substantial distinctions and the other requirements of a
reasonable classification by legislative bodies, so that the law may operate only on some, and not all,
without violating the equal protection clause. The legislative records show that the basis of the grant of
exemption to PAGCOR from corporate income tax was PAGCOR’s own request to be exempted.

Gancayco v. Quezon City 658 SCRA 853- Kenny

G.R. No. 177807 October 11, 2011

EMILIO GANCAYCO, Petitioner,


vs.
CITY GOVERNMENT OF QUEZON CITY AND METRO MANILA DEVELOPMENT AUTHORITY, Respondents.
FACTS: The Quezon City Council issued Ordinance No. 2904, which required the relevant property
owner to construct an arcade with a width of 4.50 meters and height of 5.00 meters along EDSA, from
the north side of Santolan Road to one lot after Liberty Avenue, and from one lot before Central
Boulevard to the Botocan transmission line. The ordinance covered the property of Justice Gancayco.
Subsequently, sometime in 1965, Justice Gancayco sought the exemption of a two-storey building being
constructed on his property from the application of Ordinance No. 2904 that he be exempted from
constructing an arcade on his property. The City Council acted favorably on Justice Gancayco’s request.
Decades after, the Metropolitan Manila Development Authority (MMDA) conducted operations to clear
obstructions along the sidewalk of EDSA in Quezon City pursuant to Metro Manila Council’s (MMC)
Resolution No. 02-28, Series of 2002. For failure to comply with the notice, the MMDA proceeded to
demolish the party wall, or what was referred to as the "wing walls," of the ground floor structure.
Justice Gancayco assailed the ordinance as having authorized the taking of private property without due
process of law and just compensation, because the construction of an arcade will require 67.5 square
meters from the 375 square meter property. In addition, he claimed that the ordinance was selective
and discriminatory in its scope and application when it allowed the owners of the buildings located in
the Quezon City-San Juan boundary to Cubao Rotonda, and Balete to Seattle Streets to construct
arcades at their option. He thus sought the declaration of nullity of Ordinance No. 2904 and the
payment of damages. Alternately, he prayed for the payment of just compensation should the court
hold the ordinance valid.

ISSUE: Whether petitioner may assail the ordinance on the ground of equal protection.

RULING: NO. Justice Gancayco may not question the ordinance on the ground of equal protection when
he also benefited from the exemption. It bears emphasis that Justice Gancayco himself requested for an
exemption from the application of the ordinance in 1965 and was eventually granted one. Moreover, he
was still enjoying the exemption at the time of the demolition as there was yet no valid notice from the
city engineer. Thus, while the ordinance may be attacked with regard to its different treatment of
properties that appears to be similarly situated, Justice Gancayco is not the proper person to do so.

Mendoza v. People, GR 183891, October 19, 2011- Kenny

G.R. No. 183891 October 19, 2011

ROMARICO J. MENDOZA, Petitioner,


vs.
PEOPLE OF THE PHILIPPINES, Respondent.

FACTS: Petitioner filed a motion for reconsideration on the Decision of the court affirming the
petitioner’s conviction for his failure to remit the Social Security Service (SSS) contributions of his
employees. Petitioner contends that RA No. 9903 mandates the effective withdrawal of all
pending cases against employers who would remit their delinquent contributions to the SSS within a
specified period, viz., within six months after the law’s effectivity.4 The petitioner claims that in view of
RA No. 9903 and its implementing rules, the settlement of his delinquent contributions in 2007 entitles
him to an acquittal. He invokes the equal protection clause in support of his plea.

ISSUE: Whether petitioner is included within the coverage of the condoning delinquent employers.

RULING: NO. True, the petitioner’s case was pending with the Court when RA No. 9903 was passed.
Unfortunately for him, he paid his delinquent SSS contributions in 2007. By paying outside of the
availment period, the petitioner effectively placed himself outside the benevolent sphere of RA No.
9903. This is how the law is written: it condones employers — and only those employers — with unpaid
SSS contributions or with pending cases who pay within the six (6)-month period following the law’s
date of effectivity.

RA No. 9903 creates two classifications of employers delinquent in remitting the SSS contributions of
their employees: (1) those delinquent employers who pay within the six (6)-month period (the former
group), and (2) those delinquent employers who pay outside of this availment period (the latter group).
The creation of these two classes is obvious and unavoidable when Section 2 and the last proviso of
Section 48 of the law are read together. The same provisions show the law’s intent to limit the benefit of
condonation to the former group only; had RA No. 9903 likewise intended to benefit the latter group,
which includes the petitioner, it would have expressly declared so.

SO ORDERED.

Bureau of Customs v. Teves, GR 181704, December 6, 2011- Kenny

G.R. No. 181704 December 6, 2011

BUREAU OF CUSTOMS EMPLOYEES ASSOCIATION (BOCEA), Petitioner,


vs.
HON. MARGARITO B. TEVES, in his capacity as Secretary of the Department of Finance, et.al.,
Respondents.

FACTS: RA [No.] 9335 was enacted to optimize the revenue-generation capability and collection of the
Bureau of Internal Revenue (BIR) and the Bureau of Customs (BOC). The law intends to encourage BIR
and BOC officials and employees to exceed their revenue targets by providing a system of rewards and
sanctions which included among others, the removal from the service of officials and employees whose
revenue collection falls short of the target. Petitioner contends that R.A. No. 9335 and its IRR violate the
BIR and BOC employees’ right to equal protection of the law because R.A. No. 9335 and its IRR unduly
discriminates against BIR and BOC employees as compared to employees of other revenue generating
government agencies like the Philippine Amusement and Gaming Corporation, Department of
Transportation and Communication, the Air Transportation Office, the Land Transportation Office, and
the Philippine Charity Sweepstakes Office, among others, which are not subject to attrition.
ISSUE: WHETHER REPUBLIC ACT [NO.] 9335 IS UNCONSTITUTIONAL AS THESE VIOLATE THE RIGHT OF BIR
AND BOC OFFICIALS AND EMPLOYEES TO THE EQUAL PROTECTION OF THE LAWS;

RULING: The equal protection clause recognizes a valid classification, that is, a classification that has a
reasonable foundation or rational basis and not arbitrary. With respect to RA [No.] 9335, its expressed
public policy is the optimization of the revenue-generation capability and collection of the BIR and the
BOC. Since the subject of the law is the revenue-generation capability and collection of the BIR and the
BOC, the incentives and/or sanctions provided in the law should logically pertain to the said
agencies. Moreover, the law concerns only the BIR and the BOC because they have the common distinct
primary function of generating revenues for the national government through the collection of taxes,
customs duties, fees and charges.

Pichay v. Office of the Deputy Executive Secretary (supra) – Kenny

G.R. No. 196425 July 24, 2012

PROSPERO A. PICHAY, JR., Petitioner,


vs.
OFFICE OF THE DEPUTY EXECUTIVE SECRETARY FOR LEGAL AFFAIRS INVESTIGATIVE AND
ADJUDICATORY DIVISION, ET.AL., Respondents.

FACTS: President Benigno Simeon Aquino III issued Executive Order No. 13 (E.O. 13), abolishing the
PAGC and transferring its functions to the Office of the Deputy Executive Secretary for Legal Affairs
(ODESLA). Respondent Finance Secretary Cesar V. Purisima filed before the IAD-ODESLA a complaint
affidavit2 for grave misconduct against petitioner Prospero A. Pichay, Jr., Chairman of the Board of
Trustees of the Local Water Utilities Administration (LWUA). Petitioner goes on to assail E.O. 13 as
violative of the equal protection clause pointing to the arbitrariness of limiting the IAD-ODESLA's
investigation only to presidential appointees occupying upper-level positions in the government.

ISSUE: WHETHER E.O. 13 IS UNCONSTITUTIONAL FOR VIOLATING THE EQUAL PROTECTION.

RULING: Executive Order No. 13 Does Not Violate Petitioner's Right to Due Process and the Equal
Protection of the Laws. Petitioner is a presidential appointee occupying the high-level position of
Chairman of the LWUA. Necessarily, he comes under the disciplinary jurisdiction of the President, who is
well within his right to order an investigation into matters that require his informed decision. There are
substantial distinctions that set apart presidential appointees occupying upper-level positions in
government from non-presidential appointees and those that occupy the lower positions in
government.
Alvez v. People 677 SCRA 673- Kenny

Garcia v. People 677 SCRA 750 – Kenny

G.R. No. 198554 July 30, 2012

MAJOR GENERAL CARLOS F. GARCIA, AFP (RET.), Petitioner,


vs.
THE EXECUTIVE SECRETARY, ET.AL., Respondents.

FACTS: Respondent Secretary of National Defense Voltaire T. Gazmin, issued a Memorandum10 to the
Chief of Staff, AFP for strict implementation, the Confirmation of Sentence in the Court Martial Case of
People of the Philippines Versus Major General Carlos Flores Garcia AFP. Thereafter, petitioner was
arrested and detained, and continues to be detained at the National Penitentiary, Maximum Security.
Aggrieved, petitioner filed with this Court the present petition for certiorari and petition for habeas
corpus, alternatively. Petitioner argues that the confirmation issued by the Office of the President
directing him to be confined for two (2) years in the penitentiary had already been fully served in view of
his preventive confinement which had exceeded two (2) years. Therefore, according to him, the Office
of the President no longer has the authority to order his confinement in a penitentiary.

ISSUE: Whether the application of Article 29 of the Revised Penal Code in the Articles of War is in
accordance with the Equal Protection Clause of the 1987 Constitution.

RULING: YES. According to a long line of decisions, equal protection simply requires that all persons or
things similarly situated should be treated alike, both as to rights conferred and responsibilities
imposed. In the present case, petitioner belongs to the class of those who have been convicted by any
court, thus, he is entitled to the rights accorded to them. Clearly, there is no substantial distinction
between those who are convicted of offenses which are criminal in nature under military courts and the
civil courts. Furthermore, following the same reasoning, petitioner is also entitled to the basic and time-
honored principle that penal statutes are construed strictly against the State and liberally in favor of the
accused.43 It must be remembered that the provisions of the Articles of War which the petitioner
violated are penal in nature.
Arroyo v. DOJ – Kenny

G.R. No. 199082 September 18, 2012

JOSE MIGUEL T. ARROYO, Petitioner,


vs.
DEPARTMENT OF JUSTICE, respondents.

FACTS: Acting on the discovery of alleged new evidence and the surfacing of new witnesses indicating
the occurrence of massive electoral fraud and manipulation of election results in the 2004 and 2007
National Elections, on August 2, 2011, the Comelec issued Resolution No. 9266 approving the creation of
a committee jointly with the Department of Justice (DOJ), which shall conduct preliminary investigation
on the alleged election offenses and anomalies committed during the 2004 and 2007 elections.
Petitioner Arroyo contends that the creation of the Joint Committee and Fact-Finding Team is in
violation of the equal protection clause of the Constitution because its sole purpose is the
investigation and prosecution of certain persons and incidents. They argue that there is no substantial
distinction between the allegations of massive electoral fraud in 2004 and 2007, on the one hand, and
previous and subsequent national elections, on the other hand; and no substantial distinction between
petitioners and the other persons or public officials who might have been involved in previous election
offenses. They insist that the Joint Panel was created to target only the Arroyo Administration as well as
public officials linked to the Arroyo Administration. To bolster their claim, petitioners explain that Joint
Order No. 001-2011 is similar to Executive Order No. 1 (creating the Philippine Truth Commission) which
this Court had already nullified for being violative of the equal protection clause.

ISSUE: Whether the creation of Joint Committee via the Joint Order is in accordance with the due
process and equal protection clauses of the Constitution.

RULING: YES. Joint Order No. 001-2011 cannot be nullified on the ground that it singles out the officials
of the Arroyo Administration and, therefore, it infringes the equal protection clause. The Philippine
Truth Commission of 2010 was expressly created for the purpose of investigating alleged graft and
corruption during the Arroyo Administration since Executive Order No. 177 specifically referred to the
"previous administration"; while the Joint Committee was created for the purpose of conducting
preliminary investigation of election offenses during the 2004 and 2007 elections. While GMA and Mike
Arroyo were among those subjected to preliminary investigation, not all respondents therein were
linked to GMA as there were public officers who were investigated upon in connection with their acts in
the performance of their official duties. Private individuals were also subjected to the investigation by
the Joint Committee. The equal protection guarantee exists to prevent undue favor or privilege. It is
intended to eliminate discrimination and oppression based on inequality. Recognizing the existence of
real differences among men, it does not demand absolute equality. It merely requires that all persons
under like circumstances and conditions shall be treated alike both as to privileges conferred and
liabilities enforced.78
Sto. Tomas v. Paneda 685 SCRA 245 – Kenny

G.R. No. 152710

HON. PATRICIA A. STO. TOMAS, in her capacity as Secretary of Department of Labor and Employment
(DOLE),et.al. Petitioners,
vs.
HON. JOSE G. PANEDA, in his capacity as the Presiding Judge of Branch 220, Quezon City, et.al.

FACTS: Congress enacted Republic Act (R.A.) 8042 or the Migrant Workers and Overseas Filipinos Act of
1995.The RTC of Manila declared Section 6 unconstitutional after hearing on the ground that its
definition of "illegal recruitment" is vague as it fails to distinguish between licensed and non-licensed
recruiters11 and for that reason gives undue advantage to the non-licensed recruiters in violation of the
right to equal protection of those that operate with government licenses or authorities.

ISSUE: Whether the assailed provision violates the right to equal protection of licensed recruiters.

RULING: NO. "Illegal recruitment" as defined in Section 6 is clear and unambiguous and, contrary to the
RTC’s finding, actually makes a distinction between licensed and non-licensed recruiters. By its terms,
persons who engage in "canvassing, enlisting, contracting, transporting, utilizing, hiring, or procuring
workers" without the appropriate government license or authority are guilty of illegal recruitment
whether or not they commit the wrongful acts enumerated in that section. On the other hand,
recruiters who engage in the canvassing, enlisting, etc. of OFWs, although with the appropriate
government license or authority, are guilty of illegal recruitment only if they commit any of the wrongful
acts enumerated in Section 6.

Section 2. The right to of the people to be secure in their persons, houses, papers, and effects against
unreasonable searches and seizures of whatever nature and for any purpose shall be inviolable, and
no search warrant or warrant of arrest shall issue except upon probable cause to be determined
personally by the judge after examination under oath or affirmation of the complainant and the
witnesses he may produce, and particularly describing the place to be searched and the persons or
things to be seized.

1. Purpose of Section 2

2. Scope of the Protection

Moncada v. People’s Court, 80 PHIL 1 - Bryan

As we understand it, the main, if not the sole, purpose of our constitutional inhibitions against
unreasonable searches and seizures, was to place a salutary restriction upon the powers of
government. That is to say, we believe the framers of the constitutions of the United States and of this
and other states merely sought to provide against any attempt, by legislation or otherwise, to authorize,
justify, or declare lawful, any unreasonable search or seizure. This wise restriction was intended to
operate upon legislative bodies, so as to render ineffectual any effort to legalize by statute what the
people expressly stipulated could in no event be made lawful; upon executives, so that no law violative
of this constitutional inhibition should ever be enforced; and upon the judiciary, so as to render it the
duty of the courts to denounce as unlawful every unreasonable search and seizure, whether confessedly
without any color of authority, or sought to be justified under the guise of legislative sanction. For the
misconduct of private persons, acting upon their individual responsibility and of their own volition,
surely none of the three divisions of government is responsible. If an official, or a mere petty agent of
the state, exceeds or abuses the authority with which he is clothed, he is to be deemed as acting, not for
the state, but for himself only; and therefore he alone, and not the state, should be held accountable for
his acts. If the constitutional rights of a citizen are invaded by a mere individual, the most that any
branch of government can do is to afford the citizen such redress as a possible, and bring the wrongdoer
to account for his unlawful conduct. . . . .

Nota bene: This is the only part that is in English. The rest of the case is in Spanish.

Stonehill v. Diokno, 20 SCRA 383- Bryan

Facts:
Upon application of the officers of the government (resp. prosecutors), several judges (resp.
judges) Issued a total of 42 search warrants against petitioners &/ or the corporations of w/c they
were officers, directed to any peace officer, to search the persons named and/ or the premises of their
offices, warehouses, and/ or residences, and to seize several personal property as the "subject of the
offense; stolen or embezzled or the fruits of the offense," or "used or intended to be used as the means
of committing the offense" as violation of CB Laws, Tariff and Customs Laws (TCC), NIRC and the RPC."
Alleging that the aforementioned search warrants are null & void, said petitioners filed w/ the SC this
orig. action for certiorari, prohibition, mandamus & injunction. The writ was partially lifted or
dissolved, insofar as the papers, documents, and things seized from the officers of the corporations;
but the injunction was maintained as regards those found & seized in the residences of petitioners.

Issue:
Whether the search warrants in question and the searches and seizures are valid in relation to
1.) petitioners; 2.) corporation.

Ruling:
Two points must be stressed in connection with Art. III, Sec. 2 of the Constitution: (a) that no
warrant shall Issue but upon probable cause to be determined by the judge in the manner set forth
therein; & (b) that the warrant shall particularly describe the things to be seized. None of these
requirements has been complied w/. It was stated that the natural and juridical persons has
committed a violation of CB laws, TCC, NIRC & RPC. No specific offense had been alleged in said
applications. The averments thereof w/ respect to the offense committed were abstract. As a
consequence, it was impossible for the judges who Issued the warrants to have found the existence of a
probable cause, for the same presupposes the introduction of competent proof that the party against
whom it is sought has performed particular acts, or committed specific omissions, violating a given
provision of our criminal laws. General search warrants are outlawed because they place the sanctity of
the domicile and the privacy of communication and correspondence at the mercy of the whims, caprice
or passion of peace officers. The warrants sanctioned the seizure of all records of the petitioners and the
aforementioned corporations, whatever their nature, thus openly contravening the explicit command of
our Bill of Rights. Document seized from an illegal search warrant is not admissible in court as a fruit of
a poisonous tee. However, they could not be returned, except if warranted by the circumstances.

Petitioners were not the proper party to question the validity and return of those taken from
the corporations for which they acted as officers as they are treated as personality different from that
of the corporation. Petitioners have no cause of action to assail the legality of the contested warrants
and the seizure made in pursuance thereof because said corporations have their respective
personalities, separate and distinct from the personality of petitioners. The legality of a seizure can be
contested only by the party whose rights have been impaired thereby and that the objection to an
unlawful search and seizure is purely personal and cannot be availed of by 3rd parties.

People v. Marti, 193 SCRA 57- Bryan

Facts:

On August 14, 1987, the appellant and his common-law wife, Shirley Reyes went to Manila
Packaging and Export Forwarders to send packages to Zurich, Switzerland. It was received by Anita
Reyes and ask if she could inspect the packages. Shirley refused and eventually convinced Anita to seal
the package making it ready for shipment. Before being sent out for delivery, Job Reyes, husband of
Anita and proprietor of the courier company, conducted an inspection of the package as part of
standard operating procedures. Upon opening the package, he noticed a suspicious odor which made
him took sample of the substance he found inside. He reported this to the NBI and invited agents to his
office to inspect the package. In the presence of the NBI agents, Job Reyes opened the suspicious
package and found dried-marijuana leaves inside. A case was filed against Andre Marti in violation of
R.A. 6425 and was found guilty by the court a quo. Andre filed an appeal in the Supreme Court claiming
that his constitutional right of privacy was violated and that the evidence acquired from his package was
inadmissible as evidence against him.

Issue:

Can the Constitutional Right of Privacy be enforced against private individuals?

Ruling:

The Supreme Court held based on the speech of Commissioner Bernas that the Bill of Rights governs the
relationship between the individual and the state.

The constitutional proscription against unlawful searches and seizures therefore applies as a restraint
directed only against the government and its agencies tasked with the enforcement of the law. It is
not meant to be invoked against acts of private individuals. It will be recalled that Mr Job Reyes was
the one who opened the box in the presence of the NBI agents in his place of business. The mere
presence of the NBI agents did not convert the reasonable search effected by Mr. Reyes into a
warrantless search and siezure proscribed by the constitution. Merely to observe and look at that which
is in plain sight is not a search.
The judgement of conviction finding appeallant guilty beyond reasonable doubt of the crime charged
was AFFIRMED.

Waterous Drug Corp. v. NLRC, GR 113271, Oct 16, 1997- Bryan

Facts: Antonia Melodia Catolico was hired as a pharmacist by Waterous Drug Corp.

Catolico sold to YSP Inc. 10 bottles of Voren Tablets at P384 per unit. However, the normal selling price
is P320 per unit. Catolico overcharged by P64 per unit for a total of P640. YSP sent a check payable to
Catolico as a “refund” for the jacked-up price. It was sent in an envelope addressed to her. Saldana, the
clerk of Waterous Drug Corp. opened the envelope and saw that there was a check for P640 for Catolico.

Waterous Drug Corp. ordered the termination of Catolico for acts of dishonesty.

NLRC: Dismissed the Petition. Evidence of respondents (check from YSP) being rendered inadmissible,
by virtue of the constitutional right invoked by complainants.

Petitioners: In the light of the decision in the People v. Marti, the constitutional protection against
unreasonable searches and seizures refers to the immunity of one’s person from interference by
government and cannot be extended to acts committed by private individuals so as to bring it within the
ambit of alleged unlawful intrusion by the government.

ISSUE:
Whether Waterous’ act of opening an envelope from one of its regular suppliers is contrary to the
injunction against unreasonable search and seizure and a person’s right to privacy of communication.

HELD:
NO. In light of the decision in the People v. Marti, the constitutional protection against unreasonable
searches and seizures refers to the immunity of one’s person from interference by government and
cannot be extended to acts committed by private individuals so as to bring it within the ambit of alleged
unlawful intrusion by the government. The Court finds no reason to revise the doctrine laid down in
People vs. Marti that the Bill of Rights does not protect citizens from unreasonable searches and
seizures perpetrated by private individuals.

People v. Mendoza, GR 109279, Jan 18, 1999- Bryan

Facts:
For the death of his wife Cecilia Mendoza, accused-appellant Octavio Mendoza was separately
charge with parricide and illegal possession of firearm and ammunition under two Informations.

Issue: Whether or not the memorandum receipt and mission order was obtained through unreasonable
search.

Ruling:
But this claim is belied by the overwhelming evidence pointing to accused-appellant as the
possessor of the fatal weapon. Charmaine testified that the fatal gun, when exhibited in court, was the
gun she saw on the night her mother was shot. And weeks earlier, she said, it was the same gun which
she saw with his father. Defense witness, Antonio Gabac, when asked by the Las Piñas police
investigators to surrender the gun, claimed that the same was surrendered to him by accused-
appellant shortly after the shooting incident. The possession of the fatal gun by accused-appellant is
further established by the memorandum receipt signed by accused-appellant himself and a mission
order authorizing him to carry the said weapon. But accused-appellant claims that these documents
were illegally procured in grave violation of his constitutional right to privacy of communication and
papers, and/or his right against unreasonable search and seizure.

The Solicitor General is correct in explaining that such right applies as a restraint directed only
against the government and its agencies. The case in point is People vs. Marti (193 SCRA 57 [1991])
where this Court had the occasion to rule that the constitutional protection against unreasonable
searches and seizures refers to the immunity of one’s person from interference by government and it
cannot be extended to acts committed by private individuals so as to bring it within the ambit of alleged
unlawful intrusion.

In the instant case, the memorandum receipt and mission order were discovered by accused-
appellant’s father-in-law Alipio Eusebio, a private citizen. Certainly, a search warrant is dispensable.

People v. Bongcarawan, GR 143944, July 11, 2002- Bryan

FACTS: The accused was convicted of violation of Section 16, Article III of Republic Act No. 6425
(Dangerous Drugs Act). The antecedent Facts of his conviction are as follows:

Evidence for the prosecution shows that on March 11, 1999, an interisland passenger ship, M/V Super
Ferry 5, sailed from Manila to Iligan City. At about 3:00 a.m. on March 13, 1999, the vessel was about to
dock at the port of Iligan City when its security officer, Diesmo, received a complaint from passenger
Canoy about her missing jewelry. Canoy suspected one of her co-passengers at cabin no. 106 as the
culprit. Diesmo and four (4) other members of the vessel security force accompanied Canoy to search
for the suspect whom they later found at the economy section. The suspect was identified as the
accused, Basher Bongcarawan. The accused was informed of the complaint and was invited to go back
to cabin no. 106. With his consent, he was bodily searched, but no jewelry was found. He was then
escorted by 2 security agents back to the economy section to get his baggage. The accused took a
Samsonite suitcase and brought this back to the cabin. When requested by the security, the accused
opened the suitcase, revealing a brown bag and small plastic packs containing white crystalline
substance. Suspecting the substance to be “shabu,” the security personnel immediately reported the
matter to the ship captain and took pictures of the accused beside the suitcase and its contents. They
also called the Philippine Coast Guard for assistance.

But the accused countered this by saying that the Samsonite suitcase containing the
methamphetamine hydrochloride or “shabu” was forcibly opened and searched without his consent,
and hence, in violation of his constitutional right against unreasonable search and seizure. Any evidence
acquired pursuant to such unlawful search and seizure, he claims, is inadmissible in evidence against
him.

ISSUE: WON the conviction was valid

Ruling: YES

In the case before us, the baggage of the accused-appellant was searched by the vessel security
personnel. It was only after they found “shabu” inside the suitcase that they called the Philippine
Coast Guard for assistance. The search and seizure of the suitcase and the contraband items was
therefore carried out without government intervention, and hence, the constitutional protection against
unreasonable search and seizure does not apply.

There is no merit in the contention of the accused-appellant that the search and seizure performed by
the vessel security personnel should be considered as one conducted by the police authorities for like
the latter, the former are armed and tasked to maintain peace and order. The vessel security officer in
the case at bar is a private employee and does not discharge any governmental function.

The Court finds no reason to revise the doctrine laid down in People vs. Marti that the Bill of Rights does
not protect citizens from unreasonable searches and seizures perpetrated by private individuals.

3. Requisites for a Valid Warrant

A. Probable Cause

I. Definition

Henry v. US, 361 US 98 - Bryan

Facts:
Without a warrant for search or arrest, federal officers who were investigating a theft from an
interstate shipment of whiskey twice observed cartons being placed in a motorcar in a residential
district, followed and stopped the car, arrested petitioner and another man who were in it, searched
the car, and found and seized cartons containing radios stolen from an interstate shipment. At
petitioner's trial for unlawfully possessing radios stolen from an interstate shipment, his timely motion
to suppress the evidence so seized was overruled and he was convicted.

Issue: Whether the warrantless arrest and search was valid.

Ruling:
Probable cause exists if the Facts and circumstances known to the officer warrant a prudent
man in believing that the offense has been committed. It is important, we think, that this requirement
be strictly enforced, for the standard set by the Constitution protects both the officer and the citizen. If
the officer acts with probable cause, he is protected even though it turns out that the citizen is innocent.

On the record in this case, the officers did not have probable cause for the arrest when they
stopped the car; the search was illegal; the articles seized were not admissible in evidence; and the
conviction is reversed.
For Arrest:

People v. Syjuco, 64 Phil 667- Bryan

Facts:
The crime alleged is fraud of revenue against the Government. Pursuant to a search warrant
issued, the officers searched the building occupied by Santiago Sy Juco. In the process, the authorities
seized, among others, an art metal filing cabinet claimed by Atty. Remo to be his and contained some
letters, documents and papers belonging to his clients. Also, books belonging to Salakam Lumber Co.,
Inc., were seized.

ISSUE: Whether the search and seizure is valid.

Ruling:
This court held that the oath required must be such that it constitutes a guaranty that the
person taking it has personal knowledge of the Facts of the case and that it convince the committing
magistrate, not the individual seeking the issuance of the warrant or the person making the averment
by hearsay, of the existence of the requisite of probable cause. It has likewise been held by this court
that by probable cause are meant such Facts and circumstances antecedent to the issuance thereof. It
has furthermore been held that the true test of the sufficiency of an affidavit to warrant issuance of a
search warrant is whether it has been drawn in such a manner that perjury could be charged thereon in
case the allegations contained therein prove, and that the provisions of the Constitution and the
statutes relative to searches and seizures must be construed liberally in favor of the individual who may
be affected thereby, and strictly against the State and against the person invoking them for the issuance
of the warrant ordering their execution, for the simple reason that the proceedings of search and
seizure are, by their very nature, summary and drastic ones.

Alvarez v. CFI , 64 Phil 33- Bryan

On June 3, 1936, the chief of the secret service of the Anti-Usury Board presented to Judge
David, presiding judge of CFI of Tayabas, alleging that according to reliable information, the petitioner
is keeping in his house in Infanta, Tayabas documents, receipts, lists, chits and other papers used by
him in connection with his activities as a money lender charging usurious rates of interest in violation of
the law.

In his oath the chief of the secret service did not swear to the truth of his statements upon his
knowledge of the Facts but the information received by him from a reliable person. Upon this
questioned affidavit, the judge Issued the search warrant, ordering the search of the petitioners house
at any time of the day or night, the seizure of the books and documents and the immediate delivery of
such to him (judge). With said warrant, several agents of the Anti-Usury Board entered the petitioner's
store and residence at 7 o'clock of the night and seized and took possession of various articles
belonging to the petitioner.
The petitioner asks that the warrant issued by the Court of First Instance of Tayabas, ordering
the search of his house and the seizure, at anytime of the day or night, of certain accounting books,
documents, and papers belonging to him in his residence situated in Infanta, Tayabas, as well as the
order of a later date, authorizing the agents of the Anti-Usury board to retain the articles seized, be
declared illegal and set aside, and prays that all the articles in question be returned to him.

Issue: Whether the warrant for search and seizure was valid.

Ruling:
Inasmuch as the affidavit of the agent in this case was insufficient because his knowledge of
the Facts was not personal but merely hearsay, it is the duty of the judge to require the affidavit of
one or more witnesses for the purpose of determining the existence of probable cause to warrant the
issuance of the search warrant. When the affidavit of the applicant of the complaint contains sufficient
Facts within his personal and direct knowledge, it is sufficient if the judge is satisfied that there exist
probable cause; when the applicant's knowledge of the Facts is mere hearsay, the affidavit of one or
more witnesses having a personal knowledge of the fact is necessary. We conclude, therefore, that the
warrant Issued is likewise illegal because it was based only on the affidavit of the agent who had no
personal knowledge of the Facts.

Webb v. De Leon, GR 121234, August 23, 1995 - Bryan

Facts:
The National Bureau of Investigation (NBI) filed with the Department of Justice a letter-
complaint charging petitioners Hubert Webb, Michael Gatchalian, Antonio J. Lejano and six (6) other
persons with the crime of Rape and Homicide of Carmela N. Vizconde, her mother Estrellita Nicolas-
Vizconde, and her sister Anne Marie Jennifer in their home at Paranaque, Metro Manila. Forthwith, the
Department of Justice formed a panel of prosecutors headed by Assistant Chief State Prosecutor
Jovencio R. Zuño to conduct the preliminary investigation of those charged with the rape and killing.

The DOJ Panel Issued and filed a 26-page Resolution "finding probable cause to hold
respondents for trial" and recommending that an Information for rape with homicide be filed against
petitioners and their co-respondents with the Regional Trial Court of Parañaque. The case was presided
by respondent judge Zosimo V. Escano. It was, however, the respondent judge Raul de Leon, pairing
judge of Judge Escano, who Issued the warrants of arrest against the petitioners. The case was re-
raffled to Branch 274, presided by Judge Amelita Tolentino who Issued new warrants of arrest against
the petitioners and their co-accused.

Later, petitioners charge that respondent Judge Raul de Leon and, later, respondent Judge
Amelita Tolentino Issued warrants of arrest against them without conducting the required preliminary
examination.

Issue:
Whether respondent Judges de Leon and Tolentino gravely abused their discretion when they
failed to conduct a preliminary examination before issuing warrants of arrest against petitioners.

Ruling:
Probable cause to warrant arrest is not an opaque concept in our jurisdiction. Continuing
accretions of case law reiterate that they are Facts and circumstances which would lead a reasonably
discreet and prudent man to believe that an offense has been committed by the person sought to be
arrested. Other jurisdictions utilize the term man of reasonable caution or the term ordinarily prudent
and cautious man. The terms are legally synonymous and their reference is not to a person with
training in the law such as a prosecutor or a judge but to the average man on the street. It ought to be
emphasized that in determining probable cause, the average man weighs Facts and circumstances
without resorting to the calibrations of our technical rules of evidence of which his knowledge is nil.
Rather, he relies on the calculus of common sense of which all reasonable men have an abundance.

With respect to petitioners' contention that public respondent judge failed to personally
examine and determine the existence of probable cause for the issuance of a warrant, suffice it to say
that the judge does not have to personally examine the complainant and his witnesses in order to
Issue a warrant of arrest as he can rely on the certification of the prosecutor/s.

For Search:

Burgos v. Chief of Staff, 133 SCRA 800 G.R. No. L-64261 December 26, 1984
– Rhino

FACTS: Two search warrants were issued by respondent Judge Cruz-Pano under which the premises of
the "Metropolitan Mail" and "We Forum" newspapers, respectively, were searched, and office and
printing machines, equipment, paraphernalia, motor vehicles with plate numbers and other articles
used in the printing, publication and distribution of the said newspapers, as well as numerous papers,
documents, books and other written literature alleged to be in the possession and control of petitioner
Jose Burgos, Jr. publisher-editor of the "We Forum" newspaper, were seized for being used for
subversive activities in conspiracy with, and to promote the objective of, illegal organizations.

Petitioners are now questioning the search warrant that was issued and asking said warrant to
be nullified.

ISSUE: Whether the search warrants were valid.

RULING: NO! The reasons are:


1. Probable cause for a search is defined as such facts and circumstances which would lead a
reasonably discreet and prudent man to believe that an offense has been committed and that
the objects sought in connection with the offense are in the place sought to be searched. And
when the search warrant applied for is directed against a newspaper publisher or editor in
connection with the publication of subversive materials, as in the case at bar, the application
and/or its supporting affidavits must contain a specification, stating with particularity the
alleged subversive material he has published or is intending to publish. Mere generalization will
not suffice. Thus, the broad statement in Col. Abadilla's application that petitioner "is in
possession or has in his control printing equipment and other paraphernalia, news
publications and other documents which were used and are all continuously being used as a
means of committing the offense of subversion punishable under Presidential Decree 885, as
amended ..." is a mere conclusion of law and does not satisfy the requirements of probable
cause. The “oath” (Section 2, Art. III) required must refer to the truth of the facts within the
personal knowledge of the petitioner or his witnesses.
2. The search warrants are in the nature of general warrants. They are comparable to: "books,
records, pamphlets, cards, receipts, lists, memoranda, pictures, recordings and other written
instruments concerning the Communist Party in Texas" in Stanford v. State of Texas which
invalidated the search warrant.

Prudente v. Dayrit, 180 SCRA 69 G.R. No. 82870, December 14, 1989- Rhino

FACTS: Judge Dayrit issued the assailed search warrant after examining under oath applicant ALLADIN
M. DIMAGMALIW and his witness FLORENIO C. ANGELES that there are good and sufficient reasons to
believe (probable cause) that petitioner NEMESIO PRUDENTE has in his control in the premises of
Polytechnic University of the Philippines, Anonas St., Sta. Mesa, Sampaloc, Manila, properties which are
subject of the above offense or intended to be used as the means of committing the said offense.
Petitioner now questions the validity of the subject search warrant.

ISSUE: Whether the search warrant is valid.

RULING: NO! THE APPLICANT AND HIS WITNESS HAD NO PERSONAL KNOWLEDGE OF THE FACTS AND
CIRCUMSTANCES WHICH BECAME THE BASIS FOR ISSUING THE QUESTIONED SEARCH WARRANT, BUT
ACQUIRED KNOWLEDGE THEREOF ONLY THROUGH INFORMATION FROM OTHER SOURCES OR
PERSONS: While it is true that in the application for search warrant, the applicant P/Major stated
that he verified the information he had earlier received there is nothing in the record to show or indicate
how and when said applicant verified the earlier information acquired by him as to justify his conclusion
that he found such information to be a fact. He might have clarified this point if there had been
searching questions and answers, but there were none. What the records show is the deposition of
witness, P/Lt. Angeles, as the only support to P/Major Dimagmaliw's application, and the said
deposition is based on hearsay.

TEST IN DETERMINING WHETHER THE ALLEGATIONS IN AN APPLICATION FOR SEARCH WARRANT OR


IN A SUPPORTING DEPOSITION, ARE BASED ON PERSONAL KNOWLEDGE OR NOT: In Alvarez vs. Court
of First Instance, the true test of sufficiency of a deposition or affidavit to warrant issuance of a search
warrant is whether it has been drawn in a manner that perjury could be charged thereon and the affiant
be held liable for damage caused. The oath required must refer to the truth of the facts within the
personal knowledge of the applicant for search warrant, and/or his witnesses, not of the facts merely
reported by a person whom one considers to be reliable.

OTHER GROUNDS FOR INVALIDITY: Besides, respondent Judge did not take the deposition of the
applicant as required by the Rules of Court. As held in Roan v. Gonzales, "mere affidavits of the
complainant and his witnesses are thus not sufficient. The examining Judge has to take depositions in
writing of the complainant and the witnesses he may produce and attach them to the record."
Moreover, a perusal of the deposition of P/Lt. Florenio Angeles shows that it was too brief and short.
Respondent Judge did not examine him "in the form of searching questions and answers." On the
contrary, the questions asked were leading as they called for a simple "yes" or "no" answer.

DESCRIPTION OF THE PLACE (VALID): The rule is, that a description of a place to be searched is
sufficient if the officer with the warrant can, with reasonable effort, ascertain and Identify the place
intende. In the case at bar, the application for search warrant and the search warrant itself described
the place to be searched as the premises of the Polytechnic University of the Philippines, located at
Anonas St., Sta. Mesa, Sampaloc, Manila more particularly, the offices of the Department of Military
Science and Tactics at the ground floor, and the Office of the President, Dr. Nemesio Prudente, at PUP,
Second Floor and other rooms at the second floor. The designation of the places to be searched
sufficiently complied with the constitutional injunction that a search warrant must particularly describe
the place to be searched, even if there were several rooms at the ground floor and second floor of the
PUP.

ONE SPECIFIC OFFENSE (VALID): The application for search warrant was captioned: "For Violation of PD
No. 1866 (Illegal Possession of Firearms, etc.). While the said decree punishes several offenses, the
alleged violation in this case was, qualified by the phrase "illegal possession of firearms, etc." As
explained by respondent Judge, the term "etc." referred to ammunitions and explosives. In other words,
the search warrant was issued for the specific offense of illegal possession of firearms and explosives.
Hence, the failure of the search warrant to mention the particular provision of PD No. 1-866 that was
violated is not of such a gravity as to call for its invalidation on this score. Besides, while illegal
possession of firearms is penalized under Section 1 of PD No. 1866 and illegal possession of explosives is
penalized under Section 3 thereof, it cannot be overlooked that said decree is a codification of the
various laws on illegal possession of firearms, ammunitions and explosives; such illegal possession of
items destructive of life and property are related offenses or belong to the same species, as to be
subsumed within the category of illegal possession of firearms, etc. under P.D. No. 1866.

FAILURE OF THE APPLICANT TO STATE, UNDER OATH, THE URGENT NEED FOR THE ISSUANCE OF THE
SEARCH WARRANT, HIS APPLICATION HAVING BEEN FILED ON A SATURDAY (VALID): Court's Circular
No. 19, dated 14 August 1987, which reads:
3. Applications filed after office hours, during Saturdays, Sundays and holidays shall
likewise be taken cognizance of and acted upon by any judge of the court having
jurisdiction of the place to be searched, but in such cases the applicant shall certify and
state the facts under oath, to the satisfaction of the judge, that the issuance is urgent.

it would suffice to state that the above section of the circular merely provides for a guideline, departure
from which would not necessarily affect the validity of an otherwise valid search warrant.

II. Who Determines Probable Cause?

People v. CA, GR 126005, Jan 21, 1999- Rhino

FACTS: The case arose from the fatal shooting of Petitioner Dy's mother in which the primary suspect
was Private Respondent Jonathan Cerbo, son of Private Respondent Billy Cerbo. An information was
filed against Jonathan Cerbo. Later the petitioner executed an affidavit-complaint charging private
respondent Billy Cerbo of conspiracy in the killing. Prosecutor after investigation, amended the
information by including Billy Cerbo in the murder case. A warrant of arrest was issued against Billy for
which he filed a motion to quash warrant of arrest arguing that the same was issued without probable
cause. Respondent Judge issued the assailed order dismissing the case against Billy Cerbo and recalling
the warrant for his arrest.
ISSUE: Whether the assailed order is proper.

RULING: NO! The determination of probable cause during a preliminary investigation is a function
that belongs to the public prosecutor. It is an executive function, the correctness of the exercise of
which is matter that the trial court itself does not and may not be compelled to pass upon. On the other
hand, the determination of probable cause to hold a person for trial must be distinguished from the
determination of probable cause to issue a warrant of arrest, which is a judicial function. Here, the
judge merely determines the probability, not the certainty, of guilt of the accused and, in doing so, he
need not conduct a de novo hearing. He simply personally reviews the prosecutor's initial
determination finding probable cause to see if it is supported by substantial evidence.

EXECUTIVE FUNCTION VS. JUDICIAL FUNCTION IN DETERMINING PROBABLE CAUSE:

FIRST AS TO OBJECTIVES, whether there is reasonable ground to believe that the accused is guilty of
the offense charged and should be held for trial is what the prosecutor passes upon. The judge, on the
other hand, determines whether a warrant of arrest should be issued against the accused, i.e.,
whether there is a necessity for placing him under immediate custody in order not to frustrate the ends
of justice. Thus, even if both should base their findings on one and the same proceeding or evidence,
there should be no confusion as to their distinct objectives.

SECOND AS TO SUPPORTING EVIDENCE, since their objectives are different, the judge cannot rely solely
on the report of the prosecutor in finding probable cause to justify the issuance of a warrant of arrest.
Parenthetically, the prosecutor could ease the burden of the judge and speed up the litigation process
by forwarding to the latter not only the information and his bare resolution, but also so much of the
records and the evidence on hand as to enable His Honor to make his personal and separate judicial
finding on whether to issue a warrant of arrest.

THIRD AS TO EXTENT OF EXAMINATION, it is not required that the complete or entire records of the
case during the preliminary investigation be submitted to and examined by the judge. What is required,
rather, is that the judge must have sufficient supporting documents (such as the complaint, affidavits,
counter-affidavits, sworn statements of witnesses or transcript of stenographic notes, if any) upon
which to make his independent judgment, or at the very least, upon which to verify the findings of the
prosecutor as to the existence of probable cause.

III. Kind of Evidence Needed to Establish Probable Cause

Microsoft Corp. v. Maxicorp, GR 140946, Sept. 13, 2004- Rhino

FACTS: NBI Agent Samiano, Jr. filed several applications for search warrants in the RTC against
Petitioner Maxicorp for alleged violation of Section 29 of PD 49 and Article 189 of the RPC. After
conducting a preliminary examination of the applicant and his witnesses, Judge Bayhon issued four
Search Warrants with the same date which the NBI agents used to search Maxicorp’s premises and
seized property fitting the description stated in the search warrants. Maxicorp filed a motion to quash
the search warrants alleging that there was no probable cause which denied by the RTC as well as its
MR. This was reversed by the CA.
ISSUE: Whether CA erred as there was probable cause to issue the search warrants.

RULING: YES! The determination of probable cause does not call for the application of rules and
standards of proof that a judgment of conviction requires after trial on the merits. As implied by the
words themselves, "probable cause" is concerned with probability, not absolute or even moral
certainty. The standards of judgment are those of a reasonably prudent man.

No law or rule states that probable cause requires a specific kind of evidence. Probable cause is
determined in the light of conditions obtaining in a given situation. Thus, it was improper for the Court
of Appeals to reverse the RTC’s findings simply because the sales receipt evidencing NBI Agent
Samiano’s purchase of counterfeit goods is not in his name. The sales receipt is not the only proof that
the sale of petitioners’ software occurred. During the search warrant application proceedings, NBI Agent
Samiano presented to the judge the computer unit that he purchased from Maxicorp, in which
computer unit Maxicorp had pre-installed petitioners’ software. Even if the sales receipt is disregarded,
there still remains more than sufficient evidence to establish probable cause for the issuance of the
search warrants.

Probable cause is dependent largely on the opinion and findings of the judge who conducted the
examination and who had the opportunity to question the applicant and his witnesses. For this reason,
the findings of the judge deserve great weight. The reviewing court should overturn such findings only
upon proof that the judge disregarded the facts before him or ignored the clear dictates of reason. To
demand a higher degree of proof is unnecessary and untimely. The prosecution would be placed in a
compromising situation if it were required to present all its evidence at such preliminary stage. Proof
beyond reasonable doubt is best left for trial.

IV. In General

Nala v. Barroso, GR 153087 Aug. 7, 2003- Rhino

FACTS: PO3 Alcoser applied for the issuance of a warrant to search the person and residence of
petitioner Bernard R. Nala, who was referred to in the application as “Rumolo Nala alias Long” of
“Purok 4, Poblacion, Kitaotao, Bukidnon” for illegal possession of firearm. Using the warrant, several
firearms, ammunitions and grenade were seized. Petitioner filed an Omnibus Motion seeking to quash
Search and Seizure Warrant which was denied on the basis that there was probable cause. The fact that
the items seized were not exactly the items listed in the warrant does not invalidate the same because
the items seized bear a direct relation to the crime of illegal possession of firearms.

ISSUE: Whether the search warrant is valid.

RULING: NO!

REQUISITES OF A VALID SEARCH WARRANT UNDER THE BILL OF RIGHTS AND RULE 126, SECTIONS 4
AND 5 OF THE 2000 RULES ON CRIMINAL PROCEDURE: (1) probable cause is present; (2) such presence
is determined personally by the judge; (3) the complainant and the witnesses he or she may produce
are personally examined by the judge, in writing and under oath or affirmation; (4) the applicant and
the witnesses testify on facts personally known to them; and (5) the warrant specifically describes the
person and place to be searched and the things to be seized.
FAILURE TO CORRECTLY STATE IN THE WARRANT THE FIRST NAME OF PETITIONER (RUMOLO instead
of BERNARD) DOES NOT INVALIDATE THE WARRANT: because the additional description “alias Lolong
Nala who is said to be residing at Purok 4, Poblacion, Kitaotao, Bukidnon” sufficiently enabled the police
officers to locate and identify the petitioner. What is prohibited is a warrant against an unnamed party,
and not one which, as in the instant case, contains a descriptio personae that will enable the officer to
identify the accused without difficulty.

PROBABLE CAUSE MUST BE SHOWN BY THE BEST EVIDENCE THAT COULD BE OBTAINED UNDER THE
CIRCUMSTANCES ESPECIALLY WHERE THE ISSUE IS THE EXISTENCE OF A NEGATIVE INGREDIENT OF THE
OFFENSE CHARGED ( E.G., THE ABSENCE OF A LICENSE REQUIRED BY LAW): Probable cause as applied
to illegal possession of firearms would therefore be such facts and circumstances which would lead a
reasonably discreet and prudent man to believe that a person is in possession of a firearm and that he
does not have the license or permit to possess the same. While Alcoser testified before the respondent
judge that the firearms in the possession of petitioner are not licensed, this does not qualify as
“personal knowledge” but only “personal belief” because neither he nor Nalagon verified, much more
secured, a certification (best evidence) from the appropriate government agency that petitioner was not
licensed to possess a firearm.

INADMISSIBILITY OF THE ITEMS SEIZED: In this case, it was not because: 1) there was no valid search
warrant; and 2) absent such a warrant, the right thereto was not validly waived by the petitioner. It does
not follow that because an offense is malum prohibitum, the subject thereof is necessarily illegal per se.
Motive is immaterial in mala prohibita, but the subjects of this kind of offense may not be summarily
seized simply because they are prohibited. A warrant is still necessary, because possession of any
firearm becomes unlawful only if the required permit or license therefor is not first obtained.
Admissibility of the items seized cannot be justified under the plain view doctrine. The “plain view”
doctrine applies when the following requisites concur: (a) the law enforcement officer in search of the
evidence has a prior justification for an intrusion or is in a position from which he can view a particular
area; (b) the discovery of the evidence in plain view is inadvertent; (c) it is immediately apparent to the
officer that the item he observes may be evidence of a crime, contraband or otherwise subject to
seizure. The law enforcement officer must lawfully make an initial intrusion or properly be in a position
from which he can particularly view the area. In the course of such lawful intrusion, he came
inadvertently across a piece of evidence incriminating the accused. The object must be open to eye and
hand and its discovery inadvertent.

NO PRESUMPTION OF REGULARITY: No presumption of regularity may be invoked in aid of the process


when the officer undertakes to justify an encroachment of rights secured by the Constitution. The issue
of the reasonableness of the implementation of the search and seizure warrant, i.e., whether the search
was conducted in the presence of witnesses and whether the air rifle which the trial court ordered to be
returned to petitioner was indeed among the items seized during the search, are matters that would be
best determined in the pending administrative case for grave misconduct and irregularity in the
performance of duty against the police officers who conducted the search.

Betoy v. Judge AM NO. MJJ-05-1108, Feb 26, 2006 – Rhino


FACTS: Betoy, Sr. (complainant) charges Judge Coliflores (respondent) with Grave Abuse of Discretion
and Authority, Conduct Unbecoming as a Judge and Gross Negligence Resulting to Procedural Lapses
(Dereliction of Duty) for issuing the SEARCH & SIEZURE ORDER for unlicensed firearm, ammunitions
and explosives without strict compliance with legal requirements. That almost nine months had
passed but the respondent did not even upheld his Search & Seizure Order by not conducting a judicial
inquiry from the implementing law enforcement officers as to the whereabouts of the contraband
items. Office of the Court Administrator (OCA) submitted to the Court a Memorandum wherein it
found that respondent judge was able to establish probable cause but failed to conduct a judicial
inquiry as to the whereabouts of the seized firearms and ammunitions.

ISSUE: Whether there is probable cause in the issuance of the search warrant.

RULING: NO! The same ruling in Nala v. Barroso, GR 153087 Aug. 7, 2003: no personal knowledge that
the firearms were unlicensed. Despite the glaring insufficiency of the allegations in their respective
depositions, respondent judge still failed to elicit the necessary information during his examination of
the said applicant and his witnesses to establish that complainant and his companions are indeed guilty
of illegally possessing firearms and ammunition. Thus, respondent judge fell short of the standard of
competence required of magistrates in the performance of their functions.

Respondent judge was remiss in his duty of ascertaining if a return of the warrant has been made, and if
there is none, to summon the person to whom the warrant was issued and require him to explain why
no return was made (Section 12, Rule 126 Revised Rules on Criminal Procedure). Respondent judge
should know that his duty as a magistrate does not end when he issued the search warrant.

In Dizon, Jr. v. Judge Veneracion,24 the respondent judge therein was found guilty of gross ignorance of
the law for his failure to observe the rules governing determination of probable cause. The OCA
recommendation of fine of P20,000.00 is deemed just and reasonable. (N.B.: The foregoing was culled
from A.M. No. MTJ-05-1608 February 28, 2006; the given citation did not bear result.)

20th Century Fox v. CA, 162 SCRA 655 G.R. Nos. 76649-51 August 19, 1988 - Rhino

FACTS: Petitioner 20th Century Fox Film Corporation through counsel sought the NBI’s assistance in
the conduct of searches and seizures in connection with the latter's anti-film piracy campaign. The
lower court issued the corresponding search warrants which were used by the NBI accompanied by the
petitioner's agents. Acting on a motion to lift search warrants and release seized properties filed by the
private respondents, the lower court issued an order lifting the three (3) search warrants. Petitioner
now questions such order.

ISSUE: Whether there is probable cause in the issuance of the search warrants.
RULING: NO! Section 2, Article III seeks to guard a man's prerogative to choose who is allowed entry to
his residence. In that haven of refuge, his individuality can assert itself not only in the choice of who
shall be welcome but likewise in the kind of objects he wants around him.

NO PROBABLE CAUSE: The essence of a copyright infringement is the similarity or at least substantial
similarity of the purported pirated works to the copyrighted work. Hence, the applicant must present
to the court the copyrighted films to compare them with the purchased evidence of the video tapes
allegedly pirated to determine whether the latter is an unauthorized reproduction of the former. This
linkage of the copyrighted films to the pirated films must be established to satisfy the requirements of
probable cause. Mere allegations as to the existence of the copyrighted films cannot serve as basis for
the issuance of a search warrant. The lower court did not give much credence to petitioner’s testimony
in view of the fact that the master tapes of the allegedly pirated tapes were not shown to the court
during the application.

GENERAL WARRANTS: Television sets, video cassette recorders, reminders and tape cleaners are
articles which can be found in a video tape store engaged in the legitimate business of lending or renting
out betamax tapes. In short, these articles and appliances are generally connected with, or related to a
legitimate business not necessarily involving piracy of intellectual property or infringement of copyright
laws. Hence, including these articles without specification and/or particularity that they were really
instruments in violating an Anti-Piracy law makes The search warrant too general which could result in
the confiscation of all items found in any video store.

PREVIOUSLY ISSUED SEARCH WARRANTS CAN BE RECALLED: Certainly, such action is within the power
and authority of the respondent Court to perform, provided that it is not exercised in an oppressive or
arbitrary manner. Indeed, the order of the respondent Court declaring the existence of probable cause is
not final and does not constitute res judicata.

Columbia Pictures v. CA, 262 SCRA 219 G.R. No. 111267. September 20, 1996- Rhino

FACTS: Ramos, intelligence officer of the Videogram Regulatory Board (VRB), received information
that private respondent Jinco had in his possession pirated videotapes and other materials related
thereto. Ramos ascertained the information to be true and filed a verified Application for Search
Warrant which was granted and used by the VRB. Private respondent then filed an Urgent Motion to
Lift the Search Warrant and For the Return of the Seized Articles. Relying on the ruling in 20th Century
Fox v. CA, the new judge granted the motion. Petitioners appealed to the Court of Appeals, which
affirmed the said Order in toto. Hence, this petition.

ISSUE #1: Whether the petitioners have legal personality and standing to file the appeal.

RULING: YES!

In People v. Nano, the Court declared that while the general rule is that it is only the Solicitor General
who is authorized to bring or defend actions on behalf of the People or the Republic of the Philippines
once the case is brought before this Court or the Court of Appeals, if there appears to be grave error
committed by the judge or a lack of due process, the petition will be deemed filed by the private
complainants therein as if it were filed by the Solicitor General.

ISSUE #2: Whether the application of the 20th Century Fox decision, which had not yet been
promulgated in 1986 when the search warrant in question was issued, can be made retroactive.

RULING: NO! Judicial interpretation becomes a part of the law as of the date that law was originally
passed, subject only to the qualification that when a doctrine of this Court is overruled and a different
view is adopted, and more so when there is a reversal thereof, the new doctrine should be applied
prospectively and should not apply to parties who relied on the old doctrine and acted in good
faith. The lower court could not possibly have expected more evidence from petitioners in their
application for a search warrant other than what the law and jurisprudence, then existing and
judicially accepted, required with respect to the finding of probable cause.

In fine, the supposed pronunciamento in said case regarding the necessity for the presentation of the
master tapes of the copyrighted films for the validity of search warrants should at most be understood
to merely serve as a guidepost in determining the existence of probable cause in copyright infringement
cases where there is doubt as to the true nexus between the master tape and the pirated copies. An
objective and careful reading of the decision in said case could lead to no other conclusion than that said
directive was hardly intended to be a sweeping and inflexible requirement in all or similar copyright
infringement cases.

B. Personally Determined by the Judge

Placer v. Villanueva, 126 SCRA 463 G.R. No. L-60349-62 December 29, 1983 - Rhino

FACTS: Petitioners, the City Fiscal of Butuan City and his assistants filed in the City Court of Butuan
several informations and they certified that Preliminary Investigation and Examination had been
conducted and that prima facie cases have been found. Upon receipt of said informations, respondent
judge set the hearing of the criminal cases to determine propriety of issuance of warrants of arrest.
After the hearing, respondent issued an order requiring petitioners to submit to the court affidavits of
prosecution witnesses and other documentary evidence in support of the informations to aid him in
the exercise of his power of judicial review of the findings of probable cause by petitioners. Petitioners
petitioned for certiorari and mandamus to compel respondent to issue warrants of arrest. They
contended that the fiscal’s certification in the informations of the existence of probable cause
constitutes sufficient justification for the judge to issue warrants of arrest.

ISSUE: Whether respondent city judge may, for the purpose of issuing warrants of arrest, compel the
fiscal to submit to the court the supporting affidavits and other documentary evidence presented
during the preliminary investigation.

RULING: NO! Judge may rely upon the fiscal’s certification for the existence of probable cause and on
the basis thereof, issue a warrant of arrest. But, such certification does not bind the judge to come out
with the warrant. The issuance of a warrant is not a mere ministerial function; it calls for the exercise
of judicial discretion on the part of issuing magistrate. Under Section 6 Rule 112 of the Rules of Court,
the judge must satisfy himself of the existence of probable cause before issuing a warrant of arrest. If on
the face of the information, the judge finds no probable cause, he may disregard the fiscal’s certification
and require submission of the affidavits of witnesses to aid him in arriving at the conclusion as to
existence of probable cause.

Lim v. Judge Fenix, 194 SCRA 292 G.R. Nos. 94054-57 February 19, 1991- Rhino

FACTS: The preliminary investigation was conducted by the MTC of Masbate which found the
existence of probable cause that the offense of multiple murder was committed and that all the
accused are probably guilty thereof, which was affirmed upon review by the Provincial Prosecutor who
properly filed with the RTC of Masbate four separate informations for murder. Upon petition for
change of venue, the cases were transferred to RTC Makati which issued the arrest warrants without
bail considering that both competent officers to whom such duty was entrusted by law have declared
the existence of probable cause, each information is complete in form and substance, and there is no
visible defect on its face. Respondent Judge Felix found it just and proper to rely on the prosecutor's
certification in each information as the record of the preliminary investigation was still with the RTC
Masbate. The petitioners questioned this order.

ISSUE: Whether a judge may issue a warrant of arrest without bail by simply relying on the
prosecution's certification and recommendation that a probable cause exists.

RULING: In Soliven v. Makasiar that the Judge does not have to personally examine the complainant
and his witnesses. The Prosecutor can perform the same functions as a commissioner for the taking of
the evidence. However, there should be a report and necessary documents supporting the Fiscal's bare
certification. All of these should be before the Judge. The extent of the Judge's personal examination of
the report and its annexes depends on the circumstances of each case. To be sure, the Judge must go
beyond the Prosecutor's certification and investigation report whenever necessary.

Petitioner presented before respondent judge Felix documents of recantation of witnesses whose
testimonies were used to establish a prima facie case against them. Although, the general rule is that
recantations are not given much weight in the determination of a case and in the granting of a new
trial the respondent Judge before issuing his own warrants of arrest should, at the very least, have
gone over the records of the preliminary examination conducted earlier in the light of the evidence
now presented by the concerned witnesses in view of the "political undertones" prevailing in the
cases. (N.B.: It’s JUDGE FELIX, not FENIX.)

People v. Inting, 187 SCRA 788 – Prince


G.R. No. 88919, July 25, 1990
1. OIC-Mayor violated existing election laws by transferring a government nurse to a remote
barangay without consent of the COMELEC.
2. The nurse filed a complaint with the COMELEC who sent its Provincial Election Supervisor.
3. COMELEC ordered its Election Supervisor to conduct preliminary investigations and
prosecution.
4. The Election supervisor found probable cause and filed an information before the RTC who
later issued an arrest warrant against the mayor. But the RTC set aside its order and require the
Election Supervisor to have the approval of the Provincial Fiscal.
ISSUE: Whether the Election Supervisor can conduct preliminary investigations.
HELD: CONSTITUTION EMPOWERS COMELEC TO CONDUCT PRELIMINARY INVESTIGATIONS. PETITION
GRANTED.
COMELEC, under the 1987 Constitution and the Election Code, has the exclusive power to conduct
preliminary INVESTIGATIONS with respect to election offenses.
 Judges and Prosecutors alike should distinguish the preliminary EXAMINATION which
determines probable cause for the issuance of a warrant of arrest from the preliminary
INVESTIGATION which ascertains whether the offender should be held for trial or released.
Even if the two inquiries are conducted in the course of one and the same proceeding, there
should be no confusion about the objectives. The determination of probable cause for the
warrant of arrest is made by the Judge. The preliminary investigation proper-whether there is
reasonable ground to believe that the accused is guilty of the offense charged and, therefore,
whether he should be subjected to the expense, rigors and embarrassment of trial is the
function of the Prosecutor (Castillo v. Villaluz, 171 SCRA 39 [1989]).
 The preliminary INVESTIGATION made by a Prosecutor merely assists the judge to make the
determination of probable cause. The Judge does not have to follow what the Prosecutor
presents to him. By itself, the Prosecutor's certification of probable cause is ineffectual. It is the
report, the affidavits, the transcripts of stenographic notes (if any), and all other supporting
documents behind the Prosecutor's certification which are material in assisting the Judge to
make his determination.
 Prosecutor's Preliminary INVESTIGATION is executive in nature.
 Judge's Preliminary EXAMINATION is judicial.
Judges of RTC (CFI) no longer have authority to conduct preliminary INVESTIGATIONS. That authority
was removed from them by the 1985 Rules on Criminal Procedure, effective on January 1, 1985, which
deleted all provisions granting that power to said Judges. Salta v. Court of Appeals, 143 SCRA 228:
 (1) that the conduct of a preliminary INVESTIGATION is "not a judicial function but part of the
prosecution's job, a function of the executive,"
 (2) that wherever "there are enough fiscals or prosecutors to conduct preliminary
INVESTIGATIONS, courts are counseled to leave this job which is essentially executive to them,"
and the fact "that a certain power is granted does not necessarily mean that it should be
indiscriminately exercised."
However, RTC Judges did not lose the power(granted by the 1935, 1973, and 1987 Constitutions) to
make a preliminary EXAMINATIONS for the purpose of determining whether probable cause exists to
justify the issuance of a warrant of arrest (or search warrant).

People v. Delgado, 189 SCRA 715- Prince


G.R. Nos. 93419-32, September 18, 1990
1. COMELEC sent its Provincial Election Supervisor to conduct preliminary investigation and
prosecution of election offenses.
2. After preliminary investigations, the Election Supervisor filed informations before the RTC who
later issued arrest warrants.
3. Defendants moved for suspension of the arrest warrants because they claim that there was no
preliminary investigations.
4. The RTC ordered COMELEC to conduct reinvestigation and suspended the issuance of the
arrest warrant until the submission of the reinvestigation reports.
ISSUE: Whether the RTC can order the COMELEC to re-investigate despite there was already a
preliminary investigation done by the Election Supervisor.
HELD: RTC ACQUIRED JURISDICTION; COMELEC MUST OBEY ORDERS OF RTC. THUS, COMELEC MUST
OBEY REINVESTIGATION TO SATISFY RTC'S PROBABLE CAUSE FOR THE ISSUANCE OF WARRANTS.
PETITION DISMISSED. Compare with People vs. Inting.

Sec 268 of the Omnibus Election Code: RTCs shall have exclusive original jurisdiction to try and decide
any criminal action or proceedings for violation of the Omnius Election Code, except those relating to
the offense of failure to register or failure to vote which shall be under the jurisdiction of the
metropolitan or municipal trial courts. Hence, when the COMELEC, through its duly authorized law
officer, conducts the preliminary investigation of an election offense and upon a prima facie finding of a
probable cause, files the information in the proper court, said court thereby acquires jurisdiction over
the case. Consequently, all the subsequent disposition of said case must be subject to the approval of
the court. The COMELEC cannot conduct a reinvestigation of the case without the authority of the court.
On the other hand, the RTC may order a reinvestigation since it already acquired jurisdiction and the
COMELEC must obey such order.
Because of the RTC's acquisition of jurisdiction, the records of the preliminary investigation
required to be produced by the court must be submitted by the COMELEC. The trial court may rely on
the resolution of the COMELEC to file the information, by the same token that it may rely on the
certification made by the prosecutor who conducted the preliminary investigation, in the issuance of
the warrant of arrest. Nevertheless, the court may require that the record of the preliminary
investigation be submitted to it to satisfy itself that there is probable cause which will warrant the
issuance of a warrant of arrest.

Allado v. Diokno – 232 SCRA 192- Prince


G.R. No. 113630, May 5, 1994

1. Acting upon an extrajudicial confession, the Presidential Anti-Crime Commission (PACC)


sought for a search warrant to search for the residences of the petitioners.
2. Petitioners were accused and investigated for kidnapping and murder of a German national.
3. Extrajudicial confession and sworn affidavits were submitted to the office of the prosecutor. The
prosecutor found probable cause and submitted findings and certification of probable cause
to the Judge.
4. The extrajudicial confession and sworn affidavits show manifests inconsistencies. But the
judge issued the arrest warrant anyway without questioning the complainant or witnesses of
the PACC.

ISSUE: Whether the arrest warrant is valid.

HELD: JUDGE FAILED TO PERSONALLY EXAMINE THE COMPLAINANT AND WITNESSES WHEN THE
CONFESSIONS AND AFFIDAVITS ARE INCONSISTENT. THUS, SEARCH WARRANT INVALID.

The judge committed grave abuse of discretion in issuing the warrant for the arrest of petitioners it
appearing that he did not personally examine the evidence nor did he call for the complainant and his
witnesses in the face of their incredible accounts. Instead, he merely relied on the certification of the
prosecutors that probable cause existed. For, otherwise, he would have found out that the evidence was
insufficient for probable cause.

Gozos v. Tac-an – GR 123191, Dec. 17, 1998- Prince


1. A group of drunk men gatecrashed a school party. Hence, the school officials called the police
officers.
2. The police officers came and check. One of the drunk men was searched and found to have
possessed a pistol. The searching officer asked for any license or permit to carry, and the man
showed his document. The drunk man asked for the return of his pistol and document. He
tried to grab his gun. The officer and the man wrest possession of gun until the officer took out
his service pistol and shot the man.
3. Investigator of the Ombudsman for the Military filed an information for murder before the
RTC judge.
4. The officers herein filed a Motion to Hold Issuance of Warrant and Motion to Quash with
Motion to Set Incidents for Hearing. They pray for finding of probable cause before issuance of
the arrest warrant.
5. The judge heard the parties and found probable cause only for homicide and not murder. He
ordered the prosecutor to amend the information. He issued the arrest warrant on the basis of
homicide.
ISSUE: Whether the arrest warrant was valid.
HELD: RTC JUDGE LACKS AUTHORITY TO DETERMINE THE NATURE OF THE OFFENSE. THUS, INVALID
ARREST WARRANT. Compare with People vs. Inting.
Art. III, Sec. 2 of the 1997 Constitution applies to the issuance of arrest and search warrants and
should be distinguished from a preliminary investigation.
Judges and prosecutors alike should distinguish the preliminary examination which determines
probable cause for the issuance of a warrant of arrest from the preliminary investigation proper which
ascertains whether the offender should be held for trial or released.

Flores v. Sumaljag – 290 SCRA 568- Prince


A.M. No. MTJ-97-1115, June 5, 1998
1. An Administrative case against acting MTC judge Sumaljag for gross ignorance of the law.
2. Flores and others were separately charged with falsification of public documents under the
RPC.
3. Private complainant filed and signed his complaint. Complainant and witness appeared before
the MTC judge for preliminary examination for the issuance of arrest warrants. Private
complainant and witness are timid or shy; hence the judge is compelled to use leading
questions in inquiry. The complainant and witness merely answered “Yes” to every question.
4. For other defendants, the police officer filed and signed the complaint and he was not inquired
by the judge. The private complainant did not file his complaint against other defendants.
Nevertheless, all charges have the same private complainant and witness who appeared before
the judge.
5. MTC judge issued the separate warrants of arrest

ISSUE 1) Whether the arrest warrants were necessary despite finding of probable cause.
HELD: PROBABLE CAUSE IS LIMITED TO NECESSITY TO PLACE DEFENDANTS UNDER CUSTODY; NO
PROOF OF NECESSITY. HENCE, INVALID ARREST WARRANTS. JUDGE is FINED, Php 5,000.
The rule now is that the examining judge's discretionary power to order the arrest of the accused is
limited to instances in which there is a necessity for placing him in custody "in order not to frustrate the
ends of justice."(Rule 112, sec 6.b.; Mantaring vs. Roman) The arrest of the accused can be ordered only
in the event the prosecutor files the case and the judge of the Regional Trial Court finds probable cause
for the issuance of a warrant of arrest (Samulde v. Salvani, Jr.).

ISSUE 2) Whether the judge must question the police officer as complainant.
HELD: POLICE OFFICE IS NOT A COMPLAINANT UNDER RULE 110, PAR 5; COMPLAINANT and WITNESS
QUESTIONED.
The complainant in these cases is not the chief of police. The chief of police signed the complaints in
these cases simply as prosecutor, pursuant to Rule 110, par. 5.

ISSUE 3) Whether the MTC judge may use leading questions for the issuance of arrest warrants.
HELD: COMPLAINANT AND WITNESS ARE TIMID AND SHY; NO OTHER WAY TO ELICIT ANSWERS;
JUDGE IS COMPELLED TO USE LEADING QUESTIONS; ANSWERS STILL BELONGS TO COMPLAINANT AND
WITNESS. THUS, JUDGE USES SEARCHING QUESTIONS.
The was dealing with witnesses who were reticent (timid) and had to be prodded if their
examination was to be searching. The only way to make them explain was to ask them leading
questions. If there is no other way, we see no reason why this should be considered improper. After all,
the witnesses could deny what the judge asked them if it was not true. Nonetheless, the answers were
still those of the witnesses and not those of the judge. Questions satisfactorily falls under searching
questions.

C. Personal Examination (After Examination Under Oath or Affirmation the Complainant and the
Witnesses He May Produce)

Bache & Co. v Ruiz – 37 SCRA 823- Prince


L-32409, February 27, 1971
1. For purpose of gathering documentary evidence in a NIRC violation case, authorized Revenue
Examiner filed for a search warrant before Judge Ruiz.
2. Judge Ruiz heard another case, so he ordered his Clerk of Court and stenographer to take the
deposition of the Revenue Examiner and his witness.
3. After the deposition, the judge read the TSN and deposition. Judge called the complainant and
witnesses. He warned them that if the statements in the TSNs or depositions are false, then
they are liable for perjury. The complainant and witness took the oath and signed the sworn
depositions.
4. Based on the depositions, the judge issued a single search warrant. The search warrant
mentioned many violations under the NIRC.
5. The items to be searched: "Unregistered and private books of accounts (ledgers, journals,
columnars, receipts and disbursements books, customers ledgers); receipts for payments
received; certificates of stocks and securities; contracts, promissory notes and deeds of sale;
telex and coded messages; business communications, accounting and business records; checks
and check stubs; records of bank deposits and withdrawals; and records of foreign remittances,
covering the years 1966 to 1970."
ISSUE 1: Whether the search warrant is valid.
HELD: READING TSN OR DEPOSITION IS NOT PERSONAL EXAMINATION; MORE THAN ONE OFFENSE;
ITEMS ARE STILL GENERAL DESCRIPTION. THUS, INVALID WARRANT.
 The judge failed to personally examine the complainant and his witness. Personal examination
by the judge of the complainant and his witnesses is necessary to enable him to determine the
existence or non-existence of a probable cause.
 The search warrant in question was Issued for at least four distinct offenses under the Tax Code.
“Such is the seriousness of the irregularities committed in connection with the disputed search
warrants, that this Court deemed it fit to amend Section 3 of Rule 122 of the former Rules of
Court that ‘a search warrant shall not issue but upon probable cause in connection with one
specific offense” (Stonehill vs. Diokno).
 The search warrant does not particularly describe the things to be seized. The documents,
papers and effects sought to be seized are described in the Search Warrant.

Soliven v. Makasiar, G.R. Nos L-82585, L-82827, and L-83979, November 14, 1988,167 SCRA 394.-
Prince

1. Pres Cory filed a complaint for libel against Beltran.


2. Beltran claimed that Cory hid under her bed during the coup d' etat.
3. The judge did not personally examine the complainant and witnesses.
4. An arrest warrant was issued against Beltran

ISSUE: Whether the arrest warrant is valid.

HELD: NO PROOF THAT THE JUDGE DEVIATE FROM ESTABLISHED RULES OR PROCEDURES. THUS,
VALID ARREST WARRANTS. PETITION DISMISSED.
Under Sec. 2 of the Bill of Rights, the addition of the word "personally" after the word "determined" and
the deletion of the grant of authority by the 1973 Constitution to Issue warrants to "other responsible
officers as may be authorized by law," does not require the judge to personally examine the
complainant and his witnesses in his determination of probable cause for the issuance of warrants of
arrest. Sec 2 underscores is the exclusive and personal responsibility of the issuing judge to satisfy
himself of the existence of probable cause. But the judge is not required to personally examine the
complainant and his witnesses. He shall:
1. Personally evaluate the report and the supporting documents submitted by the fiscal
regarding the existence of probable cause and, on the basis thereof, issue a warrant of arrest;
or
2. If on the basis thereof he finds no probable cause, he may disregard the fiscal's report and
require the submission of supporting affidavits of witnesses to aid him in arriving at a
conclusion as to the existence of probable cause.
3. Commits grave abuse of discretion when he does comply with no 1 or no 2.
Sound policy dictates this procedure; otherwise judges would be unduly laden with the preliminary
examination and investigation of criminal complaints instead of concentrating on hearing and deciding
cases filed before their courts.

Luna v. Plaza, 26 SCRA 310- Prince


G.R. No. L-27511, November 29, 1968
1. Luna is accused of murder.
2. T-Sgt. Partosa, as investigator, took the statements of complainant and witnesses and reduced
them in writing in form of questions and answers. T-Sgt and witnesses filed their complaint
before municipal judge Plaza including autopsy report and position of the body at the time of
death and other pertinent documents.
3. The municipal judge test the credibility of witnesses and veracity of the sworn statements by
repeatedly reading the questions and answers of the sworn statements to the witnesses.
Witnesses admitted before the court that all the answers to questions were theirs and true.
They swore to the truth of their answers and that they freely and voluntarily made the
answers. The Judge swore-in the witnesses and the latter signed the statements before the
judge.
4. Municipal Judge did not himself personally cause to be reduced to writing in the form of
questions and answers the examination of witnesses presented before him by the person who
filed the criminal complaint. Municipal Judge issued the arrest warrant.
ISSUE: Whether the arrest warrant was valid.
HELD: JUDGE MAY ADOPT THE QUESTIONS OF INVESTIGATOR AS HIS OWN SEARCHING QUESTIONS IN
PERSONALLY EXAMINATION OF COMPLAINANT OR WITNESS. THUS, VALID WARRANT.
Municipal Judge fulfilled RA No. 3828, approved June 22, 1963 and section 87 (e) of the Judiciary Act of
1948: Before a municipal judge may issue a warrant of arrest, the following conditions must first be
fulfilled:
 (1) he must examine the witnesses personally;
 (2) the examination must be under oath;
 (3) the examination must be reduced to writing in the form of searching questions and answers.
RA No. 3828 does not prohibit the municipal Judge from adopting the questions asked by a previous
investigator. Municipal Judge adopted the questions therein in his examination because he considered
them searching questions. If the municipal judge merely accepted the affidavits, without testing the
credibility of the witness and truth of the affidavit, then there is no searching questions. Municipal
judges cannot solely rely upon affidavits or sworn statements that are made to accompany the
complaints that are filed before them, in determining whether there is a probable cause for the issuance
of a warrant of arrest. The purpose of the law is to avoid malicious prosecution.
The term "searching questions and answers" means only, taking into consideration the purpose
of the preliminary examination which is to determine "whether there is a reasonable ground to believe
that an offense has been committed and the accused is probably guilty thereof so that a warrant of
arrest may be issued and the accused held for trial", such questions as have tendency to show the
commission of a crime and the perpetrator thereof. What would be searching questions would depend
on what is sought to be inquired into, such as: the nature of the offense, the date, time, and place of its
commission, the possible motives for its commission; the subject, his age, education, status, financial
and social circumstances, his attitude toward the investigation, social attitudes, opportunities to commit
the offense; the victim, his age, status, family responsibilities, financial and social circumstances,
characteristics, etc. The points that are the subject of inquiry may differ from case to case.

Kho v. Judge Makalintal, GR 94902-06, April 21, 1999- Prince


1. NBI agents and assets conducted surveillance ov2er two residences of the KHO. They disguised
themselves as repairmen and saw the firearms inside the two residences. Acting upon the
surveillance, the agents filed for search warrants before the judge.
2. NBI agents as complainants and their assets as witnesses were appeared before the judge for
inquiry. The agents brought their affidavits for examination. Judge asked: How did you know
that there are unlicensed firearms being kept by Benjamin Kho at No. 45 Bb. Ramona Tirona St.,
Phase I, BF Homes, Paranaque, Metro Manila?
3. Then, the judge issued the search warrants.
4. KHO reasoned that the question is not searching but leading or misleading. Judge should have
asked detailed account on how the agents or assets knew of the presence of unlicensed
firearms.
ISSUE: Whether the search warrants are valid.
HELD: PROBABLE CAUSE BASED UPON ACTUAL AND PERSONAL KNOWLEDGE OF COMPLAINANT AND
WITNESSES; JUDGE USED SEARCHING QUESTIONS. HENCE, VALID WARRANT. PETITION DISMISSED.
In the absence of any showing that respondent judge deviated from his duties in personal examining the
applicant and witness before him, there is no basis for doubting the reliability and correctness of his
findings and impressions.
The application for the questioned search warrants was based on the personal knowledge of the
applicants and their witnesses. To test their personal knowledge, he questioned them. And there was
nothing improper in his examination. Besides, it was within the discretion of the examining Judge to
determine what questions to ask the witnesses so long as the questions asked are germane to the pivot
of inquiry — the existence or absence of a probable cause.

Alvarez v. Court G.R. no. L-45358, January 29, 1937, 64 Phil 33- Prince
1. Acting upon intelligence from an undisclosed source, Chief of the Secret Service filed for a
search warrant before the CFI judge.
2. The Chief believed that Alvarez violated the Anti-Usury Law.
3. The Chief did not swear to an oath that he has personal knowledge.
4. The CFI judge issued the search warrant.
ISSUE: Whether the search warrant was valid.
HELD: NO OATH TO THE PERSONAL KNOWLEDGE OF APPLICANT OR WITNESSES; NO PROBABLE
CAUSE. THUS, INVALID WARRANT.
The warrant issued is likewise illegal because it was based only on the affidavit of the agent who
had no personal knowledge of the Facts. The affidavit of the agent in this case was insufficient because
his knowledge of the Facts was not personal but merely hearsay. When the affidavit of the applicant of
the complaint contains sufficient Facts within his personal and direct knowledge, it is sufficient if the
judge is satisfied that there exist probable cause; when the applicant's knowledge of the Facts is mere
hearsay, the affidavit of one or more witnesses having a personal knowledge of the fact is necessary.

Bache v. Cruz, 37 SCRA 823- Prince


L-32409, February 27, 1971
6. For purpose of gathering documentary evidence in a NIRC violation case, authorized Revenue
Examiner filed for a search warrant before Judge Ruiz.
7. Judge Ruiz heard another case, so he ordered his Clerk of Court and stenographer to take the
deposition of the Revenue Examiner and his witness.
8. After the deposition, the judge read the TSN and deposition. Judge called the complainant and
witnesses. He warned them that if the statements in the TSNs or depositions are false, then
they are liable for perjury. The complainant and witness took the oath and signed the sworn
depositions.
9. Based on the depositions, the judge issued a single search warrant. The search warrant
mentioned many violations under the NIRC.
10. The items to be searched: "Unregistered and private books of accounts (ledgers, journals,
columnars, receipts and disbursements books, customers ledgers); receipts for payments
received; certificates of stocks and securities; contracts, promissory notes and deeds of sale;
telex and coded messages; business communications, accounting and business records; checks
and check stubs; records of bank deposits and withdrawals; and records of foreign remittances,
covering the years 1966 to 1970."
ISSUE 1: Whether the search warrant is valid.
HELD: READING TSN OR DEPOSITION IS NOT PERSONAL EXAMINATION; MORE THAN ONE OFFENSE;
ITEMS ARE STILL GENERAL DESCRIPTION. THUS, INVALID WARRANT.
 The judge failed to personally examine the complainant and his witness. Personal examination
by the judge of the complainant and his witnesses is necessary to enable him to determine the
existence or non-existence of a probable cause.
 The search warrant in question was Issued for at least four distinct offenses under the Tax Code.
“Such is the seriousness of the irregularities committed in connection with the disputed search
warrants, that this Court deemed it fit to amend Section 3 of Rule 122 of the former Rules of
Court that ‘a search warrant shall not issue but upon probable cause in connection with one
specific offense” (Stonehill vs. Diokno).
 The search warrant does not particularly describe the things to be seized. The documents,
papers and effects sought to be seized are described in the Search Warrant.

Borlongan v. Pena, GR 143591, Nov. 23, 2007 - Val

FACTS: Respondent instituted a civil case for recovery of agent’s compensation and expenses,
damages, and attorney’s fees against petitioners, before the Regional Trial Court (RTC). The City
Prosecutor found probable cause for the indictment of petitioners for four (4) counts of the crime of
Introducing Falsified Documents and informations were then filed . Thereafter, the RTC Judge issued the
warrants for the arrest of the petitioners.

Petitioners filed an Omnibus Motion to Quash, Recall Warrants of Arrest and/or For
reinvestigation. Petitioners insisted that they were denied due process because of the non-observance
of the proper procedure on preliminary investigation. Specifically, they argued that since no such
counter-affidavit and supporting documents were submitted by the petitioners, the trial judge merely
relied on the complaint-affidavit and attachments of the respondent in issuing the warrants of arrest,
also in contravention with the Rules of Court.

ISSUE: Can a complaint-affidavit containing matters which are not within the personal knowledge of
the complainant be sufficient basis for the finding of probable cause?

RULING: NO. In the issuance of a warrant of arrest, the mandate of the Constitution is for the judge to
personally determine the existence of probable cause. It does not thereby mean that judges are
obliged to conduct the personal examination of the complainant and his witnesses themselves. Rather,
what is emphasized merely is the exclusive and personal responsibility of the issuing judge to satisfy
himself as to the existence of probable cause.

To this end, he may: (a) personally evaluate the report and the supporting documents submitted by
the prosecutor regarding the existence of probable cause and, on the basis thereof, issue a warrant of
arrest; or (b) if on the basis thereof he finds no probable cause, disregard the prosecutor's report and
require the submission of supporting affidavits of witnesses to aid him in determining its existence.

An arrest without a probable cause is an unreasonable seizure of a person, and violates the privacy of
persons which ought not to be intruded by the State.
People v. Mamaril, GR 147607, Jan 22 2004- Val

FACTS: After the issuance of a search warrant, a search was made in the house where the accused
lives; the search was conducted by the elements of the PNP which yielded a substantial amount of
marijuana and accused was convicted by the RTC for possession of marijuana.

Appellant prays for his acquittal on the ground that Search Warrant No. 99-51 was illegally issued
considering that there was no evidence showing that the required searching questions and answers
were made anent the application for said search warrant. There was no transcript of stenographic
notes of the proceedings in connection with the application for said search warrant. Appellant thus
asserts that it cannot be said that the judge made searching questions upon the alleged applicant and
his witnesses, which is in violation of Section 2, Article III of the Constitution.

ISSUE: Whether there was a valid issuance of the search warrant

RULING: No. The issuance of a search warrant is justified only upon a finding of probable cause.
Probable cause for a search has been defined as such facts and circumstances which would lead a
reasonably discreet and prudent man to believe that an offense has been committed and that the
objects sought in connection with the offense are in the place sought to be searched. In determining the
existence of probable cause, it is required that: (1) the judge must examine the complainant and his
witnesses personally; (2) the examination must be under oath; and (3) the examination must be reduced
in writing in the form of searching questions and answers.31

Based on the above testimony and the other evidence on record, the prosecution failed to prove that
Executive Judge Eugenio G. Ramos put into writing his examination of the applicant and his witnesses
in the form of searching questions and answers before issuance of the search warrant. The records only
show the existence of an application for a search warrant and the affidavits of the complainant’s
witnesses.

We, therefore, find that the requirement mandated by the law that the examination of the
complainant and his witnesses must be under oath and reduced to writing in the form of searching
questions and answers was not complied with, rendering the search warrant invalid. Consequently,
the evidence seized pursuant to said illegal search warrant cannot be used in evidence against appellant
in accordance with Section 3 (2),45 Article III of the Constitution.

Ortiz v. Palaypayon – 234 SCRA 391- Val

FACTS: respondent judge issued an order for the arrest of accused Juliana Lu, Rodrigo Vasquez and
herein complainant, David Ortiz, on the basis of mere affidavits by the offended party and without
conducting the preliminary investigation required by Rule 112 of the Revised Rules of Court..
Furthermore, it is alleged in the administrative complaint that herein complainant was wrongfully
included as one of the accused even if there existed no basis for his indictment of the criminal act.

Respondent judge, claims that he personally examined in writing and under oath the private
complainant and his witnesses by asking the same questions propounded to them in their sworn
statements and the same answers were given. At the same time, respondent judge maintains that it is
customary for him to adopt the sworn statements of the complainant and the witnesses if he is
satisfied of the existence of probable cause. This practice, he claims, will expedite the proceedings and
save the party litigants' time and money.

ISSUE: Whether the order for arrest was validly issued

RULING: No. The respondent judge's action clearly violates constitutional provisions and established
rules of

It is evident that there was no preliminary investigation conducted. Respondent judge did not
personally examine the complainant and her witnesses by asking searching questions and answers to
satisfy himself of the existence of probable cause as mandated by law. He simply ignored the
constitutional requirement of procedural due process. This Court cannot countenance such blatant
practice of disregarding fairly elementary legal principles and substituting it with an unorthodox and
highly irregular practice which appears convenient only to respondent judge.

A person presiding over a court of law must not only apply the law but must live and abide by it and
render justice at all times without resorting to short cuts clearly uncalled for.

The purpose of a preliminary investigation is to secure the innocent against hasty, malicious and
oppressive prosecution, and to protect him from an open and public accusation of crime, from the
trouble, expense and anxiety of a public trial, and also to protect the state from useless and expensive
trials.

D. Particularity of Description

People v. Veloso 48 Phil 169- Val

FACTS: The building located at No. 124 Calle Arzobispo, City of Manila, was used by an organization
known as the Parliamentary Club. Jose Ma. Veloso was at that time a member of the House of
Representative of the Philippine Legislature. He was also the manager of the club.

The police of Manila had reliable information that the so-called Parliamentary Club was nothing more
than a gambling house. A search warrant was then issued in the name of JOHN DOE, and the
description of the place was” namely in the building numbered 124 Calle Arzobispo, City of Manila,
Philippines Islands. By virtue of such the place was raided and when accused was presented with the
warrant, he resisted as he was not “john doe” thus the police had no authority to conduct the search.

ISSUE: Whether the description in the search warrant was sufficient in form

RULING: Yes. Form and Sufficiency of Warrant. Technical accuracy is not required. . . . Name and
description of the accused should be inserted in the body of the warrant and where the name is
unknown there must be such a description of the person accused as will enable the officer to identify
him when found.
Such a warrant must, in addition, contain the best descriptio personae possible to be obtained of the
person or persons to be apprehended, and this description must be sufficient to indicate clearly the
proper person or persons upon whom the warrant is to be served; and should state his personal
appearance and peculiarities, give his occupation and place of residence, and any other circumstances
by means of which he can be identified.

In the first place, the affidavit for the search warrant and the search warrant itself described the building
to be searched as "the building No. 124 Calle Arzobispo, City of Manila, Philippine Islands." It is the
prevailing rule that a description of a place to be searched is sufficient if the officer with the warrant
can, with reasonable effort, ascertain and identify the place intended.

As the search warrant stated that John Doe had gambling apparatus in his possession in the building
occupied by him at No. 124 Calle Arzobispo, City of Manila, and as this John Doe was Jose Ma. Veloso,
the manager of the club, the police could identify John Doe as Jose Ma. Veloso without difficulty.

Alvarez v. CFI – 64 Phil. 33- Val


FACTS: The petitioner asks that the warrant, issued by the Court of First Instance of Tayabas, ordering
the search of his house and the seizure, at any time of the day or night, of certain accounting books,
documents and papers belonging to him in his residence, as well as the order of a later date,
authorizing the agents of the Anti-Usury Board to retain the articles seized, be declared illegal and set
aside, and prays that all the articles in question be returned to him.
One of the grounds alleged by the petitioner in support of his contention that the warrant was
issued illegally is the lack of an adequate description of the books and documents to be seized. The
only description of the articles given in the affidavit presented to the judge was as follows: "that there
are being kept in said premises books, documents, receipts, lists, chits and other papers used by him
in connection with his activities as money-lender, charging a usurious rate of interest, in violation of
the law."

ISSUE: Whether there was a valid description of the things to be seized

RULING: Yes. That a detailed description of the person and place to be searched and the articles to be
seized is necessary, but whereby, by the nature of the articles to be seized, their description must be
rather general, but is not required that a technical description be given, as this would mean that no
warrant could issue;

However search and seizure made are illegal for the following reasons: (a) Because the warrant was
based solely upon the affidavit of the petitioner who had no personal knowledge of the facts of
probable cause, and (b) because the warrant was issued for the sole purpose of seizing evidence which
would later be used in the criminal proceedings that might be instituted against the petitioner, for
violation of the Anti-Usury Law;

Corro v. Lising – 137 SCRA 541- Val

FACTS: respondent RTC judge, issued a Search Warrant authorizing the search and seizure of-
1. Printed copies of Philippine Times;
2. Manuscripts/drafts of articles for publication in the Philippine Times;
3. Newspaper dummies of the Philippine Times;
4. Subversive documents, articles, printed matters, handbills, leaflets, banners;
5. Typewriters, duplicating machines, mimeographing and tape recording machines, video machines and
tapes
which have been used and are being used as instrument and means of committing the crime of
inciting to sedition .

Hence, this petition for certiorari and mandamus, praying (a) that Search Warrant No. Q-00002 issued
by respondent Judge Esteban M. Lising be declared null and void ab initio for the the articles seized
were inadequately described in the search warrant

ISSUE: Whether there was adequate description made

RULING: No. Thus, an application for search warrant must state with particularly the alleged subversive
materials published or intended to be published by the publisher and editor of the Philippine Times,
Rommel Corro. "mere generalization will not suffice." A search warrant should particularly describe
the place to be searched and the things to be seized. "The evident purpose and intent of this
requirement is to limit the things to be seized to those, and only those, particularly described in the
search warrant- to leave the officers of the law with no discretion regarding what articles they should
seize, to the end that unreasonable searches and seizures may not be committed, - that abuses may not
be committed "

In the case at bar, the search warrant issued by respondent judge allowed seizure of printed copies of
the Philippine Times, manuscripts/drafts of articles for publication, newspaper dummies, subversive
documents, articles, etc., and even typewriters, duplicating machines, mimeographing and tape
recording machines. Thus, the language used is so all embracing as to include all conceivable records
and equipment of petitioner regardless of whether they are legal or illegal. The search warrant under
consideration was in the nature of a general warrant which is constitutionally objectionable.

Pangandaman v. Casar, 159 SCRA 599 (1988) - Val

FACTS: The petitioners ask this Court to annul the warrant for their arrest issued by respondent Judge
.

Their plea is essentially grounded on the claim that the warrant for their arrest was issued by the
respondent Judge without a proper preliminary investigation.

when a criminal complaint for multiple murder was filed , respondent Judge "examined personally all
(three) witnesses under oath thru .. (his) closed and direct supervision," reducing to writing the
questions to the witnesses and the latter's answers. Thereafter the Judge "approved the complaint and
issued the corresponding warrant of arrest" against the fourteen (14) petitioners (who were named by
the witnesses) and fifty (50) "John Does." 10

ISSUE: were the warrants of arrest valid?


RULING: Insofar, however, as said warrant is issued against fifty (50) "John Does" not one of whom the
witnesses to the complaint could or would Identify, it is of the nature of a general warrant, one of a
class of writs long proscribed as unconstitutional and once anathematized as "totally subversive of the
liberty of the subject." 30 Clearly violative of the constitutional injunction that warrants of arrest should
particularly describe the person or persons to be seized, 31 the warrant must, as regards its unidentified
subjects, be voided.

Said warrant is voided to the extent that it is issued against fifty (50) "John Does."

Stonehill v. Diokno (1967) – Val

UFACTS: Upon application of the officers of the government Respondents-Judges issued, on different
dates, a total of 42 search warrants against petitioners herein and/or the corporations of which they
were officers,directed to the any peace officer, to search the persons above-named and/or the
premises of their offices, warehouses and/or residences, and to seize and take possession of the
following personal property to wit:

Books of accounts, financial records, vouchers, correspondence, receipts, ledgers, journals, portfolios,
credit journals, typewriters, and other documents and/or papers showing all business transactions
including disbursements receipts, balance sheets and profit and loss statements and Bobbins
(cigarette wrappers).

as "the subject of the offense; stolen or embezzled and proceeds or fruits of the offense," or "used or
intended to be used as the means of committing the offense," which is described in the applications
adverted to above as "violation of Central Bank Laws, Tariff and Customs Laws, Internal Revenue (Code)
and the Revised Penal Code."

Petitioners maintain that the aforementioned search warrants are in the nature of general warrants for
there is no particular description made, and that accordingly, the seizures effected upon the authority
there of are null and void.

ISSUE: Whether the search warrants are valid

RULING: No. Two points must be stressed in connection with this constitutional mandate, namely: (1)
that no warrant shall issue but upon probable cause, to be determined by the judge in the manner set
forth in said provision; and (2) that the warrant shall particularly describe the things to be seized.

None of these requirements has been complied with in the contested warrants.

Thus, the warrants authorized the search for and seizure of records pertaining to all business
transactions of petitioners herein, regardless of whether the transactions were legal or illegal. The
warrants sanctioned the seizure of all records of the petitioners and the aforementioned corporations,
whatever their nature, thus openly contravening the explicit command of our Bill of Rights — that the
things to be seized be particularly described — as well as tending to defeat its major objective: the
elimination of general warrants.
People v. Martinez – 235 SCRA 171- Val
FACTS: After receiving information that appellant was engaged in the sale of shabu at his residence , a
‘buy-bust’ operation was set up by the NBI Regional Office with the assistance of the Zamboanga City
Police which resulted in the confiscation of ‘deck’ of shabu. Pursuant to the operation, Agent Salvo
likewise obtained a search warrant, however such is now being questioned as the accused is Abelardo
Martinez and not Alexander Martinez As stated in the search warrant.

ISSUE: Whether the discrepancy in the name affects the validity of the search warrant

RULING: The discrepancy regarding the name of accused-appellant and that stated in the search
warrant cannot mitigate against his positive identification by the poseur-buyer. It has been
consistently held that "greater weight is given to the positive identification of the accused by the
prosecution witnesses than accused’s denial concerning the commission of the crime .

"The claim of the accused that his true and correct name is not Alexander Martinez but Aberlardo
Martinez does not deviate from the fact of his identity being established as a peddler of shabu. Even
granting arguendo that his (accused) claim is correct that he is not Alexander but Abelardo, that alone
does not warrant dismissal or absolving the accused of criminal liability.

‘It is therefore clear that regardless of the alleged defect of the search warrant in erroneously
designating his first name, the seized articles may still be used as evidence against accused-appellant,
having been obtained from him and as such, fruits of a lawful search incidental to a valid arrest.

Microsoft Corp. v. Maxicorp (2004) - Val


FACTS: “NBI” Agent Samiano filed several applications for search warrants in the RTC against Maxicorp
for alleged violation of Section 29 of PD 49 and Article 189 of the RPC. After conducting a preliminary
examination of the applicant and his witnesses, the Judge William M. Bayhon issued Search Warrants.
Armed with the search warrants, NBI agents conducted a search of Maxicorp’s premises and seized
property fitting the description stated in the search warrants for produced or sold the counterfeit
products.
The disputed text of the search warrants in this case states:

…e) Computer hardware, including central processing units including hard disks, CD-ROM drives,
keyboards, monitor screens and diskettes, photocopying machines and other equipment or
paraphernalia used or intended to be used in the illegal and unauthorized copying or reproduction of
Microsoft software and their manuals, or which contain, display or otherwise exhibit, without the
authority of MICROSOFT CORPORATION, any and all Microsoft trademarks and copyrights; and

ISSUE: WHETHER THE SEARCH WARRANTS ARE “GENERAL WARRANTS.”

RULING: Not as to Sec. €. A search warrant must state particularly the place to be searched and the
objects to be seized. The evident purpose for this requirement is to limit the articles to be seized only to
those particularly described in the search warrant. It is necessary to leave the officers of the law with
no discretion regarding what articles they shall seize, to the end that no unreasonable searches and
seizures be committed.
In addition, under Section 4, Rule 126 of the Rules of Criminal Procedure, a search warrant shall
issue “in connection with one specific offense.” Thus, this rule requires that the warrant must state
that the articles subject of the search and seizure are used or intended for use in the commission of a
specific offense.
It is only required that a search warrant be specific as far as the circumstances will ordinarily
allow. The description of the property to be seized need not be technically accurate or
precise. Measured against this standard we find that paragraph (e) is not a general warrant. The articles
to be seized were not only sufficiently identified physically, they were also specifically identified by
stating their relation to the offense charged. Paragraph (e) specifically refers to those articles used or
intended for use in the illegal and unauthorized copying of petitioners’ software. This language meets
the test of specificity.

However, we find paragraph (c) of the search warrants lacking in particularity. Paragraph (c) states:
c) Sundry items such as labels, boxes, prints, packages, wrappers, receptacles, advertisements
and other paraphernalia bearing the copyrights and/or trademarks owned by MICROSOFT
CORPORATION;
The scope of this description is all-embracing since it covers property used for personal or other
purposes not related to copyright infringement or unfair competition.

Burgos v. Chief of Staff, AFP 133 SCRA 890 – Mikee


FACTS:
Two warrants were issued against petitioners for the search on the premises of “Metropolitan Mail”
(Proj 6, QC) and “We Forum” (Quezon Ave, QC) newspapers and the seizure of items alleged to have
been used in subversive activities as alleged by a broad statement of the military. Petitioners
questioned the warrants for the lack of probable cause and that the two warrants issued indicated
only one and the same address for “Metropolitan Mail” in Proj 6, QC.
ISSUE:
1. Whether or not the defect in the two warrants invalidate the search warrants.
2. Whether or not there was probable cause to justify the search warrant.
3. Whether or not it is required that the property to be searched should be owned by the person
against whom the warrant is directed.
HELD:
1. NO. The defect in the indication of the same address in the two warrants was held by the
court as a typographical error and immaterial in view of the correct determination of the place sought
to be searched set forth in the application. The purpose and intent to search two distinct premises was
evident in the issuance of the two warrant.
2. NO. The statement of the military amounts to a mere conclusion of law unsupported by
particulars.
3. NO. It is sufficient that the property is under the control or possession of the person sought to
to be searched.

Frank Uy v. BIR , 344 SCRA 36- Mikee


Facts:
In Sept 1993, Rodrigo Abos, a former employee of UPC reported to the BIR that Uy Chin Ho aka Frank
Uy, manager of UPC, was selling thousands of cartons of canned cartons without issuing a report. This
is a violation of Sec 253 & 263 of the Internal Revenue Code. In Oct 1993, the BIR requested before RTC
Cebu to issue a search warrant. Judge Gozo-Dadole issued a warrant on the same day. A second
warrant was issued which contains the same substance but has only one page, the same was dated
Oct 1st 2003. These warrants were issued for the alleged violation by Uy of Sec 253. A third warrant was
issued on the same day for the alleged violation of Uy of Sec 238 in relation to sec 263. On the
strength of these warrants, agents of the BIR, accompanied by members of the PNP, on 2 Oct 1993,
searched the premises of the UPC. They seized, among other things, the records and documents of
UPC. A return of said search was duly made by Labaria with the RTC of Cebu. UPC filed a motion to
quash the warrants which was denied by the RTC. They appealed before the CA via certiorari. The CA
dismissed the appeal for a certiorari is not the proper remedy.

Issue:
Whether or not there was a valid search warrant issued.

Held:
The SC ruled in favor of UPC and Uy in a way for it ordered the return of the seized items but
sustained the validity of the warrant. The SC ruled that the search warrant issued has not met some
basic requisites of validity. A search warrant must conform strictly to the requirements of the foregoing
constitutional and statutory provisions. These requirements, in outline form, are:
(1) the warrant must be issued upon probable cause;
(2) the probable cause must be determined by the judge himself and not by the applicant or any other
person;
(3) in the determination of probable cause, the judge must examine, under oath or affirmation, the
complainant and such witnesses as the latter may produce; and
(4) the warrant issued must particularly describe the place to be searched and persons or things to be
seized.

The SC noted that there has been inconsistencies in the description of the place to be searched as
indicated in the said warrants. Also the thing to be seized was not clearly defined by the judge. He used
generic itineraries. The warrants were also inconsistent as to who should be searched. One warrant
was directed only against Uy and the other was against Uy and UPC. The SC however noted that the
inconsistencies were cured by the issuance of the latter warrant as it has revoked the two others.
Section 2, Article III of the Constitution guarantees the right of the people against unreasonable
searches and seizures.

Yousef Al-Ghoul v. CA GR 126859 Sept. 4 , 2001- Mikee


Facts:
Judge Geronimo S. Mangay, presiding judge of the Regional Trial Court, National Capital Judicial Region,
Branch 125, Kalookan City, issued search warrants 54-95 and 55-95 for the search and seizure of
certain items in Apartment No. 2 at 154 Obiniana Compound, Deparo Road, Kalookan City. On April 1,
1995, the police searched Apartment No. 8, in the same compound and found one (1) .45 caliber
pistol. Found in Apartment No. 2 were 2 M-16 rifles with 2 magazines and 20 live M-16 ammunitions.
Petitioners were charged before the Regional Trial Court of Kalookan City, Branch 123, in informations
docketed as Criminal Cases Nos. C-48666-67, accusing them with illegal possession of firearms,
ammunitions and explosives, pursuant to Presidential Decree No. 1866. Thereafter, petitioners were
arrested and detained. Petitioners filed a motion for bail on May 24, 1995, the resolution of which was
held in abeyance by the RTC pending the presentation of evidence from the prosecution to determine
whether or not the evidence presented is strong. On February 7, 1996, at the hearing for bail, the RTC
“admitted all exhibits being offered for whatever purpose that they maybe worth” after the
prosecution had finished adducing its evidence despite the objection by the petitioners on the
admissibility of said evidence.
Issue:
Whether the CA erred and gravely abused its discretion when it ruled that the search and seizure orders
in question are valid and the objects seized admissible in evidence.

Ruling:
The Court ruled that the articles seized during the search of Apartment No. 2 are of the same kind and
nature as those items enumerated in the search warrant above-quoted appears to us beyond cavil.
The items seized from Apartment No. 2 were described with specificity in the warrants in question.
The nature of the items ordered to be seized did not require, in our view, a technical description.
Moreover, the law does not require that the things to be seized must be described in precise and minute
details as to leave no room for doubt on the part of the searching authorities, otherwise, it would be
virtually impossible for the applicants to obtain a search warrant as they would not know exactly what
kind of things they are looking for.[24] Once described, however, the articles subject of the search and
seizure need not be so invariant as to require absolute concordance, in our view, between those seized
and those described in the warrant. Substantial similarity of those articles described as a class or species
would suffice. The petition is PARTIALLY GRANTED. The search conducted at Apartment No. 8 is hereby
declared illegal and the item (.45 caliber pistol) seized therein inadmissible in evidence. However, the
search at Apartment No. 2 pursuant to Search Warrant 55-95 is hereby declared valid and legal, and the
articles seized from Apartment No. 2 are found admissible in evidence.

People v. CA – 291 SCRA 400- Mikee

Paper Industries v. Asuncion, GR 122092 May 19, 1998 – Mikee


FACTS:
1. On January 25, 1995, Police Chief Inspector Napoleon B. Pascua applied for a search warrant before
the said RTC of Quezon City,
2. After propounding several questions to Police Officers Bacolod and Cicero, Judge Maximiano C.
Asuncion issued the search warrant.
3. On February 4, 1995, the police enforced the search warrant at the PICOP compound and seized a
number of firearms and explosives.
4. Believing that the warrant was invalid and the search unreasonable, the petitioners filed a "Motion
to Quash" before the trial court. Subsequently, they also filed a "Supplemental Pleading to the Motion
to Quash" and a "Motion to Suppress Evidence”, which was denied.
ISSUE: WON the warrant is valid.
HELD: No.
1. In the present case, the Search Warrant is Invalid because
a. The Trial Court failed to examine personally the complainant and the other dependents

* Except for Pascua and Bacolod, however, none of the aforementioned witnesses and policemen
appeared before the trial court.

b. SP03 Bacolod had no personal knowledge that petitioners were not licensed to possess the
subject firearms

*When questioned by the judge, Bacolod stated merely that he believed that the PICOP security
guards had no license to possess the subject firearms. This does not meet the requirement that a
witness must testify on his personal knowledge, not belief.
c. The place to be searched was not described with particularity

*The PICOP compound is made up of “200 offices/buildings, 15 plants, 84 staff houses, 1 airstrip, 3
piers/wharves, 23 warehouses, 6 POL depots/quick service outlets and some 800 miscellaneous
structures, all of which are spread out over some one hundred fifty-five hectares. Obviously, the
warrant gives the police officers unbridled and thus illegal authority to search all the structures found
inside the PICOP compound.

Malalaon v. CA, 232 SCRA 249 – Mikee


FACTS: The crime alleged is a violation of PD 1866; illegal possession of firearms act.. The officers
applied for a search warrant in Kalookan City. However, the place to be searched was in Quezon City.
The officers executed the search and seized several firearms. After the search and seizure was
conducted, the accused are now assailing the validity of the search warrant since they claim that it was
void for lack of jurisdiction. The accused buttresses their claim arguing that the criminal case was filed
in the Quezon City RTC, not in Kalookan City.

ISSUE: WON the warrant is valid.

HELD: Yes, the search warrant was valid. A warrant merely constitutes criminal process. It is not a
criminal action. The requisites, procedure and purpose for the issuance of a search warrant are
completely different from those for the institution of a criminal action. Moreover, no law or rule
imposes such a limitation on search warrants, in the same manner that no such restriction is provided
for warrants of arrest.

People v. Estrada – GR 124461, June 26, 2000- Mikee

Facts: Petitioner, People of the Philippines, seeks a partial reconsideration of the said decision that
quashed the search warrant and ordered the return of the seized goods on the ground that it failed to
satisfy the constitutional requirement for the issuance of the warrants, arguing that the seized drugs
subject of the void warrant can no longer be returned because the same are contraband goods. In its
Motion for reconsideration, petitioner attached annexes purporting to show the 52 boxes of medicines
seized were found to be illegally imported.

Issue: Whether or not the seized drugs subject of the void warrant can be returned even if the same are
contraband goods

Held:
No, even if the medicines or drugs seized were genuine and even if they contain the proper
chemicals or ingredients for their production or manufacture, if the producer, manufacturer or seller
has no permit or authority from the appropriate government agency, the drugs or medicines cannot
be returned although the search warrants were declared illegal.

4. Only a Judge May Issue a Warrant

Salazar v. Achcoso, 183 SCRA 145- Mikee


Facts:
Rosalie Tesoro of Pasay City in a sworn statement filed with the POEA, charged petitioner with illegal
recruitment. public respondent Administrator Tomas Achacoso issued a Closure and Seizure Order No.
1205 to petitioner which included the seizure of the documents and paraphernalia being used or
intended to be used as the means of committing illegal recruitment. The team confiscated assorted
costumes. Petitioner filed with POEA a letter requesting for the return of the seized properties,
because she was not given prior notice and hearing. The said Order violated due process. She also
alleged that it violated sec 2 of the Bill of Rights, and the properties were confiscated against her will
and were done with unreasonable force and intimidation.

Issue:
Whether the Philippine Overseas Employment Administration (or the Secretary of Labor) can validly
issue warrants of search and seizure (or arrest) under Article 38 of the Labor Code.

Held:
NO. Under the new Constitution, “. . . no search warrant or warrant of arrest shall Issue except upon
probable cause to be determined personally by the judge after examination under oath or affirmation
of the complainant and the witnesses he may produce, and particularly describing the place to be
searched and the persons or things to be seized”. Mayors and prosecuting officers cannot issue
warrants of seizure or arrest. The Closure and Seizure Order was based on Article 38 of the Labor Code.
The Supreme Court held, “We reiterate that the Secretary of Labor, not being a judge, may no longer
Issue search or arrest warrants. Hence, the authorities must go through the judicial process. To that
extent, we declare Article 38, paragraph (c), of the Labor Code, unconstitutional and of no force and
effect… The power of the President to order the arrest of aliens for deportation is, obviously,
exceptional. It (the power to order arrests) cannot be made to extend to other cases, like the one at bar.
Under the Constitution, it is the sole domain of the courts.” Furthermore, the search and seizure order
was in the nature of a general warrant. The court held that the warrant is null and void, because it
must identify specifically the things to be seized.

Republic (PCGG) v. Sandiganbayan, 255 SCRA 438- Mikee

Petitioner PCGG issued separate orders against private respondents Sipalay Trading Corporation and
Allied Banking Corporation (hereinafter referred to as SIPALAY and ALLIED) to effect their sequestration,
they assail the sequestration orders contending that this order is not one for sequestration but is
particularly a general search warrant which fails to meet the constitutional requisites for its valid
issuance. The Court forwarded the case to Sandiganbayan.
Issue: Was the order of sequestration a search warrant which fell outside the ambit of PCGG’s
authority?

R: YES. There can be no doubt that the order which the PCGG issued against ALLIED typifies a search.
Not only is the order captioned as SEARCH AND SEIZURE ORDER, the body thereof clearly enjoined the
branch manager to make available to the PCGG team all bank documents precisely for the purpose. It
is unauthorized because nowhere in the same Executive Order No. 1[66] (particularly Section 3) invoked
by the PCGG to justify the search and seizure order was the PCGG expressly empowered to issue such
specie of a process in pursuit of its mandated purpose of recovering ill-gotten/unexplained wealth.

Morano v. Vivo, 20* SCRA 562 – Mikee


Constitutionality of Immigration Act of 1940 was questioned where it provided that the Commissioner
of Immigration shall have the power to issue warrants and order the deportation of aliens after a
determination of existence of ground for deportation.

I: may the Commissioner on Immigration issue warrants of arrest contrary to constitutional provision
that only judges after determination of probable cause shall have the power to issue such?

R: yes.
Section 1 (3), Article III of the Constitution, we perceive, does not require judicial intervention in the
execution of a final order of deportation issued in accordance with law. The constitutional limitation
contemplates an order of arrest in the exercise of judicial power as a step preliminary or incidental to
prosecution or proceedings for a given offense or administrative action, not as a measure indispensable
to carry out a valid decision by a competent official, such as a legal order of deportation, issued by the
Commissioner of Immigration, in pursuance of a valid legislation.

In consequence, the constitutional guarantee set forth in Section 1 (3), Article III of the Constitution
aforesaid, requiring that the issue of probable cause be determined by a judge, does not extend to
deportation proceedings.6

Sy v. Domingo – Czar (“CANNOT BE FOUND; NO CITATION”)

Tron Van Nyhia v. Liway, 175 SCRA 318 – Czar

F: This is a petition for issuance of writ of habeas corpus filed by Lucien Tran Van Nghia alleging that he
was arrested without a warrant and deprived of his liberty by respondent Commissioner of
Immigration and Deportation and his agents. Lucien is a French national, who was granted the
admission to the Philippines as a temporary visitor on the basis of his representation that he is
financially capable and will invest in the Philippines. However, he has not made any investment and has
only engaged in tutoring and practice of acupressure up to date. Thus, his land lord accused petitioner
of being an undesirable alien for committing acts inimical to public safety and progress. Acting
thereon, Comm. Liwag issued a Mission Order to locate and bring the subject for proper disposition
and submit report. CID agents went to his house and invited him to go to the police station, however,
he refused to be taken in. Finally, petitioner was subdued and immediately taken to the CID
Intelligence Office. A warrant of arrest was later on issued by Comm. Liwag but there was nothing in
the records to convince this court that said warrant was served on petitioner prior to his
apprehension.

I: Whether or not the French national was illegally arrested?

H: The requirement of probable cause to be determined by a Judge does not extend to deportation
proceedings. There need be no truncated recourse to both judicial and administrative warrants in a
single deportation proceeding. The foregoing does not deviate from the ruling in Qua Chee Gan v.
Deportation Board reiterated in Vivo v. Montesa that under the express terms of our Constitution (1935
Constitution), it is therefore even doubtful whether the arrest of an individual may be ordered by
authority other than a judge if the purpose is merely to determine the existence of a probable cause,
leading to an administrative investigation. What is essential is that there should be a specific charge
against the alien intended to be arrested and deported, that a fair hearing be conducted with assistance
of counsel, if desired, and that the charge be substantiated by competent evidence. The particular
circumstances obtaining in the case at bar have seriously placed on doubt the legality and propriety of
petitioner’s apprehension by respondent Commissioner. For unlike in the Harvey case where the
warrantless capture of 2 suspected alien pedophiles was based on probable cause ascertained only after
close surveillance for a three-month period during which their activities were monitored, herein
petitioner was invited by a combined team of CID agents and police officers at his apartment unit on the
strength of a mission order issued by the individual. The essential requisite of probable cause was
conspicuously absent.

Board of Commissioners v. Judge De La Rosa, 197 SCRA 853- Czar

F: William Gatchalian, et.al was admitted in the Philippines on the basis that his grandfather Santiago
Gatchalian was a native born Filipino. They had with them a certificate of registration and identity
issued by the Philippine Consulate in Hongkong. However, the Board of Commissioners was directed by
the Secretary of Justice to review all cases where the entry was allowed on the ground that the
entrant was a Filipino citizen such included William and his companions. As a result, the decision of the
board of special inquiry recommended for the reversal of the decision of the Board of Commissioners.
Petitioner Comm. Domingo issued a mission order commanding the arrest of respondent William. The
latter appeared before Comm. Domingo and was released on the same day upon posting a cash bond.
William filed a petition for certiorari and prohibition with injunction with the RTC , which was presided
by respondent Judge Dela Rosa. Petitioners filed a motion to dismiss alleging that respondent Judge
has no jurisdiction over the Board of Commissioner and/or Board of Special Inquiry. Nonetheless
respondent Judge Dela Rosa issued the assailed order denying the motion to dismiss. Petitioner now
allege that respondent judge, among others that he gravely abused his discretion in ruling that the
issues raised in the deportation proceedings are beyond the competence and jurisdiction of petitioners,
thereby disregarding the cases of Arocha v. Vivo and Vivo v. Arca (supra), which put finality to the
decision of the Board of Commissioners that respondent Gatchalian is a Chinese citizen.

I: Whether or not the William was illegally arrested.

H: YES. Coming now to the contention of petitioners that the arrest of respondent follows as a matter of
consequence based on the warrant of exclusion issued, coupled with the Arocha and Vivo cases, the
Court finds the same devoid of merit. Sec. 3 (a) of Commonwealth Act No. 13, as amended otherwise
known as the Immigration Act of 1940 reads: Sec. 3 (a) The following aliens shall be arrested upon the
warrant of the Commissioner of Immigration or of any other officer designated by him for the purpose
and deported upon the warrant of the Commissioner of Immigration after a determination by the Board
of Commissioner of the existence of the ground for deportation as charged against the alien. From a
perusal of the above provision, it is clear that in matters of implementing the Immigration Act insofar as
deportation of aliens are concerned, the Commissioner of Immigration may issue warrants of arrest only
after a determination by the Board of Commissioners of the existence of the ground for deportation as
charged against the alien. In other words, a warrant of arrest issued by the Commissioner of
Immigration to be valid must be for the sole purpose of executing a final order of deportation. A
warrant of arrest issued by the Commissioner of Immigration for purposes of investigation only, as in
the case at bar, is null and void for being unconstitutional.

In the case at bar, it took petitioners 28 years since the BOC was rendered their decision before they
commenced deportation or exclusion proceedings against respondent William Gatchalian in 1990.
Undoubtedly, petitioners cause of action has already prescribed. Neither may an action to revive or
enforce the decision be instituted after 10 years.

Harvey v. Santiago 162 SCRA 840- Czar


F: The case stemmed from the apprehension of petitioners from their respective residences by CID
Agents by virtue of Mission Orders issued by respondent Commissioner Santiago of the CID.
Petitioners are presently detained at CID Detention Center. Seized during the petitioner’s apprehension
were rolls of photo negatives and photos of suspected child prostitutes shown in scandalous poses as
well as boys and girls engaged in sex. Posters and other literature advertising the child prostitutes were
also found. Warrants of Arrest were issued against petitioners for violation of Sec37, 45 and 46 of
Immigration Act and sec 69 of Revised Administrative Code. Trial by the Board of Special Inquiry III
commenced the same date. Petition for bail was filed 11March 1988 but was not granted by the
Commissioner of Immigration. Petitioners thereafter filed a petition for Writ of Habeas Corpus.

I: Whether or not said items were legally seized?

H: The right against unreasonable searches and seizure guaranteed by Art. III Sec 2 of 1987 Constitution
is available to all persons, including aliens, whether accused of crime or not.

The arrest of petitioners was based on probable cause determined after the close surveillance for 3
months during which period their activities were monitored. The existence of probable cause justified
the arrest and the seizure of the photo negatives, photographs and posters without warrant. Those
articles were seized as an incident to a lawful arrest and are therefore admissible in evidence.

DOCTRINE: Right against unreasonable searches and seizures available to all persons including aliens
whether accused of a crime or not. Existence of probable cause justified the arrest and the seizure of
the photo negatives, photographs and posters without warrant.

Ho vs. People – 280 SCRA 365- Czar

F: A complaint was filed by the Anti-Graft League of the Philippines against Ho, et.al, for alleged
violation of Sec. 3 (g) of RA 3019 prohibiting a public officer from entering into a contract or
transaction on behalf of the government if it is manifestly and grossly disadvantageous to the latter,
whether or not the public officer profited or will profit thereby. This was dismissed for insufficiency of
evidence. However, after review of the resolution, Special Prosecutor Tamayo recommended that they
be charged. Acting on the foregoing information, the Sandiganbayan issued the now questioned
warrant of arrest against Petitioners Ho and Narciso. Ho initially questioned the issuance thereof. They
alleged that Sandiganbayan, in determining the issuance of warrant merely relied on the information
and resolution attached thereto, in violation of the requirement of Sec. 2 Art III of the Constitution.

I: May a judge issue a warrant of arrest solely on the basis of the report and recommendation of the
investigating prosecutor, without personally determining probable cause by independently examining
sufficient evidence submitted by the parties during the preliminary investigation?

H: We should stress that the 1987 Constitution requires the judge to determine the probable cause
personally. The word personally does not appear in the corresponding provisions of our previous
Constitutions. This emphasis shown the present Constitutions intent to place a greater degree of
responsibility upon trial judges than that imposed under the previous Charters.
While affirming Soliven, Peiple v. Inting elaborated on what determination of probable cause entails,
differentiating the judges object or goal from that of the prosecutor’s. First, the determination of
probable cause is a function of the Judge. It is not for the Provincial Fiscal or Prosecutor nor for the
Election Supervisor to ascertain. Only the Judge and the Judge alone makes this determination.
Second, the preliminary inquiry made by a Prosecutor does not bind the Judge. It merely assists him to
make the determination of probable cause. The Judge does not have to follow what the Prosecutors
present to him. By itself, the Prosecutor’s certification of probable cause is ineffectual. It is the report,
the affidavits the transcripts of stenographic notes (if any) and all other supporting documents behind
the Prosecutor’s certification which are material in assisting the Judge to make his determination. And
third, Judges and Prosecutors alike should distinguish the preliminary inquiry which determines
probable cause for the issuance of a warrant of arrest from preliminary investigation proper which
ascertains whether the offender should be held for trial or released. Even if the 2 inquiries are
conducted in the course of one and the same proceeding, there should be subjected to the epense,
rigors and embarrassment of trial - is the function of the Prosecutor.

*Administrative Arrest (Exceptions to the rule that only a judge may issue a warrant):
Commissioner of Immigration and Deportation may issue warrants to carry out a final finding of a
violation. It is issued after a proceeding has taken place.

Board of Commissioners v. Judge De La Rosa, 197 SCRA 853- Czar

F: William Gatchalian, et.al was admitted in the Philippines on the basis that his grandfather Santiago
Gatchalian was a native born Filipino. They had with them a certificate of registration and identity
issued by the Philippine Consulate in Hongkong. However, the Board of Commissioners was directed by
the Secretary of Justice to review all cases where the entry was allowed on the ground that the
entrant was a Filipino citizen such included William and his companions. As a result, the decision of the
board of special inquiry recommended for the reversal of the decision of the Board of Commissioners.
Petitioner Comm. Domingo issued a mission order commanding the arrest of respondent William. The
latter appeared before Comm. Domingo and was released on the same day upon posting a cash bond.
William filed a petition for certiorari and prohibition with injunction with the RTC , which was presided
by respondent Judge Dela Rosa. Petitioners filed a motion to dismiss alleging that respondent Judge
has no jurisdiction over the Board of Commissioner and/or Board of Special Inquiry. Nonetheless
respondent Judge Dela Rosa issued the assailed order denying the motion to dismiss. Petitioner now
allege that respondent judge, among others that he gravely abused his discretion in ruling that the
issues raised in the deportation proceedings are beyond the competence and jurisdiction of petitioners,
thereby disregarding the cases of Arocha v. Vivo and Vivo v. Arca (supra), which put finality to the
decision of the Board of Commissioners that respondent Gatchalian is a Chinese citizen.

I: Whether or not the William was illegally arrested.

H: YES. Coming now to the contention of petitioners that the arrest of respondent follows as a matter of
consequence based on the warrant of exclusion issued, coupled with the Arocha and Vivo cases, the
Court finds the same devoid of merit. Sec. 3 (a) of Commonwealth Act No. 13, as amended otherwise
known as the Immigration Act of 1940 reads: Sec. 3 (a) The following aliens shall be arrested upon the
warrant of the Commissioner of Immigration or of any other officer designated by him for the purpose
and deported upon the warrant of the Commissioner of Immigration after a determination by the Board
of Commissioner of the existence of the ground for deportation as charged against the alien. From a
perusal of the above provision, it is clear that in matters of implementing the Immigration Act insofar as
deportation of aliens are concerned, the Commissioner of Immigration may issue warrants of arrest only
after a determination by the Board of Commissioners of the existence of the ground for deportation as
charged against the alien. In other words, a warrant of arrest issued by the Commissioner of
Immigration to be valid must be for the sole purpose of executing a final order of deportation. A
warrant of arrest issued by the Commissioner of Immigration for purposes of investigation only, as in
the case at bar, is null and void for being unconstitutional.

In the case at bar, it took petitioners 28 years since the BOC was rendered their decision before they
commenced deportation or exclusion proceedings against respondent William Gatchalian in 1990.
Undoubtedly, petitioners cause of action has already prescribed. Neither may an action to revive or
enforce the decision be instituted after 10 years.

5. “Of Whatever Nature and for Any Purpose”

Material Distributions v. Judge, 84 Phil 127 (1989) – Czar


F: Lope Sarreal filed a complaint seeking a money judgment against petitioners, Material Distributors,
(Phil.) Inc, on three causes of action in the total of P1, 256,229.30. He then filed a supplemental motion
for the production and inspection of the originals of Annexes A and B of the complaint.

The respondent judge, FELIPE NATIVIDAD, granted both motions and, under authority of Rule 27,
issued a subpoena duces tecum for the production and inspection of among others, the books and
papers of Material Distributors (Phil.) Inc.

Material Distributors, (Phil.) Inc questioned the issuance of the subpoena on the ground that it violated
the search and seizure clause.

I: Whether or not the trial judge, in issuing the order, has exceed his jurisdiction or acted with grave
abuse of discretion?

H: (Sec 1) Rule 21 Rules of Court states: Motion for production or inspection; order. - Upon motion of any
party showing good cause therefor and upon notice to all other parties, the court in which an action is
pending may (a) order any party to produce and permit the inspection and copying or photographing,
by or on behalf of the moving party, of any designated documents, papers, books, accounts, letters,
photographs, objects or tangible things, not privileged, which constitute or contain evidence material to
any matter involved in the action and which are in his possession, custody or control

The Court struck down the argument of Material Distributors, (Phil.) Inc and held that the subpoena
pertained to a civil procedure that "cannot be identified or confused with unreasonable searches
prohibited by the Constitution..." The books and papers therein mentioned "constitute or contain the
evidence material to the matters involved in the above entitled case.

The hypothesis that the production and inspection of books and documents in question is tantamount
to a search warrant is erroneous. The procedure outlined by Rule 21 and followed by respondent judge
place them outside the realm of the prohibited unreasonable searches.

Oklahoma Press v. Walling, 327 US 186- Czar


F: A subpoena duces tecum was issued to produce specified records to determine whether Petitioners
Oklahoma Press Pub. Co. were violating the Fair Labor Standards Act. Petitioners however claims that
the questions of coverage must be adjudicated before said subpoenas may be enforced. Furthermore,
they contend that said subpoenas violated their rights against illegal searches and seizures secure by
the Fourth Ammendment.

I: Whether or not said subpoenas were illegally issued?

H: The intent of Congress, in authorizing investigations supported by subpoenas and enforcement


orders, was to enable the Administrator to discover and procure evidence, not to prove a pending
charge or complaint, but to enable him to make one if, in his judgment, the facts thus discovered
should justify doing so.

Moreover, the corporate records sought to be inspected are not covered by the Constitutional
prohibition against self-incrimination, even though such documents may contain evidence tending to
subject any or all of the officers of a corporation to a criminal indictment.

It has been settled that corporations are not entitled to all of the constitutional protections which
private individuals have in these and related matters. As has been noted, they are not at all within the
privilege against self-incrimination, although this Court more than once has said that the privilege runs
very closely with the Fourth Amendment's search and seizure provisions.

Camara v. Municipal Court, 387 US 523 ( 1967) – Czar

F: An inspector entered an apartment building to make a routine annual inspection for possible
violations of the city’s Housing Code. The inspector was informed that the Appellant was using a part of
his leasehold as personal residence, which he was prohibited to do so. When confronted by several
times by the inspector, the Appellant did not allow the inspector to enter the premises because no he
do not have a warrant. A complaint was filed against the Appellant for alleged violations of the city’s
Housing Code. His demurrer was denied and he filed a writ of prohibition. The Court of appeals upheld
that the housing section does not violate Fourth Amendment rights because it is part of a regulatory
scheme which is essentially civil rather than criminal in nature, and that section creates a right of
inspection which is limited in scope and may not be exercise in unreasonable conditions.

I: W]hether administrative inspection programs, as presently authorized and conducted, violate Fourth
Amendment rights as those rights are enforced against the States through the Fourteenth
Amendment?”

H: General rule: Except in certain carefully defined cases, a search of private property without proper
consent is unreasonable unless it has been authorized by a valid search warrant.

Frank v. Maryland
(This was the governing doctrine, which the Court, in this case, overturned soit s landmark):

Facts: Appellant refused warrantless search of private premises for the purpose of locating and
abating a public nuisance. He was convicted for refusal and the Courtupheld it.

Ratio: Justifications for permitting administrative health and safety inspections without warrant
• Municipal fire, health and housing inspection programs are merely to determine if the
premises are complying with the minimum standards set in municipal ordinances and not to search
for evidence of criminal action. The 4th amendment interests at stake here are merely peripheral .
• They are designed to make the least possible demand on the individual occupant
• The warrant process could not function properly in this field-Inspections of entire municipal
areas are based on broad factors such as the area’s age and condition

Here’s where the Court overturns

Under this system, the occupant has no way of knowing what the search is about, what the limits are,
and WON the agent is acting upon proper authorization. He must first question the search and risk a
criminal conviction for not allowing it. The practical effect is to give the official wide discretion in the
field. Fire, housing and sanitation inspections may still be made. What we are disallowing is making
them without warrant. There is no evidence that these types of inspection programs cannot achieve
their goals within the standards of the warrant process.

Thus, warrantless searches of these kinds, like the case at bar, are violative of the 4th amendment.

Need for probable cause. Unlike searches pursuant to criminal investigation, the purpose of these kinds
of warrants is city wide compliance with the minimum physical standards for private property. The
governmental interest here is the prevention of conditions, which will become a hazard to public health
and safety. The only way to implement this is through routine periodic inspections of all structures.
Probable cause then is based on an appraisal of a certain area as a whole.

What is required is probable cause arising from knowledge of condition of the district or area, and not
necessarily individual buildings.

When a warrant must be sought routine inspections are not so urgent as to have to take effect
immediately without warrant. Furthermore, citizens usually allow routine inspections of their premises
anyway even without warrant. It is believed that warrants need only be sought when the citizen has
refused the warrantless search, unless there have been citizen complaints or other compelling reasons
to effect immediate entry. Appellant had constitutional right to insist that the inspectors obtain a
warrant to search and appellant may not be convicted for refusing consent to the inspection.

6. Warrantless Searches and Seizures

A. General Rule: Get a Search Warrant.

People v. Aminuddin, 163 SCRA 402- Czar


F: Aminnuddin was arrested shortly after disembarking from the M/V Wilcon 9 in Iloilo City. The Police
Officers who were in fact waiting for him because of a tip from one their informers simply accosted him
inspected his bag and finding what looked liked marijuana leaves, then they took him to the
headquartes for investigation. It was found to contain 3 kilos of what were later analyzed as marijuana
leaves. Information for violation of Dangerous Drugs Act was filed. In his defense, Aminnuddin
disclaimed the marijuana, averring that all he had in his bag was his clothing consisting of a jacket, 2
shirts and 2 pair of pants. He alleged that he was arbitrarily arrested and immediately handcuffed. His
bag was confiscated without a search warrant.
I: Whether the search was illegal.

H: Yes. The search was illegal. Defendant was not caught in flagrante delicto, which could allow
warrantless arrest or search. At the moment of his arrest, he was not committing a crime. Nor was he
about to do so or had just done so. To all appearances, he was like any of the other passengers
innocently disembarking from the vessel. The said marijuana therefore could not be appreciated as
evidence against the defendant, and furthermore he is acquitted of the crime as charged.

People v. Valdez, 341 SCRA 85 – Anton


FACTS:
On the strength of an informer’s tip, policemen searched the premises owned by appellant where
there were marijuana plants. they confiscated the same and used it as evidence against appellant. All of
these actions were done without securing a search warrant notwithstanding the fact that one day had
lapsed since they obtained the said tip from an informer.

ISSUE: Whether the search and seizure is valid.

RULING:

NO. The Constitution lays down the general rule that a search and seizure must be carried on the
strength of a judicial warrant. Otherwise, the search and seizure is deemed "unreasonable." Evidence
procured on the occasion of an unreasonable search and seizure is deemed tainted for being the
proverbial fruit of a poisonous tree and should be excluded. Such evidence shall be inadmissible in
evidence for any purpose in any proceeding.
In the instant case, there was no search warrant issued by a judge after personal determination of the
existence of probable cause. From the declarations of the police officers themselves, it is clear that they
had at least one (1) day to obtain a warrant to search appellant's farm. Their informant had revealed his
name to them. The place where the cannabis plants were planted was pinpointed. From the information
in their possession, they could have convinced a judge that there was probable cause to justify the
issuance of a warrant. But they did not. Instead, they uprooted the plants and apprehended the accused
on the excuse that the trip was a good six hours and inconvenient to them.

B. When is a search a “search”?

Valmonte v. General de Villa – 178 SCRA 211 (Main) and 185 SCRA 655 (MR) - Anton
FACTS:
As part of its duty to maintain peace and order, the NCRDC installed checkpoints in various parts of
Valenzuela, Metro Manila.

Petitioners Atty. Ricardo Valmonte, who is a resident of Valenzuela, Metro Manila, and the Union of
Lawyers and Advocates For People’s Rights (ULAP) sought the declaration of checkpoints in Valenzuela,
Metro Manila and elsewhere as unconstitutional. In the alternative, they prayed that respondents
Renato De Villa and the National Capital Region District Command (NCRDC) be directed to formulate
guidelines in the implementation of checkpoints for the protection of the people. Petitioners
contended that the checkpoints gave the respondents blanket authority to make searches and seizures
without search warrant or court order in violation of the Constitution.

ISSUE: Do the military and police checkpoints violate the right of the people against unreasonable search
and seizures?

RULING:
NO, military and police checkpoints DO NOT violate the right of the people against unreasonable
search and seizures. However, it must be limited to plain view.

Not all searches and seizures are prohibited. Those which are reasonable are not forbidden. A
reasonable search is not to be determined by any fixed formula but is to be resolved according to the
facts of each case.

Where, for example, the officer merely draws aside the curtain of a vacant vehicle which is parked on
the public fair grounds, or simply looks into a vehicle, or flashes a light therein, these do not constitute
unreasonable search.

ON MOTION FOR RECONSIDERATION:


The Court further ruled that routine checkpoint stop does intrude, to a certain extent, on motorist's
right to "free passage without interruption", but it cannot be denied that, as a rule, it involves only a
brief detention of travellers during which the vehicle's occupants are required to answer a brief question
or two. For as long as the vehicle is neither searched nor its occupants subjected to a body search,
and the inspection of the vehicle is limited to a visual search, said routine checks cannot be regarded
as violative of an individual's right against unreasonable search.

Guazon v. De Villa – 181 SCRA 623- Anton


FACTS:
The 41 petitioners alleged that the "saturation drive" or "aerial target zoning" that were conducted in
their place (Tondo Manila) were unconstitutional. They alleged that there is no specific target house
to be search and that there is no search warrant orwarrant of arrest served. Most of the policemen are
in their civilian clothes and without nameplates or identification cards. The residentswere rudely rouse
from their sleep by banging on the walls and windows of their houses. The residents were at the point of
high-powered guns and herded like cows. Men were ordered to strip down to their briefs for the police
to examine their tattoo marks. The residentscomplained that their homes were ransacked, tossing
their belongings and destroying their valuables. Some of their money and valuables had disappeared
after the operation. The residents also reported incidents of maulings, spot-beatings and maltreatment.
Those who were detained also suffered mental and physical torture to extract confessions and tactical
informations. The respondents said that such accusations were all lies.

ISSUE: Whether the search in the case is the one contemplated by law.

RULING:
YES. It is obvious that the search is a search contemplated by the Constitution. It is this kind of search
which is generally prohibited without first obtaining a valid warrant. It is not the police action which is
prohibited. It is the illegal search. Searches such as the one in the case at bar need be preceded by a
duly issued search warrant for it not to infringe upon a person’s constitutionally protected right.

C. No Presumption of Regularity in Search Cases

People v. Tudtud, GR 144037, Sept 26, 2003- Anton


FACTS:
Facts: The accused were apprehended due to an information that they were dealing in drugs. The
accused cry frame – up. When they alighted the bus, they were approached by a police officer while
pointing a gun at them and ordered them to open their bag which “contained” marijuana. They were
convicted by the RTC.

ISSUE: Whether the search is valid.

RULING:
NO. Because a warrantless search is in derogation of a constitutional right, peace officers who conduct it
cannot invoke regularity in the performance of official functions and shift to the accused the burden of
proving that the search was unconsented.
In any case, any presumption in favor of regularity would be severely diminished by the allegation of
appellants in this case that the arresting officers pointed a gun at them before asking them to open
the subject box.
Appellants’ implied acquiescence, if at all, could not have been more than mere passive conformity
given under coercive or intimidating circumstances and is, thus, considered no consent at all within the
purview of the constitutional guarantee. Consequently, appellants’ lack of objection to the search and
seizure is not tantamount to a waiver of his constitutional right or a voluntary submission to the
warrantless search and seizure.

Sony Music v. Judge Espanol, GR 156804, March 14, 2005- Anton


FACTS:
Relying on petitioner’s allegation, agents of the NBI applied for a search warrant before respondent
judge. The agents relied on the presumption of regularity of the performance of their service in
alleging that they personal knowledge so as to warrant the application for the search warrant. The
application was initially granted but subsequently quashed by respondent judge on the ground that
there is no probable cause.

ISSUE: Whether the agents may rely on the presumption of regularity.

RULING:
NO. To prevent stealthy encroachment upon, or gradual depreciation of the right to privacy, a liberal
construction in search and seizure cases is given in favor of the individual. Consistent with this
postulate, the presumption of regularity is unavailing in aid of the search process when an officer
undertakes to justify it. For, the presumption juris tantum of regularity cannot, by itself, prevail against
the constitutionally protected rights of an individual because zeal in the pursuit of criminals cannot
ennoble the use of arbitrary methods that the Constitution itself detests.

D. Instances of Warrantless Searches and Seizures


List:

People v. Sevilla – 339 SCRA 625- Anton


FACTS:
Above-named accused, conspiring, confederating and mutually aiding and abetting each other,
without authority of law, did then and there, wilfully, unlawfully and feloniously have in their
possession, control and custody four (4) bricks of marijuana dried leaves with fruiting tops
approximately weighing four thousand (4,000) grams.

Relying on the fact that there is a warrant of arrest served upon on appellant, policemen searched and
seized in his possession the said marijuana.

ISSUE: What are the instances of valid warrantless search and seizures?

RULING:
There are certain cases where the law itself allows a search even in the absence of a warrant.
Jurisprudence mentions the following instances under which a warrantless search and seizure may be
effected, to wit:
1. Search which is incidental to a lawful arrest ( Rule 126, Section 12, Rules of Court);
2. Seizure of evidence in “plain view”;
3. Search of a moving vehicle;
4. Consented warrantless search;
5. Customs search;
6. Stop and Frisk;
7. Exigent and emergency circumstances.
8. Right voluntarily waived
9. Aerial target zoning
10. Vessels and aircrafts
However, there is no evidence to support that the search and seizure was under any of the instances
enumerated. Neither can the police rely on the fact that there is a warrant of arrest issued for the arrest
of appellant. For the latter to be properly invoked, the search should be contemporaneous with the
arrest. In this case, it was not. Hence, the search was held to be invalid.

i. Incidental to a Lawful Arrest


Sec. 12 Rule 16, Rules of Court

Two Requisites:
1. Item to be searched was within the arrestee’s custody or area of immediate control.
2. Search was contemporaneous with an arrest.
Padilla v. CA, GR 121917 March 12, 1997- Anton
FACTS:
The lower court ordered the arrest of petitioner since high-powered firearms were found in their
possession. These firearms were seized and used against them.

ISSUE: Is the search and seizure of the firearms valid?

RULING:
YES. It is valid notwithstanding the absence of a valid search warrant for the search in the case at bar
is incidental to a lawful arrest. Their seizure without a search warrant is justified under a
search incidental to a lawful arrest. Once the lawful arrest was effected, the police may undertake a
protective search of the passenger compartment and containers in the vehicle which are within
petitioner's grabbing distance regardless of the nature of the offense. This satisfied the two-tiered test
of an incidental search: (i) the item to be searched (vehicle) was within the arrestee's custody or area of
immediate control and (ii) the search was contemporaneous with the arrest.

Espano v. CA 288 SCRA 558 (1998) – Anton


FACTS:
Acting on a tip, police officers were able to arrest petitioner as he was selling marijuana. They seized a
total of 12 bags of marijuana, 10 of which were found at the house of petitioner after his arrest. All of
the bags were used as evidence against him.

ISSUE: Can the 10 bags found at petitioner’s residence be used against him?

RULING:
NO. Only the two bags can be used. The search and seizure of the 2 bags is justified even without a
search warrant as it is incidental to a lawful arrest. The articles seized from petitioner during his arrest
were valid under the doctrine of search made incidental to a lawful arrest. The warrantless search made
in his house, however, which yielded ten cellophane bags of marijuana became unlawful since the
police officers were not armed with a search warrant at the time. Moreover, it was beyond the reach
and control of petitioner.

People v. De Lara – 236 SCRA 291- Anton


FACTS:
A surveillance was made based on an information about the rampant drug-related activities. Appellant
was caught in flagrante delicto in selling two foils of marijuana to one of the policemen’s asset.
Appellant fled towards his house. More prohibited drugs were found inside appellant’s house.

ISSUE: Are the prohibited drugs seized inside appellant’s house admissible in evidence against him?

RULING:

YES. The policemen's entry into the house of appellant without a search warrant was in hot-pursuit of
a person caught committing an offense in flagrante. The arrest that followed the hot-pursuit was valid.
The seizure of the plastic bag containing prohibited drugs was the result of appellant's arrest inside his
house. A contemporaneous search may be conducted upon the person of the arrestee and the
immediate vicinity where the arrest was made
People v. Leangsiri – 252 SCRA 213- Anton
FACTS:
Accused Leangsiri was arrested at the arrival area of the Ninoy Aquino International Airport (NAIA). He
was in the act of bringing into the country 8,225.31 grams of heroin hidden under the false bottom of
a black suitcase.
He informed the authorities that he was to deliver the contraband to three (3) people at the Las Palmas
Hotel in Manila. The arrest was made in Rm. 504 of the said hotel. Thereafter, authorities followed one
of the accused to Rm 413 where another evidence was found and was seized.

ISSUE: Whether or not the evidence found in Rm 413 is admissible against accused.

RULING:
NO. In the case at bar, appellants were arrested in Room 504 of the Las Palmas Hotel. The piece of
paper bearing Leangsiri’s name was obtained through a warrantless search of Room 413 of the same
hotel, and found tucked within the pages of appellant Amidu’s telephone and address book. Clearly, the
warrantless search is illegal and the piece of paper bearing Leangsiri’s name cannot be admitted as
evidence against appellants. The inadmissibility of this evidence will not, however, exculpate
appellants. Its exclusion does not destroy the prosecution’s case against appellants. The remaining
evidence still established their guilt beyond reasonable doubt.

People v. Cuenco – GR 128277, Nov. 16, 1998 – Kenny

G.R. No. 128277 November 16, 1998

PEOPLE OF THE PHILIPPINES, plaintiff-appellee,


vs.
FERDINAND CUENO y MATA, accused-appellant.

FACTS: Policemen conducted a search of the residence of the accused. Inside the bedroom of the
accused, SPO1 SARMIENTO found a balikbayan box which contained dirty clothes, a brick of dried
flowering tops of marijuana wrapped in a newspaper with a gross weight of 803 grams and twenty one
(21) plastic tea bags containing dried flowering tops of marijuana with a gross weight of 48.5842 grams
together with rolling papers. Appellant would question his conviction in Criminal Case No. 38-95
allegedly because of the illegality of the search made in his house.

ISSUE: Whether the search conducted in the residence of the accused is valid.

RULING: YES. The arrest of appellant has been made in the course of a buy-bust operation, thus, in
flagrante delicto. A buy-bust operation — a form of entrapment which has repeated been accepted to
be a valid means of arresting violators of the Dangerous Drugs Law 15 — is far variant from an ordinary
arrest. In lawful arrests, it becomes both the duty and the right of the apprehending officers to conduct
a warrantless search not only on the person of the suspect but also in the permissible area within his
reach, i.e., that point which is within the effective control of the person arrested, or that which may
furnish him with the means of committing violence or of escaping. Under this set-up, it becomes
advisable, if not necessary, or the peace officers to forthwith undertake a search of the house as being
within the "permissible area."
People v. Che Chun Ting – 328 SCRA 592- Kenny

G.R. Nos. 130568-69 March 21, 2000

PEOPLE OF THE PHILIPPINES, plaintiff-appellee,


vs.
CHE CHUN TING alias "DICK," accused-appellant.

FACTS: Upon seeing the door of the unit open and a man handing out to Mabel a transparent plastic bag
containing a white crystalline substance,the NARCOM agents immediately alighted and arrested the
surprised man who was positively identified by Mabel as Che Chun Ting. Then the agents conducted a
search of Unit 122. During the search SPO3 Campanilla seized a black bag with several plastic bags
containing a white crystalline substance in an open cabinet at the second floor. Accused Che Chun Ting
now contends among others, that the trial court erred in convicting him on the basis of the shabu seized
inside Unit 122, which was constitutionally inadmissible as evidence since it was seized without a search
warrant.

ISSUE: Whether the search conducted inside Unit 122 was valid.

RULING: NO. The right is not absolute and admits of certain well-recognized exceptions. For instance, a
person lawfully arrested searched for dangerous weapons or anything which may be used as proof of
the commission of the offense, without a search warrant. 11 The search may extend beyond the person
of the one arrested to include the permissible area or surroundings within his immediate control. The
accused was admittedly outside unit 122 and in the act of delivering to Mabel Cheung Mei Po a bag of
shabu when he was arrested by the NARCOM operatives. Moreover, it is borne by the records that Unit
122 was not even his residence but that of his girlfriend Nimfa Ortiz, and that he was merely a sojourner
therein. Hence, it can hardly be said that the inner portion of the house constituted a permissible area
within his reach or immediate control, to justify a warrantless search therein.

The lawful arrest being the sole justification for the validity of the warrantless search under the
exception, the same must be limited to and circumscribed by the subject, time and place of the arrest.
As to subject, the warrantless search is sanctioned only with respect to the person of the suspect, and
things that may be seized from him are limited to "dangerous weapons" or ''anything which may be
used as proof of the commission of the offense." With respect to the time and place of the warrantless
search, it must be contemporaneous with the lawful arrest. Stated otherwise, to be valid, the search
must have been conducted at about the time of the arrest or immediately thereafter and only at the
place where the suspect was arrested, or the premises or surroundings under his immediate control.

We therefore hold that the search in Unit 122 and the seizure therein of some 5,578.68 grams of
shabu do not fall within the exception, hence, were illegal for being violative of one's basic
constitutional right and guarantee against unreasonable searches and seizures.
ii. Plain View

Requisites:
1. Prior valid intrusion
2. Evidence was inadvertently discovered by the police
3. Illegality of the evidence is immediately apparent; and
4. Noticed without further search.

People v. Evaristo, 216 SCRA 413- Kenny

G.R. No. 93828 December 11, 1992

PEOPLE OF THE PHILIPPINES, plaintiff-appellee,


vs.
SANTIAGO EVARISTO and NOLI CARILLO, accused-appellants.

FACTS: Sgt. Romeroso sought Evaristo's permission to scour through the house, which was granted. In
the sala, he found, not Rosillo, but a number of firearms and paraphernalia supposedly used in the
repair and manufacture of firearms, all of which, thereafter, became the basis for the present
indictment against Evaristo.

ISSUE: Whether the lower court gravely erred in admitting Exhibits "B" to "F" in evidence considering
that those are illegally seized evidence.

RULING: NO. It is to be noted that what the above constitutional provisions prohibit
are unreasonable searches and seizures. For a search to be reasonable under the law, there must, as a
rule, be a search warrant validly issued by an appropriate judicial officer. Yet, the rule that searches and
seizures must be supported by a valid search warrant is not an absolute and inflexible rule, for
jurisprudence has recognized several exceptions to the search warrant requirement. Among
these exceptions is the seizure of evidence in plain view, adopted by this jurisdiction from the
pronouncements of the United States Supreme Court in Harris vs. U.S. 4 and Coolidge vs. New
Hampshire. 5 Thus, it is recognized that objects inadvertently falling in the plain view of an officer who
has the right to be in the position to have that view, are subject to seizure and may be introduced in
evidence. 6

The records in this case show that Sgt. Romerosa was granted permission by the appellant Evaristo to
enter his house. The officer's purpose was to apprehend Rosillo whom he saw had sought refuge
therein. Therefore, it is clear that the search for firearms was not Romerosa's purpose in entering the
house, thereby rendering his discovery of the subject firearms as inadvertent and even accidental.

People v. Tabar, 222 SCRA 144 (1993) – Kenny


G.R. No. 101124 May 17, 1993

PEOPLE OF THE PHILIPPINES, plaintiff-appellee,


vs.
CARMELINA TABAR y CARMILOTES and ROMMEL ARRIESGADO y TABAR, accused.CARMELINA TABAR
y CARMILOTES, accused-appellant.

FACTS: The trial court promulgated its decision, dated 17 December 1990, 4 finding Carmelina "guilty,
beyond reasonable doubt, for violation of Section 4, Article II RA 6425, otherwise known as the
Dangerous Drug Act of 1972 as amended by PD 1675" and sentencing her to "Reclusion Perpetua and to
pay a fine of P20,000.00 for the act of selling and distributing marijuana." Appellant maintains that the
marijuana cigarettes seized from her are inadmissible in evidence because they were obtained in
violation of the constitutional guarantee against unreasonable search and seizure.

ISSUE: Whether the evidences gathered are admissible.

RULING: The evidence for the prosecution discloses that the appellant placed the packs of marijuana
sticks under the rolled pair of pants which she was then carrying at the time she hurriedly left her
shanty after noticing the arrest of Rommel. When she was asked to spread it out, which she voluntary
did, the package containing the packs of marijuana sticks were thus exposed in plain view to the
member of the team. A crime was thus committed in the presence of the policemen. Pursuant to
Section 5, Rule 113 and Section 12 Rule 126 of the Revised Rules of Court, she could lawfully be arrested
and searched for anything which may be used as proof of the commission of an offense without the
corresponding arrest and search warrants.

Roan v. Gonzales, 145 SCRA 687- Kenny

G.R. No. 71410 November 25, 1986

JOSEFINO S. ROAN, petitioner,


vs.
THE HONORABLE ROMULO T. GONZALES, respondents.

FACTS: Petitioner claims he was the victim of an illegal search and seizure conducted by the military
authorities. The articles seized from him are sought to be used as evidence in his prosecution for illegal
possession of firearms. He asks that their admission be temporarily restrained and thereafter
permanently enjoined. The challenged search warrant was issued by the respondent judge on May 10,
1984. The petitioner's house was searched two days later but none of the articles listed in the warrant
was discovered. 3 However, the officers conducting the search found in the premises one Colt Magnum
revolver and eighteen live bullets which they confiscated. They are now the bases of the charge against
the petitioner. The respondents also argue that the Colt Magnum pistol and the eighteen bullets seized
from the petitioner were illegal per se and therefore could have been taken by the military authorities
even without a warrant.

ISSUE: Whether the seized Colt Magnum pistol and eighteen bullets are admissible in as much as the
same were not listed in the search warrant, but were found by the military after the search.

RULING: It is true that there are certain instances when a search may be validly made without warrant
and articles may be taken validly as a result of that search. For example, a warrantless search may be
made incidental to a lawful arrest, as when the person being arrested is frisked for weapons he may
otherwise be able to use against the arresting officer. Motor cars may be inspected at borders to
prevent smuggling of aliens and contraband and even in the interior upon a showing of probable
cause. Vessels and aircraft are also traditionally removed from the operation of the rule because of
their mobility and their relative ease in fleeing the state's jurisdiction. The individual may knowingly
agree to be searched or waive objections to an illegal search. And it has also been held that prohibited
articles may be taken without warrant if they are open to eye and hand and the peace officer comes
upon them inadvertently.

Clearly, though, the instant case does not come under any of the accepted exceptions. The respondents
cannot even claim that they stumbled upon the pistol and bullets for the fact is that these things were
deliberately sought and were not in plain view when they were taken. Hence, the rule having been
violated and no exception being applicable, the conclusion is that the petitioner's pistol and bullets were
confiscated illegally and therefore are protected by the exclusionary principle.

United Laboratories v. Isip – GR 163858 (June 28, 2005) – Kenny

G.R. No. 163858 June 28, 2005

UNITED LABORATORIES, INC., petitioner,


vs.
ERNESTO ISIP and/or SHALIMAR PHILIPPINES and/or OCCUPANTS, Shalimar Building, No. 1571,
Aragon Street, Sta. Cruz, Manila, respondents.

FACTS: Respondents alleged that the seized 792 bottles of Disudrin 60 ml. and 30 boxes of Inoflox 200
mg. are INADMISSIBLE as evidence against the respondents because they constitute the "fruit of the
poisonous tree." The petitioner asserts that the description in the search warrant of the products to be
seized – "finished or unfinished products of UNILAB" – is sufficient to include counterfeit drugs within
the premises of the respondents not covered by any license to operate from the BFAD, and/or not
authorized or licensed to manufacture, or repackage drugs produced or manufactured by UNILAB.
Citing the ruling of this Court in Padilla v. Court of Appeals,28 the petitioner asserts that the products
seized were in plain view of the officers; hence, may be seized by them. The petitioner posits that the
respondents themselves admitted that the seized articles were in open display; hence, the said
articles were in plain view of the implementing officers.
ISSUE: whether the search conducted by the NBI officers of the first and second floors of the Shalimar
building and the seizure of the sealed boxes which, when opened, contained Disudrin.

RULING: NO. It was thus incumbent on the NBI agents and the petitioner to prove their claim that the
items were seized based on the plain view doctrine. It is not enough to prove that the sealed boxes were
in the plain view of the NBI agents; evidence should have been adduced to prove the existence of all the
essential requirements for the application of the doctrine during the hearing of the respondents’ motion
to quash, or at the very least, during the hearing of the NBI and the petitioner’s motion for
reconsideration on April 16, 2004. The immediately apparent aspect, after all, is central to the plain view
exception relied upon by the petitioner and the NBI. There is no showing that the NBI and the petitioner
even attempted to adduce such evidence. In fact, the petitioner and the NBI failed to present any of the
NBI agents who executed the warrant, or any of the petitioner’s representative who was present at the
time of the enforcement of the warrant to prove that the enforcing officers discovered the sealed boxes
inadvertently, and that such boxes and their contents were incriminating and immediately apparent. It
must be stressed that only the NBI agent/agents who enforced the warrant had personal knowledge
whether the sealed boxes and their contents thereof were incriminating and that they were
immediately apparent.65 There is even no showing that the NBI agents knew the contents of the
sealed boxes before they were opened.

In sum then, the Court finds and so hold that the petitioner and the NBI failed to prove the essential
requirements for the application of the plain view doctrine.

People v. Doria – GR 125299, Jan. 22, 1999- Kenny

G.R. No. 125299 January 22, 1999

PEOPLE OF THE PHILIPPINES, plaintiff-appellee,


vs.
FLORENCIO DORIA y BOLADO and VIOLETA GADDAO y CATAMA @ "NENETH," accused-appellants.

The police officers found the door of "Neneth's" house open and a woman inside. "Jun" identified the
woman as his associate. 7 SPO1 Badua asked "Neneth" about the P1,600.00 as PO3 Manlangit looked
over "Neneth's" house. Standing by the door, PO3 Manlangit noticed a carton box under the dining
table. He saw that one of the box's flaps was open and inside the box was something wrapped in
plastic. The plastic wrapper and its contents appeared similar to the marijuana earlier "sold" to him by
"Jun." His suspicion aroused, PO3 Manlangit entered "Neneth's" house and took hold of the box. He
peeked inside the box and found that it contained ten (10) bricks of what appeared to be dried
marijuana leaves. The Regional Trial Court, Branch 156, Pasig City convicted the accused-appellants. The
trial court found the existence of an "organized/syndicated crime group."
ISSUE: THE COURT A QUO GRAVELY ERRED IN ADMITTING AS EVIDENCE THE MARIJUANA FRUITINGS
FOUND INSIDE THE CARTON BOX AS THESE WERE OBTAINED THROUGH A WARRANTLESS SEARCH AND
DOES NOT COME WITHIN THE PLAIN VIEW DOCTRINE.

Since the warrantless arrest of accused-appellant Gaddao was illegal, it follows that the search of her
person and home and the subsequent seizure of the marked bills and marijuana cannot be deemed legal
as an incident to her arrest. This brings us to the question of whether the trial court correctly found that
the box of marijuana was in plain view, making its warrantless seizure valid. Objects falling in plain view
of an officer who has a right to be in the position to have that view are subject to seizure even without a
search warrant and maybe introduced in evidence. 121 The "plain view" doctrine applies when the
following requisites concur: (a) the law enforcement officer in search of the evidence has a prior
justification for an intrusion or is in a position from which he can view a particular area; (b) the
discovery of the evidence in plain view is inadvertent; (c) it is immediately apparent to the officer that
the item he observes may be evidence of a crime, contraband or otherwise subject to seizure. 122 The
law enforcement officer must lawfully make an initial intrusion or properly be in a position from which
he can particularly view the area. 123 In the course of such lawful intrusion, he came inadvertently across
a piece of evidence incriminating the accused. 124 The object must be open to eye and hand and its
discovery inadvertent.

It is clear that an object is in plain view if the object itself is plainly exposed to sight. The difficulty
arises when the object is inside a closed container. Where the object seized was inside a closed package,
the object itself is not in plain view and therefore cannot be seized without a warrant. However, if the
package proclaims its contents, whether by its distinctive configuration, its transparency, or if its
contents are obvious to an observer, then the contents are in plain view and may be seized. 127 In other
words, if the package is such that an experienced observer could infer from its appearance that it
contains the prohibited article, then the article is deemed in plain view. It must be immediately
apparent to the police that the items that they observe may be evidence of a crime, contraband or
otherwise subject to seizure.

Del Rosario v. People, GR 142295, May 31, 2001- Kenny

G.R. No. 142295 May 31, 2001

VICENTE DEL ROSARIO y NICOLAS, petitioner,


vs.
PEOPLE OF THE PHILIPPINES, respondent.

FACTS: "On June 13, 1996, a search warrant (Exhibit A) was issued authorizing the search of the
residence of appellant. When the appellant came out, P/Sr. Insp. Adique informed him that they had a
search warrant and that they were authorized to search his house. After appellant gave his permission,
the police officers conducted a search of the house. The search yielded the following items: (a) a
caliber .45 pistol with Serial No. 703792 with five magazines of caliber .45 (Exhibits B and H) found at
the master's bedroom; (b) five magazines of 5.56 M-16 rifle and two radios (Exhibits C to C-4) found in
the room of appellant's daughter; and (c) a caliber .22 revolver with Serial No. 48673 (Exhibit F)
containing 8 pieces of live ammunition (Exhibit M) found in the kitchen of the house. When asked about
his license to possess the firearms, the appellant failed to produce any. This prompted the police
officers to seize the subject firearms. Petitioner submits that the search conducted at his residence
was illegal the search warrant was issued in violation of the Constitution 11 and consequently, the
evidence seized was inadmissible. He also submits that he had a license for the .45 caliber firearm and
ammunition seized in his bedroom. The other firearm, a .22 caliber revolver seized in a drawer at the
kitchen of his house, a magazine for 5.56 mm. cal. Armalite rifle, and two 2-way radios found in his
daughter's bedroom, were either planted by the police or illegally seized, as they were not mentioned
in the search warrant.

ISSUE: Whether the .22 caliber revolver seized in a drawer at the kitchen of his house, a magazine for
5.56 mm. cal. Armalite rifle and two 2-way radios found in his daughter's bedroom, were planted by the
police or were illegally seized.

RULING: With respect to the .22 caliber revolver with Serial No. 48673, that the police raiding team
found in a drawer at the kitchen of petitioner's house, suffice it to say that the firearm was not
mentioned in the search warrant applied for and issued for the search of petitioner's house. Seizure is
limited to those items particularly described in a valid search warrant. In this case, the firearm was
not found inadvertently and in plain view. It was found as a result of a meticulous search in the
kitchen of petitioner's house. This firearm, to emphasize, was not mentioned in the search warrant.
Hence, the seizure was illegal. The seizure without the requisite search warrant was in plain violation of
the law and the Constitution.51 True that as an exception, the police may seize without warrant illegally
possessed firearm or any contraband for that matter, inadvertently found in plain view. However, "[t]he
seizure of evidence in 'plain view' applies only where the police officer is not searching for evidence
against the accused, but inadvertently comes across an incriminating object."

Hence, the petitioner rightly rejected the firearm as planted and not belonging to him. The
prosecution was not able to prove that the firearm was in the effective possession or control of the
petitioner without a license. The same is true with respect to the 5.56 cal. magazine found in the
bedroom of petitioner's daughter. The seizure was invalid and the seized items were inadmissible in
evidence. With particular reference to the two 2-way radios that the raiding policemen also seized in the
bedroom of petitioner's daughter, there was absolutely no reason for the seizure.

iii. Moving Vehicle


There must be a highly reasonable suspicion amounting to probable cause that the occupant
committed a criminal activity.

Hizon v. Court of Appeals, 265 SCRA 517 (1996) – Kenny

G.R. No. 119619 December 13, 1996

RICHARD HIZON, petitioners,


vs.
HONORABLE COURT OF APPEALS and THE PEOPLE OF THE PHILIPPINES, respondents.

FACTS: On September 30, 1992 at about 2:00 in the afternoon, the Task Force Bantay Dagat reported to
the PNP Maritime Command that a boat and several small crafts were fishing by "muro ami" within the
shoreline of Barangay San Rafael of Puerto Princesa. The police immediately proceeded to the area and
found several men fishing in motorized sampans and a big fishing boat identified as F/B Robinson
within the seven-kilometer shoreline of the city. They boarded the F/B Robinson and inspected the boat
with the acquiescence of the boat captain, Silverio Gargar. The police discovered a large aquarium full
of live lapu-lapu and assorted fish weighing approximately one ton at the bottom of the boat. 2 They
checked the license of the boat and its fishermen and found them to be in order. Nonetheless, SP03
Enriquez brought the boat captain, the crew and the fishermen to Puerto Princesa for further
investigation.The petitioners primarily question the admissibility of the evidence against petitioners in
view of the warrantless search of the fishing boat and the subsequent arrest of petitioners. More
concretely, they contend that the NBI finding of sodium cyanide in the fish specimens should not have
been admitted and considered by the trial court because the fish samples were seized from the F/B
Robinson without a search warrant.

Whether the evidence is admissible in as much as it was obtained without a valid search warrant.

Search and seizure without search warrant of vessels and aircrafts for violations of customs laws have
been the traditional exception to the constitutional requirement of a search warrant. It is rooted on the
recognition that a vessel and an aircraft, like motor vehicles, can be quickly moved out of the locality or
jurisdiction in which the search warrant must be sought and secured. Yielding to this reality, judicial
authorities have not required a search warrant of vessels and aircrafts before their search and seizure
can be constitutionally effected. The same exception ought to apply to seizures of fishing vessels and
boats breaching our fishery laws. These vessels are normally powered by high-speed motors that enable
them to elude arresting ships of the Philippine Navy, the Coast Guard and other government authorities
enforcing our fishery laws.

We thus hold as valid the warrantless search on the F/B Robinson, a fishing boat suspected of having
engaged in illegal fishing. The fish and other evidence seized in the course of the search were properly
admitted by the trial court. Moreover, petitioners failed to raise the issue during trial and hence,
waived their right to question any irregularity that may have attended the said search and seizure. 23

Bagalihog v. Fernandez – 198 SCRA 614- Kenny

G.R. No. 96356 June 27, 1991

NONILLON A. BAGALIHOG, petitioner,


vs.
HON. JUDGE GIL P. FERNANDEZ, Presiding Judge of Br. 45, RTC of Masbate; and MAJOR JULITO
ROXAS,respondents.

FACTS: On March 17, 1989, Rep. Moises Espinosa was shot to death shortly after disembarking at the
Masbate Airport. Witnesses said one of the gunmen fled on a motorcycle. On the same day, the
petitioner's house, which was near the airport, was searched with his consent to see if the killers had
sought refuge there. The search proved fruitless.Two days later, Capt. Julito Roxas and his men from
the Philippine Constabulary seized the petitioner's motorcycle and took it to the PC headquarters in
Masbate. They had no search warrant. The motorcycle was impounded on the suspicion that it was one
of the vehicles used by the killers. After investigation, the petitioner and several others were charged
with multiple murder and frustrated murder for the killing of Espinosa and three of his bodyguards and
the wounding of another person.

Whether the motorcycle may be validly seized.

The mere mobility of the motorcycle did not make the search warrant redundant for it is not denied
that the vehicle remained with the petitioner until it was forcibly taken from him. The fear that it
would be dismantled or hidden was mere speculation that was not borne out by the facts. The necessity
for the immediate seizure of the motorcycle without the prior obtention of a warrant has not been
established.

It is true that are certain instances when a search when a search may be taken validly made without
warrant and articles may be taken validly as a result of that search. For example, a warrantless search
may be made incidental to a lawful arrest, as when the person being arrested is frisked for weapons he
may otherwise be able to use against the arresting officer. Motor cars may be inspected at borders to
prevent smuggling of aliens and contraband and even in the interior upon a showing of probable cause.
Vessels and aircraft are also traditionally removed from the operation of the rule because of their
mobility and their relative ease in fleeing the state's jurisdiction. The individual may knowingly agree to
be searched or waive objections to an illegal search. And it has also been held that prohibited articles
may be taken without warrant if they are open to eye and hand and the peace officer comes upon them
inadvertently.

The case at bar does not come under any of the above specified exceptions. The warrantless seizure of
the motorcycle was unquestionably violative of "the right to be let alone" by the authorities as
guaranteed by the Constitution. The vehicle cannot even be detained on the ground that it is a
prohibited article the mere possession of which is unlawful.

FRANCISCO B. ANIAG, JR. vs. COMMISSION ON ELECTIONS [237 SCRA 424 G.R. No. 104961 October 7,
1994]

Facts: Upon the issuance of declaration of gun ban by the Comelec in connection to the national & local
election, the Sgt-at-Arms of the House of Representatives requested petitioner to return the 2 firearms
issued by the House to him. In compliance, petitioner ordered his driver Arellano to pick up the
firearms in his house to return them to Congress. On his way back to the Batasan Complex, Arellano
was flagged down in a check point and police search the car. Upon finding the guns, he was
apprehended and detained and his case was referred for inquest to the City prosecutor office.
Petitioner was not made a party to the charge but was invited to shed light on the incident. Petitioner
explained the purpose how Arellano came to have the firearms boarded on the car and wrote the
prosecutor to exonerate Arellano from the charges. The prosecutor recommended dismissing the case.
The Comelec however issued a resolution filing information in violation of the gun ban against
petitioner. Petitioner moves for reconsideration to the Comelec which was denied hence this petition
contending that the search on his car was illegal and that he was not impleaded as respondent in the
preliminary investigation and his constitutional rights for due process was violated.

Issue: Whether or not petitioner was denied of due process of law.


Ruling: The court held that as a rule, a valid search must be authorized by a search warrant duly issued
by an appropriate authority. However, this is not absolute. Aside from a search incident to a lawful
arrest, a warrantless search had been upheld in cases of (1) moving vehicles (2) the seizure of evidence
in plain view and (3) search conducted at police or military checkpoints which are not illegal for as long
as the vehicle is neither searched nor its occupants subjected to a body search, and the inspection of the
vehicle is merely limited to a visual search, and (4) Stop-and-search without warrant conducted by police
officers on the basis of prior confidential information which were reasonably corroborated by other
attendant matters is also recognized by the court to be legal.

An extensive search without warrant could only be resorted to if the officers conducting the search had
reasonable or probable cause to believe before the search that either the motorist was a law offender
or that they would find the instrumentality or evidence pertaining to the commission of a crime in the
vehicle to be searched. Because there was no sufficient evidence that would impel the policemen to
suspect Arellano to justify the search they have conducted, such action constitutes an unreasonable
intrusion of the petitioner’s privacy and security of his property in violation of Section 2, Article III of
the Constitution. Consequently, the firearms obtained in violation of petitioner's right against
warrantless search cannot be admitted for any purpose in any proceeding. The manner by which
COMELEC proceeded against petitioner runs counter to the due process clause of the Constitution. The
facts show that petitioner was not among those charged by the PNP with violation of the Omnibus
Election Code. Nor was he subjected by the City Prosecutor to a preliminary investigation for such
offense. Thus the court declared the warrantless search and seizure of the firearms as illegal hence
inadmissible to court as evidence in any proceeding against the petitioner.
PEOPLE OF THE PHILIPPINES vs. IDEL AMINNUDIN y AHNI, [G.R.No. 74869 July 6, 1988]

Facts: The police agents in Iloilo City received a tip from a reliable informer that the accused,
Aminnudin, was on a vessel bound for Iloilo and is carrying with him marijuana. The said vessel was to
arrive few days after such tip. On the day of the arrival, the agents then waited at the port for the vessel.
Upon arrival of the vessel and when the suspect disembarked, they immediately frisked him and
searched his bag which contained the marijuana. Subsequently, the Aminnudin was arrested. During the
trial, the accused that alleged that he was arbitrarily arrested and immediately handcuffed and that his
bag was confiscated without a search warrant.

Issue: Whether or not there was a valid arrest effected.

Ruling: No. The police agents had enough time to secure a warrant to arrest and search the accused but
did not do so. In addition to this, the arrest did not fall into any of the exceptions of a valid warrantless
arrest because the accused-appellant was not, at the moment of his arrest, committing a crime nor was
it shown that he was about to do so or that he had just done so.

In the case at bar, there was no warrant of arrest or search warrant issued by a judge after personal
determination by him of the existence of probable cause. Contrary to the averments of the government,
the accused-appellant was not caught in flagrante nor was a crime about to be committed or had just
been committed to justify the warrantless arrest allowed under Rule 113 of the Rules of Court. Even
expediency could not be invoked to dispense with the obtention of the warrant as in the case of Roldan
v. Arca, 24 for example. Here it was held that vessels and aircraft are subject to warrantless searches
and seizures for violation of the customs law because these vehicles may be quickly moved out of the
locality or jurisdiction before the warrant can be secured.
The present case presented no such urgency. From the conflicting declarations of the PC witnesses, it is
clear that they had at least two days within which they could have obtained a warrant to arrest and
search Aminnudin who was coming to Iloilo on the M/V Wilcon 9. His name was known. The vehicle
was Identified. The date of its arrival was certain. And from the information they had received, they
could have persuaded a judge that there was probable cause, indeed, to justify the issuance of a
warrant. Yet they did nothing. No effort was made to comply with the law. The Bill of Rights was ignored
altogether because the PC lieutenant who was the head of the arresting team, had determined on his
own authority that a "search warrant was not necessary."
THE PEOPLE OF THE PHILIPPINES vs. MIKAEL MALMSTEDT [G.R. No. 91107 June 19, 1991]
Facts: Mikael Malmstedt, a Swedish national, was found, via a routine NARCOM inspection at
Kilometer 14, Acop, Tublay Mountain Province, carrying Hashish, a derivative of Marijuana. RTC La
Trinidad found him guilty for violation of the Dangerous Drugs Act. The accused filed a petition to the
Supreme Court for the reversal of the decision arguing that the search and the arrest made was illegal
because there was no search warrant.

Issue:Whether such arrest was invalid for lack of warrant.

Ruling: No. The Constitution guarantees the right of the people to be secure in their persons, houses,
papers and effects against unreasonable searches and seizures. However, where the search is made
pursuant to a lawful arrest, there is no need to obtain a search warrant. A lawful arrest without a
warrant may be made by a peace officer or a private person under the following circumstances.
Sec. 5 Arrest without warrant; when lawful. –– A peace officer or a private person may, without a
warrant, arrest a person:(a) When, in his presence, the person to be arrested has committed is actually
committing, or is attempting to commit an offense; (b) When an offense has in fact just been
committed, and he has personal knowledge of facts indicating that the person to be arrested has
committed it; and (c) When the person to be arrested is a prisoner who has escaped from a penal
establishment or place where he is serving final judgment or temporarily confined while his case is
pending, or has escaped while being transferred from one confinement to another.
In cases falling under paragraphs (a) and (b) hereof, the person arrested without a warrant shall be
forthwith delivered to the nearest police station or jail, and he shall be proceeded against in accordance
with Rule 112, Section 7.
however, under the circumstances of the case, there was sufficient probable cause for said officers to
believe that accused was then and there committing a crime.
the NARCOM officers merely conducted a routine check of the bus (where accused was riding) and the
passengers therein, and no extensive search was initially made. It was only when one of the officers
noticed a bulge on the waist of accused, during the course of the inspection, that accused was
required to present his passport. The failure of accused to present his identification papers, when
ordered to do so, only managed to arouse the suspicion of the officer that accused was trying to hide
his identity.
THE PEOPLE OF THE PHILIPPINES vs. LO HO WING alias PETER LO, LIM CHENG HUAT alias ANTONIO
LIM and REYNALDO TIA y SANTIAGO [G.R. No. 88017 January 21, 1991]

Facts: Peter Lo et al. were charged with a violation of the Dangerous Drugs Act, for the transport of
metamphetamine hydrochloride, otherwise known as "shabu". The drug was contained in tea bags
inside tin cans which were placed inside their luggages. Upon arrival from Hongkong, they boarded the
taxis at the airport which were apprehended by Criminal Investigation Service (CIS) operatives. Their
luggages were subsequently searched where the tea bags were opened and found to contain shabu.
Only Lo and Lim were convicted. Tia was discharged as a state witness, who turned out to be a "deep
penetration agent" of the CIS in its mission to bust the drug syndicate .

Issue: Whether such seizure was legal.

Ruling: YES. That search and seizure must be supported by a valid warrant is not an absolute rule. One of
the exceptions thereto is a search of a moving vehicle. The circumstance of the case clearly show that
the search in question was made as regards a moving vehicle.
Therefore, a valid warrant was not necessary to effect the search on appellant and his co-accused. It was
firmly established from the factual findings of the court that the authorities had reasonable ground to
believe that appellant would attempt to bring in contraband and transport within the country. The belief
was based on intelligence reports gathered from surveillance activities on the suspected syndicate, of
which appellant was touted to be a member. Aside from this, they were also certain as to the expected
date and time of arrival of the accused from China via Hongkong. But such knowledge was insufficient to
enable them to fulfill the requirements for the issuance of a search warrant. Still and all, the important
thing is that there was probable cause to conduct the warrantless search, which must still be present in
the case.
PEOPLE OF THE PHILIPPINES vs ALVARO SAYCON y BUQUIRAN [G.R. No. 110995 September 5, 1994]
Facts: Alvaro Saycon was charged with violating the Dangerous Drugs Act when he was found with
"shabu". At 6am a Coastguard personnel received information from a NARCOM agent that a suspected
"shabu" courier by the name of Alvaro Saycon was on board the MV Doña Virginia, which was arriving
at that moment in Dumaguete City. Upon receipt of the information, the Coastguard chief officer
ordered a combined team of NARCOM agents and Philippine Coastguard personnel to intercept the
suspect.
The vessel docked at 6:00am that same morning at Dumaguete City. Saycon alighted from the boat
carrying a black bag and went through the checkpoint manned by the Philippine Coastguard where he
was identified by a police officer of the NARCOM. Saycon was then invited to the Coastguard
Headquarters at the Pier area and he willingly went with them. At the headquarters, Saycon was asked
to open his bag and he willingly obliged. In it were personal belongings and a maong wallet. Inside that
maong wallet, there was a cigarette pack containing the suspected "shabu". When asked whether the
cigarette pack containing the suspected "shabu" was his, Saycon merely bowed his head. Then, Saycon,
his bag and the suspected "shabu" were brought to the NARCOM office for booking. When Alvaro
Saycon was arrested, the NARCOM agents did not have a warrant of arrest.
Issue: Whether the warrantless arrest valid.
Ruling: Yes. The requirement that a judicial warrant must be obtained prior to the carrying out of a
search and seizure is not absolute. "There are certain exceptions recognized in our law,"
Peace officers may lawfully conduct searches of moving vehicles without need of a warrant, it not being
practicable to secure a judicial warrant before searching a vehicle, since such vehicle can be quickly
moved out of the locality or jurisdiction in which the warrant may be sought.
While the analogy is perhaps not perfect, we consider that appellant Saycon stands in the same
situation as the driver or passenger of a motor vehicle that is stopped by police authorities and
subjected to an extensive search. In this situation, the warrantless search and arrest of appellant
Saycon would be constitutionally permissible only if the officer conducting the search had reasonable or
probable cause to believe, before the search, that Saycon who had just disembarked from the MV Doña
Virginia upon arrival of that vessel at 6:00 a.m. of 8 July 1992 at Pier I of Dumaguete city, was violating
some law or that the contents of his luggage included some instrument or the subjects matter or the
proceeds of some criminal offense.
Clearly, the NARCOM Agents had to act quickly but there was not enough time to obtain a search
warrant or a warrant of arrest. It was realistically not possible for either the NARCOM Agents or the
Coastguard Officers to obtain a judicial search warrant or warrant of arrest in the situation presented by
the case at bar.
PEOPLE OF THE PHILIPPINES vs.COURT OF FIRST INSTANCE OF RIZAL, BRANCH IX, QUEZON CITY,
presided by HON. ULPIANO SARMIENTO, JESSIE HOPE and MONINA MEDINA [G.R. No. L-41686
November 17, 1980]
Facts: Judge Ulpiano Sarmiento in Criminal Case No. Q-3781 which stalled the prosecution of
respondents Sgt. Jessie C. Hope and Monina Medina for the alleged violation of section 3601 of the
Tariff and Customs Code. The order declared as inadmissible in evidence the allegedly smuggled articles
obtained by apprehending agents in the course of a warrantless search and seizure.
the Regional Anti-Smuggling Action Center (RASAC) was informed 1week before the incident by an
undisclosed Informer that a shipment of highly dutiable goods would be transported to Manila from
Angeles City on a blue Dodge car.
a light blue Dodge car driven by Sgt. Jessie Hope who was accompanied by Monina Medina approached
the exit gate and after giving the toll receipt sped away towards Manila. The RASAC agents gave a chase
and overtook Sgt. Hope's car. the agents succeeded in blocking Sgt. Hope's car and the latter stopped.
An inspection of Sgt. Hope's car at Camp Aguinaldo yielded eleven (11) sealed boxes, contents of the
boxes revealed some "wrist watches of assorted brands” supposedly untaxed.
Issue: whether or not the seizure of the merchandise in a moving vehicle by authorized agents
commissioned to enforce customs laws without warrant of seizure breaches the constitutional immunity
against unreasonable search and seizure and therefore, such merchandise are inadmissible in evidence.
Ruling: Yes. the rules governing search and seizure had been liberalized when a moving vehicle is the
object of the search and the necessity of a prior warrant has been relaxed on the ground of practicality,
considering that before a warrant could be obtained, the place, things and persons to be searched must
be described to the satisfaction of the issuing judge — a requirement which borders on impossibility in
the case of smuggling effected by the use of a moving vehicle that can transport contraband from one
place to another with impunity.
PEOPLE OF THE PHILIPPINES vs. BONIFACIO BARROS [G.R. No. 90640 March 29, 1994]
Facts: Bonifacio Barros was charged with violating Section 4 of R.A. No. 6425, as amended (known as the
Dangerous Drugs Act of 1972). while being a passenger in a Dangwa Bus, destined for Baguio City, carry
with him as part of his baggage and transport about four (4) kilos of dried marijuana. Events transpired
as follows: both M/Sgt. Yag-as and S/Sgt. Ayan, who were seated at the back, saw accused carrying a
carton, board the bus and seated himself after putting the carton under his seat. Thereafter, M/Sgt. Yag-
as and S/Sgt. Ayan before they alighted, it being their station, inspected the carton and found out that it
contained marijuana. They invited the herein accused to the detachment for questioning as accused was
the suspected owner of the carton containing marijuana. As both P.C. officers Yag-as and Ayan saw
accused, Bonifacio Barros carrying that same carton when he boarded the bus.
Issue: whether or not the arrest of the accused was valid.
Ruling: No. When, a vehicle is stopped and subjected to an extensive search, such a warrantless search
would be constitutionally permissible only if the officers conducting the search have reasonable or
probable cause to believe, before the search, that either the motorist is a law-offender or the
contents or cargo of the vehicle are or have been instruments or the subject matter or the proceeds of
some criminal offense.
In the case at bar, however, we have been unable to find in the record of this case any circumstance
which constituted or could have reasonably constituted probable cause for the peace officers to search
the carton box allegedly owned by appellant Barros. The carrying of such a box by appellant onto a
passenger bus could not, by itself, have convinced M/Sgt. Francis Yag-as and S/Sgt. James Ayan either
that the appellant was a law violator or the contents of the box were instruments or the subject matter
or proceeds of some criminal offense. The carrying of carton boxes is a common practice among our
people, especially those coming from the rural areas since such boxes constitute the most economical
kind of luggage possible. The peace officers here involved had not received any information or "tip-off"
from an informer; no such a "tip-off" was alleged by the police officers before or during the trial. The
police officers also did not contend that they had detected the odor of dried marijuana, or appellant
Barros had acted suspiciously in the course of boarding the bus and taking a seat during the trip to
Sabangan, nor in the course of being asked whether he owned the carton box later ascertained to
contain four (4) kilos of marijuana. The testimony of the law enforcement officers who had
apprehended the accused (M/Sgt. Francis Yag-as and S/Sgt. James Ayan), and who had searched the box
in his possession, (C2C Fernando Bongyao), simply did not suggest or indicate the presence of any such
probable cause.
no circumstance which might reasonably have excited the suspicion of the two (2) police officers riding
in the same bus as appellant Barros. They asked the police officers at the checkpoint at Sabangan to
inspect the box allegedly carried by appellant Barros apparently on a mere guess that appellant Barros
might be carrying something in the nature of contraband goods. There was, in other words, nothing to
show that appellant Barros was then in the process of "actually committing" or "attempting to commit"
a crime.
MUSTANG LUMBER, INC vs. HON. COURT OF APPEALS, [G.R. No. 104988. June 18, 1996]
Facts: On 1 April 1990, Special Actions and Investigation Division (SAID),acting on information that a
huge pile of narra flitches, shorts, and slabs were seen inside the lumberyard of Mustang Lumber,
conducted a surveillance at Mustang lumberyard. The team saw a truck loaded with lauan and almaciga
lumber coming out of the lumberyard. Since the driver could not produce the required invoices and
transport documents, the team seized the truck together with its cargo and impounded them at DENR
compound. On 3 April 1990,RTC Valenzuela issued a search warrant. On same day, the team seized from
the lumberyard narra shorts, trimmings and slabs, narra lumber, and various species of lumber and
shorts. On 4 April 1990, team returned to the lumberyard and placed under administrative seizure
(owner retains physicalpossession of seized articles, only an inventory is taken) the remaining lumber
because Mustang Lumber failed to produce required documents upon demand. DENR Sec Factoran
suspended Mustang Lumber’s permit and confiscated in favor of the gov’t the seized articles. Mustang
Lumber filed for a TRO against Factoran and Robles, and questioned the validity of the April 1 and 4
seizure. RTC held that the warrantless seizure on April 1 is valid as it comes within the exceptions where
warrantless seizure is justified (search of a moving vehicle), and April 4 seizure was also valid pursuant to
the search warrant issued on April 3. CA affirmed. Mustang lumber filed a petition for review on
certiorari.
Issue: Whether or not the seizure conducted by the SAID is valid.
Ruling: Yes. the petitioner's truck was coming out from the petitioner's lumberyard loaded with lauan
and almaciga lumber of different sizes and dimensions which were not accompanied with the required
invoices and transport documents. The seizure of such truck and its cargo was a valid exercise of the
power vested upon a forest officer or employee by Section 80 of P.D. No. 705, as amended by P.D. No.
1775. Then, too, as correctly held by the trial court and the Court of Appeals in the FIRST CIVIL CASE, the
search was conducted on a moving vehicle. Such a search could be lawfully conducted without a search
warrant.
Search of a moving vehicle is one of the five doctrinally accepted exceptions to the constitutional
mandate that no search or seizure shall be made except by virtue of a warrant issued by a judge after
personally determining the existence of probable cause.
PEOPLE OF THE PHILIPPINES vs. NORIEL LACERNA y CORDERO & MARLON LACERNA y ARANADOR.
[G.R. No. 109250. September 5, 1997]
Facts: Appellant sets up the defense of denial, alleging that the blue plastic bag was owned by his uncle
who requested him to bring it to Iloilo. He also denied knowing that it contained marijuana.
Accused Marlon and Noriel Lacerna were riding in a taxicab on their way to (the) North Harbor to board
a boat bound for Iloilo City. While plying along Pier 15 their taxicab was flagged down by a patrol mobile
car. Accused Marlon Lacerna (appellant herein) was sitting in front while accused Noriel Lacerna was at
the back of the taxicab. The accused carried two bags. One bag contained their personal belongings
and the other bag contained things which their uncle Edwin Lacerna asked them to bring along. When
their taxicab was stopped, the two policemen in the Mobile car requested them that they and their
baggage be searched. Confident that they have not done anything wrong, they allowed to be searched.
During the (search), the two accused were not allowed to alight from the taxicab. After checking, the
policemen found marijuana thus led to their arrest.

The defense argues that the bricks of marijuana were inadmissible in evidence as they were obtained
through illegal search and seizure. Appellant alleges that at the time of the search and seizure, he and
his co-accused were not committing any crime as they were merely riding a taxicab on the way to Pier
15, North Harbor in Manila. Hence, the precipitate arrest and seizure violated their constitutional right
and the marijuana seized constituted “fruits of the poisonous tree.”

The Solicitor General disagrees, contending that the search and seizure were consistent with recent
jurisprudential trend liberalizing warrantless search and seizure where the culprits are riding moving
vehicles, because a warrant cannot be secured in time to apprehend the mobile target.

Issue: Whether or not the arrest of the accused was valid.


Ruling: Both contentions are inaccurate. In the case at bar, the taxicab occupied by appellant was validly
stopped at the police checkpoint by PO3 Valenzuela. It should be stressed as a caveat that the search
which is normally permissible in this instance is limited to routine checks -- visual inspection or flashing a
light inside the car, without the occupants being subjected to physical or body searches. A search of the
luggage inside the vehicle would require the existence of probable cause.
In the case at hand, however, probable cause is not evident. we hold that appellant and his baggage
were validly searched, not because he was caught in flagrante delicto, but because he freely
consented to the search. True, appellant and his companion were stopped by PO3 Valenzuela on mere
suspicion -- not probable cause -- that they were engaged in a felonious enterprise. But Valenzuela
expressly sought appellant’s permission for the search. Only after appellant agreed to have his person
and baggage checked did the actual search commence. It was his consent which validated the search,
waiver being a generally recognized exception to the rule against warrantless search

iv. Consent/Waiver

Requisites:
1.It must appear that the right exists.
2. The person involved had knowledge, either actual or constructive, of the existence of the right.
3. The person had actual intention to relinquish the right.

LEONA PASION VIUDA DE GARCIA vs. DIEGO LOCSIN [G.R. No. L-45950, June 20, 1938]
Facts: Mariano G. Almeda, an agent of the Anti-Usuary Board, obtained from the justice of the peace of
Tarlac, a search warrant to search the person, house or store of the petitioner at Victoria, Tarlac, for
“certain books, lists, chits, receipts, documents and other papers relating to her activities as usurer.”

Almeda after showing the search warrant to the petitioner’s bookkeeper, Alfredo Salas, and, without the
presence of the petitioner who was ill and confined at the time, proceeded with the execution thereof

The papers and documents seized were kept for a considerable length of time by the Anti-Usury Board
and thereafter were turned over by it to the respondent fiscal who subsequently filed six separate
criminal cases against the herein petitioner for violation of the Anti-Usury Law.

The legality of the search warrant was challenged by counsel for the petitioner in the six criminal cases
and the devolution of the documents demanded. The respondent Judge denied the petitioner’s motion
for the reason that though the search warrant was illegal, there was a waiver on the part of the
petitioner. (in Español Ü)

Issue: Whether there was a valid waiver on the part of the petitioner.

Ruling: No. While the Solicitor-General admits that, in the light of decisions of this court, the search
warrant was illegally issued, he maintains "(1) that the petitioner had waived her constitutional right by
her acquiescence after the search and seizure, and (2) that the application for the return of the
documents illegally seized was made after an unreasonable length of time after the date of seizure."

Doubtless, the constitutional immunity against unreasonable searches and seizures is a personal right
which may be waived. The waiver may be either express or implied. No express waiver has been made
in the case before us. It is urged, however, that there has been a waiver by implication. It is well-
settled that to constitute a waiver of a constitutional right, it must appear, first, that the right exists;
secondly, that the persons involved had knowledge, either actual or constructive, of the existence of
such right; and, lastly, that said person had an actual intention to relinquish the right.

the petitioner came to know later of the seizure of some of her papers and documents. But this was
precisely the reason why she sent her attorneys to the office of the Anti-Usuary Board to demand the
return of the documents seized. In any event, the failure on the part of the petitioner and her
bookkeeper to resist or object to the execution of the warrant does not constitute an implied waiver
of constitutional right.

As a general proposition, it may be admitted that waiver may be the result of a failure to object within a
reasonable time to a search and seizure illegally made. It must be observed, however, that the
petitioner, on several occasions, and prior to the filing of criminal actions against her, had demanded
verbally, through counsel, the return by the Anti-Usuary Board of the properties seized. The delay in
making demand for the return of the documents seized is not such as to result in waiver by
implication.
RUDY CABALLES y TAIÑO vs.COURT OF APPEALS [G.R. No. 136292, January 15, 2002]
Facts: While on a routine patrol in Brgy Sampalucan, Pagsanjan, Laguna, Sgt. Victorino Nocejo and Pat.
Alex de Castro spotted a passenger jeep unusually covered with kakawati leaves. Suspecting that the
jeep was loaded with smuggled goods, the two officers flagged down the vehicle. Being the driver of the
jeep, Caballes was asked by the officers as to what was loaded in the jeep, to which he did not respond,
appearing pale and nervous. The officers checked the cargo and discovered bundles of galvanized
conductor wires exclusively owned by National Power Corporation. Caballes and the vehicle with the
high-voltage wires were brought to the Pagsanjan Police Station, where he was imprisoned for 7 days.
The trial court found Caballes guilty of the crime of Theft of property. Upon appeal, the Court affirmed
the trial court’s judgment of conviction.
Petitioner contends that the statement of Sgt. Victorino Noceja that he checked the vehicle "with the
consent of the accused" is too vague to prove that petitioner consented to the search. He claims that
there is no specific statement as to how the consent was asked and how it was given, nor the specific
words spoken by petitioner indicating his alleged "consent." At most, there was only an implied
acquiescence, a mere passive conformity, which is no "consent" at all within the purview of the
constitutional guarantee.
Issue: Whether there was consent and waiver on his right to warrantless searches.
Ruling: No. In the case at bar, the evidence is lacking that the petitioner intentionally surrendered his
right against unreasonable searches. The manner by which the two police officers allegedly obtained the
consent of petitioner for them to conduct the search leaves much to be desired. When petitioner's
vehicle was flagged down, Sgt. Noceja approached petitioner and "told him I will look at the contents of
his vehicle and he answered in the positive." We are hard put to believe that by uttering those words,
the police officers were asking or requesting for permission that they be allowed to search the vehicle of
petitioner. For all intents and purposes, they were informing, nay, imposing upon herein petitioner that
they will search his vehicle. The "consent" given under intimidating or coercive circumstances is no
consent within the purview of the constitutional guaranty. In addition, in cases where this Court
upheld the validity of consented search, it will be noted that the police authorities expressly asked, in
no uncertain terms, for the consent of the accused to be searched. And the consent of the accused was
established by clear and positive proof. In the case of herein petitioner, the statements of the police
officers were not asking for his consent; they were declaring to him that they will look inside his vehicle.

People v. Agbot, 106 SCRA 325 - JC

Facts: Agbot, due to the fact that his sister did not want to return his daughter to him, threatened the
latter. After some time, he shot his sister. An investigation was conducted and without valid warrants,
seized the paltik shotgun upon questioning the accused whether it was the same gun used.
Circumstantial evidence would support the fact that it had been recently used, a spent shell found in the
accused’s domicile, and of the nature of the wound. The trial court convicted him of murder.

Issue: Whether respondent’s constitutional right was violated when the search and seizure was made

Held: No. The verity of appellant’s admission of guilt having been firmly established, the contention that
the confiscation or seizure of the gun was illegal, there being no search warrant and its use as evidence
is not permissible, clearly becomes devoid of factual or legal basis. With his confessions, his voluntarily
surrendering the weapon with which he committed the offense would be but a natural consequence
of his having admitted guilt. The taking of the gun from his house was, therefore, with consent and
acquiescence that would not constitute a violation of the constitutional guaranty against the
admissibility of illegally seized objects as evidence against an accused.

Lopez v. Commissioner of Customs, L-27968, Dec. 3, 1975 68 SCRA 320 - JC

Facts: Lopez and Velasco were accused of smuggling. Their vessel was searched to which the smuggled
items and other documents were found evidencing the fact of smuggling. With reference to the
unreasonable search and seizure, the same was made in their hotel room to which, as supported by the
fact of consent of the wife of Velasco, she assisted the raiding team when informed of their purpose,
and helped and made an inventory of the things seized with no force or threat thereat.

Issue: Whether there was consent to abate the claim of unreasonable search and seizure
Held: Yes. "There can be no question that without the proper search warrant, no public official has the
right to enter the premises of another without his consent for the purpose of search and seizure." 15 It
does not admit of doubt therefore that a search or seizure cannot be stigmatized as unreasonable and
thus offensive to the Constitution if consent be shown. Such a view is implicit in People v.
Malasugui. 16 For this immunity from unwarranted intrusion is a personal right which may be waived
either expressly or impliedly.

In essence, there must be consent.

People v. Damaso 212 SCRA 457 - JC

Facts: Damaso was arrested and convicted of the crime of subversion based on the allegation that the
raiding team was allowed to enter the house of the accused and therein, in plain view, found an M14
rifle with live ammunition.

Issue: Whether the search and seizure was valid

Held: No. The right against unreasonable searches and seizures is enshrined in the Constitution. The
purpose of the law is to prevent violations of private security in person and property, and unlawful
invasions of the sanctity of the home by officers of the law acting under legislative or judicial sanction
and to give remedy against such usurpations when. However, such right is not absolute. There are
instances when a warrantless search and seizure becomes valid, namely: (1) search incidental to an
arrest; (2) search of a moving vehicle; and (3) seizure of evidence in plain view. None of these exceptions
is present in this case.

No consent was given by the accused as he was not present. No authority was given the helper to allow
the public officers to enter the premises. Nor was the search incidental to an arrest.

People v. Asis, GR 142531, October 15, 2002 - JC

Facts: Asis and Formento were charged and convicted of robbery with homicide aggravated by abuse of
confidence, superior strength and treachery. No search warrant was issued and no consent was given by
the accused when their bags were searched for the alleged incriminating evidence.

Issue: Whether consent was validly given

Held: No. As the accused were deaf – mutes, at the time of the search and seizure, they could not have
given their consent as no interpreter was present. As to the fact of acquiescence, the accused could not
have validly objected to the same because they were ignorant of what was transpiring.

To constitute a valid waiver, it must be shown that first, the right exists; second, the person involved had
knowledge, actual or constructive, of the existence of such a right; and third, the person had an actual
intention to relinquish the right.[33] How could Appellant Formento have consented to a warrantless
search when, in the first place, he did not understand what was happening at that moment? The
prosecution witnesses themselves testified that there was no interpreter to assist him -- a deaf-mute --
during the arrest, search and seizure. Naturally, it would seem that he indeed consented to the
warrantless search, as the prosecution would want this Court to believe.

Spouses Veroy v. Layague, GR 95632, June 18, 1992 - jC

Facts: Petitioners lived in Davao City. They then transferred to Quezon City. Due to a report that their
house was being used by rebel soldiers, the commanding officer of the raiding team, sought petitioner’s
permission to enter the same. This was given on the condition that petitioner’s friend, a Major, would
accompany them. Upon entering the house, they found weapons with ammunition and other subversive
materials. They were consequently charged therewith.

Issue: Whether the items seized were admissible

Held: No. Although consent was given for the purpose of locating rebel soldiers, none was given for the
items seized therein. The raiding team had ample opportunity to secure a search warrant as evidenced
by the commanding officer’s apprehension in entering the premises without any consent therefrom;
hence, the objects seized in view of the fact that no warrant was issued are inadmissible.
People v. Omaweng, 213 SCRA 462 GR. No. 99050 - JC

Facts: Omaweng was en route to Sagada, Mountain Province, when he was arrested for transporting
and possessing marijuana. Such search was made with his consent.

Issue: Whether accused had waived his right against unreasonable search and seizure

Held: Yes. Accused was not subjected to any search which may be stigmatized as a violation of his
Constitutional right against unreasonable searches and seizures. He willingly gave prior consent to the
search and voluntarily agreed to have it conducted on his vehicle and travelling bag. Thus, the accused
waived his right against unreasonable searches and seizures. When one voluntarily submits to a search
or consents to have it made of his person or premises, he is precluded from later complaining thereof
(Cooley, Constitutional Limitations, 8th ed., vol. I, page 631.) The right to be secure from unreasonable
search may, like every right, be waived and such waiver may be made either expressly or impliedly."
Since in the course of the valid search forty-one (41) packages of drugs were found, it behooved the
officers to seize the same; no warrant was necessary for such seizure. Besides, when said packages were
identified by the prosecution witnesses and later on formally offered in evidence, the accused did not
raise any objection whatsoever.
People v. Correa, 285 SCRA 679 - JC

Facts: The accused were apprehended on board their jeep where drugs were found therein to which
search they consented. Upon arraignment, they pleaded not guilty. The RTC convicted them upon
positive identification by the arresting officer.

Issue: Whether the things seized were fruits of the poisonous tree

Held: No. When one voluntarily submits to a search or consents to have it made on his person or
premises, he is precluded from later complaining thereof (Cooley, Constitutional Limitations, 8th ed.,
vol. I, page 631). The right to be secure from unreasonable search may, like every right, be waived and
such waiver may be made either expressly or impliedly. Appellants are now precluded from assailing the
warrantless search and seizure when they voluntarily submitted to it as shown by their actuation during
the search and seizure.
People v. Ramos, 222 SCRA 557 G.R. Nos. 101804-07 May 25, 1993 - JC

Facts: Following 4 incidents of robbery, police officers proceeded to the area of residence of the accused
following the identification by the victims. Upon locating the accused, he consented to a frisk which
resulted to one .38 snub nose gun and a paltik. During trial, counsel for defendants, objected to the
presentation of the weapons as it was procured without a warrant.

Issue: Whether the consent cured the defect

Held: Yes. The rule is that a search may be conducted by law enforcers only on the strength of a search
warrant validly issued by a judge. This is enshrined in the Bill of Rights. 48 Such a rule, however, is not
without exceptions. 49 For instance, a warrantless search made be validly made as an incident to a
lawful arrest 50 or in stop and search situations. 51Another recognized exception is when the accused
himself waives his right against unreasonable search and seizure. As this Court stated in People vs.
Malasugui: 52

When one voluntarily submits to a search or consents to have it made of his person or premises, he is
precluded from later complaining thereof (Cooley, Constitutional Limitations, 8th Ed., vol. I, page 631.)
The right to be secure from unreasonable search may, like every right, be waived and such waiver may
be made either expressly or impliedly.
People v. Tudtud, GR 144037, Sept 26, 2003 - JC

Facts: The accused were apprehended due to an information that they were dealing in drugs. The
accused cry frame – up. When they alighted the bus, they were approached by a police officer while
pointing a gun at them and ordered them to open their bag which “contained” marijuana. They were
convicted by the RTC.

Issue: Whether the search was incidental to a lawful arrest

Held: No. The proscription in Section 2, Article III, however, covers only “unreasonable” searches and
seizures. The following instances are not deemed “unreasonable” even in the absence of a warrant:
Warrantless search incidental to a lawful arrest, Search of evidence in “plain view.”, Search of a moving
vehicle, Consented warrantless search, Customs search, Stop and Frisk; and Exigent and emergency
circumstances.

Appellants’ implied acquiescence, if at all, could not have been more than mere passive conformity
given under coercive or intimidating circumstances and is, thus, considered no consent at all within the
purview of the constitutional guarantee. Consequently, appellants’ lack of objection to the search and
seizure is not tantamount to a waiver of his constitutional right or a voluntary submission to the
warrantless search and seizure.

As the search of appellants’ box does not come under the recognized exceptions to a valid warrantless
search, the marijuana leaves obtained thereby are inadmissible in evidence. And as there is no evidence
other than the hearsay testimony of the arresting officers and their informant, the conviction of
appellants cannot be sustained.

People v. Tabar 222 SCRA 144 - JC

Facts: A buy bust operation was conducted when police officers received information that drugs were
being sold. The nephew of the accused sold the marijuana cigarettes to the poseur – buyer. Signaling the
police, they proceeded to apprehend the nephew. Upon seeing arrest, Tabar placed the packs of
marijuana in a rolled up pants. When asked what was inside the pants, she unrolled it and the marijuana
were found therein. The RTC convicted her for violation of the Dangerous Drugs Act of 1972.

Issue: Whether the court erred in admitting the evidence seized without a search warrant

Held: No. The appellant had effectively waived her constitutional right relative thereto by voluntarily
submitting to the seach and seizure. In People vs. Malasugui, 20 this Court ruled:

When one voluntarily submits to a search and consent to have it made of his person or premises, he is
precluded from later complaining thereof (Cooley, Constitutional Limitations, 8th ed., vol. I, pages 631).
The right to be secure from unreasonable seach may, like every right, be waived and such waiver may be
made either expressly or impliedly.

People v. Encinada – 280 SCRA 72 - Xavier


FACTS: SPO4 Nicolas Bolonia received a tip from an informant that Roel Encinada would be arriving in
Surigao City from Cebu City on board the M/V Sweet Pearl bringing with him ‘marijuana.’ After receiving
the tip, no search warrant was secured. Bolinia apprehended Encinada, carrying two small colored
plastic baby chairs in his hand, and was brought to the central police station. Bolonia, in the presence of
one local media member Nonoy Lerio, opened the package and found dried leaves known as marijuana.
Upon his arraignment, appellant pleaded “not guilty” to the charge. Later, the defense filed, with leave
of court, a “Demurrer to Evidence” questioning the admissibility of the evidence which allegedly was
illegally seized from appellant.
ISSUE: Whether the arrest and search of the accused without a warrant would fall under the doctrine of
warrantless search as an incident to a lawful arrest.
Held: NO. Raw intelligence information is not a sufficient ground for a warrantless arrest. Appellant
Encinada was not committing a crime in the presence of the Surigao City policemen. Moreover, the
lawmen did not have personal knowledge of facts indicating that the person to be arrested had
committed an offense. The search cannot be said to be merely incidental to a lawful arrest.
People v. Aruta – 288 SCRA 626- Xavier
FACTS: P/Lt. Abello was tipped off by his informant that a certain “Aling Rosa” will be arriving from
Baguio City with a large volume of marijuana. The next day, at the Victory Liner Bus terminal, after the
informer identified “Aling Rosa”, the team approached her and introduced themselves as NARCOM
agents. When Abello asked “Aling Rosa” about the contents of her bag, she handed it out to them and
they found dried marijuana leaves packed in a plastic bag marked “cash katutak.”
At the trial, instead of presenting its evidence, the defense filed a demurrer to evidence alleging
the illegality of the search and seizure of the items. In her testimony, the accused claimed that she had
just come from Choice theatre where she watched a movie “Balweg”. While about to cross the road an
old woman asked her for help in carrying a shoulder bag, when she was later on arrested by the police.
She has no knowledge of the identity of the old woman and the woman was nowhere to be found.
Also, no search warrant was presented. The trial court convicted the accused in violation of the
dangerous drugs of 1972
ISSUE: Whether the police correctly searched and seized the drugs from the accused.
HELD: NO. There was no legal basis to effect a warrantless arrest of the accused’s bag, there was no
probable cause and the accused was not lawfully arrested. The accused cannot be said to be
committing a crime, she was merely crossing the street and was not acting suspiciously for the NARCOM
agents to conclude that she was committing a crime. The essential requisite of probable cause must still
be satisfied before a warrantless search and seizure can be lawfully conducted. The police had more
than 24 hours to procure a search warrant and they did not do so. The seized marijuana was illegal and
inadmissible evidence.

v. Customs Search

Papa v. Mago, 22 SCRA 857 GR No. 27360- Xavier


FACTS: Petitioner Martin Alagao, head of the counter-intelligence unit of the Manila Police Department,
acting upon a reliable information that a certain shipment of personal effects, allegedly misdeclared and
undervalued, would be released the following day from the customs zone of the port of Manila and
loaded on two trucks. Upon orders of petitioner Ricardo Papa, Chief of Police of Manila and a duly
deputized agent of the Bureau of Customs, they conducted surveillance at gate No. 1 of the customs
zone.
The load of the two trucks, consisting of nine bales of goods, were intercepted at the Agrifina
Circle, Ermita, Manila and were seized on instructions of the Chief of Police. Upon investigation, a
person claimed ownership of the goods and showed to the policemen a "Statement and Receipts
of Duties Collected on Informal Entry No. 147-5501", issued by the Bureau of Customs in the name of a
certain Bienvenido Naguit. Respondent Mago, filed a petition for mandamus and certiorari before the
CFI Manila contending that the search and seizure is illegal for lack of a valid warrant.
ISSUE: Whether the search and seizure is illegal for lack of a valid warrant.
RULING: NO. Petitioner Ricardo G. Papa, Chief of Police of Manila, could lawfully effect the search and
seizure of the goods in question. The Tariff and Customs Code authorizes him to demand assistance of
any police officer to effect said search and seizure, and the latter has the legal duty to render said
assistance. The Code does not require said warrant in the instant case. Except in the case of the search
of a dwelling house, persons exercising police authority under the customs law may effect a search and
seizure without a search warrant in the enforcement of customs laws
Pacis v. Pamaran, 56 SCRA 16- Xavier
FACTS: Respondent Ricardo Santos is the owner of a Mercury automobile, model 1957, brought into the
country without paymentof customs duty and taxes because its original owner, Donald James Hatch,
was tax-exempt. Santos later on paid P311.00 for customs duty and taxes.
On July 22, 1964, Acting Collector of Customs Pedro Pacis was informed by the General Affairs
Administration of theDepartment of National Defense that the automobile was a “hot car.” By virtue
thereof, Pacis, through his subordinates, looked into the records of his office and found that
although the amount of P311.00 was already paid for customs duty,the amount collectible on the said
car should be P2,500.00, more or less. Based on such discrepancy, he instituted seizure proceedings and
issued a warrant of seizure and detention. The automobile was also taken by the Department
of National Defense agents and brought to the General Affairs Administration for compound.
In answer, Santos filed a criminal complaint against Pacis for usurpation of judicial functions
with the City Fiscal of Manila, Manuel Pamaran, alleging that Pacis did not have authority to issue such
warrant of seizure and detention.
ISSUE: Whether the petitioner, in the discharge of his official function, lay himself open to a criminal
prosecution for usurpation of judicial functions
HELD: It is undeniable that petitioner, as Acting Collector of Customs for the Port of Manila, had the
requisite authority for the issuance of the contested warrant of seizure and detention for the
automobile owned by respondent Ricardo Santos. What was done by him certainly could not be the
basis of a prosecution for the usurpation of judicial functions. The remedy of prohibition lies.
People v. Gatward, 267 SCRA 785- Xavier
Facts: U Aung Win, a passenger of Thai Airways which had just arrived from Bangkok. His luggage was
presented to Customs Examiner Busran Tawano. Because of U Aung’s action, Tawano became alarmed
by the failure of U Aung Win to return and suspected that the bag of U Aung Win contained illegal
articles.
Tawano inspected the luggage thoroughly and found 2 packs of heroin. U Aung win was
eventually apprehended. Follow up investigation leads to two more suspected drug couriers. The
members of the team were able to establish the identity of the two persons as Nigel Richard Gatward
and one Zaw Win Naing. Gatward was invited by the police officers for investigation. At about 3:00 PM
of 1 September 1994, Gatward’s luggage, was brought back to the NAIA from Bangkok through the Thai
airways, pursuant to the request of Erece. The luggage was found to have illegal drugs in it.
Issue: Whether Gatward’s suitcases may be searched without warrant.
Held: YES. While no search warrant had been obtained for that purpose, when Gatward checked in his
bag as his personal luggage as a passenger of KLM Flight 806 he thereby agreed to the inspection
thereof in accordance with customs rules and regulations, an international practice of strict
observance, and waived any objection to a warrantless search.
People v. Susan Canton, GR 148825, December 27, 2002- Xavier
FACTS: Susan was at the Ninoy Aquino International Airport (NAIA), being a departing passenger bound
for Saigon, Vietnam. When she passed through the metal detector booth, a beeping sound was
emitted. Consequently, Mylene Cabunoc, a civilian employee of the National Action Committee on
Hijacking and Terrorism (NACHT), frisked Susan, and felt something bulging at her abdominal
area. When Mylene inserted her hand under the skirt of Susan, she noticed that the package contained
what felt like rice granules. She also felt similar packages in front of Susan’s genital area and
thighs. When asked Susan to bring out the packages, the latter refused and said: “Money, money only.”
Mylene forthwith reported the matter to SPO4 Victorio de los Reyes, her supervisor on duty.
Upon further frisking in the ladies’ room, Mylene and Lorna discovered three packages individually
wrapped and sealed in gray colored packing tape, which were turned over to the police and later yielded
positive results for methamphetamine hydrochloride or shabu, a regulated drug. In the trial, among her
contentions was that the frisker went beyond the limits of the “Terry search” doctrine.
ISSUE: Whether the frisker went beyond the limits of the “Terry search” doctrine.
HELD: NO. In the present case, the search was made pursuant to routine airport security procedure,
which is allowed under Section 9 of Republic Act No. 6235 authorizes search for prohibited materials
or substances. This constitutes another exception to the proscription against warrantless searches and
seizures. As admitted by SUSAN and shown in Annex “D” of her Brief, the afore-quoted provision is
stated in the “Notice to All Passengers” located at the final security checkpoint at the departure
lounge. From the said provision, it is clear that the search, unlike in the Terry search, is not limited to
weapons. Passengers are also subject to search for prohibited materials or substances.
People v. Johnson – 348 SCRA 526- Xavier
FACTS: Accused appellant Leila Reyes Johnson, a naturalized American with residence in the United
States, came to visit her son’s family in the Philippines. Upon physical inspection in the airport for her
flight back to the United States, the personnel conducting the examination felt a hard object on
Johnson’s abdomen. Unsatisfied with the reasoning that she uses two sets of undergarments due to a
previous operation, the airport personnel reported to her supervisor and Johnson was led to the
women’s comfort room for closer inspection. A set of packages were found in her abdominal area which
were surrendered by the inspectors to the police officers and her passport, money and other personal
belongings were seized. She was charged for violation of the Dangerous Drugs Act. Johnson alleges that
the warrantless search and seizure conducted on her person is invalid for it violates her constitutional
rights.
ISSUE: Whether the warrantless search and seizure was invalid.
Ruling: NO. A person loses his right against a warrantless search and seizure by exposure his person and
property to the public in a manner reflecting lack of substantive expectation of privacy. The search
conducted on Johnson is well within the light of the circumstances prevalent at the time where hi-
jacking and bombings in airplanes are of prevalent concern which calls for a higher degree of security
checks. When something suspicious arises from the routine checks, it is just reasonable for the safety of
the public that a more thorough physical search be conducted. The search in the case at bar is
reasonable in the light of their minimal intrusiveness, the concern on public safety and of the other
passengers and the reduced privacy expectations in airline travel. HOWEVER, the seized personal
property like the wallet and passports should be returned because only the following objects can be
seized even with a valid search warrant:
(a) Subject of the offense;
(b) Stolen or embezzled and other proceeds or fruits of the offense; and
(c) Used or intended to be used as the means of committing an offense.

vi. Stop and Frisk Situation

Malacat: “Where a police officer observes unusual conduct which leads him reasonably to conclude in
light of his experience that criminal activity may be afoot and that the person with whom he is dealing
may be armed and that the person with whom he is dealing may be armed and presently dangerous,
where in the course of investigation of this behavior he identifies himself as a policeman and makes
reasonable inquiries, and where nothing in the initial stages of the encounter serves to dispel his
reasonable fear for his own or other’s safety, he is entitled for the protection of himself and others in
the area to conduct a carefully limited search of the outer clothing of such person in an attempt to
discover weapons which might be used to assault him.”

Malacat (1997): Probable cause is not required. However, mere suspicion or a hunch is not enough.
Rather, a “genuine reason must exist, in light of the police officer’s experience and surrounding
conditions, to warrant the belief that the person detained has weapons concealed about him.”

Terry v. Ohio 392 US 1- Xavier


FACTS: Two other men were observed to repeatedly walk past a store window and returned to a spot
where they apparently conferred with Terry by a plain clothes policeman in what the officer believed to
be "casing a job, a stick-up." The officer stopped and frisked the three men, and found weapons on two
of them. Terry was convicted of carrying a concealed weapon and sentenced to three years in jail.
ISSUE: Whether the search and seizure of Terry and the other men in violation of the Fourth
Amendment.
HELD: NO. In an 8-to-1 decision, the Court held that the search undertaken by the officer was
reasonable under the Fourth Amendment and that the weapons seized could be introduced into
evidence against Terry. Attempting to focus narrowly on the facts of this particular case, the Court found
that the officer acted on more than a "hunch" and that "a reasonably prudent man would have been
warranted in believing [Terry] was armed and thus presented a threat to the officer's safety while he
was investigating his suspicious behavior." The Court found that the searches undertaken were limited
in scope and designed to protect the officer's safety incident to the investigation.
Posadas v. CA, GR NO. 89139, August 2, 1990- Xavier
FACTS: Members of the Integrated National Police (INP) of the Davao Metrodiscom assigned with the
Intelligence Task Force Pat. Ungab and Umpar were conducting surveillance along Magallanes Street,
Davao City. They spotted petitioner Posadas carrying a "buri" bag and noticed him to be acting
suspiciously. They approached the petitioner and identified themselves asmembers of the INP.
Petitioner attempted to flee but his attempt to get away was thwarted by the two
notwithstanding his resistance. They then checked the "buri" bag of the petitioner where they found
one (1) caliber .38 revolver, two (2) rounds of live ammunition for a .38 caliber gun a smoke (tear gas)
grenade, and two (2) live ammunitions for a .22 caliber gun. The petitioner was asked to show the
necessary license or authority to possess the firearms and ammunitions but failed to do so.
ISSUE: Whether the warantless search is valid.
HELD: YES. The assailed search and seizure may still be justified as akin to a "stop and frisk" situation
whose object is either to determine the identity of a suspicious individual or to maintain the status
quo momentarily while the police officer seeks to obtain more information. This is illustrated in the
case of Terry vs. Ohio, 392 U.S. 1 (1968), where a police officer may in appropriate circumstances and in
an appropriate manner approach a person for the purpose of investigating possible criminal behaviour
even though there is no probable cause to make an arrest.
It is too much indeed to require the police officers to search the bag in the possession of the
petitioner only after they shall have obtained a search warrant for the purpose. Such an exercise may
prove to be useless, futile and much too late.
People v. Solayao 202 SCRA 255 (1996)- Xavier
FACTS: SPO3 Jose Niño with CAFGU members were conducting an intelligence patrol to verify reports on
the presence of armed persons roaming around the barangays of Caibiran. The team of Police Officer
Niño proceeded to Barangay Onion where they met the group of accused-appellant Nilo Solayao, who
were acting suspicious and they observed that the latter were drunk and that accused-appellant Solayao
himself was wearing a camouflage uniform or a jungle suit.
Upon seeing the government agents, Solayao’s companions fled. SPO3 Nino told accused-
appellant not to run away, the former identified himself as a government agent. After which he seized
the dried coconut leaves which the latter was carrying and found wrapped in it a 49-inch long
homemade firearm locally known as "latong." When he asked accused-appellant who issued him a
license to carry said firearm or whether he was connected with the military or any intelligence group,
the latter answered that he had no permission to possess the same. Thereupon, SPO3 Niño confiscated
the firearm.
ISSUE: Whether there was a violation of the constitutional guarantee against unreasonable searches and
seizures.
HELD: NO. There was justifiable cause to "stop and frisk" accused-appellant when his companions fled
upon seeing the government agents. Under the circumstances, the government agents could not
possibly have procured a search warrant first. Thus, there was no violation of the constitutional
guarantee against unreasonable searches and seizures. Nor was there error on the part of the trial court
when it admitted the homemade firearm as evidence.

Malacat v. CA 283 SCRA 159 (1997) – Kaye


Malacat vs. Court of Appeals
[GR 123595, 12 December 1997]

Facts:

On 27 August 1990, at about 6:30 p.m., allegedly in response to bomb threats reported seven days
earlier, Rodolfo Yu of the Western Police District, Metropolitan Police Force of the Integrated National
Police, Police Station No. 3, Quiapo, Manila, was on foot patrol with three other police officers (all of
them in uniform) along Quezon Boulevard, Quiapo, Manila, near the Mercury Drug store at Plaza
Miranda. They chanced upon two groups of Muslim-looking men, with each group, comprised of three
to four men, posted at opposite sides of the corner of Quezon Boulevard near the Mercury Drug Store.
These men were acting suspiciously with "their eyes moving very fast." Yu and his companions
positioned themselves at strategic points and observed both groups for about 30 minutes. The police
officers then approached one group of men, who then fled in different directions. As the policemen gave
chase, Yu caught up with and apprehended Sammy Malacat y Mandar (who Yu recognized, inasmuch as
allegedly the previous Saturday, 25 August 1990, likewise at Plaza Miranda, Yu saw Malacat and 2 others
attempt to detonate a grenade). Upon searching Malacat, Yu found a fragmentation grenade tucked
inside the latter's "front waist line." Yu's companion, police officer Rogelio Malibiran, apprehended
Abdul Casan from whom a .38 caliber revolver was recovered. Malacat and Casan were then brought to
Police Station 3 where Yu placed an "X" mark at the bottom of the grenade and thereafter gave it to his
commander. Yu did not issue any receipt for the grenade he allegedly recovered from Malacat. On 30
August 1990, Malacat was charged with violating Section 3 of Presidential Decree 1866. At arraignment
on 9 October 1990, petitioner, assisted by counsel de officio, entered a plea of not guilty. Malacat
denied the charges and explained that he only recently arrived in Manila. However, severalother police
officers mauled him, hitting him with benches and guns. Petitioner was once again searched, but
nothing was found on him. He saw the grenade only in court when it was presented. In its decision
dated 10 February 1994 but promulgated on 15 February 1994, the trial court ruled that the warrantless
search and seizure of Malacat was akin to a "stop and frisk," where a "warrant and seizure can be
effected without necessarily being preceded by an arrest" and "whose object is either to maintain the
status quo momentarily while the police officer seeks to obtain more information"; and that the seizure
of the grenade from Malacat was incidental to a lawful arrest. The trial court thus found Malacat guilty
of the crime of illegal possession of explosives under Section 3 of PD 1866, and sentenced him to suffer
the penalty of not less than 17 years, 4 months and 1 day of Reclusion Temporal, as minimum, and not
more than 30 years of Reclusion Perpetua, as maximum. On 18 February 1994, Malacat filed a notice of
appeal indicating that he was appealing to the Supreme Court. However, the record of the case was
forwarded to the Court of Appeals (CA-GR CR 15988).

In its decision of 24 January 1996, the Court of Appeals affirmed the trial court. Manalili filed a petition
for review with the Supreme Court.

Issue:

Whether the search made on Malacat is valid, pursuant to the exception of “stop and frisk.”

Held:

The general rule as regards arrests, searches and seizures is that a warrant is needed in order to validly
ef fect the same. The Constitutional prohibition against unreasonable arrests, searches and seizures
refers to those effected without a validly issued warrant, subject to certain exceptions. As regards valid
warrantless arrests, these are found in Section 5, Rule 113 of the Rules of Court. A warrantless arrest
under the circumstances contemplated under Section 5(a) has been denominated as one "in flagrante
delicto," while that under Section 5(b) has been described as a "hot pursuit" arrest.

Turning to valid warrantless searches, they are limited to the following: (1) customs searches; (2) search
of moving vehicles; (3) seizure of evidence in plain view; (4) consent searches; (5) a search incidental to
a lawful arrest; and (6) a "stop and frisk." The concepts of a "stop-and-frisk" and of a search incidental to
a lawful arrest must not be confused. These two types of warrantless searches differ in terms of the
requisite quantum of proof before they may be validly effected and in their allowable scope. In a search
incidental to a lawful arrest, as the precedent arrest determines the validity of the incidental search.
Here, there could have been no valid in flagrante delicto or hot pursuit arrest preceding the search in
light of the lack of personal knowledge on the part of Yu, the arresting officer, or an overt physical act,
on the part of Malacat, indicating that a crime had just been committed, was being committed or was
going to be committed. Plainly, the search conducted on Malacat could not have been one incidental to
a lawful arrest. On the other hand, while probable cause is not required to conduct a "stop and frisk," it
nevertheless holds that mere suspicion or a hunch will not validate a "stop and frisk." A genuine reason
must exist, in light of the police officer's experience and surrounding conditions, to warrant the belief
that the person detained has weapons concealed about him. Finally, a "stop-and-frisk" serves a two-fold
interest: (1) the general interest of effective crime prevention and detection, which underlies the
recognition that a police officer may, under appropriate circumstances and in an appropriate manner,
approach a person for purposes of investigating possible criminal behavior even without probable
cause; and (2) the more pressing interest of safety and self-preservation which permit the police officer
to take steps to assure himself that the person with whom he deals is not armed with a deadly weapon
that could unexpectedly and fatally be used against the police officer. Here, there are at least three (3)
reasons why the "stop-and-frisk" was invalid:

First, there is grave doubts as to Yu's claim that Malacat was a member of the group which attempted to
bomb Plaza Miranda 2 days earlier. This claim is neither supported by any police report or record nor
corroborated by any other police officer who allegedly chased that group.

Second, there was nothing in Malacat's behavior or conduct which could have reasonably elicited even
mere suspicion other than that his eyes were "moving very fast" — an observation which leaves us
incredulous since Yu and his teammates were nowhere near Malacat and it was already 6:30 p.m., thus
presumably dusk. Malacat and his companions were merely standing at the corner and were not
creating any commotion or trouble.

Third, there was at all no ground, probable or otherwise, to believe that Malacat was armed with a
deadly weapon. None was visible to Yu, for as he admitted, the alleged grenade was "discovered"
"inside the front waistline" of Malacat, and from all indications as to the distance between Yu and
Malacat, any telltale bulge, assuming that Malacat was indeed hiding a grenade, could not have been
visible to Yu. What is unequivocal then are blatant violations of Malacat's rights solemnly guaranteed in
Sections 2 and 12(1) of Article III of the Constitution.

A “stop-and-frisk” serves a two-fold interest: (1) the general interest of effective crime prevention and
detection, which underlies the recognition that a police officer may, under appropriate circumstances
and in an appropriate manner, approach a person for purposes of investigating possible criminal
behavior even without probable cause; and (2) the more pressing interest of safety and self-
preservation which permit the police officer to take steps to assure himself that the person with
whom he deals is not armed with a deadly weapon that could unexpectedly and fatally be used
against the police officer (Malacat v. Court of Appeals, G.R. No. 123595, 12 December 1997, J. Davide,
Jr.).

Manalili v. CA, GR 113447, October 7, 1997- Kaye


Manalili vs Court of Appeals
(October 9, 1997)

Ponente: Panaganiban
Nature: Petition for review on certiorari of a decision of the Court of Appeals
Facts:
Pat. Romeo Espiritu and Pat. Anger Lumabas were patrolling the vicinity of the Kalookan City
Cemetery due to reports of drug addicts roaming the area. They chanced upon a male (who turned out
to be petitioner Alain Manalili y Dizon) who seemed to be “high” on drugs in front of the cemetery. He
was observed to have reddish eyes and to be walking in a swaying manner. When Manalili tried to avoid
the policemen, the latter approached him and asked what he was holding in his hands. Manalili tried to
resist, but the policemen were persistent until he yielded his wallet which they examined and found to
contain crushed marijuana residue. Further examination by the Forensic Chemistry Section of the NBI
confirmed the findings. Trial court convicted Manalili of violation of Section 8, Article II, of RA 6425.
Upon appeal, the Court of Appeals affirmed the decision of the trial court.
(In his defense, Manalili claimed that he was not walking; that he was riding a tricycle until the
three policemen ordered the driver of the tricycle to stop because the driver and passenger were
allegedly under the influence of marijuana. He claimed that he was searched and his pants were turned
inside-out but nothing was found. To some extent he implied that the marijuana sample found in his
entity was framed up by the policemen.)
Issue: WON the evidence seized during a stop-and-frisk operation is admissible.
Held: Yes
Ratio:
The general rule is that a search and seizure must be validated by a previously secured judicial
warrant. However, this is not absolute and exceptions have been contemplated by the law:
1. Search incidental to a lawful rrest
2. Search of moving vehicles
3. Seizure in plain view
4. Customs search
5. Waiver by the accused themselves of their right against unreasonable search and seizure.
In the cited cases, the search and seizure may be made only with probable cause as essential
requirement.
Probable cause (in relation to search and seizure): Existence of such facts and circumstances
which could lead a reasonably discreet and prudent man to believe that an offense has been committed
and that the item, article, or object sought in connection with said offense or subject to seizure and
destruction by law is in the place to be searched.
A “stop-and-frisk” operation is another exception to the general rule. In this case, probable
cause was established with Manalili’s suspicious behaviour.

People v. Aruta, 288 SCRA 626 (1998) – Kaye


PEOPLE V ARUTA

FACTS:
In the morning of 13 Dec 1988, the law enforcement officers received information from an informant
named “Benjie” that a certain “Aling Rosa” would be leaving for Baguio City on 14 Dec 1988 and would
be back in the afternoon of the same day carrying with her a large volume of marijuana; At 6:30 in the
evening of 14 Dec 1988, Aruta alighted from a Victory Liner Bus carrying a travelling bag even as the
informant pointed her out to the law enforcement officers; NARCOM officers approached her and
introduced themselves as NARCOM agents; When asked by Lt. Abello about the contents of her
travelling bag, she gave the same to him; When they opened the same, they found dried marijuana
leaves; Aruta was then brought to the NARCOM office for investigation.

ISSUE: Whether or not the conducted search and seizure is constitutional.

HELD: The SC ruled in favor of Aruta and has noted that some drug traffickers are being freed due to
technicalities. Aruta cannot be said to be committing a crime. Neither was she about to commit one nor
had she just committed a crime. Aruta was merely crossing the street and was not acting in any manner
that would engender a reasonable ground for the NARCOM agents to suspect and conclude that she was
committing a crime. It was only when the informant pointed to Aruta and identified her to the agents as
the carrier of the marijuana that she was singled out as the suspect. The NARCOM agents would not
have apprehended Aruta were it not for the furtive finger of the informant because, as clearly illustrated
by the evidence on record, there was no reason whatsoever for them to suspect that accused-appellant
was committing a crime, except for the pointing finger of the informant. The SC could neither sanction
nor tolerate as it is a clear violation of the constitutional guarantee against unreasonable search and
seizure. Neither was there any semblance of any compliance with the rigid requirements of probable
cause and warrantless arrests. Consequently, there was no legal basis for the NARCOM agents to effect
a warrantless search of Aruta’s bag, there being no probable cause and the accused-appellant not
having been lawfully arrested. Stated otherwise, the arrest being incipiently illegal, it logically follows
that the subsequent search was similarly illegal, it being not incidental to a lawful arrest. The
constitutional guarantee against unreasonable search and seizure must perforce operate in favor of
accused-appellant. As such, the articles seized could not be used as evidence against accused-appellant
for these are “fruits of a poisoned tree” and, therefore, must be rejected, pursuant to Article III, Sec.
3(2) of the Constitution.

People v. Sy Chua, GR 136066, February 4, 2003- Kaye


PEOPLE VS. SY CHUA [396 SCRA 657; G.R. No.136066-67; 4 Feb 2003]

Facts: Accused-appellant Binad Sy Chua was charged with violation of Section 16, Article III of R.A. 6425,
as amended by R.A. 7659, and for Illegal Possession of Ammunitions and Illegal Possession of Drugs in
two separate Informations.

SPO2 Nulud and PO2 Nunag received a report from their confidential informant that accused-appellant
was about to deliver drugs that night at the Thunder Inn Hotel in Balibago, Angeles City. So, the PNP
Chief formed a team of operatives. The group positioned themselves across McArthur Highway
near Bali Hai Restaurant, fronting the hotel. The other group acted as their back up.

Afterwards, their informer pointed to a car driven by accused-appellant which just arrived and parked
near the entrance of the hotel. After accused-appellant alighted from the car carrying a sealed Zest-O
juice box, SPO2 Nulud and PO2 Nunag hurriedly accosted him and introduced themselves as police
officers. As accused-appellant pulled out his wallet, a small transparent plastic bag with a crystalline
substance protruded from his right back pocket. Forthwith, SPO2 Nulud subjected him to a
body search which yielded twenty (20) pieces of live .22 caliber firearm bullets from his left back pocket.
When SPO2 Nunag peeked into the contents of the Zest-O box, he saw that it contained a crystalline
substance. SPO2 Nulud instantly confiscated the small transparent plastic bag, the Zest-O juice box, the
twenty (20) pieces of .22 caliber firearm bullets and the car used by accused-appellant. SPO2 Nulud and
the other police operatives who arrived at the scene brought the confiscated items to the office of Col.
Guttierez at the PNP Headquarters in Camp Pepito, Angeles City.

Accused-appellant vehemently denied the accusation against him and narrated a different version of the
incident.

Accused-appellant alleged that he was driving the car of his wife to follow her and his son to Manila.
He felt sleepy, so he decided to take the old route along McArthur Highway. He stopped in front of a
small store near Thunder Inn Hotel to buy cigarettes and candies. While at the store, he noticed a
man approaches and examines the inside of his car. When he called the attention of the onlooker, the
man immediately pulled out a .45 caliber gun and made him face his car with raised hands. The man
later on identified himself as a policeman. During the course of the arrest, the policeman took out his
wallet and instructed him to open his car. He refused, so the policeman took his car keys and proceeded
to search his car. At this time, the police officer’s companions arrived at the scene in two cars. PO2
Nulud, who just arrived at the scene, pulled him away from his car in a nearby bank, while the others
searched his car.

Thereafter, he was brought to a police station and was held inside a bathroom for about fifteen minutes
until Col. Guttierez arrived, who ordered his men to call the media. In the presence of reporters, Col.
Guttierez opened the box and accused-appellant was made to hold the box while pictures were being
taken.

The lower court acquitted Sy Chua for the Illegal Possession of Ammunitions, yet convicted him for
Illegal Possession of 1,955.815 grams of shabu. Hence, this appeal to the Court.

Issues:
(1) Whether or Not the arrest of accused-appellant was lawful; and
(2) WON the search of his person and the subsequent confiscation of shabu allegedly found on him were
conducted in a lawful and valid manner.

Held: The lower court believed that since the police received information that the accused will distribute
illegal drugs that evening at the Thunder Inn Hotel and its vicinities. The police officer had to act quickly
and there was no more time to secure a search warrant. Thesearch is valid being akin to a “stop and
frisk”.

The trial court confused the concepts of a “stop-and-frisk” and of asearch incidental to a lawful arrest.
These two types of warrantless searches differ in terms of the requisite quantum of proof before they
may be validly effected and in their allowable scope.

In a search incidental to a lawful arrest, as the precedent arrest determines the validity of the
incidental search, the legality of the arrest is questioned, e.g., whether an arrest was merely used as a
pretext for conducting a search. In this instance, the law requires that there first be arrest before
a search can be made—the process cannot be reversed. Accordingly, for this exception to apply, two
elements must concur: (1) the person to be arrested must execute an overt act indicating that he has
just committed, is actually committing, or is attempting to commit a crime; and (2) such overt act is
done in the presence or within the view of the arresting officer.

We find the two aforementioned elements lacking in the case at bar. Accused-appellant did not act in a
suspicious manner. For all intents and purposes, there was no overt manifestation that accused-
appellant has just committed, is actually committing, or is attempting to commit a crime. “Reliable
information” alone, absent any overt act indicative of a felonious enterprise in the presence and within
the view of the arresting officers, is not sufficient to constitute probable cause that would justify an in
flagrante delicto arrest.

With regard to the concept of “stop-and frisk”: mere suspicion or a hunch will not validate a “stop-and-
frisk”. A genuine reason must exist, in light of the police officer’s experience and surrounding conditions,
to warrant the belief that the person detained has weapons concealed about him. Finally, a “stop-and-
frisk” serves a two-fold interest: (1) the general interest of effective crime prevention and detection for
purposes of investigating possible criminal behavior even without probable cause; and (2) the interest of
safety and self-preservation which permit the police officer to take steps to assure himself that the
person with whom he deals is not armed with a deadly weapon that could unexpectedly and fatally be
used against the police officer.

A stop-and-frisk was defined as the act of a police officer to stop a citizen on the street, interrogate him,
and pat him for weapon(s) or contraband. It should also be emphasized that a search and seizureshould
precede the arrest for this principle to apply. The foregoing circumstances do not obtain in the case at
bar. To reiterate, accused-appellant was first arrested before the search and seizure of the alleged illegal
items found in his possession. The apprehending police operative failed to make any initial inquiry into
accused-appellant’sbusiness in the vicinity or the contents of the Zest-O juice box he was carrying. The
apprehending police officers only introduced themselves when they already had custody of accused-
appellant.

In the case at bar, neither the in flagrante delicto nor the “stop and frisk” principles is applicable
to justify the warrantless arrest and consequent search and seizure made by the police operatives on
accused-appellant.

Wherefore, accused-appellant Binad Sy Chua is hereby Acquitted

vii. Exigent and Emergency Circumstances

People v. De Gracia, 233 SCRA 716 (1994) – Kaye


People vs. de Gracia [GR 102009-10, 6 July 1994]

Second Division, Regalado (J): 5 concur


Facts: The incidents took place at the height of the coup d'etat staged in December, 1989 by ultra-
rightist elements headed by the Reform the Armed Forces Movement-Soldiers of the Filipino People
(RAM-SFP)against the Government. At that time, various government establishments and military camps
in Metro Manila were being bombarded by the rightist group with their "tora-tora" planes. At around
midnight of 30 November 1989, the 4th Marine Battalion of the Philippine Marines occupied Villamor
Air Base, while the Scout Rangers took over the Headquarters of the Philippine Army, the Army
Operations Center, and Channel 4, the government television station. Also, some elements of the
Philippine Army coming from Fort Magsaysay occupied the Greenhills Shopping Center in San Juan,
Metro Manila. On 1 December 1989, Maj. Efren Soria of the Intelligence Division, National Capital
Region Defense Command, was on board a brown Toyota car conducting a surveillance of the Eurocar
Sales Office located at Epifanio de los Santos Avenue (EDSA) in Quezon City, together with his team
composed of Sgt. Crispin Sagario, M/Sgt. Ramon Briones, S/Sgt. Henry Aquino, one S/Sgt. Simon and a
Sgt. Ramos. The surveillance, which actually started on the night of 30 November 1989 at around 10:00
p.m., was conducted pursuant to an intelligence report received by the division that said establishment
was being occupied by elements of the RAM-SFP as a communication command post. Sgt. Crispin
Sagario, the driver of the car, parked the vehicle around 10 to 15 meters away from the Eurocar building
near P. Tuazon Street, S/Sgt. Henry Aquino had earlier alighted from the car to conduct his surveillance
on foot. A crowd was then gathered near the Eurocar office watching the on-going bombardment near
Camp Aguinaldo. After a while a group of 5 men disengaged themselves from the crowd and walked
towards the car of the surveillance team. At that moment, Maj. Soria, who was then seated in front, saw
the approaching group and immediately ordered Sgt. Sagario to start the car and leave the area. As they
passed by the group, then only 6 meters away, the latter pointed to them, drew their guns and fired at
the team, which attack resulted in the wounding of Sgt. Sagario on the right thigh. Nobody in the
surveillance team was able to retaliate because they sought cover inside the car and they were afraid
that civilians or bystanders might be caught in the cross-fire. As a consequence, at around 6:30 a.m. of 5
December 1989, searching them composed of F/Lt. Virgilio Babao as team leader, M/Sgt. Lacdao, Sgt.
Magallion, Sgt. Patricio Pacatang, and elements of the 16th Infantry Battalion under one Col. delos
Santos raided the Eurocar Sales Office. They were able to find and confiscate 6 cartons of M-16
ammunition, five bundles of C-4 dynamites, M-shells of different calibers, and "molotov" bombs inside
one of the rooms belonging to a certain Col. Matillano which is located at the right portion of the
building. St. Oscar Obenia, the first one to enter the Eurocar building, saw Rolando De Gracia inside the
office of Col. Matillano, holding a C-4 and suspiciously peeping through a door. De Gracia was the only
person then present inside the room. A uniform with the nametag of Col. Matillano was also found. As a
result of the raid, the team arrested de Gracia, as well as Soprieso Verbo and Roberto Jimena who were
janitors at the Eurocar building. They were then made to sign an inventory, written in Tagalog, of the
explosives and ammunition confiscated by the raiding team. No search warrant was secured by the
raiding team because, according to them, at that time there was so much disorder considering that the
nearby Camp Aguinaldo was being mopped up by the rebel forces and there was simultaneous firing
within the vicinity of the Eurocar office, aside from the fact that the courts were consequently closed.
The group was able to confirm later that the owner of Eurocar office is a certain Mr. Gutierrez and that
de Gracia is supposedly a "boy" therein. de Gracia was charged in two separate informations for illegal
possession of ammunition and explosives in furtherance of rebellion, and for attempted homicide
(Criminal Cases Q-90-11755 and Q-90-11756, respectively), which were tried jointly by the Regional Trial
Court of Quezon City, Branch 103. During the arraignment, de Gracia pleaded not guilty to both charges.
However, he admitted that he is not authorized to posses any firearms, ammunition and/or explosive.
The parties likewise stipulated that there was a rebellion during the period from November 30 up to 9
December 1989. On 22 February 1991, the trial court rendered judgment acquitting de Gracia of
attempted homicide, but found him guilty beyond reasonable doubt of the offense of illegal possession
of firearms in furtherance of rebellion and sentenced him to serve the penalty of reclusion perpetua. De
Gracia appealed.
Issue: Whether the military operatives made a valid search and seizure during the height of the
December 1989 coup d’etat.

Held: It is admitted that the military operatives who raided the Eurocar Sales Office were not armed
with a search warrant at that time. The raid was actually precipitated by intelligence reports that said
office was being used as headquarters by the RAM. Prior to the raid, there was a surveillance conducted
on the premises wherein the surveillance team was fired at by a group of men coming from the Eurocar
building. When the military operatives raided the place, the occupants thereof refused to open the door
despite the requests for them to do so, thereby compelling the former to break into the office. The
Eurocar Sales Office is obviously not a gun store and it is definitely not an armory or arsenal which are
the usual depositories for explosives and ammunition. It is primarily and solely engaged in the sale of
automobiles. The presence of an unusual quantity of high-powered firearms and explosives could not be
justifiably or even colorably explained. In addition, there was general chaos and disorder at that time
because of simultaneous and intense firing within the vicinity of the office and in the nearby Camp
Aguinaldo which was under attack by rebel forces. The courts in the surrounding areas were obviously
closed and, for that matter, the building and houses therein were deserted. Under the foregoing
circumstances, the case falls under one of the exceptions to the prohibition against a warrantless
search. In the first place, the military operatives, taking into account the facts obtaining in this case, had
reasonable ground to believe that a crime was being committed. There was consequently more than
sufficient probable cause to warrant their action. Furthermore, under the situation then prevailing, the
raiding team had no opportunity to apply for and secure a search warrant from the courts. The trial
judge himself manifested that on 5 December 1989 when the raid was conducted, his court was closed.
Under such urgency and exigency of the moment, a search warrant could lawfully be dispensed with.

*Drug, Alcohol and Blood Tests


Requisites to be valid:
1. It must be random, and
2. It must be suspicionless.

(Atty. Manuel J. Laserna, Jr. v. Dangerous Drugs Board and Philippine Drug Enforcement Agency)
G.R. No. 158633

FACTS:
Petitioner Atty. Manuel J. Laserna, Jr., as citizen and taxpayer, also seeks in his Petition for Certiorari and
Prohibition under Rule 65 that Sec. 36(c), (d), (f), and (g) of RA 9165 be struck down as unconstitutional
for infringing on the constitutional right to privacy, the right against unreasonable search and seizure,
and the right against self-incrimination, and for being contrary to the due process and equal protection
guarantees.

SEC. 36. Authorized Drug Testing.—Authorized drug testing shall be done by any
government forensic laboratories or by any of the drug testing laboratories accredited
and monitored by the DOH to safeguard the quality of the test results. x x x The drug
testing shall employ, among others, two (2) testing methods, the screening test which
will determine the positive result as well as the type of drug used and the confirmatory
test which will confirm a positive screening test. x x x The following shall be subjected
to undergo drug testing:

(c) Students of secondary and tertiary schools.—Students of secondary and


tertiary schools shall, pursuant to the related rules and regulations as contained in the
school’s student handbook and with notice to the parents, undergo a random drug
testing x x x;

(d) Officers and employees of public and private offices.—Officers and


employees of public and private offices, whether domestic or overseas, shall be
subjected to undergo a random drug test as contained in the company’s work rules and
regulations, x x x for purposes of reducing the risk in the workplace. Any officer or
employee found positive for use of dangerous drugs shall be dealt with administratively
which shall be a ground for suspension or termination, subject to the provisions of
Article 282 of the Labor Code and pertinent provisions of the Civil Service Law;

xxxx

(f) All persons charged before the prosecutor’s office with a criminal offense
having an imposable penalty of imprisonment of not less than six (6) years and one (1)
day shall undergo a mandatory drug test;

(g) All candidates for public office whether appointed or elected both in the
national or local government shall undergo a mandatory drug test.

ISSUE: Whether or not the petition is meritorious.

RULING:
WHEREFORE, the Court resolves to GRANT the petition in G.R. No. 161658 and declares Sec. 36(g) of RA
9165 andCOMELEC Resolution No. 6486 as UNCONSTITUTIONAL; and to PARTIALLY GRANT the petition
in G.R. Nos. 157870 and 158633 by declaring Sec. 36(c) and (d) of RA 9165 CONSTITUTIONAL, but
declaring its Sec. 36(f)UNCONSTITUTIONAL. All concerned agencies are, accordingly, permanently
enjoined from implementing Sec. 36(f) and (g) of RA 9165.

Unlike the situation covered by Sec. 36(c) and (d) of RA 9165, the Court finds no valid
justification for mandatory drug testing for persons accused of crimes. In the case of students, the
constitutional viability of the mandatory, random, and suspicionless drug testing for students emanates
primarily from the waiver by the students of their right to privacy when they seek entry to the school,
and from their voluntarily submitting their persons to the parental authority of school authorities. In the
case of private and public employees, the constitutional soundness of the mandatory, random, and
suspicionless drug testing proceeds from the reasonableness of the drug test policy and requirement.

We find the situation entirely different in the case of persons charged before the public
prosecutor’s office with criminal offenses punishable with six (6) years and one (1) day
imprisonment. The operative concepts in the mandatory drug testing are “randomness” and
“suspicionless.” In the case of persons charged with a crime before the prosecutor’s office, a mandatory
drug testing can never be random or suspicionless. The ideas of randomness and being suspicionless are
antithetical to their being made defendants in a criminal complaint. They are not randomly picked;
neither are they beyond suspicion. When persons suspected of committing a crime are charged, they
are singled out and are impleaded against their will. The persons thus charged, by the bare fact of being
haled before the prosecutor’s office and peaceably submitting themselves to drug testing, if that be the
case, do not necessarily consent to the procedure, let alone waive their right to privacy.[40] To impose
mandatory drug testing on the accused is a blatant attempt to harness a medical test as a tool for
criminal prosecution, contrary to the stated objectives of RA 9165. Drug testing in this case would
violate a persons’ right to privacy guaranteed under Sec. 2, Art. III of the Constitution. Worse still, the
accused persons are veritably forced to incriminate themselves.
(Aquilino Q. Pimentel, Jr. v. Commission on Elections)
G.R. No. 161658

FACTS:
On December 23, 2003, the Commission on Elections (COMELEC) issued Resolution No. 6486,
prescribing the rules and regulations on the mandatory drug testing of candidates for public office in
connection with the May 10, 2004 synchronized national and local elections.

(g) All candidates for public office x x x both in the national or local government shall undergo a
mandatory drug test.

SECTION 1. Coverage.—All candidates for public office, both national and local, in the
May 10, 2004 Synchronized National and Local Elections shall undergo mandatory drug
test in government forensic laboratories or any drug testing laboratories monitored and
accredited by the Department of Health.

Petitioner Aquilino Q. Pimentel, Jr., a senator of the Republic and a candidate for re-election in the May
10, 2004elections,[1] filed a Petition for Certiorari and Prohibition under Rule 65. In it, he seeks (1) to
nullify Sec. 36(g) of RA 9165 and COMELEC Resolution No. 6486 dated December 23, 2003 for being
unconstitutional in that they impose a qualification for candidates for senators in addition to those
already provided for in the 1987 Constitution; and (2) to enjoin the COMELEC from implementing
Resolution No. 6486.

Pimentel invokes as legal basis for his petition Sec. 3, Article VI of the Constitution, which states:

SECTION 3. No person shall be a Senator unless he is a natural-born citizen of


the Philippines, and, on the day of the election, is at least thirty-five years of age, able to
read and write, a registered voter, and a resident of the Philippines for not less than two
years immediately preceding the day of the election.

ISSUE: Whether or not the petition is meritorious. >> YES

RULING:
Pimentel’s contention is well-taken. Accordingly, Sec. 36(g) of RA 9165 should be, as it is hereby
declared as, unconstitutional. It is basic that if a law or an administrative rule violates any norm of the
Constitution, that issuance is null and void and has no effect. The Constitution is the basic law to which
all laws must conform; no act shall be valid if it conflicts with the Constitution.[8] In the discharge of their
defined functions, the three departments of government have no choice but to yield obedience to the
commands of the Constitution. Whatever limits it imposes must be observed.[9] hus, legislative power
remains limited in the sense that it is subject to substantive and constitutional limitations which
circumscribe both the exercise of the power itself and the allowable subjects of legislation.[11] The
substantive constitutional limitations are chiefly found in the Bill of Rights[12] and other provisions, such
as Sec. 3, Art. VI of the Constitution prescribing the qualifications of candidates for senators.

In the same vein, the COMELEC cannot, in the guise of enforcing and administering election laws
or promulgating rules and regulations to implement Sec. 36(g), validly impose qualifications on
candidates for senator in addition to what the Constitution prescribes. If Congress cannot require a
candidate for senator to meet such additional qualification, the COMELEC, to be sure, is also without
such power. The right of a citizen in the democratic process of election should not be defeated by
unwarranted impositions of requirement not otherwise specified in the Constitution.[1

It ought to be made abundantly clear, however, that the unconstitutionality of Sec. 36(g) of RA
9165 is rooted on its having infringed the constitutional provision defining the qualification or
eligibility requirements for one aspiring to run for and serve as senator.

7. Warrantless Arrests

Rule 113, Section 5. A peace officer or a private person may, without a warrant, arrest a person:

a. When, in his presence, the person to be arrested has committed, is actually committing, or
attempting to commit an offense;
b. When an offense has in fact been committed, and he has personal knowledge of facts indicating
that the person to be arrested has committed it; and
c. When the person to be arrested is a prisoner who has escaped from a penal establishment or place
where he is serving final judgment or temporarily confined while his case is pending or has escaped
while being transferred from one confinement to another

A. In Flagrante Delicto

People v. De La Cruz, GR 83260, April 18, 1990- Kaye


FACTS
Members of the PNP Narcotics Command received information that one “ Jun” [Doria] was engaged in
illegal drug activities, so they decided to entrap and arrest him in a buy-bust operation. He was arrested.
They frisked him but did not find the marked bills on him, and upon inquiry, he revealed that he left it at
the house of his associate “ Neneth ” [Gaddao], so he led the police team to her house.

The team found the door open and a woman inside the house. “ Jun” identified her
as “Neneth, ” and she was asked by SPO1 Badua about the marked money as PO3 Manlangit looked over
her house [he was still outside the house]. Standing by the door, PO3 Manlangit noticed a carton box
under the dining table. One of the box’ s flaps was open, and inside it was something wrapped in plastic,
and it appeared similar to the marijuana earlier sold to him by “ Jun. ” His suspicion aroused, so he
entered the house and took hold of the box. He peeked inside the box and saw 10 bricks of what
appeared to be dried marijuana leaves. SPO1 Badua recovered the marked bills from “ Neneth ” and
they arrested her. The bricks were examined and they were found to be dried marijuana leaves.

Florencio Doria and Violeta Gaddao were charged with violation of RA 6425 [Dangerous Drugs Act
of 1972], Section 4 [Sale, Administration, Delivery, Distribution and Transportation of Prohibited Drugs]
in relation to Section 21 [Attempt and Conspiracy]. RTC convicted them.

ISSUE AND HOLDING


WON RTC correctly found that the box of marijuana was in plain view, making its warrantless seizure
valid. NO
RATIO
Re: warrantless arrest
Gaddao ’s warrantless arrest was illegal because she was arrested solely on the basis of the alleged
identification made by Doria. Doria did not point to her as his associate in the drug business, but as the
person with whom he left the marked bills. This identification does not necessarily mean that Gaddao
conspired with Doria in pushing drugs. If there is no showing that the person who effected the
warrantless arrest had knowledge of facts implicating the person arrested to the perpetration of the
criminal offense, the arrest is legally objectionable.

Since the warrantless arrest of Gaddao was illegal, the search of her person and home and the
subsequent seizure of the marked bills and marijuana cannot be deemed legal as an incident to her
arrest.

“ Plain view ” issue


Objects falling in plain view of an officer who has a right to be in the position to have that view are
subject to seizure even without a search warrant and may be introduced in evidence.

Requisites
a. The law enforcement officer in search of the evidence has a prior justification for an
intrusion or is in a position from which he can view a particular area
b. The discovery of the evidence in plain view is inadvertent
c. It is immediately apparent to the officer that the item he observes may be evidence of a
crime, contraband or otherwise subject to seizure

An object is in plain view if the object itself is plainly exposed to sight. The difficulty arises when the
object is inside a closed container. Where the object seized was inside a closed package, the object
itself is not in plain view and therefore cannot be seized without a warrant. If the package is such that
an experienced observer could infer from its appearance that it contains the prohibited article, then the
article is deemed in plain view. It must be immediately apparent to the police that the items that they
observe may be evidence of a crime, contraband or otherwise subject to seizure.
In his direct examination, PO3 Manlangit said that he was sure that the contents of the box were
marijuana because he himself checked and marked the said contents. On cross-examination, however,
he admitted that he merely presumed the contents to be marijuana because it had the same plastic
wrapping as the "buy-bust marijuana." Each of the ten bricks of marijuana in the box was individually
wrapped in old newspaper and placed inside plastic bags-- white, pink or blue in color. PO3 Manlangit
himself admitted on cross-examination that the contents of the box could be items other than
marijuana. He did not know exactly what the box contained that he had to ask appellant Gaddao about
its contents. It was not immediately apparent to PO3 Manlangit that the content of the box was
marijuana; hence, it was not in plain view and its seizure without the requisite search warrant was in
violation of the law and the Constitution. It was fruit of the poisonous tree and should have been
excluded and never considered by the trial court.
The fact that the box containing about 6 kilos of marijuana was found in Gaddao ’s house Gaddao does
not justify a finding that she herself is guilty of the crime charged.
In a prosecution for illegal sale of dangerous drugs, what is material is the submission of proof that the
sale took place between the poseur-buyer and the seller and the presentation of the drug as evidence in
court.
 Prosecution established the fact that in consideration of the P1,600.00 he received, Doria sold
and delivered 970 grams of marijuana to PO3 Manlangit, the poseur-buyer
 Prosecution failed to prove that Gaddao conspired with accused-appellant Doria in the sale of
said drug

DORIA SENTENCED TO SUFFER RECLUSION PERPETUA + 500K FINE


GADDAO ACQUITTED

People v. Doria, GR 125299, January 22, 1999- Kaye


PEOPLE v DORIA, GR 125299

FACTS
Members of the PNP Narcotics Command received information that one “ Jun” [Doria] was engaged in
illegal drug activities, so they decided to entrap and arrest him in a buy-bust operation. He was arrested.
They frisked him but did not find the marked bills on him, and upon inquiry, he revealed that he left it at
the house of his associate “ Neneth ” [Gaddao], so he led the police team to her house.

The team found the door open and a woman inside the house. “ Jun” identified her
as “Neneth, ” and she was asked by SPO1 Badua about the marked money as PO3 Manlangit looked over
her house [he was still outside the house]. Standing by the door, PO3 Manlangit noticed a carton box
under the dining table. One of the box’ s flaps was open, and inside it was something wrapped in plastic,
and it appeared similar to the marijuana earlier sold to him by “ Jun. ” His suspicion aroused, so he
entered the house and took hold of the box. He peeked inside the box and saw 10 bricks of what
appeared to be dried marijuana leaves. SPO1 Badua recovered the marked bills from “ Neneth ” and
they arrested her. The bricks were examined and they were found to be dried marijuana leaves.

Florencio Doria and Violeta Gaddao were charged with violation of RA 6425 [Dangerous Drugs Act
of 1972], Section 4 [Sale, Administration, Delivery, Distribution and Transportation of Prohibited Drugs]
in relation to Section 21 [Attempt and Conspiracy]. RTC convicted them.

ISSUE:
WON RTC correctly found that the box of marijuana was in plain view, making its warrantless seizure
valid. NO

RATIO
Re: warrantless arrest
Gaddao ’s warrantless arrest was illegal because she was arrested solely on the basis of the alleged
identification made by Doria. Doria did not point to her as his associate in the drug business, but as the
person with whom he left the marked bills. This identification does not necessarily mean that Gaddao
conspired with Doria in pushing drugs. If there is no showing that the person who effected the
warrantless arrest had knowledge of facts implicating the person arrested to the perpetration of the
criminal offense, the arrest is legally objectionable.
Since the warrantless arrest of Gaddao was illegal, the search of her person and home and the
subsequent seizure of the marked bills and marijuana cannot be deemed legal as an incident to her
arrest.

“ Plain view ” issue


Objects falling in plain view of an officer who has a right to be in the position to have that view are
subject to seizure even without a search warrant and may be introduced in evidence.

Requisites
d. The law enforcement officer in search of the evidence has a prior justification for an
intrusion or is in a position from which he can view a particular area
e. The discovery of the evidence in plain view is inadvertent
f. It is immediately apparent to the officer that the item he observes may be evidence of a
crime, contraband or otherwise subject to seizure

An object is in plain view if the object itself is plainly exposed to sight. The difficulty arises when the
object is inside a closed container. Where the object seized was inside a closed package, the object
itself is not in plain view and therefore cannot be seized without a warrant. If the package is such that
an experienced observer could infer from its appearance that it contains the prohibited article, then the
article is deemed in plain view. It must be immediately apparent to the police that the items that they
observe may be evidence of a crime, contraband or otherwise subject to seizure.
In his direct examination, PO3 Manlangit said that he was sure that the contents of the box were
marijuana because he himself checked and marked the said contents. On cross-examination, however,
he admitted that he merely presumed the contents to be marijuana because it had the same plastic
wrapping as the "buy-bust marijuana." Each of the ten bricks of marijuana in the box was individually
wrapped in old newspaper and placed inside plastic bags-- white, pink or blue in color. PO3 Manlangit
himself admitted on cross-examination that the contents of the box could be items other than
marijuana. He did not know exactly what the box contained that he had to ask appellant Gaddao about
its contents. It was not immediately apparent to PO3 Manlangit that the content of the box was
marijuana; hence, it was not in plain view and its seizure without the requisite search warrant was in
violation of the law and the Constitution. It was fruit of the poisonous tree and should have been
excluded and never considered by the trial court.

The fact that the box containing about 6 kilos of marijuana was found in Gaddao ’s house Gaddao does
not justify a finding that she herself is guilty of the crime charged.

In a prosecution for illegal sale of dangerous drugs, what is material is the submission of proof that the
sale took place between the poseur-buyer and the seller and the presentation of the drug as evidence in
court.
 Prosecution established the fact that in consideration of the P1,600.00 he received, Doria sold
and delivered 970 grams of marijuana to PO3 Manlangit, the poseur-buyer
 Prosecution failed to prove that Gaddao conspired with accused-appellant Doria in the sale of
said drug

DORIA SENTENCED TO SUFFER RECLUSION PERPETUA + 500K FINEGADDAO ACQUITTED

Espiritu v. Lim, GR 85727, October 3, 1991- Kaye


G.R. No. 85727 October 3, 1991

IN THE MATTER OF APPLICATION FOR HABEAS CORPUS OF DEOGRACIAS ESPIRITU, petitioner,


vs.
BRIG. GEN.ALFREDO S. LIM, COL. RICARDO REYES, respondents.

NOTE: the case itself is a consolidation of so many cases. The decision starts with this “The Court avails
of this opportunity to clarify its ruling a begins with the statement that the decision did not rule — as
many misunderstood it to do — that mere suspicion that one is Communist Party or New People's Army
member is a valid ground for his arrest without warrant. Moreover, the decision merely applied long
existing lawsto the factual situations obtaining in the several petitions. Among these laws are th
outlawing the Communist Party of the Philippines (CPP) similar organizations and penalizing membership
therein be dealt with shortly). It is elementary, in this connection, if these laws no longer reflect the
thinking or sentiment of the people, it is Congress as the elected representative of the people — not the
Court — that should repeal, change or modify them.”

FACTS:

In G.R. No. 85727, Espiritu, on 23 November 1988, was arrested without warrant, on the basis of the
attestation of certain witnesses: that about 5:00 o'clock in the afternoon of 22 November 1988, at the
corner of Magsaysay Boulevard and Velencia St., Sta. Mesa, Manila, Espiritu spoke at a gathering of
drivers and sympathizers, where he said, among other things:

Bukas tuloy ang welga natin . . . hanggang sa magkagulona. 27 (Emphasis supplied)

and that the police authorities were present during the press conference held at the National Press Club
(NPC) on 22 November 1988 where Espiritu called for a nationwide strike (of jeepney and bus drivers)
on 23 November 1988. 28 Espiritu was arrested without warrant, not for subversion or any "continuing
offense," but for uttering the above-quoted language which, in the perception of the arresting officers,
was inciting to sedition.

ISSUE:
Whether or not the arrest of Espiritu was valid. >> YES, it was valid.

RULING:

Many persons may differ as to the validity of such perception and regard the language as falling within
free speech guaranteed by the Constitution. But, then, Espiritu had not lost the right to insist, during the
pre-trial or trial on the merits, that he was just exercising his right to free speech regardless of the
charged atmosphere in which it was uttered. But, the authority of the peace officers to make the arrest,
without warrant, at the time the words were uttered, or soon thereafter, is still another thing. In the
balancing of authority and freedom, which obviously becomes difficult at times, the Court has, in this
case, tilted the scale in favor of authority but only for purposes of the arrest (not conviction). Let it be
noted that the Court has ordered the bail for Espiritu's release to be reduced from P60,000.00 to
P10,000.00.
Let it also be noted that supervening events have made the Espiritu case moot and academic. For
Espiritu had before arraignment asked the court a quo for re-investigation, the peace officers did not
appear. Because of this development, the defense asked the court a quo at the resumption of the
hearings to dismiss the case. Case against Espiritu (Criminal Case No. 88-68385) has been provisionally
dismissed and his bail bond cancelled.

This Court reiterates that shortly after the arrests of Espiritu and Nazareno, the corresponding
informations against them were filed in court. The arrests of Espiritu and Nazareno were based on
probable cause and supported by factual circumstances. They complied with conditions set forth in
Section 5(b) of Rule 113. They were not arbitrary or whimsical arrests.

Umil v. Fidel Ramos, GR 81567, July 9, 1990- Kaye

G.R. No. 81567 October 3, 1991

IN THE MATTER OF THE PETITION FOR HABEAS CORPUS OF ROBERTO UMIL, ROLANDO DURAL and
RENATO VILLANUEVA, MANOLITA O. UMIL and NICANOR P. DURAL, FELICITAS V. SESE, petitioners,
vs.
FIDEL V. RAMOS, MAJ. GEN. RENATO DE VILLA, BRIG. GEN. RAMON MONTANO, BRIG. GEN.
ALEXANDER AGUIRRE, respondents.

NOTE: the case itself is a consolidation of so many cases. The decision starts with this “The Court avails
of this opportunity to clarify its ruling a begins with the statement that the decision did not rule — as
many misunderstood it to do — that mere suspicion that one is Communist Party or New People's Army
member is a valid ground for his arrest without warrant. Moreover, the decision merely applied long
existing lawsto the factual situations obtaining in the several petitions. Among these laws are th
outlawing the Communist Party of the Philippines (CPP) similar organizations and penalizing membership
therein be dealt with shortly). It is elementary, in this connection, if these laws no longer reflect the
thinking or sentiment of the people, it is Congress as the elected representative of the people — not the
Court — that should repeal, change or modify them.”

FACTS:

In G.R. No. 81567 (Umil case), military agents, on 1 February 1988, were dispatched to the St. Agnes
Hospital, Roosevelt Avenue, Quezon City, to verify a confidential information which was received by
their office, about a "sparrow man" (NPA member) who had been admitted to the said hospital with a
gunshot wound; that the information further disclosed that the wounded man in the said hospital was
among the five (5) male "sparrows" who murdered two (2) Capcom mobile patrols the day before, or on
31 January 1988 at about 12:00 o'clock noon, before a road hump along Macanining St., Bagong Barrio,
Caloocan City; that based on the same information, the wounded man's name was listed by the hospital
management as "Ronnie Javellon," twenty-two (22) years old of Block 10, Lot 4, South City Homes,
Biñan, Laguna.
ISSUE:
Whether or not the arrest was valid. >> YES, it was valid.

RULING:
Said confidential information received by the arresting officers, to the effect that an NPA member
("sparrow unit") was being treated for a gunshot wound in the named hospital, is deemed reasonable
and with cause as it was based on actual facts and supported by circumstances sufficient to engender
a belief that an NPA member was truly in the said hospital. The actual facts supported by
circumstances are: first — the day before, or on 31 January 1988, two (2) CAPCOM soldiers were
actually killed in Bagong Bario, Caloocan City by five (5) "sparrows" including Dural; second — a
wounded person listed in the hospital records as "Ronnie Javellon" was actually then being treated in St.
Agnes Hospital for a gunshot wound; third — as the records of this case disclosed later, "Ronnie
Javellon" and his address entered in the hospital records were fictitious and the wounded man was in
reality Rolando Dural.

In fine, the confidential information received by the arresting officers merited their immediate attention
and action and, in fact, it was found to be true. Even the petitioners in their motion for
reconsideration, 13 believe that the confidential information of the arresting officers to the effect that
Dural was then being treated in St. Agnes Hospital was actually received from the attending doctor and
hospital management in compliance with the directives of the law, 14 and, therefore, came from
reliable sources.

As to the condition that "probable cause" must also be coupled with acts done in good faith by the
officers who make the arrest, the Court notes that the peace officers wno arrested Dural are deemed to
have conducted the same in good faith, considering that law enforcers are presumed to regularly
perform their official duties. The records show that the arresting officers did not appear to have been ill-
motivated in arresting Dural. 15 It is therefore clear that the arrest, without warrant, of Dural was made
in compliance with the requirements of paragraphs (a) and (b) of Section 5, Rule 113.

People v. Sucro, 195 SCRA 388


Facts: Pat. Fulgencio went to Arlie Regalado’s house at C. Quimpo to monitor activities of Edison SUCRO
(accused). Sucro was reported to be selling marijuana at a chapel 2 meters away from Regalado’s house.
Sucro was monitored to have talked and exchanged things three times. These activities are reported
through radio to P/Lt. Seraspi. A third buyer was transacting with appellant and was reported and later
identified as Ronnie Macabante. From that moment, P/Lt.Seraspi proceeded to the area. While the
police officers were at the Youth Hostel in Maagama St. Fulgencio told Lt. Seraspi to intercept.
Macabante was intercepted at Mabini and Maagama crossing in front of Aklan Medical center.
Macabante saw the police and threw a tea bag of marijuana on the ground. Macabante admitted buying
the marijuana from Sucro in front of the chapel.

The police team intercepted and arrested SUCRO at the corner of C. Quimpo and Veterans. Recovered
were 19 sticks and 4 teabags of marijuana from a cart inside the chapel and another teabag from
Macabante.

Issues:

(1) Whether or Not arrest without warrant is lawful.

(2) Whether or Not evidence from such arrest is admissible.


Held: Search and seizures supported by a valid warrant of arrest is not an absolute rule. Rule 126, Sec 12
of Rules of Criminal Procedure provides that a person lawfully arrested may be searched for dangerous
weapons or anything, which may be used as proff of the commission of an offense, without a search
warrant.(People v. Castiller) The failure of the police officers to secure a warrant stems from the fact
that their knowledge required from the surveillance was insufficient to fulfill requirements for its
issuance. However, warantless search and seizures are legal as long as PROBABLE CAUSE existed. The
police officers have personal knowledge of the actual commission of the crime from the surveillance of
the activities of the accused. As police officers were the ones conducting the surveillance, it is presumed
that they are regularly in performance of their duties.

People v. Rodrigueza, 205 SCRA 791-


Facts: NARCOM agents staged a buy-bust operation, after gaining information that there was an
ongoing illegal traffic of prohibited drugs in Tagas, Albay. The participating agents were given money
treated with ultraviolet powder. One of the agents went to said location, asked for a certain Don.
Thereafter, the Don, herein accused, met with him and “a certain object wrapped in a plastic” later
identified as marijuana was given in exchange for P200. The agent went back to headquarters and made
a report, based on which, a team was subsequently organized and a raid was conducted in the house of
the father of the accused. During the raid, the NARCOM agents were able to confiscate dried marijuana
leaves and a plastic syringe among others. There was no authorization by any search warrant. The
accused was found positive of ultraviolet powder. The lower court, considering the evidences obtained
and testimonies from the prosecution, found him guilty of violating the Dangerous Drugs Act of 1972
and sentenced him to reclusion perpetua.

Issue: Whether or Not the lower court was correct in its judgment.

Held: The NARCOM agents’ procedure in the entrapment of the accused failed to meet the qualification
that the suspected drug dealer must be caught red-handed in the act of selling marijuana to a person
posing as a buyer, since the operation was conducted after the actual exchange. Said raid also violated
accused’ right against unreasonable search and seizure, as the situation did not fall in the circumstances
wherein a search may be validly made even without a search warrant, i.e. when the search is incidental
to a lawful arrest; when it involves prohibited articles in plain view. The NARCOM agents could not have
justified their act by invoking the urgency and necessity of the situation because the testimonies of the
prosecution witnesses reveal that the place had already been put under surveillance for quite some
time. Had it been their intention to conduct the raid, then they should, because they easily could, have
first secured a search warrant during that time. The Court further notes the confusion and ambiguity in
the identification of the confiscated marijuana leaves and other prohibited drug paraphernalia
presented as evidence against appellant:

CIC Taduran, who acted as the poseur buyer, testified that appellant sold him 100 grams of dried
marijuana leaves wrapped in a plastic bag. Surprisingly, and no plausible explanation has been advanced
therefor, what were submitted to and examined by the PCCL and thereafter utilized as evidence against
the appellant were the following items:

One (1) red and white colored plastic bag containing the following:
Exh. "A"—Thirty (30) grams of suspected dried marijuana fruiting tops contained inside a transparent
plastic bag.
Exh. "B"— Fifty (50) grams of suspected dried marijuana leaves and seeds contained inside a white
colored plastic labelled "Robertson".
Exh. "C"— Four (4) aluminum foils each containing suspected dried marijuana fruiting tops having a total
weight of seven grams then further wrapped with a piece of aluminum foil.
Exh. "D"— Five (5) small transparent plastic bags each containing suspected dried marijuana fruiting
tops having a total weight of seventeen grams.
Exh. "E"— One plastic syringe.

Evidently, these prohibited articles were among those confiscated during the so-called follow-up raid in
the house of Rodrigueza’s father. The unanswered question then arises as to the identity of the
marijuana leaves that became the basis of appellant's conviction. In People vs. Rubio, this Court had the
occasion to rule that the plastic bag and the dried marijuana leaves contained therein constitute the
corpus delicti of the crime. As such, the existence thereof must be proved with certainty and
conclusiveness. Failure to do so would be fatal to the cause of the prosecution. Conviction is reversed
and set aside and accused is acquitted.

People v. Yap, 229 SCRA 787-


Facts: Accused-appellants Edgardo Yap and Simplicio Osmeña were charged with a violation of the
Dangerous Drugs Act of 1972, as amended, for having sold on October 1, 1989 in Ozamiz City six sticks of
marijuana for a consideration of ten pesos.

Appellants asserted that the team who arrested them was not provided with a warrant of arrest when
appellants were apprehended. This specious argument is readily disposed of by Section 5(c) of Rule 113
which provides that a peace officer or a private person may effect an arrest without a warrant when an
offense has in fact just been committed, and he has personal knowledge of facts indicating that the
person to be arrested has committed it. The police officer in this case who arrested the appellants had
personal knowledge of the commission of the crime, having been present in the locus criminis and
having actually witnessed the unlawful transaction. The interval between the commission of the crime
and the time of the arrests was only four to five minutes.

A buy-bust operation is a form of entrapment employed by peace officers to apprehend a malefactor in


flagrante delicto, that is, to catch him red-handed while selling marijuana to a person acting as a poseur-
buyer. 15 Consequently, and contrary to the claim of the defense, appellants were positively identified by
the poseur-buyer, Raterta, and Sgt. Mugot (the police officer mentioned above), who were undeniably
eyewitnesses to the crime.

People v. Alolod, 266 SCRA 154- (In Flagrante Delicto was NEVER mentioned in this case)
Alolod was charged with homicide and illegal possession of firearm. He was found guilty of shooting a
victim while they were all riding a jeepney. He allegedly grabbed a plastic bag from a victim who refused
so he shot the victim (still alive at that time) and then they wrestled over the gun and fell off the jeep
and the nthe police came but Alolod was able to run away. The police officer pursued Alolod until he
caught up with him at a certain street and seized from the accused was a .38 cal. paltik revolver still
smelling of gunpowder, with four (4) live bullets and two (2) empty shells. Also found in his possession
was a plastic bag containing P17,800.00 in various denominations. According to the police officer,
Alolod confessed about the crime
Alolod’s version was different since he pleaded not guilty.
WON his conviction is proper
Ruling: Yes. After a careful assessment of the evidence, we hold that the inconsistencies alleged by the
accused are more apparent than real, and that there is no compelling reason to disturb the findings of
the trial court in this regard.

People v. Mengote – 210 SCRA 174-


Facts: The Western Police District received a telephone call from an informer that there were three
suspicious looking persons at the corner of Juan Luna and North Bay Boulevard in Tondo, Manila. A
surveillance team of plainclothesmen was forthwith dispatched to the place. The patrolmen saw two
men looking from side to side, one of whom holding his abdomen. They approached the persons and
identified themselves as policemen, whereupon the two tried to run but unable to escape because the
other lawmen surrounded them. The suspects were then searched. One of them the accused-appellant
was found with a .38 caliber with live ammunitions in it, while his companion had a fan knife. The
weapons were taken from them and they were turned over to the police headquarters for investigation.
An information was filed before the RTC convicting the accused of illegal possession of firearm arm. A
witness testified that the weapon was among the articles stolen at his shop, which he reported to the
police including the revolver. For his part, Mengote made no effort to prove that he owned the fire arm
or that he was licensed to possess it but instead, he claimed that the weapon was planted on him at the
time of his arrest. He was convicted for violation of P.D.1866 and was sentenced to reclusion perpetua.
In his appeal he pleads that the weapon was not admissible as evidence against him because it had been
illegally seized and therefore the fruit of a poisonous tree.

Issue: Whether or not the warrantless search and arrest was illegal.

Held: An evidence obtained as a result of an illegal search and seizure inadmissible in any proceeding for
any purpose as provided by Art. III sec 32 of the Constitution. Rule 113 sec.5 of the Rules of Court,
provides arrest without warrant lawful when: (a) the person to be arrested has committed, is actually
committing, or is attempting to commit an offense, (b) when the offense in fact has just been
committed, and he has personal knowledge of the facts indicating the person arrested has committed it
and (c) the person to be arrested has escaped from a penal establishment or a place where he is serving
final judgment or temporarily confined while his case is pending, or has escaped while being transferred
from one confinement to another.

These requirements have not been established in the case at bar. At the time of the arrest in question,
the accused appellant was merely looking from side to side and holding his abdomen, according to the
arresting officers themselves. There was apparently no offense that has just been committed or was
being actually committed or at least being attempt by Mengote in their presence. Moreover a person
may not be stopped and frisked in a broad daylight or on a busy street on unexplained suspicion.

Judgment is reversed and set aside. Accused-appellant is acquitted


People v. Elamparo – 329 SCRA -
Summary: A buy-bust operation led the pursuing officers of a drug runner into the house of his alleged
supplier/dealer. In the house, they witnessed the alleged dealer repacking bricks of marijuana and
arrested him.

Rule of Law: A peace officer or a private person may, without a warrant, arrest a person, when in his
presence, the person to be arrested has committed, is actually committing, or is attempting to commit
an offense.

Facts: Joel Elamparo (D) has been convicted with Illegal Possession of Drugs and penalized with reclusion
perpetua. The case was raised for automatic review.

Police Officer Baldonado of Caloocan City Police received a report from an informant that "some people
are selling shabu and marijuana somewhere in Bagong Bario, Caloocan City." Thus, Baldonado organized
a buy-bust team and deployed at a known "market" for buyers of marijuana. Thereafter, a runner
approached the poseur-buyer to confirm an order. The runner then left and returned with the
marijuana. Gaviola, the poseur-buyer and buy-bust team member, then handed over the marked money
and arrested the runner who freed himself and ran.

The buy-bust team pursued the runner, who ran inside a bungalow-type house with steel gate. Having
trapped the runner inside the house, the police officers frisked him and recovered the marked money.
The police officers likewise found Joel Elamparo (D) repacking five bricks of "marijuana" wrapped in a
newspaper on top of the round table inside the house. Elamparo (D) was then arrested.

Issues: Is the warrantless arrest valid?

Ruling: Yes. Five generally accepted exceptions to the right against warrantless searches and seizures
have been judicially formulated: (1) search incidental to a lawful arrest, (2) search of moving vehicles, (3)
seizure in plain view, (4) customs searches, and (5) waiver by the accused themselves of their right
against unreasonable search and seizure.

This case falls squarely under the plain view doctrine. In People vs. Doria, 301 SCRA 668, 710-711
(1999), the Court held that—

Objects falling in plain view of an officer who has a right to be in the position to have that view are
subject to seizure even without a search warrant and may be introduced in evidence. The "plain view"
doctrine applies when the following requisites concur (a) the law enforcement officer in search of the
evidence has a prior justification for an intrusion or is in a position from which he can view a particular
area; (b) the discovery of the evidence in plain view is inadvertent; (c) it is immediately apparent to the
officer that the item he observes may be evidence of a crime, contraband or otherwise subject to
seizure. The law enforcement officer must lawfully make an initial intrusion or properly be in a position
from which he can particularly view the area. In the course of such lawful intrusion, he came
inadvertently across a piece of evidence incriminating the accused. The object must be open to eye and
hand and its discovery inadvertent.

When the runner wrenched himself free from the grasp of Gaviola, he instinctively ran towards the
house of Elamparo (D). The members of the buy-bust team were justified in running after him and
entering the house without a search warrant for they were pursuing a fleeing criminal. Once inside the
house, the police officers cornered the runner and recovered the buy-bust money from him. They also
caught Elamparo (D) in flagrante delicto repacking the marijuana bricks which were in full view on tap of
a table.

B. Hot Pursuit

Two Requisites:

1. An offense had just been committed.


2. The person making the arrest has probable cause to believe, based on his personal knowledge of
facts and circumstances, that the person to be arrested committed it.
*There must be immediacy between the time the offense is committed and the time of the arrest.

Go v. CA – 206 SCRA 138-


Facts: Petitioner, while traveling in the wrong direction on a one-way street, almost had a collision with
another vehicle. Petitioner thereafter got out of his car, shot the driver of the other vehicle, and drove
off. An eyewitness of the incident was able to take down petitioner’s plate number and reported the
same to the police, who subsequently ordered a manhunt for petitioner. 6 days after the shooting,
petitioner presented himself in the police station, accompanied by 2 lawyers, the police detained him.
Subsequently a criminal charge was brought against him. Petitioner posted bail, the prosecutor filed the
case to the lower court, setting and commencing trial without preliminary investigation. Prosecutor
reasons that the petitioner has waived his right to preliminary investigation as bail has been posted and
that such situation, that petitioner has been arrested without a warrant lawfully, falls under Section 5,
Rule 113 and Section 7, Rule 112 of The 1985 Rules of Criminal Procedure which provides for the rules
and procedure pertaining to situations of lawful warrantless arrests. Petitioner in his petition for
certiorari assails such procedure and actions undertaken and files for a preliminary investigation.

Issues:
(1) WON warrantless arrest of petitioner was lawful.
(2) WON petitioner effectively waived his right to preliminary investigation.

Held: Petitioner and prosecutor err in relying on Umil v. Ramos, wherein the Court upheld the
warrantless arrest as valid effected 1 to 14 days from actual commission of the offenses, which however
constituted “continuing crimes,” i.e. subversion, membership in an outlawed organization, etc. There
was no lawful warrantless arrest under Section 5, Rule 113. This is because the arresting officers were
not actually there during the incident, thus they had no personal knowledge and their information
regarding petitioner were derived from other sources. Further, Section 7, Rule 112, does not apply.
Petitioner was not arrested at all, as when he walked in the police station, he neither expressed
surrender nor any statement that he was or was not guilty of any crime. When a complaint was filed to
the prosecutor, preliminary investigation should have been scheduled to determine probable cause.
Prosecutor made a substantive error, petitioner is entitled to preliminary investigation, necessarily in a
criminal charge, where the same is required appear thereat. Petition granted, prosecutor is ordered to
conduct preliminary investigation, trial for the criminal case is suspended pending result from
preliminary investigation, petitioner is ordered released upon posting a bail bond.

Ruling version 2: The general rule on arrest provides that the same is legitimate if effected with a valid
warrant. However, there are instances specifically enumerated under the law when a warrantless arrest
may be considered lawful. Despite that, the warrantless arrest of herein petitioner Rolito Go does not fall
within the terms of said rule. The police were not present at the time of the commission of the offense,
neither do they have personal knowledge on the crime to be committed or has been committed not to
mention the fact that petitioner was not a prisoner who has escaped from the penal institution. In view
of the above, the allegation of the prosecution that petitioner needs to sign a waiver of the provisions
of Article 125 of the Revised Penal Code before a preliminary investigation may be
conducted is baseless. In this connection, petitioner has all the rig ht to ask for a
preliminary investigation
to determine whether is probable cause that a crime has been committed and that petitioner is
probably guilty thereof as well asto prevent him from the hassles, anxiety and aggravation
brought by a criminal proceeding.

On the other hand, petitioner did not waive his right to have a preliminary investigation contrary to the
prosecutor's claim. The right to preliminary investigation is deemed waived when the accused fails to
invoke it before or at the t i m e o f e n t e r i n g a p l e a a t a r r a i g n m e n t . T h e f a c t s o f t h e
c a s e s h o w t h a t p e t i t i o n e r i n s i s t e d o n h i s r i g h t t o preliminary investigation before his
arraignment and he, through his counsel denied answering questions before the court unless they were
afforded the proper preliminary investigation. For the above reasons, the petition was granted and the
ruling of the appellate court was set aside and nullified. The Supreme Court however, contrary to
petitioner's allegation, declared that failure to accord the right to preliminary investigation did not
impair the validity of the information charging the latter of the crime of murder.

People v. Manlulu, 231 SCRA 701 (1994)-

Alfaro, a NARCOM agent, was stabbed and shot in a drinking spree. His
d r i n k i n g companions, Manlulu and Samson were arrested nineteen hours after the incident.
Patrolman Perez arrested Manlulu on the information given by Manlapaz, who was also
drinking with the accused and the victim. Patrolman Perez seized from Manlulu the .45
cal. Pistol and Casio wristwatch said to belong to Alfaro, without a warrant and without informing
Manlulu of his rightto counsel.

ISSUE:Whether or not the arrest and seizure of the gun and the watch was valid WON there was in
flagrante delicto

Ruling: The warrantless arrest was invalid. The killing took place at one o’clock in the morning. The arrest
and the consequent search and seizure came at around seven o’clock that evening, some nineteen
hours later. This instance cannot come within the purview of a valid warrantless arrest. Paragraph (b) Sec. 5,
Rule 113 of the 1985 Rules on Criminal Procedure provides that the a r r e s t i n g o f f i c e r m u s t h a v e
“ p e r s o n a l k n o w l e d g e ” n o r w a s t h e o f f e n s e “ i n f a c t j u s t b e e n committed.” While Pat.
Perez may have personally gathered the information which led to the a r r e s t o f M a n l u l u , t h a t i s
n o t e n o u g h . T h e l a w r e q u i r e s “ p e r s o n a l k n o w l e d g e ” . O b v i o u s l y , “personal gathering
of information” is different from personal knowledge. The rule requires thatthe arrest immediately
follows the commission of the offense, not some nineteen hours later. However, the flaw, fatal as it may
be, becomes moot in view of the eyewitness account of Manlapaz which the Court found
credible. In spite of the nullification of the arrest of accused Manlulu, and the exclusion of
real evidence, as well as his extra-judicial confession which was taken in violation of the
Constitution, still the prosecution was able to prove the guilt of the accused beyond
reasonable doubt.

People v. Rodrigueza, 205 SCRA 791 (1992)-

People v. Rodrigueza, 205 SCRA 791-


Facts: NARCOM agents staged a buy-bust operation, after gaining information that there was an
ongoing illegal traffic of prohibited drugs in Tagas, Albay. The participating agents were given money
treated with ultraviolet powder. One of the agents went to said location, asked for a certain Don.
Thereafter, the Don, herein accused, met with him and “a certain object wrapped in a plastic” later
identified as marijuana was given in exchange for P200. The agent went back to headquarters and made
a report, based on which, a team was subsequently organized and a raid was conducted in the house of
the father of the accused. During the raid, the NARCOM agents were able to confiscate dried marijuana
leaves and a plastic syringe among others. There was no authorization by any search warrant. The
accused was found positive of ultraviolet powder. The lower court, considering the evidences obtained
and testimonies from the prosecution, found him guilty of violating the Dangerous Drugs Act of 1972
and sentenced him to reclusion perpetua.

Issue: Whether or Not the lower court was correct in its judgment.

Held: The NARCOM agents’ procedure in the entrapment of the accused failed to meet the qualification
that the suspected drug dealer must be caught red-handed in the act of selling marijuana to a person
posing as a buyer, since the operation was conducted after the actual exchange. Said raid also violated
accused’ right against unreasonable search and seizure, as the situation did not fall in the circumstances
wherein a search may be validly made even without a search warrant, i.e. when the search is incidental
to a lawful arrest; when it involves prohibited articles in plain view. The NARCOM agents could not have
justified their act by invoking the urgency and necessity of the situation because the testimonies of the
prosecution witnesses reveal that the place had already been put under surveillance for quite some
time. Had it been their intention to conduct the raid, then they should, because they easily could, have
first secured a search warrant during that time. The Court further notes the confusion and ambiguity in
the identification of the confiscated marijuana leaves and other prohibited drug paraphernalia
presented as evidence against appellant:

CIC Taduran, who acted as the poseur buyer, testified that appellant sold him 100 grams of dried
marijuana leaves wrapped in a plastic bag. Surprisingly, and no plausible explanation has been advanced
therefor, what were submitted to and examined by the PCCL and thereafter utilized as evidence against
the appellant were the following items:

One (1) red and white colored plastic bag containing the following:

Exh. "A"—Thirty (30) grams of suspected dried marijuana fruiting tops contained inside a transparent
plastic bag.
Exh. "B"— Fifty (50) grams of suspected dried marijuana leaves and seeds contained inside a white
colored plastic labelled "Robertson".
Exh. "C"— Four (4) aluminum foils each containing suspected dried marijuana fruiting tops having a total
weight of seven grams then further wrapped with a piece of aluminum foil.
Exh. "D"— Five (5) small transparent plastic bags each containing suspected dried marijuana fruiting
tops having a total weight of seventeen grams.
Exh. "E"— One plastic syringe.

Evidently, these prohibited articles were among those confiscated during the so-called follow-up raid in
the house of Rodrigueza’s father. The unanswered question then arises as to the identity of the
marijuana leaves that became the basis of appellant's conviction. In People vs. Rubio, this Court had the
occasion to rule that the plastic bag and the dried marijuana leaves contained therein constitute the
corpus delicti of the crime. As such, the existence thereof must be proved with certainty and
conclusiveness. Failure to do so would be fatal to the cause of the prosecution. Conviction is reversed
and set aside and accused is acquitted.

People v. Enrile, 222 SCRA 586-

Facts: The police through their buy-bust operation caught a certain Abugatal. His capture led the police
to the house of Enrile wherein Abugatal called him and when he got out Abugatal pointed at Enrile as
the source of the illicit drugs and so the police arrested him. The marked money used in the buy-bust
operation was in Enrile’s possession. was found guilty of violating the Dangerous Drugs Act.

WON he was caught in flagrante delicto (Note: I believe this case is not really related to hot pursuit. This
is more on in flagrante delicto )

Ruling: What the policemen should have done was secure a search warrant on the basis of the
information supplied by Abugatal, and then, with such authority, proceeded to search and, if the search
was fruitful, arrest Enrile. They had no right to simply force themselves into his house on the bare (and
subsequently disallowed) allegations of Abugatal and bundle Enrile off to the police station as if he had
been caught in flagrante delicto.

The discovery of the marked money on him did not mean he was caught in the act of selling marijuana.
The marked money was not prohibited per se. Even if it were, that fact alone would not retroactively
validate the warrantless search and seizure.

The principle has been honored through the ages in all liberty-loving regimes that a man's house is his
castle that not even the mighty monarch, with all its forces, may violate. There were measures available
under the law to enable the authorities to search Enrile's house and to arrest him if he was found in
possession of prohibited articles. The police did not employ these measures.

What they did was simply intrude into Enrile's house and arrest him without the slightest heed to the
injunctions of the Bill of Rights. By so doing, they were using the tactics of the police state, where the
minions of the government place little value on human rights and individual liberties and are obssessed
only with the maintenance of peace and punishment of crime.

People v. Jayson, 282 SCRA 166 (1997)-


Accused-appellant Wenceslao Jayson was charged with violation of P.D. No. 1866 (illegal possession of
lightsaber and invisibility cloak) in the Regional Trial Court of Davao City.
Accused-appellant maintains that he acted in the good faith belief that he was authorized to carry the
firearm by virtue of the mission order and memorandum receipt issued to him by Major Francisco
Arquillano of the Davao Metropolitan District Command. He argues that the failure of the prosecution
to present as witness the government official who had certified that accused-appellant was not the
holder of a firearms license is fatal to the prosecution of this case.

With respect to the arrest, SPO1 Loreto Tenebro testified that at around 10:00 in the evening of March
16, 1991, while he and Patrolmen Camotes and Reinerio Racolas were patrolling in their car, they
received a radio message from their camp directing them to proceed to the “Ihaw-Ihaw” on Bonifacio
Street where there had been a shooting. Accordingly, they proceeded to the place and there saw the
victim, Nelson Jordan. Bystanders pointed to accused-appellant as the one who had shot Jordan. They
then arrested accused-appellant. Seized from him was a .38 caliber revolver with serial number 91955.
The firearm was covered by a mission order and memorandum receipt. Considering these facts, we hold
that the warrantless arrest and search were valid.

Rule 113, §5(b) of the Revised Rules of Criminal Procedure provides:

Sec. 5. Arrest without warrant; when lawful. — A peace officer or private person may, without a
warrant, arrest a person: . . . .

(b) When an offense has in fact just been committed, and he has personal knowledge of facts indicating
that the person to be arrested has committed it.

In the case at bar there was a shooting. The policemen summoned to the scene of the crime found the
victim. Accused-appellant was pointed to them as the assailant only moments after the shooting. In fact
accused-appellant had not gone very far (only ten meters away from the “Ihaw-Ihaw”), although he was
then fleeing. The arresting officers thus acted on the basis of personal knowledge of the death of the
victim and of facts indicating that accused-appellant was the assailant.

This Court has upheld a warrantless arrest under analogous circumstances. In People v. Tonog, Jr., the
police found the lifeless body of a person with several stab wounds. An informer pointed to the accused
as the person who had killed the victim. That afternoon, police officers arrested the accused. On their
way to the police station, a policeman noticed bloodstains on the accused’s pants which, when
examined, was found to be the same blood type “O” found on the fatal knife. The Court upheld the
warrantless arrest and ruled that the blood-stained pants, having been seized as an incident of a lawful
arrest, was admissible in evidence.

In People v. Gerente the police arrested the accused three hours after the victim had been killed. They
went to the scene of the crime where they found a piece of wood and a concrete hollow block used by
the killers in bludgeoning the victim to death. A neighbor of the accused who witnessed the killing,
pointed to him as one of the assailants. The warrantless arrest was held valid under Rule 113, §5(b)

The subsequent search of accused-appellant’s person and the seizure from him of the firearm was
likewise lawful. Rule 126, §12 states:
Sec. 12. Search incident to lawful arrest. — A person lawfully arrested may be searched for dangerous
weapons or anything which may be used as proof of the commission of an offense, without a search
warrant.

People v. Del Rosario, GR 127755, April 14, 1999 – JM

Facts: Joselito del Rosario y Pascual, Ernesto Marquez alias “Jun,” Virgilio Santos alias “Boy Santos” and
John Doe alias “Dodong” were charged with special complex crime of Robbery with Homicide for having
robbed Virginia Bernas, a 66-year old businesswoman, of P200,000.00 in cash and jewelry and on the
occasion thereof shot and killed her.

Issue: Hot pursuit.

Ruling: The arrest of del Rosario does not comply with these requirements since, as earlier explained,
the arrest came a day after the consummation of the crime and not immediately thereafter. As such,
the crime had not been "just committed" at the time the accused was arrested. Likewise, the arresting
officers had no personal knowledge of facts indicating that the person to be arrested had committed the
offense since they were not present and were not actual eyewitnesses to the crime, and they became
aware of his identity as the driver of the getaway tricycle only during the custodial investigation.

People Samus, GR 135957, April 14, 1999- JM

Facts: The accused hold the neck, strangle and thereafter bange[d] the head on the concrete pavement
floor of one DEDICACION BALISI Y SORIANO, a 61 years of age, woman, thereby inflicting upon her
fractured bones, serious and mortal wounds which directly caused her death.
Ruling: (a) when, in their presence, the person to be arrested has committed, is actually committing, or
is attempting to commit, an offense; (b) when an offense has just been committed, and they have
probable cause to believe, based on personal knowledge of facts or circumstances, that the person to be
arrested has committed it; and (c) when the person to be arrested is a prisoner who has escaped while
being transferred from one confinement to another, or from a penal establishment where he or she is
serving final judgment or is temporarily confined while the case is pending.
None of these circumstances was present when members of the Criminal Investigation Group (CIG)
arrested appellant. He was not a prisoner. The killing of Dedicacion and John Ardee Balisi was not done
in the presence of the arresting officers. Since it took place on September 2, 1996, it could not have
been considered as “having just been committed.” Evidently, they unlawfully arrested appellant on
September 10, 1996. When they did so, we cannot ascribe to them the presumption of regularity in the
performance of official functions, contrary to the court a quo’s finding.

People v. Cubcubin, GR 136267, October 2, 2001- JM

Facts: At about 3:30 in the morning of August 26, 1997, Sgt. Rogel, desk officer of the Cavite City police
station, received a telephone call that a person had been shot. F o r t h i s r e a s o n , a
p o l i c e t e a m responded to the call and found Henry P. Piamonte slumped dead on his tricycle which
was then parked on the road. A tricycle driver, who refused to divulge his name, told them accused-
appellant and the victim were last seen together coming out of the Sting Cafe, about a kilometer and a
half away from the crime scene. Another tricycle driver, gave a description fitted a person
known as alias "Jun Dulce” and led the policemen where accused-appellant lived. Upon
arriving the police asked permission if they could enter the house. After entering they
found a bloodied shirt and two spent .38 caliber shells. Accused appellant was then asked to return to
the cafe for identification. He was positively identified by the waitress to be the person with whom the
victim drank with. The police then went back to his house and there found a .38paltik pistol. Information
for murder was then filed against accused-appellant. The trial court found him guilty and sentenced him
with the capital punishment

Issue: Validity of the arrest.

Ruling: NO. The arrest of accused-appellant was effected shortly after the victim was killed. The
question, therefore, is whether there was "probable cause" for the arresting officers, to believe that
accused-appellant committed the crime. We hold that there was none. The two did not have "personal
knowledge of facts"indicating that accused-
appellant had committed the crime. Their knowledge of thecircumstances from which they allegedly
inferred that accused-appellant was probably guilty was based entirely on what they had been told by
others.

People v. Gorente, 219 SCRA 756- JM

Facts: Patrolman Jaime Urrutia of the Valenzuela Police Station received a report from the Palo Police
Detachment about a mauling incident. He went to the Valenzuela District Hospital where the victim was
brought. He was informed by the hospital officials that the victim died on arrival. Right away, Patrolman
Urrutia, proceeded to Paseo de Blas where the mauling incident took place. There they found a piece of
wood with blood stains, a hollow block and two roaches of marijuana. They were informed by Reyes,
that she saw the killing and she pointed to Gabriel Gerente as one of the three men who killed Clarito.
The policemen proceeded to the house of the appellant who was then sleeping. They told him to come
out of the house and they introduced themselves as policemen. Patrolman Urrutia frisked appellant and
found a coin purse in his pocket which contained dried leaves wrapped in cigarette foil. Only the
appellant, Gabriel Gerente, was apprehended by the police. The other suspects, Fredo and Totoy
Echigoren, are still at large.

Issue: Validity of the arrest.

Ruling: The policemen arrested Gerente only some three (3) hours after Gerente and his companions
had killed Blace. They saw Blace dead in the hospital and when they inspected the scene of the crime,
they found the instruments of death: a piece of wood and a concrete hollow block which the killers had
used to bludgeon him to death. The eye-witness, Edna Edwina Reyes, reported the happening to the
policemen and pinpointed her neighbor, Gerente, as one of the killers. Under those circumstances, since
the policemen had personal knowledge of the violent death of Blace and of facts indicating that Gerente
and two others had killed him, they could lawfully arrest.

Padilla v. CA, GR 121917, March 12, 1997- JM


Facts: Padilla figured in a hit and run accident in Oct 26, 1992. He was later on apprehended with the help of a civilian witness.
Upon arrest following high powered firearms were found in his possession. His arrest for hit and run incident modified to
include grounds of Illegal Possession of firearms. He had no papers.

Issue:WARRANTLESS ARREST: WON his was illegal and consequently, the firearmsand ammunitions taken in the course
thereof are inadmissible in evidence under the exclusionary rule

Ruling: No. Anent the first defense, petitioner questions the legality of his arrest. There is no dispute that no warrant was
issued for the arrest of petitioner, but that per se did not make his apprehension at the Abacan Bridge illegal. Warrantless
arrests are sanctioned in Sec. 5, Rule 113 of the Revised Rules on Criminal Procedure—a peace officer or a private person
may, without a warrant, arrest a person (a) when in his presence the person to be arrested has committed, is actually
committing, or is attempting to commit an offense. When caught in flagrante delicto with possession of an unlicensed
firearm and ammo, petitioner’s warrantless arrest was proper since he was actually committing another offence in the
presence of all those officers. There was no supervening event or a considerable lapse of time between the hit and run and
the actual apprehension. Because arrest was legal, the pieces of evidence are admissible. Instances when warrantless search
and seizure of property is valid.

People v. Burgos – 144 SCRA 1- JM

Facts: Due to information given by a person, who allegedly was being forcibly recruited by accused to
the NPA, the members of the Constabulary went to the house of accused, asked about his firearm and
documents connected to subversive activities. Accused pointed to where his firearm was as well as his
other documents allegedly.

Ruling: The right of the person to be secure against any unreasonable seizure of his body and any
deprivation of liberty is a most basic and fundamental one. The statute or rule, which allows
exceptions to the requirement of warrants of arrest is strictly construed. Any exception must
clearly fall within the situations when securing a warrant would be absurd or is manifestly
unnecessary as provided by the Rule. We cannot liberally construe the rule on arrests without
warrant or extend its application
beyond the cases specifically provided by law. To do so would infringe upon personal liberty
and set back a basic right so often violated and so deserving of full protection.

People v. Sucro – 195 SCRA 388- JM

Facts: Pat. Fulgencio went to Arlie Regalado’s house at C. Quimpo to monitor activities of Edison SUCRO
(accused). Sucro was reported to be selling marijuana at a chapel 2 meters away from Regalado’s house.
Sucro was monitored to have talked and exchanged things three times. These activities are reported
through radio to P/Lt. Seraspi. A third buyer was transacting with appellant and was reported and later
identified as Ronnie Macabante. From that moment, P/Lt.Seraspi proceeded to the area. While
the police officers were at the Youth Hostel in Maagama St. Fulgencio told Lt. Seraspi to intercept.
Macabante was intercepted at Mabini and Maagama crossing in front of Aklan Medical center.
Macabante saw the police and threw a tea bag of marijuana on the ground. Macabante admitted buying
themarijuana from Sucro in front of the chapel.

The police team intercepted and arrested SUCRO at the corner of C. Quimpo and Veterans. Recovered
were 19 sticks and 4 teabags ofmarijuana from a cart inside the chapel and another teabag from
Macabante.
Issues:

(1) Whether or Not arrest without warrant is lawful.

(2) Whether or Not evidence from such arrest is admissible.

Held: Search and seizures supported by a valid warrant of arrest is not an absolute rule. Rule 126, Sec 12
of Rules of Criminal Procedure provides that a person lawfully arrested may be searched for dangerous
weapons or anything, which may be used as proff of the commission of an offense, without a search
warrant.(People v. Castiller) The failure of the police officers to secure a warrant stems from the fact
that their knowledge required from the surveillance wasinsufficient to fulfill requirements for its
issuance. However, warantless search and seizures are legal as long as PROBABLE CAUSE existed.
The police officers have personal knowledge of the actual commission of the crime from
the surveillance of the activities of the accused. As police officers were the ones conducting
the surveillance, it is presumed that they are regularly in performance of their duties.

People v. Briones – 202 SCRA 708- JM

Facts: Briones, Javier and Allied mauled Gutierrez who was lying prostrate on the ground. The three later dragged Mrs.
Gutierrez inside her house and closed the door. In the morning of April 24, 1988, a neighbor went to the house
of Gutierrez. He found it in disarray, with the spouses sprawled dead on the first floor. He then informed
the Chief of Police of the incident and a team of investigators proceeded to the scene of the crime.
Subsequently, the culprits were apprehended on separate occasions without warrants.

Ruling: The arrest was unlawful originally but it was cured. “It is unequivocally clear that no valid arrest was made on the
accused, the arrest having been made without any warrant at all. Neither can the apellant’s arrest qualify as a lawful arrest
without a warrant under Sec. 5 (b) Rule 113 because the police officer who effected the arrest indubitably had no
personal knowledge of facts indicating that the person to be arrested has committed the crime. It is
eyewitness Francisco who has such personal knowledge. In sum, therefore, the warrantless arrest of the appellant is
illegal. Nevertheless, such unavailing technicality cannot render all the other proceedings, including
the conviction of the accused, void. It cannot deprive the state of its right to convict the guilty when all
the facts on record point to their culpability. In this regard, the case of De Asis v. Romero finds application. Thus, “One of
the most important of these settled rules is that any objection to the procedure followed in the matter
of the acquisition by a court of jurisdiction over the person of the accused must be opportunely raised
before he enters his plea, otherwise the objection is deemed waived.

People v. Sequino – 264 SCRA 79- JM

Facts: SPO Elpidio Luna, Luna went to the crime scene where he found an abandoned motorcycle.
People who by then had milled around the site informed Luna "that the culprit had already fled." Luna
noticed that the "bushes were compressed" and found "a piece of paper utilized as toilet paper with a
stool on it [which] was somewhat newly delivered." The paper was a bio-data sheet with the name "
Melvida, Nenito" and the entry for the father's name filled in with "Elpidio Melvida."After finding Nenito
Melvida,Luna asked Melvida to go with him to the barangay captain's house. Melvida hesitated at first,
but his companions prevailed upon him to go with Luna.The barangay captain was not home, so Luna
took Melvida to the police station instead.

Issue: Was there a valid arrest?

Ruling: Regardless of Luna's claim to the contrary, accused Nenito Melvida was arrested. An arrest "is
the taking of a person into custody in order that he may be bound to answer for the commission of an
offense, “and it is made "by an actual restraint of the person to be arrested, or by his submission to the
custody of the person making the arrest."Melvida's voluntarily going with Luna upon the latter's
"invitation" was a submission to Luna’s custody, and Luna believed that Melvida was a suspect in the
robbery charged herein, hence, Melvida was being held to answer for the commission of the said
offense. Since he was arrested without a warrant, the inquiry must now be whether a valid warrantless
arrest was effected. Rule 113 of the Rules on Criminal Procedure .There was NO valid warrantless arrest
in this case. Luna had no personal knowledge of facts indicating Melvida's guilt; at best, he had an
unreasonable suspicion. Melvida's arrest was thus illegal

People v. Nazareno – 260 SCRA 256- JM

Facts: This was a case for murder in which Ramil Regala, Narciso Nazareno, Orlando Hular and Manuel
Laureaga were arrested. Regala and Nazareno were put in a police line-up. They were identified and
pointed to as the assailants by the tricycle drivers Hernandez and de Limos. Hernandez and de Limos
executed additional sworn affidavits.

Accused-appellants claim that their arrests without warrant were illegal and justify the nullification of
the proceedings of the trial court.

Ruling: The contention is untenable. The warrantless arrest of accused-appellant Narciso Nazareno was
upheld by this Court in 1990 in a petition for habeas corpus. It appears that, on January 9, 1989,
Nazareno filed a motion for bail. As the trial court denied his motion, a petition for habeas corpus was
filed on his behalf with this Court. It was alleged that Nazareno’s arrest was illegal because it was made
without warrant fourteen days after the killing of Romulo Bunye II. This Court dismissed the petition in
its decision of July 9, 1990.He filed a motion for reconsideration which the Court also denied on the
ground that the warrantless arrest was in accordance with Rule 113, 5(b) of the Revised Rules of
Criminal Procedure. The question which Nazareno raises has thus been settled long ago in a final
decision of this Court.

Furthermore, Nazareno and Regala waived objections based on the alleged irregularity of their
arrest, considering that they pleaded not guilty to the charges against them and participated in the
trial. Any defect in their arrest must be deemed cured when they voluntarily submitted to the
jurisdiction of the court. For the legality of an arrest affects only the jurisdiction of the court over the
person of the accused. Consequently, if objections based on this ground are waived, the fact that the
arrest was illegal is not a sufficient cause for setting aside an otherwise valid judgment rendered after a
trial, free from error.The technicality cannot render the subsequent proceedings void and deprive the
State of its right to convict the guilty when all the facts on record point to the culpability of accused.
People v. Mahusay – 282 SCRA 80- JM

Facts: Accused were involved in a robbery. The following day, the incident was reported to the police. A
police team ]from the Integrated National Police Station of San Isidro, Leyte led by Sgt. Manuel Bughao
was promptly dispatched to Sitio Sumakab, Barangay Inangatan, Tabango, Leyte to arrest the suspects.

Ruling: In the case at bar, appellants were arrested on the sole basis of Bughao’s verbal report. The
arresting officers were led to suspect that, indeed, appellants had committed a crime. Thus, the arrest
was made in violation of their fundamental right against an unjustified warrantless arrest. This
notwithstanding, appellants cannot find comfort solely in this error.

The Court has ruled on several occasions that “any objection involving a warrant of arrest or the
procedure in the acquisition by the court of jurisdiction over the person of an accused must be made
before he enters his plea, otherwise the objection is deemed waived. Indeed, it is too late for appellants
to raise the question of their arrest. “When appellants were arrested and a case was filed against them,
they pleaded not guilty upon arraignment, participated in the trial and presented their respective
evidence. Appellants are thus estopped from questioning the legality of their arrest.”At the outset,
appellants should have “moved for the quashal of the information before the trial court on this
ground.”Accordingly, any irregularity in their arrest was cured when they voluntarily submitted
themselves to the jurisdiction of the trial court.

People v. Alvario – 275 SCRA 529 – Kaiser


Facts: respondent was accused of rape by a certain esterlina quintero, she allege that she was
imprisoned in the said residence by the respondent, in order to rescue the victim the police officers in
guise of returning the victim’s wallet proceeded to the said residence then immediately arrested the
respondent without warrant.
Issue: W/N there was a lawful arrest.
Held: Yes. Finally, the Court notes that during the trial, Alvario consistently protested his warrantless
arrest. Suffice it to say that his arrest falls within the purview of Rule 113, Section 5(b) of the 1985 Rules
on Criminal Procedure, viz.:
“Sec. 5. Arrest without warrant; when lawful. - A peace officer or a private person may, without a
warrant, arrest a person:(b) When an offense has in fact just been committed, and he has personal
knowledge of facts indicating that the person to be arrested has committed it; x x x.” (Underscoring
supplied).The personal knowledge of the arresting officers in the case at bar was culled from the
information supplied by the victim herself who pointed to Alvario as the man who raped her at the time
of his arrest

Larranaga v. CA – 287 SCRA 521- Kaiser


Facts: Petitioner was charged with kidnapping with serious illegal detention that was allegedly
committed 4 months ago, he was arrested by the PNP CIG without the benefit of a warrant of arrest.
Respondents claim that kidnapping with serious illegal detention is a continuing crime hence the suspect
may be arrested.

Issue: W/N he can be arrested without a warrant.


Held: No. It does not appear in the case at bar that petitioner has just committed, is actually committing
or is attempting to commit an offense when the police officers tried to arrest him on September 15,
1997. In fact, petitioner was attending classes at the Center for Culinary Arts at that time.

We reject the prosecutors' argument that petitioner was actually committing a crime at the time of the
arrest since kidnapping with serious illegal detention is a continuing crime. In the case of Parulan v.
Director of Prisons cited by the prosecutors, kidnapping with illegal detention is considered a continuing
crime where the deprivation of liberty is persistent and continuing from one place to another. The facts
show that the alleged kidnapping was committed on July 16, 1997. One of the victims, Marijoy Chiong,
was found dead in Sitio Tanawan, Barangay Guadalupe, Carcar, Cebu on July 18, 1997, while the other
victim, Jacqueline Chiong, remains missing to date. There is no showing that at the time of the arrest on
September 15, 1997, Jacqueline Chiong was being detained by petitioner who was then residing in
Quezon City. Hence, petitioner may not be considered as continually committing the crime of
kidnapping with serious illegal detention at the time of the arrest.

People v. Olivarez – GR 77865, Dec. 4, 1998


Facts: two days before the respondents were arrested a crime of robbery with homicide occurred where
one of the private respondents was working, since he no longer reported for work the police eventually
questioned him where he finally admitted of doing the crime absent the presence of counsel.

Issue: W/N a warrantless arrest may be made two days after the commission of the crime.

Held: No. At the time appellants were apprehended, two days had already lapsed after the discovery of
the crime — they were not doing nor had just done any criminal act. Neither were they caught in
flagrante delicto or had escaped from confinement. Probably aware of the illegality of the arrest they
made the arresting officers testified that appellants were merely invited to the police precinct. Such
invitation, however when construed in the light of the circumstances is actually in the nature of an
arrest designed for the purpose of conducting an interrogation. 16 Mere invitation is covered by the
proscription on a warrantless arrest because it is intended for no other reason than to conduct an
investigation. Thus, pursuant to Section 4(2), Article IV of the 1973 Constitution which was in effect at
that time, "any evidence" obtained in violation of their right under Section 3, Article IV (pertaining to
invalid warrantless arrests) 17 "shall be inadmissible for any purpose in any

Cadua v. CA – 312 SCRA 703


Facts: Petitioners were arrested by responding police officers who were to investigate a robbery
incident that has just occurred. Together with the arresting officers were the allege victims who was
able to pinpoint the petitioners while they were walking.
Issue: W/N the suspects were validly arrested.
Held: Yes. It has been ruled that personal knowledge of facts, in arrests without warrant must be based
upon probable cause, which means an actual belief or reasonable grounds of suspicion. . . . Peace
officers may pursue and arrest without warrant any person found in suspicious places or under
suspicious circumstances reasonably tending to show that such person has committed, or is about to
commit, any crime or breach of the peace. Probable cause for an arrest without warrant is such a
reasonable ground of suspicion supported by circumstances sufficiently strong in themselves as to
warrant a reasonable man in believing the accused to be guilty. Besides reasonable ground of suspicion,
action in good faith is another protective bulwark for the officer.
Petitioner could not dispute that there was an initial report to the police concerning the robbery. A radio
dispatch was then given to the arresting officers, who proceeded to Alden Street to verify the
authenticity of the radio message. When they reached said place, they met up with the complainants
who initiated the report about the robbery. Upon the officers invitation, both mother and daughter
boarded the mobile unit to join them in conducting a search of the nearby area. The accused was
spotted in the vicinity. Based on the reported statements of complainants, he was identified as a logical
suspect in the offense just committed.

People v. Cubcubin – 360 SCRA –


Facts: respondent was arrested inside his home for allegedly shooting and killing a certain HENRY PECHO
PIAMONTE. The police officers arrested him without warrant and now raises the defense that it was a
lawful hot pursuit arrest as the said crime occurred only hours before his arrest.
Issue: W/N there was a valid warrantless arrest.
Held: No. two conditions must concur for a warrantless arrest to be valid: first, the offender has just
committed an offense and, second, the arresting peace officer or private person has personal
knowledge of facts indicating that the person to be arrested has committed it. It has been held that
“‘personal knowledge of facts’ in arrests without a warrant must be based upon probable cause, which
means an actual belief or reasonable grounds of suspicion.”
In this case, the arrest of accused-appellant was effected shortly after the victim was killed. The
question, therefore, is whether there was “probable cause” for PO3 Rosal and SPO1 Malinao, Jr., the
arresting officers, to believe that accused-appellant committed the crime. We hold that there was none.
The two did not have “personal knowledge of facts” indicating that accused-appellant had committed
the crime. Their knowledge of the circumstances from which they allegedly inferred that accused-
appellant was probably guilty was based entirely on what they had been told by others, to wit: by
someone who called the PNP station in San Antonio, Cavite City at about 3:30 in the morning of August
26, 1997 and reported that a man had been killed along Julian Felipe Boulevard of the said city; by an
alleged witness who saw accused-appellant and the victim coming out of the Sting Cafe; by Danet
Garcellano, waitress at the Sting Cafe, who said that the man last seen with the victim was lean,
mustachioed, dark-complexioned and was wearing a white t-shirt and a pair of brown short pants; by a
tricycle driver named Armando Plata who told them that the physical description given by Garcellano
fitted accused-appellant, alias “Jun Dulce” and who said he knew where accused-appellant lived and
accompanied them to accused-appellant’s house. Thus, PO3 Rosal and SPO1 Malinao, Jr. merely relied
on information given to them by others.

People v. Compacion – 361 SCRA 540


Facts: the police NARCOM using search warrant from a judge of another city who does not have
jurisdiction on that area searched the residence of the respondent, they eventually found two marijuana
plants in his backyard.
Issue: W/N there was a valid search and seizure.
Held: no. In the instant case, the search and seizure conducted by the composite team in the house of
accused-appellant was not authorized by a search warrant. It does not appear either that the situation
falls under any of the above mentioned cases. Consequently, accused-appellants right against
unreasonable search and seizure was clearly violated. While the right to be secure from unreasonable
search and seizure may, like every right, be waived either expressly or impliedly, 27 such waiver must
constitute a valid waiver made voluntarily, knowingly and intelligently. The act of the accused-appellant
in allowing the members of the military to enter his premises and his consequent silence during the
unreasonable search and seizure could not be construed as voluntary submission or an implied
acquiescence to warrantless search and seizure especially so when members of the raiding team were
intimidatingly numerous and heavily armed. His implied acquiescence, if any, could not have been more
than mere passive conformity given under coercive or intimidating circumstances and is, thus,
considered no consent at all within the purview of the constitutional guarantee. Consequently, herein
accused-appellants lack of objection to the search and seizure is not tantamount to a waiver of his
constitutional right or a voluntary submission to the warrantless search and seizure.

Posadas v. Ombudsman – 341 SCRA-


Facts: Dennis Venturina, a member of Sigma Rho at the University of the Philippines, was killed in a
rumble between his fraternity and another fraternity on December 8, 1994. Petitioner Posadas, then
Chancellor of U.P. Diliman, asked the Director of the NBI for assistance in determining the persons
responsible for the crime. With that, respondent Dizon, Chief of the Special Operations Group of the
NBI, and his men went to U.P. and, on the basis of the supposed positive identification of two alleged
eyewitnesses, they attempted to arrest Francis Carlo Taparan and Raymundo Narag, officers/members
of the Scintilla Juris Fraternity, as suspects in the killing of Venturina. It appears that the two suspects
had come that day to the U.P. Police Station for a peace talk between their fraternity and the Sigma Rho
Fraternity.
Issue: W/N the NBI can arrest the suspects without warrant
Held: No. The arresting officers in this case did not witness the crime being committed. Neither are the
students fugitives from justice nor prisoners who had escaped from confinement. The question is
whether paragraph (b) applies because it is the contention of the respondents that the NBI agents had
personal knowledge of facts gathered by them in the course of their investigation indicating that the
students sought to be arrested were the perpetrators of the crime. But the Court ruled in negative.
"Personal knowledge" of facts in arrests without a warrant under Section 5 (b) of Rule 113 must be
based upon "probable cause "which means an actual belief or reasonable grounds of suspicion."
The grounds of suspicion are reasonable when, in the absence of actual belief of the arresting officers,
the suspicion that the person to be arrested is probably guilty of committing the offense is based on
actual facts.

At the time Dennis Venturina was killed, these agents were nowhere near the scene of the crime. When
respondent Dizon and his men attempted to arrest Taparan and Narag, the latter were not committing a
crime nor were they doing anything that would create the suspicion that they were doing anything
illegal

People v. Acol – 232 SCRA 406


FACTS: Holdup in passenger jeepney; driver reported to police. Two passengers who were
apprehended after they supposedly staged a hold-up inside a passenger jeepney on September 29,
1990 were haled to court, not for the felonious asportation, but for possession of the two unlicensed
firearms and bullets recovered from them which were instrumental in the commission of the robo

ISSUE: W/N the arrest was valid

HELD: With respect to the so-called warrantless arrest of accused--appellant, we are of the view that
the search falls within the purview of Section 5(b) of Rule 113 which serves as an exception to the
requisite warrant prior to arrest: When an offense has just in fact been committed, and the has
personal knowledge of facts indicating that the person to be arrested has committed it; inasmuch as
the police team was formed and dispatched to look for the persons responsible for the crime on
account of the information related by Percival Tan and Rene Araneta that they had just been robbed.
And since accused-appellant's arrest was lawful, it follows that the search made incidental thereto
was valid.
C. Escaped Prisoner

D. Waiver

E. Procedural Rules

People v. Rabang – 187 SCRA 682

Facts: Respondent was arrested based on positive identification by a witness in a crime of murder,
petitioner now questions of the legality of his arrest, and that he was not accorded of his rights when he
was arrested.

Issue: W/N may still raise the issue of legality of his arrest.

Held: No. Appellant assails the legality of his arrest by the police investigators allegedly for the reason
that it "was based solely on the basis of the tip and say-so from a telephone informant ... refused to
identify himself.

The Court considers that appellant is estopped from questioning the legality of his arrest. An
examination of the record reveals that this issue is being raised for the first time by appellant before this
Court. He had not moved for the quashing of the information before the trial court on this ground. Thus,
any irregularity attendant to his arrest was cured when he voluntarily submitted himself to the
jurisdiction of the trial court by entering a plea of not guilty "and [by] participating in the trial.
Finally, the Court considers that appellant's claim that he was arrested and exposed to accusatorial
proceedings without being informed of his constitutional rights, has not been substantiated. The
booking and information sheet and progress report marked as Exhibits "G" and "I" respectively, which
were prepared by the police investigators explicitly stated that appellant had been properly informed of
his constitutional rights. In fact, after he was informed of his constitutional rights, appellant chose to
remain silent and requested the assistance of counsel. It has not been asserted that appellant gave a
confession, judicial or extrajudicial, oral or written.
People v. Lopez – 246 SCRA 95

Velasco v. CA – 245 SCRA 677


Facts: Private respondent was detained for a case of BP22 but after he paid bail he was not released by
the NBI as another complaint pending to be filed against him for rape was being prepared. The common
law wife in behalf of the respondent filed for a writ of habeas corpus to release him.
Issue: W/N the detention and arrest of the accused was valid
Held: Yes. Even if the arrest of a person is illegal, supervening events may bar his release or discharge
from custody. What is to be inquired into is the legality of his detention as of, at the earliest, the filing of
the application for a writ of habeas corpus, for even if the detention is at its inception illegal, it may, by
reason of some supervening events, such as the instances mentioned in Section 4 of Rule 102, be no
longer illegal at the time of the filing of the application. Among such supervening events is the issuance
of a judicial process preventing the discharge of the detained person. Another is the filing of a complaint
or information for the offense for which the accused is detained, as in the instant case. By then, the
restraint of liberty is already by virtue of the complaint or information and, therefore, the writ of habeas
corpus is no longer available. Section 4 of Rule 102 reads in part as follows: "Nor shall anything in this
rule be held to authorize the discharge of a person charged with . . . an offense in the Philippines."
People v. Buluran – 325 SCRA 476
G.R. No. 113940. February 15, 2000
On May 20, 1993, appellant Cielito Buluran and three (3) John Does were charged with the crime of
murder. The Information was later amendedwhen Leonardo Valenzuela was identified as one of the
assailants. Upon arraignment, both accused entered pleas of not guilty. On February 4, 1994, the trial
court, finding conspiracy and treachery, rendered judgment convicting appellants of murder.
HELD:First. Appellants are estopped from questioning the validity of their respective arrests since they
never raised this issue before arraignment. Any objection involving a warrant of arrest or the acquisition
of jurisdiction over the person of an accused must be made before he enters his plea, otherwise the
objection is deemed waived. Second. There is no violation of the constitutional rights of the accused
during custodial investigation since neither one executed an extrajudicial confession or admission. In
this case, the basis of the conviction by the trial court was the testimonies of the three eyewitnesses,
Artemio Avendaño, Jacinto Castillo, and Gloria Castillo.

Third. The failure to accord appellants their right to preliminary investigation did not impair the validity
of the information nor affect the jurisdiction of the trial court. While the right to preliminary
investigation is a substantive right and not a mere formal or technical right of the accused, nevertheless,
the right to preliminary investigation is deemed waived when the accused fails to invoke it before or at
the time of entering a plea at arraignment.

Section 3. (1) The privacy of communication and correspondence shall be inviolable except upon
lawful order of the court, or when public safety or order requires otherwise as prescribed by law.
(2) Any evidence obtained in violation of this or the preceding section shall be inadmissible for any
purpose in any proceeding.

Scope: Tangible and Intangible Objects.

Katz v. United States, 389 U.S. 437 (1967) - Anton


FACTS:
In order to collect evidence against Katz, federal agents placed a warrantless wiretap on the public
phone booth that he used to conduct these operations. The agents listened only to Katz's conversations,
and only to the parts of his conversations dealing with illegal gambling transactions.

ISSUE: Are intangible objects such as the conversation illegally obtained in the case at bar covered by the
protection against unreasonable searches and seizures?

RULING:
YES. The US Supreme Court held that the act of FBI agents in electronically recording a conversation
made by petitioner in an enclosed public telephone booth violated his right to privacy and constituted a
“search and seizure”. Because the petitioner had a reasonable expectation of privacy in using the
enclosed booth to make a personal telephone call, the protection of the Fourth Amendment extends to
such area. In the concurring opinion of Mr. Justice Harlan, it was further noted that the existence of
privacy right under prior decisions involved a two-fold requirement: first, that a person has exhibited an
actual (subjective) expectation of privacy; and second, that the expectation be one that society is
prepared to recognize as reasonable (objective).

Factors to Determine Violation of the Right to Privacy


In the matter of the Petition for Issuance of the Writ of Habeas Corpus of Camilo I. Sabio, GR 174340,
October 17, 2006 -Anton
FACTS:
Petitioners were invited in the Senate's public hearing to deliberate on Senate Res. No. 455, particularly
"on the anomalous losses incurred by the Philippine Overseas Telecommunications Corporation (POTC),
Philippine Communications Satellite Corporation (PHILCOMSAT), and Philcomsat Holdings Corporations
(PHC) due to the alleged improprieties in the operations by their respective board of directors."
Obviously, the inquiry focus on petitioners' acts committed in the discharge of their duties as officers
and directors of the said corporations, particularly Philcomsat Holdings Corporation. They now contend
that there is a violation of their right to privacy.

ISSUE: Whether or not there is a violation to petitioner’s right to privacy.

RULING:
NO. In evaluating a claim for violation of the right to privacy, a court must determine whether a person
has exhibited a reasonable expectation of privacy and, if so, whether that expectation has been violated
by unreasonable government intrusion. In this case, petitioners have no reasonable expectation of
privacy over matters involving their offices in a corporation where the government has interest.
Certainly, such matters are of public concern and over which the people have the right to information.

Briccio Pollo v. Chairperson Karina David, GR 181881, October 18, 2011 – Anton
FACTS:
This case involves a search of office computer assigned to petitioner, employee of the CSC, who was
then charged administratively and was eventually dismissed from the service. The employee’s personal
files stored in the computer were used by the government employer as evidence of his misconduct.
Allegedly, has been lawyering for public officials with pending cases in the CSC. Petitioner contends that
there is violation of privacy.

ISSUE: Does petitioner have reasonable expectation of privacy so as to warrant a violation of the said
privacy?

RULING:
NO. Petitioner failed to prove that he had an actual (subjective) expectation of privacy either in his office
or government-issued computer which contained his personal files. Petitioner did not allege that he had
a separate enclosed office which he did not share with anyone, or that his office was always locked and
not open to other employees or visitors. Neither did he allege that he used passwords or adopted any
means to prevent other employees from accessing his computer files.

Moreover, even assuming arguendo, that petitioner had at least a subjective expectation of privacy in
his computer as he claims, such is negated by the presence of policy regulating the use of office
computers [CSC Office Memorandum No. 10, S. 2002 “Computer Use Policy (CUP)”], as in Simons. The
CSC in this case had implemented a policy that put its employees on notice that they have no
expectation of privacy in anything they create, store, send or receive on the office computers, and that
the CSC may monitor the use of the computer resources using both automated or human means. This
implies that on-the-spot inspections may be done to ensure that the computer resources were used only
for such legitimate business purposes.

Anonymous Letter-Complaint against Atty. Miguel Morales, Clerk of Court, Metropolitan Trial Court of
Manila, A.M. Nos. P-08-2519 and P-08-2520, November 19, 2008, 571 SCRA 361. - Anton
FACTS:
The two anonymous letters charge Atty. Morales with the following offenses: attending to personal
cases while using official time, office supplies, equipment and utilities, leaving the office after logging-in
in the morning only to return in the afternoon, and playing computer games whenever he was at the
office. Pursuant thereto, a spot investigation was conducted and the hard drive of Atty. Morales’s
computer yielded a pre-trial brief and a petition for relief from judgment with the name of a certain
Atty. Icaonapo.

ISSUE: Are the pleadings found in Atty. Morales's personal computer admissible in the present
administrative case against him?

RULING:
NO. There was no valid consent obtained from Atty. Morales. Acquiescence in the loss of fundamental
rights is not to be presumed and courts indulge every reasonable presumption against waiver of
fundamental constitutional rights.[52] To constitute a valid consent or waiver of the constitutional
guarantee against obtrusive searches, it must be shown that (1) the right exists; (2) that the person
involved had knowledge, either actual or constructive, of the existence of such right; and (3) the said
person had an actual intention to relinquish the right.

In this case, what is missing is a showing that Atty. Morales had an actual intention to relinquish his
right. While he may have agreed to the opening of his personal computer and the printing of
files therefrom, it is also of record that Atty. Morales immediately filed an administrative case against
said persons questioning the validity of the investigation, specifically invoking his constitutional right
against unreasonable search and seizure.

Synhumliong v. Rivera, GR 200841, June 4, 2014 - Anton

RA No. 4200, Anti-Wiretapping Law

Ramirez v. CA, 248 SCRA 590: “Private communication” in Section 1 of RA 4200 is deemed to include
“private conversations.” – Czar
F: A civil case damages was filed by petitioner Socorro Ramirez in the Quezon City RTC alleging that the
private respondent, Ester Garcia, in a confrontation in the latter’s office, allegedly vexed, insulted and
humiliated her in a “hostile and furious mood” and in a manner offensive to petitioner’s dignity and
personality,” contrary to morals, good customs and public policy.” In support of her claim, petitioner
produced a verbatim transcript of the event and sought damages. The transcript on which the civil case
was based was culled from a tape recording of the confrontation made by petitioner.

As a result of petitioner’s recording of the event and alleging that the said act of secretly taping the
confrontation was illegal, private respondent filed a criminal case before the Pasay RTC for violation of
Republic Act 4200, entitled “An Act to prohibit and penalize wire tapping and other related violations of
private communication, and other purposes.”
Petitioner filed a Motion to Quash the Information, which the RTC later on granted, on the ground that
the facts charged do not constitute an offense, particularly a violation of R.A. 4200.

The CA declared the RTC’s decision null and void and denied the petitioner’s MR, hence the instant
petition.

I: Whether or not a person privy to the private communication may be qualified as a violator under RA
4200?

H: Sec. 1 of RA 4200 clearly and unequivocally make it illegal for any person, not authorized by all the
parties to any private communication to secretly record such communication by means of a tape
recorder. The law makes no distinction as to whether the party sought to be penalized by the statute
ought to be a party other than or different from those involved in the private communication. The
statutes intent to penalize all persons unauthorized to make such recording is underscored by the use of
qualifier “any.” Consequently, as respondent Court of Appeals correctly concluded, even a person privy
to a communication who records his own private converswation with another without the knowledge of
the latter will qualify as a violator under this provision of RA 4200.

Navarro v. CA, GR 121087, August 26, 1999: The Anti-Wiretapping Law prohibits the overhearing,
intercepting, or recording of private communications. Thus, a tape recording of an altercation or
verbal exchange between a policeman and a radio reporter at a police station is admissible in
evidence. - Czar
F: Two local media men, Stanley Jalbuena, Enrique Lingan, in Lucena City wnet to the police station to
report alledged indecent show in one of the night establishment shows in the City. At the station, a
heated confrontation followed between victim Lingan and accused policeman Navarro who was then
having drinks outside the headquarters, lead to a fisticuffs. The victim was hit with the handle of the
accused's gun below the left eyebrow, followed by a fist blow, resulted the victim to fell and died under
treatment. The exchange of words was recorded on tape, specifically the frantic exclamations made by
Navarro after the altercation that it was the victim who provoked the fight. During the trial, Jalbuena,
the other media man , testified. Presented in evidence to confirm his testimony was a voice recording he
had made of the heated discussion at the police station between the accused police officer Navarro and
the deceased, Lingan, which was taken without the knowledge of the two.

I: Whether or not the voice recording is admissible in evidence in view of RA 4200, which prohibits wire
tapping.

H: he answer is affirmative, the tape is admissible in view of RA 4200, which prohibits wire tapping.
Jalbuena's testimony is confirmed by the voice recording he had made.

The law prohibits the overhearing, intercepting, or recording of private communications (Ramirez v
Cpourt of Appeals, 248 SCRA 590 [1995]). Snce the exchange between petitioner Navarro and Lingan
was not private, its tape recording is not prohibited.

Rule on the Writ of Habeas Data, AM 08-1-16-SC


Writ of Habeas Data: the remedy available to any person whose right to privacy in life, liberty or
security is violated or threatened by an unlawful act or omission of a public official or employee, or of
a private individual or entity engaged in the gathering, collecting, or storing of data or information
regarding the person, family, home and correspondence of the aggrieved party.

Not Covered

Alejano v. Cabuay, 468 SCRA 188 – Czar


F: An information for coup d’ etat was filed against the soldiers involved in the Oakwood incident. Gen.
Abaya issued a directive to all Major Service Commanders to take into custody the military personnel
under their command who took part in the said incident except those detained who were to remain
under the custody of ISAFP. Petitioners filed a petition for habeas corpus with the Supreme Court. The
Supreme Court issued a writ of habeas corpus directing respondents to make a return of the writ and to
make a return of the writ and to appear and produce the persons of the detainees before the Court of
Appeals on the scheduled hearing. On the same date, the detainees and co-accused filed a motion for
preliminary investigation, which the trial court granted. The Court of Appeals rendered its decision
dismissing the petition. Nonetheless, the appellate court ordered Gen. Cabuay, who was in charge of
implementing the regulations in the ISAFP Detention Center, to uphold faithfully the rights of the
detainees in accordance with Standing Operations Procedure No. 0263-04. The appellate court directed
Gen. Cabuay to adhere to his commitment made in court regarding visiting hours and the detainees’
right to exercise for two hours a day.

I: Whether or not the writ of habeas corpus is the appropriate remedy to be sought by the parties?

H: In a habeas corpus petition, the order to present an individual before the court is a preliminary step
in the hearing of the petition as the respondent must produce the person and eplain the cause of his
detention but such order is not a ruling on the propriety of the remedy or on the substantive matters
covered by the remedy.

The remedy of habeas corpus is not proper remedy to address the detainees’ complaint against the
regulations and conditions in the ISAFP Detention Center – the purpose is to determine whether a
person is being illegally deprived of his liberty.

In Re: Wenceslao Laureta, 148 SCRA 382 – Czar


F: In almost identical letters, personally sent to Justices Narvas, Herrera, Cruz, and Feliciano, all
members of the First Division of the Supreme Court, in a stance of dangling threats to effect a change of
the change of the Courts adverse resolutions. Eva Maravilla, thru Atty. Enceslao laureta wrote in part
that “we are pursuing further remedies in our quest for justice under the law. We intend to hold
responsible members of the First Division who participated in the promulgation of these threeminutre
resolutions in questions. For the members thereof cannot claim immunity when their action runs afoul
with penal sanctions, even in the performance of official functions; like others, none of the division
members are above the law.” True to her threats, Maravilla filed an Affidavit-Complaint before the
Tanodbayan, charging some Members of Supreme Court with having knowingly and deliberately
rendered, with bad faith, an unjust extended Minute Resolution. Moreover, Atty. Laureta reportedly
circulated copies of the complaint to the press, which was widerly publicized in almost all dalies, without
any copy furnished the Supreme Court not the members who were charged. The issue of the Daily
Express were published with a banner headline reading: ORDONEZ, 8 JUSTICES FACE GRAFT CHARGES”
thereby making it unjustly appear that the Justices of the Supreme Court were charged with graft and
corruption when the complaint was actually filed by a disgruntled litigant and her counsel after losing
her case thrice in the Supreme Court. The Tanodbayan dimissed Maravilla’s complaint and ordered the
latter and her counsel to show cause on why they should not be held in contempt and that her counsel
should not be held administratively liable.

I: Whether the letters addressed to the Supreme Court justices are matters which are covered by the
constitutional right to privacy of communication?

H: Respondent’s reliance on the privacy of communication is misplaced. Letters addressed to individual


Justices, in connection with the performance of their judicial functions become part of the judicial
record and are a matter of concern for the entire Court. The contumacious character of those letters
constrained the First Division to refer the same to the Court enbanc, en consulta and so that the Court
en banc could pass upon the judicial acts of the Division. It was only in the exercise of forbearance by
the Court that it refrained from issuing immediately a show cause order in the expectancy that after
having read the Resolution of the Court en banc, the respondents would realize the unjustness and
unfairness of their accusations.

DOCTRINE: A letter individually addressed to some Justices of the SC is not covered by the
constitutional right to privacy of communication when the same pertain to their exercise of judicial
functions.

People v. Albofera, 152 SCRA 123 - Czar


F: Albofera et.al killed Carancio, a forester who was making a list of the people who are engaged in
kaingin. Albofero warned everyone not to divulge about the killing of Carancio. The following day,
Sisneros was at his farm when Lawi-an and another passed by and called him. When Sisnero got near,
Lawi-an told him that the forester was already killed and warned him not to reveal this matter to
anybody otherwise he would be killed. The threat to his life cause Sisneros to be cautious in not
reporting at once the matter to the authorities. However, Sisneros finally reported the killing of the
forester to his brother Margarito, a CDHF member. Sisneros asked that his identity be kept in the
meantine pending the arrest of Albofera and Lawian. The police authorities arrested Albofera and
Lawian. Shortly after their arrest, they led the police authorities to the place where they buried the slain
forester. Albofera executed an extra-judicial confession stating that he was forced to join the NPA
movement for fear of his life; that the said group had ordered the arrest of Carancio which sentenced
the latter to die by stabbing. In the course of trial, the prosecution presented a letter written in Visayan
dialect dy Albofera while under detention to witness Esma several days before the latter testified. After
trial, the Regional Trial Court, found the circumstantial evidence sufficient to warrant conviction beyond
reasonable doubt of both Albofera and Lawi-an for murder, sentenced them to death, and ordered them
to indemnify the heirs of the victim in the amount of P35,000.00 "by way of moral as well as actual
damages" in its Decision. Hence, the mandatory review.

I: Whether the Albofera’s letter to Esma should be excluded as evidence in light of alleged unwarranted
intrusion or invasion of the accused’s privacy.

H: Section 4, Article IV of the 1973 Constitution (substantially reproduced in Section 3, Article III of the
1987 Constitution) implements another Constitutional provision on the security of a citizen against
unreasonable search and seizure. The production of that letter by the prosecution was not the result of
an unlawful search and seizure nor was it through unwarranted intrusion or invasion into Albofera's
privacy. Albofera admitted having sent the letter and it was its recipient, Rodrigo Esma himself, who
produced and identified the same in the course of his testimony in Court. Besides, there is nothing really
self-incriminatory in the letter. Albofera mainly pleaded that Esma change his declaration in his Affidavit
and testify in his (Albofera's) favor. Furthermore, nothing Albofera stated in his letter is being taken
against him in arriving at a
determination of his culpability.

Exclusionary Rule

Gaanan v. IAC – 145 SCRA 112 – Czar

F: Complainant Atty. Pintor and Montebon offered to withdraw the complaint for direct assault they
filed against Atty. Laconico after demanding P8,000 from him. This demand was heard by Atty. Gaanan
through a telephone extension as requested by Laconico so as to personally hear the proposed
conditions for the settlement. Atty. Pintor was subsequently arrested in an entrapment operation upon
receipt of the money.

Since Atty. Gaanan listened to the telephone conversation without complainant's consent, complainant
charged Gaanan and Laconico with violation of the Anti- Wiretapping Act (RA 4200).

I: W/N an extension telephone is among the prohibited devices in Sec. 1 of RA 4200 such that its use to
overhear a private conversation would constitute an unlawful interception of communication between 2
parties using a telephone line.

H: NO. An extension telephone cannot be placed in the same category as a dictaphone, dictagraph, or
other devices enumerated in Sec. 1 of the law as the use thereof cannot be considered as "tapping" the
wire or cable of a telephone line. This section refers to instruments whose installation or presence
cannot be presumed by the party or parties being overheard because, by their very nature, they are
not of common usage and their purpose is precisely for tapping, intercepting, or recording a
telephone.

Salcedo-Ortanez v. CA – 235 SCRA 111 – Mikee

Facts: Respondent Ortanez filed a complaint for annulment of marriage with damages against
Petitioner Salcedo-Ortanez on grounds of lack of marriage license and/or psychological incapacity.
Among the exhibits offered by respondent were 3 cassette tapes of alleged telephone conversations
between petitioner and unidentified persons. A petition was then filed by petitioner assailing the
admission of evidence of the said cassette tapes. The Court of Appeals held that tape recordings are
not inadmissible per se, depending on how they are presented and offered and on how the trial judge
utilizes them in the interest of truth and fairness and the even handed administration of justice.

Issue: Whether said tapes are admissible as evidence in the case at bar.

Held: NO. They are inadmissible. In the present case, the trial court issued the assailed order admitting
all of the evidence offered by private respondent, including tape recordings of telephone
conversations of petitioner with unidentified persons. These tape recordings were made and obtained
when private respondent allowed his friends from the military to wire tap his home telephone.
Clearly, respondents trial court and Court of Appeals failed to consider the afore-quoted provisions of
the law in admitting in evidence the cassette tapes in question. Absent a clear showing that both
parties to the telephone conversations allowed the recording of the same, the inadmissibility of the
subject tapes is mandatory under Rep. Act No. 4200. WHEREFORE, the subject cassette tapes are
declared inadmissible in evidence.

Zulueta v. CA – 253 SCRA 699 – Mikee

Facts: Petitioner Zulueta is the wife of respondent Martin. Petitioner entered the clinic of her husband,
a doctor of medicine, and in the presence of her mother, a driver and respondent’s secretary forcibly
opened the drawers and cabinets and took 157 documents consisting of private correspondence
between Martin and his alleged paramours, greeting cards, cancelled checks, diaries, passport and
photographs. These were sieved to be used as evidence in a case for legal separation and for the
disqualification from practice of medicine which petitioner filed against her husband.

Issue: Whether the evidence are admissible

Held: No. Indeed the documents and papers in question are inadmissible in evidence. The
constitutional injunction declaring "the privacy of communication and correspondence [to be]
inviolable" is no less applicable simply because it is the wife (who thinks herself aggrieved by her
husband's infidelity) who is the party against whom the constitutional provision is to be enforced. The
only exception to the prohibition in the Constitution is if there is a "lawful order [from a] court or
when public safety or order requires otherwise, as prescribed by law." Any violation of this provision
renders the evidence obtained inadmissible "for any purpose in any proceeding."

The intimacies between husband and wife do not justify any one of them in breaking the drawers and
cabinets of the other and in ransacking them for any telltale evidence of marital infidelity. A person, by
contracting marriage, does not shed his/her integrity or his right to privacy as an individual and the
constitutional protection is ever available to him or to her.

The law insures absolute freedom of communication between the spouses by making it privileged.
Neither husband nor wife may testify for or against the other without the consent of the affected
spouse while the marriage subsists. Neither may be examined without the consent of the other as to
any communication received in confidence by one from the other during the marriage, save for specified
exceptions. But one thing is freedom of communication; quite another is a compulsion for each one to
share what one knows with the other. And this has nothing to do with the duty of fidelity that each
owes to the other.

Ople v. Torres – 293 SCRA 141 – Mikee

President FIDEL V. RAMOS issued Administrative Order No. 308 entitled “ADOPTION OF A NATIONAL
COMPUTERIZED IDENTIFICATION REFERENCE SYSTEM”.

The AO seeks to have all Filipino citizens and foreign residents to have a Population Reference Number
(PRN) generated by the National Statistics Office (NSO) through the use of BIOMETRICS TECHNOLOGY .

The AO was questioned by Senator Ople on ground that the AO violates the citizen’s right to privacy
protected by the Bill of Rights of the Constitution.
Ruling: yes, it does.The AO likewise violates the right to privacy since its main purpose is to provide a
“common reference number to establish a linkage among concerned agencies through the use of
BIOMETRICS TECHNOLOGY. Biometry is the science of the application of statistical methods to
biological facts; a mathematical analysis of a biological data. It is the confirmation of an individual’s
identity through a fingerprint, retinal scan, hand geometry or facial features. Through the PRN, the
government offices has the chance of building a huge and formidable information base through the
electronic linkage of the files of every citizen. The data, however, may be gathered for gainful and useful
government purposes; but the existence of this vast reservoir of personal information constitutes a
covert invitation to misuse, a temptation that may be too great for some of our authorities to resist.

Further, the AO does not even tells us in clear and unequivocal terms how these informations gathered
shall be handled. It does not provide who shall control and access the data and under what
circumstances and for what purpose. These factors are essential to safeguard the privacy and guaranty
the integrity of the information. The computer linkage gives other government agencies access to the
information. YET, THERE ARE NO CONTROLS TO GUARD AGAINST LEAKAGE OF INFORMATIONS. WHEN
THE ACCESS CODE OF THE CONTROL PROGRAMS OF THE PARTICULAR COMPUTER SYSTEM IS BROKEN,
AN INTRUDER, WITHOUT FEAR OF SANCTION OR PENALTY, CAN MAKE USE OF THE DATA FOR
WHATEVER PURPOSE, OR WORSE, MANIPULATE THE DATA STORED WITHIN THE SYSTEM.

AO No. 308 is unconstitutional since it falls short of assuring that personal information gathered about
our people will be used only for specified purposes thereby violating the citizen’s right to privacy.

Waterous Drug Corp v. NLRC, GR 113271, October 16, 1997 – Mikee

Facts: Antonia Melodia Catolico was hired as a pharmacist by Waterous Drug Corp.

Catolico sold to YSP Inc. 10 bottles of Voren Tablets at P384 per unit. However, the normal selling
price is P320 per unit. Catolico overcharged by P64 per unit for a total of P640. YSP sent a check
payable to Catolico as a “refund” for the jacked-up price. It was sent in an envelope addressed to her.
Saldana, the clerk of Waterous Drug Corp. opened the envelope and saw that there was a check for
P640 for Catolico.

Waterous Drug Corp. ordered the termination of Catolico for acts of dishonesty.

NLRC: Dismissed the Petition. Evidence of respondents (check from YSP) being rendered inadmissible,
by virtue of the constitutional right invoked by complainants.

Petitioners contend that: In the light of the decision in the People v. Marti, the constitutional protection
against unreasonable searches and seizures refers to the immunity of one’s person from interference by
government and cannot be extended to acts committed by private individuals so as to bring it within the
ambit of alleged unlawful intrusion by the government.

Issue: W/N the check is admissible as evidence

Held: Yes. Ratio: (People vs. Marti) Marti ruling: The Bill of Rights does not protect citizens from
unreasonable searches and seizures perpetrated by private individuals.

People v. Marti – 193 SCRA 57 – Mikee


Facts: On August 14, 1987, the appellant and his common-law wife, Shirley Reyes went to Manila
Packaging and Export Forwarders to send packages to Zurich, Switzerland. It was received by Anita
Reyes and ask if she could inspect the packages. Shirley refused and eventually convinced Anita to seal
the package making it ready for shipment. Before being sent out for delivery, Job Reyes, husband of
Anita and proprietor of the courier company, conducted an inspection of the package as part of
standard operating procedures. Upon opening the package, he noticed a suspicious odor which made
him took sample of the substance he found inside. He reported this to the NBI and invited agents to his
office to inspect the package. In the presence of the NBI agents, Job Reyes opened the suspicious
package and found dried-marijuana leaves inside. A case was filed against Andre Marti in violation of
R.A. 6425 and was found guilty by the court a quo. Andre filed an appeal in the Supreme Court claiming
that his constitutional right of privacy was violated and that the evidence acquired from his package
was inadmissible as evidence against him.

Issue:Can the Constitutional Right of Privacy be enforced against private individuals?

Ruling: no, it cannot. The Supreme Court held based on the speech of Commissioner Bernas that the Bill
of Rights governs the relationship between the individual and the state.

The constitutional proscription against unlawful searches and seizures therefore applies as a restraint
directed only against the government and its agencies tasked with the enforcement of the law. It is not
meant to be invoked against acts of private individuals. It will be recalled that Mr Job Reyes was the
one who opened the box in the presence of the NBI agents in his place of business. The mere presence
of the NBI agents did not convert the reasonable search effected by Mr. Reyes into a warrantless search
and siezure proscribed by the constitution. Merely to observe and look at that which is in plain sight is
not a search.

People v. Aruta*– 288 SCRA 626 - Mikee

In the morning of 13 Dec 1988, the law enforcement officers received information from an informant
named “Benjie” that a certain “Aling Rosa” would be leaving for Baguio City on 14 Dec 1988 and would
be back in the afternoon of the same day carrying with her a large volume of marijuana; At 6:30 in the
evening of 14 Dec 1988, Aruta alighted from a Victory Liner Bus carrying a travelling bag even as the
informant pointed her out to the law enforcement officers; NARCOM officers approached her and
introduced themselves as NARCOM agents; When asked by Lt. Abello about the contents of her
travelling bag, she gave the same to him; When they opened the same, they found dried marijuana
leaves; Aruta was then brought to the NARCOM office for investigation.

Issue: were the items confiscated admissible as evidence against the respondent?

Ruling: no. In the instant case, the NARCOM agents were admittedly not armed with a warrant of
arrest. To legitimize the warrantless search and seizure of accused-appellant’s bag, she must have been
validly arrested. NARCOM had no warrant nor any cause for her arrest (not in flagrante delicto nor was
there probable cause).

Articles which are the product of unreasonable searches and seizures are inadmissible as evidence
pursuant to the doctrine pronounced in Stonehill v. Diokno. 5 This exclusionary rule was later enshrined
in Article III, Section 3(2) of the Constitution, thus:
Sec. 3(2). Any evidence obtained in violation of this or the preceding section shall be inadmissible in
evidence for any purpose in any proceeding.

From the foregoing, it can be said that the State cannot simply intrude indiscriminately into the
houses, papers, effects, and most importantly, on the person of an individual. The constitutional
provision guaranteed an impenetrable shield against unreasonable searches and seizures. As such, it
protects the privacy and sanctity of the person himself against unlawful arrests and other forms of
restraint.

Section 4. No law shall be passed abridging the freedom of speech, of expression, or of the press, or
the right of the people peaceably to assemble and petition the government for redress of grievances.

Content-based Regulation: Restraint is aimed at the message or idea of the expression. Apply the
Strict Scrutiny Test and the challenged act must overcome the clear and present danger rule.

Content-neutral Regulation: Restraint is aimed to regulate the time, place or manner of the
expression in public place without any restraint on the content of the expression. Apply the
Intermediate Approach Test wherein a regulation is justified if it is : within the constitutional power of
government, furthers an important or substantial government interest, government interest is
unrelated to the suppression of free expression, and the incident restriction on the alleged freedom of
speech and expression is no greater than is essential to the furtherance of that interest. Here, it only
requires substantial government interest for validity.

Facial Challenge Concept: A facial challenge is an exception to the rule that only persons who are
directly affected by a statute have legal standing to assail the same. This is only applicable to statutes
involving free speech, impeached on the grounds of overbreadth or vagueness. Here, the litigants are
permitted to challenge a statute not because their own rights of free expression are violated, but
because of a judicial prediction or assumption that the statute’s very existence may cause others not
before the court to refrain from constitutionally protected speech or expression.

Imbong v. Ochoa, GR 204819, April 8, 2014: While this Court has withheld the application of facial
challenges to strictly penal statues, it has expanded its scope to cover statutes not only regulating free
speech, but also those involving religious freedom, and other fundamental rights. The underlying
reason for this modification is simple. For unlike its counterpart in the U.S., this Court, under its
expanded jurisdiction, is mandated by the Fundamental Law not only to settle actual controversies
involving rights which are legally demandable and enforceable, but also to determine whether or not
there has been a grave abuse of discretion amounting to lack or excess of jurisdiction on the part of
any branch or instrumentality of the Government. – Val

FACTS: petitioners are assailing the constitutionality of RH Law . The OSG also assails the
propriety of the facial challenge lodged by the subject petitions, contending that the RH Law
cannot be challenged "on its face" as it is not a speech regulating measure.
ISSUE: Whether the Court may exercise its power of judicial review over the controversy.3]
Facial Challenge

RULING: In United States (US) constitutional law, a facial challenge, also known as a First
Amendment Challenge, is one that is launched to assail the validity of statutes concerning not
only protected speech, but also all other rights in the First Amendment. These include religious
freedom, freedom of the press, and the right of the people to peaceably assemble, and to petition
the Government for a redress of grievances. After all, the fundamental right to religious freedom,
freedom of the press and peaceful assembly are but component rights of the right to one's
freedom of expression, as they are modes which one's thoughts are externalized.

While this Court has withheld the application of facial challenges to strictly penal statues, it has
expanded its scope to cover statutes not only regulating free speech, but also those involving religious
freedom, and other fundamental rights. The underlying reason for this modification is simple. For
unlike its counterpart in the U.S., this Court, under its expanded jurisdiction, is mandated by the
Fundamental Law not only to settle actual controversies involving rights which are legally demandable
and enforceable, but also to determine whether or not there has been a grave abuse of discretion
amounting to lack or excess of jurisdiction on the part of any branch or instrumentality of the
Government.

Overbreadth Doctrine: A ground to declare a statute void when “it offends the constitutional principle
that a government purpose to control or prevent activities constitutionally subject to state regulations
may not be achieved by means which sweep unnecessarily broadly and thereby invade the area of
protected freedoms.”

Tests for Valid Government Interference to Freedom of Expression


1. Clear and Present Danger Test
2. Dangerous Tendency Test
3. Balancing of Interest Test

State Regulation of Different Types of Mass Media


1. Broadcast and Radio Media: It is subject to dual regulation: First, procure a legislative franchise.
Second, register and be subject to regulations set by the NTC.

(Divinagracia v. CBS, Inc GR 162272, April 7, 2009) – Val

FACTS: Respondents are involved in the operation of radio broadcasting services in


the Philippines, they being the grantees of legislative franchises by virtue of two laws R.A. No.
7477 and R.A. No. 7582.

Following the enactment of these franchise laws, the NTC issued four (4) Provisional
Authorities to PBS and six (6) Provisional Authorities to CBS, allowing them to install,
operate and maintain various AM and FM broadcast stations. Petitioner prayed for the
cancellation of all the Provisional Authorities or CPCs of PBS and CBS on account of the
alleged violation of the conditions set therein, as well as in its legislative franchises

ISSUE: whether the NTC, can cancel Provisional Authorities and Certificates of Public
Convenience

RULING: Yes. The scarcity of radio frequencies made it necessary for the government to step
in and allocate frequencies to competing broadcasters. As the government is put in a position to
determine who should be worthy to be accorded the privilege to broadcast from a finite and
limited spectrum, it may impose regulations to see to it that broadcasters promote the public
good deemed important by the State, and to withdraw that privilege from those who fall
short of the standards set in favor of other worthy applicants.

The operators of broadcast stations in the Philippines must secure a legislative


franchise and at the same time the CPC issued by the NTC. The complexities of our dual
franchise/license regime for broadcast media should be understood within the context of
separation of powers.

2. Print Media

The freedom of television and radio broadcasting is lesser in scope that the freedom accorded to
newspapers and print media.

(Eastern Broadcasting Corp v. Dans Jr) - Val

FACTS: This petition was filed to compel the respondents to allow the reopening of Radio
Station DYRE which had been summarily closed on grounds of national security. DYRE was
closed on the mere allegation that the radio station was used to incite people to sedition. The
petitioner also raised the issue of freedom of speech. It appears from the records that the
respondents' general charge of "inciting people to commit acts of sedition" arose from the
petitioner's shift towards what it stated was the coverage of public events and the airing of
programs geared towards public affairs.

ISSUE: whether the freedom of radio broadcasting is entitled to freedom of expression

RULING: Yes. All forms of communication are entitled to the broad protection of the
freedom of expression clause. The test for limitations on freedom of expression continues to
be the clear and present danger rule — that words are used in such circumstances and are of
such a nature as to create a clear and present danger that they will bring about the
substantive evils that the lawmaker has a right to prevent,

Necessarily, however, the freedom of television and radio broadcasting is somewhat


lesser in scope than the freedom accorded to newspaper and print media.

Radio broadcasting, more than other forms of communications, receives the most limited
protection from the free expression clause. First, broadcast media have established a uniquely
pervasive presence in the lives of all citizens, Material presented over the airwaves confronts
the citizen, not only in public, but in the privacy of his home. Second, broadcasting is
uniquely accessible to children. Bookstores and motion picture theaters may be prohibited from
making certain material available to children, but the same selectivity cannot be done in radio or
television, where the listener or viewer is constantly tuning in and out.

Private vs. Government speech


Heckler’s Veto: This involves situations in which the government attempts to ban protected speech
because it might provoke a violent response.

1. Prior Restraint: Refers to official governmental restrictions on the press or other forms of
expression in advance of actual publication or dissemination.

Valid Prior Restraint:


1. Movies, television, and radio broadcast censorship in view of its access to numerous people.
2. Pornography
3. False or misleading commercial statement
4. Advocacy of imminent lawless action
5. Danger to national security

Chavez v. Gonzales

Facts: Sometime before 6 June 2005, the radio station dzMM aired the Garci Tapes where the parties
to the conversation discussed “rigging” the results of the 2004 elections to favor President Arroyo.
Respondent Gonzalez ordered the NBI to investigate media organizations which aired the Garci Tapes
for possible violation of Republic Act No. 4200 or the Anti-Wiretapping Law.

On 11 June 2005, the NTC issued a press release warning radio and television stations that airing the
Garci Tapes is a ” cause for the suspension, revocation and/or cancellation of the licenses or
authorizations” issued to them. On 14 June 2005, NTC officers met with officers of the broadcasters
group KBP, to dispel fears of censorship.

Issue: Whether the freedom of expression may be abridged

Ruling: Yes. Freedom of expression is the foundation of a free, open and democratic society. Freedom of
expression is an indispensable condition to the exercise of almost all other civil and political rights.
Freedom of expression allows citizens to expose and check abuses of public officials. Freedom of
expression allows citizens to make informed choices of candidates for public office. Thus, the rule is that
expression is not subject to any prior restraint or censorship because the Constitution commands that
freedom of expression shall not be abridged. Over time, however, courts have carved out narrow and
well defined exceptions to this rule out of necessity.

The exceptions, when expression may be subject to prior restraint, apply in this jurisdiction to only four
categories of expression, namely: pornography, false or misleading advertisement, advocacy of
imminent lawless action, and danger to national security. All other expression is not subject to prior
restraint.

Near v. Minnesota – 238 US 697

Facts: The Saturday Press (the Press) published attacks on local officials. The Press claimed that the
chief of police had “illicit relations with gangsters.” Minnesota officials obtained an injunction in order
to abate the publishing of the Press newspaper under a state law that allowed this course of action.
The state law authorized abatement, as a public nuisance, of a “malicious, scandalous and defamatory
newspaper, or other periodical. A state court order abated the Press and enjoined the Defendants,
publishers of the Press (Defendants), from publishing or circulating such “defamatory and scandalous”
periodicals.

Ruling: The freedom of press is essential to the nature of a free state but that freedom may be
restricted by the government in certain situations.

The fact that the liberty of press may be abused by miscreant purveyors of scandal does not effect the
requirement that the press has immunity from previous restraints when it deals with official
misconduct. Subsequent punishment for such abuses as may exist is the appropriate remedy,
consistent with the constitutional privilege. Therefore, a statute authorizing such proceedings is not
consistent with the conception of the liberty of the press as historically conceived and guaranteed and
is thus, unconstitutional.

Freedman v. Maryland – 380 US 51

Facts: Defendant wished to challenge the constitutionality of a state censorship statute that required
submission of all motion pictures to the Board prior to their exhibition. He exhibited a film without
first submitting it to the Board and was subsequently convicted of violating the statute, Md. Code Ann.
art. 66A, § 2 (1957). The judgment of the trial court was affirmed, and on appeal of that decision the
reviewing court ruled in favor of defendant.

Ruling: The Court held that defendant's refusal to submit the film to the Board in violation only of § 2
did not restrict defendant to an attack on that section alone. The Court found validity in defendant's
contention that § 2 effected an invalid prior restraint on the freedom of speech because the structure
of the other provisions of the statute contributed to the infirmity of § 2, and that he did not assert that
the other provisions were independently invalid. The Court found that the statute lacked sufficient
safeguards against undue inhibition of protected expression, and that rendered the § 2 requirement of
prior submission of films to the Board an invalid previous restraint in violation of the Fourteenth
Amendment.

New York Times Co. v. US – 403 US 713 – Prince

1. In 1971, New York Times gained possession of top-secret articles based on the 1968 study,
“History of U.S. Decision Making Process on Viet Nam Policy.”
2. After studying the documents, they published several newspaper articles.
3. Dept of Justice sought and gained an injunction from the Circuit Court.
4. Circuit Court extended the restraining order to give the Court of Appeals of the District of
Columbia enough time to study the government's case.
5. Then the Circuit Court remanded to the trial court for determination of whether any of the
future publications “posed such grave and immediate danger to the security of the country as
to warrant prior restraint and a continued stay of publication.”

Issue: Whether the prior restraint was justified.


HELD: NO. The United States did not meet the heavy burden of showing justification for the
enforcement of such a restraint on the New York Times and Washington Post to enjoin them from
publishing contents of a classified study.
Tolentino v. Sec. of Finance – GR 115444, Oct. 30, 1995 – Prince

1. Petitioners assailed the constitutionality of RA No. 7716, EVAT Law.


2. The Philippine Press Institute (PPI) argues that by removing the exemption of the press from
the VAT while maintaining those granted to others, the law discriminates against the press. At
any rate, it is averred, "even nondiscriminatory taxation of constitutionally guaranteed freedom
is unconstitutional."
3. The Philippine Bible Society, Inc. claims that although it sells bibles, the proceeds derived from
the sales are used to subsidize the cost of printing copies which are given free to those who
cannot afford to pay so that to tax the sales would be to increase the price, while reducing the
volume of sale.

ISSUE: Whether sales tax on bible or newspaper sales violates religious or press freedom.

HELD: SALES TAX IS AN EXCISE TAX; EXCISE TAX DOES NOT REGULATE PRIVILEGES; EVAT IS NOT A
LICENSE TAX BUT AN EXCISE TAX. HENCE, SALES TAX ON BIBLES OR NEWSPAPERS DOES NOT
REGULATE OR REMOVE PRESS FREEDOM OR EXERCISE OF RELIGION.

The VAT is not a license tax. It is not a tax on the exercise of a privilege, much less a constitutional right.
It is imposed on the sale, barter, lease or exchange of goods or properties or the sale or exchange of
services and the lease of properties purely for revenue purposes. To subject the press to its payment is
not to burden the exercise of its right any more than to make the press pay income tax or subject it to
general regulation is not to violate its freedom under the Constitution.

A license tax, which, unlike an ordinary tax, is mainly for regulation. Its imposition on the press is
unconstitutional because it lays a prior restraint on the exercise of its right. Its application to the press
or to religious groups, in connection with the latter's sale of religious books and pamphlets, is
unconstitutional. As the U.S. Supreme Court put it, "it is one thing to impose a tax on income or property
of a preacher. It is quite another thing to exact a tax on him for delivering a sermon."

Alexander v. US – 509 U.S. 544(113 S.Ct. 2766, 125 L.Ed.2d 441)– Prince

1. Alexander owns businesses selling sexually explicit materials.


2. The government filed a criminal complaint for violations of Racketeer Influenced and Corrupt
Organizations Act (RICO).
3. He is subsequently charged with the crime of violating the obscenity laws.
4. District Court found him guilty of under the RICO. Aside from the prison term, the District Court
ordered forfeiture of his income or properties acquired through racketeering. In effect, he
lose all of his properties.
5. Alexander argued before the Court of Appeals that the forfeiture constitute a prior restraint on
freedom of speech and are overbroad.

ISSUE: Whether the forfeiture was constitutional.

HELD: Yes. RACKETEERING CAUSED HIS FORFEITURE; FORFEITURE IS FOR RACKETEERING AND NOT
FOR THE SALE OF OBSCENE MATERIALS. THUS, FORFEITURE DOES NOT PROHIBIT FREE SPEECH.
The forfeiture imposes no legal impediment to petitioner's ability to engage in any expressive activity; it
just prevents him from financing those activities with assets derived from his prior racketeering
offenses. There is no forbidding of certain communications that are issued before the communications
occur. Moreover, there is no restriction of his future speech. Besides, Petitioner's assets were forfeited
because they were directly related to past racketeering violations.

INC v. CA, 259 SCRA 529 (1996) – Prince


[G.R. NO. 119673; 26 JULY 1996]

1. Iglesia ni Cristo (INC) openly criticize beliefs and practices of other religions or Christian sects in
their program.
2. INC submitted some recordings of their episodes to the MTRCB for rating purposes. MTRCB
rated the episodes as “X-rated” since it is offensive to the faith of other religions or sects.
3. INC appealed the MTRCB's decision to the Office of President. INC reasoned that their
recordings are protected by the constitutional guarantee of free speech and expression and no
indication that the episode poses any clear and present danger.

ISSUE 1: Whether INC's program recordings of open criticisms to other faith is protected under free
exercise of religion.

HELD: MTRCB CENSORSHIP OF ONE RELIGION OPENLY FAVOR OTHER RELIGIONS; CONSTITUTION
PROHIBITS FAVOR OF ANY RELIGION. THUS, MTRCB'S CENSORSHIP IS UNCONSTITUTIONAL.

MTRCB may disagree with the criticisms of other religions but it cannot censor such criticisms. MTRCB,
in effect, openly agreed and favored other religions or Christian sects. Under the Constitutional, it is not
the task of the State to favor any religion by protecting it against an attack by another religion. The
establishment clause of freedom of religion prohibits the State from leaning towards any religion even
by censorship of criticisms. Indeed, the basis of freedom of religion is freedom of thought and it is best
served by encouraging the marketplace of dueling ideas.

ISSUE 2: Whether the criticisms of other religions is protected under free speech.
HELD: NO PROOF OF SUBSTANTIVE OR IMMINENT DANGER. THUS, RESTRAINT IS UNJUSTIFIED.
The criticisms do not give any immediate and grave danger. Only when there is an immediate and grave
danger of substantive and imminent evil to the security and welfare of the community, infringement of
religious freedom may be justified but to the smallest extent necessary to avoid the danger or evil. Prior
restraint on speech, including religious speech, cannot be justified by hypothetical fears.
The clear and present danger test applies to time, place or manner of speech, not to its content. Thus,
this test does not apply to criticisms of other religions since it is a content of speech.

ISSUE 3: Whether the MTRCB committed grave abuse of discretion.


HELD: MTRCB IS AN ADMINISTRATIVE BODY NOT AUTHORIZED TO DETERMINE FREE SPEECH. HENCE,
MTRCB COMMITTED GRAVE ABUSE OF DISCRETION.
The determination of whether the vilification, exaggeration or fabrication falls within or lies outside the
boundaries of protected speech or expression is a judicial function which cannot be arrogated by an
administrative body such as the MTRCB. A system of prior restraint may only be validly administered by
judges and not left to administrative agencies.
SWS v. COMELEC, GR 147571, May 5, 2001 – Prince

1. Election surveys refer to the measurement of opinions and perceptions of the voters as regards
a candidate's popularity, qualifications, platforms or a matter of public discussion in relation to
the election, including voter’s preference for candidates or publicly discussed issues during the
campaign period.
2. Social Weather Station intends to conduct an election survey. It assails the constitutionality of
the following:
◦ §5.4 of RA. No.9006 (Fair Election Act): Surveys affecting national candidates shall not be
published fifteen (15) days before an election and surveys affecting local candidates shall
not be published seven (7) days be- fore an election.
◦ COMELEC Resolution 3636, §24(h), dated March I, 2001: Surveys affecting national
candidates shall not be published fifteen (15) days before an election and surveys affecting
local candidates shall not be published seven (7) days be- fore an election.

ISSUE: Whether the provisions are constitutional.

HELD: NO. UNCONSTITUTIONAL FOR PRIOR RESTRAINT. Sec 5.4 of R.A. No. 9006; Sec. 24(h) of
COMELEC Resolution 3636, March 1, 2001.
 Sec 5.4 and the Sec 24(h) are unconstitutional (1) it imposes a prior restraint on the freedom of
expression, (2) it is a direct and total suppression of a category of expression even though such
suppression is only for a limited period, and (3) the governmental interest sought to be
promoted can be achieved by means other than suppression of freedom of expression.
 In sustaining the ban on media political advertisements, the grant of power to the COMELEC
under Art. IX-C, §4 is limited to ensuring "equal opportunity, time, space, and the right to reply"
as well as uniform and reasonable rates of charges for the use of such media facilities "public
information campaigns and forums among candidates."
 When utterances are not essential part of any exposition of ideas, and are of such slight social
value as a step to truth that any benefit that may be derived from them is clearly outweighed by
the social interest in order and morality, then the State can restraint.
 The provisions failed the test in United States v. O 'Brien.
 Does not fall under the exceptions discussed in Near vs. Minnesota.

Chavez v. Gonzales, GR 168338, February 15, 2008 – Prince

1. Hello Garci Scandal, Gloriagate Scandal.


2. Release of the wiretapped conversation between Pres. GMA and COMELEC Chair Garcillano and
wiretapped conversation among Garcillano, Sen. Barbers, and Mr. Arroyo.
3. DOJ Secretary warned mass-media corporations and media-men that they will be liable under the
Anti-Wiretapping Act if they possess the recordings. He ordered the NBI to investigate mass-media
corporations. GMA-7 and Philippine Daily Inquirer were included because of the release through
their joint operations of Inq7.net.
4. National Telecommunication Commission issued notice to all mass-media corporations of possible
suspension or revocation of licenses and authorizations.

ISSUE: Whether free speech or press was violated.


HELD: YES. NO CLEAR AND PRESENT DANGER; NO THREAT TO NATIONAL SECURITY. THUS,
UNJUSTIFIED RESTRAINT OF SPEECH. Compare with SWS vs. COMELEC

 The DOJ's and NTC's press statements failed the clear and present danger test.
 There is no danger to national security.
 The clear and present danger test likewise applies to Broadcasting Media although Constitution,
laws, and jurisprudence gives it lesser scope of freedom.
 It is sufficient that the press statements were made by respondents NTC and DOJ while in the
exercise of their official functions. Any act done, such as a speech uttered, for and on behalf of
the government in an official capacity is covered by the rule on prior restraint. Otherwise, the
non formalization of an act into an official order or circular will result in the easy circumvention
of the prohibition on prior restraint.
 There is chilling effect or prior restraint because NTC is empowered to revoke broadcast licenses
and DOJ is empowered to prosecute against a corporation or individual.

Newsounds Broadcasting v. Dy, GR 170270 and 179411, April 2, 2009 - Rhino

FACTS: Bombo Radyo Philippines operates several radio stations under the AM and FM band
throughout the Philippines. These stations are operated by corporations organized and incorporated by
Bombo Radyo, particularly petitioners Newsounds Broadcasting Network, Inc. (“Newsounds”) and
Consolidated Broadcasting System, Inc. Newsounds commenced relocation of its broadcasting stations,
management office and transmitters on property located in Cauayan City, Isabela. The property is
owned by CBS Development Corporation (CDC), an affiliate corporation under the Bombo Radyo
network. The property was initially classified as commercial. But later, the City Zoning Administrator
under the new administration of respondent mayor Dy refuse to issue the zoning clearance, necessary
for the renewal mayor’s permit, for the property, but required petitioner to submit “either an
approved land conversion papers from the Department of Agrarian Reform (DAR) showing that the
property was converted from prime agricultural land to commercial land. The RTC rendered a Decision
denying the petition for mandamus. Responden Mayor Dy then issued a closure order for lack of
business permit. Newsound’s only rival AM station in Cauayan and the rest of Isabela, DWDY, is
owned and operated by the family of respondent Dy.

ISSUE: Whether there is dereliction of freedom of speech, of expression or the press.

RULING: YES! The bare acts of closing the radio stations or preventing their operations are acts of
prior restraint against speech, expression or of the press. Ostensibly, the act of an LGU requiring a
business of proof that the property from which it operates has been zoned for commercial use can be
argued, when applied to a radio station, as content-neutral since such a regulation would presumably
apply to any other radio station or business enterprise within the LGU. However, the circumstances of
this case dictate that the action of the respondents in deliberately shutting down the operation of
petitioner with ill-motive, as a content-based restraint. Content-based restrictions on free speech
require strict scrutiny. The immediate implication of the application of the “strict scrutiny” test is that
the burden falls upon respondents as agents of government to prove that their actions do not infringe
upon petitioners’ constitutional rights.

MTRCB v. ABS-CBN, GR 155282, January 17, 2005 – Rhino


FACTS: Respondent ABS-CBN aired “Prosti-tuition,” an episode of TV program “The Inside Story”
produced and hosted by respondent Legarda. It depicted female students moonlighting as prostitutes
to enable them to pay for their tuition fees. In the course of the program, student prostitutes, pimps,
customers, and some faculty members were interviewed. The Philippine Women’s University (PWU)
was named as the school of some of the students involved and the facade of PWU Building at Taft
Avenue, Manila conspicuously served as the background of the episode. PWU filed letter-complaints
with petitioner MTRCB which alleged that respondents (1) did not submit “The Inside Story” to
petitioner for its review and (2) exhibited the same without its permission in violation of PD 1986.
Respondents explained that the “The Inside Story” is a “public affairs program, news documentary and
socio-political editorial,” the airing of which is protected by the constitutional provision on freedom of
expression and of the press. MTRCB sanctioned ABS-CBN ordering that all subsequent programs of the
‘The Inside Story’ and all other programs of the ABS-CBN Channel 2 of the same category shall be
submitted to the Board of Review and Approval before showing.

ISSUE: Whether Petitioner has the power to make such review.

RULING: NO! P.D. No. 1986 gives petitioner “the power to screen, review and examine “all television
programs,” emphasizing the phrase “all television programs.” It then follows that since “The Inside
Story” is a television program, it is within the jurisdiction of the MTRCB over which it has power of
review. There has been no declaration at all by the framers of the Constitution that freedom of
expression and of the press has a preferred status. The only exceptions from the MTRCB’s power of
review are those expressly mentioned in Section 7 of P. D. No. 1986, such as (1) television programs
imprinted or exhibited by the Philippine Government and/or its departments and agencies,
and (2) newsreels. P. D. No. 1986 define newsreels as “straight news reporting, as distinguished from
news analyses, commentaries and opinions. Talk shows on a given issue are not considered
newsreels. Clearly, the “The Inside Story” cannot be considered a newsreel. It is more of a public affairs
program which is described as a variety of news treatment; a cross between pure television news and
news-related commentaries, analysis and/or exchange of opinions. The SC did not rule on the
constitutionality of PD 1986 in relation to freedom of expression as the requirements to do so were not
present.

Re: Request for Radio-TV Coverage of the Estrada Trial, AM No. 01-4-03-SC, June 29, 2001 - Rhino

FACTS: The KBP sent a letter requesting this Court to allow live media coverage of the anticipated trial
of the plunder and other criminal cases filed against former President Joseph E. Estrada before the
Sandiganbayan in order "to assure the public of full transparency in the proceedings of an
unprecedented case in our history."

ISSUE: Whether the live radio and television coverage of the proceedings should be allowed.

RULING: NO! The propriety of granting or denying the instant petition involve the weighing out of the
constitutional guarantees of freedom of the press and the right to public information, on the one
hand, and the fundamental rights of the accused, on the other hand, along with the constitutional
power of a court to control its proceedings in ensuring a fair and impartial trial. When these rights race
against one another, jurisprudence tells us that the right of the accused must be preferred to win.

Due process guarantees the accused a presumption of innocence until the contrary is proved in a trial
that is not lifted above its individual settings nor made an object of public's attention and where the
conclusions reached are induced not by any outside force or influence but only by evidence and
argument given in open court, where fitting dignity and calm ambiance is demanded.

Other reasons: (1) The potential impact of television, increases the chance of prejudice that is present
in every criminal case; (2) The impact upon a witness of the knowledge that he is being viewed by a vast
audience, may affect the quality of his testimony; (3) The presence of television places on the trial judge
additional responsibilities rather than focus in ascertaining that the accused receives a fair trial; (4) The
presence of television is a form of mental - if not physical-harassment against the defendant, resembling
a police line-up or the third degree; and (5) It can negate the rule on exclusion of witnesses during the
hearings intended to assure a fair trial.

Soriano v. Laguardia, GR 164785, April 29, 2009 – Rhino

FACTS: Petitioner Soriano, as host of the program Ang Dating Daan, aired on UNTV 37, made the
following remarks:

Lehitimong anak ng demonyo; sinungaling;

Gago ka talaga Michael, masahol ka pa sa putang babae o di ba. Yung putang babae
ang gumagana lang doon yung ibaba, [dito] kay Michael ang gumagana ang itaas, o di
ba! O, masahol pa sa putang babae yan. Sabi ng lola ko masahol pa sa putang babae
yan. Sobra ang kasinungalingan ng mga demonyong ito. x x x

Based on this, two complaints were filed before the respondent MTRC against him by private
respondents members of the INC. The respondent preventively suspended the showing of Ang Dating
Daan program for 20 days, in accordance with PD 1986. Petitioner went to the SC while MR is pending
with respondent.

ISSUE: Whether there is prior restraint to freedom of speech and expression.

RULING: NO! Prior restraint means official government restrictions on the press or other forms of
expression in advance of actual publication or dissemination. The freedom of expression, as with the
other freedoms encased in the Bill of Rights, is, however, not absolute. It may be regulated to some
extent to serve important public interest, some forms of speech not being protected:
1. as when it touches upon matters of essentially private concern
2. if the utterances involved are “no essential part of any exposition of ideas, and are of such slight
social value as a step of truth that any benefit that may be derived from them is clearly
outweighed by the social interest in order and morality.”
3. LOW-VALUE EXPRESSIONS: libelous statements, obscenity or pornography, false or misleading
advertisement, insulting or “fighting words”, i.e., those which by their very utterance inflict
injury or tend to incite an immediate breach of peace and expression endangering national
security.
There is no perfect definition of “obscenity” but the latest word is that of Miller v. California which
established basic guidelines, to wit: (a) whether to the average person, applying contemporary
standards would find the work, taken as a whole, appeals to the prurient interest; (b) whether the work
depicts or describes, in a patently offensive way, sexual conduct specifically defined by the applicable
state law; and (c) whether the work, taken as a whole, lacks serious literary, artistic, political, or
scientific value. The Court finds that petitioner’s statement can be treated as obscene, at least with
respect to the average child. While the words uttered may be regarded only as indecent by an ordinary
adult, the problem is that the program was Rated G or for general viewership, and in a time slot that
would likely reach even the eyes and ears of children.

THE CLEAR AND PRESENT DANGER DOCTRINE (tackled in previous cases) IS NOT APPROPRIATE IN THIS
CASE: Since not all evils can be measured in terms of “proximity and degree” BALANCING OF INTEREST
DOCTRINE (tackled in previous cases) IS MORE APPROPRIATE: In this setting, the assertion by
petitioner of his enjoyment of his freedom of speech is ranged against the duty of the government to
protect and promote the development and welfare of the youth.

GENERAL RULE ON PRIOR RESTRAINTS ON SPEECH: Statutes imposing prior restraints on speech are
generally illegal and presumed unconstitutional breaches of the freedom of speech. EXCEPTIONS TO
PRIOR RESTRAINTS: are movies, television, and radio broadcast censorship in view of its access to
numerous people, including the young who must be insulated from the prejudicial effects of
unprotected speech. PD 1986 was passed creating the MTRCB and which requires prior permit or
license before showing a motion picture or broadcasting a TV program.

THE THREE (3) MONTHS SUSPENSION IN THIS CASE IS NOT A PRIOR RESTRAINT on the right of
petitioner to continue with the broadcast of Ang Dating Daan as a permit was already issued to him by
MTRCB for such broadcast. RATHER, THE SUSPENSION IS IN THE FORM OF PERMISSIBLE
ADMINISTRATIVE SANCTION OR SUBSEQUENT PUNISHMENT for the offensive and obscene remarks he
uttered. It is a sanction that the MTRCB may validly impose under its charter without running afoul of
the free speech clause. And the imposition is separate and distinct from the criminal action the Board
may take pursuant to Sec. 3(i) of PD 1986 and the remedies that may be availed of by the aggrieved
private party under the provisions on libel or tort, if applicable.

2. Subsequent Punishment

People v. Perez – 45 Phil. 599 G.R. No. L-21049 December 22, 1923- Rhino

FACTS: Perez, the municipal secretary of Pilar, Sorsogon, and Lodovice, a citizen of that municipality,
happening to meet one morning, in the presidencia of Pilar, they became engaged in a discussion
regarding the administration of Governor-General Wood, which resulted in Perez shouting a number
of times: "The Filipinos, like myself, must use bolos for cutting off Wood's head for having
recommended a bad thing for the Filipinos, for he has killed our independence." He was charged with
violation of Article 256 of the Penal Code having to do with contempt of ministers of the Crown or
other persons in authority. He was convicted, thus this appeal.

ISSUE: Whether the speech of Perez falls within the legal bounds of freedom of speech.

RULING: NO! Criticism is permitted to penetrate even to the foundations of Government. Criticism,
no matter how severe, on the Executive, the Legislature and the Judiciary, is within the range of liberty
of speech, unless the intention and effect be seditious. When the intention and effect of the act is
seditious, the constitutional guaranties of freedom of speech and press and of assembly and petition
must yield to punitive measures designed to maintain the prestige of constituted authority, the
supremacy of the constitution and the laws, and the existence of the State.
In this instance, the attack on the Governor-General passes the furthest bounds of free speech and
common decency. More than a figure of speech was intended. There is a seditious tendency in the
words used, which could easily produce dissatisfaction among the people and a state of feeling
incompatible with a disposition to remain loyal to the Government and obedient to the laws. The
Governor-General is the representative of executive civil authority in the Philippines and of the
sovereign power. A seditious attack on the Governor-General is an attack on the rights of the Filipino
people and on American sovereignty.

Espiritu v. General Lim, GR 85727, October 3, 1991 - Rhino

FACTS: Separate motions are now before the SC filed by the petitioners in several petitions for habeas
corpus, seeking reconsideration of the Court's decision thereof promulgated on 9 July 1990 which
dismissed the petitions on the ground that the arrests without warrant were valid.

ISSUE: Whether the arrest without warrant was made in accordance with law.

RULING: YES! As a general rule, no peace officer or person has the power or authority to arrest anyone
without a warrant of arrest, except in those cases expressly authorized by law. The law expressly
allowing arrests without warrant is found in Section 5, Rule 113 of the Rules of Court which states the
grounds upon which a valid arrest, without warrant, can be conducted. The arrests without warrant are
valid:
1. Subversion like rebellion (or insurrection) is perceived here as a continuing offense. Unlike other
so-called "common" offenses, i.e. adultery, murder, arson, etc., which generally end upon their
commission, subversion and rebellion are anchored on an ideological base which compels the
repetition of the same acts of lawlessness and violence until the overriding objective of
overthrowing organized government is attained. Dural did not cease to be, or became less of a
subversive, FOR PURPOSES OF ARREST, simply because he was, at the time of arrest, confined in
the St. Agnes Hospital.
2. Rule 113 of the Rules of Court, which requires two (2) conditions for a valid arrestt without
warrant: first, that the person to be arrested has just committed an offense, and second, that
the arresting peace officer or private person has personal knowledge of facts indicating that the
person to be arrested is the one who committed the offense. It has been ruled that "personal
knowledge of facts," in arrests without warrant must be based upon probable cause, which
means an actual belief or reasonable grounds of suspicion. The grounds of suspicion are
reasonable when, in the absence of actual belief of the arresting officers, the suspicion that the
person to be arrested is probably guilty of committing the offense, is based on actual facts, i.e.,
supported by circumstances sufficiently strong in themselves to create the probable cause of
guilt of the person to be arrested.
3. The arrest was due to "in flagrante" pursuant to search warrant.
4. In the balancing of authority and freedom, which obviously becomes difficult at times, the Court
has, in this case, tilted the scale in favor of authority but only for purposes of the arrest (not
conviction).
Mere suspicion of being a Communist Party member or a subversive is absolutely not a ground for the
arrest without warrant of the suspect. In conclusion, the Court predicated the validity of the questioned
arrests without warrant in these petitions, not on mere unsubstantiated suspicion, but on compliance
with the conditions set forth in Section 5, Rule 113, Rules of Court, a long existing law, and which, for
stress, are probable cause and good faith of the arresting peace officers, and, further, on the basis of, as
the records show, the actual facts and circumstances supporting the arrests.
Dennis v. US – 341 US 494 - Bryan

Facts:
Petitioners were indicted in July 1948 for violating a provision of the Smith Act.

They were charged for willfully and knowingly conspiring (1) to organize as the Communist Party of the
United States of America a society, group and assembly of persons who teach and advocate the
overthrow and destruction of the Government of the United States by force and violence, and (2)
knowingly and willfully to advocate and teach the duty and necessity of overthrowing and destroying
the Government of the United States by force and violence.

Petitioners were found guilty by the trial court and the decision was affirmed by the Second Circuit
Court of Appeals. The Supreme Court granted writ of certiorari, but limited it to whether section two or
three of the Smith Act violated the First Amendment and whether the same two sections violated the
First and Fifth Amendments because of indefiniteness.

Issue: When does the “clear and present danger” rule apply?

Ruling: Overthrow of the Government by force and violence is certainly a substantial enough interest
for the Government to limit speech. Indeed, this is the ultimate value of any society, for if a society
cannot protect its very structure from armed internal attack, it must follow that no subordinate value
can be protected. If, then, this interest may be protected, the literal problem which is presented is
what has been meant by the use of the phrase "clear and present danger" of the utterances bringing
about the evil within the power of Congress to punish.

Obviously, the words cannot mean that, before the Government may act, it must wait until the putsch is
about to be executed, the plans have been laid and the signal is awaited. If Government is aware that a
group aiming at its overthrow is attempting to indoctrinate its members and to commit them to a course
whereby they will strike when the leaders feel the circumstances permit, action by the Government is
required. The argument that there is no need for Government to concern itself, for Government is
strong, it possesses ample powers to put down a rebellion, it may defeat the revolution with ease needs
no answer. For that is not the question. Certainly an attempt to overthrow the Government by force,
even though doomed from the outset because of inadequate numbers or power of the revolutionists,
is a sufficient evil for Congress to prevent. The damage which such attempts create both physically and
politically to a nation makes it impossible to measure the validity in terms of the probability of success,
or the immediacy of a successful attempt. In the instant case, the trial judge charged the jury that they
could not convict unless they found that petitioners intended to overthrow the Government

Gonzales v. COMELEC – 27 SCRA 835 - Bryan

RA 4880, prohibiting the too early nomination of candidates and limiting the period of election
campaign or partisan political activity was challenged on constitutional grounds. More precisely, the
basic liberties of free speech and free press, freedom of assembly and freedom of association are
invoked to nullify the act.
Petitioner Gonzales, is a private individual, a registered voter in the City of Manila and a political leader
of his co-petitioner. There was the further allegation that the nomination of a candidate and the fixing
of period of election campaign are matters of political expediency and convenience which only political
parties can regulate or curtail by and among themselves through self-restraint or mutual understanding
or agreement and that the regulation and limitation of these political matters invoking the police power,
in the absence of clear and present danger to the state, would render the constitutional rights of
petitioners meaningless and without effect. Senator Lorenzo M. Tañada was asked to appear as amicus
curiae, and elucidated that Act No. 4880 could indeed be looked upon as a limitation on the preferred
rights of speech and press, of assembly and of association. He did justify its enactment however under
the clear and present danger doctrine, there being the substantive evil of elections, whether for
national or local officials, being debased and degraded by unrestricted campaigning, excess of
partisanship and undue concentration in politics with the loss not only of efficiency in government but
of lives as well.

Issue: Whether the freedom of speech and of the press has been violated.

Held: No. This Court is of the view that no unconstitutional infringement exists. In determining the
validity of the law, free speech as a social value must be weighed against the political process as a
social value.

The very idea of a government, republican in form, implies a right on the part of its citizens to meet
peaceably for consultation in respect to public affairs and to petition for redress of grievances. As in the
case of freedom of expression, this right is not to be limited, much less denied, except on a showing of
a clear and present danger of a substantive evil that Congress has a right to prevent. This test then as a
limitation on freedom of expression is justified by the danger or evil a substantive character that the
state has a right to prevent. Unlike the dangerous tendency doctrine, the danger must not only be clear
but also present. The term clear seems to point to a causal connection with the danger of the
substantially evil arising from the utterance questioned. Present refers to the time element. It used to
be identified with imminent and immediate danger. The danger must not only be probable but very
likely inevitable.

Eastern Broadcasting v. Dans, Jr. – 137 SCRA 628 - Bryan

Facts: This petition was filed to compel the respondents to allow the reopening of Radio Station DYRE
which had been summarily closed on grounds of national security. The petitioner raised the Issue of
freedom of speech. It appears from the records that the respondents' general charge of "inciting people
to commit acts of sedition" arose from the petitioner's shift towards what it stated was the coverage
of public events and the airing of programs geared towards public affairs.

Before the Court could promulgate a decision squarely passing upon all the Issue raised, the
petitioner through its president, Mr. Rene G. Espina suddenly filed a motion to withdraw or dismiss the
petition for it has already sold its its radio broadcasting station in in favor of someone to whom the
government willingly gave a permit to operate.

Issue:
Which test has found preference with the Supreme Court?

Held:
With the declaration of democracy, the clear and present danger test is again coming into favor.

For the guidance of inferior courts and administrative tribunals exercising quasi-judicial
functions, the Court Issue the following guidelines:

(1) The cardinal primary requirements in administrative proceedings should be followed before a
broadcast station may be closed or its operations curtailed.

(2) It is necessary to reiterate that while there is no controlling and precise definition of due process, it
furnishes an unavoidable standard to which government action must conform in order that any
deprivation of life, liberty, or property, in each appropriate case, may be valid

(3) All forms of media, whether print or broadcast, are entitled to the broad protection of the freedom
of speech and expression clause. The test for limitations on freedom of expression continues to be the
clear and present danger rule — that words are used in such circumstances and are of such a nature as
to create a clear and present danger that they will bring about the substantive evils that the lawmaker
has a right to prevent.

(4) The clear and present danger test, however, does not lend itself to a simplistic and all embracing
interpretation applicable to all utterances in all forums.

Broadcasting has to be licensed. Airwave frequencies have to be allocated among qualified


users. A broadcast corporation cannot simply appropriate a certain frequency without regard for
government regulation or for the rights of others. All forms of communication are entitled to the broad
protection of the freedom of expression clause. Necessarily, however, the freedom of television and
radio broadcasting is somewhat lesser in scope than the freedom accorded to newspaper and print
media.

(5) The clear and present danger test, therefore, must take the particular circumstances of broadcast
media into account. The supervision of radio stations-whether by government or through self-
regulation by the industry itself calls for thoughtful, intelligent and sophisticated handling.

The government has a right to be protected against broadcasts which incite the listeners to
violently overthrow it. Radio and television may not be used to organize a rebellion or to signal the start
of widespread uprising. At the same time, the people have a right to be informed. Radio and television
would have little reason for existence if broadcasts are limited to bland, obsequious, or pleasantly
entertaining utterances. Since they are the most convenient and popular means of disseminating
varying views on public Issue, they also deserve special protection.
(6) The freedom to comment on public affairs is essential to the vitality of a representative democracy.

(7) Broadcast stations deserve the special protection given to all forms of media by the due process
and freedom of expression clauses of the Constitution.
WHEREFORE, the case having become moot and academic, the petitioner's motion to withdraw or
dismiss the petition is hereby GRANTED.

Ayer Prod. PTY. LTD. V. Judge Capulong – 160 SCRA 865 - Bryan
Facts:
Petitioner Ayer Productions, envisioned, sometime in 1987, for commercial viewing and for
Philippine and international release, the historic peaceful struggle of the Filipinos at EDSA. The
proposed motion picture entitled "The Four Day Revolution" would be essentially a re-enactment of the
events that made possible the EDSA revolution.

Private respondent Enrile replied that "[he] would not and will not approve of the use,
appropriation, reproduction and/or exhibition of his name, or picture, or that of any member of his
family in any cinema or television production, film or other medium for advertising or commercial
exploitation". Private respondent filed a Complaint with application for Temporary Restraining Order
and Writ of Preliminary Injunction seeking to enjoin petitioners from producing the movie "The Four
Day Revolution". The complaint alleged that petitioners' production of the mini-series without private
respondent's consent and over his objection, constitutes an obvious violation of his right of privacy.

Issue: Whether right to privacy is supreme over the freedom of speech and of expression.

Ruling:
NO. Motion pictures are important both as a medium for the communication of Ideas and the
expression of the artistic impulse. This freedom is available in our country both to locally-owned and to
foreign-owned motion picture companies even if they are commercial. Indeed there is such a thing as
the right to privacy. But this cannot be invoked to resist publication of matters of public interest, what
the right to privacy protects is the right against unwarranted intrusions and wrongful publication of
the private affairs and activities of individuals which are outside the sphere of legitimate public concern.
Enrile’s role in that revolution is a matter of public interest because he was a principal figure in that
event.

The line of equilibrium in the specific context of the instant case between the constitutional
freedom of speech and of expression and the right of privacy, may be marked out in terms of a
requirement that the proposed motion picture must be fairly truthful and historical in its presentation of
events. There must, in other words, be no knowing or reckless disregard of truth in depicting the
participation of private respondent in the EDSA Revolution. There must, further, be no presentation of
the private life of the unwilling private respondent and certainly no revelation of intimate or
embarrassing personal Facts. The proposed motion picture should not enter into matters of essentially
private concern. To the extent that "The Four Day Revolution" limits itself in portraying the participation
of private respondent in the EDSA Revolution to those events which are directly and reasonably related
to the public Facts of the EDSA Revolution, the intrusion into private respondent's privacy cannot be
regarded as unreasonable and actionable. Such portrayal may be carried out even without a license
from private respondent.

Kelley v. Johnson – 425 US 238 - Bryan

Facts: Respondent's predecessor, individually and as president of the Suffolk County Patrolmen's
Benevolent Association, brought this against petitioner's predecessor, the Commissioner of the Suffolk
County Police Department. The Commissioner of the Suffolk County Police Department had
promulgated Order No. 71-1, which established hair-grooming standards applicable to male members
of the police force. The regulation was directed at the style and length of hair, sideburns, and
mustaches; beards and goatees were prohibited, except for medical reasons; and wigs conforming to
the regulation could be worn for cosmetic reasons. The regulation was attacked as violative of
respondent patrolman's right of free expression under the First Amendment and his guarantees of due
process and equal protection under the Fourteenth Amendment, in that it was "not based upon the
generally accepted standard of grooming in the community," and placed "an undue restriction" upon
his activities therein.

Issue:
Whether respondent can demonstrate that there is no rational connection between the
regulation, based as it is on the county's method of organizing its police force, and the promotion of
safety of persons and property.

Ruling:
A county regulation limiting the length of county policemen's hair held not to violate any
right guaranteed to respondent policeman by the Fourteenth Amendment.

The promotion of safety of persons and property is unquestionably at the core of the State's
police power, and virtually all state and local governments employ a uniformed police force to aid in the
accomplishment of that purpose. Choice of organization, dress, and equipment for law enforcement
personnel is a decision entitled to the same sort of presumption of legislative validity as are state
choices designed to promote other aims within the cognizance of the State's police power we think
Suffolk County's police regulations involved here are entitled to similar weight.

The regulation challenged here did not violate any right guaranteed respondent by the
Fourteenth Amendment to the United States Constitution, and the Court of Appeals was therefore
wrong in reversing the District Court's original judgment dismissing the action.

When the State has an interest in regulating one's personal appearance, as it certainly does in
this case, there must be a weighing of the degree of infringement of the individual's liberty interest
against the need for the regulation. This process of analysis justifies the application of a reasonable
regulation to a uniformed police force that would be an impermissible intrusion upon liberty in a
different context.

Brandenburg v. Ohio – 395 US 444 - Bryan

Facts:
Brandenburg, a leader in the Ku Klux Klan, made a speech at a Klan rally and was later
convicted under an Ohio criminal syndicalism law. The law made illegal advocating "crime, sabotage,
violence, or unlawful methods of terrorism as a means of accomplishing industrial or political reform,"
as well as assembling "with any society, group, or assemblage of persons formed to teach or advocate
the doctrines of criminal syndicalism."

Issue:
Did Ohio's criminal syndicalism law, prohibiting public speech that advocates various illegal activities,
as protected by the First and Fourteenth Amendments?
Ruling:
Legal provision: Amendment 1: Speech, Press, and Assembly
The Court's Per Curiam opinion held that the Ohio law violated Brandenburg's right to free
speech. The Court used a two-pronged test to evaluate speech acts: (1) speech can be prohibited if it is
"directed at inciting or producing imminent lawless action" and (2) it is "likely to incite or produce
such action." The criminal syndicalism act made illegal the advocacy and teaching of doctrines while
ignoring whether or not that advocacy and teaching would actually incite imminent lawless action. The
failure to make this distinction rendered the law overly broad and in violation of the Constitution.

Miriam College Foundation v. CA, GR 127930, December 15, 2000 - Kaiser


Facts: The members of the editorial board of the Miriam College Foundation’s school paper were
subjected to disciplinary sanction by the College Discipline Committee after letters of complaint were
filed before the Board following the publication of the school paper that contains obscene, vulgar, and
sexually explicit contents. the defendants, wrote to the Committee to transfer the case to the DECS
which they alleged to have the jurisdiction over the issue. Pushing through with the investigation ex
parte the Committee found the defendants guilty and imposed upon them disciplinary sanctions.
Defendants filed before the court for prohibition with preliminary injunction on said decision of the
Committee questioning the jurisdiction of said Discipline Board over the defendants.

Issue: WON the Discipline Board of Miriam College has jurisdiction over the defendants.

Held: The court resolved the issue before it by looking through the power of DECS and the Disciplinary
Committee in imposing sanctions upon the defendants. Section 5 (2), Article XIV of the Constitution
guarantees all institutions of higher learning academic freedom. This institutional academic freedom
includes the right of the school or college to decide for itself, its aims and objectives, and how best to
attain them free from outside coercion or interference save possibly when the overriding public welfare
calls for some restraint. Such duty gives the institution the right to discipline its students and inculcate
upon them good values, ideals and attitude. The right of students to free speech in school is not
always absolute. The court upheld the right of students for the freedom of expression but it does not
rule out disciplinary actions of the school on the conduct of their students. Further, Sec. 7 of the of the
Campus Journalism Act provides that the school cannot suspend or expel a student solely on the basis
of the articles they write EXCEPT when such article materially disrupts class work of involve substantial
disorder or invasion of the rights of others. Therefore the court ruled that the power of the school to
investigate is an adjunct of its power to suspend or expel. It is a necessary corollary to the enforcement
of rules and regulations and the maintenance of a safe and orderly educational environment conducive
to learning. That power, like the power to suspend or expel, is an inherent part of the academic freedom
of institutions of higher learning guaranteed by the Constitution. The court held that Miriam College has
the authority to hear and decide the cases filed against respondent students.

3. Speech and the Electoral Process

Sanidad v. COMELEC – 181 SCRA 529

Facts: This is a petition for certiorari assailing the constitutionality of Section 19 of Comelec Resolution
No. 2167 on the ground that it violates the constitutional guarantees of the freedom of expression and
of the press. Resolution No. 2167, which provides:
Section 19. Prohibition on columnists, commentators or announcers. — During the plebiscite campaign
period, on the day before and on the plebiscite day, no mass media columnist, commentator,
announcer or personality shall use his column or radio or television time to campaign for or against
the plebiscite Issue.

Issue: Whether Section 19 of Comelec Resolution No. 2167 is unconstitutional.

Held: The Supreme Court ruled that Section 19 of Comelec Resolution No. 2167 is unconstitutional. It is
clear from Art. IX-C of the 1987 Constitution that what was granted to the Comelec was the power to
supervise and regulate the use and enjoyment of franchises, permits or other grants issued for the
operation of transportation or other public utilities, media of communication or information to the end
that equal opportunity, time and space, and the right to reply, including reasonable, equal rates
therefor, for public information campaigns and forums among candidates are ensured. The evil sought
to be prevented by this provision is the possibility that a franchise holder may favor or give any undue
advantage to a candidate in terms of advertising space or radio or television time. This is also the
reason why a "columnist, commentator, announcer or personality, who is a candidate for any elective
office is required to take a leave of absence from his work during the campaign period (2nd par.
Section 11(b) R.A. 6646). It cannot be gainsaid that a columnist or commentator who is also a candidate
would be more exposed to the voters to the prejudice of other candidates unless required to take a
leave of absence.

National Press Club v. COMELEC – 207 SCRA 1

Facts: Petitioners in these cases consist of representatives of the mass media which are prevented from
selling or donating space and time for political advertisements. It is principally argued by petitioners
that Section 11 (b) of Republic Act No. 66461 invades and violates the constitutional guarantees
comprising freedom of expression. Petitioners maintain that the prohibition imposed by Section 11 (b)
amounts to censorship, because it selects and singles out for suppression and repression with criminal
sanctions, only publications of a particular content, namely, media-based election or political
propaganda during the election period of 1992.

Issue: Whether Section 11 (b) of Republic Act No. 6646 constitutional.

Held: Yes. It seems a modest proposition that the provision of the Bill of Rights which enshrines freedom
of speech, freedom of expression and freedom of the press has to be taken in conjunction with Article
IX (C) (4) which may be seen to be a special provision applicable during a specific limited period — i.e.,
"during the election period." In our own society, equality of opportunity to proffer oneself for public
office, without regard to the level of financial resources that one may have at one's disposal, is clearly an
important value. One of the basic state policies given constitutional rank by Article II, Section 26 of the
Constitution is the egalitarian demand that "the State shall guarantee equal access to opportunities for
public service and prohibit political dynasties as may be defined by law." The essential question is
whether or not the assailed legislative or administrative provisions constitute a permissible exercise of
the power of supervision or regulation of the operations of communication and information enterprises
during an election period, or whether such act has gone beyond permissible supervision or regulation of
media operations so as to constitute unconstitutional repression of freedom of speech and freedom of
the press. The Court considers that Section 11 (b) has not gone outside the permissible bounds of
supervision or regulation of media operations during election periods.
Adiong v. COMELEC – March 31, 1992

FACTS: On January 13, 1992, the COMELEC promulgated Resolution No. 2347 pursuant to its powers
granted by the Constitution, the Omnibus Election Code, Republic Acts Nos. 6646 and 7166 and other
election laws. Section 15(a) of the resolution provides:

Sec. 15. Lawful Election Propaganda. “ The following are lawful election propaganda:
(a) Pamphlets, leaflets, cards, decals… Provided, That decals and stickers may be posted only in any of
the authorized posting areas provided in paragraph (f) of Section 21 hereof.
Section 21 (f) of the same resolution provides:
Sec. 21(f). Prohibited forms of election propaganda.
It is unlawful:…
(f) To draw, paint, inscribe, post, display or publicly exhibit any election propaganda in any place,
whether public or private, mobile or stationary, except in the COMELEC common posted areas and/or
billboards…

Petitioner Blo Umpar Adiong, a senatorial candidate in the May 11, 1992 elections assails the
COMELEC’s Resolution insofar as it prohibits the posting of decals and stickers in “mobile” places like
cars and other moving vehicles. According to him such prohibition is violative of Section 82 of the
Omnibus Election Code and Section 11(a) of Republic Act No. 6646.

ISSUE: Whether the COMELEC may prohibit the posting of decals and stickers on “mobile” places,
public or private, and limit their location or publication to the authorized posting areas that it fixes.

HELD: YES. The petition is hereby GRANTED. The portion of Section 15 (a) of Resolution No. 2347 of the
COMELEC providing that “decals and stickers may be posted only in any of the authorized posting areas
provided in paragraph (f) of Section 21 hereof” is DECLARED NULL and VOID. The COMELEC’s
prohibition on posting of decals and stickers on “mobile” places whether public or private except in
designated areas provided for by the COMELEC itself is null and void on constitutional grounds. The
prohibition unduly infringes on the citizen’s fundamental right of free speech enshrined in the
Constitution (Sec. 4, Article III). Significantly, the freedom of expression curtailed by the questioned
prohibition is not so much that of the candidate or the political party. The regulation strikes at the
freedom of an individual to express his preference and, by displaying it on his car, to convince others
to agree with him.

Osmena v. COMELEC – 288 SCRA 447

Facts: Emilio Osmena and Pablo Garcia, candidates for public office in the 1998 elections, seek to
invalidate provision of RA6646 (Electoral Reform Law of 1987), which prohibits mass media from
selling or giving free of charge print space or air time for campaign or other political purposes, except
to the COMELEC. They contend that the ban has not only failed to level the playing field, but actually
worked to the grave disadvantage of the poor candidates by depriving them of a medium which they
can afford to pay while their affluent rival scan always resort to other means of reaching voters.

Issue: W/N the ad ban is constitutional.

Held: YES. There is actually no suppression of political ads but only a regulation of time and manner of
advertising. The term political “ad ban” is actually misleading, as although the provision prohibits the
sale or donation of print space and air time to political candidates, it mandates the COMELEC to procure
and itself allocate to the candidates space and time in the media. In this case, there is no total ban on
political ads, much less restriction on the content of the speech.

ABS-CBN v. COMELEC – 323 SCRA 811


Facts: COMELEC passed a resolution issuing a restraining order on ABSCBN from conducting exit polls
after the 1998 elections, upon the belief that such project might conflict with the official COMELEC
count, as well as the unofficial quick count of the Namfrel. ABSCBN prayed for a TRO against the
COMELEC resolution, which was granted by the court. The exit polls were then actually conducted and
reported by the media without any difficulty or problem

Issue: W/N the COMELEC’s absolute ban on exit polling is valid.

Held: NO ( I think YES). The assailed COMELEC resolution is too broad, since its application without
qualification as to whether the polling is disruptive or not. Concededly, the Omnibus Election Code
prohibits disruptive behavior around the voting centers. There is no showing, however, that exit polls
or the means to interview voters cause chaos in voting centers. Neither has any evidence been
presented proving that the presence of exit poll reporters near an election precinct tends to create
disorder or confuse the voters

SWS v. COMELEC – 357 SCRA 496 - JM

Facts: Petitioner SWS and KPC states that it wishes to conduct an election survey throughout the
period of the elections and release to the media the results of such survey as well as publish them
directly. Petitioners argue that the restriction on the publication of election survey results constitutes a
prior restraint on the exercise of freedom of speech without any clear and present danger to justify
such restraint.

Issue: Are the Comelec Resolutions prohibiting the holding of pre-polls and exit polls and the
dissemination of their results through mass media, valid and constitutional?

Ruling: No. The Court held that Section (5)4 is invalid because (1) it imposes a prior restraint on the
freedom of expression, (2) it is a direct and total suppression of a category of expression even though
such suppression is only for a limited period, and (3) the governmental interest sought to be promoted
can be achieved by means other than suppression of freedom of expression.

It has been held that "[mere] legislative preferences or beliefs respecting matters of public convenience
may well support regulation directed at other personal activities, but be insufficient to justify such as
diminishes the exercise of rights so vital to the maintenance of democratic institutions.”

Penera v. COMELEC, GR 181613, November 25, 2009 - JM

Facts: On 11 September 2009, the Supreme Court affirmed the COMELEC’s decision to
disqualify petitioner Rosalinda Penera (Penera) as mayoralty candidate in Sta. Monica, Surigao del
Norte, for engaging in election campaign outside the campaign period, in violation of Section 80 of
Batas Pambansa Blg. 881 (the Omnibus Election Code).
Penera moved for reconsideration, arguing that she was not yet a candidate at the time of the
supposed premature campaigning, since under Section 15 of Republic Act No. 8436 (the law authorizing
the COMELEC to use an automated election system for the process of voting, counting of votes, and
canvassing/consolidating the results of the national and local elections), as amended by Republic Act No.
9369, one is not officially a candidate until the start of the campaign period.

Ruling: Contrary to the assailed Decision, Section 15 of R.A. 8436, as amended, does not
provide that partisan political acts done by a candidate before the campaign period are unlawful, but
may be prosecuted only upon the start of the campaign period. Neither does the law state that
partisan political acts done by a candidate before the campaign period are temporarily lawful, but
becomes unlawful upon the start of the campaign period. Besides, such a law as envisioned in the
Decision, which defines a criminal act and curtails freedom of expression and speech, would be void for
vagueness.

(Note: the case barely touches the constitutional issue under freedom of expression. But from what I
can gather, the acts of Penera were only considered partisan political acts which cannot be made
unlawful before the campaign period. To do so would curtail the freedom of expression.)

4. Commercial Speech

Rubin v. Coors Brewing – 131 L. Ed. 2d 532 - JM

Facts: Because 5(e)(2) of the Federal Alcohol Administration Act (FAAA or Act) prohibits beer labels
from displaying alcohol content, the federal Bureau of Alcohol, Tobacco and Firearms (BATF) rejected
respondent brewer's application for approval of proposed labels that disclosed such content.
Respondent filed suit for relief on the ground that the relevant provisions of the Act violated the First
Amendment's protection of commercial speech. The Government argued that the labeling ban was
necessary to suppress the threat of "strength wars" among brewers, who, without the regulation,
would seek to compete in the marketplace based on the potency of their beer. The District Court
invalidated the labeling ban, and the Court of Appeals affirmed. Although the latter court found that
the Government's interest in suppressing "strength wars" was "substantial" under the test set out in
Central Hudson Gas & Electric Corp. v. Public Serv. Comm'n of N. Y., 447 U.S. 557 , the court held that
the ban violates the First Amendment because it fails to advance that interest in a direct and material
way.

Ruling: Section 5(e)(2) violates the First Amendment's protection of commercial speech. Pp. 3-15.

(a) In scrutinizing a regulation of commercial speech that concerns lawful activity and is not
misleading, a court must consider whether the governmental interest asserted to support the
regulation is "substantial." If that is the case, the court must also determine whether the regulation
directly advances the asserted interest and is no more extensive than is necessary to serve that interest.
Central Hudson, supra, at 566. Here, respondent seeks to disclose only truthful, verifiable, and
nonmisleading factual information Page II concerning alcohol content.

Cincinnati v. Discovery Network – 123 L. Ed. 2d 99 - JM


Facts: Discovery Network, Inc., is engaged in the business of providing adult educational, recreational,
and social programs to individuals in the Cincinnati area. It advertises those programs in a free
magazine that it publishes nine times a year. Although these magazines consist primarily of
promotional material pertaining to. Discovery's courses, they also include some information about
current events of general interest.

Ruling: That "the regulatory scheme advanced by the City of Cincinnati completely prohibiting the
distribution of commercial handbills on the public right of way violates the First Amendment." The
court found that both publications were "commercial speech" entitled to First Amendment protection
because they concerned lawful activity and were not misleading. While it recognized that a city "may
regulate publication dispensing devices pursuant to its substantial interest in promoting safety and
esthetics on or about the public right of way.

Central Hudson Gas v. Public Service Commission of New York, 447 US 557 - JM

Facts: In the winter of 1973-74 there existed an electricity shortage in the State of New York.
Accordingly the Appellee, the Public Service Commission (Appellee), imposed a ban on all advertising
that promotes the use of electricity. By 1976 the electricity shortage subsided, causing the Appellee to
determine whether or not to continue the ban. Upon further inquiry, the Appellee decided to continue
the ban, causing the Appellant, Central Hudson Gas and Electric Corp. (Appellant), to file suit claiming
that the regulation of the Appellee was infringing on their First and Fourteenth Amendment
constitutional rights involving commercial speech.

Ruling: The court ruled that a regulation that completely bans an electric utility from advertising to
promote the use of electricity violates the First and Fourteenth Amendments.

The court instituted a four-step analysis for commercial speech to the Commission's arguments in
support of its ban on promotional advertising:

Is the expression protected by the First Amendment?


For speech to come within that provision, it must concern lawful activity and not be misleading.
Is the asserted governmental interest substantial?
Does the regulation directly advance the governmental interest asserted?
Is the regulation more extensive than is necessary to serve that interest?

Pharmaceutical v. Secretary of Health, GR 173034, October 9, 2007 - JM

Facts: Executive Order No. 51 (Milk Code) was issued by President Corazon Aquino on October 28,
1986 by virtue of the legislative powers granted to the president under the Freedom Constitution. One
of the preambular clauses of the Milk Code states that the law seeks to give effect to Article 11 of the
International Code of Marketing of Breastmilk Substitutes (ICMBS), a code adopted by the World Health
Assembly (WHA) in 1981. From 1982 to 2006, the WHA adopted several Resolutions to the effect that
breastfeeding should be supported, promoted and protected, hence, it should be ensured that
nutrition and health claims are not permitted for breastmilk substitutes. (advertisments and
promotions of breastmilk substitutes were prohibited.)

Ruling: Thus, unlike what has been done with the ICMBS whereby the legislature enacted most of the
provisions into law which is the Milk Code, the subsequent WHA Resolutions, specifically providing for
exclusive breastfeeding from 0-6 months, continued breastfeeding up to 24 months, and absolutely
prohibiting advertisements and promotions of breastmilk substitutes, have not been adopted as a
domestic law.

It is propounded that WHA Resolutions may constitute "soft law" or non-binding norms, principles and
practices that influence state behavior.

City of Laduc v. Gilleo – 129 L. Ed. 2d 36 - Mac

Gilleo placed a sign on her front lawn saying no to war in the Persian gulf and then this was lost so she
replaced it with another sign on her window. There was an ordinance which prohibited this.

WON the ordinance is legal

Ruling: No because it violates the first amendment which includes the prohibition on making a law
that abridges the freedom of speech and of press

5. Libel (Unprotected Speech)

Policarpio v. Manila Times – 5 SCRA 148 – Mac

Policarpio filed a libel suit against Manila Times for publishing two defamatory libelous and false
articles/items about her being sued. The article contained false statements as to the conduct of the
charges against policarpio and this had the effect that policarpio was guilty. In essence, the news article
described policarpio in a worse predicament than she was actually in.

Issue: Whether the expression was libelous

Held: If Manila times knew of this inaccuracy and still printed it then the article is malicious and if they
did not know about the inaccuracy they are still guilty of negligence

Petitioner won the case. The court ordered manila times to pay policarpio

Lopez v. CA – 34 SCRA 116 – Mac

Manila chronicle published the picture of cruz to refer to another person named cruz who created a
hoax about mass murder in Babuyan Islands. The newspaper did make a correction.

WON it is still liable ruling:

Yes because mistake is not an excuse to absolve the publisher because libel is harmful on its face and
correction of error in publishing does not wipe out the responsibility arising from the wrong
publication.
New York Times Co. c. Sullivan – 376 US 254 – Mac

Facts: This case concerns a full-page ad in the New York Times which alleged that the arrest of the Rev.
Martin Luther King, Jr. for perjury in Alabama was part of a campaign to destroy King's efforts to
integrate public facilities and encourage blacks to vote. L. B. Sullivan, the Montgomery city
commissioner, filed a libel action against the newspaper and four black ministers who were listed as
endorsers of the ad, claiming that the allegations against the Montgomery police defamed him
personally. Under Alabama law, Sullivan did not have to prove that he had been harmed; and a
defense claiming that the ad was truthful was unavailable since the ad contained factual errors.
Sullivan won a $500,000 judgment.

Issue: Did Alabama's libel law, by not requiring Sullivan to prove that an advertisement personally
harmed him and dismissing the same as untruthful due to factual errors, unconstitutionally infringe on
the First Amendment's freedom of speech and freedom of press protections?

Ruling: The Court held that the First Amendment protects the publication of all statements, even false
ones, about the conduct of public officials except when statements are made with actual malice (with
knowledge that they are false or in reckless disregard of their truth or falsity). Under this new standard,
Sullivan's case collapsed.

1964 established guidelines for determining whether public officials and public figures could win
damage suits for libel.

To do so, individuals must prove that the defamatory statements were made w/ "actual malice" and
reckless disregard for the truth. The Supreme Court ruled in favor of New York Times.

Rosenbloom v. Metromedia, Inc. – 403 US 254 – Mac

Respondent's radio station, which broadcast news reports every half hour, broadcast news stories of
petitioner's arrest for possession of obscene literature and the police seizure of "obscene books," and
stories concerning petitioner's lawsuit against certain officials alleging that the magazines he distributed
were not obscene and seeking injunctive relief from police interference with his business. These latter
stories did not mention petitioner's name, but used the terms "smut literature racket" and "girlie-book
peddlers." Following petitioner's acquittal of criminal obscenity charges, he filed this diversity action in
District Court seeking damages under Pennsylvania's libel law. The jury found for petitioner and
awarded $25,000 in general damages; and $725,000 in punitive damages, which was reduced by the
court on remittitur to $250,000. The Court of Appeals reversed, holding that the New York Times Co. v.
Sullivan, 376 U.S. 254, standard applied, and "the fact that plaintiff was not a public figure cannot be
accorded decisive significance."

Held: The judgment is affirmed. Pp. 40-62.

MR. JUSTICE BRENNAN, joined by THE CHIEF JUSTICE and MR. JUSTICE BLACKMUN, concluded that the
New York Times standard of knowing or reckless falsity applies in a state civil libel action brought by a
private individual for a defamatory falsehood uttered in a radio news broadcast about the individual's
involvement in an event of public or general interest. Pp. 40-57.

MR. JUSTICE BLACK concluded that the First Amendment protects the news media from libel judgments
even when statements are made with knowledge that they are false. P. 57.

MR. JUSTICE WHITE concluded that, in the absence of actual malice as defined in New York Times,
supra, the First Amendment gives the news media a privilege to report and comment upon the official
actions of public servants in full detail, without sparing from public view the reputation or privacy of an
individual involved in or affected by any official action. Pp. 59-62.
Shortcut version- (1971) Rosenbloom published a nudist magazine and Metromedia radio station
defamed Rosenbloom. Rosenbloom sued for libel and won but the Supreme Court reversed the decision
extending the rule of "actual malice" to public issues even when private persons are involved- this
decision was later reversed in the Gertz v. Welch landmark case that protected private people.

Gerts v. Robert Wlech – 418 US 323 – Mac

The court set a new standard for determining whether a person is a public figure and determined that
the plaintiff, Elmer Gertz, was a private citizen and not a public figure in this action.
The court said that a private citizen does not have to meet the actual malice standard of Times v.
Sullivan to recover damages.

A private citizen may sue for actual injury, which the court said includes impairment of reputation,
personal humiliation and mental anguish, as well as financial loss. Punitive damages, the court said,
can be awarded upon a showing of actual malice.

The decision allows each state to determine the degree of fault that must be proved by the plaintiff.
What has emerged as the most common standard is negligence. The best definition of negligence is a
departure from normal publishing standards or procedures.
Justice Lewis F. Powell Jr. wrote the opinion in a 5-4 decision.

Hustler v. Magazine – 485 US 46 – Kaye

Facts:
A lead story in the November 1983 Issue of Hustler Magazine featured a "parody" of an
advertisement, modeled after an actual ad campaign, claiming that Falwell, a Fundamentalist minister
and political leader, had a drunken incestuous relationship with his mother in an outhouse. Falwell
sued to recover damages for libel, invasion of privacy, and intentional infliction of emotional distress.
Falwell won a jury verdict on the emotional distress claim and was awarded a total of $150,000 in
damages. Hustler Magazine appealed.

Issue:
Whether freedom of speech protection is extended to the making of patently offensive
statements about public figures, resulting perhaps in their suffering emotional distress?

Held:
Yes. In a unanimous opinion the Court held that public figures, such as Jerry Falwell, may not
recover for the intentional infliction of emotional distress without showing that the offending
publication contained a false statement of fact which was made with "actual malice." The Court added
that the interest of protecting free speech, under the First Amendment, surpassed the state's interest
in protecting public figures from patently offensive speech, so long as such speech could not
reasonably be construed to state actual Facts about its subject.

In Re Jurado AM No. 90-5-2373, 4 LR 19 Aug’09


In Re Jurado – 243 SCRA 299
Facts:
This proceeding treats of Emiliano P. Jurado, a journalist who writes in a newspaper of general
circulation, the "Manila Standard." He describes himself as a columnist, who "incidentally happens to
be a lawyer," remarking that while he values his membership in the law profession, "such membership
is neither a critical nor indispensable adjunct in the exercise of his occupation as a newspaperman."His
column in the "Manila Standard" is entitled "Opinion."

Jurado had been writing about alleged improprieties and irregularities in the judiciary over several
months. Other journalists had also been making reports or comments on the same subject. At the same
time, anonymous communications were being extensively circulated, by hand and through the mail,
about alleged venality and corruption in the courts. And all these were being repeatedly and
insistently adverted to by certain sectors of society.

In light of these abnormal developments, the Chief Justice took an extraordinary step. He Issued
Administrative Order No. 11-93 dated January 25, 1993, "Creating an Ad Hoc Committee to Investigate
Reports of Corruption in the Judiciary.

Issue:
Whether the Judges attacked have the remedy in action for libel.

Held:
It might be suggested that judges who are unjustly attacked have a remedy in an action for libel. This
suggestion has, however, no rational basis in principle. In the first place, the outrage is not directed to
the judge as a private individual but to the judge as such or to the court as an organ of the
administration of justice. In the second place, public interests will gravely suffer where the judge, as
such, will, from time to time, be pulled down and disrobed of his judicial authority to face his assailant
on equal grounds and prosecute cases in his behalf as a private individual. The same reasons of public
policy which exempt a judge from civil liability in the exercise of his judicial functions, most
fundamental of which is the policy to confine his time exclusively to the discharge of his public duties,
applies here with equal, if not superior, force [Hamilton v. Williams, 26 Ala. 529; Busteed v. Parson, 54
Ala. 403; Ex parte McLeod, 120 Fed. 130; Coons v. State, 191 Ind. 580; 134 N. E. 194].

WHEREFORE, the Court declares Atty. Emil [Emiliano] P. Jurado guilty of contempt of court and
in accordance with Section 6, Rule 71 of the Rules of Court, hereby sentences him to pay a fine of one
thousand pesos [P1,000,00].

Vasquez v. CA – GR 118971 Sept. 15, 1999

Facts:
Petitioner Rodolfo R. Vasquez is a resident of the Tondo Foreshore Area. Sometime in April 1986, he
and some 37 families from the area went to see then National Housing Authority (NHA) General
Manager Lito Atienza regarding their complaint against their Barangay Chairman, Jaime Olmedo, a
public official. After their meeting with Atienza and other NHA officials, petitioner and his companions
were met and interviewed by newspaper reporters at the NHA compound concerning their complaint.
The next day, April 22, 1986, the following exerpts of the news article appeared in the newspaper Ang
Tinig ng Masa. In the article, published were supposed allegations by Vasquez that (1)
“nakipagsabwatan umano si Chairman Jaime Olmedo upang makamkam ang may 14 na lote ng lupa”;
(2) ang mga lupa ay ilegal na patituluhan, nagawa ito ni Olmedo sa pakikipagsabwatan sa mga project
manager at legal officers ng NHA; (3) kasangkot din umano si Olmedo sa mga ilegal na pasugalan sa
naturang lugar at maging sa mga nakawan ng manok. x x x”
Based on the newspaper article, Olmedo filed a complaint for libel against petitioner alleging that the
latter’s statements cast aspersions on him and damaged his reputation.
On May 28, 1992, the trial court rendered judgment finding petitioner guilty of libel and sentencing him
to pay a fine of P1,000.00. On appeal, the Court of Appeals affirmed in toto. Hence, this petition for
review.

Issue:
Whether the actual malice standard in New York Times versus Sullivan is to be applied in prosecutions
for criminal libel.

Held:
The standard of actual malice in New York Times versus Sullivan is to be applied in criminal
prosecutions for libel. For that matter, even if the defamatory statement is false, no liability can attach
if it relates to official conduct, unless the public official concerned proves that the statement was
made with actual malice — that is, with knowledge that it was false or with reckless disregard of
whether it was false or not. In this case, the prosecution failed to prove not only that the charges made
by petitioner were false but also that petitioner made them with knowledge of their falsity or with
reckless disregard of whether they were false or not. A rule placing on the accused the burden of
showing the truth of allegations of official misconduct ana/or good motives and justifiable ends for
making such allegations would not only be contrary to Art. 361 of the Revised Penal Code. It would,
above all, infringe on the constitutionally guaranteed freedom of expression. Instead of the claim that
petitioner was politically motivated in making the charges against complainant, it would appear that
complainant filed this case to harass petitioner. It is curious that the ones most obviously responsible for
the publication of the allegedly offensive news report, namely, the editorial staff and the periodical
itself, were not at all impleaded. The charge was leveled against the petitioner and, "curiouser" still, his
clients who have nothing to do with the editorial policies of the newspaper.

Borjal v. CA – GR. 126466 Jan. 14, 1999

Facts:
A civil action for damages based on libel was filed before the court against Borjal and Soliven for
writing and publishing articles that are allegedly derogatory and offensive against Francisco
Wenceslao, attacking among others the solicitation letters he send to support a conference to be launch
concerning resolving matters on transportation crisis that is tainted with anomalous activities.
Wenceslao however was never named in any of the articles nor was the conference he was organizing.
The lower court ordered petitioners to indemnify the private respondent for damages which was
affirmed by the Court of Appeals. A petition for review was filed before the SC contending that private
respondent was not sufficiently identified to be the subject of the published articles.

Issue:
Whether there are sufficient grounds to constitute guilt of petitioners for libel.

Held:
In order to maintain a libel suit, it is essential that the victim be identifiable although it is not
necessary that he be named. It is also not sufficient that the offended party recognized himself as the
person attacked or defamed, but it must be shown that at least a third person could identify him as
the object of the libelous publication. These requisites have not been complied with in the case at bar.
The element of identifiability was not met since it was Wenceslaso who revealed he was the organizer
of said conference and had he not done so the public would not have known.

The concept of privileged communications is implicit in the freedom of the press and that privileged
communications must be protective of public opinion. Fair commentaries on matters of public interest
are privileged and constitute a valid defense in an action for libel or slander. The doctrine of fair
comment means that while in general every discreditable imputation publicly made is deemed false,
because every man is presumed innocent until his guilt is judicially proved, and every false imputation is
deemed malicious, nevertheless, when the discreditable imputation is directed against a public person
in his public capacity, it is not necessarily actionable. In order that such discreditable imputation to a
public official may be actionable, it must either be a false allegation of fact or a comment based on a
false supposition. If the comment is an expression of opinion, based on established Facts, then it is
immaterial that the opinion happens to be mistaken, as long as it might reasonably be inferred from the
Facts.

The questioned article dealt with matters of public interest as the declared objective of the
conference, the composition of its members and participants, and the manner by which it was intended
to be funded no doubt lend to its activities as being genuinely imbued with public interest. Respondent
is also deemed to be a public figure and even otherwise is involved in a public Issue. The court held that
freedom of expression is constitutionally guaranteed and protected with the reminder among media
members to practice highest ethical standards in the exercise thereof.

Vicario v. CA – GR 124491 June 1, 1999

Facts:
Petitioner was charged with libel by the Provincial Prosecutor of Catarman, Northern Samar,
with Judge Proceso Sidro of the Municipal Circuit Trial Court of Mondragon San Roque, Northern
Samar, as complaining witness. According to the Information, the crime was committed when Vicario
allegedly distributed and circulated in the vicinity of the Northern Samar Provincial Hospital in
Catarman photocopies of page 7 of the 20 March 1992 Issue of the Philippine Daily Inquirer which
contained the following article:

SAMAR JUDGE WHO POCKETED BOND CHARGED WITH GRAFT


OMBUDSMAN Conrado Vasquez yesterday filed with the Sandiganbayan graft charges against a
Northern Samar judge who pocketed the P1,000.00 cash bond posted by a respondent in one of several
cases pending in his sala.
Charged was Judge Proceso Sidro of the Northern Samar municipal circuit trial court in Mondragon.
Investigation showed that Sidro failed to deposit the cash bond with his clerk-of-court, and refused to
return the money even after the accused who filed the bond was already acquitted in the case.

Private complainant Sidro alleged that petitioner's act greatly prejudiced his reputation as a
member of the bench and caused him great distress. Petitioner Vicario on the other hand disclaimed
responsibility for the distribution of the alleged libelous article, at the same time asserting that the libel
suit against him was ill-motivated for he had filed a criminal charge for graft and corruption against
Judge Sidro before the Ombudsman and an administrative complaint for dishonesty with the Supreme
Court, both due to the latter's unjustified refusal and failure to return petitioner's cash bond of
P1,000.00.

Issue:
WON the act of merely distributing a photocopy of an article in a newspaper reporting that
graft charges had been filed against a judge named therein constitutes libel?.

Held:
YES. A person's liability for libel need not, admittedly, stem from the fact that he was the
original publisher of the discreditable act. The maker of a libelous republication or repetition, although
not liable for the results of the primary publication, is liable for the consequence of a subsequent
publication which he makes or participates in making. It is no justification that the defamatory matter
is previously published by a third person, provided malice is present.

However, in this case, there was no evidence at all offered to show that petitioner himself
photocopied the article. Nor was evidence sufficiently adduced to prove that he himself distributed
photocopies of the news item to so many people. It was therefore reversible error of the courts below
to conclude that petitioner was liable for the republication of the news article alleged to be libelous.

Granting arguendo the correctness of the finding by the lower courts that petitioner did at least
distribute a machine copy of the article to one Amador Montes, an acknowledged "batos" of Judge
Sidro, was there sufficient basis to ascribe malice in his act?

It is established doctrine that the malice that attends the dissemination of the article alleged to
be libelous must attend the distribution itself. It cannot be merely a resentment against a person,
manifested unconnectedly several months earlier or one displayed at a much later date, as what
happened in this case. A fine-tooth comb dissection of the testimony of prosecution witness Amador
Montes reveals none that would indicate, much less hint at, the attitude and mental frame of Vicario at
the time he allegedly handed over the photocopy of the news item in question to Montes. If at all, as
can be gathered from the testimonial narration, Vicario's attitude could only be described as
noncommittal.

In order to constitute malice, ill will must be personal. So if the ill will is engendered by one's
sense of justice or other legitimate or plausible motive, such feeling negatives actual malice. The anger
observed by trial court to have been shown by the petitioner towards private complainant at the time
the former offered his testimony in defense of libel cannot be properly considered as malice, either in
fact or in law, that accompanied the dissemination of an alleged libelous publication. For the anger
discerned of petitioner on the witness stand could also mean anger not only borne out of a sense of
justice frustrated by the continued refusal of Judge Sidro to return to him his cash bond, but also at
being criminally sued in court for an act which he stoutly believed was not imputable to him. This state
of mind cannot be appropriately considered malice and applied retroactively to the time of the
distribution of the alleged libelous article unless clear and convincing evidence shows otherwise; and,
there is no such contrary evidence in the case at bar. Since there is no indication about the cause of such
display of "intense hatred" by the petitioner for Judge Sidro, the Court will grant him the benefit of the
doubt under the "equipoise doctrine."
There was nothing defamatory in the news item. This much was found by the trial court itself,
noting that the published article was merely a factual report about the filing by the Ombudsman of the
charge of corruption against the judge with the Sandiganbayan. Of course, it does not necessarily mean
that if the news article complained of is not libelous because it is a privileged matter, he who repeats
the publication is likewise free from accountability for the re-utterance. We recognize that a person's
liability for libel does not necessarily proceed from the fact that he was the original publisher of the
discreditable act. The maker of a libelous republication or repetition, although not liable for the results
of the primary publication, is liable for the consequences of a subsequent publication which he makes or
participates in making so long as the elements of libel are satisfied. But in every case malice must be
present, something which has not been shown in the case at bar.

The law presumes that malice is present in every defamatory imputation. However, on this
score, Art. 354 of the Revised Penal Code provides an exemption —

Art. 354. Requirement for publicity. — Every defamatory imputation is presumed to be malicious, even if
it be true, if no good intention and justifiable motive for making it is shown, except in the following
cases . . . . 2. A fair and true report, made in good faith, without any comments or remarks, of any
judicial, legislative or other official proceedings which are not of confidential nature, or of any
statement, report or speech delivered in said proceedings, or of any other act performed by public
officers in the exercise of their functions (emphasis supplied).
Paragraph 2 aforequoted refers to a qualifiedly privileged communication, the character of which is a
matter of defense that may be lost by positive proof of express malice on the part of the accused. Once
it is established that the article is of a privileged character, the onus of proving actual malice rests on the
plaintiff who must then convince the court that the offender was prompted by malice or ill will. When
this is accomplished the defense of privilege becomes unavailing. Since the prosecution failed to
establish express malice on the part of petitioner by positive proof, its cause perforce must fail.

WHEREFORE, this petition is GRANTED. The decision of the Regional Trial Court of Catarman,
Northern Samar, dated 11 March 1993 finding petitioner guilty of libel, and that of the Court of Appeals
dated 28 February 1996 affirming his conviction are REVERSED and SET ASIDE. Petitioner ROQUE
VICARIO Y MENDEZ is ACQUITTED of the crime charged.

Pader v. People – 325 SCRA 117 - Xavier


FACTS: Atty. Benjamin C. Escolango was conversing with his political leaders at the terrace of his house
at Morong, Bataan when petitioner appeared at the gate and shouted “putang ina mo Atty.
Escolango. Napakawalanghiya mo!” The latter was dumbfounded and embarrassed. At that time, Atty.
Escolango was a candidate for vice mayor of Morong, Bataan in the elections of May 8, 1995. MCTC,
Bagac, Bataan rendered decision convicting petitioner of grave oral defamation.
ISSUE: whether petitioner is guilty of slight or serious oral defamation.
HELD: YES. Unquestionably, the words uttered were defamatory. However, the factual backdrop of
the case, the oral defamation was only slight. In resolving the issue, we are guided by a doctrine of
ancient respectability that defamatory words will fall under one or the other, depending not only upon
their sense, grammatical significance, and accepted ordinary meaning judging them separately, but
also upon the special circumstances of the case, antecedents or relationship between the offended
party and the offender, which might tend to prove the intention of the offender at the time.
The expression “putang ina mo” is a common enough utterance in the dialect that is often
employed, not really to slander but rather to express anger or displeasure. Somehow, the trial court
failed to appreciate the fact that the parties were also neighbors; that petitioner was drunk at the time
he uttered the defamatory words; and the fact that petitioner’s anger was instigated by what Atty.
Escolango did when petitioner’s father died. In which case, the oral defamation was not of serious or
insulting nature.
Fermin v. People, GR 157643, March 28, 2008 - Xavier
FACTS: Spouses Annabelle Rama Gutierrez and Eduardo (Eddie) Gutierrez, filed two criminal
complaints for libel against Cristinelli (“Cristi”) Fermin, publisher of Gossip Tabloid, and Bogs Tugas,
editor-in-chief of Gossip Tabloid, where the RTC convicted them for publication of a libelous article in
Gossip Tabloid which reads that Spouses Gutierrez (1) converted for their personal use the money paid
to them by fellow Filipinos in America in their business of distributing high-end cookware); (2) returned
to the Philippines to evade prosecution in America; and (3) that Annabelle Rama Gutierrez lost the
earnings from their business through irresponsible gambling in casinos.

CA affirmed the conviction of petitioner, but acquitted Tugas on account of non-participation in


the publication of the libelous article. In the SC, Petitioner posits that, to sustain a conviction for libel
under Article 360 of the Revised Penal Code, it is mandatory that the publisher knowingly participated
in or consented to the preparation and publication of the libelous article.
ISSUE: Whether the subject article in the June 14, 1995 issue of Gossip Tabloid is not libelous, is covered
by the mantle of press freedom, and is merely in the nature of a fair and honest comment.
HELD: NO. In determining whether a statement is defamatory, the words used are to be construed in
their entirety and should be taken in their plain and ordinary meaning as they would naturally be
understood by persons reading them, unless it appears that they were used and understood in another
sense. There is (1) evident imputation of the crime of malversation; (2) of vices or defects for being
fugitives from the law; and (3) of being a wastrel. The attribution was made publicly, considering
that Gossip Tabloid had a nationwide circulation. The victims were identified and identifiable. More
importantly, the article reeks of malice, as it tends to cause the dishonor, discredit, or contempt of the
complainants.
Further, Fermin’s defense that proof of knowledge of and participation in the publication of the
offending article is not required, if the accused has been specifically identified as “author, editor, or
proprietor” or “printer/publisher” of the publication, as petitioner and Tugas are in this case. In the
same vein, we note that the CA erred in acquitting Tugas. Moreover, his alibi of confinment at the
Mother of Perpetual Help Clinic in Angeles City, is unavailing, in view of the testimony of his attending
physician that Tugas’ medical condition did not prevent him from performing his work. But, of course,
we cannot reinstate the ruling of the trial court convicting Bogs Tugas because with his acquittal by the
CA, we would run afoul of his constitutional right against double jeopardy.

6. Obscenity (Unprotected Speech)

Miller v. California – 37 L. Ed. 2d 419 – Xavier


FACTS: Defendant Miller was convicted under the California Penal Code for mailing advertisements for
“adult” material to non-soliciting recipients. In determining whether speech is obscene, the basic
guidelines for the trier of fact must be: (a) whether “the average person, applying contemporary
community standards” would find the material, taken as a whole, appeals to the prurient interest of
sex, (b) whether the work depicts or describes, in a patently offensive way, sexual conduct specifically
defined by the applicable state law, and (c) whether the work, taken as a whole, lacks serious literacy,
artistic, political, or scientific value.
ISSUE: Whether state statutes may regulate obscene material without limits.
HELD: NO. In determining whether speech is obscene, the basic guidelines for the trier of fact must be:
(a) whether “the average person, applying contemporary community standards” would find the
material, taken as a whole, appeals to the prurient interest of sex, (b) whether the work depicts or
describes, in a patently offensive way, sexual conduct specifically defined by the applicable state law,
and (c) whether the work, taken as a whole, lacks serious literacy, artistic, political, or scientific value.

The Supreme Court of the Untied States (Supreme Court) does not adopt as a constitutional
standard the “utterly without redeeming social value” test. If a state law that regulates obscene
material is thus limited, as written or construed, First Amendment constitutional values are
adequately protected by the ultimate power of appellate courts to conduct an independent review of
constitutional claims when necessary.
Gonzales v. Kalaw-Katigbak – 137 SCRA 717 - Xavier
FACTS: Petitioner was the producer of the movie Kapit sa Patalim which the Board of Review for
Motion Pictures and Televisions allowed on condition that certain deletions were made and that it was
shown on adults only. The petitioner brought an action, claiming violation of their freedom of
expression.
ISSUE: Whether cencorship would constitute violation of freedom of expression
HELD: YES. Motion pictures are important both as a method for the communication of ideas and the
expression of the artistic impulse. The power of the Board is limited to the classification of films. For
freedom of expression is the rule and restrictions the exception. The power to impose prior restraint is
not to be presumed, rather the presumption is against its validity. Censorship is allowable only under
the clearest proof of a clear and present danger of a substantive evil to public safety, public morals,
public health or any other legitimate public interest. The Board committed an abuse of discretion in
subjecting petitioner to difficulty and travail before the movie was classified as "For adults only" without
deletion. However there is not enough votes to consider the abuse of discretion grave as it explained
that there were reasons for its action because of the scenes showing women erotically dancing naked
and kissing and caressing each other like lesbians.
Pita v. CA – 178 SCRA 362 – Xavier
FACTS: An Anti-Smut Campaign initiated by the Mayor of the City of Manila, Ramon D. Bagatsing, seized
and confiscated magazines, publications and other reading materials believed to be obscene,
pornographic and indecent and later burned the seized materials in public at the University belt along
C.M. Recto Avenue, Manila. Among the publications seized, and later burned, was "Pinoy Playboy"
magazines published and co-edited by plaintiff Leo Pita.
Plaintiff claimed that the magazine is a decent, artistic and educational magazine which is not
per se obscene, and that the publication is protected by the Constitutional guarantees of freedom of
speech and of the press.
ISSUE: Whether the magazines in question are obscene, pornographic and indecent thus not covered by
freedom of speech and of the press.
HELD: NO. Ultimately "whether a picture is obscene or indecent must depend upon the circumstances
of the case and that the question is to be decided by the "judgment of the aggregate sense of the
community reached by it." The government authorities in the instant case have not shown the required
proof to justify a ban and to warrant confiscation of the literature. There was no lawful court order: (1)
finding the said materials to be pornography, and (2) authorizing them to carry out a search and
seizure, by way of a search warrant.
Freedom of the press is not without restraint as the state has the right to protect society from
pornographic literature that is offensive to public morals, as indeed we have laws punishing the author,
publishers and sellers of obscene publications. Using the Kottinger rule: the test of obscenity is
"whether the tendency of the matter charged as obscene, is to deprave or corrupt those whose minds
are open to such immoral influences and into whose hands a publication or other article charged as
being obscene may fall." Another is whether it shocks the ordinary and common sense of men as an
indecency.

Barnes v. Glen Theater – 498 US 439 - Xavier


FACTS: Glen Theatre and the Kitty Kat Lounge in South Bend, Indiana, operated entertainment
establishments with totally nude dancers. An Indiana law regulating public nudity required
dancers to wear "pasties" and a "G-string" when they perform. The Theatre and Lounge sued
to stop enforcement of the statute.

ISSUE: Whehter a state prohibition against complete nudity in public places violate the First
Amendment's freedom of expression guarantee.

HELD: NO. Chief Justice Rehnquist, in a plurality opinion, conceded that nude dancing was a
form of expressive activity. But he maintained that the public indecency statute is justified
despite the incidental limitations on such expressive activity. The statute "furthers a
substantial government interest in protecting order and morality." The proscription on public
nudity is unrelated to the erotic message the dancers seek to convey.

FCC v Pacifica Foundation – 438 US 726 - JC

Facts: During a mid-afternoon weekly broadcast, a New York radio station aired George Carlin's
monologue, "Filthy Words." Carlin spoke of the words that could not be said on the public airwaves.
His list included shit, piss, fuck, cunt, cocksucker, motherfucker, and tits. The station warned listeners
that the monologue included "sensitive language which might be regarded as offensive to some." The
FCC received a complaint from a man who stated that he had heard the broadcast while driving with
his young son.

Issue: Whether the government may regulate the right to free speech

Held: Yes. The Court held that limited civil sanctions could constitutionally be invoked against a radio
broadcast of patently offensive words dealing with sex and execration. The words need not be obscene
to warrant sanctions. Audience, medium, time of day, and method of transmission are relevant
factors in determining whether to invoke sanctions.

Renton v. Playtime Theater – 475 US 41 - JC

Facts: The city of Renton, Washington, enacted a zoning ordinance that prohibited adult motion
picture theaters from locating within 1,000 feet of "any residential zone, single- or multiple-family
dwelling, church, park, or school." Playtime Theatres, Inc., challenged the ordinance and sought a
permanent injunction against its enforcement.

Issue: Whether the ordinance violated free speech


Held: No. The Court held that the zoning ordinance did not violate the First and Fourteenth
Amendments. The Court held that the ordinance was a form of “Content Neutral” time, place, and
manner regulation, not a ban on adult theaters altogether. The Court reasoned that the law was not
aimed at the content of the films shown at adult motion picture theaters, "but rather the secondary
effects of such theaters on the surrounding community." The Court found that the ordinance was
designed to serve a substantial governmental interest in preserving the quality of life and allowed for
"reasonable alternative avenues of communication."

Bethel School District v. Fraser – 478 US 675 - JC

Facts: At a school assembly of approximately 600 high school students, Matthew Fraser made a
speech nominating a fellow student for elective office. In his speech, Fraser used what some observers
believed was a graphic sexual metaphor to promote the candidacy of his friend. As part of its
disciplinary code, Bethel High School enforced a rule prohibiting conduct which "substantially
interferes with the educational process . . . including the use of obscene, profane language or gestures."
Fraser was suspended from school for two days.

Issue: Whether the right to free speech prevents a school from punishing a student for giving a lewd
speech

Held: No. The First Amendment did not prevent the School District from disciplining respondent for
giving the offensively lewd and indecent speech at the assembly. Under the First Amendment, the use
of an offensive form of expression may not be prohibited to adults making what the speaker considers
a political point, but it does not follow that the same latitude must be permitted to children in a public
school. It is a highly appropriate function of public school education to prohibit the use of vulgar and
offensive terms in public discourse. Nothing in the Constitution prohibits the states from insisting that
certain modes of expression are inappropriate and subject to sanctions. The inculcation of these values
is truly the work of the school, and the determination of what manner of speech is inappropriate
properly rests with the school board.

Hazelwood School District v. Kuhlmeier – 484 US 260 - JC

Facts: The Spectrum, the school-sponsored newspaper of Hazelwood East High School, was written and
edited by students. In May 1983, Robert E. Reynolds, the school principal, received the pages proofs
for the May 13 issue. Reynolds found two of the articles in the issue to be inappropriate, and ordered
that the pages on which the articles appeared be withheld from publication. Cathy Kuhlmeier and two
other former Hazelwood East students brought the case to court.

Issue: Whether the deletion of the articles violated the right to free speech

Held: No. The Court held that the First Amendment did not require schools to affirmatively promote
particular types of student speech. The Court held that schools must be able to set high standards for
student speech disseminated under their auspices, and that schools retained the right to refuse to
sponsor speech that was "inconsistent with 'the shared values of a civilized social order.'" Educators
did not offend the First Amendment by exercising editorial control over the content of student speech
so long as their actions were "reasonably related to legitimate pedagogical concerns." The actions of
principal Reynolds, the Court held, met this test.

Fernando v. CA, GR 159751, December 6, 2006 - JC

Facts: Petitioners were charged and convicted of violation of Art 201 of the RPC for selling magazines
and VHS tapes depicting acts of sexual intimacy.

Issue: Whether the State may regulate obscenity

Held: Yes. As obscenity is an unprotected speech which the State has the right to regulate, the State in
pursuing its mandate to protect, as parens patriae, the public from obscene, immoral and indecent
materials must justify the regulation or limitation.

One such regulation is Article 201 of the Revised Penal Code. To be held liable, the prosecution must
prove that (a) the materials, publication, picture or literature are obscene; and (b) the offender sold,
exhibited, published or gave away such materials. Necessarily, that the confiscated materials are
obscene must be proved.

Soriano v. Laguardia, GR 164785, April 29, 2009 - JC

Facts: Petitioner uttered on his show, Ang Dating Daan, certain cuss words which prompted MTRCB to
suspend the airing of his show per PD 1896.

Issue: Whether his constitutional guarantee of religion, speech and expression were violated

Held: No. The Court rules otherwise. It has been established in this jurisdiction that unprotected speech
or low-value expression refers to libelous statements, obscenity or pornography, false or misleading
advertisement, insulting or “fighting words”, i.e., those which by their very utterance inflict injury or
tend to incite an immediate breach of peace and expression endangering national security.

The Court finds that petitioner’s statement can be treated as obscene, at least with respect to the
average child. Hence, it is, in that context, unprotected speech. In Fernando v. Court of Appeals, the
Court expressed difficulty in formulating a definition of obscenity that would apply to all cases, but
nonetheless stated the ensuing observations on the matter:

There is no perfect definition of “obscenity” but the latest word is that of Miller v. California which
established basic guidelines, to wit: (a) whether to the average person, applying contemporary
standards would find the work, taken as a whole, appeals to the prurient interest; (b) whether the
work depicts or describes, in a patently offensive way, sexual conduct specifically defined by the
applicable state law; and (c) whether the work, taken as a whole, lacks serious literary, artistic,
political, or scientific value. But, it would be a serious misreading of Miller to conclude that the trier of
facts has the unbridled discretion in determining what is “patently offensive.” x x x What remains clear is
that obscenity is an issue proper for judicial determination and should be treated on a case to case basis
and on the judge’s sound discretion.[35]
Following the contextual lessons of the cited case of Miller v. California, a patently offensive utterance
would come within the pale of the term obscenity should it appeal to the prurient interest of an
average listener applying contemporary standards.

7. Assembly and Petition

NAVARRO vs. CITY MAYOR ANTONIO J. VILLEGAS, [G.R. No. L-31687 February 26, 1970]

Facts: the petitioner, acting in behalf of the Movement of a Democratic Philippines, wrote a letter to
the respondent, the Mayor of the city of Manila, applying to hold a rally at Plaza Miranda February 26,
1970, from 4-11pm. On the same day, the respondent wrote a reply, denying his request on the
grounds that, they have temporarily adopted the policy of not issuing any permit for the use of Plaza
Miranda for rallies or demonstration during weekdays due to certain events. On the same letter, the
respondent gave the petitioner an option to use the Sunken Garden for its rally, and for it to be held
earlier for it to end before dark. The petitioner filed suit contesting the Mayor’s action on the ground
that it violates the petitioner’s right to peaceably assemble and petition the government for redress
of grievances.

Issue: Whether the respondents act on denying the request of the petitioner violates the petitioners’
Right to peaceable assembly.

Ruling: No. The right of peaceable assemble is subject to regulation under the police power of the
state. The right to freedom of speech and peaceful assembly, though granted by the Constitution, is
not absolute for it may be regulated in order that it may not be injurious to the equal enjoyment of
others having an equal right of community and society, This power may be exercised under the police
power of the state, which is the power of the state, which is the power to prescribe regulations to
promote the health, morals, peace, education, and good order, safety and general welfare of the
people. While the privilege of the citizen to use streets and parks for communication may be regulated
in the interest of all, said privilege is not absolute. It must be exercised insubordination to the general
comfort and convenience and in consonance with peace and good order, but it must not guise of
regulation be abridged or denied.

PHILIPPINE BLOOMING MILLS EMPLOYMENT ORGANIZATION, vs. PHILIPPINE BLOOMING MILLS CO.,
INC. [G.R. No. L-31195 June 5, 1973]

Facts: Philippine Blooming Employees Organization (PBMEO) decided to stage a mass demonstration in
front of Malacañang to express their grievances against the alleged abuses of the Pasig Police.

After learning about the planned mass demonstration, Philippine Blooming Mills Inc., called for a
meeting with the leaders of the PBMEO. During the meeting, the planned demonstration was
confirmed by the union. But it was stressed out that the demonstration was not a strike against the
company but was in fact an exercise of the laborers inalienable constitutional right to freedom of
expression, freedom of speech and freedom for petition for redress of grievances.

The company asked them to cancel the demonstration for it would interrupt the normal course of
their business which may result in the loss of revenue. This was backed up with the threat of the
possibility that the workers would lose their jobs if they pushed through with the rally.
A second meeting took place where the company reiterated their appeal that while the workers may be
allowed to participate, those from the 1st and regular shifts should not absent themselves to
participate, otherwise, they would be dismissed. Since it was too late to cancel the plan, the rally took
place and the officers of the PBMEO were eventually dismissed for a violation of the ‘No Strike and No
Lockout’ clause of their Collective Bargaining Agreement.

Issue: Whether the workers who joined the strike may be dismissed for violation of the CBA.

Ruling: No. While the Bill of Rights also protects property rights, the primacy of human rights over
property rights is recognized. Because these freedoms are "delicate and vulnerable, as well as
supremely precious in our society" and the "threat of sanctions may deter their exercise almost as
potently as the actual application of sanctions," they "need breathing space to survive," permitting
government regulation only "with narrow specificity."

Property and property rights can be lost thru prescription; but human rights are imprescriptible. In the
hierarchy of civil liberties, the rights of free expression and of assembly occupy a preferred position as
they are essential to the preservation and vitality of our civil and political institutions; and such
priority "gives these liberties the sanctity and the sanction not permitting dubious intrusions."

The freedoms of speech and of the press as well as of peaceful assembly and of petition for redress of
grievances are absolute when directed against public officials or "when exercised in relation to our right
to choose the men and women by whom we shall be governed.”

JOSE B.L. REYES, in behalf of the ANTI-BASES COALITION (ABC) vs. RAMON BAGATSING, [G.R. No. L-
65366 November 9, 1983]

Facts: retired Justice Reyes in behalf of the Anti-Bases Coalition, sought a rally permit from Luneta Park
to front gate of the US Embassy in Manila. Mayor Bagatsing denied the petition. He issued City
Ordinance No. 7295 to prohibit rallying 500 meter radius around the Embassy.

Issue:Whether the Mayor violated the petitioners' constitutional right.

Ruling: Yes, the mayor's ordinance which prohibit the petitioners to rally violates their constitutional
right to free speech and peaceable assembly. The invocation of the right to freedom of peaceable
assembly carries with it the implication that the right to free speech has likewise been disregarded. It is
settled law that as to public places, especially so as to parks and streets, there is freedom of access.
Nor is their use dependent on who is the applicant for the permit, whether an individual or a group.
There can be no legal objection, absent of clear and present danger of a substantive evil, on the choice
of Luneta and US Embassy as the place for the peaceful rally. Time immemorial Luneta has been used
for purposes of assembly, communicating thoughts between citizens, and discussing public questions.
Moreover, denial of permit for a rally in front of the US Embassy is justified only in the presence of a
clear and present danger to life or property of the embassy.

While the General rule is that a permit should recognize the right of the applicants to hold their
assembly at a public place of their choice, another place may be designated by the licensing authority
if it be shown that there is a clear and present danger of a substantive evil if no such change were
made.
PHILIPPINE COMMERCIAL & INDUSTRIAL BANK vs. PHILNABANK EMPLOYEES' ASSOCIATION [G.R. No.
L-29630 July 2, 1981]

Facts: defendant Philnabank Employees' Association (PEMA) declared a strike. During the said strike,
which lasted up to the following day, members of the (PEMA) paraded and displayed placards in front
of the PNB building at Escolta, Manila, one of which contained the following words: "PCIB BAD
ACCOUNTS TRANSFERRED TO PNB-NIDC?" It is an admitted fact that PCIB stands for plaintiff Philippine
Commercial and Industrial Bank, while PNB refers to Philippine National Bank, and NIDC stands for
National Investment Development Corporation, a subsidiary of the PNB."

Plaintiff sued for Libel "that the writing on the placard in question is a baseless and malicious
aspersion that the plaintiff was a party to a fraud, in that it was able to recoup on bad debts or other
uncollected accounts by fraudulent, questionable and immoral transfer thereof to the PNB or NIDC.
Malice was likewise attributed to defendant labor union and its officers.

the lower court ruled: The plaintiff has also miserably failed to prove any damage caused to it by the
supposed libelous placard subject matter of this action.

Issue: Whether respondent should be held guilty of libel to which was committed in the exercise of their
right to peaceably assemble.

Ruling: No. There was a labor controversy resulting in a strike. The labor union made use of its
constitutional right to picket. this Court has been committed to the view that peaceful picketing is
part of the freedom of speech guarantee of the Constitution. labor disputes give rise to strong
emotional response, then the decision reached by the lower court becomes even more acceptable. It is
a fact of industrial life, both in the Philippines as in the United States, that in the continuing
confrontation between labor and management, it is far from likely that the language employed would
be both courteous and polite. Such being the case. there is no affront either to reason or to the law in
the complaint for libel being dismissed. In pricing reliance on the constitutional right of freedom of
expression, this Court once again makes manifest its adherence to the principle first announced by
Justice Malcolm as ponente in the leading case of United States v. Bustos. In no uncertain terms, it
made clear that the judiciary, in deciding suits for libel, must ascertain whether or not the alleged
offending words may be embraced by the guarantees of free speech and free press.

CRISPIN MALABANAN, EVELIO JALOS, BEN LUTHER LUCAS, SOTERO LEONERO, and JUNE LEE,vs. THE
HONORABLE ANASTACIO D. RAMENTO [G.R. No. L-62270 May 21, 1984]

Facts: Petitioners were officers of the Supreme Student Council of Respondent University. They sought
and were granted by the school authorities a permit to hold a meeting from 8am to 12am. Pursuant to
such permit, along with other students, they held a general assembly at the Veterinary Medicine and
Animal Science (VMAS) Basketball Court. The place indicated in such permit, not in the basketball
court as therein stated, but at the second floor lobby. At such gathering, they manifested in vehement
and vigorous language their opposition to the proposed merger of the Institute of Animal Science.
They continued their language severely critical of the university authorities and using megaphones in the
process. There was, as a result, disturbance of classes being held. Also, non academic employees
within hearing distance, stopped their work because of noise created. They were asked to explain why
they should not be held liable for holding an assembly. Petitioners were suspended for one academic
year.
Issue: Whether the suspension of students for one academic year was violative of the constitutional
rights of freedom of assembly and free speech?

Ruling: Yes, necessarily their exercise to discuss matters affecting their welfare or involving public
interest is not subjected to previous restraint or subsequent punishment unless there be a showing of
clear and present danger to a substantive evil that the State has a right to prevent. The peaceable
character of an assembly could be lost, however, by an advocacy or disorder. If assembly is to be held in
school premises, permit must be sought from its school authorities who are devoid to deny such
request. In granting such permit, there may be conditions as to the time and place of an assembly to
avoid disruption of classes or stoppage of work of non-academic personnel. However, in violation of
terms, penalty incurred should not be disproportionate to the offense. The excitement of the occasion,
the propensity of speakers to exaggerate and the exuberance of the youth should be taken into
consideration.

LUZVIMINDA DE LA CRUZ, vs., COURT OF APPEALS, CIVIL SERVICE COMMISSION and THE SECRETARY
OF THE DEPARTMENT OF EDUCATION, CULTURE AND SPORTS. [G.R. No. 126183. March 25, 1999]

Facts: Petitioners are public school teachers from various schools in Metro Manila who were
simultaneously charged, preventively suspended, and eventually dismissed in October 1990 by the
Secretary of the Department of Education, Culture and Sports (DECS) in connection with the
administrative complaints filed before its office by their respective principals for participating in a
mass action/strike and subsequently defying the return-to-work order by DECS constituting grave
misconduct., gross neglect of duty, gross violation of Civil Service Law, Rules and Regulations and
reasonable office regulations, refusal to perform official duty, gross insubordination conduct prejudicial
to the best interest of the service and absence without official leave (AWOL), in violation of Presidential
Decree 807, otherwise known as the Civil Service Decree of the Philippines. Petitioners contend they are
merely participating in a peaceful assembly to petition the government for redress of their grievances
in the exercise of their constitutional right and insist their assembly does not constitutes as a strike as
there is no actual disruption of classes.

Issue: Whether the petitioners’ exercise of their right to freedom to assembly and petition were valid.

Ruling: No.The court held that previous jurisprudence laid down a rule that public teachers in the
exercise of their right to ventilate their grievances by petitioning the government for redress should be
done within reasonable limits so as not to prejudice the public welfare. The conduct of mass protests
during school days while abandoning classes is highly prejudicial to the best interest of public service.
The court stresses that teachers are penalized not because they exercised their right to peaceably
assemble but because of the manner by which such right was exercised, i.e., going on unauthorized
and unilateral absences thus disrupting classes in various schools in Metro Manila which produced
adverse effects upon the students for whose education the teachers were responsible.

Bangalisan v. CA, GR 124678, July 23, 1997 – Kenny

G.R. No. 124678 July 31, 1997

DELIA BANGALISAN, ET.AL., petitioners,


vs.
HON. COURT OF APPEALS, ET.AL., respondents.
FACTS: Petitioners, except Rodolfo Mariano, were among the 800 public school teachers who staged
"mass actions" on September 17 to 19, 1990 to dramatize their grievances concerning, in the main, the
alleged failure of the public authorities to implement in a just and correct manner certain laws and
measures intended for their material benefit. On September 17, 1990, the Secretary of the Department
of Education, Culture and Sports (DECS) issued a Return-to-Work Order. Petitioners failed to comply
with said order. They were simultaneously placed under preventive suspension. Petitioners contend
that the Court of Appeals committed grave abuse of discretion when it upheld the resolutions of the CSC
that penalized petitioners whose only offense was to exercise their constitutional right to peaceably
assemble and petition the government for redress of grievances. Petitioners contend, however, that
they were not on strike but were merely exercising their constitutional right peaceably to assemble
and petition the government for redress of grievances.

ISSUE: Whether the teachers are may be penalized for their acts.

RULING: YES. We find such pretension devoid of merit. It is not the exercise by the petitioners of their
constitutional right to peaceably assemble that was punished, but the manner in which they exercised
such right which resulted in the temporary stoppage or disruption of public service and classes in
various public schools in Metro Manila. For, indeed, there are efficient but non-disruptive avenues,
other than the mass actions in question, whereby petitioners could petition the government for redress
of grievances.

It bears stressing that suspension of public services, however temporary, will inevitably derail services
to the public, which is one of the reasons why the right to strike is denied government employees. 12 It
may be conceded that the petitioners had valid grievances and noble intentions in staging the "mass
actions," but that will not justify their absences to the prejudice of innocent school children. Their
righteous indignation does not legalize an illegal work stoppage.

Ruiz v. Gordon, 126 SCRA 233 – Kenny


Facts: Hector S. Ruiz, Coordinator of Olongapo Citizen's Alliance for National Reconciliation, filed a
petition for mandamus against Richard Gordon to be allowed to hold a parade/march from Gordon
Avenue to the Rizal Triangle starting at 1:00 P.M. Respondents replied by stating the request for a
prayer rally was received in the Office of the Mayor and that respondent had repeatedly announced in
his regular program on Sunday over the radio (DWGO) and at the Monday morning flag ceremony
before hundreds of government employees that he would grant the request of any group that would
like to exercise their freedom of speech and assembly.

Issue: Can the petition be granted?

Ruling: NO. Petition dismissed. The Reyes case was given some discussion in the course of this petition
as to the role of the judiciary in petitions for permits to hold peaceable assembles.

"The applicants for a permit to hold an assembly should inform the licensing authority of the date, the
public place where and the time when it will take place. If it were a private place, only the consent of
the owner or the one entitled to its legal possession is required. Such application should be filed well
ahead in time to enable the public official concerned to appraise whether there may be valid objections
to the grant of the permit or to its grant but at another public place. It is an indispensable condition to
such refusal or modification that the clear and present danger test be the standard for
the decision reached. If he is of the view that there is such an imminent and grave danger of a
substantive evil, the applicants must be heard on the matter.” Thereafter, his decision must be
transmmitted to them at the earliest opportunity. They can have recourse to the proper judicial
authority. Free speech and peaceable assembly, along with the other intellectual freedoms, are highly
ranked in our scheme of constitutional values. It cannot be too strongly stressed that on the judiciary, —
even more so than on the other departments — rests the grave and delicate responsibility of assuring
respect for and deference to such preferred rights.

As shown both in the manifestation and the answer, this action for mandamus could have been
obviated if only petitioner took the trouble of verifying on November 23 whether or not a permit had
been issued. A party desirous of exercising the right to peaceable assembly should be the one most
interested in ascertaining the action taken on a request for a permit. Necessarily, after a reasonable
time or, if the day and time was designated for the decision on the request, such party or his
representative should be at the office of the public official concerned. If he fails to do so, a copy of
the decision reached, whether adverse or favorable, should be sent to the address of petitioner.

BAYAN v. Ermita – GR 169838, April 25, 2006 – Kenny

G.R. No. 169838 April 25, 2006

BAYAN, KARAPATAN, KILUSANG MAGBUBUKID NG PILIPINAS (KMP), Petitioners,


vs.
EDUARDO ERMITA, Respondents.

FACTS: Petitioners Bayan, et al., contend that Batas Pambansa No. 880 is clearly a violation of the
Constitution and the International Covenant on Civil and Political Rights and other human rights
treaties of which the Philippines is a signatory. They argue that B.P. No. 880 requires a permit before
one can stage a public assembly regardless of the presence or absence of a clear and present danger.
It also curtails the choice of venue and is thus repugnant to the freedom of expression clause as the
time and place of a public assembly form part of the message for which the expression is sought.
Petitioners Jess del Prado, et al., in turn, argue that B.P. No. 880 is unconstitutional as it is a curtailment
of the right to peacefully assemble and petition for redress of grievances because it puts a condition for
the valid exercise of that right. It also characterizes public assemblies without a permit as illegal and
penalizes them and allows their dispersal. Thus, its provisions are not mere regulations but are actually
prohibitions.

ISSUE: Whether B.P. 880 violates the Constitution as it abridges the freedom of the right of the people
to peaceably assemble and petition the government for redress of grievances.

RULING: NO. In sum, this Court reiterates its basic policy of upholding the fundamental rights of our
people, especially freedom of expression and freedom of assembly. In several policy addresses, Chief
Justice Artemio V. Panganiban has repeatedly vowed to uphold the liberty of our people and to nurture
their prosperity. He said that "in cases involving liberty, the scales of justice should weigh heavily
against the government and in favor of the poor, the oppressed, the marginalized, the dispossessed
and the weak. Indeed, laws and actions that restrict fundamental rights come to the courts with a heavy
presumption against their validity. These laws and actions are subjected to heightened scrutiny."
B.P. No. 880 cannot be condemned as unconstitutional; it does not curtail or unduly restrict freedoms;
it merely regulates the use of public places as to the time, place and manner of assemblies. Far from
being insidious, "maximum tolerance" is for the benefit of rallyists, not the government. The delegation
to the mayors of the power to issue rally "permits" is valid because it is subject to the constitutionally-
sound "clear and present danger" standard.

GSIS v. Kapisanan, GR 170132, December 6, 2006 – Kenny

G.R. No. 170132 December 6, 2006

GOVERNMENT SERVICE INSURANCE SYSTEM (GSIS) and WINSTON F. GARCIA, in his capacity as GSIS
President & General Manager, petitioners,
vs.
KAPISANAN NG MGA MANGGAGAWA SA GSIS, respondents.

A four-day October 2004 concerted demonstration, rallies and en masse walkout waged/held in front
of the GSIS main office in Roxas Boulevard, Pasay City took place, which appeared to be targeting
petitioner Garcia and his management style. While the Mayor of Pasay City allegedly issued a rally
permit, the absence of the participating GSIS employees was not covered by a prior approved leave.
Thereafter, GSIS directed 131 union and non-union members to show cause why they should not be
charged administratively for their participation in said rally.

Whether petitioner may be prohibited from filing formal charges against the employees who
participated in the said demonstrations.

NO. It should be stressed right of that the civil service encompasses all branches and agencies of the
Government, including government-owned or controlled corporations (GOCCs) with original charters,
like the GSIS,9 or those created by special law.10 As such, employees of covered GOCCs are part of the
civil service system and are subject to circulars, rules and regulations issued by the Civil Service
Commission (CSC) on discipline, attendance and general terms/conditions of employment, inclusive of
matters involving self-organization, strikes, and demonstrations and like concerted actions.

With the view we take of the events that transpired on October 4-7, 2004, what respondent's members
launched or participated in during that time partook of a strike or, what contextually amounts to the
same thing, a prohibited concerted activity. Indeed, for four straight days, participating KMG members
and other GSIS employees staged a walk out and waged or participated in a mass protest or
demonstration right at the very doorstep of the GSIS main office building. The record of
attendance27 for the period material shows that, on the first day of the protest, 851 employees, or forty
eight per cent (48%) of the total number of employees in the main office (1,756) took to the streets
during office hours, from 6 a.m. to 2 p.m.,28leaving the other employees to fend for themselves in an
office where a host of transactions take place every business day. On the second day, 707 employees
left their respective work stations, while 538 participated in the mass action on the third day. A smaller
number, i.e., 306 employees, but by no means an insignificant few, joined the fourth day activity.

In Re Valmonte, 296 SCRA – Kenny


Petitioner questions the setting up of checkpoins in the area of Valenzuela and Metro Manila without
proper guidelines/standard for its conduct. Petitioner raise the issue of possible harassment and of
searches and seizures without violations of any specific violation of law.

Issue: Whether the checkpoints infringed on an individual’s right to liberty

Held: The SC dismissed the petition. Dissenting Opinion of Justice Cruz.

I dissent. The sweeping statements in the majority opinion are as dangerous as the checkpoints it
would sustain and fraught with serious threats to individual liberty. The bland declaration that
individual rights must yield to the demands of national security ignores the fact that the Bill of Rights
was intended precisely to limit the authority of the State even if asserted on the ground of national
security. What is worse is that the searches and seizures are peremptorily pronounced to be reasonable
even without proof of probable cause and much less the required warrant. The improbable excuse is
that they are aimed at 'establishing an effective territorial defense, maintaining peace and order, and
providing an atmosphere conducive to the social, economic and political development of the National
Capital Region." For these purposes, every individual may be stopped and searched at random and at
any time simply because he excites the suspicion, caprice, hostility or malice of the officers manning the
checkpoints, on pain of arrest or worse, even being shot to death, if he resists.

In Re Petition to Annul 98-7-02 SC - 72 – Kenny

A.M. 98-7-02-SC dated July 7, 1998

Re: Guidelines on the Conduct of Demonstrations, Pickets, Rallies and Other Similar Gatherings in the
Vicinity of the Supreme Court and All Other Courts.

En Banc.

Considering the inherent and regulatory power of the courts to control their proceedings in order to
permit the fair and impartial administration of justice and the constitutional rights, pleading, practice,
and procedure in all courts, and complementing further the Per Curiam Resolution of the Court in the
case of Nestle Philippines, Inc. v. Hon. Augusto S. Sanchez, et al., dated September 30, 1987, the Court
resolves to adopt formally the following policy and procedural guidelines, regarding the conduct of
demonstrations, pickets, rallies and other similar gatherings in the vicinity of the grounds and adjacent
areas of the Supreme Court and all other courts:

1. Courts are the defenders of the people's rights, especially their freedom of expression and assembly.
Free speech and peaceable assembly, along with the other intellectual freedoms, are highly ranked in
our scheme of constitutional values. These freedoms, however, are not absolute. The right of a citizen
to use the streets for communication of views on national questions must be balanced with the need
of our courts for an atmosphere that will enable them to dispense justice free from bias and
unnecessary pressure. The courts would not exist and survive to protect the people's most revered
rights if they were unable to preserve the integrity of judicial proceedings and the dignity of the
institution from all forms of distracting, degrading and prejudicial influences that threaten the fair and
orderly administration of justice.
2. Demonstrators, picketers, rallyists and all other similar persons are enjoined from holding any
activity on the sidewalks and streets adjacent to, in front of, or within a radius of two hundred (200)
meters from, the outer boundary of the Supreme Court Building, any Hall of Justice, and any other
building that houses at least one (1) court sala. Such activities unquestionably interrupt and hamper the
working condition in the salas, offices and chambers of the courts.

3. Demonstrators, picketers, rallyists and their sympathizers must keep all public thoroughfares free
and unimpeded to allow the smooth flow of vehicular and pedestrian traffic. At no time should
ingress to and egress from the premises of the courts and the offices of the courts and the offices of the
justices, judges, and court officials and employees be obstructed.

4. Demonstrators, picketers, rallyists and their sympathizers are prohibited from camping out on the
streets, sidewalks or pavement adjacent to, in front of, or within a radius of two hundred (200) meters
from, the outer boundary of the Supreme Court Building, any Hall of Justice, and any other building that
houses at least one (1) court sala. No provisional shelters and kitchens, pickets' quarters, and other
similar makeshift structures shall be established in said areas.

5. Lawyers of parties with cases pending in courts have a duty to properly apprise their clients on
matters of decorum and proper attitude toward courts of justice when engaged in demonstrations,
pickets, rallies and similar activities. As officers of the court, they must help to preserve the dignity of
the courts and to insulate the courts from all forms of influence that may adversely affect judicial
impartiality and violate a party's right to due process.

6. Any violation of this resolution shall be treated as contempt of court. Members of the Bar violating
this resolution may, in addition, be subject to the administrative sanctions of fine, imprisonment,
suspension from the practice of law or disbarment as circumstances may warrant.

The Clerk of Court is directed to forthwith cause publication of these Guidelines in two (2) newspapers
of general circulation. The Guidelines shall take effect upon the expiration of fifteen (15) days after such
publication.

The Clerk of Court is further directed to furnish all lower courts, the Integrated Bar of the Philippines,
the PNP, and all agencies of local governments in charge of issuing permits to hold demonstrations,
rallies, pickets and similar activities, around the vicinity of courts of justice with copies of this Resolution.

Section 5. No law shall be made respecting an establishment of religion, or prohibiting the free
exercise thereof. The free exercise and enjoyment of religious profession and worship, without
discrimination or preference, shall forever be allowed. No religious test shall be required for the
exercise of civil or political rights.

Purpose

I. Non-Establishment Clause

Aglipay v. Ruiz, 64 Phil 201


Facts: The 33rd International Eucharistic Congress organized by the Roman Catholic Church took place
sometime in 1936. In commemoration thereof. then Director of Posts, Juan Ruiz, initiated the
production of certain stamps the design of which would have in their center a chalice, with grape and
stalks of wheat as border design. Eventually, the stamps were produced and some were sold pursuant
to Act No. 4052, which provides for appropriation.

Gregorio Aglipay, the head of the Philippine Independent Church, assailed the production and sale of
such stamps. Aglipay contends that the funding of said stamps commemorative to a particular
religious event is in violation of Sec 13, Article 6 of the Philippine Constitution which prohibits the
appropriation or usage of public money for the use or benefit of any church or denomination.

ISSUE: Whether the production of the said stamps violate the Constitution.

HELD: No. The sale of stamps is not in violation of the Constitution. In fact, what was emphasized on
the stamps was not the religious event itself but rather the City of Manila as being the seat of such
event. Act No. 4052 on the other hand did not appropriate any public money to a religious event. Act
No. 4052 appropriated the sum of P60,000.00 for the cost of plates and printing of postage stamps with
new designs and other expenses incident thereto, and merely authorizes the Director of Posts, with the
approval of the Secretary of Public Works and Communications, to dispose of the amount appropriated
in the manner indicated and “as often as may be deemed advantageous to the Government”. The fact
that the fund is being used for such is only incidental to the function of Director of Posts and under his
discretion.

On religious freedom

The Supreme Court noted however that the elevating influence of religion is recognized here as
elsewhere. Evidence would be our preamble where we implored the aid of divine providence to
establish an ideal government. It should also be further noted that religious freedom as a constitutional
mandate is not an inhibition of profound reverence to religion.

Garces v. Estenzo, 104 SCRA 510

Two resolutions of the Barangay Council of Valencia, Ormoc City were passed:

a. Resolution No. 5- Reviving the traditional socio-religious celebration every fifth of April. This
provided for the acquisition of the image of San Vicente Ferrer and the construction of a waiting shed.
Funds for the said projects will be obtained through the selling of tickets and cash donations.

b. Resolution No. 6- The chairman or hermano mayor of the fiesta would be the caretaker of the image
of San Vicente Ferrer and that the image would remain in his residence for one year and until the
election of his successor. The image would be made available to the Catholic Church during the
celebration of the saint’s feast day.

These resolutions have been ratified by 272 voters, and said projects were implemented. The image
was temporarily placed in the altar of the Catholic Church of the barangay. However, after a mass,
Father Sergio Marilao Osmeña refused to return the image to the barangay council, as it was the
church’s property since church funds were used in its acquisition.
Resolution No. 10 was passed for the authorization of hiring a lawyer for the replevin case against the
priest for the recovery of the image.

Resolution No. 12 appointed Brgy. Captain Veloso as a representative to the case. The priest, in his
answer assailed the constitutionality of the said resolutions. The priest with Andres Garces, a member
of the Aglipayan Church, contends that Sec. 8 Article IV1 and Sec 18(2) Article VIII) 2 of the constitution
was violated.

Issue: Whether any freedom of religion clause in the Constitution violated.

Held: No. As said by the Court this case is a petty quarrel over the custody of the image. The image was
purchased in connection with the celebration of the barrio fiesta and not for the purpose of favoring
any religion nor interfering with religious matters or beliefs of the barrio residents. Any activity
intended to facilitate the worship of the patron saint(such as the acquisition) is not illegal. Practically,
the image was placed in a layman’s custody so that it could easily be made available to any family
desiring to borrow the image in connection with prayers and novena. It was the council’s funds that
were used to buy the image, therefore it is their property. Right of the determination of custody is their
right, and even if they decided to give it to the Church, there is no violation of the Constitution, since
private funds were used. Not every government activity which involves the expenditure of public
funds and which has some religious tint is violative of the constitutional provisions regarding
separation of church and state, freedom of worship and banning the use of public money or property.

School District v. Schempp, 394 RS 203

Facts: Both cases involved the reading of Bible passages prior to class in public schools. Schempp
challenged a Pennsylvania law that stated that, "at least ten verses from the Holy Bible shall be read,
without comment, at the opening of each public school day. Any child shall be excused from such Bible
reading, or attending such Bible reading, upon written request of his parent or guardian." This was
disallowed by a federal district court.

Murray challenged the requirement of the Baltimore school board that the Lord's Prayer be recited
prior to the beginning of the day's classes. This was upheld by both a state court and the Maryland
Court of Appeals.

Issue:W/N the reading of the bible in class violates of the Establishment clause

Held: Yes. The test for assessing the violation of the Establishment Clause can be summarized as "what
are the purpose and primary effect of the enactment. If either is the advancement or inhibition of
religion then the enactment exceeds the scope of legislative power as circumscribed by the
Constitution. That is to say that to withstand the structures of the Establishment Clause there must be
a secular legislative purpose and a primary effect that neither advances nor inhibits religion." To
violate the Free Exercise Clause, it is only necessary to show a coercive effect of a law's enactment. That
these might merely be "minor encroachments on the First Amendment" is irrelevant. The Court does
not disallow the comparative study of religion in public schools, but these religious observances were
not instituted for such studies.

Board of Education v. Allen, 392 US 236


Facts: New York's Education Law required local public school authorities to lend textbooks free of
charge to all students in grades 7 to 12, even those attending private schools.

Some local school boards challenged this regulation, arguing that it violated both State and Federal
constitutions. The boards asked for an an order barring the Commissioner of Education (Allen) from
removing board members from office for failing to comply with the regulation, and an order that
stopped any use of state funds to buy textbooks that would be lent to parochial students.

A trial court found in favor of the school boards, an appellate court reversed this decision, and the New
York Court of Appeals agreed that the regulation was constitutional.

Issue: W/N the lending of the books was constitutional

Held: Yes. The Supreme Court gave six reasons why the law in question did not violate the Establishment
Clause:

The primary purpose of the statute was to advance education in general, not advance religious
education in particular.

There was no evidence of specifically religious books being loaned.

Parochial schools also perform the task of secular education, and so helping them does not
automatically help religion.

There is no evidence of unconstitutional state involvement with religion.

There is no evidence of anyone being coerced into the practice of religion.

The focus of the Court decision was mostly the fact that the law had a secular purpose:

The express purpose of 701 was stated by the New York Legislature to be furtherance of the
educational opportunities available to the young. Appellants have shown us nothing about the
necessary effects of the statute that is contrary to its stated purpose. The law merely makes available to
all children the benefits of a general program to lend school books free of charge. Books are furnished at
the request of the pupil and ownership remains, at least technically, in the State. Thus no funds or
books are furnished to parochial schools, and the financial benefit is to parents and children, not to
schools.

Lemon v. Kurtzman, 403 US 602


Facts: Pennsylvania and Rhode Island statutes provided state aid to church-related elementary and
secondary schools. A group of individual taxpayers and religious liberty organizations filed suit,
challenging the constitutionality of the program. They claimed that, since the program primarily aided
parochial schools, it violated the Establishment Clause.

Issue: Whether states can create programs that provide financial support to nonpublic elementary and
secondary schools by way or reimbursement for the cost of teachers’ salaries, textbooks, and
instructional materials in specified secular subjects (Pennsylvania) -- or pay a salary supplement directly
to teachers of secular subjects in religious schools (Rhode Island).
Held: In a unanimous decision, the Court held that both programs violate the Establishment Clause
because they create excessive entanglement between a religious entity and the state. The Court
looked to three factors in determining the constitutionality of the contested programs, factors that
would become known as the Lemon test. First, whether the legislature passed the statute based on a
secular legislative purpose. The Court could find no evidence that the goal of the Pennsylvania or Rhode
Island legislatures was to advance religion. Instead the Court relied on the stated purpose, that the bill
was designed to improve "the quality of the secular education in all schools covered by the compulsory
attendance laws." Second, the Court questioned whether the programs had the primary effect of
advancing or inhibiting religion. It bypassed this prong by examining the third prong and finding a
violation there, thus obviating the need for analysis of this point. The third factor, and the point at which
the Court found the constitutional defect, was over the issue of excessive entanglement. Here, the
Court held that the state’s oversight and auditing requirements and the propensity for political
divisiveness generated by this kind of aid program would entangle the state and the religious entity in
unconstitutional ways.

Tilton v. Richardson, 403 US 672

Facts: The federal Higher Education Facilities Act of 1963 provided construction grants to church-
sponsored higher educational institutions. The grants were to be used for the construction of non-
religious school facilities. The Act also stipulated that twenty years after the grant had been given,
schools were free to use the buildings for any purpose.

Issue: Did the Act violate the Religion Clauses of the First Amendment?

Held: In a 5-to-4 decision, the Court held that only the 20-year limitation portion of the Act violated the
Religion Clauses of the First Amendment. The Court invalidated the 20-year clause, arguing that
subsidizing the construction of facilities used for non-secular purposes would have the effect of
advancing religion. The Court held that the church-related institutions in question had not used their
federally-funded facilities for religious activities, and that the facilities were "indistinguishable from a
typical state university facility." The Court also held that the Act did not excessively entangle the
government with religion, noting that college students were less susceptible to religious indoctrination,
that the aid was of "nonideological character," and that one-time grants did not require constant state
surveillance.

Country of Allegheny v. American Civil Liberties Union, 57 LW 504- Val

FACTS: This litigation concerns the constitutionality of two recurring holiday displays located on public
property in downtown Pittsburgh. The first depicting the Christian nativity scene, was placed on the
Grand Staircase of the Allegheny County Courthouse. The second of the holiday displays in question
was an 18-foot Chanukah menorah or candelabrum, which was placed just outside the City-County
Building next to the city's 45-foot decorated Christmas tree. Respondents, filed suit seeking
permanently to enjoin the county from displaying the creche and the city from displaying the
menorah on the ground that the displays violated the Establishment Clause of the First Amendment

ISSUE: Whether there was a violation of the free exercise clause


RULING: When viewed in its overall context, the creche display violates the Establishment Clause. In
sum, Lynch teaches that government may celebrate Christmas in some manner and form, but not in a
way that endorses Christian doctrine. Here, Allegheny County has transgressed this line. It has chosen
to celebrate Christmas in a way that has the effect of endorsing a patently Christian message: Glory to
God for the birth of Jesus Christ. Under Lynch, and the rest of our cases, nothing more is required to
demonstrate a violation of the Establishment Clause. The display of the creche in this context, therefore,
must be permanently enjoined.

However it is not "sufficiently likely" that residents of Pittsburgh will perceive the combined display
of the tree, the sign, and the menorah as an "endorsement" or "disapproval . . . of their individual
religious choices." that the display of the menorah is not an endorsement of religious faith, but simply a
recognition of cultural diversity. The court also found that "the displays had a secular purpose," and
"did not create an excessive entanglement of government with religion.

Zobrest v. Catalina, No. 92-94 June 18, 1993- Val

FACTS: Petitioners, a deaf child and his parents, filed this suit after respondent school district refused
to provide a sign-language interpreter to accompany the child to classes at a Roman Catholic high
school. They alleged that the Individuals with Disabilities Education Act (IDEA) and the Free Exercise
Cause of the First Amendment required respondent to provide the interpreter and that the
Establishment Clause did not bar such relief. The District Court granted respondent summary judgment
on the ground that the interpreter would act as a conduit for the child's religious inculcation, thereby
promoting his religious development at government expense in violation of the Establishment Clause.

ISSUE: Whether the provision of an interpreter is in violation of the Establishment clause

RULING: No. The Establishment Clause does not prevent respondent from furnishing a disabled child
enrolled in a sectarian school with a sign-language interpreter in order to facilitate his education.
Government programs that neutrally provide benefits to a broad class of citizens defined without
reference to religion are not readily subject to an Establishment Clause challenge just because
sectarian institutions may also receive an attenuated financial benefit. . The service in this case is part of
a general government program that distributes benefits neutrally to any child qualifying as disabled
under the IDEA, without regard to the sectarian-nonsectarian, or public-nonpublic nature of the school
the child attends. Here, the child is the primary beneficiary, and the school receives only an incidental
benefit. There is no absolute bar to the placing of a public employee in a sectarian school.

Capitol Square Review Board v. Pinetter & Ku Klus Klan, US No. 94-780, June 29, 1995- Val

FACTS: In 1993, the Ku Klux Klan organization attempted to place an unattended cross on Capitol
Square, the state-house plaza in Columbus, Ohio, during the 1993 Christmas season. Ohio law makes
Capitol Square a forum for discussion of public questions and for public activities, and gives the
Advisory Board responsibility for regulating access to the square. The Board denied the application of
the Ku Klux Klan to erect the cross on Establishment Clause grounds.
ISSUE: Whether the placing of the cross violates the Establishment Clause

RULING: No. Religious expression cannot violate the Establishment Clause where it (1) is purely private
and (2) occurs in a traditional or designated public forum, publicly announced and open to all on equal
terms. Those conditions are satisfied here, and therefore the State may not bar respondents' cross from
Capitol Square.

The display was private religious speech that "is as fully protected under the Free Speech Clause as
secular private expression." Because Capitol Square is designated as a traditional public forum, any
group may express their views there, and the Board may regulate the content of the Klan's expression
on the plaza only if a restriction is necessary and narrowly drawn to serve a compelling state interest.

Lee v. Welsman, US No. 90-1014, June 24, 1992 - JM

Facts: It has been the custom of Providence school officials to provide invited clergy with a pamphlet
entitled "Guidelines for Civic Occasions," prepared by the National Conference of Christians and Jews.
The Guidelines recommend that public prayers at nonsectarian civic ceremonies be composed with
"inclusiveness and sensitivity," though they acknowledge that "[p]rayer of any kind may be
inappropriate on some civic occasions." The principal gave Rabbi Gutterman the pamphlet before the
graduation, and advised him the invocation and benediction should be nonsectarian.

Ruling: Our society would be less than true to its heritage if it lacked abiding concern for the values of its
young people, and we acknowledge the profound belief of adherents to many faiths that there must be
a place in the student's life for precepts of a morality higher even than the law we today enforce. We
express no hostility to those aspirations, nor would our oath permit us to do so. A relentless and all-
pervasive attempt to exclude religion from every aspect of public life could itself become inconsistent
with the Constitution.... The sole question presented is whether a religious exercise may be conducted
at a graduation ceremony in circumstances where, as we have found, young graduates who object are
induced to conform. No holding by this Court suggests that a school can persuade or compel a student
to participate in a religious exercise. That is being done here, and it is forbidden by the Establishment
Clause of the First Amendment.

Manosca v. CA, 252 SCRA 412 - JM

Facts: Petitioners inherited a piece of land when the parcel was ascertained by the NHI to have been
the birth site of Felix Y. Manalo, the founder of Iglesia Ni Cristo, it passed Resolution No. 1, declaring
the land to be a national historical landmark. Petitioners moved to dismiss the complaint on the main
thesis that the intended expropriation was not for a public purpose and, incidentally, that the act
would constitute an application of public funds, directly or indirectly, for the use, benefit, or support of
Iglesia ni Cristo, a religious entity, contrary to the provision of Section 29(2), Article VI, of the 1987
Constitution.

Issue: The expropriation of the land whereat Manalo was born, valid and constitutional?

Held: Yes. The taking to be valid must be for public use. There was a time when it was felt that a literal
meaning should be attached to such a requirement. Whatever project is undertaken must be for the
public to enjoy, as in the case of streets or parks. Otherwise, expropriation is not allowable. It is not so
any more. As long as the purpose of the taking is public, then the power of eminent domain comes into
play. As just noted, the constitution in at least two cases, to remove any doubt, determines what is
public use. One is the expropriation of lands to be subdivided into small lots for resale at cost to
individuals. The other is the transfer, through the exercise of this power, of utilities and other private
enterprise to the government. It is accurate to state then that at present whatever may be beneficially
employed for the general welfare satisfies the requirement of public use.

Islamic Dawah v. ES, GR 153888, July 9, 2003 - JM

Facts: Petitioner contends that the subject EO violates the constitutional provision on the separation
of Church and State. It is unconstitutional for the government to formulate policies and guidelines on
the halal certification scheme because said scheme is a function only religious organizations, entity or
scholars can lawfully and validly perform for the Muslims. According to petitioner, a food product
becomes halal only after the performance of Islamic religious ritual and prayer. Thus, only practicing
Muslims are qualified to slaughter animals for food. A government agency like herein respondent OMA
cannot therefore perform a religious function like certifying qualified food products as halal.

Ruling: OMA was created in 1981 through Executive Order No. 697 (EO 697) “to ensure the integration
of Muslim Filipinos into the mainstream of Filipino society with due regard to their beliefs, customs,
traditions, and institutions.” OMA deals with the societal, legal, political and economic concerns of the
Muslim community as a “national cultural community” and not as a religious group. Thus, bearing in
mind the constitutional barrier between the Church and State, the latter must make sure that OMA
does not intrude into purely religious matters lest it violate the non-establishment clause and the “free
exercise of religion” provision found in Article III, Section 5 of the 1987 Constitution.

Taruc v. Dela Cruz, 453 SCRA 123

Petitioners were lay members of the Philippine Independent Church (PIC). On June 28, 1993,
Bishop de la Cruz declared petitioners expelled/excommunicated from the Philippine Independent
Church. Because of the order of expulsion/excommunication, petitioners filed a complaint for damages
with preliminary injunction against Bishop de la Cruz before the Regional Trial Court.They contended
that their expulsion was illegal because it was done without trial thus violating their right to due
process of law.

Issue:
Whether courts have jusrisdiction in purely ecclesiastical matters

Held:
No. The expulsion/excommunication of members of a religious institution/organization is a
matter best left to the discretion of the officials, and the laws and canons, of said
institution/organization. It is not for the courts to exercise control over church authorities in the
performance of their discretionary and official functions. Rather, it is for the members of religious
institutions/organizations to conform to just church regulations.
“Civil Courts will not interfere in the internal affairs of a religious organization except for the
protection of civil or property rights. Those rights may be the subject of litigation in a civil court, and
the courts have jurisdiction to determine controverted claims to the title, use, or possession of church
property.” Obviously, there was no violation of a civil right in the present case
UCCP v. Bradford, 674 SCRA 92

BUCCI wanted to be separate from UCCP. UCCP did not allow this.

Issue: Whether this is a purely ecclesiastical matter

Ruling: It is not a purely ecclesiastical affair

An ecclesiastical affair is one that concerns doctrine, creed or form of worship of the church, or
the adoption and enforcement within a religious association of needful laws and regulations for the
government of the membership,

UCCP and BUCCI, being corporate entities and grantees of primary franchises, are subject to
the jurisdiction of the SEC.

Imbong v. Ochoa, GR 204819, April 8, 2014

This case is about the consititutionality of the RH law. Ruling:


R H l a w d o e s n o t v i o l a t e g u a r a n t e e o f r e l i g i o u s f r e e d o m v i a t h e s t a t e sponsored
procurement of contraceptives, which contravene the religious beliefs of the people including the
petitioners. This is because in doing so, t h e s t a t e w o u l d b e a d h e r i n g t o o n e r e l i g i o n s ,
m a k i n g a d e f a c t o s t a t e religion which is contrary to religious freedom.

The separation of church and 4tate shall be inviolable

There limits to the exercise of religious freedom compelling state interest test

RH law does not violate the guarantee of religious freedom by re3uiring would be spouses,
as a condition for the issuance of a marriage license, to attend a seminar on parenthood, family
planning, breastfeeding and infant nutrition

However, the R H L a w v i o l a t e s t h e g u a r a n t e e o f r e l i g i o u s f r e e d o m b y
compelling medical health practitioners, hospitals, and health care
providers, under pain of penalty, to refer patients to other institutions despite their conscientious
objections

II. Free Exercise of Religion

Tests
a) Clear and Present Danger Test: When words are used in such circumstance and of such nature as to
create a clear and present danger that will bring about the substantive evil that the State has a right
to prevent.
b) Compelling State Interest Test: When a law of general application infringes religious exercise, albeit
incidentally, the state interest sought to be promoted must be so paramount and compelling as to
override the free exercise claim. Three-step test:
1. Has the statute or government action created a burden on the free exercise of religion?
2. Is there a sufficiently compelling state interest to justify this infringement of religious liberty?
3. Has the state in achieving its legitimate purposes used the least intrusive means possible so that the
free exercise is not infringed any more than necessary to achieve the legitimate goal of the state?

(Estrada v. Escritor) – Mac

Complainant Alejandro Estrada wrote to Judge Jose F. Caoibes, Jr., requesting for an investigation of
rumors that respondent Soledad Escritor, court interpreter, is living with a man not her husband. They
allegedly have a child of eighteen to twenty years old. Estrada is not personally related either to
Escritor or her partner. Nevertheless, he filed the charge against Escritor as he believes that she is
committing an immoral act that tarnishes the image of the court, thus she should not be allowed to
remain employed therein as it might appear that the court condones her act.

Respondent Escritor testified that when she entered the judiciary in 1999, she was already a widow,
her husband having died in 1998. She admitted that she has been living with Luciano Quilapio, Jr.
without the benefit of marriage for twenty years and that they have a son. But as a member of the
religious sect known as the Jehovah's Witnesses and the Watch Tower and Bible Tract Society, their
conjugal arrangement is in conformity with their religious beliefs. In fact, after ten years of living
together, she executed on July 28, 1991 a "Declaration of Pledging Faithfulness," insofar as the
congregation is concerned, there is nothing immoral about the conjugal arrangement between Escritor
and Quilapio and they remain members in good standing in the congregation.

ISSUE: Whether there is compelling state interest to prosecute Excritor

HELD:
Benevolent neutrality recognizes that government must pursue its secular goals and interests but at
the same time strives to uphold religious liberty to the greatest extent possible within flexible
constitutional limits. Thus, although the morality contemplated by laws is secular, benevolent
neutrality could allow for accommodation of morality based on religion, provided it does not offend
compelling state interests. It still remains to be seen if respondent is entitled to such doctrine as the
state has not been afforded the chance has demonstrate the compelling state interest of prohibiting the
act of respondent, thus the case is remanded to the RTC.

Benevolent neutrality is inconsistent with the Free Exercise Clause as far as it prohibits such exercise
given a compelling state interest. It is the respondent’s stance that the respondent’s conjugal
arrangement is not immoral and punishable as it comes within the scope of free exercise protection.
Should the Court prohibit and punish her conduct where it is protected by the Free Exercise Clause, the
Court’s action would be an unconstitutional encroachment of her right to religious freedom. The Court
cannot therefore simply take a passing look at respondent’s claim of religious freedom, but must instead
apply the “compelling state interest” test. The government must be heard on the issue as it has not
been given an opportunity to discharge its burden of demonstrating the state’s compelling interest
which can override respondent’s religious belief and practice.
c) Conscientious Objector Test: Persons who are conscientiously opposed to participation in war in
any form by reason of religious training and belief may be exempted from combat training and service
in the armed forces. Religious training and belief means an individual’s belief in relation to a Supreme
Being involving duties superior to those arising from any human relation, but does not include
essentially political, sociological or philosophical views or a merely personal code.

Victoriano v. Elizalde, 59 SCRA 94

Victoriano, an Iglesia ni Cristo member, has been an employee of the Elizalde Rope Factory (ERF) since
1958. He was also a member of the EPWU (this is a union). Under the CBA between ERF and EPWU, a
close shop agreement is being enforced which means that employment in the factory relies on the
membership in the EPWU; that in order to retain employment in the said factory one must be a
member of the said Union. In 1962, Victoriano tendered his resignation from EPWU claiming that as per
RA 3350 he is an exemption to the close shop agreement by virtue of his being a member of the INC
because apparently in the INC, one is forbidden from being a member of any labor union. It was only in
1974 that his resignation from the Union was acted upon by EPWU which notified ERF about it. ERF then
moved to terminate Victoriano due to his non-membership from the EPWU. EPWU and ERF reiterated
that he is not exempt from the close shop agreement because RA 3350 is unconstitutional and that
said law violates the EPWU’s and ERF’s legal/contractual rights.

ISSUE: Whether RA 3350 is unconstitutional.

HELD: The right to religion prevails over contractual or legal rights. As such, an INC member may
refuse to join a labor union and despite the fact that there is a close shop agreement in the factory
where he was employed, his employment could not be validly terminated for his non-membership in
the majority therein. Further, the right to join a union includes the right not to join a union. The law is
not unconstitutional. It recognizes both the rights of unions and employers to enforce terms of contracts
and at the same time it recognizes the workers’ right to join or not to join union. But the RA recognizes
as well the primacy of a constitutional right over a contractual right.

Cantwell v. Connecticut, 310 US 296

Facts of the Case

Jesse Cantwell and his son were Jehovah's Witnesses; they were proselytizing (means attempting to
convert) a predominantly Catholic neighborhood in Connecticut. The Cantwells distributed religious
materials by travelling door-to-door and by approaching people on the street. After voluntarily hearing
an anti-Roman Catholic message on the Cantwells' portable phonograph, two pedestrians reacted
angrily. The Cantwells were subsequently arrested for violating a local ordinance requiring a permit
for solicitation and for inciting a breach of the peace.

Question

Did the solicitation statute or the "breach of the peace" ordinance violate the Cantwells' First
Amendment free speech or free exercise rights?
Conclusion

Yes. In a unanimous decision, the Court held that while general regulations on solicitation were
legitimate, restrictions based on religious grounds were not. Because the statute allowed local officials
to determine which causes were religious and which ones were not, it violated the First and Fourteenth
Amendments. The Court also held that while the maintenance of public order was a valid state
interest, it could not be used to justify the suppression of "free communication of views." The
Cantwells' message, while offensive to many, did not entail any threat of "bodily harm" and was
protected religious speech.

US v. Ballard – 322 US 78 - Kaye


US v. Ballard – 322 US 78

Facts:
Guy Ballard was convicted of using and conspiring to use mails to defraud. He was a follower
of the 'I Am' movement and believed that the words of St. Germain, the divine messenger, were
transmitted through him. Ballard also claimed to possess the power to heal people and claimed to have
had success in doing so in the past. He solicited contributions via mail in exchange for offering his
healing abilities. The government asserted that he 'well knew' that these claims were false and he used
them to defraud others of their money. In the initial trial, the jury was told not to consider Ballard's
religious beliefs; instead they were merely to determine whether the defendant believed that he
possessed the ability to heal others.

Issue:
Is Respondent being unconstitutionally persecuted for his religious beliefs?

Held:
The First Amendment protects the right to choose and practice a religion of the individual’s
choice. This includes freedom to believe and freedom to act. Just because a religious doctrine cannot be
proven does not mean that it is not a religion and not protected by the United States Constitution.

American Bible Society v. City of Manila – 104 Phil. 386 KAYE

The constitutional guaranty of the free exercise and enjoyment of religious profession and worship
carries with it the right to disseminate religious information. Any restraint of such right could only be
justified like other restraints of freedom of expression on the grounds that there is clear and present
danger of any substantive evil which the State has the right to prevent.

In the course of its ministry, plaintiff's Philippine agency has been distributing and selling bibles and/or
gospel portions thereof (except during the Japanese occupation) throughout the Philippines and
translating the same into several Philippine dialects. On May 29 1953, the acting City Treasurer of the
City of Manila informed plaintiff that it was conducting the business of general merchandise since
November, 1945, without providing itself with the necessary Mayor's permit and municipal license, in
violation of Ordinance No. 3000, as amended, and Ordinances Nos. 2529, 3028 and 3364, and required
plaintiff to secure, within three days, the corresponding permit and license fees, together with
compromise covering the period from the 4th quarter of 1945 to the 2nd quarter of 1953, in the total
sum of P5,821.45 (Annex A).
Issue: Whether plaintiff may be compelled to acquire a permit in violation of free exercise of religion

Held: No. It may be true that in the case at bar the price asked for the bibles and other religious
pamphlets was in some instances a little bit higher than the actual cost of the same but this cannot
mean that appellant was engaged in the business or occupation of selling said "merchandise" for
profit. For this reason. The Court believe that the provisions of City of Manila Ordinance No. 2529, as
amended, cannot be applied to appellant, for in doing so it would impair its free exercise and
enjoyment of its religious profession and worship as well as its rights of dissemination of religious
beliefs.

With respect to Ordinance No. 3000, as amended, the Court do not find that it imposes any charge upon
the enjoyment of a right granted by the Constitution, nor tax the exercise of religious practices.

It seems clear, therefore, that Ordinance No. 3000 cannot be considered unconstitutional, however
inapplicable to said business, trade or occupation of the plaintiff. As to Ordinance No. 2529 of the City of
Manila, as amended, is also not applicable, so defendant is powerless to license or tax the business of
plaintiff Society.

WHEREFORE, defendant shall return to plaintiff the sum of P5,891.45 unduly collected from it.

Ebranilag v. Divison Superintendent – 219 SCRA 256; (MR) 251 SCRA

Facts:
All the petitioners in these two cases were expelled from their classes by the public school
authorities in Cebu for refusing to salute the flag, sing the national anthem and recite the patriotic
pledge. Jehovah's Witnesses admittedly teach their children not to salute the flag, sing the national
anthem, and recite the patriotic pledge for they believe that those are "acts of worship" or "religious
devotion" which they "cannot conscientiously give to anyone or anything except God”. They feel
bound by the Bible's command to "guard ourselves from idols. They consider the flag as an image or
idol representing the State. They think the action of the local authorities in compelling the flag salute
and pledge transcends constitutional limitations on the State's power and invades the sphere of the
intellect and spirit which the Constitution protects against official control.

Issue:
Whether school children who are members of a religious sect known as Jehovah's Witnesses
may be expelled from school (both public and private), for refusing, on account of their religious beliefs,
to take part in the flag ceremony which includes playing (by a band) or singing the Philippine national
anthem, saluting the Philippine flag and reciting the patriotic pledge

Held:
NO. Religious freedom is a fundamental right which is entitled to the highest priority and the
amplest protection among human rights, for it involves the relationship of man to his Creator. The right
to religious profession and worship has a two-fold aspect, freedom to believe and freedom to act on
one's belief. The first is absolute as long as the belief is confined within the realm of thought. The
second is subject to regulation where the belief is translated into external acts that affect the public
welfare. Since they do not engage in disruptive behavior, there is no warrant for their expulsion. The
sole justification for a prior restraint or limitation on the exercise of religious freedom is the existence of
a grave and present danger of a character both grave and imminent, of a serious evil to public safety,
public morals, public health or any other legitimate public interest, that the State has a right (and duty)
to prevent." Absent such a threat to public safety, the expulsion of the petitioners from the schools is
not justified. We are not persuaded that by exempting the Jehovah's Witnesses, this religious which
admittedly comprises a "small portion of the school population" will shake up our part of the globe and
suddenly produce a nation "untaught and uninculcated in and unimbued with reverence for the flag,
patriotism, love of country and admiration for national heroes. After all, what the petitioners seek only
is exemption from the flag ceremony, not exclusion from the public schools where they may study the
Constitution, the democratic way of life and form of government, and learn not only the arts, science,
Philippine history and culture but also receive training for a vocation or profession and be taught the
virtues of "patriotism, respect for human rights, appreciation for national heroes, the rights and duties
of citizenship, and moral and spiritual values. Forcing a small religious group, through the iron hand of
the law, to participate in a ceremony that violates their religious beliefs, will hardly be conducive to love
of country or respect for duly constituted authorities. The expulsion of members of Jehovah's Witnesses
from the schools where they are enrolled will violate their right as Philippine citizens, under the 1987
Constitution, to receive free education, for it is the duty of the State to "protect and promote the right
of all citizens to quality education and to make such education accessible to all. While the highest regard
must be afforded their right to the exercise of their religion, "this should not be taken to mean that
school authorities are powerless to discipline them" if they should commit breaches of the peace by
actions that offend the sensibilities, both religious and patriotic, of other persons. If they quietly stand at
attention during the flag ceremony while their classmates and teachers salute the flag, sing the national
anthem and recite the patriotic pledge, we do not see how such conduct may possibly disturb the peace,
or pose "a grave and present danger of a serious evil to public safety, public morals, public health or any
other legitimate public interest that the State has a right. The petition for certiorari and prohibition is
GRANTED. The expulsion orders Issued by the public respondents against the petitioners are hereby
ANNULLED AND SET ASIDE.

Wisconsin v. Yoder – 406 US 205

Facts:
Jonas Yoder and Wallace Miller, both members of the Old Order Amish religion, and Adin
Yutzy, a member of the Conservative Amish Mennonite Church, were prosecuted under a Wisconsin
law that required all children to attend public schools until age 16. The three parents refused to send
their children to such schools after the eighth grade, arguing that high school attendance was contrary
to their religious beliefs.

Issue:
Did Wisconsin's requirement that all parents send their children to school at least until age 16
violate the First Amendment by criminalizing the conduct of parents who refused to send their children
to school for religious reasons?

Held: YES. In a unanimous decision, the Court held that individual's interests in the free exercise of
religion under the First Amendment outweighed the State's interests in compelling school attendance
beyond the eighth grade. In the majority opinion by Chief Justice Warren E. Burger, the Court found
that the values and programs of secondary school were "in sharp conflict with the fundamental mode
of life mandated by the Amish religion," and that an additional one or two years of high school would
not produce the benefits of public education cited by Wisconsin to justify the law.

Goldman v. Weinberger – 54 LW 4298

Facts:
Goldman joined the United States Air Force as an inactive reserve in 1973. He received a
Health Professions scholarship to work towards a PhD in Psychology at the Loyola University of
Chicago. Subsequently, Goldman entered service at March Air Force Base in Riverside, California as a
commissioned officer and clinical psychologist at the on-base mental health clinic. As an Orthodox Jew
and rabbi, Goldman's faith required him to wear a yarmulke to show that he is aware that God is a
higher power and above him.

For years, Goldman wore his yarmulke without controversy by staying near his station at the
clinic and wearing his service cap above the yarmulke while outdoors. In 1981, however, he was
required to testify as a defense witness at a court-martial. As a result, the prosecution witness was
discredited, and subsequently a government attorney lodged a complaint. Subsequently, his
commanding officer at the hospital, Colonel Joseph Gregory, informed him that he was violating Air
Force Regulation 35-10, which states that "headgear will not be worn... while indoors except by armed
security police in the performance of their duties." The officer then ordered him to not wear the
yarmulke while in uniform outside the hospital.

Goldman refused this order, and instead his attorney filed a complaint to the Air Force General
Counsel. Gregory then ordered that Goldman cease wearing his yarmulke even when within the
hospital. Goldman requested to be allowed to report for duty in civilian clothes until the Issue was
settled in court, but he was denied this and was threatened with court-martial. It was at this point that
Goldman sued the Secretary of Defense, Caspar Weinberger for Free Exercise Clause violations. He
was favored at the District Court of Washington, D.C., but that decision was reversed in the Court of
Appeals. The Supreme Court granted the writ of certiorari.

Held:
The Supreme Court upheld the military provision by a 5-4 vote.

Because Goldman alleged that this was a Free Exercise violation, he indicated that the defense
had to pass the Sherbert test- by demonstrating a "compelling interest" for the violation. He then
submitted evidence that there was not a compelling interest for preventing the display of religious
apparel, because it presented no danger to military discipline. However, the majority opinion, written by
Rehnquist, held that this was of no consequence- it contended that the Sherbert test did not apply
because the Free Exercise Clause and even the First Amendment in general did not apply to the military
in the same way that it did to civilian society. The justification for this was a need to "foster instinctive
obedience, unity, commitment, and esprit de corps."

Our review of military regulations challenged on First Amendment grounds is far more
deferential than constitutional review of similar laws or regulations designed for civilian society.
...[W]hen evaluating whether military needs justify a particular restriction on religiously-motivated
conduct, courts must give great deference to the professional judgment of military authorities
concerning the relative importance of a particular military interest.

Military officials were not constitutionally required to condition their professional decisions to
the Constitution because the military was found to be a "specialized society separate from civilian
society" and "to accomplish its mission the military must foster instinctive obedience, unity,
commitment, and esprit de corps."

Things like obedience, unity, commitment, and esprit de corps are, in turn, fostered by a sense
of cohesivenes and common purpose among its members. An important means by which cohesiveness
is created by the military is through strict regulation and commonality of the uniforms worn. Members
are expected to subordinate their own desires and beliefs to the larger needs and objectives of the
service. Thus, headgear worn for personal reasons could not be permitted.

Significance:
The purpose of the military and its need to foster cohesiveness were regarded as appropriate
justifications to restrict the religious rights of individuals. The Court did not try to evaluate the merit of
military's claims about the importance of regulating the appearance of its members, deciding that the
military should be left to make such final decisions rather than the courts.

This was one of the first cases decided by the "Reagan Court" - that is to say, a court made up
mostly by justice appointed by President Ronald Reagan. This court consistently refused to apply the
"compelling interest" test in questions of religious freedom and thus consistently found in favor of the
government infringing upon the liberty of religious minorities based upon weaker justifications.

German v. Baranganan – 135 SCRA 514

Facts:
On Oct 2, 1984, petitioners composed of about 50 businessmen, students and office employees
and who were members of the August Twenty-One Movement (ATOM), converged at J.P. Laurel Street,
Manila, for the purpose of hearing Mass at the St. Jude Chapel, which adjoins the Malacañang grounds
located in the same street. Wearing yellow t-shirts, they marched down said street with raised clenched
fists and shouts of anti-government invectives. Along the way, however, they were barred by
respondent Major lsabelo Lariosa, upon orders of his superior and co-respondent Gen. Santiago
Barangan, from proceeding any further, on the ground that St. Jude Chapel was located within the
Malacañang security area. When their efforts to enter the church became apparently futile, they opted
to stay outside, kneeling on the sidewalk in front of the barricades and prayed the Holy Rosary.
Afterwards, they sang Bayan ko with clenched fists of protest against the violation of their rights and
thereafter dispersed peacefully. Because of the alleged warning given them by respondent Major
Lariosa that any similar attempt by petitioners to enter the church in the future would likewise be
prevented, petitioners took this present recourse.

Issue/ Held:
1. WON petitioners’ constitutionally protected freedom to exercise religion
(Sec 8, Art IV of the 1973 Consti) was violated NO
2. WON petitioners’ freedom of locomotion was violated (Sec 5, Art IV of the 1973
Consti) NO

RATIO:
1. Petitioners' alleged purpose in converging at J.P. Laurel Street was to pray and hear
mass at St. Jude church. At the hearing of this petition, respondents assured petitioners and the Court
that they have never restricted, and will never restrict, any person or persons from entering and
worshipping at said church. They maintain, however, that petitioners' intention was not really to
perform an act of religious worship, but to conduct an anti-government demonstration at a place close
to the very residence and offices of the President of the Republic. Respondents further lament
petitioners' attempt to disguise their true motive with a ritual as sacred and solemn as the Holy Sacrifice
of the Mass. Undoubtedly, the yellow t-shirts worn by some of the marchers, their raised clenched
fists, and chants of anti-government slogans strongly tend to substantiate respondents allegation.

These allegations cannot but cast serious doubts on the sincerity and good faith of petitioners
in invoking the constitutional guarantee of freedom of religious worship and of locomotion. While it is
beyond debate that every citizen has the undeniable and inviolable right to religious freedom, the
exercise thereof, and of all fundamental rights for that matter, must be done in good faith. As Art 19 of
the Civil Code admonishes: "Every person must, in the exercise of his rights and in the performance of
his duties… observe honesty and good faith."

Even assuming that petitioners' claim to the free exercise of religion is genuine and valid, still
respondents reaction to the October 2, 1984 mass action may not be characterized as violative of the
freedom of religious worship. Since 1972, when mobs of demonstrators crashed through the
Malacañang gates and scaled its perimeter fence, the use by the public of J.P. Laurel Street and the
streets approaching it have been restricted. While travel to and from the affected thoroughfares has not
been absolutely prohibited, passers-by have been subjected to courteous, unobtrusive security checks.
The reasonableness of this restriction is readily perceived and appreciated if it is considered that the
same is designed to protect the lives of the President and his family, as well as other government
officials, diplomats and foreign guests transacting business with Malacañang. The restriction is also
intended to secure the several executive offices within the Malacañang grounds from possible external
attacks and disturbances. These offices include communications facilities that link the central
government to all places in the land. Unquestionably, the restriction imposed is necessary to maintain
the smooth functioning of the executive branch of the government, which petitioners' mass action
would certainly disrupt.

Cantwell v Connecticut: The constitutional inhibition on legislation on the subject of religion has
a double aspect. On the one hand, it forestalls compulsion by law of the acceptance of any creed or the
practice of any form of worship. Freedom of conscience and freedom to adhere to such religious
organization or form of worship as the individual may choose cannot be restricted by law. On the other
hand, it safeguards the free exercise of the chosen form of religion. Thus the amendment embraces two
concepts –freedom to believe and freedom to act. The first is absolute, but in the nature of things, the
second cannot be.

In the case at bar, petitioners are not denied or restrained of their freedom of belief or choice of
their religion, but only in the manner by which they had attempted to translate the same into action.
This curtailment is in accord with the pronouncement of this Court in Gerona v. Secretary of Education,
thus:

The realm of belief and creed is infinite and limitless bounded only by one's imagination and
thought. So is the freedom of belief, including religious belief, limitless and without bounds. One may
believe in most anything, however strange, bizarre and unreasonable the same may appear to others,
even heretical when weighed in the scales of orthodoxy or doctrinal standards. But between the
freedom of belief and the exercise of said belief, there is quite a stretch of road to travel. If the exercise
of said religious belief clashes with the established institutions of society and with the law, then the
former must yield and give way to the latter. The government steps in and either restrains said exercise
or even prosecutes the one exercising it.

2. Suffice it to say that the restriction imposed on the use of J.P. Laurel Street, the wisdom
and reasonableness of which have already been discussed, is allowed under the fundamental law, the
same having been established in the interest of national security.

Petition dismissed.

Teehankee, dissenting:
I vote to grant the petition on the ground that the right of free worship and movement is a
preferred right that enjoys precedence and primacy and is not subject to prior restraint except where
there exists the clear and present danger of a substantive evil sought to be prevented. There was and is
manifestly no such danger in this case.

1. The right to freely exercise one's religion is guaranteed in Section 8 of our Bill of Rights.
Freedom of worship, alongside freedom of expression and speech and peaceable assembly, along with
the other intellectual freedoms, are highly ranked in our scheme of constitutional values. It cannot be
too strongly stressed that on the judiciary—even more so than on the other departments—rests the
grave and delicate responsibility of assuring respect for and deference to such preferred rights. No
verbal formula, no sanctifying phrase can, of course, dispense with what has been so felicitously termed
by Justice Holmes 'as the sovereign prerogative of judgment.' Nonetheless, the presumption must be to
incline the weight of the scales of justice on the side of such rights, enjoying as they do precedence and
primacy."

2. In the free exercise of such preferred rights, there is to be no prior restraint although there
may be subsequent punishment of any illegal acts committed during the exercise of such basic rights.
The sole justification for a prior restraint or limitation on the exercise of these basic rights is the
existence of a grave and present danger of a character both grave and imminent, of a serious evil to
public safety, public morals, public health or any other legitimate public interest, that the State has a
right (and duty) to prevent.

3. The burden to show the existence of grave and imminent danger that would justify prior
restraint and bar a group of persons from entering the church of their choice for prayer and worship lies
on the military or police officials who would so physically restrain them. Indeed, there is no precedent in
this time and age where churchgoers whose right of free exercise of their religion is recognized have
been physically prevented from entering their church on grounds of national security. On the other
hand, it does not lie within the competence or authority of such officials to demand of churchgoers that
they show and establish their "sincerity and good faith… in invoking the constitutional guarantee of
freedom of religious worship and of locomotion" as a pre-condition, as seems to be the thrust of the
majority decision.

Respondents themselves in the Solicitor General's comment admit that "true, there were only
about 80 persons in petitioners' group on October 2 and this number could hardly pose the danger
feared," but expressed the fear that petitioners' ranks could within hours reach hundreds if not
thousands and "peaceful dispersal becomes impossible as in recent demonstrations and rallies."
Respondents were in full control and there is no question as to the capability of the security forces to
ward off and stop any untoward move. They had placed an advance checkpoint as far back as the Sta.
Mesa Rotonda and could stop the flow of people in the church if they deemed it unmanageable. There
definitely was no clear and present danger of any serious evil to public safety or the security of
Malacañang.

Tolentino v. Sec. of Finance – 249 SCRA 628 - Xavier


FACTS: The value-added tax (VAT) is levied on the sale, barter or exchange of goods and properties as
well as on the sale or exchange of services. RA 7716 seeks to widen the tax base of the existing VAT
system and enhance its administration by amending the National Internal Revenue Code. The
Philippine Bible Society, Inc. claims that although it sells bibles, the proceeds derived from the sales
are used to subsidize the cost of printing copies which are given free to those who cannot afford to
pay so that to tax the sales would be to increase the price, while reducing the volume of sale.
ISSUE: Whether VAT infringes upon the free exercise of religion.
HELD: NO. Granting that to be the case, the resulting burden on the exercise of religious freedom is so
incidental as to make it difficult to differentiate it from any other economic imposition that might
make the right to disseminate religious doctrines costly. Otherwise, to follow the petitioner's
argument, to increase the tax on the sale of vestments would be to lay an impermissible burden on the
right of the preacher to make a sermon.

On the other hand the registration fee of P1,000.00 imposed by §107 of the NIRC, as amended
by §7 of R.A. No. 7716, although fixed in amount, is really just to pay for the expenses of registration and
enforcement of provisions such as those relating to accounting in §108 of the NIRC. That the PBS
distributes free bibles and therefore is not liable to pay the VAT does not excuse it from the payment of
this fee because it also sells some copies. At any rate whether the PBS is liable for the VAT must be
decided in concrete cases, in the event it is assessed this tax by the Commissioner of Internal Revenue.
Centeno v. Villalon-Pornillos – 236 SCRA 197 - Xavier
FACTS: Petitioner Centeno et al, officers of a civic organization known as the Samahang Katandaan ng
Nayon ng Tikay launched a fund drive for the purpose of renovating the chapel of Barrio Tikay,
Bulacan, and solicited from Judge Adoracion G. Angeles a contribution of P1,500.00. It is admitted that
the solicitation was made without a permit from the Department of Social Welfare and Development.
An information was filed against petitioner for violation of Presidential Decree No. 1564, or the
Solicitation Permit Law.

Petitioner avers that solicitations for religious purposes cannot be penalized under the law
for, otherwise, it will constitute an abridgment or restriction on the free exercise clause guaranteed
under the Constitution.
ISSUE: Whether to subject to State regulation solicitations made for a religious purpose would constitute
an abridgment of the right to freedom of religion guaranteed under the Constitution.
HELD: NO. Solicitation for religious purposes may be subject to proper regulation by the State in the
exercise of police power. However, in the case at bar, considering that solicitations intended for a
religious purpose are not within the coverage of Presidential Decree No. 1564, petitioner cannot be
held criminally liable therefor.

Church of the Lukumi v. City of Hialeach – No. 91-948, June 11, 1993 - Xavier
FACTS: The Church of Lukumi Babalu Aye practiced the Afro-Caribbean-based religion of Santeria.
Santeria used animal sacrifice as a form of worship in which an animal's carotid arteries would be cut
and, except during healing and death rights, the animal would be eaten. Shortly after the
announcement of the establishment of a Santeria church in Hialeah, Florida, the city council adopted
several ordinances addressing religious sacrifice. The ordinances prohibited possession of animals for
sacrifice or slaughter, with specific exemptions for state-licensed activities.
ISSUE: Whether the city of Hialeah's ordinance, prohibiting ritual animal sacrifices, violate the First
Amendment's Free Exercise Clause.
HELD: YES. The Court held that the ordinances were neither neutral nor generally applicable. The
ordinances had to be justified by a compelling governmental interest and they had to be narrowly
tailored to that interest. The core failure of the ordinances were that they applied exclusively to the
church. The ordinances singled out the activities of the Santeria faith and suppressed more religious
conduct than was necessary to achieve their stated ends. Only conduct tied to religious belief was
burdened. The ordinances targeted religious behavior, therefore they failed to survive the rigors of strict
strutiny.
Lamb’s Chapel v. School Disctrict – No.91-2024, June 7, 1993 - Xavier
FACTS: A New York law authorized schools to regulate the after-hour use of school property and
facilities. The Center Moriches School District, acting under the statute, prohibited the use of its
property by any religious group. The District refused repeated requests by Lamb's Chapel to use the
school's facilities for an after-hours religious-oriented film series on family values and child rearing.
The Chapel brought suit against the School District in federal court.
ISSUE: Whether the District violate the First Amendment's Free Exercise Clause when it denied Lamb's
Chapel the use of school premises to show religious-oriented films.
HELD: YES. The Supreme Court's holding consisted of two parts. First, the District violated freedom of
speech by refusing the Chapel's request to show movies on school premises solely because such
movies were religiously oriented. While non-public schools are permitted under New York law to
restrict access to their premises based on subject matter or speaker identity, such restrictions must be
reasonable and "viewpoint neutral." In this case, the District's restriction was neither reasonable nor
viewpoint neutral, since it allowed the presentation of all other views about family values and child
rearing - except those which were presented from a religious perspective. Second, a grant of permission
to the Chapel to use the District's premises would not have amounted to an establishment of religion.
This is because the showing of the films would neither be school-sponsored during school hours nor
closed to the public.
In re Request of Muslim Employees in the Different Court of Iligan City, 477 SCRA 648 – Xavier
FACTS: Several Muslim employees in the different courts in the Iligan city request that they be allowed
to enjoy the following privileges:
1. to hold office hours from 7:30 a.m. to 3:30 p.m. without lunch break or coffee breaks during the
month of Ramadan;
2. to be excused from work from 10:00 a.m. to 2:00 p.m. every Friday (Muslim Prayer Day) during the
entire calendar year.
Judge Salazar forwarded the said letter-request to the Office of the Court Administrator
(OCA). He expressed his conformity with the first request, i.e., allowing them to hold office from 7:30
a.m. to 3:30 p.m. without any break during the month of Ramadan. However, he expressed some
misgivings about the second request, i.e., excusing them from work from 10:00 a.m. to 2:00 p.m. every
Friday during the entire calendar year.
CSC promulgated Resolution No. 81-1277 and Resolution No. 00-0227 that Muslim employees
are excused from work from 10:00 a.m. to 2:00 p.m. every Friday subject to certain conditions.
ISSUE: Whether the employees may be exempted from compliance with the prescribed government
working hours by reason of the free exercise clause.
HELD: QUALIFED. The Court recognizes that the observance of Ramadan and the Friday Muslim Prayer
Day is integral to the Islamic faith. However, while the observance of Ramadan and allowing the
Muslim employees in the Judiciary to hold flexible office hours from 7:30 a.m. to 3:30 p.m. without any
break during the month of Ramadan finds support in Section 3 (a) of P.D. No. 291, as amended by P.D.
No. 322, there is no such basis to excuse them from work from 10:00 a.m. to 2:00 p.m. every Friday,
the Muslim Prayer Day, during the entire calendar year.
Section 5, Rule XVII of the Omnibus Rules Implementing Book V of E.O. No. 292 enjoins all civil
servants, of whatever religious denomination, to render public service of no less than eight hours a day
or forty (40) hours a week. The performance of religious practices, whether by the Muslim employees
or those belonging to other religious denominations, should not prejudice the courts and the public. In
fine, the remedy of the Muslim employees, with respect to their request to be excused from work from
10:00 a.m. to 2:00 p.m. every Friday during the entire calendar year, is legislative, which is to ask
Congress to enact a legislation expressly exempting them from compliance with the prescribed
government working hours.
Estrada v. Escritor – AM P-021651, August 4, 2003 (Compelling State Interest Test) - Xavier
FACTS: Complainant Alejandro Estrada wrote to Judge Caoibes, Jr., presiding judge of RTC that
respondent Soledad Escritor, court interpreter in said court, is living with a man not her husband and as
such should be administratively liable and that to allow her to remain employed therein will appear as
if the court allows such act.
Respondent Escritor, testified that when she entered the judiciary in 1999, she was already a
widow, her husband having died in 1998. She admitted that she has been living with Luciano Quilapio,
Jr. without the benefit of marriage for twenty years and that they have a son. But as a member of the
religious sect known as the Jehovah’s Witnesses and the Watch Tower and Bible Tract Society, their
conjugal arrangement is in conformity with their religious beliefs. In fact, after ten years of living
together, she executed a “Declaration of Pledging Faithfulness.”
ISSUE: Whether the respondent’s right to religious freedom should carve out an exception from the
prevailing jurisprudence on illicit relations for which government employees are held administratively
liable.
HELD: Should the Court prohibit and punish her conduct where it is protected by the Free Exercise
Clause, the Court’s action would be an unconstitutional encroachment of her right to religious freedom.
We cannot therefore simply take a passing look at respondent’s claim of religious freedom, but must
instead apply the “compelling state interest” test.

The case is remanded to the Office of the Court Administrator and the Solicitor General is
ordered to intervene in the case to properly settle the issue in the case at bar, the government should
be given the opportunity to demonstrate the “compelling state interest” it seeks to uphold in opposing
the respondent’s stance that her conjugal arrangement is not immoral and punishable as it comes within
the scope of free exercise protection.

Imbong v. Ochoa, GR 204819, April 8, 2014 - JC

Facts: As to Freedom of Religion: Petitioners contend that the RH Law violates the right to religious
freedom. The petitioners contend that the RH Law violates the constitutional guarantee respecting
religion as it authorizes the use of public funds for the procurement of contraceptives. For the
petitioners, the use of public funds for purposes that are believed to be contrary to their beliefs is
included in the constitutional mandate ensuring religious freedom.

It is also contended that the RH Law threatens conscientious objectors of criminal prosecution,
imprisonment and other forms of punishment, as it compels medical practitioners 1] to refer patients
who seek advice on reproductive health programs to other doctors; and 2] to provide full and correct
information on reproductive health programs and service, although it is against their religious beliefs
and convictions.

Issue: Whether the RH Law infringes on the right of free exercise of religion

Held: No. Exception: Life Threatening Cases

The Court properly recognizes a valid exception set forth in the law. While generally healthcare service
providers cannot be forced to render reproductive health care procedures if doing it would contravene
their religious beliefs, an exception must be made in life-threatening cases that require the
performance of emergency procedures. In these situations, the right to life of the mother should be
given preference, considering that a referral by a medical practitioner would amount to a denial of
service, resulting to unnecessarily placing the life of a mother in grave danger. Thus, during the oral
arguments, Atty. Liban, representing CFC, manifested: "the forced referral clause that we are objecting
on grounds of violation of freedom of religion does not contemplate an emergency."

In a conflict situation between the life of the mother and the life of a child, the doctor is morally obliged
always to try to save both lives. If, however, it is impossible, the resulting death to one should not be
deliberate.

Principle of Double-Effect. - May we please remind the principal author of the RH Bill in the House of
Representatives of the principle of double-effect wherein intentional harm on the life of either the
mother of the child is never justified to bring about a "good" effect. In a conflict situation between the
life of the child and the life of the mother, the doctor is morally obliged always to try to save both lives.
However, he can act in favor of one (not necessarily the mother) when it is medically impossible to save
both, provided that no direct harm is intended to the other. If the above principles are observed, the
loss of the child's life or the mother's life is not intentional and, therefore, unavoidable. Hence, the
doctor would not be guilty of abortion or murder. The mother is never pitted against the child because
both their lives are equally valuable.

Accordingly, if it is necessary to save the life of a mother, procedures endangering the life of the child
may be resorted to even if is against the religious sentiments of the medical practitioner. As quoted
above, whatever burden imposed upon a medical practitioner in this case would have been more than
justified considering the life he would be able to save.

Family Planning Seminars

Anent the requirement imposed under Section 15 as a condition for the issuance of a marriage license,
the Court finds the same to be a reasonable exercise of police power by the government. A cursory
reading of the assailed provision bares that the religious freedom of the petitioners is not at all violated.
All the law requires is for would-be spouses to attend a seminar on parenthood, family planning
breastfeeding and infant nutrition. It does not even mandate the type of family planning methods to be
included in the seminar, whether they be natural or artificial. As correctly noted by the OSG, those who
receive any information during their attendance in the required seminars are not compelled to accept
the information given to them, are completely free to reject the information they find unacceptable,
and retain the freedom to decide on matters of family life without the intervention of the State.

III. No Religious Test

Torcaso v. Watkins, 367 US 488 - JC

Facts: Appellant was appointed by the Governor of Maryland to the office of Notary Public, but he was
denied a commission because he would not declare his belief in God, as required by the Maryland
Constitution. Claiming that this requirement violated his rights under the First and Fourteenth
Amendments, he sued in a state court to compel issuance of his commission, but relief was denied. The
State Court of Appeals affirmed, holding that the state constitutional provision is self-executing,
without need for implementing legislation, and requires declaration of a belief in God as a qualification
for office.

Issue: Whether the declaration of belief in God is a religious test

Held: This Maryland test for public office cannot be enforced against appellant, because it
unconstitutionally invades his freedom of belief and religion guaranteed by the First Amendment and
protected by the Fourteenth Amendment from infringement by the States.

Pamil v. Teleron – 86 SCRA 413 - JC

Facts: In 1971, Fr. Margarito Gonzaga, a priest, won the election for mayoralty in Albuquerque, Bohol.
He was also proclaimed as a mayor therein. Pamil, a rival candidate file a quo warranto case against
Gonzaga questioning the eligibility of Gonzaga. He argued that as provided for in the Revised
Administrative Code; “in no case shall there be elected or appointed to a municipal office ecclesiastics,
soldiers in active service, persons receiving salaries or compensation from provincial or national
funds, or contractors for public works of the municipality.” In this case, the elected mayor is a priest.
However, Judge Teleron ruled that the Administrative Code is repealed by the Election Code of 1971
which allowed the prohibitions of the revised administrative code.

Issue: Whether the prohibition regarding ecclesiastics is constitutional

Held: Kinda NO. Decision is indecisive, the said law, in the deliberations of the court, failed to obtain the
majority vote of eight (8) which is needed in order for this law to be binding upon the parties in this
case. For this, the petition must be granted and the decision of the lower court reversed and set aside.
Fr. Gonzaga is hereby ordered to vacate the mayoralty position. It is also pointed out that how can one
who swore to serve the Church’s interest above all be in duty to enforce state policies which at times
may conflict with church tenets. This is in violation of the separation of the church and state. The
Revised Administrative Code still stands because there is no implied repeal.

McDaniel v. Paty – 435 US 618 - JC

Facts: McDaniel was an ordained minister who was barred from serving in the Tennessee’s
constitutional convention. A statute prevented Ministers of the Gospel, or priest[s] of any
denomination whatever from taking part. McDaniel alleged that his First Amendment rights were
violated by the restriction.

Issue: Whether the Tennessee law infringed free exercise of religion

Held: Yes. The Supreme Court abolished clergy disqualification provisions, holding they
unconstitutionally violated ministers’ First Amendment free exercise rights. Religious liberty meant
clergy should not be forced to choose between their ministerial calling and serving the public through
office holding.

The Tennessee disqualification also violates the Establishment Clause. Government generally may not
use religion as a basis of classification for the imposition of duties, penalties, privileges, or benefits.
Specifically, government may not fence out from political participation, people such as ministers
whom it regards as overinvolved in religion. The disqualification provision employed by Tennessee here
establishes a religious classification that has the primary effect of inhibiting religion."

The disqualification of clergy from holding public office had a historical basis as eleven of the original
thirteen colonies had such provisions. The statute prevents McDaniel from simultaneously exercising
two of his fundamental rights. While Tennessee may have originally had a legitimate interest in keeping
clergy from participation, this interest has not been shown to exist any longer. Safeguards exist to
ensure that, if elected, clergy will not create too close an alliance between church and state.

Ang Ladlad v. COMELEC, GR 190582, April 8, 2010 - JC

Facts: Ang Ladlad argued that the denial of accreditation, insofar as it justified the exclusion by using
religious dogma, violated the constitutional guarantees against the establishment of
religion. Petitioner also claimed that the Assailed Resolutions contravened its constitutional rights to
privacy, freedom of speech and assembly, and equal protection of laws, as well as constituted
violations of the Philippines’ international obligations against discrimination based on sexual
orientation.
Issue: Whether the denial violated the non – establishment of religion

Held: Yes. Our Constitution provides in Article III, Section 5 that “[n]o law shall be made respecting an
establishment of religion, or prohibiting the free exercise thereof.” At bottom, what our non-
establishment clause calls for is “government neutrality in religious matters.”[24] Clearly,
“governmental reliance on religious justification is inconsistent with this policy of neutrality.”[25] We
thus find that it was grave violation of the non-establishment clause for the COMELEC to utilize the
Bible and the Koran to justify the exclusion of Ang Ladlad.

Rather than relying on religious belief, the legitimacy of the Assailed Resolutions should depend,
instead, on whether the COMELEC is able to advance some justification for its rulings beyond mere
conformity to religious doctrine.

IV. Ecclesiastical Matters

Austria v. NLRC, 310 SCRA 293 - JC

Facts: Petitioner was a minister with the Seventh Day Adventist. He was dismissed due to non –
remittance of tithes collection. Petitioner then filed a complaint for illegal dismissal and a decision was
rendered in his favor. The SDA appealed the same to the NLRC and after much ado, the case was
dismissed for lack of jurisdiction on the ground that the case involved an ecclessiastical affair to which
the State cannot interfere.

Issue: Whether the case is purely ecclesiastical

Held: No. The case at bar did not concern a purely religious affair as to bar the State from taking
cognizance thereof. What is involved here is the relationship of the church as an employer and the
minister as an employee. There was no compliance of the requirement that there should be a written
notice specifying the grounds for termination and giving the employee reasonable opportunity to
explain his side. Here, petitioner was not given enough opportunity to properly prepare for his
defense. At any rate, the validity of the dismissal cannot be sustained. There was no basis for the loss
of confidence and breach of trust as it was petitioner’s wife who collected the money and failed to remit
the same. On the ground of serious misconduct and commission of an offense against the person of the
employer’s duly authorized representative, the same was unmeritorious as petitioner’s actuations
cannot be considered grave enough to be considered as serious misconduct to merit the ultimate
penalty of dismissal. Then also, there was no proof that petitioner committed gross and habitual
neglect of duties. Hence, since petitioner was illegally dismissed, he is entitled to reinstatement with full
backwages.

ALFREDO LONG and FELIX ALMERIA,vs. LYDIA BASA, ANTHONY SAYHEELIAM and YAO CHEK [G.R. Nos.
134963-64. September 27, 2001]

Facts: The Board of Directors observed that certain members of the Church In Quezon City (CHURCH),
including petitioners herein, exhibited "conduct which was dishonorable, improper and injurious to
the character and interest of the CHURCH" by "introducing to the members doctrines and teachings
which were not based on the Holy Bible" and the Principles of Faith embraced by the CHURCH.
Petitioners were given warnings and later expelled.

Petitioners insist that the expulsion is void since it was rendered without prior notice to them or, in a
constitutional context, without due process.

On the other hand, respondents assert that the expulsion is in accordance with the By-laws of the
CHURCH.

Issue: Whether the court can rule on the validity of the expulsion which pertains to religious matters.

Ruling: No. It must be emphasized that the issue of the validity of the expulsion had long been resolved
and declared valid by the SEC en banc in its decision. The petitioners themselves admitted in their
present petition that they did not appeal anymore, thereby rendering the same final and conclusive. As
such, the expulsion order is now inextricably binding on the parties concerned and can no longer be
modified, much less reversed.

"…in matters purely ecclesiastical the decisions of the proper church tribunals are conclusive upon the
civil tribunals. A church member who is expelled from the membership by the church authorities, or a
priest or minister who is by them deprived of his sacred office, is without remedy in the civil courts,
which will not inquire into the correctness of the decisions of the ecclesiastical tribunals."

Dominador Taruc vs. Bishop Dela Cruz [G.R. No. 144801. March 10, 2005]

Facts: Petitioners were lay members of the Philippine Independent Church (PIC). Petitioners led by
Taruc clamoured for the transfer of parish priest Rustom Florano for the reason that Fr. Florano’s wife
belonged to a political party opposed to petitioner Taruc. Respondent did not find the reason as valid
and denied the request. Petitioner Taruc tried to organize an open mass to be celebrated by a certain
Fr. Renato Z. Ambong, not a member of the clergy. Bishop de la Cruz declared petitioners
expelled/excommunicated from the Philippine Independent Church. Because of the order of
expulsion/excommunication, petitioners filed a complaint for damages with preliminary injunction
against Bishop de la Cruz before the Regional Trial Court.They contended that their expulsion was
illegal because it was done without trial thus violating their right to due process of law.

Issue: Whether or not there was a violation of religious rights.

Ruling: No. The expulsion/excommunication of members of a religious institution/organization is a


matter best left to the discretion of the officials, and the laws and canons, of said
institution/organization. It is not for the courts to exercise control over church authorities in the
performance of their discretionary and official functions. Rather, it is for the members of religious
institutions/organizations to conform to just church regulations. “Civil Courts will not interfere in the
internal affairs of a religious organization except for the protection of civil or property rights. Those
rights may be the subject of litigation in a civil court, and the courts have jurisdiction to determine
controverted claims to the title, use, or possession of church property.” Obviously, there was no
violation of a civil right in the present case.
United Church Of Christ In The Philippines, Inc., V. Bradford United Church Of Christ, Inc. G.R. No.
171905, June 20, 2012

Facts: A long time ago (in a galaxy far far away) by circumstance, the Bradford Evangelical Church, a
separate entity from UCCP, transferred its synodical connection to and became a constituent Church of
the UCCP. BUCCI disaffiliated from UCCP due to the dispute of BUCCI started construction of a fence that
encroached upon the right-of way allocated by UCCP for Cebu conference inc. and Visayas jurisdiction.
UCCP filed before SEC a complaint/protest for rejection/annulment of Amended Articles and
Incorporation and disuse of BUCCI’s name. UCCP maintains that the issue on validity of disaffiliation of
respondents is purely an ecclesiastical affair. SEC upheld the right of BUCCI to continue using its
corporate name.

Issue: Whether the case involved is purely ecclesiastical thus the courts and SEC have no jurisdiction.

Ruling:No. An ecclesiastical affair is one that concerns doctrine, creed or form of worship of the
church, or the adoption and enforcement within a religious association of needful laws and
regulations for the government of the membership, and the power of excluding from such
associations those deemed unworthy of membership. SEC may exercise jurisdiction matters that are
legal and corporate.

Based on this definition, an ecclesiastical affair involves the relationship between the church and its
members and relate to matters of faith, religious doctrines, worship and governance of the
congregation. To be concrete, examples of this so-called ecclesiastical affairs to which the State
cannot meddle are proceedings for excommunication, ordinations of religious ministers,
administration of sacraments and other activities attached with religious significance.

UCCP and BUCCI, being corporate entities and grantees of primary franchises, are subject to the
jurisdiction of the SEC. Section 3 of Presidential Decree No. 902-A provides that SEC shall have absolute
jurisdiction, supervision and control over all corporations. Even with their religious nature, SEC may
exercise jurisdiction over them in matters that are legal and corporate.

Section 6. The liberty of abode and of changing the same within the limits prescribed by law shall not
be impaired except upon lawful order of the court. Neither shall the right to travel be impaired except
in the interest of national security, public safety, or public health, as may be provided by law.

Article 13 (2) Universal Declaration of Human Rights

Article 13.

 (1) Everyone has the right to freedom of movement and residence within the borders of each state.
 (2) Everyone has the right to leave any country, including his own, and to return to his country.

Article 12 (4) Covenant on Civil and Political Rights

Article 12

1. Everyone lawfully within the territory of a State shall, within that territory, have the right to liberty of
movement and freedom to choose his residence.
2. Everyone shall be free to leave any country, including his own.

3. The above-mentioned rights shall not be subject to any restrictions except those which are provided by
law, are necessary to protect national security, public order (ordre public), public health or morals or the
rights and freedoms of others, and are consistent with the other rights recognized in the present Covenant.

4. No one shall be arbitrarily deprived of the right to enter his own country.

Watch-list, hold departure orders and lookout order

Reverend Father Robert P. Reyes v. CA, GR 182161, December 3, 2009

Facts: Petitioner was among those arrested in the Manila Peninsula Hotel siege on November 30,
2007. DOJ Secretary Raul Gonzales issued Hold Departure Order (HDO) No. 45 ordering respondent
Commissioner of Immigration to include in the Hold Departure List of the Bureau of Immigration and
Deportation (BID) the name of petitioner and 49 others relative to the aforementioned case in the
interest of national security and public safety.

the RTC issued an Order dismissing the charge for Rebellion against petitioner and 17 others for lack of
probable cause. petitioner’s counsel wrote the DOJ Secretary requesting the lifting of HDO No. 45 in
view of the dismissal of Criminal Case No. 07-3126.

petitioner filed the instant petition claiming that despite the dismissal of the rebellion case against
petitioner, HDO No. 45 still subsists; petitioner was held by BID officials at the NAIA as his name is
included in the Hold Departure List; the Secretary of Justice has not acted on his request for the lifting
of HDO No. 45. Petitioner further maintained that immediate recourse to the Supreme Court for the
availment of the writ of amparo is exigent as the continued restraint on petitioner’s right to travel is
illegal.

The petition for a writ of amparo is anchored on the ground that respondents violated petitioner’s
constitutional right to travel. Petitioner argues that the DOJ Secretary has no power to issue a Hold
Departure Order (HDO) and the subject HDO No. 45 has no legal basis since Criminal Case No. 07-3126
has already been dismissed. Petitioner maintains that the writ of amparo does not only exclusively
apply to situations of extrajudicial killings and enforced disappearances but encompasses the whole
gamut of liberties protected by the Constitution. Petitioner argues that “[liberty] includes the right to
exist and the right to be free from arbitrary personal restraint or servitude and includes the right of the
citizens to be free to use his faculties in all lawful ways.” Part of the right to liberty guaranteed by the
Constitution is the right of a person to travel.

Issue: whether petitioner’s right to liberty has been violated or threatened with violation by the issuance
of the subject HDO, which would entitle him to the privilege of the writ of amparo.

Ruling:No. The Court, in Secretary of National Defense et al. v. Manalo et al., made a categorical
pronouncement that the Amparo Rule in its present form is confined to these two instances of
“extralegal killings” and “enforced disappearances,”

Here, the restriction on petitioner’s right to travel as a consequence of the pendency of the criminal
case filed against him was not unlawful. Petitioner has also failed to establish that his right to travel
was impaired in the manner and to the extent that it amounted to a serious violation of his right to
life, liberty and security, for which there exists no readily available legal recourse or remedy.

is seeking the extraordinary writ of amparo due to his apprehension that the DOJ may deny his motion
to lift the HDO. Petitioner’s apprehension is at best merely speculative. Thus, he has failed to show
any clear threat to his right to liberty actionable through a petition for a writ of amparo. The absence
of an actual controversy also renders it unnecessary for us on this occasion to pass upon the
constitutionality of DOJ Circular No. 17, Series of 1998 (Prescribing Rules and Regulations Governing
the Issuance of Hold Departure Orders); and Circular No. 18, Series of 2007 (Prescribing Rules and
Regulations Governing the Issuance and Implementation of Watchlist Orders and for Other Purposes).

Return to One’s Country

FERDINAND E. MARCOS, vs. HONORABLE RAUL MANGLAPUS, [G.R. No. 88211 September 15, 1989]

Facts: This petition for mandamus and prohibition asks the Courts to order the respondents to issue
travel documents to Mr. Marcos and the immediate members of his family and to enjoin the
implementation of the President's decision to bar their return to the Philippines.

The petitioners contend that the President is without power to impair the liberty of abode of the
Marcoses because only a court may do so "within the limits prescribed by law." Nor may the President
impair their right to travel because no law has authorized her to do so. They advance the view that
before the right to travel may be impaired by any authority or agency of the government, there must
be legislation to that effect.

The petitioners further assert that under international law, the right of Mr. Marcos and his family to
return to the Philippines is guaranteed.

Issue: whether, in the exercise of the powers granted by the Constitution, the President may prohibit
the Marcoses from returning to the Philippines.

Ruling: Yes. Exercise of Residual Powers. It must be emphasized that the individual right involved is not
the right to travel from the Philippines to other countries or within the Philippines. These are what the
right to travel would normally connote. Essentially, the right involved is the right to return to one's
country, a totally distinct right under international law, independent from although related to the
right to travel. Thus, the Universal Declaration of Humans Rights and the International Covenant on Civil
and Political Rights treat the right to freedom of movement and abode within the territory of a state,
the right to leave a country, and the right to enter one's country as separate and distinct rights.

The request or demand of the Marcoses to be allowed to return to the Philippines cannot be considered
in the light solely of the constitutional provisions guaranteeing liberty of abode and the right to travel,
subject to certain exceptions, or of case law which clearly never contemplated situations even remotely
similar to the present one. It must be treated as a matter that is appropriately addressed to those
residual unstated powers of the President which are implicit in and correlative to the paramount duty
residing in that office to safeguard and protect general welfare. In that context, such request or
demand should submit to the exercise of a broader discretion on the part of the President to
determine whether it must be granted or denied.
The return of the Marcoses to the Philippines will cause the escalation of violence against the State,
that would be the time for the President to step in and exercise the commander-in-chief powers granted
her by the Constitution to suppress or stamp out such violence. The State, acting through the
Government, is not precluded from taking pre- emptive action against threats to its existence if,
though still nascent they are perceived as apt to become serious and direct.

Liberty of Abode and Right to Travel

ZACARIAS VILLAVICENCIO, ET AL.vs. JUSTO LUKBAN, ET AL., [G.R. No. L-14639, March 25, 1919]

Facts: Justo Lukban as Manila City's Mayor together with Anton Hohmann, the city's Chief of Police,
took custody of about 170 women at the night of October 25 beyond the latters consent and
knowledge and thereafter were shipped to Davao where they were signed as laborers. Said women are
inmates of the houses of prostitution situated in Gardenia Street, in the district of Sampaloc.

Issue: whether said acts of the mayor or chief of police violate the liberty of abode of the said
prostitutes.

Ruling: Yes. Only Alien prostitutes can be expelled from the Philippine Islands in conformity with an
Act of congress. The Governor-General can order the eviction of undesirable aliens after a hearing from
the Islands. Act No. 519 of the Philippine Commission and section 733 of the Revised Ordinances of the
city of Manila provide for the conviction and punishment by a court of justice of any person who is a
common prostitute. Act No. 899 authorizes the return of any citizen of the United States, who may have
been convicted of vagrancy, to the homeland. New York and other States have statutes providing for the
commitment to the House of Refuge of women convicted of being common prostitutes. Always a law!
Even when the health authorities compel vaccination, or establish a quarantine, or place a leprous
person in the Culion leper colony, it is done pursuant to some law or order. But one can search in vain
for any law, order, or regulation, which even hints at the right of the Mayor of the city of Manila or
the chief of police of that city to force citizens of the Philippine Islands — and these women despite
their being in a sense lepers of society are nevertheless not chattels but Philippine citizens protected
by the same constitutional guaranties as are other citizens — to change their domicile from Manila to
another locality. On the contrary, Philippine penal law specifically punishes any public officer who, not
being expressly authorized by law or regulation, compels any person to change his residence. No
official, no matter how high, is above the law.

Rubi v. Provincial Board of Mindoro – Kenny

G.R. No. L-14078 March 7, 1919


RUBI, ET AL. (manguianes), plaintiffs,
vs.
THE PROVINCIAL BOARD OF MINDORO, defendant.

FACTS: This is an application for habeas corpus in favor of Rubi and other Manguianes of the Province
of Mindoro. It is alleged that the Maguianes are being illegally deprived of their liberty by the
provincial officials of that province. Rubi and his companions are said to be held on the reservation
established at Tigbao, Mindoro, against their will. It thus appears that the provincial governor of
Mindoro and the provincial board thereof directed the Manguianes in question to take up their
habitation in Tigbao, a site on the shore of Lake Naujan, selected by the provincial governor and
approved by the provincial board. The action was taken in accordance with section 2145 of the
Administrative Code of 1917, and was duly approved by the Secretary of the Interior as required by said
action. Petitioners, however, challenge the validity of this section of the Administrative Code.

Whether the constitutional right to liberty of abode of the Manguaianes are violated in as much as they
were directed to settle in a particular reservation.

NO. One cannot hold that the liberty of the citizen is unduly interfered without when the degree of
civilization of the Manguianes is considered. They are restrained for their own good and the general
good of the Philippines. This restriction was intended to promote their better education,
advancement, and protection. Nor can one say that due process of law has not been followed. To go
back to our definition of due process of law and equal protection of the law, there exists a law; the law
seems to be reasonable; it is enforced according to the regular methods of procedure prescribed; and
it applies alike to all of a class.

The public policy of the Government of the Philippine Islands is shaped with a view to benefit the
Filipino people as a whole. The Manguianes, in order to fulfill this governmental policy, must be
confined for a time, as we have said, for their own good and the good of the country.

Silverio v. CA – 195 SCRA 760 – Kenny

Facts: Petitioner was charged with violation of Section 2 (4) of the revised securities act. Respondent
filed to cancel the passport of the petitioner and to issue a hold departure order. The RTC ordered the
DFA to cancel petitioner’s passport, based on the finding that the petitioner has not been arraigned
and there was evidence to show that the accused has left the country without the knowledge and the
permission of the court.

Issue: Whether the right to travel may be impaired by order of the court.

Held: YES. Article III, Section 6 of the 1987 Constitution should be interpreted to mean that while the
liberty of travel may be impaired even without Court Order, the appropriate executive officers or
administrative authorities are not armed with arbitrary discretion to impose limitations. They can
impose limits only on the basis of "national security, public safety, or public health" and "as may be
provided by law." Holding an accused in a criminal case within the reach of the Courts by preventing
his departure from the Philippines must be considered as a valid restriction on his right to travel so
that he may be dealt with in accordance with law. The offended party in any criminal proceeding is the
People of the Philippines. It is to their best interest that criminal prosecutions should run their course
and proceed to finality without undue delay, with an accused holding himself amenable at all times to
Court Orders and processes.

Santiago v. Vasquez – 217 SCRA 633 – Kenny

Facts: An information was filed against petitioner with the Sandiganbayan for violation of the Anti
Graft and Corrupt Practices Act. The order of arrest was issued with bail for release fixed at Php.
15,000 so she filed a motion for acceptance of cash bail bond. On the same day the Sandiganbayan
issued a resolution authorizing the petitioner to post cash bond which the later filed in the amount of
Php.15, 000. Her arraignment was set, but petitioner asked for the cancellation of her bail bond and
that she be allowed provisional release on recognizance. The Sandiganbayan deferred it. The
Sandiganbayan issued a hold departure order against petitioner, by reason of the announcement she
made that she would be leaving for the U.S. to accept a fellowship a Harvard. In the instant motion she
submitted before the S.C. she argues that her right to travel is impaired.

Issue: Whether petitioner’s right to travel is impaired.

Ruling: NO. The hold – departure order is but an exercise of the respondent court’s inherent power to
preserve and maintain the effectiveness of its jurisdiction over the case and over the person of the
accused. By posting bail, the accused holds herself amenable at all times to the orders and processes of
the court, thus, she, may be legally prohibited from leaving the country during the pendency of the
case. Lastly, parties with pending cases should apply for permission to leave the country from the very
same courts which, in the first instance, are in the best position to pass upon such applications and to
impose the appropriate conditions therefor, since they are conversant with the facts of the cases and
the ramifications or implications thereof.

Marcos v. Sandiganbayan – 247 SCRA 127 – Kenny

FACTS: Former First Lady Imelda Romualdez Marcos is the defendant in several criminalcases for
violations of the Anti-Graft and Corrupt Practices Act (RA 3019), nowpending in the Sandiganbayan and
in the regular courts. In two of these cases, she was found guilty by the First Division of
theSandiganbayan and was sentenced to an indeterminate penalty of 9-12 years, with perpetual
disqualification from office. A motion for reconsideration is pending. After her conviction, she filed a
Motion for Leave to Travel Abroad to seek diagnostic tests and treatment in China because of a
seriousand life-threatening medical condition requiring facilities not available in thePhilippines. This
was denied by the Sandiganbayan.

Whether the Sandiganbayan erred in denying petitioner’s motion to travel.

RULING: NO. The person’s right to travel is subject to the usual constraints imposed by the very
necessity of safeguarding the system of justice. Whether the accused should be permitted to leave the
country for humanitarian reasons is a matter addressed to the court’s discretion.

Yap v. CA, GR 141529, June 6, 2001 – Kenny

G.R. No. 141529 June 6, 2001


FRANCISCO YAP, JR., aka EDWIN YAP, petitioner,
vs.
COURT OF APPEALS and THE PEOPLE OF THE PHILIPPINES, respondents.

FACTS: Accused-appellant Francisco Yap, Jr., a.k.a. Edwin Yap was ALLOWED TO POST BAIL in the
amount of Five Million Five Hundred Thousand (P5,500,000.00) Pesos, subject to the following
conditions, viz. :

(1) He (accused-appellant) secures a certification/guaranty from the Mayor of the


place of his residence that he is a resident of the area and that he will remain to be a
resident therein until final judgment is rendered or in case he transfers residence, it
must be with prior notice to the court;

(2) The Commission of lmmigration and Deportation (CID) is hereby directed to issue a
hold departure order against accused-appellant; and

(3) The accused-appellant shall forthwith surrender his passport to the Division Clerk of
Court for safekeeping until the court orders its return;

(4) Any violation of the aforesaid conditions shall cause the forfeiture of accused-
appellant's bail bond, the dismissal of appeal and his immediate arrest and confinement
in jail.

ISSUE: Whether the CA unduly restricted petitioner’s constitutional liberty of abode and travel in
imposing the aforementioned conditions for the grant of bail.

RULING: Under the circumstances of this case, we find that appropriate conditions have been imposed
in the bail bond to ensure against the risk of flight, particularly, the combination of the hold-departure
order and the requirement that petitioner inform the court of any change of residence and of his
whereabouts.

The right to change abode and travel within the Philippines, being invoked by petitioner, are not
absolute rights. The order of the Court of Appeals releasing petitioner on bail constitutes such lawful
order as contemplated by the Article III, Section 6. The condition imposed by the Court of Appeals is
simply consistent with the nature and function of a bail bond, which is to ensure that petitioner will
make himself available at all times whenever the Court requires his presence. Besides, a closer look at
the questioned condition will show that petitioner is not prevented from changing abode; he is merely
required to inform the court in case he does so.

Mirasol v DPWH, 490 SCRA 318 – Kenny

Facts: Petitioners sought the declaration of nullity of certain administrative issuances of the DPWH for
being inconsistent with RA 2000, entitled “Limited Access Highway Act.” Among others, is AO1 which
requires motorcycles shall have an engine displacement of at least 400cc.
Issue: Whether the prohibition on the use of motorcycles in toll ways unduly deprive petitioners of their
right to travel.

Ruling: NO. A toll way is not an ordinary road. As a facility designed to promote the fastest access to
certain destinations, its use, operation, and maintenance require close regulation. Public interest and
safety require the imposition of certain restrictions on toll ways that do not apply to ordinary roads. As
a special kind of road, it is but reasonable that not all forms of transport could use it. The right to travel
does not mean the right to choose any vehicle in traversing a toll way. The right to travel refers to the
right to move from one place to another. Petitioners can traverse the toll way any time they choose
using private or public four-wheeled vehicles. Petitioners are not denied the right to move from Point
A to Point B along the toll way. Petitioners are free to access the toll way, much as the rest of the
public can. The mode by which petitioners wish to travel pertains to the manner of using the toll way,
a subject that can be validly limited by regulation.

Petitioners themselves admit that alternative routes are available to them. Their complaint is that these
routes are not the safest and most convenient. Even if their claim is true, it hardly qualifies as an undue
curtailment of their freedom of movement and travel. The right to travel does not entitle a person to
the best form of transport or to the most convenient route to his destination. The obstructions found in
normal streets, which petitioners complain of (i.e., potholes, manholes, construction barriers, etc.), are
not suffered by them alone.

OAS v. Judge Macarine, 677 SCRA 1 - Anton


FACTS:

OCA Circular No. 49-2003 requires that all foreign travels of judges and court personnel, regardless of
the number of days, must be with prior permission from the Court. The complete requirements should
be submitted to and received by the OCA at least two weeks before the intended time of travel. No
action shall be taken on requests for travel authority with incomplete requirements.3

Judges and personnel who shall leave the country without travel authority issued by the OCA shall be
subject to disciplinary action

Respondent traveled to Hong Kong without securing the necessary travel authority. He was disciplined
as such.

ISSUE: Is the right to travel absolute?

RULING:
NO. True, the right to travel is guaranteed by the Constitution.1âwphi1 However, the exercise of such
right is not absolute. Section 6, Article III of the 1987 Constitution allows restrictions on one’s right to
travel provided that such restriction is in the interest of national security, public safety or public health
as may be provided by law. This, however, should by no means be construed as limiting the Court’s
inherent power of administrative supervision over lower courts. OCA Circular No. 49-2003 does not
restrict but merely regulates, by providing guidelines to be complied by judges and court personnel,
before they can go on leave to travel abroad. To "restrict" is to restrain or prohibit a person from doing
something; to "regulate" is to govern or direct according to rule.
Human Security Act, Section 26: In cases where evidence of guilt is not strong, and the person charged
with the crime of terrorism or conspiracy to commit terrorism is entitled to bail and is granted the same,
the court, upon application by the prosecutor, shall limit the right of travel of the accused to within the
municipality or city where he resides or where the case is pending, in the interest of national security
and public safety. Travel outside said municipality or city, without the authorization of the court, shall be
deemed a violation of the terms and conditions of his bail, which shall then be forfeited under the Rules
of Court.

Human Security Act, Section 26: In cases where evidence of guilt is not strong, and the person charged
with the crime of terrorism or conspiracy to commit terrorism is entitled to bail and is granted the
same, the court, upon application by the prosecutor, shall limit the right of travel of the accused to
within the municipality or city where he resides or where the case is pending, in the interest of
national security and public safety. Travel outside said municipality or city, without the authorization
of the court, shall be deemed a violation of the terms and conditions of his bail, which shall then be
forfeited under the Rules of Court.

Section 7. The right of the people to information on matters of public concern shall be recognized.
Access to official records, and to documents and papers pertaining to official acts, transactions, or
decisions, as well as to government research data used as basis for policy development, shall be
afforded the citizen, subject to such limitations as may be provided by law.

Right to Information

Scope of the Right

Chavez v. PEA, GR 133250, July 9, 2002- Anton


FACTS:
Petitioner as taxpayer filed the instant Petition for Mandamus with Prayer for the Issuance of a Writ
of Preliminary Injunction and Temporary Restraining Order. Petitioner contends the government stands
to lose billions of pesos in the sale by PEA of the reclaimed lands to AMARI. Petitioner prays that PEA
publicly disclose the terms of any renegotiation of the JVA, invoking Section 28, Article II, and Section
7, Article III, of the 1987 Constitution on the right of the people to information on matters of public
concern.

Due to the approval of the Amended JVA by the Office of the President, petitioner now prays that on
"constitutional and statutory grounds the renegotiated contract be declared null and void."

ISSUE: Whether the constitutional right to information includes information on on-going neogtiations
BEFORE a final agreement.

RULING:
YES. The right to information "contemplates inclusion of negotiations leading to the consummation of
the transaction." Certainly, a consummated contract is not a requirement for the exercise of the right
to information. Otherwise, the people can never exercise the right if no contract is consummated, and
if one is consummated, it may be too late for the public to expose its defects.

Requiring a consummated contract will keep the public in the dark until the contract, which may be
grossly disadvantageous to the government or even illegal, becomes a fait accompli.
However, the right to information does not compel PEA to prepare lists, abstracts, summaries and the
like relating to the renegotiation of the JVA. 34 The right only affords access to records, documents
and papers, which means the opportunity to inspect and copy them. One who exercises the right must
copy the records, documents and papers at his expense. The exercise of the right is also subject to
reasonable regulations to protect the integrity of the public records and to minimize disruption to
government operations, like rules specifying when and how to conduct the inspection and copying.

Limitation on the Right

Chavez v. PCGG, GR 130716, Dec. 9, 1988 - Anton


FACTS:
Petitioner, instituted a case against public respondent to make public any negotiations and/or
agreements pertaining to the latter's task of recovering the Marcoses' ill-gotten wealth. The
respondents argued that the action was premature since he has not shown that he had asked the
respondents to disclose the negotiations and agreements before filing the case.

ISSUE: May respondent be compelled to make public the negotiations?

RULING:
YES. The assets and properties referred to supposedly originated from the government itself. To all
intents and purposes, therefore, they belong to the people. As such, upon reconveyance they will be
returned to the public treasury, subject only to the satisfaction of positive claims of certain persons as
may be adjudged by competent courts. Another declared overriding consideration for the expeditious
recovery of ill-gotten wealth is that it may be used for national economic recovery.

The negotiations, therefore, do not fall under any of the exceptions laid down under this case, to wit:
No right to information in the following:
1. National security matters and intelligence information
2. Trade secrets and banking transactions
3. Criminal matters
4. Other confidential information which includes diplomatic correspondence, closed door Cabinet
meetings and executive sessions of either Houses of Congress, and the internal deliberations of the
Supreme Court.

In Re: Production of Court Records, 14 February 2012- Anton

In General: Access to court records, Government contract negotiations, Diplomatic negotiations, etc.

Legaspi v. CSC, 150 SCRA 530- Anton


FACTS:
The petitioner invokes his constitutional right to information on matters of public concern in a
special civil action for mandamus against the CSC pertaining to the information of civil service
eligibilities of certain persons employed as sanitarians in the Health Department of Cebu City. The
standing of the petitioner was challenged by the Solicitor General of being devoid of legal right to be
informed of the civil service eligibilities of government employees for failure of petitioner to provide
actual interest to secure the information sought.

ISSUE: Whether the petitioner may invoke his constitutional right to information in the case at bar.

RULING: YES. The Court delves into determining whether the information sought for by the petitioner
is of public interest. All appointments in the Civil Service Commission are made according to merit and
fitness while a public office is a public trust. Public employees therefore are accountable to the people
even as to their eligibilities to their positions in the government. The court also noted that the
information on the result of the CSC eligibility examination is released to the public therefore the
request of petitioner is one that is not unusual or unreasonable. The public, through any citizen, has
the right to verify the civil eligibilities of any person occupying government positions.

Bantay Republic Act v. COMELEC, GR 177271, May 4, 2007- Anton


FACTS:
Comelec en banc issued Resolution 07-0724 under date April 3, 2007 virtually declaring the nominees'
names confidential and in net effect denying petitioner Rosales' basic disclosure request. Comelec's
reason for keeping the names of the party list nominees away from the public is deducible from the
excerpts of the news report appearing in the April 13, 2007 issue of the Manila Bulletin, is that there is
nothing in R.A. 7941 that requires the Comelec to disclose the names of nominees, and that party list
elections must not be personality oriented.

ISSUE: Whether respondent Comelec, by refusing to reveal the names of the nominees of the various
party-list groups, has violated the right to information and free access to documents as guaranteed by
the Constitution; and
Whether respondent Comelec is mandated by the Constitution to disclose to the public the names of
said nominees.

YES. It has been repeatedly said in various contexts that the people have the right to elect their
representatives on the basis of an informed judgment. While the vote cast in a party-list elections is a
vote for a party, such vote, in the end, would be a vote for its nominees, who, in appropriate cases,
would eventually sit in the House of Representatives. The Court frowns upon any interpretation of the
law or rules that would hinder in any way the free and intelligent casting of the votes in an election.

YES. COMELEC has a constitutional duty to disclose and release the names of the nominees of the
party-list groups named in the herein petitions. The right to information is a public right where the real
parties in interest are the public, or the citizens to be precise, but like all constitutional guarantees,
however, the right to information and its companion right of access to official records are not
absolute. The people's right to know is limited to "matters of public concern" and is further subject to
such limitation as may be provided by law. But no national security or like concerns is involved in the
disclosure of the names of the nominees of the party-list groups in question. Doubtless, the Comelec
committed grave abuse of discretion in refusing the legitimate demands of the petitioners for a list of
the nominees of the party-list groups subject of their respective petitions. Mandamus, therefore, lies.

Valmonte v. Belmonte, Jr., 170 SCRA 256 – Czar

F: Valmonte wrote to Belmonte, requesting to be furnished the list of names of the opposition
members who were able to secure a clean loan of P2 Million each on guaranty of Mrs. Marcos and also
to be furnished with the certified true copies of the documents evidencing their loans. Due to serious
legal implications, President and General Manger Belmonte referred the letter to the Deputy General
Counsel of GSIS, Tiro. Tiro replied that it is his opinion that a confidential relationship exist between
the GSIS and all its borrowers. Thus, it is their duty to preserve this confidentiality and that it would
not be proper for the GSIS to breach this confidentiality unless so ordered by the courts. Apparently,
not having received the reply of GSIS Deputy General, Valmonte wrote another letter saying that having
failed to reply, it is their understanding that they may do whatever action necessary within the premises
to pursue our their desired objective in pursuance of public interest. Valmonte, et.al later on filed a
special civil action for mandamus with preliminary injunction invoking their right to their right to
information.

I: Whether Valmonte, et. al. are entitled as citizens and taxpayers to inquire upon GSIS records on
behest loans given by the former First Lady Imelda Marcos to Batasang Pambansa members belonging
to the UNIDO and PDP-Laban political parties.

H: The GSIS is a trustee of contributions from the government and its employees and the
administrator of various insurance programs for the benefit of the latter. Undeniably, its funds assume
a public character. More particularly, Secs. 5(b) and 46of PD 1146, as amended (the Revised
Government Service Insurance Act of 1977),provide for annual appropriations to pay the contributions,
premiums, interest and other amounts payable to GSIS by the government, as employer, as well as the
obligations which the Republic of the Philippines assumes or guarantees to pay. Considering the nature
of its funds, the GSIS is expected to manage its resources with utmost prudence and in strict
compliance with the pertinent laws or rules and regulations. Thus, one of the reasons that prompted
the revision of the old GSIS law(CA 186, as amended) was the necessity "to preserve at all times the
actuarial solvency of the funds administered by the Systems [Second Whereas Clause, PD1146.]
Consequently, as Feliciano Belmonte himself admits, the GSIS "is not supposed to grant 'clean loans.'"
It is therefore the legitimate concern of the public to ensure that these funds are managed properly
with the end in view of maximizing the benefits that accrue to the insured government employees.
Moreover, the supposed borrowers were Members of the defunct Batasang Pambansa who themselves
appropriated funds for the GSIS and were therefore expected to be the first to see to it that the GSIS
performed its tasks with the greatest degree of fidelity and that all its transactions were above board. In
sum, the public nature of the loanable funds of the GSIS and the public office held by the alleged
borrowers make the information sought clearly a matter of public interest and concern. Still, Belmonte
maintains that a confidential relationship exists between the GSIS and its borrowers. It is argued that a
policy of confidentiality restricts the indiscriminate dissemination of information. Yet, Belmonte has
failed to cite any law granting the GSIS the privilege of confidentiality as regards the documents subject
of the present petition. His position is apparently based merely on considerations of policy. The judiciary
does not settle policy issues. The Court can only declare what the law is, and not what the law should
be. Under our system of government, policy issues are within the domain of the political branches of the
government, and of the people themselves as the repository of all State power.

Aquino-Sarmiento v. Morato, 203 SCRA 515 – Czar


F: Petitioner, a member of respondent MTRCB, wrote its records officer requesting that she be
allowed to examine the boards records pertaining to the voting slips accomplished by the individual
board members after a review of the movies and television productions. The record officer informed
petitioner that she has to secure prior clearance from Morato, chairman of MTRCB to gain access to
the records sought to be examined. However, Morato denied the request on the ground that whatever
the members of the board sit in judgment partake the nature of conscience votes as such, are purely
personal. Petitioner on the other hand argues that the records she wishes to eamine are public in
character and that he (Morato) have no authority to deny any citizen seeking examination of the
board records.

I: Whether petitioner has a right to be given the information sought

H: Yes. There can be no invasion of privacy in the case at bar since what is sought to be divulged is a
product of action undertaken in the course of performing official functions. To declare otherwise
would be to clothe every public official with an impregnable mantle of protection against public
scrutiny for their official acts.

Echegaray v. Sec. of Justice, GR 132601, Oct. 12, 1988 – Czar


F: Echegaray’s counsel filed a Petition for Prohibition, Injunction and/or Temporary Restraining Order
to enjoin respondents Secretary of Justice and Director of the Bureau of Prisons from carrying out the
execution by lethal injection of petitioner under R.A. No. 8177 and its implementing rules as these are
unconstitutional and void. These are the questioned provisions:

"SEC. 19. EXECUTION PROCEDURE. - Details of the procedure prior to, during and after administering
the lethal injection shall be set forth in a manual to be prepared by the Director. The manual shall
contain details of, among others, the sequence of events before and after execution; procedures in
setting up the intravenous line; the administration of the lethal drugs; the pronouncement of death;
and the removal of the intravenous system.

Said manual shall be confidential and its distribution shall be limited to authorized prison personnel."

I: Does the 2nd paragraph of Section 19 unduly suppress the convict’s and his counsel’s constitutional
right to information and as parties in interest?

H: YES. Petition is PARTIALLY GRANTED insofar as Sections 17 and 19 of the Rules and Regulations to
Implement Republic Act No. 8177 are concerned, which are hereby declared INVALID because Section
19 unjustifiably makes the manual confidential, hence unavailable to interested parties including the
accused/convict and counsel.

As to the second paragraph of section 19, the Court finds the requirement of confidentiality of the
contents of the manual even with respect to the convict unduly suppressive. It sees no legal
impediment for the convict, should he so desire, to obtain a copy of the manual. The contents of the
manual are matters of public concern "which the public may want to know, either because these
directly affect their lives, or simply because such matters naturally arouse the interest of an ordinary
citizen."

The incorporation in the Constitution of a guarantee of access to information of public concern is


recognition of the essentiality of the free flow of ideas and information in a democracy. In the same way
that free discussion enables members of society to cope with the exigencies of their time, access to
information of general interest aids the people in democratic decision-making by giving them a better
perspective of the vital issues confronting the nation.
Gonzales v. Narvasa, GR 140835, August 14, 2000 – Czar Preparatory Commission on Constitutional
Reform
F: Petitioner Gonzales, in his capacity as a citizen and taxpayer, assails the constitutionality of the
creation of the PCCR and of the positions of presidential consultants, advisers and assistants.
Petitioner asks this Court to enoin the PCCR and the presidential consultants, advisers and assistants
from acting as such and to enjoin Executive Secretary Zamora from enforcing their advice and
recommendations. In addition petitioner seeks to enjoin COA from passing in audit expenditures for
the PCCR and the presidential consultants. Finally, Petitioner orders Executive Secretary Zamora to
answer his letter, requesting for the names of executive officials holding multiple positions in
government, copies of their appropriations, and a list of the recipients of luxury vehicles seized by the
Bureau of Customs and turned over to Malacanang.

I: Whether the public is entitled to be furnished such documents?

H: Under both the 1973 and 1987 Constitution, this is a self-executory provision which can be invoked
by any citizen before the courts, though Congress may provide for reasonable conditions upon the
access to information such as those found in RA 6713 otherwise known as “Code of Conduct and Ethical
Standards for Public Officials and Employees. This law provides that, in the performance of their duties,
all public official and employees are obliged to respond to letters sent by the public within 15 working
days from receipt thereof and to ensure the accessibility of all public documents for inspection by the
public within reasonable working hours, subject to the reasonable claims of confidentiality.
COMPLAINT was dimissed but was ordered to furnish the documents asked to furnish.

RE: Request for Radio-TV Coverage, 365 SCRA 248 – Czar Pro Hac Vice – for this event; for this turn

F: Several people, including journalist and media practitioners were killed while on their way to Shariff
Aguak in Maguindanao. The tragic incident which we came to know as Magauinadanao Massacre.
Almost a year later, various media entities, relatives, journalist and members of the academe filed a
petition before this Court praying that live television and radio coverage of the trial be allowed, and
the recording devices be permitted. Petitioners seek the lifting of the absolute ban on live television
and radio coverage of court proceedings. They urge the Court to revisit the 1991 Re: Live TV and Radio
Coverage of the Hearing of President Aquinos Libel Case and 2001 ruling in Re: Request Radio and Tv
Coverage Trial in Sandiganbayan of the Plunder Cases against the former Pres. Estrada which rulings,
they contend violate the doctrine that proposed restriction on constitutional rights are to be narrowly
construed and outright prohibition cannot stand when regulation is a viable alternative. This present
petitions asserts the exercise of freedom of press, right to information, right to a fair trial and etc.

I: WON Petitioner can be granted with Pro Hac Vice for the Radio and TV coverage of the Maguindanao
Massacre Case.

H: The Court PARTIALLY GRANTS PRO HAC VICE the request for live broadcast by television and radio of
the trial court proceedings of the Maguindanao Massacre cases, subject to the guidelines herein
outlined.

The Court cannot gloss over what advances technology has to offer in distilling the abstract discussion of
key constitutional precepts into the workable context. Technology per se has always been neutral. It is
the use and regulation thereof that need fine-tuning. Law and technology can work to the advantage
and furtherance of the various rights herein involved, within the contours of defined guidelines.

The indication of "serious risks" posed by live media coverage to the accused’s right to due process,
left unexplained and unexplored in the era obtaining in Aquino and Estrada, has left a blow to the
exercise of press freedom and the right to public information. The rationale for an outright total
prohibition was shrouded, as it is now, inside the comfortable cocoon of a feared speculation which no
scientific study in the Philippine setting confirms, and which fear, if any, may be dealt with by
safeguards and safety nets under existing rules and exacting regulations.

For a win-win situation that shall not compromise rights in the criminal administration of justice,
sacrifice press freedom and allied rights, and interfere with the integrity, dignity and solemnity of
judicial proceedings. Compliance with regulations, not curtailment of a right, provides a workable
solution to the concerns raised in these administrative matters, while, at the same time, maintaining
the same underlying principles upheld in the two previous cases.

RE: Request for Live Radio-TV Coverage, 365 SCRA 62 – Mikee


Facts:
This is a motion for reconsideration of the decision denying petitioners' request for permission to
televise and broadcast live the trial of former President Estrada before the Sandiganbayan. The
motion was filed by the Secretary of Justice, as one of the petitioners, who argues that there is really
no conflict between the right of the people to public information and the freedom of the press, on the
one hand, and, on the other, the right of the accused to a fair trial; that if there is a clash between
these rights, it must be resolved in favor of the right of the people and the press because the people,
as the repository of sovereignty, are entitled to information; and that live media coverage is a
safeguard against attempts by any party to use the courts as instruments for the pursuit of selfish
interests.

ISSUE: WON a live radio-tv coverage be allowed in the case at bar.


HELD:
Considering the significance of the trial before the Sandiganbayan of former President Estrada and the
importance of preserving the records thereof, the Court believes that there should be an audio-visual
recording of the proceedings. The recordings will not be for live or real time broadcast but for
documentary purposes. Only later will they be available for public showing, after the Sandiganbayan
shall have promulgated its decision in every case to which the recording pertains.

The trial shall be recorded in its entirety, except such portions thereof as the Sandiganbayan may
decide should not be held public pursuant to Rule 119, §21 of the Revised Rules of Criminal Procedure.
The audio-visual recordings shall be made under the supervision and control of the Sandiganbayan or
its Division as the case may be.

There are several reasons for such televised recording. First, the hearings are of historic significance.
They are an affirmation of our commitment to the rule that "the King is under no man, but he is under
God and the law." (Quod Rex non debet esse sub homine, sed sub Deo et Lege.) Second, the Estrada
cases involve matters of vital concern to our people who have a fundamental right to know how their
government is conducted. This right can be enhanced by audio visual presentation. Third, audio-visual
presentation is essential for the education and civic training of the people.
The recordings will be useful in preserving the essence of the proceedings in a way that the cold print
cannot quite do because it cannot capture the sights and sounds of events. Thus, many important
purposes for preserving the record of the trial can be served by audio-visual recordings without
impairing the right of the accused to a fair trial.

Hilado v. Reyes, 496 SCRA 282 (Access to Court Records) – Mikee


Atty. Paredes, an associate of petitioner’s counsel, was denied access to the folder-record of the case
regarding the settlement of the estate of Roberto s. Benedicto. The denial was made by Presiding
Judge on the ground that only parties or those with authority from the parties are allowed to inquire
or verify the status of the case and to go over the records. Paredes requested to be furnished certified
true copies of the order and the transcript of stenographic notes but this was denied again. Thus, she
filed a petition for mandamus to compel public respondent to allow them to access, examine and
obtain copies of any and all documents part of the records of the case; she contended that the records
of the case are public records to which the public has the right to access under Sec. 7 of Art. III.

Issue: are court records documents subject to the right to information clause?

The right to information on “matters of public concern or of public interest” is both the purpose and
the limit of the constitutional right of access to public documents.

Unlike court orders and decisions, pleadings and other documents filed by parties to a case need not
be matters of public concern or interest. Information regarding the financial standing of a person at
the time of his death and the manner by which his private estate may be settled is not matter of
general, public concern or one in which a citizen or the public has an interest by which its legal rights
or liabilities may be affected. If the information sought is not a matter of public concern or interest, the
denial of access thereto does not violate the citizen’s constitutional right to information.

Access to court records may be permitted at the discretion and subject to the supervisory and
protective powers of the court, after considering the actual use or purpose for which the request for
access is based and the obvious prejudice to any of the parties.

As long then as any party, counsel or person has a legitimate reason to have a copy of court records
and pays court fees, a court may not deny access to such records.

Sabio v. Gordon, 504 SCRA 704 – Mikee


The constitutionality of EO No.1 creating the Philippine Commission on Good Government was raised.
The assailed provision provides in Sec. 4b that “No member or staff of the Commission shall be
required to testify or produce evidence in any judicial, legislative or administrative proceeding
concerning matters within its official cognizance.
The Philippine Senate conducted an inquiry in aid of legislation regarding anomalous transactions of
PHC and PHILCOMSAT. Petitioner, Chariman of PCGG was invited as one of the resource persons, he
declined the invitation invoking the above provision

Issue: is the EO constitutional in relation to the right to information

R: No, among other reasons.. Section 4b also runs counter to Article II sec.28 and Article IIISec. 7, which
twin provisions seek to promote transparency in policy-making and in the operations of the
government, as well as provide the people sufficient information to enable them to exercise
effectively their constitutional rights. Armed with the right information, citizens can participate in
public discussions leading to the formulation of government policies and their effective implementation.
In Valmonte v Belmonte, Jr. , the court explained that an informed citizenry is essential to the existence
and proper functioning of any democracy.

A Statute may be declared unconstitutional because its purpose or effect violates the Constitution or its
basic principles.

Bantay v. COMELEC, 523 SCRA 1 - Mikee

Petitioners impugn Comelec Resolution denying their request for the release or disclosure of the
names of the nominees of the 14 accredited participating partylist groups in the May 1007 elections.
The names were necessary for the petition to cancel the accreditation of the partylist groups on the
ground that Comelec granted the accreditations without the required simultaneous determination of
the qualification of the nominees.

Issue: Did Comelec’s refusal to reveal the names of the nominees violate the right to information as
guaranteed by the Constitution?

R: yes, it does.
Assayed against the non-disclosure stance of the Comelec and the given rationale therefore is the right
to information enshrined in the self-executory Section 7, Article III of the Constitution. Complementing
and going hand in hand with the policy of full disclosure and transparency in Government. We refer to
Section 28, Article II.

By weight of jurisprudence, any citizen can challenge any attempt to obstruct the exercise of his right
to information and may seek its enforcement by mandamus. However, like all constitutional
guarantees, the right to information and its companion right of access to official records are not
absolute. The people’s right to know is limited to “matters of public concern” and is further subject to
such limitation as may be provided by law.

Berdin v. Mascarinas, 526 SCRA 592 - Mikee


The Sangguniang Bayan of Tubigon, Bohol enacted Tax Ordinance No. 88-11-36 increasing the taxes
and fees of the municipality. Its validity was questioned on the assertion that the ordinance does not
exist by virtue of respondent officials’ delay in furnishing them with a copy of the questioned
ordinance.

Issue: Was the delay in furnishing petitioners a copy of the ordinance in violation of their right to
information?

Ruling: no. The delay was due to their non-payment of the cost of reproduction.

While access to official records may not be prohibited, it certainly may be regulated. The right of the
people to information on matters of public concern is recognized under Sec. 7, Art III of the 1987
Constitution and is subject to such limitations as may be provided by law. Thus, While access to official
records may not be prohibited, it certainly may be regulated. The regulation may come either from
statutory law and from the inherent power of an officer to control his office and the records under his
custody and to exercise some discretion as to the manner in which persons desiring to inspect,
examine, or copy the record may exercise their rights. The Municipal Trreasurer in the case at bar
exercised this discretion by requiring petitioners to pay for the cost of reproduction of Tax Ordinance.
Such a requirement is reasonable under the circumstances considering that the ordinance is quite
voluminous consisting of more than a hundred pages.

Chang v. NHA, 530 SCRA 335 – Val

FACTS: The NHA prepared the feasibility studies of the Smokey Mountain low-cost housing project
which resulted in the formulation of the the Smokey Mountain Development and Reclamation Project
(SMDRP; the Project). The Project aimed to convert the Smokey Mountain dumpsite into a habitable
housing project. Once finalized, the Plan was submitted to President Aquino for her approval.
Petitioner asserts his right to information on all documents such as contracts, reports,
memoranda, and the like relative to SMDRP.
Petitioner asserts that matters relative to the SMDRP have not been disclosed to the public like
the current stage of the Project, the present financial capacity of RBI, the complete list of investors in
the asset pool, the exact amount of investments in the asset pool and other similar important
information regarding the Project.

ISSUE: whether respondents must be compelled to disclose all information related to the smokey
mountain development and reclamation project.

RULING: Yes. Sec. 28, Art. II compels the State and its agencies to fully disclose “all of its transactions
involving public interest.” Thus, the government agencies, without need of demand from anyone,
must bring into public view all the steps and negotiations leading to the consummation of the
transaction and the contents of the perfected contract. Such information must pertain to “definite
propositions of the government,” meaning official recommendations or final positions reached on the
different matters subject of negotiation. The government agency, however, need not disclose “intra-
agency or inter-agency recommendations or communications during the stage when common assertions
are still in the process of being formulated or are in the exploratory stage.” The limitation also covers
privileged communication like information on military and diplomatic secrets; information affecting
national security; information on investigations of crimes by law enforcement agencies before the
prosecution of the accused; information on foreign relations, intelligence, and other classified
information.

The other aspect of the people’s right to know apart from the duty to disclose is the duty to
allow access to information on matters of public concern under Sec. 7, Art. III of the Constitution. The
gateway to information opens to the public the following: (1) official records; (2) documents and papers
pertaining to official acts, transactions, or decisions; and (3) government research data used as a basis
for policy development.

Senate v. Ermita – GR 169777, April 20, 2006 - Val

Facts: This is a petition for certiorari and prohibition proffer that the President has abused power by
issuing E.O. 464 “Ensuring Observance of the Principles of Separation of Powers, Adherence to the Rule
on Executive Privilege and Respect for the Rights of Public Officials Appearing in Legislative Inquiries in
Aid of Legislation Under the Constitution, and for Other Purposes”. Petitioners pray for its declaration as
null and void for being unconstitutional.

The Committee of the Senate Issued invitations to various officials of the Executive
Department for them to appear as resource speakers in a public hearing on the railway project, others
on the Issue of massive election fraud in the Philippine elections, wire tapping, and the role of military
in the so-called “Gloriagate Scandal”.

Said officials were not able to attend due to lack of consent from the President as provided by
E.O. 464, Section 3 which requires all the public officials enumerated in Section 2(b) to secure the
consent of the President prior to appearing before either house of Congress.

Issue: Whether E.O. 464 violates the right of the people to information on matters of public concern.

Held: E.O 464 is concerned only with the demands of Congress for the appearance of executive
officials in the hearings conducted by it, and not with the demands of citizens for information
pursuant to their right to information on matters of public concern. Petitioners are not amiss in
claiming, however, that what is involved in the present controversy is not merely the legislative power
of inquiry, but the right of the people to information. While Congress is composed of representatives
elected by the people, it does not follow, except in a highly qualified sense, that in every exercise of
its power of inquiry, the people are exercising their right to information.

It is in the interest of the State that the channels for free political discussion be maintained to
the end that the government may perceive and be responsive to the people’s will. Yet, this open
dialogue can be effective only to the extent that the citizenry is informed and thus able to formulate its
will intelligently.

The impairment of the right of the people to information as a consequence of E.O. 464 is,
therefore, in the sense explained above, just as direct as its violation of the legislature’s power of
inquiry. Resort to any means then by which officials of the executive branch could refuse to divulge
information cannot be presumed valid. Otherwise, we shall not have merely nullified the power of our
legislature to inquire into the operations of government, but we shall have given up something of much
greater value – our right as a people to take part in government.

Suplico v. NEDA, GR 178830, July 14, 2008 - Val

FACTS: Petitioners in G.R. Nos. 178830 and 179613 pray that they be furnished certified true copies of
the “contract or agreement covering the NBN project as agreed upon with ZTE Corporation.”
Respondents also insist that there is no perfected contract in this case that would prejudice the
government or public interest. Explaining the nature of the NBN Project as an executive agreement,
respondents stress that it remained in the negotiation stage. The conditions precedent for the
agreement to become effective have not yet been complied with.
Respondents further oppose petitioners’ claim of the right to information, which they contend
is not an absolute right. They contend that the matters raised concern executive policy, a political
question which the judicial branch of government would generally hesitate to pass upon.

ISSUE: Whether petitioner can validly invoke their right to information in the case
RULING: Yes. It appears that during one of the Senate hearings on the NBN project, copies of the
supply contract were readily made available to petitioners. Evidently, the said prayer has been
complied with and is, thus, mooted.
It is simply impossible for this Court “to annul and set aside the award of the ZTE-DOTC
Broadband Deal” without any evidence to support a prior factual finding pointing to any violation of law
that could lead to such annulment order. For sure, the Supreme Court is not the proper venue for this
factual matter to be threshed out.

Neri v. Senate – GR 180643, March 25, 2008; MR Sept. 4, 2008

Facts: DOTC entered into a contract with Zhing Xing Telecommunications Equipment (ZTE) on for the
purpose of supplying equipment and services of a National Broadband Network (NBN) Project.
The Senate Committees initiated the investigation by inviting officials and persons involved in
the specified project. Secretary Romulo L. Neri was one of those invited to the hearings. On three
occasions the Petitioner was invited, he only appeared once.
Neri; appeared before the respondent committees and testified on the matters concerning the
National Broadband Project, a project awarded to a Chinese company ZTE. However when he was
probed further on PGMA’s and petitioner’s discussions relating to the NBN Project, petitioner refused
to answer, invoking executive privilege. The questions that he refused to answer were:

1. whether or not PGMA followed up the NBN Project.


2. whether or not PGMA directed him to prioritize it.
3. whether or not PGMA directed him to approve it.

The petitioner did not appear before the respondent committees upon orders of the President invoking
exec privilege. He explained that the questions asked of him are covered by exec privilege. He was cited
in contempt of respondent committees and an order for his arrest and detention until such time that he
would appear and give his testimony.

Issue: Will the claim of executive privilege in this case violates the right of the people to information
on matters of public concern)

Held: The Issue on the right to public information is subject to limitation as may be provided by
law. These are in addition to what our body of jurisprudence classifies as confidential and what our
Constitution considers as belonging to the larger concept of executive privilege. Clearly, there is a
recognized public interest in the confidentiality of certain information. We find the information subject
of this case belonging to such kind.
More than anything else, though, the right of Congress or any of its Committees to obtain
information in aid of legislation cannot be equated with the people's right to public information. The
former cannot claim that every legislative inquiry is an exercise of the people's right to information.
Be that as it may, the right to information must be balanced with and should give way, in appropriate
cases, to constitutional precepts particularly those pertaining to delicate interplay of executive-
legislative powers and privileges which is the subject of careful review by numerous decided cases.

Akbayan v. Aquino – GR 170516, July 16, 2008

Facts:
Petition for mandamus and prohibition was filed by the petitioners, as congresspersons,
citizens and taxpayers, requesting respondents to submit to them the full text of the Japan-Philippines
Economic Partnership Agreement (JPEPA).

JPEPA which has been referred to as a ‘mega treaty’ is a comprehensive plan for opening up of
markets in goods and services as well as removing barriers and restrictions on investments. It is a deal
that encompasses even our commitments to the WTO.

Petitioner emphasize that the refusal of the government to disclose the said agreement
violates their right to information on matters of public concern and of public interest.

Respondent herein invoke executive privilege. They relied on the ground that the matter
sought involves a diplomatic negotiation then in progress, thus constituting an exception to the right
to information and the policy of full disclosure of matters that are of public concern like the JPEPA.

Issue: Whether the information sought by the petitioners is of public concern and are still covered by
the doctrine of executive privilege?

Held: From the nature of the JPEPA as an international trade agreement, it is evident that the
Philippine and Japanese offers submitted during the negotiations towards its execution are matters of
public concern. This, respondents do not dispute. They only claim that diplomatic negotiations are
covered by the doctrine of executive privilege, thus constituting an exception to the right to
information and the policy of full public disclosure.

The privileged character of diplomatic negotiations has been recognized in this jurisdiction.
information on inter-government exchanges prior to the conclusion of treaties and executive
agreements may be subject to reasonable safeguards for the sake of national interest.Disclosing these
offers could impair the ability of the Philippines to deal not only with Japan but with other foreign
governments in future negotiations.

The Court holds that, in determining whether an information is covered by the right to
information, a specific “showing of need” for such information is not a relevant consideration, but only
whether the same is a matter of public concern. When, however, the government has claimed executive
privilege, and it has established that the information is indeed covered by the same, then the party
demanding it, if it is to overcome the privilege, must show that that information is vital, not simply for
the satisfaction of its curiosity, but for its ability to effectively and reasonably participate in social,
political, and economic decision-making.

Province of North Cotabato v. GRP Peace Panel, 568 SCRA 402 – Prince
GR Nos. 183591, 183752, 183893, 183951, 183962, October 14, 2008

7. MOA-AD
8. Petitioners filed for Mandamus and Prohibition with Prayer for the Issuance of Writ of
Preliminary Injunction and Temporary Restraining Order.
9. Petitioners demand disclosure and release of the complete and official copies of the MOA-AD
including its attachments, and to prohibit the signing of the MOA-AD, pending the disclosure
of the contents of the MOA-AD and the holding of a public consultation thereon.

ISSUE: Whether the subject documents may be disclosed.

HELD: YES. MUST DISCLOSE. PRESIDENTIAL ADVISOR ON PEACE GRAVELY ABUSE DISCRETION because:
NO PUBLIC CONSULTATION AND PLEBISICITE; PUBLIC CONSULTATION AND PLEBISCITE REQUIRE THE
RIGHT TO BE INFORMED; TERRITORIAL INTEGRITY AND SOVEREIGNTY IS A PUBLIC CONCERN; RIGHT
TO INFORMATION AND DUTY TO DISCLOSE ARE SELF-EXECUTORY; PROCESS OF NEGOTIATIONS OF AN
INTERNATIONAL OR EXECUTIVE AGREEMENTS ARE NOT EXEMPTED.

• Right to be informed is a self-executory constitutional right. Section 28, the duty to disclose is
self-executory as the records of the deliberations of the Constitutional Commission so disclose.
Significantly, it is absurd to say that the broader right to information on matters of public
concern is already enforceable while the correlative duty of the State to disclose its
transactions involving public interest is not enforceable until there is an enabling law.

• Prior Consultation Required: Public consultation is a species of the right to information. Clearly,
E.O. No. 3 contemplates not just the conduct of a plebiscite to effectuate "continuing"
consultations. E.O. No. 3 establishes petitioners' right to be consulted on the peace agenda, as a
corollary to the constitutional right to information and disclosure. Thus, PAPP Esperon
committed grave abuse of discretion because there was no public consultation and plebiscite.

• GRP peace panel admit that the MOA-AD is indeed of public concern. The MOA-AD subject of
the present cases is of public concern, involving as it does the sovereignty and territorial
integrity of the State, which directly affects the lives of the public at large.

• Matters of public concern covered by the right to information include steps and negotiations
leading to the consummation of the contract. The right to information "contemplates inclusion
of negotiations leading to the consummation of the transaction.” (Chavez vs. PEA).

Guingona v. COMELEC, 620 SCRA 448 – Prince


 PCOS machines, automated election.
 Alleged anomalous or irregular transactions between Comelec and Smartmatic.
 PCOS machines malfunctioned during tests.
 Petitioners pray that the SC orders the COMELEC to disclose completely the preparations of
the 2010 election.
ISSUE: Whether the COMELEC must disclose the preparations of elections.
HELD: COMELEC MUST DISCLOSE because: ELECTION AND DISBURSEMENT OF PUBLIC FUNDS ARE
TRANSCENDENTALLY IMPORTANT AND OF PUBLIC CONCERN.

9. The people’s right to information is limited to matters of public concern (Legazpi v. CSC).
Election of public officers, the citizen's right of suffrage, disbursement of public funds are
public concern and transcendental important. The requirement of personal interest is satisfied
by the mere fact that the petitioner is a citizen, and therefore, part of the general public which
possesses the right. There is no need to show any special interest in the result. Moreover, The
right to information goes hand-in-hand with the constitutional policies of full public disclosure
and honesty in the public service.

10. The duty to disclosed information of public concern is not discretionary on the agency who
holds custody of the information.

11. The postulate of public office is a public trust, institutionalized in the Constitution to protect the
people from abuse of governmental power, would certainly be mere empty words if access to
such information of public concern is denied (Valmonte v. Belmonte, Jr.).

Antolin v. Domondon, 623 SCRA 163 – Prince


G.R. No. 165036, July 5, 2010
1. Antolin failed CPA Board Exam.
2. She requested the Board of Accountancy for rechecking. She requested for the answer sheets,
questionaires, answer keys, and the grading system.
3. The BOARD released her answer sheets only pursuant to existing IRR.
4. She filed an injunction suit with the RTC for the release of all papers.

ISSUE: Whether a licensure exam sheets may be released by the concerned agency.

HELD: BOARD MUST DISCLOSE because---NATIONAL BOARD EXAMS ARE PUBLIC CONCERN; THE
COURT HAS THE POWER TO DETERMINE PUBLIC CONCERN MATTERS.

It is for the courts to determine on a case by case basis whether the matter at issue is of interest or
importance, as it relates to or affects the public. We are prepared to concede that national board
examinations such as the CPA Board Exams are matters of public concern. The public and the
examinees, in particular, would understandably be interested in the fair and competent administration
of these exams in order to ensure that only those qualified are admitted into the accounting
profession. And as with all matters pedagogical, these examinations could be not merely quantitative
means of assessment, but also means to further improve the teaching and learning of the art and
science of accounting.

Center for People v. COMELEC, 631 SCRA 41 – Prince


G.R. No. 189546, September 21, 2010
 Center for People's Empowerment in Governnance is an NGO.
 Source code is the human-readable instructions that determine the operations of machines
such as the PCOS and computers.
 Petitioner requested the COMELEC to release the copies of the source code for the Automated
Election System.
 COMELEC release all source codes except the one for the Data Capture System.
 COMELEC had not fully paid its obligation with the Software Corporation.
 COMELEC explained that the Data Capture System determines the registration of voters. If the
source codes are release, others may exploit the number of voters per candidate.

ISSUE: Whether the source code must be released.


HELD: COMELEC RELEASES SOURCE CODE because: SEC 12 OF RA 9369 EXPRESSLY PROVIDES THE
DISCLOSURE OF ALL SOURCE CODES.

The pertinent portion of Section 12 of R.A. 9369 is clear in that "once an AES technology is selected for
implementation, the Commission shall promptly make the source code of that technology available
and open to any interested political party or groups which may conduct their own review thereof." The
COMELEC has offered no reason not to comply with this requirement of the law. Indeed, its only excuse
for not disclosing the source code was that it was not yet available when CenPEG asked for it and,
subsequently, that the review had to be done, apparently for security reason, "under a controlled
environment." The elections had passed and that reason is already stale.

Francisco v. TRB, 633 SCRA 470 – Prince


G.R. No. 166910, October 19, 2010
6. Toll Regulatory Board (TRB) is a government agency empowered to grant toll operation
certificates TOC.
7. TRB issued TOC and entered into toll-operation contracts with Philippine National
Construction Corporation (PNCC) which received a legislative franchise. Toll-operation
contracts stipulated that the revenues are channeled to maintenance of the NLEX and SLEX.
8. At one point, the TRB modified the terms and conditions of the franchises. Then, petitioners
complain that toll fees are too high and requested government to release all accounts of the
operation.
9. PNCC reasoned that the there is a confidentiality clause in the contracts.

ISSUE: Whether the non-disclosure clause overcomes the duty to disclose.

HELD: NON-DISCLOSURE CLAUSE DOES NOT PROTECT THE CONTRACT because--- COURT and COA ARE
EMPOWERED TO LOOK INTO CONTRACTS; HIGHWAYS ARE PUBLIC UTILITIES; DISBURSEMENT OF
PUBLIC FUNDS AND PUBLIC UTILITY ARE PUBLIC CONCERNS.
 Sec 22, Chap 4, Subtitle B, Title 1, Book V of the Administrative Code of 1987 expressly
authorizes the COA to examine the aforementioned documents in connection with the fixing of
rates of every nature including the fixing of toll fees. Any public utility unreasonably denying
COA access to the documents, unnecessarily obstructs the examination and audit and may be
adjudged liable "of concealing any material information concerning its financial status, shall be
subject to the penalties provided by law."
 The alleged confidentiality clause in the CITRA STOA does not necessarily translate into a
violation of Article III, Section 7. The Court can take judicial notice that most commercial
contracts, including finance-related project agreements carry the standard confidentiality clause
to protect proprietary data and/or intellectual property rights.
 The toll-operation contracts' likewise stipulate disclosure of information to any persons
authorized by law and to judicial and quasi-judicial bodies.

Initiatives v. PSALM, 682 SCRA 602 – Prince


L-192088, October, 9 2012
 Petitioner is an NGO.
 POWER SECTOR ASSETS AND LIABILITIES MANAGEMENT CORPORATION (PSALM) is created
under the EPIRA law with purpose of privatizing and liquidating Napocor's assets within 25
years.
 Korean Water Resources Corporation won the bid.
 Petitioners claim that there was no prior consultation since PSALM withheld vital information.
 After request, PSALM did not furnish information about the corporate bidders.

ISSUE: Whether PSALM must disclose information about the bidders and the bidding process.

HELD: DISCLOSE ALL INFORMATION ABOUT THE BID.

6. EPIRA was enacted to provide for "an orderly and transparent privatization" of NPC’s assets
and liabilities. It mandated that "all assets of NPC shall be sold in an open and transparent
manner through public bidding."
7. Constitution seeks to promote transparency in policy-making and in the operations of the
government, as well as provide the people sufficient information to exercise effectively other
constitutional rights.
8. Armed with the right information, citizens can participate in public discussions leading to the
formulation of government policies and their effective implementation. An informed citizenry is
essential to the existence and proper functioning of any democracy.
9. Chavez v. Public Estates Authority :The constitutional right to information includes official
information on on-going negotiations before a final contract. The information, however, must
constitute definite propositions by the government and should not cover recognized exceptions
like privileged information, military and diplomatic secrets and similar matters affecting national
security and public order. In addition, Congress has prescribed other limitations on the right to
information in several legislations.
10. DUTY TO ALLOW ACCESS TO INFORMATION DISTINGUISH FROM DUTY TO DISCLOSE: The duty to
disclose covers only transactions involving public interest, while the duty to allow access has a
broader scope of information which embraces not only transactions involving public interest,
but any matter contained in official communications and public documents of the government
agency.
11. DUTY TO ALLOW ACCESS TO INFORMATION: Such relief must be granted to the party
requesting access to official records, documents and papers relating to official acts,
transactions, and decisions that are relevant to a government contract.

Belgica v. Executive Secretary, GR 208566, November 19, 2013 - Rhino

FACTS: Aside from seeking the Court to declare the Pork Barrel System unconstitutional petitioners
equally pray that the Executive Secretary and/or the DBM be ordered to release to the CoA and to the
public: (a) "the complete schedule/list of legislators who have availed of their PDAF and Various
Infrastructures including Local Projects (VILP) from the years 2003 to 2013, specifying the use of the
funds, the project or activity and the recipient entities or individuals, and all pertinent data thereto"
and (b) "the use of the Executive‘s lump-sum, discretionary funds, including the proceeds from the
Malampaya Funds and remittances from the PAGCOR from 2003 to 2013, specifying the project or
activity and the recipient entities or individuals, and all pertinent data thereto." Petitioners‘prayer is
grounded on Section 28, Article II and Section 7, Article III of the 1987 Constitution.

ISSUE: Whether the right to information should be granted.


RULING: NO! Case law instructs that the proper remedy to invoke the right to information is to file a
petition for mandamus. In the case of Valmonte v. Belmonte Jr.: it has been clarified that the right to
information does not include the right to compel the preparation of "lists, abstracts, summaries and
the like." It is essential that the "applicant has a well -defined, clear and certain legal right to the thing
demanded and that it is the imperative duty of defendant to perform the act required." Hence,
without the foregoing substantiations, the Court cannot grant a particular request for information. The
request of the petitioners fails to meet this standard, there being no duty on the part of respondent to
prepare the list requested. While petitioners pray that said information be equally released to the CoA,
it must be pointed out that the CoA has not been impleaded as a party to these cases nor has it filed any
petition before the Court to be allowed access to or to compel the release of any official document
relevant to the conduct of its audit investigations.

Section 8. The right of the people, including those employed in the public and private sectors, to form
unions, associations, or societies for purposes not contrary to law shall not be abridged.

Scope

Volkschel Labor Union v. Bureau of Labor Relations, 137 SCRA 42 G.R. No. L-45824 June 19, 1985-
Rhino

FACTS: Petitioner is affiliated with the respondent Associated Labor Union for Metal Workers. Both
unions, using the name Volkschel Labor Union Associated Labor Union for Metal Workers, jointly
entered into a collective bargaining agreement with respondent companies. Later, a majority of
petitioner's members decided to disaffiliate from respondent federation in order to operate on its own
as an independent labor group. Confronted with the predicament of whether or not to continue
deducting from employees' wages and remitting union dues to respondent, respondent companies
sought the legal opinion of the respondent Bureau as regards the controversy between the two unions.
The bureau ruled that petitioner's members should continue paying their dues to respondent in the
concept of agency fees.

ISSUE #1: Whether petitioner union's disaffiliation from respondent federation is valid.

RULING: YES! A local union, being a separate and voluntary association, is free to serve the interest
of all its members including the freedom to disaffiliate when circumstances warrant. This right is
consistent with the Constitutional guarantee of freedom of association. A disaffiliation does not
disturb the enforceability and administration of a collective agreement; it does not occasion a change of
administrators of the contract nor even an amendment of the provisions thereof.

ISSUE #2: Whether respondent companies have the right to effect union dues collections despite
revocation by the employees of the check-off authorization.

RULING: NO! Without said affiliation, the employer has no link to the mother union. The obligation of
an employee to pay union dues is coterminous with his affiliation or membership. "The employees'
check-off authorization, even if declared irrevocable, is good only as long as they remain members of
the union concerned." A contract between an employer and the parent organization as bargaining
agent for the employees is terminated by the disaffiliation of the local of which the employees are
members.
Right to Association

Occena v. COMELEC, 127 SCRA 404 – Rhino

FACTS: This petition for prohibition seeks the declaration as unconstitutional of Sections 4 and 22 of
Batas Pambansa Blg. 222, otherwise known as the Barangay Election Act of 1982, insofar as it prohibits
any candidate in the Barangay election "from representing or allowing himself to be represented as a
candidate of any political party ... or prohibits a political party, political group, political committee ...
from intervening in the nomination of a candidate in the barangay election or in the filing of his
certificate of candidacy, or giving aid or support directly or indirectly, material or otherwise, favorable
to or against his campaign for election." The petitioner contends that the ban on the intervention of
political parties in the election of barangay officials is violative of the constitutional guarantee of the
right to form associations and societies for purposes not contrary to law.

ISSUE: Whether the provision in question is unconstitutional.

RULING: NO! The right to form associations or societies for purposes not contrary to law is neither
absolute nor illimitable; it is always subject to the pervasive and dominant police power of the state
and may constitutionally be regulated or curtailed to serve appropriate and important public interests.
Political parties may freely be formed although there is a restriction on their activities, i.e., their
intervention in the election of barangay officials is proscribed. But the ban is narrow, not total. It
operates only on concerted or group action of political parties. Members of political and kindred
organizations, acting individually, may intervene in the barangay election. The barangay is the basic unit
not only of our social structure but also of our political structure. As much as possible, we believe that it
would be a more prudent policy to insulate the barangays from the influence of partisan politics. The
primary purpose of the prohibition then is also to avert the clear and present danger of another
substantive evil, the denial of the equal protection of the laws. The candidates must depend on their
individual merits and not on the support of political parties or organizations.

UPCSU v. Laguesma – 286 SCRA 15 - Rhino

FACTS: Petitioner is a union of supervisory employees. It appears that the union filed a petition for
certification election on behalf of the route managers at Pepsi-Cola Products Philippines, Inc.
However, its petition was denied by the med-arbiter and, on appeal, by the Secretary of Labor and
Employment, on the ground that the route managers are managerial employees and, therefore,
ineligible for union membership under the first sentence of Art. 245 of the Labor Code, which provides:

Ineligibility of managerial employees to join any labor organization; right of supervisory


employees. — Managerial employees are not eligible to join, assist or form any labor organization.

Supervisory employees shall not be eligible for membership in a labor organization of the
rank-and-file employees but may join, assist or form separate labor organizations of their own.

Petitioner brought this suit challenging the validity of the order dated, as reiterated in the order,
of the Secretary of Labor and Employment. Its petition was dismissed by the Third Division. But
petitioner filed a motion for reconsideration, pressing for resolution its contention that the first
sentence of Art. 245 of the Labor Code, so far as it declares managerial employees to be ineligible to
form, assist or join unions, contravenes Art. III, §8 of the Constitution which provides:

The right of the people, including those employed in the public and private sectors, to form
unions, associations, or societies for purposes not contrary to law shall not be abridged.

For this reason, the petition was referred to the Court en banc.

ISSUE: Whether Art. 245, insofar as it prohibits managerial employees from forming, joining or assisting
labor unions, violates Art. III, Section 8 of the Constitution.

RULING: NO! The said ban under Article 245 is valid because the right guaranteed in Art. III, Sect. 8 is
subject to the condition that its exercise should be for purposes “not contrary to law.” In the case of
Art. 245, there is a rational basis for prohibiting managerial employees from forming or joining labor
organizations. The rationale thus: “because if these managerial employees would belong to or be
affiliated with a Union, the latter might not be assured of their loyalty to the Union in view of evident
conflict of interest. The Union can also become company-dominated with the presence of managerial
employees in the Union membership. (N.B.: Should be 288 SCRA 15.)

Bel-Air Village Association v. Dionisio, 174 SCRA 589 - Rhino

FACTS: Plaintiff filed a complaint against the defendant in the municipal court of Makati, Rizal, for the
collection of the amount of P 2,100 plus penalty of 12% per annum and P 751.30 as attorney's fees
and expenses of litigation. The sum of P 2,100 represents the association dues assessed on the lot
owned by the defendant as member of the plaintiff association. There is no dispute that Transfer
Certificate of Title No. 81136 covering the subject parcel of land issued in the name of the defendant
contains an annotation to the effect that the lot owner becomes an automatic member of the
respondent Bel-Air Association and must abide by such rules and regulations laid down by the
Association in the interest of the sanitation, security and the general welfare of the community. It is
likewise not disputed that the provision on automatic membership was expressly annotated on the
defendant's Transfer Certificate of Title and on the title of his predecessor-in-interest. The municipal
court ruled in plaintiff’s.

ISSUE: Whether such annotation collides with the constitutional guarantee of freedom of association is
not tenable.

RULING: NO! The transaction between the defendants and the original seller (defendant's immediate
predecessor) of the land covered by TCT No. 81136 is a sale and the conditions have been validly
imposed by the said vendor/the same not being contrary to law, morals and good customs and public
policy. The fact that it has been approved by the Land Registration Commission did not make it a
governmental act subject to the constitutional restriction against infringement of the right of
association. The constitutional proscription that no person can be compelled to be a member of an
association against his will applies only to government acts and not to private transactions like the
one in question.
The defendant cannot legally maintain that he is compelled to be a member of the association against
his will because the limitation is imposed upon his ownership of property. If he does not desire to
comply with the annotation or lien in question he can at any time exercise his inviolable freedom of
disposing of the property and free himself from the burden of becoming a member of the plaintiff
association. After all, it is not imposed upon him personally but upon his ownership of the property. The
limitation and restriction is a limitation that follows the land whoever is its owner. It does not inhere in
the person of the defendant.

Padcom Condominium Association v. Ortigas Center Association, Inc, 382 SCRA 222 - Rhino

FACTS: Petitioner Padcom Condominium Corporation (hereafter PADCOM) owns and manages the
Padilla Office Condominium Building (PADCOM Building) located in Ortigas Center which was acquired
from Tierra Development Corporation (TDC) in 1975. TDC’s predecessor was the Ortigas & Company,
Limited Partnership under a Deed of Sale which the terms and conditions that the transferee and its
successor-in-interest must become members of an association for realty owners and long-term lessees
in the area later known as the Ortigas Center. In 1982, respondent Ortigas Center Association, Inc.
(hereafter the Association) was organized to advance the interests and promote the general welfare of
the real estate owners and long-term lessees of lots in the Ortigas Center. It sought the collection of
membership dues in the amount of two thousand seven hundred twenty-four pesos and forty
centavos (P2,724.40) per month from PADCOM. The corporate books showed that PADCOM owed the
Association P639,961.47, representing membership dues, interests and penalty charges from April
1983 to June 1993. Respondent then sued for money claims. PADCOM now contends that it could not
be compelled to become a member without violating its right to freedom of association. And since it
was not a member of the Association, it was not liable for membership dues, interests and penalties.

ISSUE: Whether there is a violation of the right to join association.

RULING: NO! PADCOM was never forced to join the association. It could have avoided such
membership by not buying the land from TDC. Nobody forced it to buy the land when it bought the
building with the annotation of the condition or lien on the Certificate of Title thereof and accepted the
Deed. PADCOM voluntarily agreed to be bound by and respect the condition, and thus to join the
Association.
The stipulation of automatic membership was also annotated at the back of Transfer Certificate of Title
No. 457308 issued to TDC and to the TCT of petitioner as any lien annotated on previous certificates of
title should be incorporated in or carried over to the new transfer certificates of title.

Government Employees (Right to Strike)


Tupas v. NHC, 173 SCRA 33 - Bryan

Facts:
Respondent National Housing Corporation (hereinafter referred to as NHC) is a corporation
organized in 1959 in accordance with Executive Order No. 399, otherwise known as the Uniform Charter
of Government Corporations, dated January 1, 1951. Petitioner Trade Unions of the Philippines and
Allied Services (TUPAS, for brevity) is a legitimate labor organization with a chapter in NHC.

TUPAS filed a petition for the conduct of a certification election with Regional Office No. IV of
the Department of Labor in order to determine the exclusive bargaining representative of the workers
in NHC. It was claimed that its members comprised the majority of the employees of the corporation.
The petition was dismissed by med-arbiter Eusebio M. Jimenez in an order, holding that NHC "being a
government-owned and/or controlled corporation its employees/workers are prohibited to form, join
or assist any labor organization for purposes of collective bargaining.

From this order of dismissal, TUPAS appealed to the Bureau of Labor Relations where, acting
thereon in BLR, Director Carmelo C. Noriel reversed the order of dismissal and ordered the holding of a
certification election. This order was, however, set aside by Officer-in-Charge Virgilio S.J. Sy in his
resolution upon a motion for reconsideration of respondent NHC.

In the instant petition for certiorari, TUPAS seeks the reversal of the said resolution and prays
that a certification election be held among the rank and file employees of NHC.

Issue:
Whether government employees have the right to form unions.

Ruling:
The workers or employees of NHC undoubtedly have the right to form unions or employees'
organizations. The right to unionize or to form organizations is now explicitly recognized and granted to
employees in both the governmental and the private sectors. The Bill of Rights Section 8 provides that
"the right of the people, including those employed in the public and private sectors, to form unions,
associations or societies for purposes not contrary to law shall not be abridged."

SSS Employees v. CA, 175 SCRA 686 - Bryan

Facts:
The petitioners went on strike after the SSS failed to act upon the union’s demands concerning
the implementation of their CBA. SSS filed before the court action for damages with prayer for writ of
preliminary injunction against petitioners for staging an illegal strike. The court Issued a temporary
restraining order pending the resolution of the application for preliminary injunction while petitioners
filed a motion to dismiss alleging the court’s lack of jurisdiction over the subject matter. Petitioners
contend that the court made reversible error in taking cognizance on the subject matter since the
jurisdiction lies on the DOLE or the National Labor Relations Commission as the case involves a labor
dispute. The SSS contends on one hand that the petitioners are covered by the Civil Service laws, rules
and regulation thus have no right to strike. They are not covered by the NLRC or DOLE therefore the
court may enjoin the petitioners from striking.

Issue: Whether or not government employees have the right to strike.

Ruling:
The Constitutional provisions enshrined on Human Rights and Social Justice provides guarantee
among workers with the right to organize and conduct peaceful concerted activities such as strikes.
On one hand, Section 14 of E.O No. 180 provides that “the Civil Service law and rules governing
concerted activities and strikes in the government service shall be observed, subject to any legislation
that may be enacted by Congress” referring to Memorandum Circular No. 6, s. 1987 of the Civil Service
Commission which states that “prior to the enactment by Congress of applicable laws concerning strike
by government employees enjoins under pain of administrative sanctions, all government officers and
employees from staging strikes, demonstrations, mass leaves, walk-outs and other forms of mass action
which will result in temporary stoppage or disruption of public service.” Therefore in the absence of any
legislation allowing govt. employees to strike they are prohibited from doing so.

In Sec. 1 of E.O. No. 180 the employees in the civil service are denominated as
“government employees” and that the SSS is one such government-controlled corporation with an
original charter, having been created under R.A. No. 1161, its employees are part of the civil service and
are covered by the Civil Service Commission’s memorandum prohibiting strikes. But the current ban on
them against strikes is statutory and may be lifted by statute.

MPSTA v. Secretary of Education, GR 95445, August 6, 1991 - Bryan

Facts: September 17, 1990 fell on a Monday, which was also a regular school day. There is no
question that the some 800 teachers who joined the mass action did not conduct their classes on that
day; instead, as alleged in the petition in G.R. No. 95590, they converged at the LiwasangBonifacio in
the morning whence they proceeded to the National Office of the Department of Education, Culture
and Sport (DECS) for a whole-day assembly.

Issue: Do public teachers have the right to strike?

Ruling : No,the Manila public school teachers association has no right to strike during class hours.
The dissenting opinions, however, would anchor their defense of the public school teachers
on their right to petition the government for redress of grievances.

Jacinto v. CA, GR 124540, November 4, 1997 - Bryan

Facts:
Petitioners are public school teachers from various schools in Metropolitan Manila. Between
the period September 17 to 21, 1990, they incurred unauthorized absences in connection with the
mass actions then staged; and on September 17, 1990, DECS Secretary Isidro Cariño immediately
issued a return-to-work order. They were administratively charged with gross misconduct; gross
neglect of duty, etc. for joining unauthorized mass actions; ignoring report-to-work directives;
unjustified abandonment of teaching posts; non-observance of Civil Service law, rules and regulations;
non-compliance with reasonable office rules and regulations; and incurring unauthorized absences
without leave, etc.
Issue: Were the public school teachers penalized for the exercise of their right to assemble peacefully
and to petition the government for redress of grievances?

Ruling: Improper Exercise of the Right to Peaceful Assembly and to Petition for a Redress of Grievances.
There is no question as to the petitioners’ rights to peaceful assembly to petition the government for a
redress of grievances and, for that matter, to organize or form associations for purposes not contrary to
law, as well as to engage in peaceful concerted activities. Although the Constitution vests in them the
right to organize, to assemble peaceably and to petition the government for a redress of grievances,
there is no like express provision granting them the right to strike. Rather, the constitutional grant of
the right to strike is restrained by the proviso that its exercise shall be done in accordance with law.

It cannot be denied that the mass action or assembly staged by the petitioners resulted in the non-
holding of classes in several public schools during the corresponding period.

GSIS v. Kapisanan, GR 170132 - Bryan

Facts:
A four-day October 2004 concerted demonstration, rallies and en masse walkout waged/held
in front of the GSIS main office in Roxas Boulevard, Pasay City, started it all. Forming a huge part of the
October 4 to October 7, 2004 mass action participants were GSIS personnel, among them members of
the herein respondent Kapisanan Ng Mga Manggagawa sa GSIS ("KMG" or the "Union"), a public sector
union of GSIS rank-and-file employees. Contingents from other government agencies joined causes
with the GSIS group. The mass action's target appeared to have been herein petitioner Garcia and his
management style. While the Mayor of Pasay City allegedly issued a rally permit, the absence of the
participating GSIS employees was not covered by a prior approved leave.

Issue: Whether the employees of the GSIS have a right to strike.

Ruling:
NO. It may be, as the appellate court urged¸ that the freedom of expression and assembly and
the right to petition the government for a redress of grievances stand on a level higher than economic
and other liberties. Any suggestion, however, about these rights as including the right on the part of
government personnel to strike ought to be, as it has been, trashed. We have made this abundantly
clear in our past determinations. For instance, in Alliance of Government Workers v. Minister of Labor
and Employment, a case decided under the aegis of the 1973 Constitution, an en banc Court declared
that it would be unfair to allow employees of government corporations to resort to concerted activity
with the ever present threat of a strike to wring benefits from Government. Then came the 1987
Constitution expressly guaranteeing, for the first time, the right of government personnel to self-
organization to complement the provision according workers the right to engage in "peaceful concerted
activities, including the right to strike in accordance with law."

We hold, citing MPSTA v. Laguio, Jr., that employees in the public service may not engage in
strikes or in concerted and unauthorized stoppage of work; that the right of government employees to
organize is limited to the formation of unions or associations, without including the right to strike.
Membership in the Philippine Bar

In Re: Edillon, 84 SCRA 554 - 11 - Bryan


A.M. No. 1928 August 3, 1978

Facts:

The respondent Marcial A. Edillon is a duly licensed practicing attorney in the Philippines. The
IBP Board of Governors recommended to the Court the removal of the name of the respondent from
its Roll of Attorneys for “stubborn refusal to pay his membership dues” to the IBP since the latter’s
constitution notwithstanding due notice.

Edilion contends that the provision providing for the IBP dues constitute an invasion of his
constitutional rights in the sense that he is being compelled, as a pre-condition to maintaining his
status as a lawyer in good standing, to be a member of the IBP and to pay the corresponding dues, and
that as a consequence of this compelled financial support of the said organization to which he is
admittedly personally antagonistic, he is being deprived of the rights to liberty and property
guaranteed to him by the Constitution. Hence, the respondent concludes, the above provisions of the
Court Rule and of the IBP By-Laws are void and of no legal force and effect.

Issue: WON to compel a lawyer to be a member of the Integrated Bar is not violative of his
constitutional freedom to associate. 6

Ruling:

Bar integration does not compel the lawyer to associate with anyone. He is free to attend or not
attend the meetings of his Integrated Bar Chapter or vote or refuse to vote in its elections as he
chooses. The only compulsion to which he is subjected is the payment of annual dues. The Supreme
Court, in order to further the State’s legitimate interest in elevating the quality of professional legal
services, may require that the cost of improving the profession in this fashion be shared by the subjects
and beneficiaries of the regulatory program — the lawyers.

Such compulsion is justified as an exercise of the police power of the State. Why? The right to practise
law before the courts of this country should be and is a matter subject to regulation and inquiry. And, if
the power to impose the fee as a regulatory measure is recognize, then a penalty designed to enforce its
payment, which penalty may be avoided altogether by payment, is not void as unreasonable or
arbitrary.

Section 9. Private Property shall not be taken for public use without just compensation.
Expropriation in General

Heirs of Alberto Suguitan v. City of Mandaluyong, March 14, 2000 – Kenny


[G.R. No. 135087. March 14, 2000]

HEIRS OF ALBERTO SUGUITAN, petitioner, vs. CITY OF MANDALUYONG, respondent.

Sangguniang Panlungsod of Mandaluyong City issued Resolution No. 396, S-1994[3] authorizing then
Mayor Benjamin S. Abalos to institute expropriation proceedings over the property of Alberto Suguitan.
Petitioner assert that the city of Mandaluyong may only exercise its delegated power of eminent domain
by means of an ordinance as required by section 19 of Republic Act (RA) No. 7160,[9] and not by means
of a mere resolution. Petitioners refute respondent's contention that only a resolution is necessary upon
the initiation of expropriation proceedings and that an ordinance is required only in order to
appropriate the funds for the payment of just compensation.

Whether a resolution is sufficient to authorize the City Mayor to expropriate the said property.

NO. The courts have the obligation to determine whether the following requisites have been complied
with by the local government unit concerned:

1. An ordinance is enacted by the local legislative council authorizing the local chief
executive, in behalf of the local government unit, to exercise the power of eminent
domain or pursue expropriation proceedings over a particular private property.cal

2. The power of eminent domain is exercised for public use, purpose or welfare, or for
the benefit of the poor and the landless.

3. There is payment of just compensation, as required under Section 9, Article III of the
Constitution, and other pertinent laws.

4. A valid and definite offer has been previously made to the owner of the property
sought to be expropriated, but said offer was not accepted.

In the present case, the City of Mandaluyong seeks to exercise the power of eminent domain over
petitioners' property by means of a resolution, in contravention of the first requisite. The law in this case
is clear and free from ambiguity. Section 19 of the Code requires an ordinance, not a resolution, for the
exercise of the power of eminent domain.

NHA v. Heirs of Isidro Guivelondo, GR 15441, June 19, 2003 – Kenny

[G.R. No. 154411. June 19, 2003]


NATIONAL HOUSING AUTHORITY, petitioner, vs. HEIRS OF ISIDRO GUIVELONDO, COURT OF APPEALS,
HON. ISAIAS DICDICAN, Presiding Judge, Regional Trial Court, Branch 11, Cebu City, and PASCUAL Y.
ABORDO, Sheriff, Regional Trial Court, Branch 11, Cebu City, respondents.

Petitioner seeks the dismissal or discontinuance of the expropriation proceedings. More specifically,
petitioner alleges that the intended public use was rendered nugatory by the unreasonable just
compensation fixed by the court, which is beyond the means of the intended beneficiaries of the
socialized housing project.

Whether the expropriation proceedings may be discontinued.

NO. Housing is a basic human need. Shortage in housing is a matter of state concern since it directly and
significantly affects public health, safety, the environment and in sum, the general welfare. The public
purpose of the socialized housing project is not in any way diminished by the amount of just
compensation that the court has fixed. The need to provide decent housing to the urban poor dwellers
in the locality was not lost by the mere fact that the land cost more than petitioner had expected. It is
worthy to note that petitioner pursued its petition for certiorari with the Court of Appeals assailing the
amount of just compensation and its petition for review with this Court which eloquently indicates that
there still exists a public use for the housing project. It was only after its appeal and petitions for review
were dismissed that petitioner made a complete turn-around and decided it did not want the property
anymore.
Respondent landowners had already been prejudiced by the expropriation case. Petitioner cannot
be permitted to institute condemnation proceedings against respondents only to abandon it later when
it finds the amount of just compensation unacceptable.

Mactan v. Lozada, 613 SCRA 618 (Reversion) – Kenny

G.R. No. 176625

MACTAN-CEBU INTERNATIONAL AIRPORT AUTHORITY and AIR TRANSPORTATION OFFICE, Petitioners,


vs.
BERNARDO L. LOZADA, SR., and the HEIRS OF ROSARIO MERCADO, namely, VICENTE LOZADA, MARIO
M. LOZADA, MARCIA L. GODINEZ, VIRGINIA L. FLORES, BERNARDO LOZADA, JR., DOLORES GACASAN,
SOCORRO CAFARO and ROSARIO LOZADA, represented by MARCIA LOZADA GODINEZ, Respondents.

Subject of this case is Lot No. 88-SWO-25042 (Lot No. 88), with an area of 1,017 square meters, more or
less, located in Lahug, Cebu City which was subjected to expropriation proceedings, initiated by the
Republic of the Philippines (Republic), represented by the then Civil Aeronautics Administration (CAA),
for the expansion and improvement of the Lahug Airport. During the pendency of the expropriation
proceedings, respondent Bernardo L. Lozada, Sr. acquired Lot No. 88 from Deiparine. Consequently,
Transfer Certificate of Title (TCT) No. 9045 was issued in Lozada’s name. Eventually, the trial court
rendered judgment in favor of the Republic and ordered the latter to pay Lozada the fair market value of
Lot No. 88, adjudged at P3.00 per square meter. The projected improvement and expansion plan of the
old Lahug Airport, however, was not pursued. Lozada, with the other landowners requested CAA to be
allowed to repurchase the lots, as per previous agreement. The CAA replied that there might still be a
need for the Lahug Airport to be used as an emergency DC-3 airport. It reiterated, however, the
assurance that "should this Office dispose and resell the properties which may be found to be no longer
necessary as an airport, then the policy of this Office is to give priority to the former owners subject to
the approval of the President." After some time, petitioners initiated a complaint for the recovery of
possession and reconveyance of ownership of Lot No. 88.

Whether petitioner may recover the said lot.

Indeed, it is apparent that the acquisition by the Republic of the expropriated lots was subject to the
condition that the Lahug Airport would continue its operation. The condition not having materialized
because the airport had been abandoned, the former owner should then be allowed to reacquire the
expropriated property. In light of these premises, we now expressly hold that the taking of private
property, consequent to the Government’s exercise of its power of eminent domain, is always subject to
the condition that the property be devoted to the specific public purpose for which it was taken.
Corollarily, if this particular purpose or intent is not initiated or not at all pursued, and is peremptorily
abandoned, then the former owners, if they so desire, may seek the reversion of the property, subject
to the return of the amount of just compensation received. In such a case, the exercise of the power of
eminent domain has become improper for lack of the required factual justification

Vda De Ouna v. Republic, 642 SCRA 384 (Reversion) - Kenny


Soon after the MCIAA jettisoned the Lahug Airport expansion project, informal settlers entered and
occupied Lot No. 763-A which, before its expropriation, belonged to the Ouanos. The Ouanos then
formally asked to be allowed to exercise their right to repurchase the aforementioned lot, but the
MCIAA ignored the demand. On August 18, 1997, the Ouanos instituted a complaint before the Cebu
City RTC against the Republic and the MCIAA for reconveyance, docketed as Civil Case No. CEB-20743.

Answering, the Republic and MCIAA averred that the Ouanos no longer have enforceable rights
whatsoever over the condemned Lot No. 763-A, the decision in Civil Case No. R-1881 not having found
any reversionary condition.

WHETHER ABANDONMENT OF THE PUBLIC USE FOR WHICH THE SUBJECT PROPERTIES WERE
EXPROPRIATED ENTITLES PETITIONERS TO REACQUIRE THEM.

YES. A condemnor should commit to use the property pursuant to the purpose stated in the petition for
expropriation, failing which it should file another petition for the new purpose. If not, then it behooves
the condemnor to return the said property to its private owner, if the latter so desires. The government
cannot plausibly keep the property it expropriated in any manner it pleases and, in the process,
dishonor the judgment of expropriation. The notion, therefore, that the government, via expropriation
proceedings, acquires unrestricted ownership over or a fee simple title to the covered land, is no longer
tenable. The taking of a private land in expropriation proceedings is always conditioned on its continued
devotion to its public purpose. As a necessary corollary, once the purpose is terminated or peremptorily
abandoned, then the former owner, if he so desires, may seek its reversion, subject of course to the
return, at the very least, of the just compensation received. Given the foregoing disquisitions, equity and
justice demand the reconveyance by MCIAA of the litigated lands in question to the petitioners.

Power to Undertake Expropriation Case

Iron and Steel Authority v. CA, 249 SCRA 538 – Kenny

G.R. No. 102976 October 25, 1995

IRON AND STEEL AUTHORITY, petitioner,


vs.
THE COURT OF APPEALS and MARIA CRISTINA FERTILIZER CORPORATION, respondents.

FACTS: Petitioner Iron and Steel Authority ("ISA") was created by Presidential Decree (P.D.) No. 272,
generally, to develop and promote the iron and steel industry in the Philippines. It has a term of five (5)
years counting from 9 August 1973. 1 When ISA's original term expired on 10 October 1978, its term was
extended for another ten (10) years by Executive Order No. 555 dated 31 August 1979. Petitioner ISA
initiated expropriation proceedings in respect of occupancy rights of private respondent MCFC relating
to the subject public land as well as the plant itself and related facilities and to cede the same to the
NSC. The case proceeded to trial. While the trial was ongoing, however, the statutory existence of
petitioner ISA expired on 11 August 1988. MCFC then filed a motion to dismiss, contending that no valid
judgment could be rendered against ISA which had ceased to be a juridical person. Petitioner ISA filed its
opposition to this motion.

Whether the Republic of the Philippines is entitled to be substituted for ISA in view of the expiration of
ISA's term.

We consider that the ISA is properly regarded as an agent or delegate of the Republic of the Philippines.
The Republic itself is a body corporate and juridical person vested with the full panoply of powers and
attributes which are compendiously described as "legal personality." The Republic of the Philippines is
entitled to be substituted in the expropriation proceedings as party-plaintiff in lieu of ISA, the statutory
term of ISA having expired. Put a little differently, the expiration of ISA's statutory term did not by itself
require or justify the dismissal of the eminent domain proceedings. The legislative authority, a long time
ago, enacted a continuing or standing delegation of authority to the President of the Philippines to
exercise, or cause the exercise of, the power of eminent domain on behalf of the Government of the
Republic of the Philippines.

Philippine Press Institute v. COMELEC, 244 SCRA 272 – Kenny

G.R. No. L-119694 May 22, 1995

PHILIPPINE PRESS INSTITUTE, INC., for and in behalf of 139 members, represented by its President,
Amado P. Macasaet and its Executive Director Ermin F. Garcia, Jr., petitioner,
vs.
COMMISSION ON ELECTIONS, respondent.
Petitioner assailed Comelec Resolution No. 2772 as unconstitutional and void on the ground that it
violates the prohibition imposed by the Constitution upon the government, and any of its agencies,
against the taking of private property for public use without just compensation.

Whether the said resolution constitutes of taking of private property for public use without just
compensation.

The taking of private property for public use is, of course, authorized by the Constitution, but not
without payment of "just compensation" (Article III, Section 9). And apparently the necessity of paying
compensation for "Comelec space" is precisely what is sought to be avoided by respondent Commission,
whether Section 2 of Resolution No. 2772 is read as petitioner PPI reads it, as an assertion of authority
to require newspaper publishers to "donate" free print space for Comelec purposes, or as an
exhortation, or perhaps an appeal, to publishers to donate free print space, as Section 1 of Resolution
No. 2772-A attempts to suggest. There is nothing at all to prevent newspaper and magazine publishers
from voluntarily giving free print space to Comelec for the purposes contemplated in Resolution No.
2772. Section 2 of Resolution No. 2772 does not, however, provide a constitutional basis for compelling
publishers, against their will, in the kind of factual context here present, to provide free print space for
Comelec purposes. Section 2 does not constitute a valid exercise of the power of eminent domain.

Telebap v. COMELEC – 289 SCRA 337 - Kenny

TELEBAP vs. COMELEC, G.R. NO. 132922, April 21, 1998 (289 SCRA 337)

Facts: TELEBAP and GMA Network together filed a petition to challenge the validity of Comelec Time
due to the fact that said provisions: (1) have taken properties without due process of law and without
just compensation; (2) it denied the radio and television broadcast companies the equal protection of
the laws; and (3) that it is in excess of the power given to the Comelec to regulate the operation of
media communication or information during election period.

Held: Petitioners' argument is without merit, All broadcasting, whether by radio or by television stations,
is licensed by the government. Airwave frequencies have to be allocated as there are more individuals
who want to broadcast than there are frequencies to assign. A franchise is thus a privilege subject,
among other things, to amended by Congress in accordance with the constitutional provision that "any
such franchise or right granted . . . shall be subject to amendment, alteration or repeal by the Congress
when the common good so requires."
Indeed, provisions for COMELEC Time have been made by amendment of the franchises of radio and
television broadcast stations and, until the present case was brought, such provisions had not been
thought of as taking property without just compensation. Art. XII, Section 11 of the Constitution
authorizes the amendment of franchises for "the common good."

In truth, radio and television broadcasting companies, which are given franchises, do not own the
airwaves and frequencies through which they transmit broadcast signals and images. They are merely
given the temporary privilege of using them. Since a franchise is a mere privilege, the exercise of the
privilege may reasonably be burdened with the performance by the grantee of some form of public
service.

ESTATE OR HEIRS OF THE LATE EX-JUSTICE JOSE B. L. REYES, vs. CITY OF MANILA, respondent. [G.R. No.
132431. February 13, 2004]
Facts: The case esteemed from the decision of the C.A. for the issuance of protective custody in favor of
respondent on the contested 11 parcels of land owned by the petitioner situated at Sta. Cruz District,
Manila. The land in question was initially occupied and leased by different tenants, members of
Sampaguita Bisig ng Magkakabitbahay, Incorporated (SBMI). petitioners obtained a favorable judgments
against respondents ab pursuant to the decision rendered by the MTC Manila. While the case is under
adjudication, the respondents City of Manila intervenes and file a complaint for expropriation based on
its approved Ordinance No 7818 authorizing the City Mayor of Manila to expropriate certain parcels of
land with an aggregate area of 9,930square meters, more or less. It argued that the purpose of
expropriation is for a socialize housing project of the city which would otherwise benefit its
underprivileged and homeless citizens. However, petitioner turned down the offer on various reasons
among them, the failure of both to arrive at an amicable offer for the settlement of the case.

Issue: Whether respondent City deprived petitioners of their property without dueprocess of law.

Ruling: Yes. RA 7279 Sections 9 states the order of priority in the acquisition of property subject of any
expropriation intended for public purpose. Section 10 thereof, made a significant pronouncement that
“the expropriation may be resorted to only when other modes of acquisition have been exhausted: xxx
underscoring supplied”. Before respondent City can exercise its power of eminent domain, the same
must be sanctioned and must not violate any law. Reiterating the provision of R.A. 7279, it would bear
stressing that private lands rank last in the order of priority for purposes of socialized housing. In the
same vein, expropriation proceedings are to be resorted to only after the other modes of acquisition
have been exhausted. Compliance with these conditions is mandatory because these are the only
safeguards of oftentimes helpless owners of private property against violation of due process when their
property is forcibly taken from them for public use. The state in its paramount interest of promoting
public good and general welfare cannot simply ignore the rights of its citizens and such must take
precedence over the interest of private property owners. Individual rights affected by the exercise of
such right are also entitled to protection, bearing in mind that the exercise of this superior right cannot
override the guarantee of due process extended by the law to owners of the property to be
expropriated.

DIOSDADO LAGCAO v. JUDGE GENEROSA G. LABRA [G.R. No. 155746, October 13, 2004]
Facts: The Province of Cebu donated 210 lots to the City of Cebu. But then, in late 1965, the 210 lots,
including Lot 1029, reverted to the Province of Cebu. Consequently, the province tried to annul the sale
of Lot 1029 by the City of Cebu to the petitioners. This prompted the latter to sue the province for
specific performance and damages in the then Court of First Instance.

The court a quo ruled in favor of petitioners and ordered the Province of Cebu to execute the final deed
of sale in favor of petitioners. The Court of Appeals affirmed the decision of the trial court. After
acquiring title, petitioners tried to take possession of the lot only to discover that it was already
occupied by squatters. Thus petitioners instituted ejectment proceedings against the squatters. The
Municipal Trial Court in Cities (MTCC) ordering the squatters to vacate the lot. On appeal, the RTC
affirmed the MTCC’s decision and issued a writ of execution and order of demolition. However, when
the demolition order was about to be implemented, Cebu City Mayor Alvin Garcia wrote two letters to
the MTCC, requesting the deferment of the demolition on the ground that the City was still looking for a
relocation site for the squatters. Acting on the mayor’s request, the MTCC issued two orders suspending
the demolition. Unfortunately for petitioners, during the suspension period, the Sangguniang
Panlungsod (SP) of Cebu City passed a resolution which identified Lot 1029 as a socialized housing site
pursuant to RA 7279. Petitioners filed with the RTC an action for declaration of nullity of Ordinance No.
1843 for being unconstitutional.

Issue: Whether the Ordinance No. 1843 is unconstitutional as it sanctions the expropriation of their
property for the purpose of selling it to the squatters, an endeavor contrary to the concept of “public
use” contemplated in the Constitution.

Ruling: Yes. RA 7279 is the law that governs the local expropriation of property for purposes of urban
land reform and housing. Sections 9 and 10 thereof provide:
SEC 9. Priorities in the Acquisition of Land. − Lands for socialized housing shall be acquired in the
following order: (a) Those owned by the Government or any of its subdivisions, instrumentalities, or
agencies, including government-owned or controlled corporations and their subsidiaries; (b) Alienable
lands of the public domain; (c) Unregistered or abandoned and idle lands; (d) Those within the declared
Areas or Priority Development, Zonal Improvement Program sites, and Slum Improvement and
Resettlement Program sites which have not yet been acquired; (e) Bagong Lipunan Improvement of Sites
and Services or BLISS which have not yet been acquired; and (f) Privately-owned lands.

We have found nothing in the records indicating that the City of Cebu complied strictly with Sections 9
and 10 of RA 7279. Ordinance No. 1843 sought to expropriate petitioners’ property without any attempt
to first acquire the lands listed in (a) to (e) of Section 9 of RA 7279. Likewise, Cebu City failed to establish
that the other modes of acquisition in Section 10 of RA 7279 were first exhausted. Moreover, prior to
the passage of Ordinance No. 1843, there was no evidence of a valid and definite offer to buy
petitioners’ property as required by Section 19 of RA 7160.[20] We therefore find Ordinance No. 1843
to be constitutionally infirm for being violative of the petitioners’ right to due process.

Instead of looking for a relocation site during the suspension period, the city council suddenly enacted
Ordinance No. 1843 for the expropriation of petitioners’ lot. It was trickery and bad faith, pure and
simple. The unconscionable manner in which the questioned ordinance was passed clearly indicated
that respondent City transgressed the Constitution, RA 7160 and RA 7279.
Rights of Owner Before Expropriation

GREATER BALANGA DEVELOPMENT CORPORATION vs. MUNICIPALITY OF BALANGA, BATAAN [G.R.


No. 83987 December 27, 1994]
Facts: This case involves a parcel of land situated in Barrio San Jose, Municipality of Balanga, Province of
Bataan. It is registered in the name of petitioner Greater Balanga Development Corporation. Petitioner
is a domestic corporation owned and controlled by the Camacho family, which donated to the
Municipality of Balanga the present site of the Balanga Public Market. The lot in dispute lies behind the
Balanga Public Market.

petitioner conducted a relocation survey of the area. It discovered that certain portions of the property
had been "unlawfully usurped and invaded" by the Municipality of Balanga, which had
"allowed/tolerated/abetted" the construction of shanties and market stalls while charging market fees
and market entrance fees from the occupants and users of the area.

petitioner applied with the Office of the Mayor of Balanga for a business permit to engage in business in
the said area which granting petitioner the privilege of a "real estate dealer/privately-owned public
market operator" under the trade name of Balanga Public Market. However, the Sangguniang Bayan of
Balanga passed Resolution No. 12, s-88 annulling the Mayor's permit issued to petitioner and advising
the Mayor to revoke the permit "to operate a public market."

Issue: whether respondent have the right to control such property.

Ruling: No. As may be gleaned from said Resolution, the main reason for the revocation of the Mayor's
permit was the controversy engendered by the filing of Civil Case No. 3803 before the Regional Trial
Court, Balanga, Bataan involving the ownership of certain portions of Lot 261-B, the land from which Lot
261-B-6-A-3 was derived.

A close scrutiny of the records reveals that the Sangguniang Bayan did not establish or maintain any
public market on the subject lot. The resolution merely mentioned the plan to acquire the lot for
expansion of the public market adjacent thereto. Until expropriation proceedings are instituted in
court, the landowner cannot be deprived of its right over the land. Of course, the Sangguniang Bayan
has the duty in the exercise of its police powers to regulate any business subject to municipal license
fees and prescribe the conditions under which a municipal license already issued may be revoked (B.P.
Blg. 337, Sec. 149 [1] [r]). But the "anxiety, uncertainty, restiveness" among the stallholders and traders
cannot be a valid ground for revoking the permit of petitioner. After all, the stallholders and traders
were doing business on property not belonging to the Municipal government. Indeed, the claim that the
executive order and resolution were measures "designed to promote peace and order and protect the
general welfare of the people of Balanga" is too amorphous and convenient an excuse to justify
respondents' acts.

Moreover, we find that the manner by which the Mayor revoked the permit transgressed petitioner's
right to due process. The alleged violation of Section 3A-06(b) of the Balanga Revenue Code was not
stated in the order of revocation, and neither was petitioner informed of this specific violation until the
Rejoinder was filed in the instant case. In fact, with all the more reason should due process have been
observed in view of the questioned Resolution of the Sangguniang Bayan.

In view of the undisputed fact that the respondent Municipality is not the owner of Lot 261-B-6-A-3,
then there is no legal basis for it to impose and collect market fees and market entrance fees. Only the
owner has the right to do so.
BIENVENIDO VELARMA, vs. COURT OF APPEALS and JOSEFINA PANSACOLA [G.R. No. 113615. January
25, 1996]

Facts: This case arose from an “ejectment suit” filed by private respondent against petitioner before the
Regional Trial Court. The trial court found that private respondent had satisfactorily established her
ownership over the parcel of land in question. It also found that petitioner entered and occupied private
respondent’s land “without authority of law and against the will of the owner. the claim of petitioner
that “by virtue of an agreement between the former owner (Publio Pansacola) and the Municipality of
Mauban x x x the lot [being occupied by petitioner] became the property of the government, and
therefore, [respondent] has no cause of action against [petitioner]” was “baseless and unwarranted,”
since no deed had ever been executed to “perfect the deal” between the municipality and Publio for the
exchange of a portion of the abandoned provincial road with a portion of the lot owned by Publio, such
that the Pansacola spouses later demanded that petitioner vacate the land and sought the help of the
barangay council. They eventually instituted the criminal case against petitioner for violation of the Anti-
Squatting Law.
Petitioner insists that private respondent has no cause of action against him because the land on which
his house stands belongs to the government. Petitioner’s dwelling is situated on the shoulder of the
new provincial highway, part of which was constructed on a portion of the land belonging to and
titled in the name of private respondent’s husband. According to petitioner, “while it is conceded that
the premises [occupied by him] is still within the area covered by [private respondent’s] title,
nonetheless, x x x [the subject premises] x x x already belong to the government by virtue of its
exchange of the abandoned road and bridge.”

Petitioner’s claim is anchored on a document entitled “Minutes of the Meeting of the Sangguniang
Bayan of Mauban, Quezon” dated November 5, 1974. Therein, Publio Pansacola signified before the
Sangguniang Bayan of Mauban his agreement to the transfer of that portion of his land traversed by the
new provincial highway and its shoulder in exchange for a corresponding portion of the old abandoned
provincial road.

Issue: Will the lot owner’s agreement to sell the property to the government as evidenced by the
minutes of a meeting of the Sangguniang Bayan, absent a formal deed, constitute a sufficient ground to
defeat a forcible entry suit?

Ruling: No. As found by the trial court, the said minutes of the meeting of the Sangguniang Bayan do not
mention the execution of any deed to perfect the agreement. An engineer was appointed to survey the
old abandoned road, but this act does not in any manner convey title over the abandoned road to the
Pansacola spouses nor extinguish their ownership over the land traversed by the new provincial
highway. No evidence was introduced by petitioner to show that the survey was actually undertaken
and a specific portion of the abandoned road partitioned and conveyed to the Pansacolas. It must be
stressed that the agreement to transfer the property was made in 1974. More than twenty years later,
no actual transfer had yet been made. Unless and until the transfer is consummated, or expropriation
proceedings instituted by the government, private respondent continues to retain ownership of the land
subject of this case.

SOLANDA ENTERPRISES, INC. vs. COURT OF APPEALS and LUIS MANLUTAC [G.R. No. 123479. April 14,
1999]
Facts: Private Respondent Manlutac has built a residential house Rivera St., Tondo, Manila, is among the
numerous tenants/lessees of the Quijano estate for a period of more than 40 years now. The original
owners of the subject estate and at the same time the lessors of private respondent, sold thru a Deed of
Absolute Sale, the realty in favor of petitioner for Thirty Million Pesos without giving the private
respondent and the other tenants the chance to exercise their preemptive rights as accorded to them by
PD No. 1517, subject lots being declared as urbanized lands.

the tenants including private respondent, thru a letter addressed to Ceferina Quijano, signified their
acceptance of the proposed sale and concurrence [in] the selling price [of] P2,000.00 per square meter.
Mrs. Quijano sent a letter to private respondent informing him that Solanda Enterprises was already the
owner of the subject property and that the payment of monthly rental should thence be made to
Solanda Enterprises.

Solanda Enterprises registered the land in its name. the tenants including [private respondent] filed with
the RTC a complaint against Solanda Enterprises and Ceferina Quijano for annulment of sale,
reconveyance and damages. Solanda Enterprises, Inc., filed with the MTC a complaint for ejectment
against Luis Manlutac on the ground that the oral contract of lease on a parcel of land located had
expired.

the MTC rendered a summary judgment in favor of petitioner. the Regional Trial Court rendered the
assailed decision. However, upon private respondent’s petition, this Court, in its Resolution of January
11, 1995, reversed the appellate court. CA ruled that, for more than ten years, private respondent had
been a legitimate tenant of a parcel of land “which is within the Urban Zone.” Hence, he had the right of
first refusal to purchase the same at a reasonable price and within a reasonable time.

Issues: Whether respondent court committed reversible errors in applying PD 1517 when the land in
dispute was not located within an area for priority development (APD) and an urban land reform zone
(ULRZ).

Ruling: We agree. A close reading of Proclamation No. 1967 reveals that, before a preemptive right can
be exercised, the disputed land should be situated in an area declared to be both an APD and a ULRZ.

Although the Court did not categorically rule that an urban tenant had the right of first refusal only
when there was a declaration that the disputed land was within both an APD and a ULRZ, the fact
remains that there was such a conjunction in the said case.

The Court further notes a certification from the Housing and Land Use Regulatory Board (HLURB) that
the said property, covered by TCT No. 198580, is outside any APD and ULRZ specified in Proclamation
1967 and the Locational Reference Handbook.

Ordinance No. 7806 of the City of Manila, “Authorizing the Honorable City Mayor to work for the
Acquisition, Either by Negotiation or Expropriation, of a Parcel of Land xxx Owned by Juan O. Quijano
xxx,” expresses merely an intent to expropriate the land in dispute. Said ordinance does not show that
the disputed land, when expropriated, will be awarded or sold to the private respondent. It does not
demonstrate or touch upon private respondent’s right of possession over the disputed land. It is, at
best, speculative and not a bar to an action for ejectment.

REPUBLIC OF THE PHILIPPINES vs. SALEM INVESTMENT CORPORATION, MARIA DEL CARMEN ROXAS
DE ELIZALDE, CONCEPCION CABARRUS VDA. DE SANTOS [G.R. No. 137569. June 23, 2000]

Facts: Batas Pambansa Blg. 340 was passed authorizing the expropriation of parcels of lands in the
names of defendants in this case. the Republic of the Philippines filed the present case for expropriation
pursuant to B.P. Blg. 340. Among the defendants named in the complaint were Milagros and Inocentes
De la Rama as registered owners of Lot 834, a portion of which (Lot 834-A) was part of the expropriated
property. Upon the deposit of P12,970,350.00 representing 10 percent of the approximate market
value of the subject lands, a writ of possession was issued in favor of the government.

Meanwhile, a case for specific performance to sell the subject lot was initiated by Guerrero to compel
the sale of the land against De La Rama. the Pasay City Regional Trial Court, declared Guerrero the
rightful owner of the 920-square meter expropriated property and ordered payment to him of just
compensation for the taking of the land.

Issue: Whether Guerrero is entitled to receive the just compensation of the expropriation proceedings.
Ruling: No. there are 3 stages of expropriation. The first is concerned with the determination of the
authority of the plaintiff to exercise the power of eminent domain and the propriety of its exercise in
the context of the facts involved in the suit. It ends with an order, if not dismissal of the action, "of
condemnation declaring that the plaintiff has a lawful right to take the property sought to be
condemned, for the public use or purpose declared in the complaint, upon the payment of just
compensation to be determined as of the date of the filing of the complaint"…xxx.
The second phase of the eminent domain action is concerned with the determination by the court of
"the just compensation for the property sought to be taken." This is done by the court with the
assistance of not more than three (3) commissioners…xxx

It is only upon the completion of these two stages that expropriation is said to have been
completed. Moreover, it is only upon payment of just compensation that title over the property
passes to the government. Therefore, until the action for expropriation has been completed and
terminated, ownership over the property being expropriated remains with the registered owner.
Consequently, the latter can exercise all rights pertaining to an owner, including the right to dispose of
his property, subject to the power of the State ultimately to acquire it through expropriation.

Thus, the De la Ramas still had authority to transfer ownership of their land and convey all rights,
including the right to receive just compensation, to Guerrero.

1. Elements of “Taking”

Republic vs. Vda. de Castellvi [GR L-20620, August 15, 1974]


Facts: Petitioner Republic of the Philippines (Philippine Air Force) occupied the land situated in Carmen
M. vda. de Castellvi by virtue of a contract of lease. Before the expiration of the contract of lease, the
Republic sought to renew the same but Castellvi refused, intending to subdivide the lots for sale to the
general public; filed civil case for ejectment of Armed Forces of the Philippines.

In view of the difficulty for the army to vacate the premises due to permanent installations and other
facilities, AFP filed expropriation proceedings and was placed in possession of the lands.

In its complaint, the Republic alleged, among other things, that the fair market value of the above-
mentioned lands, according to the Committee on Appraisal for the Province of Pampanga, was not more
than P2,000 per hectare (P.20/sqm), or a total market value of P259,669.10 when AFP first had the
“taking” of the said property by virtue of the special lease agreement. Respondents allege that their
lands are residential with a fair market value of not less than P15/sqm.

The trial court rendered its decision, finding that the unanimous recommendation of the commissioners
of P10.00 per square meter for the 3 lots subject of the action is fair and just compensation

Issue: Whether the “taking” of the properties under expropriation commenced with the filing of the
action or upon the occupation of the premises by the AFP.

Ruling: The "taking" of Catellvi's property for purposes of eminent domain cannot be considered to
have taken place in 1947 when the Republic commenced to occupy the property as lessee. Elements B &
E were not present when Republic entered the properties in 1947.

Elements/Requisites of “taking” of property for purposes of eminent domain:


A. Expropriator must enter a private property.
B. Entrance into private property must be for more than a momentary period.
C. Entry into the property should be under warrant or color of legal authority.
D. Property must be devoted to a public use or otherwise informally appropriated or injuriously
affected.
E. Utilization of the property for public use must be in such a way as to oust the owner and
deprive him of all beneficial enjoyment of the property.

the “just compensation” is to be determined as of the date of the filing of the complaint. Herein, it is
undisputed that the Republic was placed in possession of the Castellvi property, by authority of the
court, on 10 August 1959.

The “taking” of the Castellvi property for the purposes of determining the just compensation to be paid
should not be paid based on 1947 fair market value amount.

Garcia v. CA – 102 SCRA 597 - JC

Facts: Napocor sought to expropriate the property of petitioner. The RTC pegged it at P15.00. The CA
pegged it at P0.07 per square meter.

Issue: Just compensation

Held: The final question involves the determination of the just compensation. Just compensation is the
market value of the property. It should be determined at the time of the taking. It is the price which it
will command where it is offered for sale by one who desires, but is not obliged to sell, and is bought by
one who is under no necessity of having it.

City of Government v. Judge Ericta – 122 SCRA 759 - JC

Facts: Petitioners issued an ordinance which mandated that 6% of the total land area of cemeteries be
set aside for paupers and residents of Quezon City. Private respondent challenges its validity as it is
tantamount to taking or confiscation without just compensation.

Issue: Whether the ordinance is valid

Held: No. The SC held the law as an invalid exercise of police power. There is no reasonable relation
between the setting aside of at least six (6) percent of the total area of all private cemeteries for charity
burial grounds of deceased paupers and the promotion of health, morals, good order, safety, or the
general welfare of the people. The ordinance is actually a taking without compensation of a certain area
from a private cemetery to benefit paupers who are charges of the municipal corporation. Instead of
building or maintaining a public cemetery for this purpose, the city passes the burden to private
cemeteries.

US v. Causby – 328 US 256 - JC

Facts: Thomas Lee Causby owned a chicken farm outside of Greensboro, North Carolina. The farm was
located near an airport used regularly by the United States military. According to Causby, noise from the
airport regularly frightened the animals on his farm, resulting in the deaths of several chickens. The
problem became so severe that Causby was forced to abandon his business. Under an ancient doctrine
of the common law, land ownership extended to the space above and below the earth. Using this
doctrine as a basis, Causby sued the United States, arguing that he owned the airspace above his farm.
By flying planes in this airspace, he argued, the government had confiscated his property without
compensation, thus violating the Takings Clause of the Fifth Amendment. The United States Court of
Claims accepted Causby's argument, and ordered the government to pay compensation.

Issue: Whether there was taking of property thus justifying just compensation

Held: Yes, to an extent. In a 5-2 opinion authored by Justice William O. Douglas, the Court concluded
that the ancient common law doctrine "has no place in the modern world." Justice Douglas noted that,
were the Court to accept the doctrine as valid, "every transcontinental flight would subject the operator
to countless trespass suits. Common sense revolts at the idea." However, while the Court rejected the
unlimited reach above and below the earth described in the common law doctrine, it also ruled that, "if
the landowner is to have full enjoyment of the land, he must have exclusive control of the immediate
reaches of the enveloping atmosphere." Without defining a specific limit, the Court stated that flights
over the land could be considered a violation of the Takings Clause if they led to "a direct and
immediate interference with the enjoyment and use of the land." Given the damage caused by the
particularly low, frequent flights over his farm, the Court determined that the government had
violated Causby's rights, and he was entitled to compensation.

People v. Fajardo – 104 Phil 443 - JC

Facts: The municipal council of baao, camarines sur stating among others that construction of a building,
which will destroy the view of the plaza, shall not be allowed and therefore be destroyed at the
expense of the owner, enacted an ordinance. Herein appellant filed a written request with the
incumbent municipal mayor for a permit to construct a building adjacent to their gasoline station on a
parcel of land registered in Fajardo's name, located along the national highway and separated from the
public plaza by a creek. The request was denied, for the reason among others that the
proposed building would destroy the view or beauty of the public plaza. Defendants reiterated their
request for a building permit, but again the mayor turned down the request. Whereupon, appellants
proceeded with the construction of the building without a permit, because they needed a place of
residence very badly, their former house having been destroyed by a typhoon and hitherto they had
been living on leased property. Thereafter, defendants were charged in violation of the ordinance and
subsequently convicted. Hence this appeal.

Issue: Whether the ordinance is valid

Held: A Municipal Ordinance is unreasonable and oppressive if it operates to permanently deprive


appellants of the right to use their own property; it then oversteps the bounds of police power
without just compensation. We do not overlook that the modern tendency is to regard the
beautification of neighborhoods as conducive to the comfort and happiness of residents. But while
property may be regulated in the interest of the general welfare and, in its pursuit, the State may
prohibit structures offensive to sight (Churchill and Tait v. Rafferty, 32 Phil., 580), the State may not,
under guise of police power, permanently divest owners of the beneficial use of their property and
practically confiscate them solely to preserve or assure the aesthetic appearance of the community. To
legally achieve that result, the landowner should be given just compensation and an opportunity to be
heard.
Republic v. PLDT – 26 SCRA 620 - JC

Facts: Public petitioner commenced a suit against private respondent praying for the right of the Bureau
of Telecommunications to demand interconnection between the Government Telephone System and
that of PLDT, so that the Government Telephone System could make use of the lines and facilities of the
PLDT. Private respondent contends that it cannot be compelled to enter into a contract where no
agreement is had between them.

Issue: Whether the interconnection between PLDT and the Government Telephone System can be a
valid object for expropriation.

Held: Yes, in the exercise of the sovereign power of eminent domain, the Republic may require the
telephone company to permit interconnection as the needs of the government service may require,
subject to the payment of just compensation. The use of lines and services to allow inter-service
connection between the both telephone systems, through expropriation can be a subject to an
easement of right of way.

NPC v. Jocson – 206 SCRA 520 - JC

Facts: Petitioner filed eminent domain cases for the acquisition of right of way easement of private
lands. Because they could not come to an agreement as to just compensation, the eminent domain
cases were initiated. Respondent judge issued an order determining the provisional market value based
on tax declarations. Petitioner deposited the same. The landowners objected because it was too low;
the judge ordered petitioner to pay the differential and held in abeyance the writ of possession until full
payment and the same shall have been received by the landowners.

Issue: Whether the procedure adopted by the judge is improper

Held: The court ruled that PD No. 42 provides that upon filing in court complaints on eminent domain
proceeding and after due notice to the defendants, plaintiff will have the right to take possession of the
real property upon deposit of the amount of the assessed value with PNB to be held by the bank subject
to orders and final disposition of the court. The respondent judge failed to observe this procedure by
failure to issue the writ of possession to the petitioner despite its effort to deposit the amount in
compliance to the mandate of law. Furthermore, the respondent judge erred in increasing the
provisional value of properties without holding any hearing for both parties. The instant petition was
granted by the court setting aside the temporary restraining order and directing respondent judge to
cease and desist from enforcing his orders.

There are 2 stages in the action of expropriation:

1. Determination of the authority of the plaintiff to exercise the power of eminent domain and the
propriety of its exercise in the context of the facts involved in the suit.
2. Eminent domain action is concerned with the determination by the Court of the "just compensation
for the property sought to be taken." This is done by the Court with the assistance of not more than
three (3)commissioners whose findings are deemed to be final

Penn Central Transportation v. NY City 438 US 104 - JC


Facts: The New York City Landmarks Preservation Law of 1965 empowered the city to designate certain
structures and neighborhoods as "landmarks" or "landmark sites." Penn Central, which owned the
Grand Central Terminal (opened in 1913), was not allowed to construct a multistory office building
above it.

Issue: Whether the restriction is tantamount to taking of property

Held: No. The Court held that the restrictions imposed did not prevent Penn Central from ever
constructing above the terminal in the future. New York's objection was to the nature of the proposed
construction and not to construction in general implemented to "enhance" the Terminal. Preventing the
construction of a 50-plus story addition above the station was a reasonable restriction substantially
related to the general welfare of the city.

Ruckelshaus v. Monsanto – 467 US 986 – Xavier


FACTS: The Federal Insecticide, Fungicide, and Rodenticide Act (FIFRA) authorizes the Environmental
Protection Agency (EPA) to use data (including trade secrets) submitted by a prior pesticide applicant for
registration of a persticide product in evaluating the application of a subsequent applicant, and to
disclose publicly some of the submitted data so long as the second applicant paid just compensation for
the data.
Appellee, a company headquartered in Missouri, is an inventor, producer, and seller of
pesticides, alleged that the data-consideration and data-disclosure provisions of FIFRA effected a
"taking" of property without just compensation, in violation of the Fifth Amendment.
Issue: Whether the data-consideration and data-disclosure provisions of FIFRA effected a "taking" of
property without just compensation, in violation of the Fifth Amendment.
Held: NO. As long as appellee is aware of the conditions under which the data are submitted, and the
conditions are rationally related to a legitimate Government interest, a voluntary submission of data in
exchange for the economic advantages of a registration can hardly be called a taking.
Furthermore, a factor for consideration in determining whether a governmental action short of
acquisition or destruction of property has gone beyond proper "regulation" and effects a "taking" is
whether the action interferes with reasonable investment-backed expectations. In this case, appellee
could not have had a reasonable, investment-backed expectation that EPA would keep the data
confidential beyond the limits prescribed in the amended statute itself.
NPC v. Manubay – 437 SCRA 60 – Xavier
FACTS: National Power Corporation commenced its 350 KV Leyte-Luzon HVDC Power Transmission
Project. The project, which is for a public purpose, aims to transmit the excess electrical generating
capacity coming from Leyte Geothermal Plant to various load centers in the entire country into a single
power grid. The transmission lines necessarily crossed over certain lands owned by private entities,
among them is respondent Manubay Agro-Industrial Development Corporation.
Petitioner NPC contends that the valuation of the expropriated property -- fixed by the RTC and
affirmed by the CA -- was too high a price for the acquisition of an easement of a mere aerial right of
way, because respondent would continue to own and use the subject land anyway. Petitioner argues
that in a strict sense, there is no “taking” of property, but merely an imposition of an encumbrance or a
personal easement/servitude under Article 614 of the Civil Code. Hence, the latter should be
compensated only for what it would actually lose.
ISSUE: Whether there is no “taking” of property, but merely an imposition of an encumbrance or a
personal easement/servitude thus entitling to a reduced valuation of price for the acquisition.
HELD: NO. The acquisition of such an easement falls within the purview of the power of eminent
domain. Thus, the latter is entitled to payment of a just compensation, which must be neither more
nor less than the monetary equivalent of the land. Even granting arguendo that what petitioner
acquired over respondent’s property was purely an easement of a right of way, still, the Court cannot
sustain its view that it should pay only an easement fee, and not the full value of the property.
Furthermore, the determination of just compensation is a judicial function.
NPC v. San Pedro – 503 SCRA 333 – Xavier
FACTS: For the construction of its San Manuel-San Jose 500 KV Transmission Line and Tower, NPC
negotiated with Maria Mendoza San Pedro for an easement of right of way over her property. The
payment voucher for the residential portion of the lot valued at P6,000,000.00 (at P600.00 per square
meter) but, NPC said it would only pay P230.00 per sq m for the residential portion and P89.00 per sq m
for the agricultural portion. Trial ensued.
NPC argued that the property involved was actually and principally used as agricultural, though
declared as agricultural/residential lots; hence, only the easement fee of right of way should be paid, as
the principal purpose for which the lot was devoted would not be impaired by the construction of
transmission lines.
ISSUE: Whether easement of right-of-way is “taking” thus subject to a reasonable estimate of just
compensation.
HELD: YES. Indeed, expropriation is not limited to the acquisition of real property with a
corresponding transfer of title or possession. In the case at bar, the easement of right-of-way is
definitely a taking under the power of eminent domain. Considering the nature and effect of the
installation of the 230 KV Mexico-Limay transmission lines, the limitation imposed by NPC against the
use of the land for an indefinite period deprives private respondents of its ordinary use. As such,
easement of right-of-way entitles one to just compensation for private property condemned for public
use.
NPC v. Tiangco – 514 SCRA 674 - Xavier
FACTS: NPC requires 19,423 square meters of the respondents’ aforementioned property, across which
its 500Kv Kalayaan-San Jose Transmission Line Project will traverse. After repeated unsuccessful
negotiations with the respondents, NPC filed with the RTC. NPC made it clear that it is interested only
in acquiring an easement of right-of-way over the respondents’ property and that ownership of the
area over which the right-of-way will be established shall remain with the respondents. For this
reason, NPC claims that it should pay, in addition to the agreed or adjudged value of the improvements
on the area, only an easement fee in an amount equivalent to ten per cent (10%) of the market value
of the property as declared by the respondents or by the Municipal Assessor, whichever is lower, as
provided for under Section 3-A of Republic Act No. 6395, as amended by Presidential Decree 938.
ISSUE: Whether there is “taking” of property notwithsanding only an easement of right-of-way for the
establishment of a safe and free passage for the overhead transmission lines is assumed and that
ownership of the area over which the right-of-way will be established shall remain with the
respondents.
HELD: YES. There is taking. True, an easement of a right-of-way transmits no rights except the easement
itself, and the respondents would retain full ownership of the property taken. However, if the easement
is intended to perpetually or indefinitely deprive the owner of his proprietary rights through the
imposition of conditions that affect the ordinary use, free enjoyment and disposal of the property or
through restrictions and limitations that are inconsistent with the exercise of the attributes of
ownership, or when the introduction of structures or objects which, by their nature, create or increase
the probability of injury, death upon or destruction of life and property found on the land is necessary,
then the owner should be compensated for the monetary equivalent of the land.
In eminent domain cases, the time of taking is the filing of the complaint, if there was no actual
taking prior thereto. Hence, in this case, the value of the property at the time of the filing of the
complaint on November 20, 1990 should be considered in determining the just compensation due the
respondents.
LBP v. Imperial – 515 SCRA 449 – Xavier
FACTS: Respondent Juan H. Imperial owned five parcels of land. Pursuant to the Land Reform Program
under Presidential Decree No. 27 and Executive Order No. 228, the Department of Agrarian Reform
(DAR) placed these lands under its Operation Land Transfer (OLT) and were subsequently distributed to
deserving farmer beneficiaries. Imperial filed a complaint for determination and payment of just
compensation.
The petitioner claims that the areas used as feeder road, right of way, and barrio site, should not
be included in the computation since they are not devoted to or suitable for agriculture in
contemplation of Rep. Act No. 6657 and the title to these areas remained with the respondent.
ISSUE: Whether the areas used as feeder road, right of way, and barrio site should be considered as
compensable.
HELD: YES. Although such areas were not strictly used for agricultural purposes, the same were
diverted to public use. For this reason, the respondent should be compensated for what he actually lost
and that should include not only the areas distributed directly to the tenant beneficiaries but also
those areas used as feeder road, right of way, and barrio site, which were undoubtedly diverted to the
use of the public. The only area that ought to be excluded is the portion or portions retained by the
respondent as owner-cultivator for his own use.
NPC v. Bongbong – 520 SCRA 290 – Xavier
FACTS: As early as 1996, the National Power Corporation (NPC) negotiated with the spouses Bongbong
to use a portion of the property for the construction of a 230 KV LCIP Malitbog-Tabango CETL TWR SITE
1046 for the Leyte-Cebu Interconnection Project. The spouses Bongbong demanded that the NPC
pay P8,748,448.00 which they alleged to be the just and reasonable value for their land and
improvements. NPC refused and trial ensued for just compensation before the RTC.
Petitioner insists that if any amount should be paid to respondents, it should only be an
easement fee of 10% the value of the property, not the full value, since it acquired only a simple right-
of-way easement for the passage of its overhead transmission lines. It points out that its charter
authorizes the acquisition only of a right-of-way easement for its transmission lines and the payment
of an easement fee.
ISSUE: Whether the easement fee of 10% the value of the property, not the full value, since it acquired
only a simple right-of-way easement for the passage of its overhead transmission lines.
HELD: NO. The Court has consistently held that the determination of just compensation is a judicial
function. No statute, decree, or executive order can mandate that its own determination shall prevail
over the court’s findings.
True, an easement of a right of way transmits no rights except the easement itself, and owner
retains full ownership of the property. The acquisition of such easement is, nevertheless, not gratis.
Considering the nature and the effect of the installation of power lines, the limitations on the use of
the land for an indefinite period would deprive owner of normal use of the property. For this reason,
the latter is entitled to payment of just compensation which must be neither more nor less than the
monetary equivalent of the land.
Tan v. Republic - 423 SCRA 203 – Xavier
FACTS: Petitioner Julita P. Tan acquired a property from the San Antonio Development Corporation
(SADC) by assuming SADC’s "obligation of paying all imposable taxes due said land." Prior to the transfer
of the property to petitioner PEA wrote SADC requesting permission to enter the property for the
purpose of constructing the southern abutment of the Zapote Bridge at the Coastal Road to which SADC
agreed on condition that PEA should pay a monthly rental of P10,000.00. PEA also proposed to SADC to
start their negotiation for its acquisition of the latter’s property but which never materialized.
Later, despite the construction of the Coastal Road was completed. PEA continuously refuses to
pay petitioner Tan any compensation. Trial commenced and PEA alleged therein, among others, that its
liability for just compensation is based on the zonal value of the land at the time of the taking in 1985.
Petitioner, on the other hand, claimed that PEA should pay for the whole area consisting of 7,161 square
meters at P20,000.00 per square meter (a whopping total of P143,200,000.00 plus interest of 12% per
annum), the zonal value set by the BIR pursuant to Republic Act No. 8974.
ISSUE: Whether the just compensation for petitioner’s property should be based on the BIR zonal
valuation in 1985 when petitioner entered the subject property.1awphi1.nét
HELD: NO. Just compensation is determined as of the (1) date of the taking of the property or (2) the
filing of the complaint, whichever came first.
We have made it clear that there was no taking of the property in 1985 by PEA for purposes of
expropriation. As shown by the records, PEA filed with the RTC its petition for expropriation on
September 22, 2003. The trial court, therefore, was correct in ordering respondent, through PEA, upon
the filing of its complaint for expropriation, to pay petitioner just compensation on the basis of the BIR
zonal valuation of the subject property at P20,000.00 per square meter.
NPC v. Ibrahim – 526 SCRA 149 - Xavier
FACTS: Respondent Ibrahim claimed that they were owners of several parcels of land. In 1978, NPC
without permission from respondent Ibrahim, constructed underground tunnels in the sub-terrain area
respondent’s lands. However, it was only discovered in 1992, when Omar G. Maruhom’s request to
install a motorized deep well was rejected by the Marawi City Water District. Respondent Ibrahim
instituted an action against the NPC for recovery of possession of land and damages before the RTC. At
the trial, Petitioner NPC contends that the underground tunnels in this case constitute an easement
upon the property of respondents which does not involve any loss of title or possession.
ISSUE: Whether the underground tunnels in this case constitute an easement upon the property of
respondents which does not involve any loss of title or possession.
HELD: NO. The underground tunnels impose limitations on respondents’ use of the property for an
indefinite period and deprive them of its ordinary use. Based upon the foregoing, respondents are
clearly entitled to the payment of just compensation. Notwithstanding the fact that petitioner only
occupies the sub-terrain portion, it is liable to pay not merely an easement fee but rather the full
compensation for land. This is so because in this case, the nature of the easement practically deprives
the owners of its normal beneficial use.

NAPOCOR vs PUREFOODS
G.R. No. 160725
September 12, 2008

National Power Corporati-on, Petitioner, vs PUREFOODS Corporation, Solid Development Corporation,


Jose Ortega Jr., Silvestre Bautista, Alfredo Cabande, Heirs of Victor Trinidad and Moldex Reality
Incorporated, Respondents

Facts:
NAPOCOR, a government-owned and controlled corporation created for the purpose of undertaking
development of hydroelectric power generation, the production of electricity from nuclear, geothermal
and other sources and the transmission of electric power on a nationwide basis, is also empowered to
acquire property incident to or necessary, convenient or proper to carry out its purpose. It is also
empowered to enter private property in the lawful performance of its business purposed provided that
the owners of such private property shall be indemnified for any damage that may be caused thereby
and exercise the right of eminent domain.

In order to construct and maintain its Northwestern Luzon Project , NAPOCOR had to acquire an
easement if right-of-way over certain parcels of land situated in the towns of Angat, San Rafael and San
Ildefonso and in the city of San Jose del Monte ---- all in the province of Bulacan.

On November 5, 1997, NAPOCOR filed a special civil action for eminent domain before the RTC of
Malolos, Bulacan. The defendants were the vendors and vendees of the affected parcels of land.

The complaint alleged that the defendants were either the registered owners or the claimants of the
affected pieces of properties. It also alleged that the public purpose of the Northwestern Luzon Project,
as well as the urgency and necessity of acquiring easements of right-of-way over the said parcels of land
consisting of 62,426.50 sq.m. It also averred that the affected properties were selected by NAPOCOR in
a manner compatible with the greatest public good and the least private injury and had not been
expropriated for public use and that the negotiations between the NAPOCOR and the defendants failed.
The complaint prayed, among others, that the RTC issue a writ of possession in favor of the NAPOCOR in
the event that it would be refused entry to the affected properties. Only the defendants herein filed
their respective answers.

Respondents Heirs of Trinidad claimed that they should be indemnified for the value of the affected
property based on the prevailing market purchase price of P 750/sq.m. They also added that there are
other parcels of land within the area which are more suitable for NAPOCOR’s project.

Respondent Moldex, alleged that the expropriation of part if the landholding in which it has propriety
interest would divest the peripheral area of its value and render the same totally useless and thus must
be compensated for the loss of the peripheral area as well.

Purefoods Corporation and SDC prayed for the dismissal of the complaint. SDC averred that the taking
would not serve any public purpose and that the selection of its property for expropriation would not be
compatible with the greatest public good and the least private injury. Purefoods who also prayed for the
dismissal of the complaint argued that NAPOCOR failed to append copies of the pertinent Torrens titles
to the complaint. It also averred that NAPOCOR’s offer was excessively low, undervalued and obsolete
and that its action had caused extreme prejudice to its investment and further delay in the construction
and development of its piggery business, thereby adversely affecting its operation.

On December 19, 1997, NAPOCOR filed its Urgent Ex Parte Motion for the Issuance of Writ of Possession
wherein it alleged that it had deposited with the Land Bank of the Philippines the amount of P 126,
565.42 as provisional valuation of the properties sought to be expropriated and that it had sent a Notice
to Take Possession of the said properties. The RTC directed the clerk of court to issue a writ of
possession on January 6, 1998.
The RTC issued an order dated June 14, 1999, after the pre-trial conference, reflecting he parties’
agreement to limit the issues to the amount of just compensation and to whether respondent Moldex
was entitles to just compensation on the devaluation of the peripheral area within its property.

The RTC appointed a second set of commissioners after the first set of appointed commissioners failed
to discharge their duties. On May 18, 2001, the commissioners submitted separate reports to the RTC
which formed part of the case records. The commissioners recommended that the compensation due
from NAPOCOR be based on the fair market value of P 600.00/sq.m. for the properties belonging to
Moldex and P 400.00/sq.m. for the undeveloped and underdeveloped properties belonging to the rest
of the respondents. In the rendered decision of the RTC dated December 17, 2001, it fixed the amount
of P600.00/sq.m. for the land of Moldex as just compensation and P400.00/sq.m. for the properties of
the other respondents and payment of interest of 12% per annum from the finality of its decision until
full payment thereof.

Moldex sought for reconsideration of the said decision but was denied by the RTC in its Order dated
December 7, 2001. Both Moldex and NAPOCOR filed separate appeals before the Court of Appeals.

Moldex argues that the RTC erred in the following instances: (1) in ruling that just compensation should
be paid at P600/sq.m. and not P1,600/sq.m; (2) in not imposing interest of 12% per annum reckoned
from the taking until the finality of the decision and; (3) in not ordering the payment of just
compensation for the peripheral portion of the affected property.

NAPOCOR assailed the RTC’s valuations of the properties at P600/sq.m and P400/sq.m, contending that
the same are not based on the value of the properties at the time of the taking when the properties
were still agricultural in nature. It claimed that only an easement fee, which should not exceed 10% of
the declared market value, should be paid to the respondents. It also questioned the awared of interest
of 12% per annum from the finality of the decision until the full payment of the amount adjudged.

On November 7, 2003, the CA affirmed the RTCs decision in all respect except for the period during
which the interest of 12% per annum would accrue.

Moldex sought the reconsideration of the November 7, 2003 decision of the CA while NAPOCOR,
through the OSG, elevated the case to the SC via petition for review of certiorari.

ISSUE: Whether or not only the easement fee of 10% of the market value of the expropriated properties
should be paid for the affected owners.
RULING: No. The question of just compensation for an easement of right-of-way over a parcel of land
that will be traversed by NAPOCOR’s transmission lines has already been answered in the case of
NAPOCOR vs Manubay Agro-Industrial Development Corporation. In the said case, it was held that
because of the nature of the easement, which will deprive normal use of ther land for an indefinite
period, just compensation must be based on the full market value of the affected properties. Also in the
case of NAPOCOR vs Aguirre-Paderanga, the court noted that the passage of NAPOCOR’s transmission
lines over the affected property causes not only actual damage but also restriction on the agricultural
and economic activity normally undertaken on the entire property. While in the said case NAPOCOR was
seeking to acquire only an easement of the right-of-way, the court nonetheless rules that the just
compensation in the amount of only 10% of the market value of the property was not enough to
indemnify the incursion of the affected property.
The Court explained therein that expropriation is not limited to the acquisition of real property
with corresponding transfer of title or possession. The right-of-way easement resulting in a restriction or
limitation on property rights over the land traversed by transmission lines, as in the present case, also
falls within the ambit of the terms “expropriation”. In eminent domain or expropriation proceedings, the
general rule is that the just compensation to which the owner of the condemned property is entitles is
the market value and market value is the “sum of money which a person desirous but not compelled to
buy, and an owner willing but not compelled to sell, would agree on as a price to be given and receive
therefor. The aforementioned rule, however, is modified where only a part of a certain property is
expropriated. In such case the owner is not restricted to compensation for the portion actually taken. In
addition to the market value of the portion taken, he is also entitled to recover the consequential
damage, if any, to the remaining part of the property. At the same time, from the total compensation
must be deducted the value of the consequential benefits.”

While section 3(a) of R.A. No. 6395, as amended, and the implementing rule of R.A. No. 8974 indeed
state that only 10% of the market value of the property is due to the owner of the property subject to an
easement of right-of-way, said rule is not binding on the court. Well-settled is the rule that the
determination of “just compensation” in eminent domain cases is a judicial function.

Thus, the decision of the RTC and CA with regard to the value of just compensation of P 400/sq.m.
of the properties of the respondents except for Moldex which is P600/sq.m. is affirmed.

NPC v CAPIN
FACTS: Petitioner already admitted that it had taken portions of respondents' lands for the construction
of its power lines and transmission towers pursuant to its Interconnection Project. However, the parties
could not agree on the amount of just compensation or damages that petitioner should pay respondents
for the lands taken. Respondents insist that they be paid the full market value of the portions of their
lots taken by the petitioner, while petitioner believed that it was only bound to pay respondents
easement fees, which was equivalent to 10% of the market value of the respondents' lots as indicated in
their tax declarations, pursuant to Section 3-A of petitioner's Charter.

ISSUE: Whether the construction of its power lines and transmission towers over one’s land constitute
taking. >>>yes.

RULING:
The foregoing facts considered, the acquisition of the right-of-way easement falls within the purview of
the power of eminent domain. In the case at bar, the easement of right-of-way is definitely a taking
under the power of eminent domain. Considering the nature and effect of the installation of the 230 KV
Mexico-Limay transmission lines, the limitation imposed by NPC against the use of the land for an
indefinite period deprives private respondents of its ordinary use. Having established that petitioner's
acquisition of right-of-way easement over the portions of respondents' lots was definitely a taking under
the power of eminent domain, petitioner then is liable to pay respondents just compensation and not
merely an easement fee.

PNOC VS MAGLASANG
Facts: Petitioner insists that contrary to the findings of the two courts below, the determination of just
compensation should be reckoned prior to the time of the filing of the complaint for expropriation.
According to petitioner in Civil Case No. 3267-O, petitioner took possession of the land on January 1,
1992 when PNOC leased the same from its administrator as evidenced by a Lease Agreement for the
period of January 1, 1992 to December 31, 1992.Thus, taking, for purposes of computing just
compensation, should have been reckoned from January 1, 1992.

Ruling:

The Court rejected the State’s contention that a lease on a year to year basis can give rise to a
permanent right to occupy, since by express legal provision a lease made for a determinate time, ceases
upon the day fixed, without need of a demand.Neither can it be said that the right of eminent domain
may be exercised by simply leasing the premises to be expropriated.Where, as here, the owner was
compensated and not deprived of the ordinary and beneficial use of his property by its being diverted to
public use, there is no taking within the constitutional sense.

The time of the taking may be reckoned in 1994.For Lot 11900, on October 24, 1994, the date of the
filing of the complaint although the plaintiff took possession of the property in 1991 due to a lease
contract executed between plaintiff and defendant yet the intention to expropriate was manifested only
upon the filing of the complaint

NPC v CA
FACTS: In 1978, National Power Corporation (NAPOCOR), took possession of a 21,995 square meter land
which is a portion of Lot 1 of the subdivision plan (LRC) Psd-116159 situated in Marawi City, owned by
Mangondato, and covered by Transfer Certificate of Title No. T-378-A, under the mistaken belief that it
forms part of the public land reserved for use by NAPOCOR for hydroelectric power purposes under
Proclamation No. 1354 of the President of the Philippines dated December 3, 1974. In 1979, when
NAPOCOR started building its Agus I HE (Hydroelectric Plant) Project, Mangondato demanded
compensation from NAPOCOR. NAPOCOR refused to compensate insisting that the property is public
land and that it had already paid ‘financial assistance’ to Marawi City in exchange for the rights over the
property.

Mangondato claimed that the subject land is his duly registered private property covered by Transfer
Certificate of Title No. T-378-A in his name, and that he is not privy to any agreement between
NAPOCOR andMarawi City and that any payment made to said city cannot be considered as payment to
him.

ISSUE: When is There “Taking” of Property?


RULING: “A number of circumstances must be present in the ‘taking’ of property for purposes of
eminent domain: (1) the expropriator must enter a private property; (2) the entrance into private
property must be for more than a momentary period; (3) the entry into the property should be under
warrant or color of legal authority; (4) the property must be devoted to a public use or otherwise
informally appropriated or injuriously affected; and (5) the utilization of the property for public use must
be in such a way to oust the owner and deprive him of all beneficial enjoyment of the property.”

In this case, the petitioner’s entrance in 1978 was without intent to expropriate or was not made under
warrant or color of legal authority, for it believed the property was public land covered by Proclamation
No. 1354. When the private respondent raised his claim of ownership sometime in 1979, the petitioner
flatly refused the claim for compensation, nakedly insisted that the property was public land and
wrongly justified its possession by alleging it had already paid “financial assistance” to Marawi City in
exchange for the rights over the property. Only in 1990, after more than a decade of beneficial use, did
the petitioner recognize private respondent’s ownership and negotiate for the voluntary purchase of the
property. A Deed of Sale with provisional payment and subject to negotiations for the correct price was
then executed.

Clearly, this is not the intent nor the expropriation contemplated by law. This is a simple attempt at a
voluntary purchase and sale. Obviously, the petitioner neglected and/or refused to exercise the power
of eminent domain.

Only in 1992, after the private respondent sued to recover possession and petitioner filed its Complaint
to expropriate, did petitioner manifest its intention to exercise the power of eminent domain.

NPC v VILLAMOR

FACTS:
Due to its Leyte-Cebu Interconnection Project, NPC’s 230 KV Talisay-Compostela transmission lines and
towers have to pass parcels of land in the City of Danao and Municipality of Carmen, both situated in the
province of Cebu. Two of these lands situated in Cantipay, Carmen, Cebu are owned by respondent
Carlos Villamor (Villamor). On these lands stand fruit-bearing trees, such as mango, coconut, avocado,
soursop or guyabano, jackfruit, tamarind, breadfruit, sugar apple or atis, Spanish plum or siniguelas and
banana; and non-fruit bearing trees, such as mahogany and gemilina. Petitioner contends that under
Section 3A of its charter, RA 6395, where private property will be traversed by transmission lines, NPC
shall only acquire an easement of right of way since the landowner retains ownership of the property
and can devote the land to farming and other agricultural purposes. Moreover, in the present case,
since the lands are agricultural with no sign of commercial activity, the amount of P450 per square
meter awarded by the trial court as market value of the property is excessive and unreasonable.

ISSUE: Whether the in the case at bar, “taking” is present. >>yes.

RULING: Easement of right of way falls within the purview of the power of eminent domain. In installing
the 230 KV Talisay-Compostela transmission lines which traverse respondent’s lands, a permanent
limitation is imposed by petitioner against the use of the lands for an indefinite period. This deprives
respondent of the normal use of the lands. In fact, not only are the affected areas of the lands
traversed by petitioner’s transmission lines but a portion is used as the site of its transmission
tower. Because of the danger to life and limbs that may be caused beneath the high-tension live wires,
the landowner will not be able to use the lands for farming or any agricultural purposes.

NPC v MARUHOM

FACTS: Lucman G. Ibrahim and his co-heirs are owners of a 70,000-square meter lot in Saduc, Marawi
City. Sometime in 1978, NPC, without respondents’ knowledge and consent, took possession of the
subterranean area of the land and constructed therein underground tunnels. The tunnels were used by
NPC in siphoning the water of Lake Lanao and in the operation of NPC’s projects. Respondents
demanded that NPC pay damages and vacate the subterranean portion of the land, but the demand was
not heeded.

ISSUE: Whether the in the case at bar, “taking” is present. >>yes.


RULING: Indeed, expropriation is not limited to the acquisition of real property with a corresponding
transfer of title or possession. The right-of-way easement resulting in a restriction or limitation on
property rights over the land traversed by transmission lines also falls within the ambit of the
term expropriation. An easement is compensable by the full value of the property despite the fact that
NPC was only after a right-of-way easement, if by such easement it perpetually or indefinitely deprives
the land owner of his proprietary rights by imposing restrictions on the use of the property. It is,
therefore, clear that NPC’s acquisition of an easement of right-of-way on the lands of respondents
amounted to expropriation of the portions of the latter’s property for which they are entitled to a
reasonable and just compensation.

The Office of the Solicitor General petitioner


vs.
Ayala Land Incorporated, Robinson’s Land Corporation, Shangri-La-Plaza Corporation and SM Prime
Holdings, respondents.

Facts: This is a petition for review on certiorari seeking the reversal and setting aside of the decision of
the court of appeals which affirmed the decision of the Makati RTC in two civil cases and the resolution
of the appellate court in the same case which denied the motion for reconsideration filed by the OSG.
Respondents herein are operators of shopping malls in various locations in Metro Manila that have
parking facilities (inside the main buildings, in separate buildings and/or in adjacent lots solely provided
for parking use). The respondents are also the one which maintains the parking spaces and in turn, they
collect parking fees subject to their imposed parking rates. The Senate Committee on Trade and
Commerce and on Justice and Human Rights conducted a joint investigation to inquire on the legality of
the parking fees and to find out the basis and reasonableness of the parking rates. More importantly, to
determine the legality of the policy of the shopping malls denying liability in cases of theft, robbery or
carnapping by invoking the waiver clause at the back of the parking tickets. After the public hearings,
the Senate Committees jointly concluded that the collection parking fee is contrary to the National
Building Code and that the reasonable interpretation of the code is that the parking spaces are for free;
thus, the Committee recommended that the Office of the Solicitor General should institute the
necessary action to enjoin the collection of parking fees as well as to enforce the penal sanctions of the
National Building Code. Two civil cases arise and by being of the same subject matter, the RTC Makati
issued an order to consolidate the cases. The court ruled that the respondents are not obligated to
provide parking spaces that are free of charge, compelling them to do so would be an unlawful taking of
property right without just compensation. The petitioners sought for relief by filing a Motion for
Reconsideration in the Court of Appeals but the appellate court denied the appeal and affirmed the joint
decision by the RTC. Hence, this present petition with a single assignment of error that the Court of
Appeals erred in affirming the ruling of the lower court.

Issue: Whether or not the property right of the respondents can be taken so as to provide free parking
spaces for the general public welfare.

Held: The court affirmed the previous decision that the respondents are not obliged to provide free
parking spaces. There is no pertaining provision in the National Building Code that expressly provides
the same. The law is clear and unequivocal that it needs no further interpretation, it only provides for
measurement requirements of the parking spaces. The OSG cannot rely on their invoked provisions;
they even failed to consider the substantial differences and legal backgrounds on the jurisprudence they
are insisting. Wherefore, the instant petition is hereby DENIED. Previous ruling AFFIRMED. No Costs
NPC v. Tuazon – 653 SCRA 84

RESPONDENTS DEMANDS THAT THEY BE PAID FULL VALUE OF THEIR LAND AS JUST COMPENSATION.
NAPOCOR ARGUES THAT IT SHALL ONLY PAY EASEMENT FEE PURSUANT TO SECTION 3-A(B) ITS
CHARTER, R.A. 6395, WHICH PRESCRIBES A FORMULA FOR EASEMENT FEE. IS NAPOCOR CORRECT?

NO. THE DETERMINATION OF JUST COMPENSATION IS A JUDICIAL FUNCTION. THE FORMULA


PROVIDED IN NAPOCOR’S CHARTER IS NOT BINDING ON THE COURT. IT IS ONLY A GUIDE.

2. Public Use

Sumulong v. Guerrero – 154 SCRA 461

Sumulong v. Guerrero 154 SCRA 461 (1987) F: On December 5, 1977, the National Housing Authority
filed a complaint for the expropriation of 25 hectares of land in Antipolo, Rizal pursuant to PD 1224
authorizing the expropriation of private lands for socialized housing. Among those lands sought to be
expropriated are the petitioners'' lands. They brought this suit in the SC challenging the
constitutionality of PD 1224. HELD: Petitioners contend that socialized housing for the purpose of
condemnation proceedings is not public use since it will benefit only a handful of people. The "public
use" requirement is an evolving concept influences by changing conditions. Urban renewal or
redevelopment and the construction of low-cost housing is recognized as a public purpose, not only
because of the expanded concept of public use but also because of specific provisions in the
Constitution. Shortage in housing is a matter of state concern since it directly and significantly affects
public health, safety, the environment and, in sum, the general welfare. Petitioners claim that there
are vast areas of lands in Rizal hundreds of hectares of which are owned by a few landowners only.
Why should the NHA pick their small lots? Expropriation is not confined to landed estates. The test to
be applied for a valid expropriation of private lands was the area of the land and not the number of
people who stood to be benefitted. The State acting through the NHA is vested with broad discretion
to designate the property. The property owner may not interpose objections merely because in their
judgment some other property would have been more suitable. The provisions on just compensation
found in PD 1224, 1259, and 1313 are the same provisions which were declared unconstitutional in
EPZA v. Dulay (1987) for being encroachments on judicial prerogatives.

Phil. Columbian Assn. v. Hon. Panis – 228 SCRA 668


City of Manila filed before the RTC to expropriate the property which is subject to an ejectment case
wherein the petitioner (is about to win; hence, petitioner will have possession of the property).
Petitioner contends that the expropriation is not proper because it does not comply with public use
requirement. Petitioner argues that under the 1987 Constitution, there must be a law expressly
authorizing local governments to undertake urban land reform

Ruling: Expropriation is proper because the charter of manila expressly provides power to the city to
expropriate
Manosca v. CA – 252 SCRA 412

Facts: The National Historical Institute declared the parcel of land owned by Petitioners as a national
historical landmark, because it was the site of the birth of Felix Manalo, the founder of Iglesia ni Cristo.
The Republic of the Philippines filed an action to appropriate the land. Petitioners argued that the
expropriation was not for a public purpose.

Issue: Whether or Not the taking or exercise of eminent domain may be granted.

Held: Public use should not be restricted to the traditional uses. The taking is for a public use because of
the contribution of Felix Manalo to the culture and history of the Philippines.

Province of Camarines Sur v. CA – 222 SCRA 173


On December 22, 1988, the Sangguniang Panlalawigan of the Province of Camarines Sur passed
Resolution No. 129, Series of 1988, authorizing the Provincial Governor to purchase or expropriate
property contiguous to the provincial capitol site, in order to establish a pilot farm for non-food and
non-traditional agricultural crops and a housing project for provincial government employees.
WON the pilot farm is for public use or purpose
Ruling Yes.
Under the new concept,"public use" means public advantage, convenience or benefit, which tends to
contribute to the general welfare and the prosperity of the whole community, like a resort complex for
tourists or housing project. The expropriation of the property authorized by Resolution 129, Series of
1988, is for a public purpose. The establishment of a pilot development center would inure to the direct
benefit and advantage of the people of the Province of Camarines Sur

Lagcao v. Judge Labra – GR 155746, Oct. 13, 2004 –


The Sangguniang Panlungsod of Cebu City passed a resolution which identified Lot 1029 as a socialized
housing site pursuant to RA 7279. Petitioners filed with the RTC an action for declaration of
nullity of Ordinance No. 1843 for being unconstitutional.

WON the Ordinance No. 1843 is unconstitutional as it sanctions the expropriation of


their property for the purpose of selling it to the squatters, an endeavor contrary to the concept
of “public use” contemplated in the Constitution.
Ruling: We have found nothing in the records indicating that the City of Cebu complied strictly with
Sections 9 and 10 of RA 7279. Ordinance No. 1843 sought to expropriate petitioners’ property without
any attempt to first acquire the lands listed in (a) to (e) of Section 9 of RA 7279. Likewise, Cebu City
failed to establish that the other modes of acquisition in Section 10 of RA 7279 were first exhausted.
Moreover, prior to the passage of Ordinance No. 1843, there was no evidence of a valid and definite
offer to buy petitioners’ property as required by Section 19 of RA 7160.1[20] We therefore find
Ordinance No. 1843 to be constitutionally infirm for being violative of the petitioners’ right to due
process.
Note: RA 7279 is the law that governs the local expropriation of property for purposes of urban land
reform and housing. Sections 9 and 10 thereof provide:
SEC 9. Priorities in the Acquisition of Land. − Lands for socialized housing shall
be acquired in the following order:

(a) Those owned by the Government or any of its subdivisions,


instrumentalities, or agencies, including government-owned or
controlled corporations and their subsidiaries;

(b) Alienable lands of the public domain;

(c) Unregistered or abandoned and idle lands;

(d) Those within the declared Areas or Priority Development, Zonal


Improvement Program sites, and Slum Improvement and Resettlement
Program sites which have not yet been acquired;

(e) Bagong Lipunan Improvement of Sites and Services or BLISS which have
not yet been acquired; and

(f) Privately-owned lands.

Where on-site development is found more practicable and advantageous to the


beneficiaries, the priorities mentioned in this section shall not apply. The local
government units shall give budgetary priority to on-site development of government
lands. (Emphasis supplied).

SEC. 10. Modes of Land Acquisition. − The modes of acquiring lands for
purposes of this Act shall include, among others, community mortgage, land swapping,
land assembly or consolidation, land banking, donation to the Government, joint
venture agreement, negotiated purchase, and expropriation: Provided, however, That
expropriation shall be resorted to only when other modes of acquisition have been
exhausted: Provided further, That where expropriation is resorted to, parcels of land
owned by small property owners shall be exempted for purposes of this Act.

Reyes v. NHA, GR 147511, Jan 20, 2003 –


Facts: National Housing Authority filed several expropriation complaints on the sugarland owned by the
petitioners Reyes. The land is located in Dasmarinas, Cavite. The purpose of the expropriation is for the
expansion of the Dasmarinas Resettlement Project to accommodate the squatters who were relocated
from Manila. The trial court rendered judgment ordering the expropriation of these lots with payment
of just compensation. It was affirmed by the Supreme Court.

The petitioners Reyes alleged the failure of the respondents to comply with the Supreme Court order, so
they filed a complaint for forfeiture of their rights before the RTC of Quezon City. They also said that
NHA did not relocate squatters from Manila on the expropriated lands which violate the reason for
public purpose. The petitioners prayed that NHA be enjoined from disposing and alienating the
expropriated properties and that judgment be rendered forfeiting all its rights and interests under the
expropriation judgment.

In the answer of NHA, they already paid a substantial amount to the petitioners. Thus, several issues are
already raised in the expropriation court.

The trial court dismissed the case. It held that NHA did not abandon the public purpose because the
relocation of squatters involves a long and tedious process. It also entered into a contract with a
developer for the construction of a low-cost housing to be sold to qualified low income beneficiaries.
The payment of just compensation is independent of the obligation of the petitioners to pay capital
gains tax. Lastly, the payment of just compensation is based on the value at the time the property was
taken.

The Court of Appeals affirmed the decision.

Issue: Whether or not the property expropriated is taking for public purpose.

Held: The decision appealed is modified.

The 1987 Constitution explicitly provides for the exercise of the power of eminent domain over the
private properties upon payment of just compensation. Sec. 9, Article III states that private property
shall not be taken for public use without just compensation. The constitutional restraints are public use
and just compensation.

The expropriation judgment declared that NHA has a lawful right to take petitioners properties “for the
public use or purpose of expanding the Dasmarinas Resettlement Project”.

The “public use” is synonymous with “public interest”, “public benefit”, “public welfare”, and “public
convenience”. The act of NHA in entering a contract with a real estate developer for the construction of
low cost housing cannot be taken to mean as a deviation from the stated public purpose of their taking.

Expropriation of private lands for slum clearance and urban development is for a public purpose even if
the developed area is later sold to private homeowners, commercial firms, entertainment and service
companies and other private concerns.

The expropriation of private property for the purpose of socialized housing for the marginalized sector is
in furtherance of the social justice provision under Section 1, Article XIII of the Constitution.

When land has been acquired for public use in fee simple unconditionally, either by the exercise of
eminent domain or by purchase, the former owner retains no rights in the land, and the public use may
be abandoned, or the land may be devoted to a different use, without any impairment of the estate or
title acquired, or any reversion to the former owner

Masikip v. Pasig, 479 SCRA 391


Lourdes Dela Paz Masikip is the registered owner of a parcel of land, which the City of Pasig sought to
expropriate a portion thereof for the “sports development and recreational activities” of the residents
of Barangay Caniogan. This was in January 1994. Masikip refused.

On March 23, 1994, City of Pasig sought again to expropriate said portion of land for the alleged purpose
that it was “in line with the program of the Municipal Government to provide land opportunities to
deserving poor sectors of our community.”

Petitioner protested, so City of Pasig filed with the trial court a complaint for expropriation. The Motion
to Dismiss filed by Masikip was dismissed by the rial court on the ground that there was genuine
necessity to expropriate the property. Case was elevated to the Court of Appeals, which dismissed
petition for lack of merit.

Hence, this petition.

ISSUE:

W/N there was genuine necessity to expropriate the property

HELD:

Eminent domain is “the right of a government to take and appropriate private property to the public
use, whenever the public exigency requires it, which can be done only on condition of providing a
reasonably compensation therefor.” It is the power of the State or its instrumentalities to take private
property for public use and is inseparable from sovereignty and inherent in government.

This power is lodged in the legislative branch of government. It delegates the power thereof to the
LGUs, other public entities and public utility corporations, subject only to constitutional limitations.
LGUs have no inherent power of eminent domain and may exercise it only when expressly authorized by
statute.

Sec. 19, LGC: LGU may, through its chief executive and acting pursuant to an ordinance, exercise the
power of eminent domain for public use, purpose or welfare for the benefit of the poor and landless,
upon payment of just compensation, pursuant to the provisions of the Constitution and pertinent laws.

Provided:

(1) power of eminent domain may not be exercised unless a valid and definite offer has been previously
made to the owner and such offer was not accepted;
(2) LGU may immediately take possession of the property upon the filing of expropriation proceedings
and upon making a deposit with the proper court of at least 15% fair market value of the property based
on the current tax declaration; and
(3) amount to be paid for expropriated property shall be determined by the proper court, based on the
fair market value at the time of the taking of the property

There is already an established sports development and recreational activity center at Rainforest Park in
Pasig City. Evidently, there is no “genuine necessity” to justify the expropriation. The records show that
the Certification issued by the Caniogan Barangay Council which became the basis for the passage of
Ordinance No. 4, authorizing the expropriation, indicates that the intended beneficiary is the Melendres
Compound Homeowner’s Association, a private, non-profit organization, not the residents of Caniogan.
Didipio v. Earth Savers v. Guzon, 485 SCRA 586 – JM
Didipio v. Earth Savers v. Guzon, 485 SCRA 586 – JM

Facts: Didipio petitioned to have the law and the RR to be annulled as it is unconstitutional and it
constitutes unlawful taking of property. In seeking to nullify Rep. Act No. 7942 and its implementing
rules DAO 96-40 as unconstitutional, petitioners set their sight on Section 76 of Rep. Act No. 7942 and
Section 107 of DAO 96-40 which they claim allow the unlawful and unjust “taking” of private property
for private purpose in contradiction with Section 9, Article III of the 1987 Constitution mandating that
private property shall not be taken except for public use and the corresponding payment of just
compensation. They assert that public respondent DENR, through the Mining Act and its Implementing
Rules and Regulations, cannot, on its own, permit entry into a private property and allow taking of land
without payment of just compensation.

Ruling: The SC noted the requisites of eminent domain. They are;


(1) the expropriator must enter a private property;
(2) the entry must be for more than a momentary period.
(3) the entry must be under warrant or color of legal authority;
(4) the property must be devoted to public use or otherwise informally appropriated or injuriously
affected;
(5) the utilization of the property for public use must be in such a way as to oust the owner and deprive
him of beneficial enjoyment of the property.

Further, mining is a public policy and the government can invoke eminent domain to exercise entry,
acquisition and use of private lands.

MINING IS A PUBLIC POLICY AND THE GOVERNMENT CAN INVOKE EMINENT DOMAIN TO EXERCISE
ENTRY, ACQUISITION AND USE OF PRIVATE LANDS.

Barangay v. CA, 581 SCRA 649 – JM

Manapat v. CA, 536 SCRA 32 – JM


Manapat v. CA, 536 SCRA 32 - JM

Facts: President Ferdinand E. Marcos issued Presidential Decree (PD) No. 1072, appropriating P1.2M out
of the President’s Special Operations Funds to cover the additional amount needed for the expropriation
of Grace Park. The National Housing Authority (NHA), PHHC’s successor, then filed several expropriation
proceedings over the already subdivided lots for the purpose of developing Grace Park under the Zonal
Improvement Program (ZIP) and subdividing it into small lots for distribution and resale at a low cost to
the residents of the area. Petitioners in these consolidated cases were among those who purchased
individual subdivided lots of Grace Park directly from RCAM and/or PRC.

Ruling: In a more recent decision, we had occasion to declare that the fact that the property is less than
½-hectare and that only a few would actually benefit from the expropriation does not diminish its public
use character, inasmuch as “public use” now includes the broader notion of indirect public benefit or
advantage, including in particular, urban land reform and housing.
THE FACT THAT ONLY AN ACTUAL FEW WOULD BENEFIT FROM THE EXPROPRIATION DOES NOT
DIMINISH ITS PUBLIC USE CHARACTER. “PUBLIC USE” NOW INCLUDES A NOTION PF INDIRECT
PUBLIC BENEFIT.

Mactan v. Tudtud, GR 174012, November 14, 2008 - JM

Facts: The Republic, represented by the then Civil Aeronautics Administration (CAA), as successor
agency of the NAC, to file a complaint for the expropriation of Lot Nos. 744-A, 745-A, 746, 747, 761-
A,762-A, 763-A, 942, and 947, among others. Soon after the transfer of the aforesaid lots to MCIAA,
Lahug Airport completely ceased operations, Mactan Airport having opened to accommodate incoming
and outgoing commercial flights. On the ground, the expropriated lots were never utilized for the
purpose they were taken as no expansion of Lahug Airport was undertaken. This development
prompted the former lot owners to formally demand from the government that they be allowed to
exercise their promised right to repurchase.

Ruling: When land has been acquired for public use in fee simple, unconditionally, either by the exercise
of eminent domain or by purchase, the former owner retains no rights in the land, and the public use
may be abandoned, or the land may be devoted to a different use, without any impairment of the estate
or title acquired, or any reversion to the former owner.
However, a condemnor should commit to use the property pursuant to the purpose stated in the
petition for expropriation, failing which it should file another petition for the new purpose. If not, then
it behooves the condemnor to return the said property to its private owner, if the latter so desires. The
government cannot plausibly keep the property it expropriated in any manner it pleases and, in the
process, dishonor the judgment of expropriation.

THE CONDEMNOR SHOULD COMMIT TO USE THE PROPERTY PURSUANT TO THE STATED PETITION FOR
APPROPRIATION. THE GOVERNMENT CANNOT PLAUSIBLY KEEP THE EXPROPRIATED
PROPERTY IN ANY MANNER IT PLEASES AND DISHONOR THE JUDGEMENT OF EXPROPRIATION.

City of Manila v. Tan Te, 658 SCRA 88(socialized housing) - JM

Facts: On March 15, 1998, then Manila City Mayor Joselito L. Atienza approved Ordinance No. 7951 – an
expropriation measure enacted on February 3, 1998 by the city council – authorizing him to acquire by
negotiation or expropriation certain pieces of real property along Maria Clara and Governor Forbes
Streets where low-cost housing units could be built and then awarded to bona fide residents therein.
For this purpose, the mayor was also empowered to access the city’s funds or utilize funding facilities of
other government agencies. In the aggregate, the covered property measures 1,425 square meters, and
includes the 475-square-meter lot owned by respondent Melba Tan Te.

Ruling: The term “public use” has acquired a more comprehensive coverage. To the literal import of the
term signifying strict use or employment by the public has been added the broader notion of indirect
public benefit or advantage. The restrictive view of public use may be appropriate for a nation which
circumscribes the scope of government activities and public concerns and which possesses big and
correctly located public lands that obviate the need to take private property for public purposes.
Neither circumstance applies to the Philippines. We have never been a laissez-faire state. And the
necessities which impel the exertion of sovereign power are all too often found in areas of scarce public
land or limited government resources.

Specifically, urban renewal or development and the construction of low-cost


housing are recognized as a public purpose, not only because of the expanded concept
of public use but also because of specific provisions in the Constitution. x x x The 1987
Constitution

LOW COST HOUSING IS CONSIDERED AS PUBLIC USE EVEN ONLY A FEW WILL BENEFIT FROM THE
SAME. THE TERM, “PUBLIC USE” IS BROAD AS TO INCLUDE A NOTION OF INDIRECT PUBLIC BENEFIT OR
ADVANTAGE.

3. Just Compensation

City of Manila v. Estrada – 25 Phil 208 - JM

Facts: The city of Manila sought to expropriate an entire parcel of land with its improvements for use in
connection with a new market at that time being erected in the district of Paco. A complaint was filed
setting forth the necessary allegations, answer joined, and commissioners were appointed, who, after
viewing the premises and receiving evidence, and being unable to agree, submitted two reports to the
court. The court duly rendered its decision, confirming the majority report as to the improvements, but
reducing the price of the land from P20 per square meter, as fixed by the majority report, to P15 per
square meter. Motions for a new trial having been made by both parties and denied by the court, both
parties appealed from that part of the decision fixing the value of the land at P15 per square meter. The
record was therefore elevated to this court for a review of the evidence and assigned errors of the
parties. This court held that P10 per square meter was just compensation for the land, and rendered its
decision accordingly.

The court justifies such action, first, upon the ground that the great preponderance of the evidence
submitted to the commissioners showed that P10 per square meter was just compensation for the land
taken, and, second, upon the power of the court to revise the report of the commissioners when the
amount awarded is grossly inadequate or grossly excessive.

Issue: The question now arises, when may the court, with propriety, overrule the award of the
commissioners in whole or in part and substitute its own valuation of the condemned property?

Ruling: From a mere reading of section 246 and the remarks just made, it should be clear that the court
is permitted to act upon the commissioners' report in one of several ways, at its own discretion. The
whole duty of the court in considering the commissioners' report is to satisfy itself that just
compensation will be made to the defendant by its final judgment in the matter, and in order to fulfill its
duty in this respect the court will be obliged to exercise its discretion in dealing with the report as the
particular circumstances of the case may require.

THE COURTS MAY SUBSTITUTE IT’S OWN VALUATION OVER THE AWARD OF THE COMMISSIONER OF
THE CONDEMNED PROPERTY AT IT’S OWN DISCRETION.

WITH JUST COMPENSATION


Manila Railroad v. Paredes – 31 Phil. 118 – JM

Facts: Sometime prior to the first of April, 1913, the Railroad Company entered upon and took
possession of a long, narrow strip of land, running between the municipalities of Pagbilao and Lopez in
the Province of Tayabas, for use as a roadbed for a railroad which it was engaged in building under its
charter. The Railroad Company claims that it took possession of this strip of land with the consent of the
various owners and occupants claiming title thereto, and with the understanding that it would pay the
owners of all the lands thus taken a price to be agreed upon thereafter, or to be fixed in condemnation
proceedings; and that, not having been able to agree upon a price with the owners of the land, it was
later compelled to institute proceedings for the condemnation of the land thus taken

Ruling: In this jurisdiction the constitutional prohibition against the taking of property without just
compensation contains no express provision requiring pre-payment; and following the weight of
authority, we are of opinion that there is no prohibition against the legislative enactment of a form of
procedure whereby immediate possession of lands involved in expropriation proceedings may be taken,
provided always that due provision is made to secure the prompt adjudication and payment of just
compensation to the owners. We believe that such provision is made for the adjudication and payment
of just compensation to the owners of the lands affected by the condemnation proceedings authorized
under Act No. 1592; and we conclude that these legislative provisions cannot be successfully attacked
on the ground that they contravene the prohibitions against the taking of property without due process
of law or without just compensation.

THE TAKING OF THE PROPERTY IN THIS CASE WAS HELD VALID EVEN THOUGH NO PRE-PAYMENT.
THERE IS NO EXPRESS PROVISION REQUIRING PRE-PAYMENT NOR IS THERE A PROHIBITION AGAINST
THE LEGISLATIVE ENACTMENT OF A FORM OR PROCEDURE WHEREBY IMMEDIATE POSSESSION OF
LANDS INVOLVED IN EXPROPRIATION PROCEEDING.

Santos v. Land Bank – GR 137431, Sept. 7, 2000 – Kaiser


Facts: petitioner Edgardo Santos is the plaintiff in Agrarian Case No. RTC 94-3206 for the determination
of just compensation regarding properties which were taken by DAR under P.D. No. 27 in 1972. On
August 12, 1997. Petitioner filed a motion for the issuance of an alias writ of execution before the
Regional Trial Court, praying that the payment of the compensation be in proportion of P8,629,179.36 in
bonds and P32,499,745 in cash, alleging that the cash portion should include the amounts in the
Decision representing the interest payments. upon appeal by landbank payment was to be made
majority in bonds rather than cash.
Issue: W/N the compensation paid more in bonds is constitutional.
Held: Yes. "It cannot be denied from these cases that the traditional method for the payment of just
compensation is money and no other. And so, conformably, has just compensation been paid in the past
solely in that medium. However, we do not deal here with the traditional exercise of the power of
eminent domain. This is not an ordinary expropriation where only a specific property of relatively
limited area is sought to be taken by the State from its owner for a specific and perhaps local purpose.
What we deal with here is a revolutionary kind of expropriation.
Accepting the theory that payment of the just compensation is not always required to be made fully in
money, we find further that the proportion of cash payment to the other things of value constituting the
total payment, as determined on the basis of the areas of the lands expropriated, is not unduly
oppressive upon the landowner. It is noted that the smaller the land, the bigger the payment in money,
primarily because the small landowner will be needing it more than the big landowners, who can afford
a bigger balance in bonds and other things of value. No less importantly, the government financial
instruments making up the balance of the payment are 'negotiable at any time.' The other modes, which
are likewise available to be landowner at his option, are also not unreasonable because payment is
made in shares of stock, LBP bonds, other properties or assets, tax credits, and other things of value
equivalent to the amount of just compensation.

Municipality of Daet v. CA – 129 SCRA 665


Facts: The Municipality of Daet instituted condemnation proceedings against private respondent Li Seng
Giap & Co. Inc. on August 9, 1962 before the Court of Firs Instance of Camarines Norte for the purpose
of acquiring and subsequently converting the following described property owned by private
respondent as a public park. They eventually started to contest on the right value to be paid to the
respondents.
Issue: W/N the value of the land shall be based on the fmv at the time of the taking
Held: yes. We hold that the decision of the Court of Appeals fixing the market value of the property to
be that obtaining, at least, as of the date of the rendition of the judgment on December 2, 1969 as
prayed by private respondent, which the Court fixed at P200.00 per square meter is in conformity with
doctrinal rulings herein above cited that the value should be fixed as of the time of the taking of the
possession of the property because firstly, at the time judgment was rendered on December 2, 1969,
petitioner had not actually taken possession of the property sought to be expropriated and secondly,
We find the valuation determined by the Court of Appeals to be just, fair and reasonable at the amount
of 200 per sq.m.

NPC v. CA – 129 SCRA 665


Facts: The plaintiffs therein, now private respondents, sought to recover actual and other damages for
the loss of lives and the destruction to property caused by the inundation of the town of Norzagaray,
Bulacan on 26-27 October 1978. The flooding was purportedly caused by the negligent release by the
defendants of water through the spillways of the Angat Dam. Petitioners now are claiming act of God as
a defense so that they would not be liable for the damages that occurred.
Issue: W/N Act of God is a defense
Held: No. Accordingly, petitioners cannot be heard to invoke the act of God or force majeure to escape
liability for the loss or damage sustained by private respondents since they, the petitioners, were guilty
of negligence. The event then was not occasioned exclusively by an act of God or force majeure; a
human factor — negligence or imprudence — had intervened. The effect then of the force majeure in
question may be deemed to have, even if only partly, resulted from the participation of man. Thus, the
whole occurrence was thereby humanized, as it were, and removed from the laws applicable to acts of
God.

EPZA v. Dulay – 149 SCRA 305 - Kaiser

Maddumba v. GSIS – 182 SCRA 281


Facts: Petitioner after winning in a public auction wants to pay the GSIS Gov’t. issued bonds, GSIS is
refusing to accept the bond on its face value.
Issue: W/N the bonds should be priced at its face Value.
Held: Yes. Acceptance of Land Bank bonds, instead of money, undoubtedly involves a certain degree of
sacrifice for the landowner. This, of course, is in addition to the fact that, in case of expropriation of land
covered by land reform, the landowner will seldom get the compensation he desires. Thus, discounting
the Land Banks bonds, and thereby reducing their effective value, entails and imposes an additional
burden on his part. It is, in fact, in consideration of this sacrifice that we extended the rule on liberality
in the interpretation of the provisions of Republic Act No. 3844, then known as the Agricultural Land
Reform Code, in favor not only of the actual tillers but the landowners as well. Ita semper fiat relatio ut
valeat dispositio. The interpretation must always be such that the disposition may prevail.
The preamble of PD 251 eloquently articulates government intent to implement the state policy of
'diverting landlord capital in agriculture to industrial development' by 'mobilization and harnessing
properly all available government resources for the realization of the desired agrarian reform program.'
For agrarian reform cannot be fully realized without the intervention of the government particularly in
the payment of just compensation. Surely, the tenant by himself does not have and cannot afford the
wherewithal to defray the cost of the land transferred to him. It is only with the full support and active
assistance of the government principally through its financial institutions that payment of just
compensation to the landowner may be realized.

Berkenkotter v. CA – 216 SCRA 584


Facts: The sole issue for resolution in this case is the just compensation to be paid for a parcel of land
sought to be expropriated for the use of the Apolinario R. Apacible School of Fisheries, a government
institution, in Nasugbu, Batangas.
Issue: How should the property be valuated.
Held: To determine just compensation, the trial court should first ascertain the market value of the
property, to which should be added the consequential benefits which may arise from the expropriation.
4 If the consequential benefits exceed the consequential damages, these items should be disregarded
altogether as the basic value of the property should be paid in every case.
The market value of the property is the price that may be agreed upon by parties willing but not
compelled to enter into the contract of sale 6 Not unlikely, a buyer desperate to acquire a piece of
property would agree to pay more, and a seller in urgent need of funds would agree to accept less, than
what it is actually worth. The price agreed upon in these cases would not represent the market value of
the property.
Among the factors to be considered in arriving at the fair market value of the property are the cost of
acquisition, the current value of like properties, its actual or potential uses, and in the particular case of
lands, their size, shape, location, and the tax declarations thereon.
It is settled that just compensation is to be ascertained as of the time of the taking, which usually
coincides with the commencement of the expropriation proceedings. Where the institution of the action
precedes entry into the property, the just compensation is to be ascertained as of the time of the filing
of the complaint.
However the court ruled that they only be paid 19.18 per sq.m. the same amount they sold the property
to private individuals on the same area

Meralco v. Pineda – 206 SCRA 196


Facts: For the purpose of constructing a 230 KV Transmission line from Barrio Malaya to Tower 220 at
Pililla, Rizal, the Manila Electric Company (MERALCO) needed portions of the land of Teofilo Arayon,
Sr.,Gil de Guzman, Lucito Santiago and Teresa Bautista (simple fee owners), consisting of an aggregate
area of237,321 square meters. Despite MERALCO's offers to pay compensation and attempts to
negotiate with Arayon, et. al., the parties failed to reach an agreement. On 29 October 1974, a
complaint for eminent domain was filed by MERALCO against 42 defendants (including Teofilo Arayon
Sr., Gil de Guzman, Lucito Santiago, and Teresa Bautista) with the Court of First Instance (now Regional
Trial Court) of Rizal, Branch XXII, Pasig, Metro Manila.
Issue: Whether the court can dispense with the assistance of a Board of Commissioners in an
expropriation proceeding and determine for itself the just compensation
Held: No.an expropriation case where the principal issue is the determination of just compensation, a
trial before the Commissioners is indispensable to allow the parties to present evidence on the issue of
just compensation. The appointment of at least 3 competent persons as commissioners to ascertain just
compensation for the property sought to be taken is a mandatory requirement in expropriation cases.
While it is true that the findings of commissioners may be disregarded and the court may substitute its
own estimate of the value, the latter may only do so for valid reasons, i.e., where the Commissioners
have applied illegal principles to the evidence submitted to them or where they have disregarded a clear
preponderance of evidence, or where the amount allowed is either grossly inadequate or excessive

NPC v. CA – 254 SCRA 577


Facts: A contract was forged between the government through the National Power Corporation
(NAPOCOR) and PECORP, Inc. (PECORP, formerly Pacific Equipment Corporation, as party-CONTRACTOR
on 27 June1974 for the construction of the Mariveles Dam 1 and appurtenant structures of the water
supply system of the Bataan Export Processing Zone at Mariveles, Bataan. It was agreed upon that the
contract is of a "Cost-Plus a Percentage" type — meaning, PECORP will be paid a certain percentage as
fee based on the "Actual Final Cost" of the work, and what constitutes "Actual Final Cost" is the total
cost to NAPOCOR of all the work performed by PECORP which includes cost of materials and supplies,
structures, furnitures, charges, etc. and all other expenses as are inherent in a Cost- Plus and Percentage
Contract and necessary for the prosecution of the work that are approved by NAPOCOR. However other
works in PECORPS contract were sub-contracted to other contractors. PECORP seek for arbritration and
eventually did not proceed with the contract. Roughly 5 years after, PECORP on 14 June 1979 presented
to NAPOCOR 4 claims, i.e. (1) Fee on the cost of drilling and grouting which is 10% of the Actual Final
Cost of P6,962,519.50, or P696.251.95; (2) Fee on the minimum guaranteed equipment rental which is
10% of the Actual Final Cost of P1.67 million, or P167,000.00; (3) Fee on the inventory of unused stocks
and POL, P155,844.95; and (4) Reimbursement of Medical Hospital expenses re: TK-001 Accident case,
or P50,085.93, coupled with a request for arbitration.
Issue: W/N PECORP should be paid.
Held: Yes.as (a) there was actually no withdrawal from the "construction of the work," but only a
transfer of a part of the construction, which is the drilling and grouting work, and (b) said drilling and
grouting still forms part of the project as a mere NAPOCOR-GROGUN sub-contract. Since the NAPOCOR-
GROGUN Contract did not amend nor nullify the "cost plus" provision of the NAPOCOR-Pecorp Contract,
therefore, appellee Pecorp is still entitled to the said 10% fee. Further, the allegation that PECORP
withdrew its claim for fee on the minimum guaranteed equipment rental hours of P167,000.00 is
without merit, as it is clear that withdrawal is only a proposal conditioned upon NAPOCOR's
adjudication, endorsement and approval of all the 3 other claims. However, as the record shows,
NAPOCOR refused to certify for arbitration all the said 3 other claims, hence, the withdrawal was
rendered null and void. These were the findings of the Court of Appeals which were approved by the
Supreme Court.

Land Bank v. CA – 249 SCRA 149; (MR) 258 SCRA 404 - Bryan

FACTS:
Private respondents are landowners whose landholdings were acquired by the DAR and
subjected to transfer schemes to qualified beneficiaries under the Comprehensive Agrarian Reform Law
(CARL, Republic Act No. 6657).
Aggrieved by the alleged lapses of the DAR and the Landbank with respect to the valuation and payment
of compensation for their land pursuant to the provisions of RA 6657, private respondents filed with this
Court a Petition for Certiorari and Mandamus with prayer for preliminary mandatory injunction. Private
respondents questioned the validity of DAR Administrative Order and sought to compel the DAR to
expedite the pending summary administrative proceedings to finally determine the just compensation
of their properties, and the Landbank to deposit in cash and bonds the amounts respectively
"earmarked", "reserved" and "deposited in trust accounts" for private respondents, and to allow them
to withdraw the same.

On October 20, 1994, the respondent court rendered the assailed decision in favor of private
respondents. Petitioners filed a motion for reconsideration but respondent court denied the same.

Petitioners submit that respondent court erred in (1) declaring as null and void DAR Administrative
Order No. 9, Series of 1990, insofar as it provides for the opening of trust accounts in lieu of deposit in
cash or in bonds, and (2) in holding that private respondents are entitled as a matter of right to the
immediate and provisional release of the amounts deposited in trust pending the final resolution of the
cases it has filed for just compensation.

The last major challenge to CARP is that the landowner is divested of his property even before actual
payment to him in full of just compensation, in contravention of a well-accepted principle of eminent
domain.

ISSUE:
WON the “compensation” in the above case is the same as the just compensation contemplated in the
Constitution in the event of expropriation of property.

RULING:
Within the context of the State's inherent power of eminent domain, just compensation means
not only the correct determination of the amount to be paid to the owner of the land but also the
payment of the land within a reasonable time from its taking. Without prompt payment, compensation
cannot be considered "just" for the property owner is made to suffer the consequence of being
immediately deprived of his land while being made to wait for a decade or more before actually
receiving the amount necessary to cope with his loss. Administrative regulations cannot extend the law
and amend a legislative enactment for settled is the rule that administrative regulations must be in
harmony with the provisions of the law.

Panes v. VISCA – 264 SCRA 708 - Bryan

FACTS:
On March 21, 1977, then President Marcos issued Presidential Decree (P.D.) No. 1107 establishing the
Philippine Root Crops Research and Training Center (hereafter, Root Crops Center) in the Visayas State
College of Agriculture (hereafter VISCA) located at Baybay, Leyte. Pursuant to the purposes of the Root
Crops Center, VISCA was authorized under P.D. No. 1107 to acquire by negotiated sale or expropriation,
private agricultural properties in Barrios Pangasugan to the extent of 250 hectares and in Guadalupe,
Baybay, Leyte to the extent of 75 hectares.
Clothed by P.D. No. 1107 with the power to expropriate lands situated within the aforecited barrios,
respondent VISCA filed a complaint for expropriation against petitioners.

On May 15, 1978, petitioners filed their answer to the complaint. They alleged that (1) the lands sought
to be expropriated were not within the area specified under P.D. No. 1107; (2) the amount of
P74,050.00 did not constitute just compensation; (3) P.D. No. 794 providing that the just compensation
shall not be in excess of the current and fair market value declared by the owner or administrator, or
such market value as determined by the provincial assessor, which is lower, was unconstitutional; (4)
likewise unconstitutional was P.D. No. 1107 for impairing the freedom of contract and violating the
equal protection clause; and (5) there was no public necessity for the acquisition by respondent VISCA of
petitioners’ lands. Petitioners also averred, by way of counterclaim, that because of the institution of
the expropriation suit against them, they suffered anguish and anxiety for which they should be
indemnified with damages.

Issue: Whether or not the just compensation can be a executive decision.

RULING:
However, the finding of respondent appellate court insofar as it found that respondent VISCA
has the right to a writ of possession upon compliance with the requirements of P.D. No. 1533 in relation
to P.D. Nos. 1107 and 42, i.e., payment of an amount equivalent to 10% of the amount of compensation
for the property which is, under P.D. 42, the amount equivalent to the assessed value of the subject
property for purposes of taxation, has been rendered ineffectual by our ruling in Export Processing Zone
Authority v. Dulay.

P.D. No. 1533 determines the just compensation in expropriation cases to be the fair and current market
value declared by the owner of the property sought to be expropriated or such market value as
determined by the assessor, whichever is lower. As such, the determination of just compensation, by
virtue of the enactment of P.D. No. 1533, was converted from being a judicial prerogative to an
executive decision. Because the executive determination of just compensation in eminent domain
proceedings renders the courts inutile in a matter which under the Constitution is reserved to them
for final determination, we declared P.D. No. 1533 to be unconstitutional and void in the case of
Export Processing Zone Authority v. Dulay Similar provisions regarding executive determination of just
compensation found in Presidential Decree Nos. 42, 76, 464, 794, 1224, 1259, 1313, and 1517 were
therefore likewise nullified for being unconstitutional.

Republic v. CA – 263 SCRA 758 - Bryan

FACTS:
Private respondent Acil Corporation owned several hectares of Land Linoan, Montevista, Davao
del Norte, which the government took pursuant to the Comprehensive Agrarian Reform Law (R.A. No.
6657)..

Private respondent rejected the government’s offer, pointing out that nearby lands planted to the same
crops were valued at the higher price of P24,717.40 per hectare. The matter was brought before the
Provincial Agrarian Reform Adjudicator (PARAD) who sustained the initial valuation made by the LBP.
On December 12, 1992, private respondent filed a Petition for Just Compensation in the Regional Trial
Court of Tagum, Davao del Norte, sitting as a Special Agrarian Court. Private respondent prayed that
DAR be ordered to pay P24,717.40 per hectare. However, the RTC dismissed its petition on the ground
that private respondent should have appealed to the Department of Agrarian Reform Adjudication
Board (DARAB), pursuant to the latter’s Revised Rules of Procedure, before recourse to it (the RTC)
could be had.

ISSUE: Whether in cases involving claims for just compensation under R.A. No. 6657 an appeal from the
decision of the provincial adjudicator to the DARAB must first be made before a landowner can resort to
the RTC.

RULING:
The DAR is an administrative agency which cannot be granted jurisdiction over cases of eminent domain
and over criminal cases.

Thus, under the law, the Land Bank of the Philippines is charged with the initial responsibility of
determining the value of lands placed under land reform and the compensation to be paid for their
taking.[6] Through notice sent to the landowner pursuant to §16(a) of R.A. No. 6657, the DAR makes an
offer. In case the landowner rejects the offer, a summary administrative proceeding is held[7] and
afterward the provincial (PARAD), the regional (RARAD) or the central (DARAB) adjudicator as the case
may be, depending on the value of the land, fixes the price to be paid for the land. If the landowner
does not agree to the price fixed, he may bring the matter to the RTC acting as Special Agrarian Court.
This in essence is the procedure for the determination of compensation cases under R.A. No. 6657. In
accordance with it, the private respondent’s case was properly brought by it in the RTC, and it was error
for the latter court to have dismissed the case. In the terminology of §57, the RTC, sitting as a Special
Agrarian Court, has “original and exclusive jurisdiction over all petitions for the determination of just
compensation to landowners.”

NPC v. Henson – GR 129998, December 29 1998 - Bryan

Facts:
The National Power Corporation (NPC) originally instituted with the Regional Trial, San
Fernando, Pampanga, a complaint for eminent domain, for the taking for public use of five (5) parcels of
land, owned or claimed by respondents, with a total aggregate area of 58,311 square meters, for the
expansion of the NPC Mexico Sub-Station.

The trial court issued an order appointing three (3) commissioners to aid the court in the
reception of evidence to determine just compensation for the taking of the subject property. After
receiving the evidence and conducting an ocular inspection, the commissioners submitted to the court
their individual reports.

Commisioner Mariano C. Tiglao, in his report dated September 10, 1992, recommended that the
fair market value of the entire 63,220 square meters property be fixed at P350.00 per square meter.
Commissioner Arnold P. Atienza, in his report dated February 24, 1993, recommended that the fair
market value be fixed at P375.00 per square meter. Commissioner Victorino Orocio, in his report dated
April 28, 1993, recommended that the fair market value be fixed at P170.00 per square meter.

However, the trial court did not conduct a hearing on any of the reports.

On May 19, 1993, the trial court rendered judgment fixing the amount of just compensation to
be paid by petitioner for the taking of the entire area of 63,220 square meters at P400.00 per square
meter, with legal interest thereon

The CA affirmed the RTC decision, hence this appeal.

Issue: What is the fair market value of the subject lot as to constitute just compensation?

Ruling:
The parcels of land sought to be expropriated are undeniably idle, undeveloped, raw agricultural
land, bereft of any improvement. Except for the Henson family, all the other respondents were
admittedly farmer beneficiaries under operation land transfer of the Department of Agrarian Reform.
However, the land has been re-classified as residential. The nature and character of the land at the time
of its taking is the principal criterion to determine just compensation to the landowner.

Commissioner Atienza recommended a fair market value at P375.00 per square meter. This
appears to be the closest valuation to the market value of lots in the adjoining fully developed
subdivision. Considering that the subject parcels of land are undeveloped raw land, the price of P375.00
per square meter would appear to the Court as the just compensation for the taking of such raw land.

Santos v. Landbank, GR 137431, Sept. 7, 2000 - Bryan

Facts:
Petitioner Edgardo Santos is the plaintiff in Agrarian Case No. RTC 94-3206 for the
determination of just compensation regarding properties which were taken by the Department of
Agrarian Reform (DAR). The Regional Trial Court, sitting as an Agrarian Court rendered judgment
ordering Defendant Land Bank of the Philippines to pay plaintiff the amount of P45,698,805.34 PESOS, in
the manner provided by The Comprehensive Agrarian Reform Law (RA 6657), by way of full payment of
the said just compensation.
The Land Bank remitted the amount of P948,857.52 to the Clerk of Court on December 24, 1997
and released the amount of P3,621,023.01 in cash and Land Bank Bond in the amount of P41,128,024.81
to the petitioner.
Petitioner filed a motion for the issuance of an alias writ of execution before the Regional Trial
Court, praying that the payment of the compensation be in proportion of P8,629,179.36 in bonds and
P32,499,745 in cash, alleging that the cash portion should include the amounts in the Decision
representing the interest payments.
Petitioner filed a motion to hold the Land Bank in contempt for its refusal to release the balance
of the garnished amount in cash or certified check. Petitioner contends that the bank is estopped from
questioning its alleged undertaking to pay him in cash.
The RTC held that the payment of just compensation must be computed in the manner provided
for in Section 18, Republic Act No. 6657, in which payment was modified to P5,792,084.37 in Cash and
P35,336,840.16 in government instruments or bonds to fully satisfy the Judgment.
Issue: Whether or not just compensation needs to always be made fully in money.

Ruling:
Payment of the just compensation is not always required to be made fully in money, we find
further that the proportion of cash payment to the other things of value constituting the total payment,
as determined on the basis of the areas of the lands expropriated, is not unduly oppressive upon the
landowner. It is noted that the smaller the land, the bigger the payment in money, primarily because the
small landowner will be needing it more than the big landowners, who can afford a bigger balance in
bonds and other things of value.
Petitioner's claim for damages against the bank must likewise be denied because, as already explained,
it was well within its rights in resisting the former's claim.

Sigre v. CA, GR 109568, Aug. 8 2002 - Bryan

Facts:
Presidential Decree No. 27, issued by then Pres. Ferdinand E. Marcos, proclaimed the entire
country as a "land reform area" and decreed the emancipation of tenants from the bondage of the soil,
transferring to them the ownership of the land they till. To achieve its purpose, the decree laid down a
system for the purchase by tenant-farmers, long recognized as the backbone of the economy, of the
lands they were tilling. Owners of rice and corn lands that exceeded the minimum retention area were
bound to sell their lands to qualified farmers at liberal terms and subject to conditions.

In a not-so-novel attempt to challenge the long-settled constitutionality of Presidential Decree


No. 27, private respondent Lilia Y. Gonzales, as co-administratrix of the Estate of Matias Yusay, filed with
the Court of Appeals on September 15, 1992, a petition for prohibition and mandamus docketed as CA-
G.R. SP No. 28906, seeking to prohibit the Land Bank of the Philippines (LBP) from accepting the
leasehold rentals from Ernesto Sigre (predecessor of petitioner Rolando Sigre), and for LBP to turn over
to private respondent the rentals previously remitted to it by Sigre. It appears that Ernesto Sigre was
private respondent's tenant in an irrigated rice land located in Barangay Naga, Pototan, Iloilo. He was
previously paying private respondent a lease rental of sixteen (16) cavans per crop or thirty-two (32)
cavans per agricultural year. In the agricultural year of 1991-1992, Sigre stopped paying his rentals to
private respondent and instead, remitted it to the LBP pursuant to the Department of Agrarian Reform's
Memorandum Circular No. 6, Series of 1978, which set the guidelines in the payment of lease
rental/partial payment by farmer-beneficiaries under the land transfer program of P.D. No. 27.

Issue: Whether or not PD 27, setting forth the value of land for purposes of just compensation, is
constitutional.

Ruling:
The objection that P.D. 27 is unconstitutional as it sets limitations on the judicial prerogative of
determining just compensation is bereft of merit. P.D. 27 provides:

"For the purpose of determining the cost of the land to be transferred to the tenant-farmer
pursuant to this Decree, the value of the land shall be equivalent to two and one half (2 ½) times the
average harvest of three normal crop years immediately preceding the promulgation of this Decree;"
The determination of just compensation under P.D. No. 27, like in Section 16 (d) of R.A. 6657 or
the CARP Law, is not final or conclusive.
Clearly therefrom, unless both the landowner and the tenant-farmer accept the valuation of the
property by the Barrio Committee on Land Production and the DAR, the parties may bring the dispute to
court in order to determine the appropriate amount of compensation, a task unmistakably within the
prerogative of the court.

NHA v. Heirs of Isidro, GR 154411, June 19 2001 - Bryan

Facts:
• On February 23, 1999, petitioner National Housing Authority filed with the Regional Trial Court
of Cebu City, Branch 11, an Amended Complaint for eminent domain against Associacion
Benevola de Cebu, Engracia Urot and the Heirs of Isidro Guivelondo for the purpose of the
public use of Socialized housing.
• On April 17, 2000, the Commissioners submitted their report wherein they recommended that the
just compensation of the subject properties be fixed at P11,200.00 per square meter wherein a
partial judgment has been rendered.
• After the report on the just compensation has completed, both parties filed an MR on the amount for
the just compensation stating that it has no adequate basis and support. Both MR was denied by
the court.
• While the judgment has been rendered in the RTC and an entry of judgment and the motion for
execution has been issued, NHA filed a petition for certiorari to the Court of Appeals. The CA
denied the petition on the ground that the Partial Judgment and Omnibus Order became final
and executory when petitioner failed to appeal the same.
Wherefore, the Petitioner NHA filed an appeal to the Supreme Court.

Issue: Whether or not the intended public use was rendered nugatory by the unreasonable just
compensation fixed by the court.
Ruling:
The public purpose of the socialized housing project is not in any way diminished by the amount
of just compensation that the court has fixed. The need to provide decent housing to the urban poor
dwellers in the locality was not lost by the mere fact that the land cost more than petitioner had
expected. It is worthy to note that petitioner pursued its petition for certiorari with the Court of
Appeals assailing the amount of just compensation and its petition for review with this Court which
eloquently indicates that there still exists a public use for the housing project. It was only after its
appeal and petitions for review were dismissed that petitioner made a complete turn-around and
decided it did not want the property anymore.

Respondent landowners had already been prejudiced by the expropriation case. Petitioner
cannot be permitted to institute condemnation proceedings against respondents only to abandon it
later when it finds the amount of just compensation unacceptable.

Mactan v. Urgello – 520 SCRA 515 - Bryan

Facts:
Respondent, Milagros Urgello, was the owner of Lot No. 913-E of the Banilad Estate in Cebu City.
Sometime in the 1950s, the then Civil Aeronautics Administration (CAA) filed a complaint before
the then Court of First Instance of Cebu to expropriate Lot No. 913-E-3 for the projected expansion of
the Lahug Airport.

The case reached the Court of Appeals in which, on joint motion of the parties, a judgment
based on a compromise agreement was rendered. In that agreement, the CAA agreed to purchase Lot
No. 913-E-3 for P3,105.00, subject to the resolutory condition that in the event that the Republic of the
Philippines would no longer use it as an airport, its title or ownership would revert to respondent or her
heirs upon reimbursement of the purchase price of P3,105.00.

It appears that on April 27, 1966, the Mactan Airport commenced its operations and the
Philippine Airlines stopped using the Lahug Airport. Filipinas Airways and Air Manila ceased too to use
the Lahug Airport at the end of 1966 and thereafter used the Mactan Airport.

On August 2, 1983, the Bureau of Air Transportation (BAT) by which the CAA was later known,
and the Bureau of Equipment of the then Ministry of Public Works and Highways (MPWH), entered into
a Memorandum of Agreement whereby the BAT was to lease several parcels of land, including Lot No.
913-E-3, to MPWH for 25 years to be used as the site of the latter’s Seventh Regional Base Shop
Complex.

The MPWH soon started building fences along the perimeters of Lot No. 913-E-3.

After the dismantling of the hangars and taxiways from the Lahug Airport and putting up of a
repair shop of the Bureau of Equipment of the MPWH, the BAT erected a fence, over the objection of
respondent, enclosing portions of her Lot Nos. 913-E-2 and 913-E-4.

Issue: Whether or not there was undue taking of private property.

Ruling:
As shown in the records, the aforementioned lots were never included in the expropriation
proceedings over Lot No. 913-E-3. These lots, without the respondent’s consent, were unilaterally taken
by the government when BAT fenced the same. Such an act violates the constitutional mandate that
"[Private] property shall [not] be taken for public use without just compensation" (Section 9, Art. III,
Constitution). Likewise, it is deprivation of property without due process of law. [Is it] not that justice
and equity demand that what is not yours should be returned to its rightful owner.

Contrary to petitioners DPWH’s and ATO’s undertakings in the January 17, 1990 Compromise
Agreement, they failed to reconvey Lot No. 913-E-3 to respondent despite her return of the purchase
price therefor. Such failure amounts to expropriation without just compensation.

San Roque v. Republic – 532 SCRA 493 – Rhino

FACTS: At the outset, an expropriation proceeding was began over parcels of land forming part of Lot
933. Just compensation was allegedly paid, however, the vouchers were lost during the war and the
title of the subject parcel of land was not transferred to the government. Eventually, the land was
subdivided and new titles were issued by the Register of Deeds of Cebu which were acquired by
defendant-appellee. Plaintiff-appellant filed the present case alleging that it is the owner of the subject
parcels of land by virtue of the expropriation. Defendant-appellee alleged that the expropriation was
never consummated because the government did not pay any compensation.

ISSUE: Whether there was a valid expropriation.

RULING: NO! By the very admission of the Republic, there was no record of payment of compensation
to the land owners. In Republic v. Lim, we emphasized that no piece of land can be finally and
irrevocably taken from an unwilling owner until compensation is paid. Without full payment of just
compensation, there can be no transfer of title from the landowner to the expropriator. Time and
again, we have declared that eminent domain cases are to be strictly construed against the
expropriator. The payment of just compensation for private property taken for public use is an
indispensable requisite for the exercise of the State’s sovereign power of eminent
domain. Failure to observe this requirement renders the expropriation ineffectual,
notwithstanding the avowed public purpose. To disregard this limitation on the exercise of
governmental power to expropriate is to ride roughshod over private rights.

4. Judicial Review

De Knecht v. Bautista – 100 SCRA 660 G.R. No. L-51078 October 30, 198 – Rhino

FACTS: The plan to extend EDSA to Roxas Boulevard to be ultimately linked to the Cavite Coastal Road
Project, originally called for the expropriation of properties along Cuneta Avenue in Pasay City. Later on,
however, the Ministry of Public Highways decided to make the proposed extension pass through
Fernando Rein and Del Pan Streets. Because of the protests of residents of the latter, the Commission on
Human Settlements recommended the reversion to the original plan, but the Ministry argued the new
route will save the government P2 million. The government filed expropriation proceedings against the
owners of Fernando Rein and Del Pan streets, among whom was petitioner De Knecht. Petitioner De
Knecht filed a motion to dismiss on the ground that the court had no jurisdiction over the subject matter
of the action because the complaint failed to allege that the instant project for expropriation bore the
approval of the Ministry of Human Settlements and the Metro Manila Government nor pursuant to
laws. She alleges that the choice of properties to be expropriated made by the Ministry of Public
Highways was arbitrary and erroneous. The respondent judge issued a writ of possession.

ISSUE #1: Whether the court has jurisdiction.

RULING: YES! With due recognition then of the power of Congress to designate the particular property
to be taken and how much thereof may be condemned in the exercise of the power of expropriation, it
is still a judicial question whether in the exercise of such competence, the party adversely affected is the
victim of partiality and prejudice. That the equal protection clause will not allow. The failure to meet
tile exacting standard of due process would likewise constitute a valid objection to the exercise of this
congressional power. It is recognized, however, that the government may not capriciously or
arbitrarily choose what private property should be taken.
For the purpose of obtaining a judicial declaration of nullity, it is enough if the respondents or
defendants named be the government officials who would give operation and effect to official action
allegedly tainted with unconstitutionality. Thus, where the statute assailed was sought to be enforced
by the Land Tenure Administrative and the Solicitor General, the two officials may be made respondents
in the action without need of including the Executive Secretary as a party in the action.

ISSUE #2: Whether there is a genuine need to expropriate the properties owned by De Knecht and
others similarly situated on the ground that the choice of properties to be expropriated seemed
arbitrarily made by the DPWH.

RULING: NO! The choice of Fernando Rein and Del Pan Streets is arbitrary and should not receive
judicial approval. The Human Settlements Commission concluded that the cost factor is so minimal
(P269,196 and not P2M) that it can be disregarded in making a choice between the two lines. The factor
of functionality strongly militates against the choice of Fernando Rein and Del Pan Streets (2nd line)
because it is longer by 3 meters and less straight than Cuneta Avenue (1st line), while the factor of social
and economic impact bears grievously on the residents of Cuneta Avenue as there are more residents.
While the issue would seem to boil down to a choice between people, on one hand, and progress and
development, on the other, it is to be remembered that progress and development are carried out for
the benefit of the people.

Manotoc v. NHA – 150 SCRA 89 - Rhino

FACTS: Petitioners challenge the constitutionality of P.D. 1669 and 1670, pertaining to the expropriation
of the Tambunting Estate and the Estero de Sunog-Apog respectively. The questioned properties were
owned by herein petitioners.

The dispute started when the National Government, through its Zonal Improvement Program (ZIP),
included the said properties as part of the properties to be expropriated by the Government so that the
ZIP will push through. Petitioners contend that the expropriation, by virtue of the P.D., was null and void
because it deprived them of their constitutional right to due process and equal protection of the laws.
Furthermore, Not only are the owners given absolutely no opportunity to contest the expropriation,
plead their side, or question the amount of payments fixed by decree, but the decisions, rulings, orders,
or resolutions of the NHA are expressly declared as beyond the reach of judicial review. An appeal may
be made to the Office of the President but the courts are completely enjoined from any inquiry or
participation whatsoever in the expropriation of the subdivision or its incidents.

ISSUE: Whether the power of expropriation exercised by the NHA may be expressly declared as beyond
the reach of judicial review.

RULING: NO! That is tantamount to depriving one person to his constitutional right of due process.
“Although due process does not always necessarily demand that a proceeding be had before a court of
law, it still mandates some form of proceeding wherein notice and reasonable opportunity to be heard
are given to the owner to protect his property rights. We agree with the public respondents that there
are exceptional situations when, in the exercise of the power of eminent domain, the requirement of
due process may not necessarily entail judicial process. But where it is alleged that in the taking of a
person's property, his right to due process of law has been violated, the courts will have to step in and
probe into such an alleged violation.”

In the exercise of its sovereign right the State is not subject to any limitation other than those
imposed by the Constitution which are: first, the taking must be for a public use; secondly, the
payment of just compensation must be made: and thirdly, due process must be observed in the
taking.

Republic v. De Knecht – 182 SCRA 141 - Rhino

FACTS: De Knecht was one of the owners of several properties along the Fernando Rein-Del Pan streets
which the Government sought to expropriate to give way to the extension of EDSA and the construction
of drainage facilities. De Knecht filed a case to restrain the Government from proceeding with the
expropriation. Her prayer was denied by the lower court but upon certiorari, the SC reversed the lower
court decision and granted the relief asked for by De Knecht ruling that the expropriation was arbitrary.
The case was remanded to the lower court. No further action was taken despite the SC decision until
two years later, in 1983, when the Government moved for the dismissal of the case on the ground that
the Legislature has since enacted BP 340 expropriating the same properties for the same purpose. The
lower court denied the motion.

ISSUE: Whether an expropriation proceeding that was determined by a final judgment of this Court may
be the subject of a subsequent legislation for expropriation.

RULING: YES! While it is true that said final judgment of this Court on the subject becomes the law of
the case between the parties, it is equally true that the right of petitioner to take private properties for
public use upon payment of just compensation is so provided in the Constitution and the laws.

Such expropriation proceeding may be undertaken by the petitioner not only by voluntary negotiation
with the land owners but also by taking appropriate court action or by legislation. When BP 340 was
passed, it appears that it was based on supervening events that occurred after the 1980 decision of the
SC on the De Knecht case was rendered. The social impact factor which persuaded the Court to consider
this extension to be arbitrary had disappeared. Only private respondent remains as the solitary obstacle
to this project that will solve not only the drainage and flood control problem but also minimize the
traffic bottleneck in the area. Moreover, the said decision is no obstacle to the legislative arm of the
Government in thereafter making its own independent assessment of the circumstances then prevailing
as to the propriety of undertaking the expropriation of properties in question and thereafter by enacting
the corresponding legislation as it did in this case. The Court agrees in the wisdom and necessity of
enacting BP 340. B.P. Blg. 340 therefore effectively superseded the aforesaid final and executory
decision of this Court.

Militante v. CA, GR 107040, April 12, 2000 - 72 – Rhino


FACTS: Petitioner Militante is the registered owner of three (3) contiguous parcels of land where
twenty-four (24) squatter families. Petitioner wrote the Human Settlements Regulatory Commission
(HSRC) seeking a declaration of non-coverage from the Urban Land Reform Program of the government
pursuant to PD 1315 which provides automatic expropriation and for the NHA to immediately take
possession, control and disposition of the expropriated properties with the power of demolition of their
improvements. A certificate declaring petitioner's lots "outside the declared Urban Land Reform Zone"
was issued. With this certificate, petitioner asked the NHA to relocate the squatters on his land. For
refusal by the squatters to enter into a dialogue, the NHA issued a demolition clearance which was not
executed. Four years later, the NHA subjected petitioner’s lot to expropriation but petitioner’s offer is
too expensive. Petitioner requested for a revalidation of his demolition clearance and relocation of the
squatters. Respondent Annabelle Carangdang, NHA Project Manager in Bagong Barrio, refused to
implement the clearance to eject the squatters on petitioner's land. Petitioner then filed with the
respondent Court of Appeals a "Petition for Prohibition and Mandamuswith Declaration as Inexistent
and Unconstitutional Presidential Decree No. 1315" against the NHA and Carangdang. In a decision
dated April 24, 1992, the respondent Court of Appeals dismissed the petition and held that petitioner
failed to overcome the presumption of the decree's constitutionality.

ISSUE: Whether PD 1315 providing for automatic expropriation is unconstitutional.

RULING: NO! The unconstitutionality of PD 1315 cannot be entertained for the real objective of the
petitioner was compel the NHA to eject the squatters in his property. Otherwise, it "might spawn legal
and social ramifications which cannot just be lightly ignored." In Alfonso v. Pasay City as authority for
the view that "if property is taken by the government without the benefit of expropriation proceedings
and is devoted to public use, such as a road, after many years, the property owner may no longer bring
an action for recovery of his land, but may simply demand payment of just compensation for his land."
A careful reading of the Alfonso case, however, will show that this Court did not rule that the only
remedy of an aggrieved landowner in such a situation is to "simply demand payment of just
compensation." To be sure, this Court contemplated the remedy of restoring possession to the
aggrieved landowner. If it did not order the remedy, it was only because it was no longer feasible as the
lot involved had already been converted to a road. In the case at bar, the landowners concerned may
not opt for the right to be paid just compensation. The process is not an easy one and may take years
especially in light of the budget difficulties of the government. We take judicial notice of the fact that
the current budget deficit of the government amounts to P8.9 billion.

Section 10. No law impairing the obligation of contracts shall be passed.

Clemens v. Nolting, 42 Phil 702, 1922 G.R. No. L-17959 January 24, 1922 - Rhino

FACTS: That plaintiff is a citizen of the United States, temporarily residing in the city of Manila,
Philippine Islands. That defendant, William T. Nolting, is the duly appointed, qualified and acting Auditor
of the Government of the Philippine Islands. The Secretary of War, through the Bureau of Insular
Affairs, employed plaintiff on behalf of the Government of the Philippine Islands. The chief accountant
of the Bureau of Public Works of the Government of the Philippine Islands tendered plaintiff a warrant
on the Treasurer of the Philippine Islands in the sum of P666.66, Philippine currency, in full payment of
his salary for the month of January, 1921 but received the same under protest claiming that per contract
he should be paid $333.33 in United States currency, or a sum in Philippine currency of P739.99. After
repeated demands, he was issued a warrant for the difference of P73.33. He now collects the same
insisting that he be paid in United States money or in Philippine pesos of an equivalent commercial value
as per contract. The respondent contends that under Act No. 2776 (“the Philippine silver peso shall be a
legal tender for all debts, public and private”), a debt of the Government, payable in "dollars," may be
paid in Philippine currency at the rate of two to one even though the debt grew out of a special contract
which provided that the same should be paid in "dollars."

ISSUE: Whether Act No. 2776 is valid.

RULING: NO! A contract to pay a certain sum in money, without any stipulation as to the kind of money
in which it shall be paid, may always be satisfied by payment of that sum in any currency which is lawful
money at the place and time at which payment is to be made. That is the general rule, under both the
common and the civil law. But when the contract stipulates the specie or kind or character of money for
the performance of the contract, it must be satisfied in the medium of payment mentioned in the
contract. An interference with the terms of a legal contract by legislation is unwarranted and illegal. A
contract is not fulfilled by the delivery of one thing which is different from the thing the contract
provides for.

While the respondent contends, under the laws in force in the Philippine Islands, that a debt of the
Government payable in dollars may be paid in Philippine currency at the rate of two to one, he
overlooks the fact that section 1613 makes the Philippine silver peso and the gold coins of the United
States at the rate of one dollar for two pesos, a legal tender in the Philippine Islands for all debts, public
and private, and not the Philippine paper peso. If the Government can discharge a contract, payable in
dollars, by tendering Philippine paper pesos, then merchants and others who contract debts payable
expressly in dollars may also discharge their debts in a like manner. If such doctrine should be
announced, then no manufacturer or person would take the risk of contracting obligations here for
future payments.

Home Building and Loan Assn. v. Blaisdell – 290 US 398 - Rhino

FACTS: In the midst of the Great Depression, Minnesota passed a law declaring an emergency and
saying that during the emergency period courts could extend the time periods in which mortgagers
could pay back their debts to their lenders. Pursuant to the statute, Blaisdell’s period of redemption was
extended, unquestionably modifying the lender’s contractual rights of foreclosure. The constitutionality
of the law was brought into question.

ISSUE: Did the Minnesota law violate the provision of the United States Constitution which prohibits a
state from impairing the obligations of contracts?

RULING: NO! The law was a legitimate use of its police powers since Minnesota faced massive
economic difficulties. The Contract Clause was adopted by the Framers during the distressed economic
times following the Revolutionary War. It was established to counter the “ignoble array of legislative
schemes” of State legislatures which were designed to defeat the obligations of debtors owed to
creditors by interfering with contractual arrangements. The Framers feared that if private contracts
were not respected, the destruction of credit would result, and credit was essential to the prosperity of
the Nation’s economy. Therefore, the Contract Clause was not intended to confer absolute rights onto
private parties entering into contracts. Rather, it was intended to serve as qualified right, over which
the State retained some power to control, for the benefits of the Nation as a whole. Thus, the
question is not whether legislative action affects contracts, but instead whether legislative action is
reasonably appropriate to the achievement of a legitimate end. This is the test under the Contract
Clause in this case, the legislation was addressed to a legitimate end – an emergency existed in
Minnesota. Plus, the conditions upon which the period of redemption under the contract was extended
do not appear unreasonable. A State action that impairs a private contract is not valid unless it is
reasonably related to the achievement of a legitimate State end.

Rutter v. Esteban – 93 Phil. 68 – Rhino

FACTS: Rutter sold to Esteban two parcels of land situated in Manila for P9,600 of which P4,800 were
paid outright, and the balance was made payable as follows: P2,400 on or before August 7, 1942, and
P2,400 on or before August 27, 1943, with interest at the rate of 7 percent per annum. To secure
the payment of said balance of P4,800, a first mortgage has been constituted in favor of the plaintiff.
Esteban failed to pay the two installments as agreed upon, as well as the interest that had accrued and
so Rutter instituted an action to recover the balance due, the interest due and the attorney's fees.
The complaint also contains a prayer for sale of the properties mortgaged in accordance with law.
Esteban claims that this is a prewar obligation contracted and that he is a war sufferer, having filed his
claim with the Philippine War Damage Commission for the losses he had suffered as a consequence of
the last war; and that under section 2 of RA 342 (moratorium law), payment of his obligation cannot be
enforced until after the lapse of eight years. The complaint was dismissed. A MR was made which assails
the constitutionality of RA 342.

ISSUE: Whether RA 342 is unconstitutional on non-impairment clause grounds.

RULING: YES! The moratorium is postponement of fulfillment of obligations decreed by the state
through the medium of the courts or the legislature. Its essence is the application of police power. The
economic interests of the State may justify the exercise of its continuing and dominant protective power
notwithstanding interference with contracts. The question is not whether the legislative action
affects contracts incidentally, or directly or indirectly, but whether the legislation is addressed to a
legitimate end and the measures taken are reasonable and appropriate to that end.

However based on the President’s general SONA that economy has stabilized and consistent with what
the Court believes to be as the only course dictated by justice, fairness and righteousness, declared that
the continued operation and enforcement of RA 342 at the present time is unreasonable and
oppressive, and should not be prolonged and should be declared null and void and without effect. This
holds true as regards Executive Orders Nos. 25 and 32, with greater force and reason considering that
said Orders contain no limitation whatsoever in point of time as regards the suspension of the
enforcement and effectivity of monetary obligations.

Del Rosario v. De los Santos – L-20589-90 – Prince


Del Rosario v. De los Santos – L-20589-90 – Prince
 Constitutionality of Section 14 of the Agricultural Tenancy Act of 1955, which empowers a
tenant "to change the tenancy contract from one of share tenancy to the leasehold tenancy and
vice versa and from one crop-sharing arrangement to another of the share tenancy.
 Court of Agrarian Relations decided in favor of the tenants.
ISSUE: Whether lease contracts may be intervened.
HELD: Obligations of Contracts yields to Police Power as long as the end are not arbitrary or
oppressive and the exercise of vested rights is not contrary to Public Policy.

ooooooooooooooooooooooooooooooooooooooooooo
Abella v. NLRC – 152 SCRA 140 – Prince
L-71813, July 20, 1987

 Hacienda owner vs. agricultural workers.


 Tenant employed its own workers.
 Tenant and Hacienda owner terminated leasehold contract.
 Tenant fired the workers but workers continued to till on the land
 Illegal dismissal and backwages suit against hacienda owner before labor arbiter. Arbiter and
NLRC found the dismissal valid but awarded separation pay.
ISSUE: Whether validly terminated contracts may be affected by a law.
HELD: POLICE POWERS MAY AFFECT OBLIGATIONS OF VALIDLY TERMINATED CONTRACTS because:::
LABOR SOCIAL LEGISLATIONS.

OOOOOOOOOOOO
Phil. Vet. Bank Employees v. Phil. Vet. Bank – 189 SCRA 14 – Prince
G.R. No. 67125, August 24, 1990
6. PVB is in financial difficulties.
7. PVB is placed under the Receivership of the Central Bank Monetary Board.
8. PVB, in order to reduce losses, planned for retrenchment and reorganization.
9. The Employee union questioned the retrenchment and reorganization before the NLRC since
their security of tenure are affected.
10. Meanwhile, the Monetary Board ordered the liquidation the assets of the PVB.
11. Veterans Federation of the Philippines intervened and seek to prevent the liquidation. VFP
prepared some rehabilitation of the Bank.
12. Ancillary petition to pay wages as ordered by the NLRC. Meanwhile, depositors and stockholders
seeks to recover their interests.
13. The Employee's Union and stockholders reasoned that the liquidation is improper since it
impairs the PVB charter as a contract between the PVB stockholders and the Government.
Issue: Whether the liquidation was proper.
Held: PROPER LIQUIDATION::
3. Banking System is imbued with public interest.
4. Contracts are protected when by the Constitutional guaranty only when they do not affect
public interest and social order.
5. CB bank is duty bound to liquidate the remaining assets of the bank.

Presley v. Bel-Air Village Association – 201 SCRA 13 - Prince


Presley v. Bel-Air Village Association – 201 SCRA 13 – Prince
G.R. No. 86774, August 21, 1994
5. Almendras bought a house and lot, located at Jupiter Street, with Bel-Air.
6. TCT's annotated a Deed of Restriction (DOR) that says that the house and lot are exclusively for
residential purpose.
7. A City Zoning Ordinance converted the street from residential to commercial zone.
8. Almendrase leased the house and lot to Presley's parents who made a pan-de-sal bakery.
9. Bel-Air sent a letter of desistance to Almendras.
10. Almendras contends that there was no written contract between him and Bel-Air and that as
soon as Bel-Air stops providing basic sanitation and security services, he stops payment.
ISSUE: Whether the Zoning Ordinance can impair the DOR.
HELD: DOR IS CONTRARY TO LAW AND PUBLIC ORDER.
5. Deed of Restrictions are in the nature of contractual obligations.
6. However, these contractual stipulations on the use of the land even if said conditions are
annotated on the torrens title can be impaired if necessary to reconcile with the legitimate
exercise of police power. (Ortigas & Co. Limited Partnership v. Feati Bank and Trust Co., 94 SCRA
533 [1979]).

Tolentino v. Sec. of Finance – 235 SCRA 630 - Prince


Tolentino v. Sec. of Finance – 235 SCRA 630 – Prince
G.R. Nos. 115455 and others, August 25, 1994

1. Constitutionality of RA No. 7716, EVAT Law.


2. Petitioners raise the issue that the passage of the law shall impair obligation of contracts.
Accordingly, the application of the tax to existing contracts of the sale of real property by
installment or on deferred payment basis would result in substantial increases in the monthly
amortizations to be paid because of the 10% VAT.The additional amount, it is pointed out, is
something that the buyer did not anticipate at the time he entered into the contract
ISSUE: Whether RA 7716 impairs obligations of contracts.
HELD: STATE CAN INTERFERE INTO CONTRACTS WHEN CONTRARY TO LAW, PUBLIC POLICY OR ORDER.
Even though such taxation may affect particular contracts, as it may increase the debt of one person and
lessen the security of another, or may impose additional burdens upon one class and release the
burdens of another, still the tax must be paid unless prohibited by the Constitution, nor can it be said
that it impairs the obligation of any existing contract in its true legal sense. The policy of protecting
contracts against impairment presupposes the maintenance of a government which retains adequate
authority to secure the peace and good order of society. Indeed not only existing laws but also "the
reservation of the essential attributes of sovereignty, is to read into contracts as a postulate of the legal
order." Contracts must be understood as having been made in reference to the possible exercise of the
rightful authority of the government and no obligation of contract can extend to the defeat of that
authority.

Siska Development v. Office of the President – 231 SCRA 674 - Prince


Siska Development v. Office of the President – 231 SCRA 674 – Prince
G.R. No. 93176, April 22, 1994
1. Sering spouses delayed payment with Siska Development. Hence, Siska sent notice of rescission,
although not stipulated, of the contract to sell but Siska still accepted the delayed payments.
2. Sering spouses sent their delayed payments because they did not receive the notice of
rescission on time. Sering spouses sent another payment but Siska did not accept because of the
rescission.
3. Sering spouse filed for an action for specific performance before the CFI. But CFI dismissed the
case because jurisdiction falls under the National Housing Authority (NHA). NHA denied specific
performance but ordered Siska to refund the Sering spouses.
4. On appeal, the Office of the President, pursuant to the Maceda law, ordered Siska to deliver the
property to the Sering Spouses upon full payment.
5. Maceda law requires that the buyer receives the notice of rescission for cancellation to effect.
Moreover, the cancellation includes full refund to the buyer.
Issue: Whether the Maceda Law impairs obligations of contracts.
HELD: MACEDA LAW MERELY PROVIDES REMEDIES; IT DID NOT CHANGE TERMS AND CONDITIONS OF
CONTRACTS. THUS, IT DOES NOT IMPAIR CONTRACTS.
Even before the effectivity of the Maceda Law, jurisprudence made necessary a notice of rescission. The
requirement of notice of the rescission under the Maceda Law does not change the time or mode of
performance or impose new conditions or dispense with the stipulations regarding the binding effect of
the contract. Neither does it withdraw the remedy for its enforcement. At most, it merely provides for a
procedure in aid of the remedy of rescission.

Miners Association v. Factoran – 240 SCRA 100 - Prince


Miners Association v. Factoran – 240 SCRA 100 – Prince
G.R. No. 98332, January 16, 1995

 EO No. 211- Pursuant to the 1987 Constitution, it prescribes the interim procedures in the
processing and approval of applications for the exploration, development and utilization of
minerals order ensuring the continuity of mining operations and activities.
 EO No. 279 that authorizes the Secretary of the DENR to negotiate and enter into, for and in
behalf of the Government.
 Secretary of the DENR in turn promulgated AO Nos. 57: "Guidelines of Mineral Production
Sharing Agreement."Art 9 says that all existing mining leases or agreements which were granted
after the effectivity of the 1987 Constitution pursuant to EO No. 211, except small scale mining
leases and those pertaining to sand and gravel and quarry resources covering an area of 20
hectares or less, shall be converted into production-sharing agreements within one year from
the effectivity.
 Sec of DENR issued AO No. 82: Enumerates the persons or entities required to submit Letter of
Intent and Mineral Production Sharing Agreement within two years from the effectivity of the
previous order or until July 17, 1991. Failure to do so within the prescribed period shall cause
the abandonment of mining, quarry and sand and gravel claims.
 Miner's Association contends that both orders violate the non-impairment of contract clause on
the ground that the first unduly pre-terminates existing mining agreements and automatically
converts them into production-sharing agreements within a year, and the other declares that
failure to submit LOIs and MPAs within two years shall cause the abandonment of their mining,
quarry and sand gravel permits.
ISSUE: Whether the AOs impaired mining contracts.
HELD: CONTRACTUAL RIGHTS YIELD TO LAW, PUBLIC POLICY, WELFARE OR ORDER; MINERAL LANDS
SERVES PUBLIC WELFARE OR RIGHTS. THUS, MINERAL LANDS IS NOT COVERED UNDER THE NON-
IMPAIRMENT OF CONTRACT CLAUSE.
Non-impairment of contracts clause are restricted to contracts which respect property, or some object
or value, and confer rights which may be asserted in a court of justice, and have no application to
statute relating to public subjects within the domain of the general legislative powers of the State, and
involving the public rights and public welfare of the entire community affected by it. They do not
prevent a proper exercise by the State of its police powers. By enacting regulations reasonably
necessary to secure the health, safety, morals, comfort, or general welfare of the community, even the
contracts may thereby be affected; for such matter can not be placed by contract beyond the power of
the State shall regulates and control them.

Juarez v. CA – 214 SCRA 475 – Prince


Juarez v. CA – 214 SCRA 475 - Prince
G.R. No. 93474, October 7, 1992
 Lessor Araneta leased the land to Ocampo. Then, Ocampo built the family home on the
Araneta's land. Ocampo left the house and live with her daughter but she leased the house to
Roberto. Now, Juarez, another daughter, possessed the house. She likewise leased the house to
Roberto.
 Araneta sold the land to Susanna Realty and Susanna sold the same land to Cetus Development.
Under BP 877, Cetus filed and ejectment suit before the MTC against Juarez for subleasing the
land without its consent.
 MTC dismissed the suit because the lease was made before the effectivity of BP 877. RTC
affirmed the MTC but CA reversed the RTC. CA reasoned that the lease was terminated before
the effectivity of BP 877 and was renewed within the effectivity of the law.
 Juarez argued that the law is not retroactive. If retroactive, then it impairs contracts.

Issue: Whether BP 877 impairs the lease contract.


HELD: HOUSING IS A PUBLIC INTEREST; LESSEE ABUSED HER RIGHTS TO THE PREJUDICE OF THE
LESSOR; THE STATE SERVES PUBLIC INTEREST AND PROTECTS THE RIGHTS TO ALL PARTIES IN A
CONTRACT. THUS, THE LAW MAY INTERFERE CONTRACTS.PETITION DENIED.
As long as the contract affects the public welfare one way or another requiring the interference of the
State, then the police power prevails over the impairment clause.
The purpose of the law in interfering with such contracts is to protect both the landlord and the tenant
from their mutual impositions that can only cause detriment to society as a whole. The practice such as
above discussed have to be the subject of government regulation and even prohibition and cannot seek
legitimate refuge in the impairment clause.

FPIB v. CA – 252 SCRA 259

FACTS: Petitioner Bank was under a conservator placed by the Central Bank of the Philippines during the
time that the negotiation and perfection of the contract of sale took place. Petitioners energetically
contended that the conservator has the power to revoke or overrule actions of the management or the
board of directors of a bank, under Section 28-A of Republic Act No. 265.

ISSUE: Whether the power granted to the conservator in the law in effect allows impairment of
contracts

RULING: No. While admittedly, the Central Bank law gives vast and far-reaching powers to the
conservator of a bank, it must be pointed out that such powers must be related to the "(preservation of)
the assets of the bank, (the reorganization of) the management thereof and (the restoration of) its
viability." Such powers, enormous and extensive as they are, cannot extend to the post-
facto repudiation of perfected transactions, otherwise they would infringe against the non-impairment
clause of the Constitution . If the legislature itself cannot revoke an existing valid contract, how can it
delegate such non-existent powers to the conservator under Section 28-A of said law?

Obviously, therefore, Section 28-A merely gives the conservator power to revoke contracts that are,
under existing law, deemed to be defective — i.e., void, voidable, unenforceable or rescissible. Hence,
the conservator merely takes the place of a bank's board of directors. Ineluctably, his power is not
unilateral and he cannot simply repudiate valid obligations of the Bank. His authority would be only to
bring court actions to assail such contracts — as he has already done so in the instant case. A contrary
understanding of the law would simply not be permitted by the Constitution. Neither by common sense.
To rule otherwise would be to enable a failing bank to become solvent, at the expense of third parties,
by simply getting the conservator to unilaterally revoke all previous dealings which had one way or
another or come to be considered unfavorable to the Bank, yielding nothing to perfected contractual
rights nor vested interests of the third parties who had dealt with the Bank.

CMMA v. POEA – 243 SCRA 666

FACTS: Petitioner Conference of Maritime Manning Agencies, Inc., urge us to annul Resolution No. 01,
of the Governing Board of the Philippine Overseas Employment Administration (POEA) and POEA
Memorandum Circular No. 05 on the ground that:

The resolution and the memorandum circular are unconstitutional because they violate non-impairment
of obligation of contracts clauses of the Constitution.

Memorandum Circular No. 05, informed them .that Governing Board Resolution No. 01 adjusted the
rates of compensation and other benefits, and that: VI. Upon effectivity, the new compensation and
other benefits shall apply to any Filipino seafarer already on-board any vessel provided, that the case of
action occurs after the said compensation and benefits take effect;

ISSUE: Whether or not POEA Memorandum Circular No. 05, is violative of the non impaitrment of
contracts clause of the constitution

RULING: There is no merit; in the claim that the resolution and memorandum circular violate the
contract clause of the Bill of Rights. The constitutional prohibition against impairing contractual
obligations is not absolute and is not to be read with literal exactness. It does not prevent a proper
exercise by the State of its police power by enacting regulations reasonably necessary to secure the
health, safety, morals; comfort, or general welfare of the community, even though contracts may
thereby be affected, for such matters cannot be placed by contract beyond the power of the State to
regulate and control them.

Verily, the freedom to contract is not absolute; all contracts and all rights are subject to the police
power of the State . The challenged resolution and memorandum circular being valid implementations
of E.O. No. 797, which was enacted under the police power of the State, they cannot be struck down on
the ground that they violate the contract clause. To hold otherwise is to alter long-established
constitutional doctrine and to subordinate the police power to the contract clause.
PNB v. O.P. – 252 SCRA 5
PHILIPPINE NATIONAL BANK, vs. OFFICE OF THE PRESIDENT,
FACTS: Private respondents were buyers on installment of subdivision lots from Marikina Village, Inc.
Notwithstanding the land purchase agreements, the subdivision developer mortgaged the lots in favor
of the petitioner, Philippine National Bank. Private respondents duly complied with their obligations as
lot buyers and constructed their houses on the lots in question.

The Subdivision developer defaulted and PNB foreclosed on the mortgage. As highest bidder at the
foreclosure sale, the bank became owner of the lots.

Acting on suits brought by private respondents, the HLURB Office of Appeals, Adjudication and Legal
Affairs (OAALA) ruled that PNB - without prejudice to seeking relief against Marikina Village, Inc. - may
collect from private respondents only the "remaining amortizations, in accordance with the land
purchase agreements they had previously entered into with" Marikina Village, Inc., and cannot compel
private respondents to pay all over again for the lots they had already bought from said subdivision
developer. The Housing and Land Use Regulatory Board affirmed this decision which was concurred by
the Office of the President.

ISSUE: Whether or not The Office of the President erred in applying P.D. 957 because said law was
enacted only on July 12, 1976, while the subject mortgage was executed on December 18, 1975;

RULING: It is obvious and indubitable that P.D. 957 was intended to cover even those real estate
mortgages, like the one at issue here, executed prior to its enactment, and such intent must be given
effect if the laudable purpose of protecting innocent purchasers is to be achieve:

While P.D. 957 did not expressly provide for retroactivity in its entirety, yet the same can be plainly
inferred from the unmistakable intent of the law to protect innocent lot buyers from scheming
subdivision developers. As between these small lot buyers and the gigantic financial institutions which
the developers deal with, it is obvious that the law - as an instrument of social justice - must favors the
weak.

The petitioner complains that the retroactive application of the law would violate the impairment
clause. The argument does not impress. The impairment clause is now no longer inviolate; As long as the
contract affects the public welfare one way or another so as to require the interference of the State,
then must the police power be asserted, and prevail, over the clause.

Despite the impairment clause, a contract valid at the time of its execution may be legally modified or
even completely invalidated by a subsequent law. If the law is a proper exercise of the police power, it
will prevail over the contract.

Eugenio v. Drilon – 252 SCRA 106


FLORENCIO EUGENIO, vs EXECUTIVE SECRETARY FRANKLIN M. DRILON,

FACTS: Private respondent purchased on installment basis from petitioner two lots .Acting on
complaints for non-development, the National Housing Authority rendered a resolution inter
aliaordering petitioner to cease and desist from making further sales of lots in said village or in any
project owned by him.

Private respondent filed with the Office of Appeals, Adjudication and Legal Affairs (OAALA) of the
Human Settlements Regulatory Commission (HSRC), a complaint alleging that, in view of the above NHA
resolution, he suspended payment of his amortizations. OAALA rendered a decision upholding the right
of petitioner to cancel the contract with private respondent and dismissed private respondent's
complaint.

On appeal, the Commission Proper of the HSRC reversed the OAALA and, applying P.D. 957, ordered
petitioner to complete the subdivision development and to reinstate private respondent's purchase
contract over one lot, and as to the other, ordered the management of E & S Delta Village to
immediately refund to the herein private respondent all payments made thereon, plus interests
computed at legal rates from date of receipt hereof until fully paid."

Petitioner avers that inasmuch as the land purchase agreements were entered into in 1972, prior to the
effectivity of P.D. 957 in 1976, said law cannot govern the transaction.

ISSUE: Whether or not the Executive Secretary erred in applying P.D. 957 and in concluding that the
non-development of the E & S Delta Village justified private respondent's non-payment of his
amortizations.

RULING: We hold otherwise, and herewith rule that respondent Executive Secretary did not abuse his
discretion, and that P.D. 957 is to be given retroactive effect so as to cover even those contracts
executed prior to its enactment in 1976.

P.D. 957 did not expressly provide for retroactivity in its entirety, but such can be plainly inferred from
the unmistakable intent of the law. P.D. 957 was enacted with no other end in view than to provide a
protective mantle over helpless citizens who may fall prey to the manipulations and machinations of
"unscrupulous subdivision and condominium sellers", and such intent is nowhere expressed more
clearly than in its preamble

From a dedicated reading of the preamble, it is manifest and unarguable that the legislative intent must
have been to remedy the alarming situation by having P.D. 957 operate retrospectively even upon
contracts already in existence at the time of its enactment.

Meralco v. Province of Laguna – 306 SCRA 750

Facts: Republic Act No. 7160, otherwise known as the “Local Government Code of 1991,” was enacted to
take effect on 01 January 1992 directing local government units to create their own sources of revenue
and to levy taxes, fees and charges, subject to the limitations expressed therein, consistent with the
basic policy of local autonomy. Pursuant to this respondent province enacted Laguna Provincial
Ordinance No. 01-92, which ordered the levying of franchise tax.

On the basis of the above ordinance, respondent Provincial Treasurer sent a demand letter to
MERALCO for the corresponding tax payment of 50% of gross annual receipts. MERALCO contended that
the imposition of a franchise tax under Provincial Ordinance No. 01-92, insofar, contravened the
provisions of Section 1 of P.D. 551 which provided that “the franchise tax shall be two per cent (2%) of
their gross receipts”

Issue: Whether or not the local government code as well as the resolution of the Province of Laguna
which levied franchise tax against Meralco contravenes sec. 10 of article III which states that no law
impairing the obligations of contracts shall be passed

Ruling: The Supreme Court held that although tax exemptions contained in special franchises as being in
the nature of contracts and a part of the inducement for carrying on the franchise, these exemptions,
nevertheless, are far from being strictly contractual in nature. Contractual tax exemptions, in the real
sense of the term and where the non-impairment clause of the Constitution can rightly be invoked,
are those agreed to by the taxing authority in contracts, such as those contained in government bonds
or debentures, lawfully entered into by them under enabling laws in which the government, acting in
its private capacity, sheds its cloak of authority and waives its governmental immunity.

These contractual tax exemptions, however, are not to be confused with tax exemptions
granted under franchises. A franchise partakes the nature of a grant which is beyond the purview of the
non-impairment clause of the Constitution. The 1987 Constitution, is explicit that no franchise for the
operation of a public utility shall be granted except under the condition that such privilege shall be
subject to amendment, alteration or repeal by Congress as and when the common good so requires.

Lim v. Pacquing – 240 SCRA 649 – Val


FACTS: The Municipal Board of Manila nonetheless passed Ordinance No. 7065 entitled “An Ordinance
Authorizing the Mayor To Allow And Permit The Associated Development Corporation To Establish,
Maintain And Operate A Jai-Alai In The City Of Manila, Under Certain Terms And Conditions And For
Other Purposes.” Later, Presidential Decree No. 771 was issued “Revoking All Powers and Authority of
Local Government(s) To Grant Franchise, License or Permit And Regulate Wagers Or Betting By The
Public On Horse And Dog Races, Jai-Alai Or Basque Pelota, And Other Forms Of Gambling”,

Section 3 thereof, expressly revoked all existing franchises and permits issued by local governments.

Associated Development Corporation (ADC) was granted a valid and legal franchise by Ordinance No.
7065 to operate the jai-alai. and assails the constitutionality of P.D. No. 771.

ISSUE: Whether PD 771 violated the non-impairment clause


RULING: No. It should be remembered that a franchise is not in the strict sense a simple contract but
rather it is more importantly, a mere privilege specially in matters which are within the government's
power to regulate and even prohibit through the exercise of the police power. Thus, a gambling
franchise is always subject to the exercise of police power for the public welfare.
By their very nature, franchise are subject to amendment, alteration or revocation by the State
whenever appropriate. Under the exercise of its police power, the State through its requirement for
permits, licenses and franchises to operate, undertakes to regulate what would otherwise be an illegal
activity punished by existing penal laws.

Ortigas v. Feati Bank – 94 SCRA 533 – Val


Facts:
Plaintiff is engaged in real estate business . Plaintiff upon sale to defendant of Lots no. 5 and 6 imposed
building restrictions in their contract and limited it to residential buildings only. Defendant maintains
that , per ResolutionNo.27 of the Municipal Council of Mandaluyong, the area along the western part of
EDSA from Shaw Boulevard to the Pasig River, has been declared a commercial and industrial zone.
Defendant-appellee began construction of a building devoted to banking purposes but which it claims
could also be used exclusively for residential purposes. The following day, the plaintiff demanded in
writing that the construction of the commercial building be stopped but the defendant refused to
comply contending that the construction was in accordance with the zoning regulations.

Issue: Whether the said Resolution can nullify or supersede the contractual obligations assumed by
defendant-appellee.

Held:Yes. While non-impairment of contacts is constitutionally guaranteed, the rule is not absolute since
it has to be reconciled with the legitimate exercise of police power.
Invariably described as the “most essential, insistent and illimitable of powers” and the
“greatest and most powerful attribute of government”, the exercise of police power may be judicially
inquired into and corrected only if it is capricious, whimsical, unjust or unreasonable, there having been
a denial of due process or a violation of any other applicable constitutional guarantee.
Resolution No.27, S-1960 declaring the western part of EDSA from Shaw Boulevard to the Pasig
River as an industrial or commercial zone was passed in the exercise of police power to
safeguard/promote the health, safety, peace, good order and general welfare of the people in the
locality. Judicial notice may be taken of the conditions prevailing in the area, especially where Lots Nos.
5 and 6 are located. EDSA supports an endless stream of traffic and the resulting activity, noise and
pollution which are hardly conducive to the health, safety or welfare of the residents in its route.

Juarez v. CA – 214 SCRA 475 – Mikee


Facts:
Angela Ocampo subleased the house she was renting to Robert Capuchino. The house was later sold to
Cetus Development Corporation.
Immediately after acquiring the lot, Cetus filed a complaint for ejectment against the petitioner with the
Municipal Court of Manila on the ground that the lessee had subleased the property without its consent
in violation of BP 877. The case was dismissed by the court held that the state was inapplicable because
the sublease was made prior to its effectivity. The RTC also affirmed such rule. The CA however reversed
the ruling.
Issue:
Whether or not BP 877 which was applied retrospectively, violated the impairment clause.
Held:
NO, it did not violate the impairment clause. The impairment clause is now no longer inviolate, as long
as the contract affects the public welfare one way or another so as to require the interference of the
State, then must the police power be asserted, and prevail, over the impairment clause. This time, the
law would operate retroactively , to affect existing contracts of sub-lease concluded before the law
came into effect. It ruled that BP 877 was applicable because the original contract of lease did not
specify a fixed term and payment of the rental was made on a monthly basis. The contract was deemed
terminated from month to month. Hence, when it was renewed in July 1985, it became subject to BP
877, which had come into effect on June 12, 1985
FPIB v. CA – 252 SCRA 259 – Mikee
FACTS:
• The original plaintiffs Demetria and Janolo negotiated with the defendant Producer Bank of the
Philippines (now First Philippine International Bank) to purchase the bank’s six parcels of land .After the
meeting, plaintiff Janolo, following the advice of defendant Rivera, made a formal purchase offer to the
bank which was also accepted by it.
Then petitioner Bank was under a conservator placed by the Central Bank of the Philippines during the
time that the negotiation and perfection of the contract of sale took place. Petitioners energetically
contended that the conservator has the power to revoke or overrule actions of the management or the
board of directors of a bank, under Section 28-A of Republic Act No. 265 (otherwise known as the
Central Bank Act). Thus, the bank did not honor the contract.
ISSUE:
Whether the bank conservator has the unilateral power to repudiate the authority of the bank officers
and/or to revoke the contract.
HELD:
•the powers granted to the conservator of a bank, enormous and extensive as they are, cannot extend
to the post-facto repudiation of perfected transactions, otherwise they would infringe against the
non-impairment clause of the Constitution.
NO. The Central Bank law gives vast and far-reaching powers to the conservator of a bank suffering
liquidity, it must be pointed out that such powers must be related to the "preservation of the assets of
the bank, the reorganization of the management thereof and the restoration of its viability."
• Such powers, enormous and extensive as they are, cannot extend to the post-facto repudiation
of perfected transactions, otherwise they would infringe against the non-impairment clause of the
Constitution. If the legislature itself cannot revoke an existing valid contract, much more can it delegate
such non-existent powers to the conservator under Section 28-A of said law.
• Obviously, therefore, Section 28-A merely gives the conservator power to revoke contracts that
are, under existing law, deemed to be defective — i.e., void, voidable, unenforceable or rescissible.
Hence, the conservator merely takes the place of a bank's board of directors. What the said board
cannot do — such as repudiating a contract validly entered into under the doctrine of implied authority
— the conservator cannot do either. Ineluctably, his power is not unilateral and he cannot simply
repudiate valid obligations of the Bank
CMMA v. POEA – 243 SCRA 106 – Mikee
FACTS:
• EO 797 provided for the creation of POEA (Philippine Overseas Employment Administration)
which was mandated to protect the rights of overseas Filipino workers to fair and equitable employment
practices . Pursuant to the objectives set forth in EO 797, the Governing Board of POEA issued the
assailed Resolution 1 which stipulates:
• In case of death of the seaman during the term of his Contract, the employer shall pay his
beneficiaries the Philippine Currency equivalent to the amount of US$50,000 and an additional amount
of US$7,000 to each child under the age of twenty-one (21) but not exceeding four children at the
exchange rate prevailing during the time of payment. Petitioner, an incorporated association of licensed
Filipino manning agencies, which hire and recruit Filipino seamen for and in behalf of their respective
foreign shipowner-principals, seeks the nullification of the said Resolution on the ground that it amounts
to discrimination against foreign shipowners and principals employing Filipino seamen thereby giving
undue advantage to foreign employers whose covered overseas Filipino workers are not seamen.
Petitioners also contest undue delegation of legislative powers and the impairment of contracts.
ISSUE: Whether EO 797 and Resolution No. 1 violate the non-impairment clause.
HELD:
• NO.
• The constitutional prohibition against impairing contractual obligations is not absolute and is not
to be read with literal exactness. It is restricted to contracts with respect to property or some object of
value and which confer rights that may be asserted in a court of justice; it has no application to statutes
relating to public subjects within the domain of the general legislative powers of the State and involving
the public rights and public welfare of the entire community affected by it.
• It does not prevent a proper exercise by the State of its police power by enacting regulations
reasonably necessary to secure the health, safety, morals; comfort, or general welfare of the
community, even though contracts may thereby be affected, for such matters cannot be placed by
contract beyond the power of the State to regulate and control them.
JMM v. CA – (supra) – Mikee
With the introduction of the Entertainment Industry Advisory Council, a plan was presented to POEA to
screen and train performing artists seeking to go abroad. In pursuant to the proposal POEA and the
secretary of DOLE sought a 4 step plan to realize the plan which included an Artist’s Record Book which
a performing artist must acquire prior to being deployed abroad. The Federation of Talent Managers of
the Philippines assailed the validity of the said regulation as it violated the right to travel, abridge
existing contracts and rights and deprives artists of their individual rights. JMM intervened to bolster the
cause of FETMOP. The lower court ruled in favor of EIAC.
Issue: is the regulation valid?
Yes, it is a futile gesture on the part of petitioners to invoke the non-impairment clause of the
Constitution to support their argument that the government cannot enact the assailed regulatory
measures because they abridge the freedom to contract. In Philippine Association of Service Exporters,
Inc. vs. Drilon, we held that "[t]he non-impairment clause of the Constitution... must yield to the loftier
purposes targeted by the government." 15 Equally important, into every contract is read provisions of
existing law, and always, a reservation of the police power for so long as the agreement deals with a
subject impressed with the public welfare.

PNB v. OP – 252 SCRA 5 – Mikee


FACTS:
• Private respondents were buyers on installment of subdivision lots from Marikina Village, Inc.
• Private respondents duly complied with their obligations as lot buyers and constructed their
houses on the lots in question.
• Notwithstanding the land purchase agreements it executed over said lots, the subdivision
developer mortgaged the lots in favor of the petitioner, Philippine National Bank.
• The Subdivision developer defaulted and PNB foreclosed on the mortgage. As highest bidder at
the foreclosure sale, the bank became owner of the lots.
• Acting on suits brought by private respondents, the Housing and Land Use Regulatory Board
(HLURB) Office of Appeals, Adjudication and Legal Affairs (OAALA) ruled that PNB - without prejudice to
seeking relief against Marikina Village, Inc. - may collect from private respondents only the "remaining
amortizations, in accordance with the land purchase agreements they had previously entered into with"
Marikina Village, Inc., and cannot compel private respondents to pay all over again for the lots they had
already bought from said subdivision developer.
• The HLURB affirmed this decision which was concurred by the Office of the President invoking
P.D. 957 “The Subdivision and Condominium Buyers’ Protective Decree,” which obliges petitioner Bank
to accept the payment of the remaining unpaid amortizations tendered by private respondents.
• Petitioner averred that P.D. 957 is not applicable because it was enacted after the execution of
the mortgage.
ISSUE: Whether the retroactive application of P.D. 957 would violate the non-impairment clause.
HELD:
• NO.
• While P.D. 957 did not expressly provide for retroactivity in its entirety, yet the same can be
plainly inferred from the unmistakable intent of the law to protect innocent lot buyers from scheming
subdivision developers. As between these small lot buyers and the gigantic financial institutions which
the developers deal with, it is obvious that the law - as an instrument of social justice - must favor the
weak. Truly, this Court cannot allow the injustice that will be wrought by a strictly prospective
application of the law.
The petitioner complains that the retroactive application of the law would violate the non-impairment
clause. The argument does not impress. The impairment clause is now no longer inviolate; As long as the
contract affects the public welfare one way or another so as to require the interference of the State,
then must the police power be asserted, and prevail, over the clause.
Eugenio v. Drilon – 252 SCRA 106 - Mikee
Private Respondent purchased on installment b
asis from Petitioner, two lots.Private respondent suspended payment of his amortizations because of
non-development on the property. Petitioner then sold one of the two lots to spousesRelevo and the
title was registered under their name. Respondent prayed forannulment of sale and reconveyance of
the lot to him. Applying P.D. 957 “TheSubdivision and Condominium Buyers’ Protective Decree”, the
Human SettlementsRegulatory Commission ordered Petitioner to complete the development,
reinstatePrivate Respondent’s purchase contract over one lot and immediately refund him ofthe
payment (including interest) he made for the lot sold to the spouses. Petitionerclaims that the Exec. Sec.
erred in applying P.D. 957 saying it should have not beengiven retroactive effect and that non-
development does not justify the non-paymentof the amortizations.
ISSUE:
W/N the Executive Secretary acted with grave abuse of discretion when he
decided P.D. 957 will be given retroactive effect.
HELD:
No. Respondent Executive Secretary did not act with grave abuse ofdiscretion and P.D. 957 is to given
retroactive effect so as to cover even thosecontracts executed prior to its enactment in 1976. P.D. 957
did not expressly providefor retroactivity in its entirety, but such can be plainly inferred from the
unmistakableintent of the law. “The intent of the statute is the law.”
JMM v. CA – (supra Substantive) – Mikee—already above

C & M Timber v. Alcala – 273 SCRA 402 – Czar


F: Filipinas Loggers Devt Corp. (FLDC) requested a timber concession over the same area covered by
petitioner’s TLA No. 106, alleging that the same had been cancelled pursuant to a presidential directive
banning all forms of logging in the area. The request was approved but later on the TLA of FLDC, on the
ground that in spite of the suspension order, said concessionaire had continued logging operations in
violation of forestry rules and regulations. C&M Timber seeks to nullify the order and resolution of the
Office of the President, declaring as of no force and effect Timber License Agreement No. 106 issued to
Petitioner. TLA No. 106 with expiry date June 30 1997, covering 67,680 hectares of forest land in the
municipalities of Dipaculao and Dinalongan in the Province of Aurora and the Municipality of Maddela in
Quirino province.

I: Whether the cancellation of TLAs constitutes impairment of the obligation of its contracts.

H: As to petitioners contention that the cancellation of its license constitutes an impairment of the
obligation of its contract, suffice it for us to quote what we held in Felipe Ysmael & Co Inc v. Deputy
Executive Secretary Timber licenses, permits and license agreements are the principal instruments by
which the State regulates the utilization and disposition of forest resources to the end that public
welfare is promoted. And it can hardly be gainsaid that they merely evidence a privilege granted by the
State to qualified entities and do not vest in the latter a permanent or irrevocable right to the particular
concession area and the forest products therein. They may be validly amended, modified, replaced, or
rescinded by the Chief Executive when national interests of the due process of law clause.

DOCTRINE: Timber licenses, permits and license agreements are the principal instruments by which the
State regulates the utilization and disposition of forest resources to the end that public welfare is
promoted – they are not deemed contracts within the purview of the due process of law clause.

Republic v. Agana – 269 SCRA 1 – Czar


F: Respondents filed a complaint anchored on private respondent’s alleged rights to collect dividends
under the preferred shares in question and to have petitioner redeem the same under the terms and
conditions of the stock certificates. Petitioner filed a motion to dismiss the complaint on the ground that
the trial court has no jurisdiction over the subject matter and that the action was unenforceable under
substantive law. Trial copurt denied the Motion to Dismiss and rendered a decision in favor of private
respondents. Hence this petition arguing that the respondent judge erred in saying that said directive
which prohibits the respondent from redeeming any preferred shares on the ground that the said
redemption would reduce the assets of the Bank to the prejudice of its depositors and creditors.
Respondent Judge insists that directive constitutes an impairment of obligations of contracts.

I: Whether or not erred in rendering that said directive constitutes an impairment in the impairment of
obligations of contracts?

H: The redemption of said shares cannot be allowed. Redemption of preferred shares was prohibited
for just and valid reason. The directive issued by the Central Bank Governor was obviously meant to
preserve the status quo and to prevent the financial adverse repercussions, not only to its depositors
and creditors, but also to the banking industry as a whole. The directive, in limiting the exercise of right
granted by law to a corporate entity, may thus be considered as an exercise of police power. The
respondent judge insists that the directive constitutes an impairment of obligations of contracts. It has,
however, been settled that the Constitutional guaranty of non-impairment of obligations of contracts is
limited by the exercise of the police power of the state, the reason being that public welfare is superior
to private rights.

Producers v. NLRC – GR 118069, November 16, 1998 - Czar


F: Petitioner was placed by the Central Bank of the Philippines under a conservator for the purpose of
protecting its assets. Private respondents sought the implementation of the CBA regarding the
retirement plan. Deadlock continued for at least 6 months when the private respondent, to resolve the
issue, decided to file a case against the petitioner for unfair labor practice and flagrant violation of the
CBA provisions. Local Arbiter dismissed private respondents complaint on the premise that considering
that the Bank is under conservatorship, the latter is under no compulsion to implement the resolutions
issued by the LMRC. On appeal, the NLRC reversed the findings and ordered the petitioner to implement
the provisions of the CBA.

I: Whether or not Conservator may validly revoke the CBA entered into between the Bank and the Union
employees
H: Obviously, therefore, Section 28-A merely gives the conservator power to revoke contracts that are,
under existing law, deemed to be defective - i.e., void, voidable, unenforceable or rescissible. Hence,
the conservator merely takes the place of a bank's board of directors. What the said board cannot do -
such as repudiating a contract validly entered into under the doctrine of implied authority - the
conservator cannot do either. Ineluctably, his power is not unilateral and he cannot simply repudiate
valid obligations of the Bank. His authority would be only to bring court actions to assail such contracts -
as he has already done so in the instant case. A contrary understanding of the law would simply not be
permitted by the Constitution. Neither by common sense. To rule otherwise would be to enable a
failing bank to become solvent, at the expense of third parties, by simply getting the conservator to
unilaterally revoke all previous dealings which had one way or another come to be considered
unfavorable to the Bank, yielding nothing to perfected contractual rights nor vested interests of the
third parties who had dealt with the Bank."

It bears repeating that apart from the non-impairment clause, what is also well-settled, to the point of
being trite, is the principle that when the conflicting interests of labor and capital are weighed on the
scales of social justice, the dominant influence of the latter must be counter-balanced by the sympathy
and compassion the law must accord the under-privileged worker.[6]

Blaquera v. Alcala – GR109406, September 11, 1998 – Czar


F: Petitioners theorize that AO 29 and AO 268 violate EO 292 and since the latter is a law, it prevails over
executive issuances. Petitioners likewise assert that AO 29 and AO 268encroach upon the constitutional
authority of the Civil Service Commission to adopt measures to strengthen the merit and rewards
system and to promulgate rules, regulations and standards governing the incentive awards system of
the civil service. Issued by the then President Corazon Aquino ("President Aquino") in the exercise of her
legislative powers under the 1987 Constitution, EO 292, or the Administrative Code of 1987, provided
for the following incentive award system:"

Sec. 31. Career and Personnel Development Plans.— Each department or agency shall prepare a career
and personnel development plan which shall be integrated into a national plan by the Commission. Such
career and personnel development plans which shall include provisions on merit promotions,
performance evaluation, in-service training, including overseas and local scholarships and training
grants, job rotation, suggestions and incentive award systems, and such other provisions for employees'
health, welfare, counseling, recreation and similar services.

Sec. 35. Employee Suggestions and Incentive Award System. — There shall be established a government-
wide employee suggestions and incentive awards system which shall be administered under such rules,
regulations, and standards as maybe promulgated by the Commission.

On February 21, 1992, President Aquino issued AO 268 which granted "each official and employee of the
government the productivity incentive benefits in a maximum amount equivalent to thirty percent
(30%) of his one (1) month basic salary but in no case shall such amount be less than two thousand
pesos (P2,000.00)," 29 for those who have rendered at least one year of service as of December 31,
1991. 30 Said AO carried the prohibition, provided in Section 7 thereof, which reads:"

SEC. 7. The productivity incentive benefits herein authorized shall be granted only for Calendar Year
1991. Accordingly, all heads of agencies, including the governing boards of government-owned or -
controlled corporations and financial institutions, are hereby strictly prohibited from
authorizing/granting productivity incentive benefits or other allowances of similar nature for Calendar
Year 1992 and future years pending the result of a comprehensive study being undertaken by the Office
of the President in coordination with the Civil Service Commission and the Department of Budget and
Management on the matter.

On January 19, 1993, President Ramos issued AO 29 which granted productivity incentive benefits to
government employees in the maximum amount of P1,000.00 for the calendar year 1992 but reiterated
the proscription under Section 7 of AO 268, thus:"

SEC. 2. The prohibition prescribed under Section 7 of Administrative Order No. 268 is hereby reiterated.
Accordingly, all heads of government offices/agencies, including government-owned and/or controlled
corporations, as well as their respective governing boards are hereby enjoined and prohibited from
authorizing/granting Productivity Incentive Benefits or any and all similar forms of allowances/benefits
without prior approval and authorization via Administrative Order by the Office of the President.
Henceforth, anyone found violating any of the mandates in this Order, including all officials/employees
and the COA Auditor-in-Charge of such government office/agency found to have taken part thereof,
shall be accordingly and severely dealt with in accordance with the applicable provisions of existing
penal laws

I: Whether or not it constitutes an impairment of contractual obligations?

H: Anent petitioners' contention that the forcible refund of incentive benefits is an unconstitutional
impairment of a contractual obligation, suffice it to state that "[n]ot all contracts entered into by the
government will operate as a waiver of its non-suability; distinction must be made between its sovereign
and proprietary acts. The acts involved in this case are governmental. Besides, the Court is in
agreement with the Solicitor General that the incentive pay or benefit is in the nature of a bonus which
is not a demandable or enforceable obligation.

Philreca v. Sec. of DILG, GR 1543076, June 10, 2003 – Czar


F: With the passage of PD 269, electric cooperatives were decreed to be permanently exempted from
income taxes, and exempted from paying national or local government taxes until it has become
completely free of indebtedness incurred by borrowing.

From 1971 to 1978, the Philippine government and the electric cooperatives entered into 6 loan
agreements with USAID to finance their electrification projects. These obligations still exist.

However, the petitioners contend that their tax exemptions were invalidly withdrawn by the passage of
the Local Government Code. They assail Sections 193 and 234 for giving different tax treatments on
electric cooperatives registered under PD 269 and those registered under RA 6938, thereby violating the
equal protection clause.

I: Whether or not it constitutes an impairment of contracts?

H: It is ingrained in jurisprudence that the constitutional prohibition on the impairment of the obligation
of contracts does not prohibit every change in existing laws. To fall within the prohibition, the change
must not only impair the obligation of the existing contract, but the

Republic v. Rosemoor Mining and Development Corp. 426 SCRA 517 - Czar
F: Four respondents were granted permission to look for marble deposits in the mountains of Biak-na-
Bato. When they discovered marble deposits in Mount Mabio, they applied for license to exploit said
marble deposits and they were issued such license. However, in a letter, Maceda cancelled their license.
Proclamation No. 84 was then issued conforming the cancellation of the license. The RTC granted under
respondent’s license already became a property right, which is protected under due process clause. CA
affirmed the decision adding that the Constitution provides for the non-impairment of obligations and
contracts which implies that the license of respondents must be respected.

I: Whether or not the cancellation of TLAs constitutes impairment of contracts?

H: A mining license that contravenes a mandatory provision of law under which it is granted is void.
Being a mere privilege, a license does not vest absolute rights in the holder. Thus without offending the
due process and the non-impairment clauses of the Constitution, it can be revoked by the State in the
public interest.

Chavez v. COMELEC – 437 SCRA 415 - Czar

F: Chavez entered into formal agreements with certain establishments to endorse their products. He
later on filed his certificate of candidacy. COMELEC issued Resolution No. 6520, which contained the
provision assailed which directs the removal of the print, television and radio ads. Otherwise, they shall
be presumed to have conducted premature campaigning in violation of Sec. 80 of the Omnibus Election
Code. Petitioner allege that Comelec Resolution is an invalid exercise of police power arguing that the
billboards, while they exhibit his name and image, do not at all announce his candidacy for any public
office nor solicit support for such candidacy.

I:Whether or not Sec. 32 constitutes an impairment of contract as guaranteed in the Constitution?

H: The non-impairment clause of the Constitution must yield to the loftier purposes targeted by the
Government. Equal opportunity to proffer oneself for public office, without regard to the level of
financial resources one may have at his disposal, is indeed of vital interest to the public. The State has
the duty to enact and implement rules to safeguard the interest. Time and again, this Court has said that
contracts affecting public interest contain an implied reservation of the police power as a postulate of
the existing legal order. This power can be activated at anytime to change the provisions of the contract,
or even abrogate it entirely, for the promotion or protection of the general welfare. Such an act will not
militate against the impairment clause, which is subject to and limited by the paramount police power.
PETITION DISMISSED.

Alvarez v. PICOP - 508 SCRA 498 – Anton

Alvarez v. PICOP - 508 SCRA 498 – Anton

FACTS:
PICOP filed with the Department of Environment and Natural Resources (DENR) an application to have
its Timber License Agreement (TLA) No. 43 converted into an IFMA. According to PICOP, the government
is bound by contract, a 1969 Document signed by then President Ferdinand Marcos, to enter into an
Integrated Forest Management Agreement (IFMA) with PICOP. Secretary of DENR opposed it on the
ground that the presidential warranty is a mere permit. PICOP contended that such denial is a violation
of the non-impairment clause.
ISSUE: Whether or not there is a violation of the non-impairment clause.

RULING:
NO. Since timber licenses are not contracts, the non-impairment clause cannot be invoked. The
Presidential Warranty cannot, in any manner, be construed as a contractual undertaking assuring PICOP
of exclusive possession and enjoyment of its concession areas. Such an interpretation would result in
the complete abdication by the State in favor of PICOP of the sovereign power to control and supervise
the exploration, development and utilization of the natural resources in the area.

The 1969 Document is not a contract recognized under the non-impairment clause and, even if we
assume for the sake of argument that it is, it did not enjoin the government to issue an IFMA in 2002
either. These are the essential elements in PICOP’s cause of action, and the failure to prove the same
warrants a dismissal of PICOP’s Petition for Mandamus, as not even PICOP’s compliance with all the
administrative and statutory requirements can save its Petition now.

Lepanto v. WMC – 507 SCRA 315 - Anton

Lepanto v. WMC – 507 SCRA 315 - Anton


FACTS:
The Philippine Government and WMC Philippines, the local wholly-owned subsidiary of WMC Resources
International Pty. Ltd. (WMC Resources) executed a Financial and Technical Assistance Agreement,
denominated as the Columbio FTAA No. 02-95-XI (Columbio FTAA) for the purpose of large scale
exploration, development, and commercial exploration of possible mineral resources. Later, the FTAA
was transferred to Sagittarius Mining with the mere consent of the DENR Secretary without the approval
of the President. Petitioner contends that the transfer is a violation of Sec. 40 of the Philippines Mining
Act, requiring the approval by the President to effectuate the transfer. Be it noted that the FTAA was
entered into before the enactment of the Philippine Mining Act.

ISSUE: Can the FTAA be subjected to Sec. 40 of the Philippine Mining Act without violating the non-
impairment of obligations clause?

RULING:
NO. Such would constitute an impairment of obligations proscribed by the Constitution. Section 40 of
the Philippine Mining Act of 1995 requiring the approval of the President with respect to assignment or
transfer of FTAAs, if made applicable retroactively to the Columbio FTAA, would be tantamount to an
impairment of the obligations under said contract as it would effectively restrict the right of the parties
thereto to assign or transfer their interests in the said FTAA.

By imposing a new condition apart from those already contained in the agreement, before the parties to
the Columbio FTAA may assign or transfer its rights and interest in the said agreement, Section 40 of the
Philippine Mining Act of 1995, if made to apply to the Columbio FTAA, will effectively modify the
terms of the original contract and thus impair the obligations of the parties thereto and restrict the
exercise of their vested rights under the original agreement. Such modification to the Columbio FTAA,
particularly in the conditions imposed for its valid transfer is equivalent to an impairment of said
contract violative of the Constitution.
Republic v. Caguioa – 536 SCRA 193 – Anton
Republic v. Caguioa – 536 SCRA 193 - Anton
FACTS:
On the basis of Section 6 of R.A. No. 9334, SBMA issued a Memorandum declaring that effective all
importations of cigars, cigarettes, distilled spirits, fermented liquors and wines into the SBF, including
those intended to be transshipped to other free ports in the Philippines, shall be treated as ordinary
importations subject to all applicable taxes, duties and charges, including excise taxes. SBMA issued a
Memorandum directing the departments concerned to require locators/importers in the SBF to pay the
corresponding duties and taxes on their importations of cigars, cigarettes, liquors and wines before said
items are cleared and released from the freeport.

Private respondents assail the constitutionality of R.A. No. 9334 on the ground that it violates the non-
impairment of obligations.

ISSUE: Whether or not there is a violation of the non-impairment clause.

RULING:
NO. The rights granted under the Certificates of Registration and Tax Exemption of private respondents
are not absolute and unconditional as to constitute rights in esse – those clearly founded on or granted
by law or is enforceable as a matter of law.

These certificates granting private respondents a "permit to operate" their respective businesses are in
the nature of licenses, which the bulk of jurisprudence considers as neither a property nor a property
right. The licensee takes his license subject to such conditions as the grantor sees fit to impose, including
its revocation at pleasure. A license can thus be revoked at any time since it does not confer an absolute
right.

While the tax exemption contained in the Certificates of Registration of private respondents may have
been part of the inducement for carrying on their businesses in the SBF, this exemption, nevertheless, is
far from being contractual in nature in the sense that the non-impairment clause of the Constitution can
rightly be invoked.

Land Bank v. Republic – 543 SCRA 453 – Anton

Land Bank v. Republic – 543 SCRA 453 – Anton


FACTS:
Lourdes Farms, Inc. mortgaged the property in dispute, which is classified as Marshy and under water
during high tide, which used to be a portion of a dry river bed near the mouth of Davao River, to
petitioner LBP. Bureau of Lands discovered and found out that the land is still within the forest zone,
hence, cannot be appropriated. It resolved that the sales patent in favor of Bugayong (the original
owner) was improperly and illegally issued and that the Director of Lands had no jurisdiction to dispose
of the subject land. Petitioner contended that such restraint by the government constitutes an
impairment of obligations.

ISSUE: Whether or not there is impairment.


RULING:

NO. There is no impairment of contract but a valid exercise of police power of the State. The
constitutional guarantee of non-impairment of contracts may not likewise be used by LBP to validate its
interest over the land as mortgagee. The State’s restraint upon the right to have an interest or
ownership over forest lands does not violate the constitutional guarantee of non-impairment of
contracts. Said restraint is a valid exercise of the police power of the State.

Preservation of our forest lands could entail intrusion upon contractual rights as in this case but it is
justified by the Latin maxims Salus populi est suprema lex and Sic utere tuo ut alienum non laedas, which
call for the subordination of individual interests to the benefit of the greater number.

Serrano v. Gallant – 582 SCRA 254 – Anton

Serrano v. Gallant – 582 SCRA 254 - Anton


FACTS:
This case involved a Labor Case for money claims. The Labor Arbiter awarded the said claim. The said
award was affirmed by the NLRC on appeal but it substantially reduced the award on the basis
of Sec. 10, R.A. 8042 which reads:

Sec. 10. Money Claims. - x x x In case of termination of overseas employment without just, valid or
authorized cause as defined by law or contract, the workers shall be entitled to the full
reimbursement of his placement fee with interest of twelve percent (12%) per annum, plus his
salaries for the unexpired portion of his employment contract or for three (3) months for every
year of the unexpired term, whichever is less.

Petitioner contends that the said clause in R.A. 8042 violates the non-impairment clause.

ISSUES: Whether or not the subject clause violates Section 10, Article III of the Constitution on non-
impairment of contracts;

RULING:
NO. The prohibition is aligned with the general principle that laws newly enacted have only a
prospective operation, and cannot affect acts or contracts already perfected; however, as to
laws already in existence, their provisions are read into contracts and deemed a part thereof.
Thus, the non-impairment clause under Section 10, Article III is limited in application to laws
about to be enacted that would in any way derogate from existing acts or contracts by
enlarging, abridging or in any manner changing the intention of the parties thereto.

As aptly observed by the OSG, the enactment of R.A. No. 8042 in 1995 preceded the execution of the
employment contract between petitioner and respondents in 1998. Hence, it cannot be argued
that R.A. No. 8042, particularly the subject clause, impaired the employment contract of the
parties. Rather, when the parties executed their 1998 employment contract, they were deemed
to have incorporated into it all the provisions of R.A. No. 8042.

But even if the Court were to disregard the timeline, the subject clause may not be declared
unconstitutional on the ground that it impinges on the impairment clause, for the law was
enacted in the exercise of the police power of the State to regulate a business, profession or
calling, particularly the recruitment and deployment of OFWs, with the noble end in view of
ensuring respect for the dignity and well-being of OFWs wherever they may be employed. Police
power legislations adopted by the State to promote the health, morals, peace, education, good
order, safety, and general welfare of the people are generally applicable not only to future
contracts but even to those already in existence, for all private contracts must yield to the
superior and legitimate measures taken by the State to promote public welfare.

Alvarez v. PICOP – 606 SCRA 444 – Anton

Alvarez v. PICOP – 606 SCRA 444 - Anton


FACTS:
PICOP filed with the Department of Environment and Natural Resources (DENR) an application to have
its Timber License Agreement (TLA) No. 43 converted into an IFMA. According to PICOP, the government
is bound by contract, a 1969 Document signed by then President Ferdinand Marcos, to enter into an
Integrated Forest Management Agreement (IFMA) with PICOP. Secretary of DENR opposed it on the
ground that the presidential warranty is a mere permit. PICOP contended that such denial is a violation
of the non-impairment clause.

ISSUE: Is the 1969 document considered a contract under the non-impairment clause?

RULING:
NO. The Court does not subscribe to PICOP’s argument that the Presidential Warranty dated September
25, 1968 is a contract protected by the non-impairment clause of the 1987 Constitution.

An examination of the Presidential Warranty at once reveals that it simply reassures PICOP of the
government’s commitment to uphold the terms and conditions of its timber license and guarantees
PICOP’s peaceful and adequate possession and enjoyment of the areas which are the basic sources of
raw materials for its wood processing complex. The warranty covers only the right to cut, collect, and
remove timber in its concession area, and does not extend to the utilization of other resources, such
as mineral resources, occurring within the concession.

The Presidential Warranty cannot be considered a contract distinct from PTLA No. 47 and FMA No. 35.
We agree with the OSG’s position that it is merely a collateral undertaking which cannot amplify
PICOP’s rights under its timber license.

NOTE: I do not know how this case is different from the other case of the same title.

Surigao v. ERC - 632 SCRA 96 – Anton

Surigao v. ERC - 632 SCRA 96 – Anton


FACTS:
The National Electrification Administration (NEA) entered into a loan agreement with the Asian
Development Bank (ADB). The proceeds of the loan were intended for use by qualified rural electric
cooperatives, SURNECO included, in their rehabilitation and expansion projects. The loan agreement
imposed a 15% system loss cap, but provided a Power Cost Adjustment Clause authorizing cooperatives
to charge and show "system losses in excess of 15%" as a separate item in their consumer’s bill. Thus,
the cooperatives charged their consumer-members "System Loss Levy" for system losses in excess of the
15% cap. R.A. 7832 set a cap which is different from that which is specified in the loan agreement.
Petitioner contends that the said imposition of a different cap is an impairment of an obligation.

ISSUE: Whether or not there is an impairment of obligation.

RULING:
NO. The cap as set by the law took effect upon the enactment of the said law which was before the loan
agreement was entered into, Hence, it cannot be considered as a substantial change so as to constitute
an impairment of obligations. Even assuming, merely for argument’s sake, that the ERC issuances
violated the NEA and ADB covenant, the contract had to yield to the greater authority of the State’s
exercise of police power. It has long been settled that police power legislation, adopted by the State to
promote the health, morals, peace, education, good order, safety, and general welfare of the people
prevail not only over future contracts but even over those already in existence, for all private contracts
must yield to the superior and legitimate measures taken by the State to promote public welfare.

Hacienda Luisita v. Pac – 653 SCRA 154 - 40 – Anton

FACTS:
Respondent revoked the approved Stock Distribution Plan petitioner. In addition, RA 6657, in relation to
DAO 10 had the effect of subjecting petitioners’ landholdings to compulsory distribution after its
approved SDP has been implemented. Petitioners contend that this is an impairment of
contractual obligations created under the Stock Distribution Option Agreement (SDOA).

ISSUE: Whether or not there is an impairment of contractual obligations.

RULING:
NO. A law authorizing interference, when appropriate, in the contractual relations between or among
parties is deemed read into the contract and its implementation cannot successfully be resisted
by force of the non-impairment guarantee. There is, in that instance, no impingement of the
impairment clause, the non-impairment protection being applicable only to laws that derogate
prior acts or contracts by enlarging, abridging or in any manner changing the intention of the
parties. Impairment, in fine, obtains if a subsequent law changes the terms of a contract
between the parties, imposes new conditions, dispenses with those agreed upon or withdraws
existing remedies for the enforcement of the rights of the parties. Necessarily, the
constitutional proscription would not apply to laws already in effect at the time of contract
execution, as in the case of RA 6657, in relation to DAO 10, vis-à-vis HLI’s SDOA.

Section 11. Free access to the courts and quasi-judicial bodies and adequate legal assistance shall not
be denied to any person by reason of poverty.

Indigent Party: One who is authorized by the court to prosecute his action or defense as an indigent
upon an ex parte application and hearing showing that he has no money or property sufficient and
available for food, shelter and basic necessities for himself and his family. (Rules of Court, Rule 3,
Section 21)
Legal Provisions on Free Access
1. RA 6035: stenographers are required to give free transcript of stenographic notes to indigent and
low-income litigants.
2. Rules of Court, Rule 3, Section 21
3. Constitution, Article 3, Section 12: the court appoints a counsel de officio for an accused who
cannot afford to engage the service of a counsel de parte.
4. Rule on the Writ of Amparo, Section 4: No docket or other lawful fees shall be required for the filing
of the petition.
5. Rule on the Writ of Habeas Data, Section 4: No docket and other lawful fees are required from
indigent petitioner.

You might also like